Download as docx, pdf, or txt
Download as docx, pdf, or txt
You are on page 1of 667

БАЗА

ВСТУП

1. На гістологічному препараті в сполучній тканині знайдено великі клітини, заповнені базофільною


метахроматичною зернистістю; гістохімічно встановлено, що гранули містять гепарин та гістамін. Які клітини
найбільш вірогідно знайдено в препараті?
A. @Тучні клітини
B. Фібробласти
C. Макрофаги
D. Плазмоцити
E. Адипоцити
2. У недоношеного немовля – дихальна недостатность. Недостатність якого з компонентів з
аерогематичного бар'єру обумовлює дану патологію?
A. @Сурфактант
B. Альвеолоцити
C. Базальна мембрана альвеолоцитів
D. Базальна мембрана ендотеліоцитів
E. Капілярний ендотелій
3. На препараті одного з відділів дихальної системи виявлено трубчастий орган, в якому визначається
невисокий епітелій, добре розвинена м’язова оболонка, відсутні залози та хрящ. Назвіть цей орган:
A. @Малий бронх
B. Трахея
C. Гортань
D. Великий бронх
E. Середний бронх
4. Препарат сполучної тканини дерми забарвили Судан-III і гематоксилін-еозином. Виявлено групи
великих полігональних клітин, помаранчевого кольору з розташованими на периферії сплощеними ядрами.
Назвіть тканину?
A. Ретикулярна сполучна тканина
B. Пластинчаста кістка
C. Бурий жир
D. @Білий жир
E. Гіаліновий хрящ
5. Гістологічний препарат органа порожнини рота виявляє, що передня поверхня органа обмежена
багатошаровим плоским епітелієм без зроговіння, а задня - багаторядним війчастим епітелієм. Який це орган?
A. Щока
B. Губа
C. Ясна
D. Тверде піднебіння
E. @М'яке піднебіння
6. Препарат м'якого піднебіння, з оральною та назальною поверхнями. Епітелій оральної поверхні виявляє
ознаки альтерації. Який епітелій було пошкоджено?
A. Багаторядний війчастий
B. Багатошаровий призматичний без кератинізації
C. Плаский з кератинізацією
D. Багатошаровий кубічний без кератинізації
E. @Багатошаровий плоский без кератинізації
7. Мікропрепарат підщелепної слинної залози виявляє клітини кошикоподібної форми, сконцентровані
навколо ацинусів та вивідних проток, що оточують основи серозних клітин і називаються міоепітеліоцитами. До
якої тканини відносяться ці клітини:
A. Спеціалізованої сполучної
B. Епітеліальної
C. М’якої сполучної
D. Нервової
E. @М'язової
8. Мікроскопічне дослідження органа виявило ацинуси, що складаються з 10-15 конусних клітин з
базофільною цитоплазмою, круглим ядром і добре розвиненою гранульованою ендоплазматичною сіткою.
Ацинус оточений базальною мембраною з міоепітеліальними клітинами. Назвіть орган.
A. Підязична залоза
B. @Завушна залоза
C. Підшлункова залоза
D. Печінка
E. Легені
9. Під час гістологічного дослідження шлунка з'ясувалося, що кількість парієтальних клітин у складі залоз
різко зменшена або повністю відсутня. Слизову оболонку якої частини шлунка було досліджено?
A. Стравохідну
B. Дна шлунка
C. Тіла шлунка
D. Кардіальної частини
E. @Пілоричної частини
10. На електронній мікрофотограмі виявляється макрофаг з еритроцитами на різних етапах диференціації,
розташованих вздовж синусоїдів. Ці клітини якого органа?
A. @Червоний кістковий мозок
B. Селезінка
C. Мигдалина
D. Лімфатичний вузол
E. Тимус
11. При мікроскопічному дослідженні червоного кісткового мозку виявлено кілька капілярів через стінки,
яких мігрують зрілі кров'яні клітини. Який тип капілярів це?
A. Фенестраційний
B. @Синусоїдальний
C. Вісцеральний
D. Лімфатичний
E. Соматичний
12. 6-місячна дитина задихається, лежачи на спині. При пальпації на передній стінці трахеї до яремної
вирізки грудини виявлено пухлиноподібну масу, що тягнеться до переднього середостіння. Що з наведеного
нижче може притиснути трахею?
A. Паратироїдні лімфатичні вузли
B. Паращитовидная залоза
C. Щитовидна залоза
D. @Вилочкова залоза
E. Навколо трахеальні лімфатичні вузли
13. Гістологічний зразок представляє орган, який має як кортикальну, так і мозкову речовину. Коркова
речовина складається з зовнішньої зони, яка містить лімфатичні вузлики, а також паракортикальну зону.
Медулярна речовина містить медулярні шнури, пазухи та трабекули. Який орган має ці морфологічні ознаки?
A. Наднирники
B.Нирка
C.Тимус
D. Селезінка
E. @Лімфатичний вузол
14. У гістологічному зразку паренхіма органа представлена лімфоїдною тканиною, яка утворює
лімфатичні вузли, що розташовані дифузним чином і містять центральну артерію. Що має таку морфологічну
структуру?
A. Тимус
B.@Селезінка
C.Мигдалина
D. Лімфатичний вузол
E. Червоний кістковий мозок
15.Інтралобулярні капіляри печінки мають широкий нерівномірний просвіт. Базальна мембрана в
основній частині капіляра відсутня. Який це тип капілярів?
A. Соматичний
B. Посткапілярний
C. Прекапілярний
D. @Синусоїдний
E. Вісцеральний
16. Електронно-мікроскопічне вивчення коркової речовини нирок виявило структури, вистелені
призматичним епітелієм з війчастою облямівкою та глибокими складками клітинної мембрани з базального боку.
Між ними розміщена велика кількість мітохондрій. До якої частини нефрона відносяться дані структури:
A. Дистальний прямий каналець
B. @Проксимальний каналець
C. Ниркове тільце
D. Дистальний звивистий каналеці
E. Петля Генле
17. Електронна мікрофотографія нефрона продемонструвала приносну артеріолу клубочка, під ендотелієм
вміщує - гігантські клітини, які містять секреторні гранули. Назвіть тип цих клітин:
A. Гладком’язові
B. Проміжна
C. @Юкстагломерулярні
D. Мезангіальні
E. Юкставаскулярні
18. Гістологічний препарат нирки показує частину дистальних канальців, що йдуть між аферентним і
еферентним артеріолами. Клітини, що утворюють стінку канальців, мають щільні ядра; базальна мембрана
відсутня. Таке структурне утворення називається:
A. Юкстово-судинні клітини
B.@Macula dens (щільна пляма)
C.Нефротелій
D. Мезангіальні клітини
E. Юкстагломерулярні клітини
19. Гістологічний препарат представлений паренхіматозним органом, який має коркову та мозкову
речовину. Коркова складається з груп епітеліоцитів, між якими є кровоносні капіляри, групи формують три зони.
Мозкова представлена хроматофінними клітинами та венозними синусоїдами. Який орган має ці морфологічні
особливості?
A. @Наднирник
B. Нирка
C. Щитовидна залоза
D. Лімфатичний вузол
E. Тимус
20. Гістологічний препарат виявляє кровоносну судину. Її внутрішній шар складається з ендотелію,
субендотеліального простору та внутрішньої еластичної мембрани. Середній шар містить велику кількість
гладких міоцитів. Дана морфологічна характеристика типова для:
A. Капіляра
B. @Артерії м'язового типу
C. Вени безм’язового типу
D. Артерії еластичного типу
E. Вени м’язового типу
21. 54-річний чоловік звернувся до лікарні зі скаргами на біль у правій підреберній області, блювоту з
кров’ю. Об'єктивно: збільшена печінка, варикозне розширення вен шлунка та стравоходу. Порушення функції
якої судини має місце?
A. Aorta abdominalis
B. Vena hepatica
C. Vena cava inferior
D. @Vena porta
E. Vena cava superior
22. Де повинен бути вставлений катетер для евакуації лімфи з грудної лімфатичної протоки?
A. Нижня порожниста вена
B. Верхня порожниста вена
C. Ліва пахова вена
D. @Лівий венозний кут
E. Правий венозний кут
23. Запальний процес змінив субсерозний шар навколо шийки матки, спричинюючи інтенсивний
больовий синдром. Де відбувається патологічний процес?
A. Міометрій
B. Периметрій
C. Мезометрій
D. @Параметрій
E. Ендометрій
24. На препараті яєчника, забарвленому гематоксиліном-еозіном, визначається фолікул, в якому клітини
фолікулярного епітелію розміщені в 1-2 шари та мають кубічну форму, навколо овоциту видно оболонку яскраво-
червоного кольору. Назвіть цей фолікул:
A. @Первинний
B. Примордіальний
C. Вторинний
D. Зрілий
E. Атретичний
25. Під час ембріогенезу трофобласт перетворюється на рудиментарний орган, який має ендокринну
функцію. Назвіть даний рудиментарний орган.
A. @Ворсинки хоріона
B. Жовтяний мішок
C. Амніон
D. Пупкова хорда
E. Алантоіс
26. Під час пубертатного періоду клітини чоловічих статевих залоз починають виробляти чоловічий
статевий гормон тестостерон, що відповідає за формування вторинних статевих ознак. Які клітини чоловічих
статевих залоз виробляють цей гормон?
A. Сперматозоїди
B.@Клітини Лейдіга
C.Клітини Сертолі
D. Сустеноцити
E. Підтискувальні клітини
27. Молода людина скаржиться на розлад сечовипускання. Вивчення зовнішніх статевих органів показало,
що уретра розщеплена, і сеча витікає з цього отвору. Яка аномалія розвитку зовнішніх статевих органів?
A. @Епіспадія
B. Гіпоспадія
C. Парафімоз
D. Фімоз
E. Гермафродитизм
28. Вивчення геніталій новонародженого хлопчика виявило зяяння уретри, що відкривається на нижній
частині його члена. Назвіть дану мальформацію?
A. Монорхізм
B. Гермафродитизм
C. Епіспадія
D. @Гіпоспадія
E. Крипторхізм

УЛЬТРАСТРУКТУРА

1. Електронним мікроскопічним дослідженням клітини виявлено округлі везикули, що оточені мембраною і


містять багато різних гідролітичних ферментів. Відомо, що ці органели забезпечують внутрішньоклітинне
травлення та захисні функції. Ці елементи:

A. Ендоплазматичний ретикулум
B. Рибосоми
C. Мітохондрії
D. Центросоми
E. @Лізосом
2. Мікроскопічним аналізом клітин серця людини виявлені деякі овальні оганелі, їх туніка формується
двома мембранами: зовнішня гладка, внутрішня - кристи. Біохімічний аналіз визначив присутність ферменту
АТФ-синтетази. Які органели були проаналізовані?
A. Центросоми
B. Рибосоми
C. @Мітохондрії
D. Лізосоми
E. Ендоплазматичний ретикулум
3. Позначені амінокислоти аланін та триптофан були введені в мишу, щоб вивчити локалізацію біосинтезу
білка в її клітинах. Навколо яких органел буде спостерігатися накопичення помічених амінокислот?
A. Лізосом
B. @Рибосом
C. Агранулярного ендоплазматичного ретикулума
D. Клітинного центру
E. Гольджі апарату
4. При електронномікроскопічному дослідженні біоптату печінки виявлено, що між чисельними
мітохондріями знаходиться велика кількість плоских цистерн і міхурців з секреторними гранулами, стінки яких
утворені мембранами. Про гіперплазію складових якої ультраструктури йде мова?
A. Лізосом
B. Мікротрубочок
C. @Апарату Гольджі
D. Ендоплазматичного ретикулуму
E. Піноцитозних міхурців
5. Ультраструктурне дослідження біопсії печінки показало, що між мітохондріями існували численні
плоскі цистерни та міхурці з секреторними гранулами, обмеженими мембраною. Назвіть структуру клітини з
гіперпластичними складовими?
A. @Гольджі апарат
B.Піноцитозні бульбашки
C.Ендоплазматичний ретикулум
D. Лізосоми
E. Мікротубули
6. Комплекс Гольджі експортує речовини з клітини за рахунок злиття мембранного міхура з клітинною
мембраною. Вивільнює вміст пухирця. Який процес це?
A. Сприяння дифузії
B. @Екзоцитоз
C. Активний транспорт
D. Усі відповіді є помилковими
E. Ендоцитоз
7. Ультрамікроскопічне дослідження "темної" гепатоцелюлярної популяції в клітинній цитоплазмі
виявило розвинений гранульований ендоплазматичний ретикулум. Яку функцію має ця органела?
A. Дезінтоксикаційна
B.Осадження іонів кальцію
C.@Синтез білків плазми крові
D. Виробництво жовчі
E. Синтез вуглеводів
8. Життєвий цикл клітини включає в себе процес ауторедублікації ДНК. У результаті цього процесу
монохроматидні хромосоми перетворюються на біхроматидні. Це явище спостерігається протягом наступного
періоду клітинного циклу:
A. G2
B. G1
C. @S
D. G0
E. М
9. Електронна мікрофотографія являє собою клітину, яка не має ядра та ядерної мембрани. Хромосоми
мають вільне положення, центріоли мігрують до полюсів. Назвіть фазу клітинного циклу:
A. @Профаза
B. Телофаза
C. Метафаза
D. Міжфаза
E. Анафаза
10. Переміщення дочірніх хроматид до полюсів клітини спостерігається в мітотично діленій клітині. На
якому етапі мітотичного циклу ця клітина?
A. Інтерфаза
B.Метафаза
C.Профаза
D. @Анафаза
E.Телофаза
11. Вивчення фаз мітотичного циклу кореня цибулі виявило клітину, в якій хромосоми розташовані в
екваторіальній площині, утворюючи зірку. Яка стадія клітинного мітозу це?
A. Профаза
B.Анафаза
C.Телофаза
D. Інтерфаза
E. @Метафаза
12. У генетичній лабораторії під час роботи з молекулами ДНК білих щурів лінії Вістара було заміщено
ще один нуклеотид. При цьому в пептиді заміщували лише одну амінокислоту. Яка мутація дала такі зміни?
A. Переміщення
B. Зсув кадру зчитування
C. @Трансгресія
D. Видалення
E. Дублювання
13. Дослідження пацієнта показало знижений вміст іонів магнію, необхідних для приєднання рибосом до
гранулярного ендоплазматичного ретикулуму. Відомо, що це викликає порушення біосинтезу білка. Яка стадія
біосинтезу білка буде порушена?
A. Подвоєння
B.@Трансляція
C.Активація амінокислоти
D. Тиражування
E. Транскрипція
14. У хворого виявлено зниження вмісту іонів магнію, які потрібні для прикріплення рибосом до
гранулярної ендоплазматичної сітки. Відомо, що це призводить до порушення біосинтезу білка. Який саме етап
біосинтезу білка буде порушено?
A. @Трансляція
B. Транскрипція
C. Реплікація
D. Активація амінокислот
E. Термінація
15. У постсинтетичний період мітотичного циклу був знищений синтез білків-тубулінів, які беруть участь
у формуванні мітозу. Це може спричинити погіршення:
A. Хромосомну деспіралізацію
B.Цитокінез
C.Тривалості мітозу
D. @Розділення хромосом
E. Хромосомної спіралізації
16. У період постсинтезу мітотичного циклу порушувався синтез білків тубуліну. Ці білки беруть участь у
будівництві веретена поділу. Це може призвести до порушення:
A. Розсіювання хромосом
B.Тривалості мітозу
C.Цитокінезу
D. Спіралізації хромосом
E. @Депаралізації хромосом
17. 46 хромосом були виявлені при каріотипному огляді 5-річної дівчини. Одна з 15-ти пар хромосом
більше, ніж зазвичай, через пов'язану хромосому з 21 пари. Який тип мутації має ця дівчина?
A. Видалення
B. Недостатність
C. Інверсія
D. Дублювання
E. @Перекладка
18. У деяких регіонах Південної Африки виявляється розповсюджена серповидно- клітинна анемія, в якій
еритроцити мають форму серпа в результаті заміщення глутаміну валіном у молекулі гемоглобіну. Яка причина
цього захворювання?
A. @Генна мутація
B.Порушення механізмів реалізації генетичної інформації
C.Геномічні мутації
D. Перетинаючі мутації
E. Трансдукція
19. Експертиза 7-річної дитини виявила такі симптоми: низькій зріст, широке округле обличчя, тісно
розміщені очі, вузькі пальці, напіввідкриту порожнину рота. Виявлено дефект клапана. Ці клінічні дані, швидше
за все, характерні для синдрому Дауна. Назвіть причину такої патології:
A. Нерозходження статевих хромосом
B.Часткова моносомія
C.@Трисомія 21 хромосоми
D. Трисомія 13 хромосоми
E. X-хромосомна трисомія
20. При обстеженні 7-річної дитини виявлено наступні синдроми: низький зріст, широке округле обличчя,
близько розміщені очі з вузькими очними щілинами, напіввідкритий рот. Діагностовано також ваду серця. Ці
клінічні ознаки найбільш характерні для хвороби Дауна. Вкажіть причину даної патології.
A. @Трисомія 21-ої хромосоми
B. Трисомія 13-ої хромосоми
C. Трисомія 20-ої хромосоми
D. Часткова моносомія
E. Нерозходження статевих хромосом
21. У дитини, 5 місяців, яка померла від пневмонії, при макроскопічному обстеженні знайдено: косий
розріз очей, западаюча спинка носа, високе піднебіння, низьке розташування малих вушних раковин, а під час
розтину - дефект розвитку серця і магістральних судин. При генетичному дослідженні виявлено трисомію за 21
парою аутохромосом. Який з перелічених діагнозів найбільш вірогідний?
A. @Хвороба Дауна
B. Синдром Патау
C. Синдром Шерешевського-Тернера
D. Синдром Едвардса
E. Синдром Марфана
22. Аномальна диз'юнкція хромосом при мейозі призвела до утворення: яйцеклітини з 22 аутосом і
полярним тілом з 24 хромосомами. Якщо така яйцеклітина буде запліднена звичайним сперматозоїдом (22+X),
дитина може мати наступний синдром:
A. Синдром Клайнфельтера
B.Трисомія X
C.@Синдром Тернера
D. Синдром Дауна
E. Синдром Едвардса
23. У дівчини виявлено диспропорцію тіла, крилоподібні складки на шиї. При цитогенетичному
дослідженні в ядрах лейкоцитів не виявлені "барабанні палички", а в ядрах букального епітелію відсутні тільця
Барра. Який найбільш вірогідний діагноз?
A. @Синдром Шерешевського-Тернера
B. Синдром Клайнфельтера
C. Синдром Дауна
D. Синдром Патау
E. Синдром Едвардса
24. 8-місячна дитина має незакрите піднебіння, ряд дефектів очей, мікроцефалію, розлад серцево-судинної
системи. Цитогенетичний аналіз показав 47 хромосом з додатковою 13-ю хромосомою. Який діагноз можна
поставити на основі клінічних спостережень та цитогенетичних досліджень?
A. Синдром Дауна
B.Синдром Едвардса
C.@Синдром Патау
D. Кишковий синдром
E. Синдром Клайнфельтера
25. При розтині трупа новонародженого хлопчика виявлені: полідактилія, мікроцефалія, незарощення
верхньої губи та верхнього піднебіння, а також гіпертрофія паренхіматозних органів. Вказані вади відповідають
синдрому Патау. Яка найбільш ймовірна причина даної патології?
A. @Трисомія 13-ої хромосоми
B. Трисомія 18-ої хромосоми
C. Трисомія 21-ої хромосоми
D. Нерозходження статевих хромосом
E. Часткова моносомія
26. 28-річна жінка повідомила лікаря про безпліддя. Експертиза показала недорозвинені яєчники і матку,
нерегулярний менструальний цикл. Вивчення статевого хроматину показало 2 тіла Барра в більшості соматичних
клітин. Яка хромосомна хвороба є найбільш вірогідною у цьому випадку?
A. Синдром Тернера
B.Синдром Патау
C.Синдром Едвардса
D. @Трисомія-X синдром
E. Синдром Клайнфельтера
27. У індивіда з підозрою на синдром Прадера-Віллі під час картування хромосом з використанням
бендингу відмічено відсутність смужки q-12 хромосоми 15. Якому дефекту відповідає виявлена зміна?
A. @Делеція
B. Інверсія
C. Дуплікація
D. Інсерція
E. Транслокація
28. У 19-річної дівчинки клінічно виявлено таку групу синдромів: низький зріст, статевий інфантилізм,
відставання в інтелектуальному та статевому розвитку, вада серця. Які найбільш ймовірні причини даної
патології?
A. @Нерозходження статевих хромосом
B. Трисомія по 13-й хромосомі
C. Трисомія по 18-й хромосомі
D. Трисомія по 20-й хромосомі
E. Часткова моносомія
29. У новонародженої дитини виявлено наступну патологію: аномалія розвитку нижньої щелепи та
гортані, що супроводжується характерними змінами голосу, а також мікроцефалія, вада серця, чотирьохпалість.
Яка найбільш йморівна причина таких аномалій?
A. @Часткова моносомія короткого плеча 5-ої хромосоми
B. Часткова моносомія короткого плеча 7-ої хромосоми
C. Часткова моносомія короткого плеча 9-ої хромосоми
D. Часткова моносомія короткого плеча 11-ої хромосоми
E. Трисомія 21-ої хромосоми
РЕГЕНЕРАЦІЯ. ПРОЦЕСИ АДАПТАЦІЇ ТА КОМПЕНСАЦІЇ
ВАРІАНТ №

1. У дитини, 12 років, хворої на поліомієліт, скелетні м'язи слабкі, об'єм їх зменшений, шкіра суха, бліда. При
морфологічному дослідженні біоптату м'яких тканин виявлені характерні морфологічні зміни. Визначите
характер патологічного процесу в м'яких тканинах.
A. @Атрофія
B. Гіпертрофія
C. Гіперплазія
D. Метаплазія
E. Гіпоплазія
2. На розтині померлої, 86 років, що страждала на атеросклероз судин головного мозку, виявлено атрофію кори
головного мозку. Як називається ця атрофія за етіологією?
A. @Від недостатнього кровопостачання
B. Від тиску
C. Від дії фізичних і хімічних чинників
D. Нейротична
E. Дисфункціональна
3. При розтині тіла жінки, 69 років, яка тривалий час страждала на гіпертонічну хворобу, патологоанатом виявив,
що обидві нирки щільної консистенції, значно зменшені, поверхня їх дрібнозерниста. Про що свідчать ці зміни?
A. Атрофію від тиску
B. Старечу атрофію
C. @Атрофію від недостатнього кровопостачання
D. Дисфункціональну атрофію
E. Гіпоплазію
4. Після видалення зуба у пацієнта через деякий час розвинулась атрофія країв зубної лунки. Назвіть цей вид
атрофії.
A. @Дисфункціональна
B. Внаслідок недостатності кровообігу
C. Від тиску
D. Нейротична
E. Від дії фізичних та хімічних факторів
5. Пiдлiток скаржиться на стоншення м'язiв i зменшення об'єму гомiлки, якi виникли пiсля перелому стегнової
кiстки без пошкодження нервів, який тривало не загоювався. Як називається така атрофiя м'язiв?
A. Нейротична
B. Викликана стисканням
C. Вiд дiї фiзичних факторiв
D. @Дисфункцiональна
E. Від недостатнього кровопостачання
6. У 40-річного чоловіка зі стенозуючим (без метастазів) раком стравоходу визначаються наступні зміни: атрофія
скелетних м'язів, жирової клітковини. Шкіра - землисто-коричневого кольору, епідерміс стоншений, серце -
зменшене за розмірами. Міокард і печінка - бурого кольору. Поставте діагноз.
A. Міастенія
B. @Аліментарна кахексія
C. Ракова кахексія
D. Бура атрофія
E. Хвороба Аддісона
7. Хворий на рак шлунка з множинними метастазами помер від ракової кахексії. Назвіть характерні зміни серця,
виявлені на розтині.
A. @Бура атрофія міокарда
B. Амілоїдна кардіомегалія
C. Дилатаційна кардіоміопатія
D. Гіпертрофічна кардіоміопатія
E. "Тигрове" серце
8. На розтині померлого від гострого інфаркту міокарда хворого, що страждав на часті гемороїдальні кровотечі,
виявлено: соковитий, червоного кольору кістковий мозок діафіза стегнової кістки. Який процес розвинувся в
кістковому мозку?
A. Вікарна гіпертрофія
B. Гіпертрофічні розростання
C. @Компенсаторна гіперплазія
D. Гормональна гіперплазія
E. Робоча гіпертрофія
9. На розтині померлого, 73 років, виявлено: збільшену, м'яку, еластичну, злегка горбисту передміхурову залозу,
що на розрізі складається з окремих вузлів, розділених прошарками сполучної тканини. При мікроскопії виявлено
збільшення кількості залозистих елементів. Величина часточок і кількість залозистих структур - різна. Який
процес має місце в передміхуровій залозі?
A. М′язово-фіброзна нодулярна гіперплазія
B. @Змішана нодулярна гіперплазія
C. Аденокарцинома
D. Залозиста нодулярна гіперплазія
E. Недиференційований рак
10. У ході гістологічного дослідження біоптату, що був отриманий в результаті роздільного діагностичного
вишкрібання стінки матки у 45 річної жінки, з порушеннями оваріально-менструального циклу, виявлені:
збільшення кількості ендометріальних залоз, деякі - пилкоподібно звивисті, деякі залози – кістозно розширені.
Діагностуйте захворювання.
A. Плацентарний поліп
B. Атипова гіперплазія ендометрію
C. Залозистий поліп ендометрію
D. Аденокарцинома ендометрію
E. @Залозисто-кістозна гіперплазія ендометрію
11. Хворого, 65 років, прооперовано в урологічному відділенні з приводу доброякісної гіперплазії передміхурової
залози. Під час операції виявлено, що стінка сечового міхура потовщена до 1 см. Це можна пояснити:
A. Вікарною гіпертрофією
B. Гормональною гіпертрофією
C. @Робочою гіпертрофією
D. Гормональною гіперплазією
E. Захисною гіперплазією
12. При електронномікроскопічному дослідженні м'язових клітин серця було діагностовано гіпертрофію
міокарда, оскільки цитоплазма кардіоміоцитів містила велику кількість:
A. Гліальних філаментів
B. Нейрофіламентів
C. Віметину
D. @Десміну
E. Цитокератину
13. У хворого, який помер при легенево-серцевій недостатності: серце збільшене за розмірами, стінка правого
шлуночка на розрізі потовщена, порожнина - розширена. Визначите характер патологічного процесу
A. @Гіпертрофія
B. Гіперпластичні розростання запальної природи
C. Метаплазія
D. Атрофія
E. Склероз
14. При розтинi померлого хворого, що довгий час страждав на гiпертонiчну хворобу, виявлено рiзко збiльшене
серце, масою 800.0 г. Назвiть вид компенсаторної гiпертрофiї серця.
A. Вiкарна
B. @Робоча
C. Нейрогуморальна
D. Гiпертрофiчнi розростання
E. Вакатна
15. У хворого, померлого вiд гiпертонiчної хвороби, на розтинi виявлене збiльшене серце масою 600 г з
потовщеною до 2 см стiнкою лiвого шлуночка i розширеною порожниною шлуночка. Уточнiть рiзновид
пристосувальної перебудови серця.
A. Вiкарна гіпертрофія
B. @Ексцентрична гіпертрофія
C. Концентрична гіпертрофія
D. Ексцентрична атрофiя
E. Вакантна гiпертрофiя
16. При розтині померлого, 57 років, що страждав на гіпертонічну хворобу і помер від декомпенсації серцевої
недостатності, виявлено збільшене серце з розширеними порожнинами (маса 550 г, товщина стінки лівого
шлуночка 2,5 см). Мікроскопічно: кардіоміоцити значно збільшені за розмірами, з жировою дистрофією і
гіперхромними діжкоподібними ядрами. Який з перерахованих патологічних процесів найбільш вірогідні у серці?
A. Компенсована гіпертрофія
B. Концентрична гіпертрофія
C. Ангіогенний кардіосклероз
D. Кардіоміопатія
E. @Декомпенсована гіпертрофія
17. При розтині тіла померлого, 57 років, який страждав на гіпертонічну хворобу та помер від серцевої
декомпенсації, знайдено збільшене серце з розширеними порожнинами (маса - 550 г, товща стінки лівого
шлуночка - 2,5 см). мікроскопічно кардіоміоцити значно збільшені у розмірах, з жировою дистрофією та
гіперхромними діжкоподібними ядрами. Який з перелічених патологічних процесів найбільш вірогідний у серці?
A. @Ексцентрична гіпертрофія
B. Міокардит
C. Концентрична гіпертрофія
D. Ангіогенний кардіосклероз
E. Кардіоміопатія
18. У хворого 10 років тому було видалено праву легеню з приводу пухлини, об'єм лівої легені збільшився на
50%. Який процес розвинувся в лівій легені?
A. Нейрогуморальна гіпертрофія
B. Робоча гіпертрофія
C. Несправжня гіпертрофія
D. @Вікарна гіпертрофія
E. Гипертрофічні розростання
19. Хворий помер від серцевої недостатності, в анамнезі - пульмонектомія з приводу кісти правої легені. На
розтині виявлено збільшену за розмірами ліву легеню. Назвіть патологічний процес у лівій легені.
A. Нейрогуморальна гіпертрофія
B. Дисфункціональна атрофія
C. @Вікарна гіпертрофія
D. Дисциркуляторна атрофія
E. Нейротична атрофія
20. Після травматичного пошкодження печінки згодом відбулося повне відновлення будови та функції печінки.
Як називається такий вид регенерації?
A. @Реституція
B. Субституція
C. Патологічна
D. Звичайна
E. Фізіологічна
21. Назвіть форму регенерації серцевого м'яза при інфаркті міокарда і вкажіть її джерело.
A. Патологічна, фібробласти і кардіоміоцити
B. Внутрішньоклітинна, кардіоміоцити
C. @Репаративна, фібробласти
D. Фізіологічна, кардіоміоцити
E. Клітинна, фібробласти
22. Чоловiк, 74 рокiв, помер вiд хронiчної серцевої недостатностi. На аутопсiї в серцi виявлено старий
постiнфарктний рубець. Гiстологiчно: дiлянки фiброзу, гiпертрофiя кардiомiоцитiв. Проявом якої регенерацiї є
описанi змiни?
A. Патологiчної
B. @Субституцiї
C. Фiзiологiчної
D. Реституцiї
E. Метаплазiї
23. Хворий переніс повторний інтрамуральний інфаркт міокарда. Після лікування та реабілітації був виписаний у
задовільному стані під нагляд дільничого терапевта. Через 2 роки загинув у автомобільній катастрофі.
Встановити характер патологічного процесу в міокарді, який було виявлено на розтині.
A. @Крупноосередковий кардіосклероз
B. Дрiбноосередковий кардіосклероз
C. Атрофiя
D. Некроз
E. Гiперплазiя
24. У хворого з ампутованою нижньою кінцівкою у куксі з'явилися хворобливі вузлики. При мікроскопічному
дослідженні виявлено ампутаційні невроми. Назвіть патологічний процес, з яким пов'язаний розвиток невром.
A. @Патологічна регенерація
B. Повна репаративна регенерація
C. Запалення
D. Неповна репаративна регенерація
E. Метаплазія
25. У чоловіка, 38 років, після лікування опіку лівої руки сформувався келоїдний рубець. Цей процес є
прикладом:
A. @Патологічної регенерації
B. Вікарної гіпертрофії
C. Захисної гіперплазії
D. Метаплазії
E. Репаративної регенерації
26. У патоморфологічне відділення доставлено шматочок тканини з краю хронічної виразки шлунка. При
гістологічному дослідженні у ньому виявлено: некроз, грануляційна тканина, надмірний розвиток волокнистої
сполучної тканини і метаплазія епітелію. Який вид компенсаторно-пристосувального процесу має місце?
A. @Порушена регенерація з порушенням зміни фаз
B. Гіпертрофія
C. Фізіологічна регенерація
D. Репаративна регенерація (субституція)
E. Репаративна регенерація (реституція)
27. До патогiстологiчного вiддiлення надiслано шматочок тканини шлунка з наявнiстю виразки. При
гiстологiчному дослiдженнi в стiнцi виразки виявлено: некроз та лейкоцити, грануляцiйна тканина, надмiрний
розвиток склеротичної тканини та метаплазiя епiтелiю слизової в краї виразки. Про який вид регенерацiї свiдчать
цi змiни?
A. Гiпертрофiя
B. Фiзiологiчна регенерацiя
C. @Спотворена патологiчна регенерацiя з порушенням змiни фаз
D. Повна репаративна регенерацiя (реституцiя)
E. Неповна репаративна регенерацiя (субституцiя)
28. У хворого, 65 років, який зловживав палінням, при мікроскопічному дослідженні: слизова оболонка бронхів
місцями вистелена незроговілим багатошаровим епітелієм замість війчастого однорядного епітелію. Зміни в
епітелії слизової оболонки бронха - це прояви:
A. Гіпертрофії
B.Регенерації
C.@Метаплазії
D. Гіперплазії
E.Організації
29. Мікроскопічне дослідження біоптату слизової оболонки, взятого з центрального бронха шахтаря, 56 років,
виявило зміну циліндричного епітелію на зрілий багатошаровий. Цю зміну епітелію слід назвати:
A. Адаптація епітелію
B. @Метаплазія епітелію
C. Репаративна регенерація
D. Лейкоплакія
E. Гіперплазія епітелію
30. У біоптаті слизової оболонки бронха в хворого на хронічний бронхіт були виявлені ділянки багатошарового
плоского незроговілого епітелію. Діагностуйте патологічний процес у слизовій оболонці бронха.
A. Атрофія
B. Гіперплазія
C. Дисплазія
D. Злоякісна пухлина
E. @Метаплазія
31. У біоптаті слизової оболонки бронха хворого, 50 років, який 20 років хворів на хронічний бронхіт, знайдено
стоншення її, кістоподібне перетворення слизових залоз, осередки заміни призматичного епітелію на
багатошаровий плоский. Який з перелічених патологічних процесів найбільш вірогідний?
A. @Метаплазія
B. Гіперплазія
C. Гетеротопія
D. Гетероплазія
E. Дисплазія
32. У хворого на хронічний цистит у біоптаті слизової оболонки сечового міхура разом з перехідним епітелієм
виявлені осередки багатошарового плоского незроговілого епітелію. Який процес лежить в основі зазначених
змін в епітелії?
A. Гіперплазія
B.Дистрофія
C.Гіперкератоз
D. Дисплазія
E.@Метаплазія
33. При гiстологiчному дослiдженнi бiоптату, отриманого з нижньої третини стравоходу чоловiка, 57 років, з
симптомами тривалого рефлюксу шлункового вмiсту, виявлено наступнi змiни: у слизовiй оболонцi на мiсцi
багатошарового плоского епітелiю визначається одношаровий залозистий призматичний епiтелiй, з ознаками
продукцiї cлизу. Вкажiть патологiчний процес, який виник у слизовiй оболонцi:
A. @Метаплазiя
B. Гiперплазiя
C. Гiпертрофiя
D. Органiзацiя
E. Регенерацiя
34. Хвора, 45 років, скаржиться на біль і кровотечу з каріозної порожнини під час їжі. Раніше спостерігався
неспровокований біль. Під час огляду: на жувальній поверхні - глибока каріозна порожнина, виповнена тканиною
червоного кольору, під час зондування відмічається біль і кровотеча. Яка тканина розрослася у каріозній
порожнині?
A. Фіброзна
B. @Грануляційна
C. Жирова
D. Епітеліальна
E. М’язова
35. Головний клiнiко-анатомiчний прояв реакцiї строми мiокарда на хронiчну гiпоксiю органа:
A. @Розростання сполучної тканини (кардiосклероз)
B. Iнфiльтрацiя строми ліпідами
C. Формування неспецифiчної гранульоми
D. Атрофiя строми
E. Ослизнення строми
36. Пiсля загоєння рани на її мiсцi утворився рубець. Яка речовина є основним компонентом цього рiзновиду
сполучної тканини?
A. @Колаген
B. Еластин
C. Гiалуронова кислота
D. Хондроїтин-сульфат
E. Кератансульфат
37. У хворого на цукровий діабет після оперативного втручання (розкриття абсцесу задньої області шиї),
загоєння рани відбувалося впродовж 1,5 місяців, з рани постійно виділявся гнійний ексудат. Після загоєння рани
на її місці утворився грубий рубець. Який з видів загоєння мав місце у даному клінічному випадку?
A. Загоєння під струпом
B.@Вторинне загоєння
C.Епіталізація
D. Первинне загоєння
E.Змішане загоєння
38. Після падіння у дитини виник перелом однієї з кісток передпліччя, який через певний час загоївся Назвіть
форму регенерації та клітини, які приймали в цьому участь.
A. Клітинна, остеокласти, остеобласти
B. Патологічна, остеокласти
C. Фізіологічна, остеобласти
D. Внутрішньоклітинна, остеоцити
E. @Репаративна, остеокласти і остеобласти
39. При гістологічному дослідженні ділянки перелому стегнової кістки було виявлено грануляційну тканину в
стадії дозрівання, зрілу сполучну тканину та помірну кількість примітивних кісткових балочок з незначним
вмістом вапна; в оточуючих ділянках – осередкові лімфо-макрофагальні інфільтрати та гемосидероз. Діагностйте
вид ураження кістки.
A. Несправжній суглоб
B. Попередня сполучнотканинна мозоля
C. Попередня кісткова мозоля
D. @Попередня кістково-хрящова мозоля
E. Остаточна кісткова мозоля

ПАРЕНХІМАТОЗНІ ДИСТРОФІЇ
ВАРІАНТ №
1. На розтині тіла хворого, який страждав на лейкоз, і помер від наростаючої хронічної анемії, серце
збільшене в розмірах, м'яз серця на розрізі тьмяний, дряблий, блідо-сірого кольору, під ендокардом визначаються
жовті плями і смуги. Який патологічний процес розвинувся у серці?
A. Вакуольна дистрофія
B. Гіаліново-крапельна дистрофія
C. @Паренхіматозна жирова дистрофія
D. Мезенхімальна жирова дистрофія
E. Робоча гіпертрофія
2. На розтині трупа на підставі характерних макроскопічних змін діагностовано паренхіматозну жирову
дистрофію міокарда. Яка образна назва серця при цій дистрофії?
A. «Бичаче»
B. «Волохате»
C. «Панцирне»
D. «Легеневе»
E. @«Тигрове»
3. У померлого від хронічної серцево-судинної недостатності на розтині виявлено "тигрове серце". З боку
ендокарда виявляється жовто-біла посмугованість, міокард тьмяний, глинисто-жовтий. Який процес зумовив
дану патологію?
A. Вуглеводна дистрофія
B. Гіаліново- крапельна дистрофія
C. @Жирова дистрофія
D. Зерниста дистрофія
E. Амілоїдоз
4. Хвора на дифтерію померла від серцевої недостатності. На розтині: порожнини серця розширені,
міокард тьмяний, з боку ендокарда спостерігається жовта посмугованість. При гістологічному досліджені в
кардіміоцитах виявлені жирові вакуолі. Образна назва серця?
A. “Волове”
B. @“Тигрове”
C. “Волохате”
D. “Левове”
E. “Панцирне”
5. При розтині тіла, померлої від серцевої недостатності, серце збільшене в об'ємі, дрябле; міокард -
глинисто-жовтий, тьмяний; з боку ендокарда визначається жовто-біла посмугованість ("тигрове серце").
Мікроскопічно: в групах кардіоміоцитів відсутня поперечна посмугованість, цитоплазма кардіоміоцитів містить
дрібні краплі, які фарбуються суданом IV - в чорний колір. Ваш діагноз?
A. Кардіосклероз
B. @Жирова дистрофія міокарда
C. Ревматичний міокардит
D. Ожиріння серця
E. Міомаляція
6. У чоловіка, 62 років, який помер при наростаючих явищах серцевої недостатності, на розтині виявлено
збільшене в об'ємі серце. Серце дряблої консистенції, камери розтягнуті, міокард на розрізі тьмяний, глинисто-
жовтий. З боку ендокарда визначається жовто-біла посмугованість, яка особливо виражена в сосочкових м'язах.
Який патологічний процес найбільш вірогідний?
A. Ожиріння серця
B. Дилатаційна кардіоміопатія
C. Міомаляція
D. Кардіосклероз
E. @Жирова дистрофія міокарда
7. У дитини з діагностованою дифтерією зіва, померлої від гострої серцевої недостатності, на розтині
виявлено, що порожнини серця розширені в поперечнику. М'яз серця тьмяний, дряблий, на розрізі строкатий, з
жовтуватими ділянками. У цитоплазмі деяких кардіоміоцитів зі збереженою цитоплазмою виявлено дрібні
вакуолі. На заморожених зрізах вакуолі забарвлюються суданом III в оранжевий колір. Який вид дистрофії
виявлено в кардіоміоцитах?
A. @Жирова
B. Вуглеводна
C. Балонна
D. Гіаліново-крапельна
E. Гідропічна
8. У дівчини, 18 років, з'явився різкий біль при ковтанні, збільшились лімфатичні вузли шиї, підвищилась
температура до 39oC. На слизовій оболонці мигдаликів - блідо-жовті плівки, які насилу відділяються з
утворенням дефекту. Діагностовано дифтерію. Стан прогресивно погіршувався. Хвора померла на 8-й день
захворювання при наростаючих явищах серцевої недостатності. Які гістологічні зміни в кардіоміоцитах ймовірно
будуть виявлені?
A. Гідропічна дистрофія
B. @Жирова дистрофія
C. Гіаліново-крапельна дистрофія
D. Балонна дистрофія
E. Слизова дистрофія
9. Жінка, 36 років, яка хворіла на дифтерію, померла від гострої серцевої недостатності. На розтині:
порожнини серця розширені, міокард тьмяний, строкатий, на розрізі з жовтуватими ділянками. Який процес
виявлено в кардіоміоцитах?
A. Вуглеводна дистрофія
B. Балонна дистрофія
C. @Жирова дистрофія
D. Гіаліново-крапельна дистрофія
E. Гідропічна дистрофія
10. У жінки з важкою інтоксикацією, обумовленою сепсисом, який і став безпосередньою причиною
смерті, на розтині виявлено "тигрове серце". Мікроскопічно в цитоплазмі кардіоміоцитів виявлені ліпіди. Який
морфогенетичний механізм розвитку найчастіше лежить в основі даної дистрофії?
A. @Декомпозиція
B. Інфільтрація
C. Трансформація
D. Порушений синтез
E. Транссудація
11. При мікроскопічному дослідженні серця з використанням судану IV в цитоплазмі кардіоміоцитів
виявлені відкладення чорного кольору. Який морфогенетичний механізм розвитку лежить в основі даної
дистрофії?
A. @Декомпозиція
B. Інфільтрація
C. Трансформація
D. Порушений синтез
E. Інверсія
12. У жінки похилого віку з постійними матковими кровотечами "масткого" характеру при клінічному
обстеженні відмічені прояви ішемії міокарда та збільшення печінки. Вкажіть тип морфологічних змін в серці та
печінці, характерний для вказаних клінічних проявів.
A. Амілоїдоз
B. Білкова дистрофія
C. Вуглеводна дистрофія
D. Кардіосклероз, фіброз печінки
E. @Жирова дистрофія
13. У хворої на цукровий дiабет при обстеженнi виявлено збiльшення розмiрiв печiнки. У кровi
пiдвищення рiвня триглiцеридiв, лiпопротеїдiв. Який вид дистрофiї можна чекати у бiоптатi з печiнки?
A. Мукоїдне набухання
B. @Паренхiматозна жирова дистрофія
C. Гiалiноз
D. Амiлоїдоз
E. Гiалiново-крапельна дистрофія
14. При електронномікроскопічному дослідженні біоптату печінки хворого на цукровий діабет в ядрах
гепатоцитів виявлено пошкодження пор нуклеолеми та "дірчасті, пусті ядра". Який з перелічених патологічних
процесів найбільш вірогідний?
A. Каріолізис
B. @Дійсні ядерні включення
C. Дисфункціональний набряк ядра
D. Вірусообумовлені ядерні включення
E. Ядерні цитоплазматичні включення
15. У померлого від легенево-серцевої недостатності, на розтині виявлено різко збільшену недокрівну
печінку, тістоподібної консистенції, жовтого кольору. При забарвленні гематоксиліном та еозином виявлені
різних розмірів вакуолі в цитоплазмі гепатоцитів. Яка це дистрофія?
A. @Паренхіматозна жирова
B. Паренхіматозна вуглеводна
C. Гіаліново-крапельна
D. Спадкова жирова
E. Гідропічна
16. У жінки, 45 років, яка померла від хронічної алкогольної інтоксикації, на аутопсії виявлено: різко
збільшену печінку, тістоподібної консистенції, жовтуватого кольору. Мікроскопічно в цитоплазмі гепатоцитів
при фарбуванні гематоксиліном та еозином виявляються різних розмірів оптично порожні вакуолі. Який вид
дистрофії має місце?
A. Вуглеводна
B. Гіаліново-крапельна
C. Рогова
D. @Паренхіматозна жирова
E. Гідропічна
17. При огляді ротової порожнини чоловіка, який палить багато років, на слизовій оболонці язика
виявлено бляшку білого кольору. Ваш діагноз:
A. @Рогова дистрофія
B. Балонна дистрофія
C. Жирова дистрофія
D. Гіаліново-крапельна дистрофія
E. Гідропічна дистрофія
18. У хворого на слизовій оболонці порожнини рота з'явився осередок сірувато-білого кольору, щільний,
який виступає над слизовою оболонкою. Гістологічно в цій ділянці виражений гіперкератоз. Який патологічний
процес розвинувся в слизовій оболонці?
A. @Лейкоплакія
B. Гіаліноз
C. Осередковий іхтіоз
D. Локальний пухлинний амілоїдоз
E. Лейкодерма
19. При огляді порожнини рота на слизовій оболонці щоки визначається щільна пляма білого кольору
діаметром близько 1 см, яка дещо підвищується над рівнем слизової оболонки. Як називається цей патологічний
процес?
A. @Лейкоплакія
B. Еритроплакія
C. Карніфікація
D. Організація
E. Мукоїдне набухання
20. При профілактичному огляді робочих, пов'язаних з виробництвом кам'яновугільних смол, у ротовій
порожнині виявлені ділянки потовщення і зроговіння слизової оболонки, переважно щік, білястого кольору, з
шорсткою поверхнею, безболісні. Про яку патологію йдеться?
A. Папіломатоз
B. Глосит
C. @Лейкоплакія
D. Стоматит
E. Метаплазія
21. На вентральній поверхні язика у чоловіка з протезом на нижній щелепі виявлено щільні сірі бляшкі
неправильної форми з нерівною поверхнею з чіткими межами. При гістологічному дослідженні утворення
виявляються: збережена структура багатошарового плоского епітелію, потовщення його за рахунок шипуватого і
базального шарів, гіперкератозу, акантозу; спостерігається лімфомакрофагальна інфільтрація у сполучній
тканині. Встановити діагноз.
A. Еритроплакія
B. @Лейкоплакія
C. Папілома
D. Рак на місці
E. Кондилома
22. При зовнішньому огляді новонародженого виявлялися виражені зміни шкірних покривів усього тіла.
Шкіра суха, тьмяна з нерівною поверхнею і наявністю сірих пластинок, що відшаровуються. З яким видом
дистрофії це пов'язано?
A. Гідропічною
B. Гіаліново-краплинною
C. Зернистою
D. Жировою
E. @Роговою
23. При зовнішньому огляді новонародженого виявлялися виражені зміни шкірних покривів всього тіла.
Шкіра суха, тьмяна із нерівною поверхнею та з наявністю сірих пластин, які відшаровуються. З яким видом
дистрофії пов'язана ця патологія?
A. Стромально-судинною білковою
B. Стромально-судинною жировою
C. Змішаною
D. @Паренхіматозною білковою
E. Паренхіматозною жировою
24. У мертвонародженої дитини шкіра потовщена, нагадує панцир черепахи, вушні раковини
недорозвинуті. Гістологічно: надмірне зроговіння шкіри, атрофія зернистого шару епідермісу, відсутні запальні
зміни. Яке захворювання можна припустити ?
A. @Іхтіоз
B. Еритроплакія
C. Дерматоміозит
D. Лейкоплакія
E. Ксеродермія
25. Під час аутопсійного дослідження нирки було встановлено: макроскопічно - нирки звичайної форми та
розмірів, корковий шар помірно потовщений; мікроскопічно: епітеліальні клітини звивистих канальців збільшені
в розмірах, ядро зміщене на периферію, цитоплазма майже не фарбується, містить велики вакуолі, вміст яких
суданонегативний та ШИК негативний. Діагностуйте вид дистрофії.
A. Гіаліновокраплинна
B. Жирова
C. Рогова
D. @Гідропічна
E. Вуглеводна
26. У померлого, що за життя страждав на хронічний гломерулонефрит, при мікроскопичному
дослідженні нирок виявлено: клітини епітелію ниркових канальців головних відділів збільшені в розмірах, їх
цитоплазма заповнена вакуолями, що містять прозору суданонегативну рідину, а ядро зміщено на периферію.
Вкажіть, яка з паренхіматозних дистрофій найбільш вірогідна?
A. Білкова
B. Рогова
C. Гіаліново-крапельна
D. @Гідропічна
E. Жирова
27. У чоловіка 32 роки діагностовано вірусний гепатит В. Хворому проведено пункційну біопсію печінки.
При гістологічному дослідженні біоптатів печінки відмічається порушення балчастої будови з поліморфізмом
гепатоцитів, в яких багато фігур мітозу. Гепатоцити збільшені в об’ємі, цитоплазма заповнена вакуолями, які
містять прозору рідину. Який вид дистрофії характерний для даного захворювання?
A. @Гідропічна
B. Зерниста
C. Вуглеводна
D. Гіаліново-крапельна
E. Жирова
28. При мікроскопичному дослідженні біоптату нирки хворого, який страждає на цукровий діабет,
виявлено: епітелій вузького та дистального сегментів канальців нирок високий зі світлою пінистою цитоплазмою.
При забарвленні карміном Беста в цитоплазмі епітелію та просвітах канальців знайдено зерна червоного кольору.
Вкажіть яка це з перелічених видів паренхіматозних дистрофій?
A. Білкова
B. @Вуглеводна
C. Гіаліново-крапельна
D. Гідропічна
E. Жирова
29. До лікаря звернулась мати з приводу поганого самопочуття дитини - відсутність апетиту, поганий сон,
дратівливість. При біохімічному дослідженні в крові виявлено відсутність ферменту глюкоцереброзидази. Для
якої патології це характерно?
A. @Хвороба Гоше
B. Хвороба Тея-Сакса
C. Хвороба Німана-Піка
D. Хвороба Гірке
E. Хвороба Помпе
30. Хвороба Тея-Сакса характеризується відсутністю ферменту α-гексозамінідази та накопиченням
гангліозидів у лізосомах нервових клітин, що зв’язано з генетичним дефектом, який визначається як:
A. @Дефектність одного гена
B. Транслокація гена
C. Дефектність кількох генів
D. Трисомія 21 хромосоми
E. Дефектність теломера Х-хромосоми
31. При електронномікроскопічному дослідженні пунктату печінки дитини, 5 років, яка страждає на
циклічну нейтропенію, аденопатію, гепатоспленомегалію та рецидивуючі інфекції, виявлено утворення
гігантських органел, у тому числі гігантських лізосом. Який з перелічених діагнозів найбільш вірогідний?
A. @Синдром Чедіака Хігасі
B. Хвороба Тея-Сакса
C. Хвороба Садхоф
D. Ювенільний гангліозідоз
E. Хвороба Дабіна-Джонса
32. Дитина 4 років померла від легенево-серцевої недостатності. На аутопсії знайдено ущільнення
підшлункової залози, в бронхах – обтураційні ателектази та бронхоектази. В печінці – жирова інфільтрація.
Мікроскопічно в підшлунковій залозі відмічається: кістозно розширені протоки, атрoфія паренхiми залози,
дифузний фіброз та лімфо-гістiоцитарна інфільтрація. Ваш діагноз?
A. @Фетальний муковісцидоз
B. Інфекційна фетопатія
C. Гаметопатія
D. Ембріопатія
E. Хвороба Гіршпрунга
33. На розтині померлої дитини, 5 років, яка впродовж усього життя страждала на гострі респіраторні
інфекції, хронічний бронхіт, пневмонії, та системне ураження екскреторних залоз, у легенях виявлено
обтураційнi ателектази та бронхоектази, з поверхні розрізу – стікає густа гнійно-слизова маса. Мікроскопічно –
явища гнійного бронхіту та пневмонії, дрібні бронхоектази та абсцеси. Який з перелічених діагнозів найбільш
вірогідний?
A. @Легенева форма муковісцидозу
B. Бронхопневмонія
C. Бронхоектатична хвороба
D. Хронічна пневмонія
E. Хронічний бронхіт

СТРОМАЛЬНО-СУДИННІ ДИСТРОФІЇ
ВАРІАНТ №

1. Виберіть процес, що виникає під час розвитку судинно-стромальних диспротеїнозів:


A. Нагромадження інтерстиціальної рідини
B. @Нагромадження білків плазми крові в інтерстиції
C. Нагромадження клітин крові в інтерстиції
D. Нагромадження ліпідів в інтерстиції
E. Нагромадження ліпідів у стінці судин
2. Внаслідок гіпоксії й активації гіалуронідази відбувається нагромадження глікозаміногліканів в
основній речовині сполучної тканини і стінках судин. Вкажіть назву патології:
A. Фібриноїдне набухання
B. Гіаліноз
C. Фібриноїдний некроз
D. @Мукоїдне набухання
E. Набряк
3. Виберіть процес, який переважає під час розвитку мукоїдного набухання.
A. Плазморагія
B. Інтерстицій містить значну кількість фібрину
C. Відбувається проліферація фібробластів
D. Колагенові волокна розпадаються на фібрили
E. @Збільшується кількість глікозаміногліканів сполучної тканини
4. У чоловіка з мітральним стенозом ревматичної етіології в кардіохірургічному відділенні проведено
біопсію тканини вушка лівого передсердя. Мікроскопічно: зміни колагенових волокон – набряк, набухання,
базофілія (при фарбуванні гематоксилін-еозином), що свідчить про загострення процесу і наявність мукоїдного
набухання. Яке необхідне додаткове гістохімічне фарбування препарату для підтвердження діагнозу?
A. Судан ІІІ
B. Конго-рот
C. ШИК-реакція
D. Кармін Беста
E. @Толуїдиновим-синім
5. Оперативно висічено сполучну тканину деформованого мiтрального клапана, що дає базофiльну
реакцію при забарвленні гематоксилiн-еозином, а під впливом толуїдинового синього фарбується в рожевий
колiр (метахромазiя). Які зміни сполучної тканини виявляються такими типами реакцій?
A. Фiбриноїдний некроз
B. Набряк
C. Петрифiкацiя
D. @Мукоїдне набухання
E. Гiалiноз
6. При гемотрансфузії у хворої раптово з'явився озноб, розвинулася картина анафілактичного шоку.
Реанімаційні заходи виявилися неефективними, настала смерть. При гістологічному дослідженні внутрішніх
органів виявлено: осередкове потовщення базальних мембран капілярів та артеріол, їх набряк і базофілія. При
фарбуванні толуїдиновим синім у цих ділянках - виражена метахромазія. Морфологічні зміни в капілярах та
артеріолах є проявом:
A. Амілоїдозу
B. Гіалінозу
C. @Мукоїдного набухання
D. Фібриноїдного набухання
E. Фібриноїдного некрозу
7. Під час дослідження біоптату шкіри хворого на алергічний васкуліт виявлено: стінка судин потовщена,
гомогенна, пікрофуксином забарвлюється у жовтий колір, ШИК-позитивна. Назвіть патологічний процес у
стінках судин?
A. Мукоїдне набухання
B. @Фібриноїдне набухання
C. Амілоїдоз
D. Гіаліноз
E. Ліпідоз
8. При дослідженні біоптату шкіри хворого на алергічний васкуліт виявлено: стіна судин потовщена,
гомогенна, пікрофуксином забарвлена в жовтий колір, ШИК-позитивна. Назвіть різновид мезенхимальної
дистрофії.
A. Амілоїдоз
B. Мукоїдне набухання
C. Гиаліноз
D. @Фібриноїдне набухання
E. Ліпідоз
9. У померлого, 58 років, на розтині мітральний клапан деформований, потовщений, змикається не до
кінця. Мікроскопічно: ділянки колагенових волокон еозинофільні, дають позитивну реакцію на фібрин. Ваш
діагноз:
A. Амілоїдоз
B. Фібринозне запалення
C. Мукоїдне набухання
D. Гіаліноз
E. @Фібриноїдне набухання
10. Жінка, 28 років, померла від серцевої недостатності, під час дослідження серця було встановлено:
макроскопічно - стулки мітрального клапана ущільнені, потовщені та помірно деформовані; мікроскопічно -
пучки колагенових фібрил гомогенізовані, еозинофільні, метахромазія та компоненти імунних комплексів не
виявляються, оточує їх незначний макрофагальний інфільтрат. Діагностуйте вид ураження сполучної тканини.
A. Мукоїдне набухання
B. Амілоїдоз
C. @Фібриноїдне набухання
D. Гіаліноз
E. Склероз
11. При якому виді пошкодження відбувається деструкція волокон та основної речовини сполучної
тканини:
A. Амілоїдоз
B. @Гіаліноз
C. Мукоїдне набухання
D. Фібриноїдне набухання
E. Ліпідоз
12. Хворий, 66 років, за 10 років до смерті переніс перитоніт. На розтині: капсула печінки та селезінки
місцями різко потовщена, ущільнена, напівпрозора. Найвірогідніше це:
A. @Гіаліноз
B. Некроз
C. Мукоїдне набухання
D. Фібриноїдне набухання
E. Амілоїдоз
13. При рoзтинi померлої виявлено ознаки ревматичної вади серця: потовщенi, деформованi клапани, якi
набули хрящоподiбної консистенцiї, з блискучою поверхнею. Який процес спостерiгається в клапанах серця?
A. Амiлоїдоз
B. Ожирiння
C. @Гiалiноз
D. Кальциноз
E. Колагеноз
14. При макроскопічному дослідженні видаленого шлунка, лікар виявив в антральному відділі на малій
кривизні глибокий дефект, який доходить до м'язового шару, діаметром 1,5см, округлої форми з рівними краями.
В дні дефекту визначалась напівпрозора, щільна ділянка, яка на вигляд нагадувала гіаліновий хрящ. Який процес
розвинувся в дні дефекту шлунка?
А. Загальний гіаліноз
B. Амілоїдоз
C. Мукоїдне набухання
D. Фібриноїдні зміни
E. @Місцевий гіаліноз
15. Під час дослідження біоптату шкіри хворого на алергічний васкуліт виявлено: стінка судин
потовщена, гомогенна, пікрофуксином забарвлюється у червоний колір, ШИК-позитивна. Який процес
розвинувся в стінках судин?
A. Мукоїдне набухання
B. Фібриноїдне набухання
C. Амілоїдоз
D. @Гіаліноз
E. Ліпідоз
16. 78-річний чоловік, котрий біля 40 років хворів на артеріальну гіпертензію, помер від крововиливу у
головний мозок. Гістологічне дослідження тканин внутрішніх органів виявило, що стінки артеріол та капілярів
потовщені, еластичні мембрани і м’язовий шар у них атрофовані та заміщені слабо еозинофільними гомогенними
масами, просвіт цих судин щілиноподібний. Як називається описаний процес?
A. Стеатоз
B. @Гіаліноз
C. Ліпідоз
D. Диспротеїноз
E. Ксантоматоз
17. У померлого, на розтині виявлено дещо зменшену селезінку блідо-рожевого кольору. При
мікроскопічному дослідженні фолікули зменшені в об’ємі, стінки артеріол і трабекули потовщені, представлені
гомогенними еозинофільними, ШИК-позитивними масами. При додатковому фарбуванні пікрофуксином маси
забарвлюються в червоний колір. Ці зміни свідчать про наявність:
А. @Гіалінозу
B. Амілоїдозу
C. Мукоїдного набрякання
D. Фібриноїдного набухання
E. Склерозу (фіброзу)
18. До якого виду дистрофій належить амілоїдоз:
A. Жирова
B. Вуглеводна
C. Мінеральна
D. @Білкова
E. Гідропічна
19. Яку хімічну будову має компонент F амілоїду:
A. Глобулярні білки
B. @Фібрилярні білки
C. Глюкопротеїди
D. Нейтральні жири
E. Полісахариди
20. Чоловік, 19 років, з раннього дитинства страждав на бронхоектатичну хворобу. Помер від ниркової
недостатності. На розтині виявлено: окрім множинних бронхоектатичних каверн, заповнених гнійним ексудатом,
збільшені в розмірах нирки щільної консистенції; корковий шар - потовщений, білого кольору, щільний.
Піраміди нирки - анемічні, чіткі. Назвати процес у нирках:
A. @Вторинний амілоїдоз
B. Гломерулонефрит
C. Хронічний пієлонефрит
D. Вроджений кістоз нирок
E. Вторинний нефросклероз
21. На розтині тіла померлого від ниркової недостатності, який протягом останніх 5-ти років хворів на
бронхоектатичну хворобу, виявлені збільшені в розмірах нирки щільної консистенції з потовщеним корковим
шаром білого кольору та сальним блиском. Про яке захворювання нирок можна думати?
A. @Вторинний амілоїдоз
B. Гломерулонефрит
C. Первинний амілоїдоз
D. Хронічний пієлонефрит
E. Некротичний нефроз
22. У хворого 53-х років, який тривало страждав на бронхоектатичну хворобу та кровохаркання, з’явилися
набряки на обличчі й у ділянці попереку; у сечі - білок 33 мг/л. Смерть настала від легеневої кровотечі.
Результати аутопсії: нирки збільшені в об’ємі, ущільнені, поверхня розрізу має сальний вигляд. Гістологічно
відзначено відкладення у клубочках і за ходом канальців гомогенних еозинофільних мас, які вибірково
забарвлюються конго-рот і дають метахромазію з метиловим фіолетовим. Який патологічний процес мав місце в
нирках у даному випадку?
A. @Амілоїдоз
B. Гіаліноз
C. Жирова дистрофія
D. Мукоїдне набухання
E. Фібриноїдне набухання
23. При аутопсії тіла померлого, середніх років, що тривало страждав на бронхоектатичну хворобу,
виявлено, що обидві наднирковові залози різко збільшені в об'ємі за рахунок коркового шару, щільні, бліді,
сального вигляду. Мікроскопічно за ходом ретикулярної строми, у стінках судин - відкладання безструктурних,
гомогенних, еозинофільних, конго-рот-позитивних мас. Який процес зумовив ці зміни?
A. Мукоїдне набухання
B. Гіаліноз
C. @Амілоїдоз
D. Фібриноїдне набухання
E. Ліпідоз
24. Жінка, 56 років, впродовж 20 років хворіє на хронічний фіброзно-кавернозний туберкульоз легень,
надійшла в нефрологічне відділення з явищами уремії. Прижиттєва проба на наявність у нирках амілоїду
виявилася позитивною. Яка це форма амілоїдозу?
А. Сімейний вроджений
B. Первинний системний
C. Обмежений (місцевий)
D. @Вторинний системний
E. Сенильний (старечий)
25. Чоловік, 63 років, хворіє на циротичний туберкульоз легень впродовж 24 років, поступив у
нефрологічне відділення з явищами уремії. Прижиттєва проба на наявність у нирках амілоїду виявилася
позитивною. Яка форма амілоїдозу в даному випадку має місце?
A. Сімейний вроджений
B. @Вторинний системний
C. Первинний системний
D. Обмежений (місцевий)
E. Сенильний (старечий)
26. Хворий на фіброзно-кавернозний туберкульоз помер від ниркової недостатності. На розтині – запах
сечі, гіпертрофія лівого шлуночка серця, фібринозний перикардит, фібринозно-геморагічний ентероколіт. Нирки
дещо зменшені в розмірах, щільні, з множинними втягненнями. Гістологічно на препаратах, забарвлених Конго-
рот - червоні маси у клубочках та стінках судин, атрофія більшості нефронів, нефросклероз. Дайте
характеристику нирки при цій патології.
A. Вторинно-зморщені
B. Первинно-зморщені
C. @Амілоїдно-зморщені
D. Атеросклеротично-зморщені
E. Пієлонефритично-зморщені
27. Хворий, який страждав на хронічний гнійний остеомієліт, помер від хронічної ниркової недостатності.
При розтині виявлено великі щільні нирки біло-жовтого кольору з сальним блиском на розрізі. Ваш діагноз:
A. @Амілоїдоз нирок
B. Хронічний гломерулонефрит
C. Підгострий гломерулонефрит
D. Септичний нефрит
E. Гострий некротичний нефроз
28. Чоловік, 40 років, впродовж 10 років після перелому великогомілкової кістки страждав на хронічний
остеомієліт. Три роки тому з'явився нефротичний синдром. Смерть настала від уремії. На секції: нирки щільні,
білі, з рубцями в корковому шарі, на розрізі - з сальним блиском. Ваш діагноз:
A. Хронічний гломерулонефрит
B. Первинний амілоїдоз
C. Ідіопатичний амілоїдоз
D. @Вторинний амілоїдоз
E. Хронічний пієлонефрит
29. Оберіть захворювання, яке може супроводжуватися розвитком амілоїдозу з нагромадженням АА
амілоїду:
А. Гострий бронхіт
B. @Ревматоїдний артрит
C. Мієломна хвороба
D. Плазмоцитома
E. Рак шлунка
30. При розтині трупа померлого, 61 року, який страждав на ревматоїдний артрит, виявлено збільшені у
розмірах нирки, дуже щільні за консистенцією, жовто-білі з воскоподібним блиском, з осередками рубцевих змін
на поверхні. При фарбуванні конго-рот виявлено відкладення гомогенних червоних мас у капілярних петлях
клубочків, у стінках артеріол, артерій, у базальній мембрані канальців та у стромі. Розвитком якого процесу
ускладнився ревматоїдний артрит?
A. @Вторинний амілоїдоз нирок
B. Постінфекційний гломерулонефрит
C. Швидкопрогресуючий гломерулонефрит
D. Гострий некротичний нефроз
E. Фібропластичний гломерулонефрит
31. У жінки, 56 років, після перенесеного, 13 рокiв тому, шигельозу, сформувався хронічний абсцес
печінки. У даний час з'явилися ознаки ниркової недостатності. Назвіть ускладнення, пов'язане з хронічним
абсцесом?
A. @Амiлоїдоз нирок
B. Постстрептококовий гломерулонефрит
C. Осередковий iнтерстицiальний нефрит
D. Пролiферативний гломерулонефрит
E. Подагрична нирка
32. У жiнки вiком 56 рокiв, яка тривалий час (впродовж 13 рокiв) страждає на хронiчний пiлефлебiтичний
абсцес печiнки пiсля перенесеного шигельозу з'явилися ознаки ниркової недостатностi. Яке ускладнення в
нирках, може бути пов'язане з хронiчним абсцесом i, що могло послужити причиною цьому
A. @Амiлоїдоз нирок
B. Постстрептококовий гломерулонефрит
C. Осередковий iнтерстицiальний нефрит
D. Пролiферативний гломерулонефрит
E. Подагрична нирка
33. Хворий на фіброзно-кавернозний туберкульоз легень помер від хронічної легенево-серцевої
недостатності. Впродовж останніх місяців спостерігалась виражена протеїнурія. На розтині: нирки збільшені,
щільні, поверхня на розрізі - воскоподібна. Які зміни в нирках при цій формі туберкульозу могли викликати
протеїнурію?
A. @Амілоїдоз нирок
B. Туберкульоз нирок
C. Гломерулонефрит
D. Нефролітіаз
E. Некротичний нефроз
34. На розтині виявлено нирки, збільшені в розмірах, щільні, тканина сірувато-жовтого кольору, з
сальним блиском. Який патологічний процес лежить в основі описаних змін?
А. Мукоїдне набухання
B. Гіаліноз
C. Жирова дистрофія
D. @Амілоїдоз
E. Гемохроматоз
35. В яких структурах нирки нагромаджується амілоїд:
A. Миски – субепітеліально
B. @Клубочки
C. Просвіт порожнини капсули Боумена-Шумлянського
D. Чашечки – субепітеліально
E. Інтима ниркової артерії
36. Макроскопічно печінка збільшена в розмірах, ущільнена, тканина сірувато-жовтого кольору, нагадує
старе сало. Який патологічний процес лежить в основі розвитку "сальної" печінки?
A. @Амілоїдоз
B. Жирова дистрофія
C. Гіаліноз
D. Ожиріння
E. Мукоїдне набухання
37. Під час розтину тіла померлого, 56 років, який страждав на фіброзно-кавернозний туберкульоз легень,
знайдено збільшену у розмірах щільну селезінку. На розрізі тканина її коричнево-рожевого кольору, гладенька, з
воскоподібною поверхнею. Назвіть патологічний процес у селезінці?
A. @Сальна селезінка
B. Глазурована селезінка
C. Ціанотична індурація
D. Порфірна селезінка
E. Сагова селезінка
38. При розтині тіла померлої, 40 років, яка страждала на ревматоїдний артрит виявлено збільшену,
щільну селезінку. На розрізі її тканина коричнево-червоного кольору зі збільшеними фолікулами, які мають
вигляд напівпрозорих, сірувато-білуватих зерен. Вкажіть, який процес найбільш вірогідний:
A. @Сагова селезінка
B. Глазурована селезінка
C. Сальна селезінка
D. Гіаліноз селезінки
E. Порфірна селезінка
39. Чоловік. 52 років, помер від раптової зупинки серця. На розтині виявлено симетричний тип ожиріння
ІІІ ступеня, розрив стінки правого шлуночка з гемоперикардом; під епікардом - надлишкове відкладення жиру.
Мікроскопічно – жирова тканина з епікарда проникає в міокард з атрофією м'язових волокон. Який з перелічених
патологічних процесів найбільш вірогідний?
А. Ішемічна хвороба серця
B. Жирова дистрофія міокарда
C. @Просте ожиріння серця
D. Гіпертонічна хвороба
E. Гострий інфаркт міокарда
40. При мікроскопії атеросклеротичної бляшки серед колагенових волокон виявлено безліч оптично
порожніх вакуолей. При дослідженні замороженого зрізу в поляризованому світлі у цій ділянці виявляються
кристалоїдні структури з яскравою подвійною переломністю світла. При фарбуванні осмієвою кислотою ділянка
набуває чорного кольору. Ці зміни свідчать про:
A. Амілоїдоз
B. Паренхіматозну жирову дистрофію
C. Гіаліноз
D. Вакуольну (жирову) дистрофію
E. @Ліпідоз
41. Ожиріння II ступеня – це збільшення маси тіла на:
A. 20 %
B. 75 %
C. @30-40 %
D. 10-15 %
E. 45 %
42. Під час розтину тіла померлого, що за життя страждав на важку форму гіпотиреозу, виявлено:
сполучна тканина, строма органів, жирова і хрящова тканини набряклі, напівпрозорі, слизоподібні.
Мікроскопічно у тканинах виявляються зірчасті клітини з відростками, між якими знаходиться слиз. Назвіть вид
дистрофії:
A. Стромально-судинна жирова
B. @Стромально-судинна вуглеводна
C. Стромально-судинна білкова
D. Паренхіматозна білкова
E. Паренхіматозна жирова
43. У чоловіка, який хворіє на ревматоїдний артрит, при мікроскопічному дослідженні біоптату капсули
колінного суглоба виявлено: колагенові волокна, які при збереженні пучкової структури, виражено набряклі,
потовщені, при фарбуванні гематоксилін-еозином слабо базофільні. При додаткових методиках фарбування
(толуїдиновим синім) ця ділянка фарбується в червоно-фіолетовий колір. Така мікроскопічна картина свідчить
про збільшення вмісту:
A. Нейтральних мукополісахаридів
B. Глікогену
C. Ліпопротеїдів
D. Складного гіаліну
E. @Глікозаміногліканів

ЗМІШАНІ ДИСТРОФІЇ
ВАРІАНТ №1

1. При черевному тифі некротизовані пейєрові бляшки тонкої кишки, забарвлюються у жовто-коричневий
колір. Який пігмент просякає некротизовану тканину?
A. Індол
B. Гемоглобін
C. Ліпофусцин
D. @Білірубін
E. Меланін
2. Чоловік, 28 років, надійшов зі скаргами на нудоту, блювання, біль у правому підребер’ї. Об’єктивно:
жов- тяничність шкіри, склер; температура тіла підвищена, печінка збільшена, сеча темна, кал гіпохолічний.
Гіпербілі- рубінемія (білірубін прямий та непрямий), білірубінурія, уробілінурія, гіпопротеїнемія, зниження
зсідання крові.Для якого з перелічених нижче станів найбільш характерні ці зміни?
A. @Клітинно-паренхіматозна жовтяниця
B. Підпечінкова жовтяниця
C. Надпечінкова гемолітична жовтяниця
D. Гострий холецистит
E. Гострий панкреатит
3. Чоловіка, 50 років, госпіталізовано в хірургічне відділення зі скаргами на поступовий розвиток
жовтушності склер, шкірних покривів, темну сечу, знебарвлений кал. Смерть настала від прогресуючої
інтоксикації. На розтині: в голівці підшлункової залози виявлено пухлинний вузол розмірами 5х4см, що стискав
загальну жовчну протоку. Порушення обміну якої речовини має місце в даному випадку?
A. @Білірубіну
B. Ліпофусцину
C. Меланіну
D. Феритину
E. Гемосидерину
4. Хворий надiйшов до клiнiки зi скаргами на загальну слабкiсть, порушення сну. Шкiра має жовтий
колiр. У кровi: збiльшена кiлькiсть прямого бiлiрубiну, жовчних кислот. Кал ахолiчний. Для якого стану
характернi цi змiни?
A. @Механiчна жовтяниця
B. Гемолiтична жовтяниця
C. Надпечiнкова жовтяниця
D. Синдром Жiльбера
E. Хронiчний холецистит
5. Хвора, 48 років, надійшла до клініки зі скаргами на слабкість, дратівливість, порушення сну.
Об’єктивно: шкіра та склери жовтого кольору. У крові: підвищення рівня загального білірубіну з переважанням
прямого. Кал - ахолічний. Сеча - темного кольору (жовчні пігменти). Яка жовтяниця має місце в хворої?
A. @Механічна
B. Гемолітична
C. Паренхіматозна
D. Синдром Жільбера
E. Синдром Кріглера-Найяра
6. У хворого після отруєння грибами з’явилося жовте забарвлення шкіри та склер, темний колір сечі. Який
пігмент спричинює забарвлення сечі?
A. Білівердин
B. Кон’югований білірубін
C. @Некон’югований білірубін
D. Вердоглобін
E. Стеркобілін
7. У хворого з жовтяницею встановлено: пiдвищення у плазмi кровi вмiсту загального бiлiрубiну за
рахунок непрямого (вiльного), в калi та сечi - високий вмiст стеркобiлiну, рiвень прямого (зв’язаного) бiлiрубiну
в плазмi кровi в межах норми. Про який вид жовтяницi можна думати?
A. @Гемолiтична
B. Паренхiматозна (печiнкова)
C. Механiчна
D. Жовтяниця немовлят
E. Хвороба Жильбера
8. У чоловіка, 52 років, з діагнозом «підгострий септичний ендокардит» при огляді лікар відзначив
загальну блідість іктеричність шкіри, склер та видимих слизових оболонок. У крові виявлено збільшену кількість
непрямого білірубіну. Жовте забарвлення шкіри, склер і слизових оболонок - це прояв:
А. Печінкової жовтяниці
B. @Надпечінкової жовтяниці
C. Гемосидерозу
D. Жирової дистрофії
E. Підпечінкової жовтяниці
9. У жінки, доставленої після укусу гюрзи (отруйна змія), виявлено різко виражений внутрішньосудинний
гемоліз. На розтині: селезінка, кістковий мозок і лімфатичні вузли мали буре забарвлення. При мікроскопічному
дослідженні в цитоплазмі макрофагів виявлено велику кількість пігменту коричневого кольору. Який пігмент
накопичився в тканинах?
A. Гематин
B. Гематоїдин
C. @Гемосидерин
D. Ліпофусцин
E. Білірубін
10. Чоловік, 58 років, тривалий час хворів на гемобластоз. На розтині виявлено: кістковий мозок,
селезінка, печінка, лімфатичні вузли - коричневого кольору. Проведено гістохімічну реакцію Перлса.
Встановлено, що ретикулярні, ендотеліальні і гістіоцитарні елементи цих органів містять гранули синього
кольору. Який пігмент виявлено при застосуванні зазначеного забарвлення?
А. Гематоїдин
B. Білірубін
C. @Гемосидерин
Д. Гемомеланін
E. Гематопорфірин
11. При обстеженнi хворого, 47 рокiв, робiтника анiлiнової фабрики, виявлено: хронiчне недокрiв'я i
жовтяничне забарвлення шкiри. При дослiдженнi бiоптату з кiсткового мозку виявлено коричнево-жовтий
пiгмент у цитоплазмi макрофагiв (реакцiя на залiзо за Перлсом позитивна). Про який з нижченаведених пiгментiв
iде мова?
A. Лiпофусцин
B. Гематин
C. Бiлiрубiн
D. @Гемосидерин
E. Гематоїдин
12. У харкотинні хворого з мітральною вадою серця виявлені клітини, які вміщують бурий пігмент.
Реакція Перлса позитивна. Який це пігмент?
A. Порфірин
B. Гематоїдин
C. Меланін
D. @Гемосидерин
E. Білірубін
13. У хворого з вадою мiтрального клапана з'явився кашель, мокротиння іржавого кольору. Який пiгмент
обумовив такий колiр мокротиння?
A. Меланiн
B. Гемомеланiн
C. Гемоглобiн
D. Сiрчасте залiзо
E. @Гемосидерин
14. У харкотинні хворого з мітральною вадою серця виявлені клітини, які вміщують бурий пігмент.
Реакція Перлса позитивна. Назвіть дані клітини:
A. Пирогова-Лангханса
B. @Серцевих вад
C. Мікуліча
D. Тутона
E. Вірхова
15. Хворий, 56 років, помер від хронічної серцевої недостатності в результаті ревматичної вади серця. На
розтині: легені бурого кольору, збільшені, щільні. Ваш діагноз?
А. Хронічна емфізема
B. Гострий бронхіт
C. Стільникові легені
D. Хронічний бронхіт
E. Бура індурація легень
16. У чоловіка, який за життя страждав на мітральний стеноз, під час розтину виявлені ущільнені легені
бурого кольору. Про який патологічний процес в легенях йде мова?
A. Гемохроматоз
B. Ліпофусциноз
C. Жовтяниця
D. Гемомеланоз
E. @Гемосидероз
17. Після перенесеного геморагічного інсульту у хворого виникла кіста головного мозку. Через 2 роки
хворий помер внаслідок грипозної пневмонії. На розтині в мозку виявлено: кісту зі стінками біло-іржавого
відтінку, реакція Перлса позитивна. Який з процессів найбільш вірогідний у стінці кісти?
A. Інфільтрація білірубіном
B. Загальний гемосидероз
C. Місцевий гемомеланоз
D. @Місцевий гемосидероз
E. Первинний гемохроматоз
18. У хворого на гостру виразкову хворобу шлунка, що ускладнилась шлунковою кровотечею, блювотні
маси темно-коричневого кольору («кавова гуща»). Наявність якого пігменту в блювотних масах визначає їх
забарвлення?
A. Білірубіну
B. Гемоглобіну
C. @Солянокислого гематину
D. Гемомеланіну
E. Сульфіду заліза
19.У хворого на виразкову хворобу шлунка з кровотечею, при ендоскопії у шлунці виявлено рідину
кольору кавової гущі. Який пігмент зумовив такий колір вмісту шлунка?
A. Феритин
B. Гемосидерин
C. Білірубін
D. @Солянокислий гематин
E. Порфірин
20. У хворого при гастроскопiї виявлено виразку слизової оболонки шлунка з штампованими краями i
буро-чорним дном, а при гастробiопсiї пiдтверджено гостру виразку з коричнево-чорним пiгментом в її
некротичному шарi. Який це пiгмент?
A. @Солянокислий гематин
B. Гемосидерин
C. Порфiрин
D. Феритин
E. Бiлiрубiн
21. При розтині померлого, що страждав на малярію: виражена жовтушність шкіри, склер та слизових
оболонок. Селезінка збільшена, аспідно-сірого кольору. Цей колір селезінки зумовлений наявністю:
A. Меланіну
B. @Гемомеланіну
C. Ліпофусцину
D. Гемосидерину
E. Гемопорфірину
22. Хворого доставлено в лікарню у стані глибокої церебральної коми. Відомо, що за життя у нього мали
місце періодичні приступи лихоманки. На секції виявлено забарвлення у сіро-аспідний колір головного мозку,
лімфатичних вузлів, збільшених печінки і селезінки. Гістологічно в зазначених органах мав місце гемомеланоз і
гемосидероз. Який з діагнозів найбільш вірогідний?
A. @Малярія
B. Гемолітична анемія
C. Аддисонова хвороба
D. Септицемія
E. Чорна віспа
23. Під час розтину трупа чоловіка, 55 років, який впродовж останніх восьми років хворів на хронічну
форму тропічної малярії, виявлено, що сіра речовина головного мозку та селезінка аспідно-сірого кольору. Який
пігмент зумовив таке забарвлення?
A. @Гемомеланін
B. Ліпофусцин
C. Гематопорфірин
D. Меланін
E. Гемосидерин
24. У фрагменті шкіри 1х2 см, доставленому для гістологічного дослідження, виявлено новоутворення
бурого кольору діаметром 0,5 см. Мікроскопічно пухлина складається з невусних клітин у вигляді тяжів і гнізд,
розташованих у дермі, з бурим пігментом у цитоплазмі, який дає негативну реакцію Перлса. Який пігмент
найбільш вірогідний?
А. Гемосидерин
B. Гематоїдин
C Гемомеланін
D. Білірубін
E. @Меланін
25. У хворого, з двобічним ураження надниркових залоз, з’явилося темне коричневе забарвлення шкірних
покривів, при гістохімічному дослідженні шкіри реакція Перлса негативна. Який пігмент зумовив зміну кольору
шкіри?
A. @Меланін
B. Гемосидерин
C. Порфірин
D. Ліпофусцин
E. Білівердін
26. У чоловіка, 34 років, з туберкульозом надниркових залоз, шкіра сірувато-коричневого кольору,
артеріальний тиск знижений, адинамія та зниження рівня 17-оксикортикостероїдів у сечі та плазмі крові.
Порушення якого пiгменту зумовило клiнiчнi прояви у хворого?
A. Лiпохрому
B. Бiлiрубiну
C. Лiпофусцину
D. @Меланiну
E. Гемосидерину
27. У хворого, який страждав на рак нирки з множинними метастазами, також і у надниркові залози,
з'явилися: темно-коричневий колір шкірних покривів, слабкість, гіпотонія, адинамія. Який пігмент зумовив зміну
кольору шкіри?
A. Порфірин
B. @Меланін
C. Білівердін
D. Гемосидерин
E. Ліпофусцин
28. Пiсля перенесеного сепсису у хворої, 27 рокiв, з’явився бронзовий колiр шкiри, характерний для
аддiсонової хвороби. Механiзм гiперпiгментацiї полягає в пiдвищеннi секрецiї такого гормону:
A. @Меланоцитстимулюючого
B. Соматотропного
C. Гонадотропного
D. B-лiпотропного
E. Тиреотропного
29. При розтині тіла чоловіка, 87 років, звертає на себе увагу виражене виснаження і коричнево-бурий
колір серця, печінки. В кардіоміоцитах, гепатоцитах і цитоплазмі нейронів виявлено накопичення гранул
золотисто-коричневого кольору. Який пігмент виявлено?
A. @Ліпофусцин
B. Гемомеланін
C. Білірубін
D. Гемосидерин
E. Цероїд
30. У чоловіка, який за життя хворів на бронхоектатичну хворобу, пневмосклероз з вираженими явищами
кахексії, на розтині: серце зменшене в розмірах, стінки стоншені, дряблої консистенції, на розрізі тканина бурого
кольору. Відкладання якого пігменту спостерігається в міокарді?
A. @Ліпофусцину
B. Гемосидерину
C. Гематоїдину
D. Цероїду
E. Ліпохрому
31. У виснаженого хворого, який помер від раку стравоходу, на розтині виявлено: атрофію печінки, серця,
зникнення жирової клітковини. При гістологічному дослідженні в цитоплазмі кардіоміоцитів поблизу ядер
виявлені відкладення буро-жовтих зерен і глибок; негативна реакція Перлса. Яка це речовина?
А. Меланін
B. @Ліпофусцин
C. Гемосидерин
D. Ферритин
E. Гемомеланін
32. На розтині померлого від хроніосепсису, виявлено: атрофію скелетних м’язів, буру атрофію міокарда,
печінки. Порушення обміну якого пігменту виявлено у померлого?
A. Гемомеланіну
B. @Ліпофусцину
C. Гемосидерину
D. Ліпохрому
E. Меланіну
33. Жінка, 62 років, з діагнозом «Рак шлунка з множинними метастазами» померла внаслідок кахексії.
Назвіть характерні зміни серця, виявлені на розтині.
А. Дилатаційна кардіоміопатія
B. Атрофія
C."Тигрове серце"
D. Гіпертрофічна кардіоміопатія
E. @Бура атрофія міокарда
34. Хворий, який страждав на рак шлунка, помер внаслідок ракової кахексії. На розтині виявлені
характерні зміни в серці. Як назвати таке серце?
А. Панцирне
B. Волохате
C. @Буре атрофічне
D. Тигрове
E. Волове
35. Пацiєнт, 46 рокiв, звернувся до лiкаря зi скаргами на болi в дрiбних суглобах нiг та рук. Суглоби
збiльшенi, мають вигляд потовщених вузлiв. У сироватцi встановлено пiдвищений вмiст уратiв. Це може бути
спричинене:
A. @Порушенням обмiну пуринiв
B. Порушенням обмiну вуглеводiв
C. Порушенням обмiну лiпiдiв
D. Порушенням обмiну пiримiдинiв
E. Порушенням обмiну амiнокислот
36. Чоловік, 70 років, звернувся до лікаря зі скаргами на біль у дрібних суглобах рук і ніг. Суглоби
деформовані, болючі. Виявлено підвищений рівень солей сечової кислоти в крові та сечі. Про порушений обмін
яких речовин йде мова?
A. Калію
B. Кальцію
C. Хромопротеїдів
D. Ліпопротеїдів
E. @Нуклеопротеїдів
37. На розтині трупа хворого, страждаючого хронічною нирковою недостатністю в нирках знайдені
камені - урати. З порушенням якого обміну зв'язано це захворювання?
A. Білірубіну
B. Ліпопротеїдів
C. Гемосидерину
D. Хромопротеїдів
E. @Нуклеопротеїдів
38. На розтинi трупа жiнки, яка хворiла на нирковокам'яну хворобу, ускладненою двобічним
гiдронефрозом, виявлено деформацiю дрiбних суглобiв через наявнiсть утворень кам’янистої щiльностi. За життя
в кровi пiдвищився вмiст сечової кислоти. Змiни у дрiбних суглобах зумовленi вiдкладенням?
A. Лiпiдiв
B. Гiалiну
C. Холестерину
D. @Уратiв
E. Фосфатiв
39. Як назвати процес накопичення вапна в осередках туберкульозу, iнфарктах, атеросклеротичних
бляшках, старих рубцях?
A. Метаболiчне обвапнування
B. Фiбриноїдний некроз
C. Метастатичне обвапнування
D. Органiзацiя процесу
E. @Дистрофiчне обвапнування
40. Під час розтину тіла хворої, яка страждала на ревматизм з комбінованою мітральною вадою - стулки
мітрального клапана різко потовщені, зрощені між собою, кам'янистої щільності, визначається хрускіт при їх
розрізі. Який патологічний процес зумовив кам'янисту щільність стулок клапана?
А. Метаболічне звапнування
B. Метастатичне звапнування
C. Фібриноїд
D. @Дистрофічне звапнування
E. Амілоїдоз
41. На аутопсії жінки, яка хворіла на хронічну дизентерію, при гістологічному дослідженні внутрішніх
органів у стромі та паренхімі міокарда, нирок, у слизовій оболонці шлунка, у сполучній тканині легень виявлені
аморфні відкладення фіолетового кольору, що дають позитивну реакцію за Коссом. Яке ускладнення розвинулось
у хворої?
A. Метастатичне звапніння
B. Дистрофічне звапніння
C. Метаболічне звапніння
D. Амілоїдоз
E. Гіаліноз

НЕКРОЗ, АПОПТОЗ,
БІОЛОГІЧНА СМЕРТЬ
ВАРІАНТ №

1. У новонародженої дитин при огляді виявлено дефект у ділянці твердого піднебіння у вигляді
порожнини. Внаслідок порушення якого процесу він утворився?
A. @Апоптозу
B. Атрофії
C. Некрозу
D. Дистрофії
E. Запалення
2. У новонародженої дитини при огляді виявлено дефект в ділянці твердого піднебіння у вигляді
порожнини. Внаслідок якого процесу він утворився?
A. Атрофії
B. Некрозу
C. Гістогенетичного апоптозу
D. @Морфогенетичного апоптозу
E. Філогенетичного апоптозу
3. Під час огляду новонародженої дитини виявили зрощення пальців верхніх кінцівок (синдактилія). Така
вада розвитку є результатом порушення:
A. @Морфогенетичного апоптозу
B. Гістогенетичного апоптозу
C. Філогенетичного апоптозу
D. Некрозу
E. Атрофії
4. Дослідник при мікроскопічному і електронно-мікроскопічному вивченні печінки звернув увагу на те,
що деякі окремо розташовані клітини розпалися на дрібні фрагменти, оточені мембраною. У деяких з них наявні
органели, інші включають фрагменти ядра, що розпалося. Перифокальна запальна реакція відсутня. Дослідник
розцінив ці зміни, як:
A. Гіпоплазію
B. Дистрофію
C. Атрофію
D. Некроз
E. @Апоптоз
5. Хворого, 22 років, (в 20 років перехворів вірусним гепатитом В) госпіталізовано в клініку зі скаргами
на невизначений біль у правому підребер'ї, явища диспепсії, втрату ваги. В біоптаті печінкової тканини виявлено
незворотну внутрішньоклітинну зміну, що є типовою для перенесеного вірусного гепатиту В. Назвіть цю зміну.
A. Внутрішньоклітинне ожиріння
B. Тьмяний набряк печінкових клітин
C. Скупчення в ядрах клітин глікогену
D. @Тільця Каунсілмена
E. Гідропічна дистрофія
6. Чоловік, 45 років, раптово помер. На розтині в задній стінці лівого шлуночка серця виявлено інфаркт
міокарда. Які найбільш вірогідні зміни у будові кардіоміоцитів можна побачити в осередку інфаркту
мікроскопічно?
А. Звапнування
B. Жирову дистрофію
C. Вуглеводну дистрофію
D. @Каріолізис
E. Білкову дистрофію
7. Під впливом патогенних факторів може настати некротична загибель клітини, яка супроводжується
порушенням утворення енергії, внаслідок чого виникають фактори, які ведуть до загибелі ядра та розриву
клітинної мембрани. Головний з цих факторів:
A. Ушкодження апарата Гольджі
B. Порушення саркоплазматичного ретикулуму
C. Гіпертрофія мітохондрій
D. Зміни вмісту циклічних нуклеотидів
E. @Розрив лізосом зі звільненням активованих протеаз
8. При електронномікроскопічному дослідженні клітини було встановлено, що вона загинула внаслідок
некрозу, а не апоптозу, оскільки для апоптозу не є характерним:
A. Фагоцитоз апоптозних тілець
B. Розщеплення ДНК
C. Збереження цілісності органел
D. Відсутність запальної відповіді
E. @Дифузна локалізація хроматину в клітині
9. У хворого на туберкульоз у біоптаті нирки при гістологічному дослідженні в осередку казеозного
некрозу виявлено: безладно розсипані дрібні зерна хроматину. Наслідком чого є виявлені зміни?
A. Пікнозу ядер
B. Каріолізису
C. @Каріорексису
D. Мітотичної активності ядер
E. Апоптозу
10. Наявність зерен хроматину в осередку казеозного некрозу є проявом:
A. Апоптозу
B. Мітотичної активності ядер
C. @Каріорексису
D. Пікнозу ядер
E. Каріолізису
11. У пацієнта, який багато років тому хворів на сифіліс, з приводу якого він не лікувався, під час операції
хірург у нижньому краю печінки помітив чітко відмежовану, блідо-жовтувату, дряблу ділянку тканини і
резектував її. При макроскопічному дослідженні патологоанатом побачив, що ця ділянка представлена сухуватою
безструктурною, пастоподібною, жовтувато-білуватою масою. Він вирішив, що це:
A. Інфаркт
B. @Казеозний (сирнистий) некроз
C. Воскоподібний некроз
D. Фібриноїдний некроз
E. Стеатонекроз
12. На розтині тіла чоловіка, 35 років, у другому сегменті правої легені виявлено осередок ущільнення
діаметром 5 см, оточений тонкою капсулою, представлений сухою крихкою тканиною з тьмяною поверхнею. Для
якого захворювання характерні такі морфологічні зміни?
A. @Туберкулема
B. Рак легені
C. Хондрома
D. Туморозна форма силікозу
E. Післязапальний пневмосклероз
13. На розтині трупа померлого від черевної тифу, 57 років, виявлено, що м’язи передньої черевної стінки
і стегон щільні, білувато-жовтого кольору, нагадують стеаринову свічку. Проявом якого патологічного процесу є
описані зміни у м’язах:
A. @Воскоподібного некрозу
B. Апоптозу
C. Фібриноїдного некрозу
D. Колікваційного некрозу
E. Казеозного некрозу
14. При розтині трупа чоловіка, який помер від черевного тифу, виявлено, що м'язи передньої черевної
стінки і стегна щільні, ламкі, білувато-жовтуватого кольору, нагадують воскову свічку. Описана зміна є проявом:
A. @Ценкеровського некрозу
B. Фібриноїдного некрозу
C. Казеозного некрозу
D. Колікваційного некрозу
E. Апоптозу
15. У чоловiка, 58 рокiв, померлого при наростаючих явищах хронiчної серцевої недостатностi,
дiагностовано ревматичний гранульоматозний мiокардит. Мiкроскопiчно в мiокардi спостерiгаються гранульоми,
що складаються з макрофагiв з гiперхромними ядрами та свiтлою цитоплазмою, в центрi - осередок некрозу.
Який характер має некроз в серединi гранульоми?
A. @Фiбриноїдний
B. Ценкеровський
C. Казеозний
D. Колiквацiйний
E. Жировий
16. При гістологічному дослідженні нирок молодої жінки, 25 років, яка померла під час пологів, в епітелії
канальців головних відділів нефрона виявлені: конденсація хроматину ядер, розпад їх на грудочки і лізис, а також
плазморексис і цитоліз. Який патологічний процес виявлено в епітелії канальців нирок?
A. Амілоїдоз
B. Гідропічна дистрофія
C. @Некроз
D. Жирова дистрофія
E. Гіаліноз
17. Робітник хімічної промисловості звернувся до лікаря зі скаргою на стирання емалі. При огляді
виявлено розповсюджене руйнування коронок зубів з утворенням замісного дентину. Який з перелічених
діагнозів найбільш вірогідний?
A. Флюороз
B. Ерозія зубів
C. Клиноподібні дефекти
D. @Некроз твердих тканин зубів
E. Середній карієс
18. Лікар-стоматолог оглянув хворого, який скаржився на гострий зубний біль. У малому кореневому зубі
виявлено каріозну порожнину, що проникає аж до пульпи. Пульпа має вигляд сіро-чорної маси з гнильним
запахом. Назвіть патологічний процес у пульпі.
A. Атрофія
B. Серозне запалення
C. @Гангрена
D. Гіаліноз
E. Кальциноз
19. У хворого, 77 років, який страждав на атеросклероз, з'явився біль у правій стопі. Стопа збільшена в
розмірі, шкірні покрови чорного кольору, мацеровані, демаркаційна зона не виражена. Який патологічний процес
у стопі діагностовано патологоанатомом:
A. Нома
B. @Волога гангрена
C. Суха гангрена
D. Секвестр
E. Коагуляційний некроз
20. У хворого на атеросклероз, 70 років, з'явився біль у лівій стопі. До лікаря не звертався. На момент
огляду стопа збільшена в об'ємі, тканини дряблі, чорного кольору, мацеровані. Демаркаційна зона не виражена
Діагностуйте патологічний процес:
A. @Волога гангрена
B. Муміфікація
C. Коагуляційнний некроз
D. Суха гангрена
E. Секвестр
21. У хворого, який тривалий час страждає на переміжну кульгавість, тканини пальців стопи сухі, чорного
кольору, нагадують мумію. На невеликій відстані від почорнілої ділянки розташована двоколірна лінія (червона
лінія прилягає до практично незмінених тканин, а біло-жовта - до змінених тканин). Який вид некрозу в даного
хворого?
A. @Гангрена
B. Пролежень
C. Секвестр
D. Інфаркт
E. Мацерація
22. У хворого на цукровий діабет з'явився різкий біль у правій стопі. Об'єктивно: великий палець стопи
чорного кольору, тканини стопи набряклі, осередки відшарування епідермісу, виділення з неприємним запахом.
Яка клініко-морфологічна форма некрозу розвинулася в хворого?
A. Секвестр
B. Пролежень
C. Інфаркт
D. Гангрена суха
E. @Гангрена волога
23. До хірурга звернувся чоловік, 60-ти років, який тривалий час хворіє на цукровий діабет. Об'єктивно:
тканини правої стопи чорного кольору, щільні, з чіткими краями. Який діагноз поставив хірург?
A. Трофічна виразка
B. Волога гангрена
C. Пролежень
D. @Суха гангрена
E. Газова гангрена
24. У пацієнтки хірургічного відділення, 77 років, встановлено діагноз “Паховий лімфаденіт”. Об’єктивно:
у паховій області розміром з горошину рожево-червоного кольору болюча припухлість, зі скаргами на нудоту.
При спостереженні розміри припухлості прогресивно збільшувалися, у хворої з’явилася блювота з домішками
жовчі. Інтраопераційно: при розрізі шкіри над припухлістю відчувався важкий неприємний запах, підлеглі
тканини мали вигляд напіврідкої сіруватої кашоподібної маси. При лапаротомії – ділянка тонкого кишківника
ціанотичного кольору, роздута, стінка набрякла, перистальтика не виявляється. Частина стінки кишки
знаходиться у звуженому паховому кільці і не виділяється з нього. Назвіть процес у стінці кишки внаслідок
пристінкового защемлення кили?
A. Секвестр
B. Суха гангрена
C. Сухий пролежнь
D. Вологий пролежень
E. @Волога гангрена
25. Хвору, 70 рокiв, прооперовано з приводу "гострого живота". Пiд час операцiї виявлено близько 80 см
клубової кишки чорного кольору, очеревина тьмяна, покрита фiбринозним нашаруванням, перистальтика не
визначається, просвiт верхньої брижової артерiї обтурований сухим бурим згустком кровi, який легко кришиться
та прикрiплений до стiнки судини. Який процес розвинувся в кишцi?
A. Бiлий інфаркт
B. Пролежень
C. Коагуляцiйний некроз
D. @Гангрена
E. Бiлий iнфаркт з геморагiчним вiнчиком
26. Жiнка, 69 рокiв, довго хворiла на атеросклероз. Поступила в хірургічне вiддiлення з симптомами
гострого живота. При лапаротомiї виявленi: тромбоз мезентерiальної артерiї, петлi тонкої кишки набряклi,
багряно-чорного кольору, на їх серознiй оболонцi фiбринознi нашарування. Який патологiчний процес
розвинувся у кишцi хворої?
A. @Волога гангрена
B. Суха гангрена
C. Iшемiчний інфаркт
D. Секвестр
E. Коагуляцiйний некроз
27. Під час огляду дитини, яка перехворіла на кір, у м’яких тканинах щік та промежини виявлено нечітко
відмежовані, набряклі, червоно-чорного кольору ділянки, в яких виявляється помірна флюктуація. Яке
ускладнення розвинулося у дитини?
A. Пролежень
B. @Волога гангрена
C. Газова гангрена
D. Трофічна виразка
E. Суха гангрена
28. У дитини, після перенесеного кору, при огляді виявлено у м'яких тканинах щік і промежини нечітко
відмежовані, набряклі, червоно-чорного кольору ділянки, які злегка флуктують. Яке ускладнення розвинулося у
дитини?
A. @Волога гангрена (нома)
B. Суха гангрена
C. Трофічна виразка
D. Газова гангрена
E. Пролежень
29. У дитини, 6 років, госпіталізованої з коровою пневмонією, стоматолог виявив на слизовій оболонці
щоки ділянку брудно-сірого кольору розмірами 2х2,5 см без чітких меж. М'які тканини набряклі, тьмяні, з
неприємним запахом. Який найбільш вірогідний діагноз встановлено стоматологом?
A. @Нома
B. Гангренозний стоматит
C. Пустульозний стоматит
D. Флегмонозний стоматит
E. Виразковий стоматит
30. У лежачого хворого з недостатністю кровообігу, нерухомого після інсульту, шкіра і м’які тканини над
крижами почорніли і набрякли, після відторгнення епідермісу в чорних тканинах відкрились виразки. Який
процес розвинувся у хворого?
A. @Пролежні
B. Суха гангрена
C. Флегмона
D. Інфаркт
E. Абсцес
31. Під час розтину тіла жінки, яка померла внаслідок пухлинної дисемінації муцинозної
цистаденокарциноми і тривалий час мала вимушене положення в ліжку, були виявлені великі ділянки некрозу
шкіри та підлеглих м’яких тканин крижової ділянки. Діагностуйте форму некрозу.
A. Ценкеровський некроз
B. Інфаркт
C. Секвестр
D. Сирнистий
E. @Пролежень
32. При розтині померлого, 46 років, у міокарді лівого шлуночка виявлено великий осередок жовто-сірого
кольору, а в коронарній артерії - свіжий тромб. Поставте діагноз:
А. Міокардит
B. Кардіосклероз
C. @Інфаркт міокарда
D. Амілоїдоз
E. Кардіоміопатія
33. На розтині померлого, 48 років, виявлено обтурацію просвіту середньої мозкової артерії тромбом. У
тім`яно-скроневій ділянці лівої півкулі головного мозку - осередок кашоподібної консистенції сірого кольору.
Найвірогідніше це:
A. Гангрена
B. Секвестр
C. @Інфаркт
D. Казеозний некроз
E. Фібриноїдний некроз
34. При розтині померлого, 48 років, у правій скроневій частці головного мозку виявлено великий
осередок розм'якшення сірого кольору, кашоподобної консистенції. В артеріях основи мозку - численні біло-
жовті потовщення інтими, які різко звужують просвіт. Ваш діагноз?
А. @Ішемічний інфаркт
B. Абсцес мозку
C. Крововилив
D. Геморагічний інфаркт
E. Набряк мозку
35. У померлого, 75 років, який довгий час страждав на атеросклероз церебральних судин, на аутопсії
виявлені: тромбоз правої середньої мозкової артерії, великий осередок сірого кашоподібного розм’якшення
мозкової тканини неправильної форми. Який патологічний процес розвинувся в головному мозку?
A. Казеозний некроз
B. @Ішемічний інфаркт
C. Геморагічний інфаркт
D. Волога гангрена
E. Коагуляційний некроз
36. Хворого, 65 рокiв, який страждає на атеросклероз, госпiталiзовано до хiрургiчного вiддiлення з
приводу розлитого гнiйного перитонiту. Пiд час операцiї дiагностовано тромбоз брижових артерiй. Яка найбiльш
iмовiрна причина перитонiту?
A. @Геморагiчний інфаркт
B. Iшемiя ангiоспастична
C. Iшемiчний інфаркт
D. Стаз
E. Iшемiя компресiйна
37. Жінку, 68 років, яка страждає на атеросклероз, госпіталізовано до хірургічного відділення з приводу
розлитого гнійного перитоніту. Під час операції діагностовано тромбоз брижових артерій. Яка найбільш імовірна
причина перитоніту?
A. @Геморагічний інфаркт
B. Ішемія ангіоспастична
C. Стаз
D. Ішемія компресійна
E. -
38. У померлого на розтині виявлено розрив лівої середньої мозкової артерії з формуванням, з великого
осередку, порожниною виповненою згортками крови. Назвіть патологічний процесс:
А. Волога гангрена
B. Коагуляційний некроз
C. Абсцес
D. @Гематома
E. Секвестр
39. На розтинi тiла померлого, який страждав на гiпертонiчну хворобу, у лiвiй гемiсферi мозку виявлено
порожнину округлої форми 4х5 см з iржавою стiнкою, заповнену жовтуватою прозорою рiдиною. Назвiть
патологiю, яка розвинулась у головному мозку хворого:
A. @Кiста
B. Iшемiчний iнфаркт
C. Геморагiчне просякнення
D. Гематома
E. Абсцес
40. Паціент, 72 років, з діагнозом «Інфаркт правої гемісфери». Через рік під час комп'ютерної томографії
мозку в правій півкулі виявлено порожнину з гладкими стінками, заповнену ліквором. Який патологічний процес
знайдено в головному мозку?
А. @Постінфарктну кісту
B. Гідроцефалію
C. Сіре розм'якшення мозку
D. Інфаркт мозку
E. Гематому
41. У хворого, 58 років, через 6 місяців після операції та хіміотерапії з приводу злоякісної пухлини кореня
язика, виникли порушення ковтання та утруднення проходження твердої їжі. Виявлені множинні метастази у
лімфатичні вузли шиї, легені та печінку, хворий схуд на 30,0 кг. Помер при наростаючих явищах анемії та
інтоксикації. Назвати вид смерті:
A. Насильницька смерть
B. @Смерть від захворювання
C. Біологічна смерть
D. Клінічна смерть
E. Природна смерть
42. У породіллі, 28 років, через 1 годину після пологів виникла раптова атонічна маткова кровотеча.
Хвора різко зблідла, настала короткочасна зупинка дихання, серцебиття, зниження артеріального тиску. Через
декілька хвилин - повторна зупинка серцевої та дихальної функцій, розширення зіниць очей з відсутністю реакції
на світло. В результаті своєчасних реанімаційних заходів життя породіллі було врятоване. Назвати патологічний
стан, що виник у хворої:
A. @Клінічна смерть
B. Біологічна смерть
C. Непритомність
D. Кома
E. Післяреанімаційна хвороба
43. Після автокатастрофи у водія відбулась зупинка серцевої і дихальної діяльності. Після виконання
реанімаційних заходів лікарями швидкої допомоги, постраждалому було відновлено серцебиття та дихання. В
якому стані знаходився водій після аварії?
A. Біологічна смерть
B. Природна смерть
C. @Клінічна смерть
D. Фізіологічна смерть
E. Соціальна смерть
44. У приймальне відділення стаціонару доставлено невідомого у непритомному стані. Черговий лікар
запідозрив смерть людини. Які з ознак смерті є недостовірними (орієнтовними)?
A. Ознака Бєлоглазова (феномен "котячого ока")
B. Плями Лярше
C. @Нетривалі: апное, асистолія, арефлексія
D. Трупне задубіння
E. Зниження ректальної температури до 20°С
45. При огляді померлого, 72 років, патологоанатом зазначив, що м'язи померлого мають дуже щільну
консистенцію, суглоби згинаються і розгинаються погано. Як називається ця патологоанатомічна ознака смерті?
A. Трупне висихання
B. @Трупне задубіння
C. Трупне охолодження
D. Трупне розкладання
E. Трупні гіпостази
46. При зовнiшньому дослiдженнi трупа чоловiка, 69 рокiв, який помер 4 години тому, патологоанатом
вiдмiтив, що м’язи померлого мають дуже щiльну консистенцiю, суглоби згинаються та розгинаються важко. Як
називається ця патологоанатомiчна ознака смертi?
A. @Трупне заклякання
B. Трупне висихання
C. Трупне охолодження
D. Трупне розкладання
E. Трупнi гіпостази
47. На розтині померлого, 68 років, який помер на 10 добу в реанімаційному відділенні в результаті
черепно-мозкової травми, виявлено ущільнення скелетної мускулатури всього тіла, труднощі при згинанні голови
та кінцівок в суглобах. Визначити послідовність задубіння м’язів після смерті:
A. Парадоксальний
B. Висхідний
C. Ретроградний
D. Змішаний
E. @Низхідний
48. Хворий помер внаслідок туберкульозу легень. Які ознаки біологічної смерті хворого встановив
патологоанатом?
A. Зупинка дихання
B. Зупинка кровообігу
C. @Трупне заклякання
D. Розвиток інфаркту міокарда
E. Розрив аневрізми мозку
49. Аутопсію проведено на 2-й день після смерті. При огляді трупа спостерігається виражене трупне
задубіння, яке зумовлене:
A. Висиханням тканин трупа [муміфікація]
B. Аутолізом і гниттям тканин
C. Трупним гемолізом і перерозподілом крові
D. Трупними гіпостазами
E. @Зникненням з м'язів АТФ з накопиченням молочної кислоти
50. Під час огляду трупа відмічено: мутні рогівки, сухі шкірні покриви з жовто-бурими пергаментного
вигляду плямами. Встановіть вид посмертних змін.
А. @Трупне висихання
B. Перерозподіл крові
C. Трупні плями
D. Трупне задубіння
E. Охолодження трупа
51. У тяжко травмованої людини поступово настала біологічна смерть. Свідченням цього є:
A. @У клітинах відбувається аутоліз і розкладання
B. Непритомність
C. Відсутність серцебиття і дихання
D. Невпорядкованість хімічних процесів
E. Відсутність рухливості
52. Хворий, 64 років, на цукровий діабет помер в результаті трансмурального інфаркту міокарда. На
розтині встановлено: розрив стінки лівого шлуночка, гемотампонада порожнини перикарда (550,0 мл),
гіпостатична пневмонія, атеросклероз розповсюдженого характеру, набряк і трофічні виразки нижніх кінцівок та
ознаки посмертних змін. Назвати пізню ознаку смерті:
A. Охолодження (algor mortis)
B. Задубіння м’язів (rigor mortis)
C. @Трупна емфізема
D. Потьмяніння рогівки очей
E. Трупні плями

ПОРУШЕННЯ КРОВОБІГУ 1
ВАРІАНТ №

1. Після перенесеної психічної травми у пацієнтки періодично відбувається підвищення артеріального


тиску, що супроводжується головним болем, серцебиттям, загальною слабкістю. Який механізм лежить в основі?
A. @Підвищення тонусу артеріол
B. Збільшення маси циркулюючої крові
C. Зниження хвилинного об’єму крові
D. Тахікардія
E. Веноконстрикція
2. Студент на екзамені не зміг вірно відповісти на питання екзаменаційного білету, що супроводжувалося
почервонінням шкіри обличчя, відчуттям жару і невпевненістю поведінки. Який вид артеріальної гіперемії
розвинувся у даному випадку?
A. @Нейротонічна
B. Нейропаралітична
C. Метаболічна
D. Патологічна
E. Постішемічна
3. У чоловіка, 46 років, після емоційної реакції, викликаної гнівом, виник напад загрудинного болю. На
ЕКГ були встановлені ознаки порушення коронарного кровообігу. Який вид порушень міг обумовити ці явища?
A. Венозний стаз
B. Артеріальна гіперемія
C. Істинний стаз
D. Венозна гіперемія
E. @Ангіоспастична ішемія
4. При закупорці магістральної артерії верхньої кінцівки тромбом, спостерігається почервоніння шкіри
вище місця обтурації. Назвіть вид артеріальної гіперемії, яка виникає при затрудненні кровотоку по магістральній
артерії.
A. Вакатна
B. Запальна
C. @Колатеральна
D. Післяанемічна
E. Ангіоневротична
5. У хворого порушення периферичного кровообігу, основою якого є обмеження припливу артеріальної к
рові. При цьому має місце зниження місцевої температури, шкіра даної ділянки бліда. Це зумовлене:
A. Лімфостазом
B. @Ішемією
C. Артеріальною гіперемією
D. Сладж - феноменом
E. Венозною гіперемією
6. Хворомий, якому швидко видалили близько 10 літрів асцитичної рідини з черевної порожнини, раптом
знепритомнів. Ваш діагноз:
А. Тромбоз церебральних вен
B. Тромбоз церебральних артерій
С. Крововилив у головний мозок
D. Вакатна гіперемія
E. @Недокрів’я головного мозку
7. Під час емоційно напруженої роботи раптово помер молодий чоловік. На розтині виявлено
нерівномірне кровонаповнення міокарда. Гістохімічно – зниження вмісту глікогену. Електронно мікроскопічно –
деструкція мітохондрій, контрактури міофібрил. Вкажіть імовірний розлад кровообігу?
A. @Гостра ішемія
B. Хронічна ішемія
C. Вакатна артеріальна гіперемія
D. Гостра венозна гіперемія
E. Ангіоневротична артеріальна гіперемія
8. При короткочасній ішемії міокарда (до 10 хвилин) спостерігаються оборотні пошкодження
кардіоміоцитів внаслідок іонного дисбалансу в них. Назвіть зміни, які при цьому не спостерігаються в
кардіоміоцитах.
A. Порушення внутрішньо-клітинного розподілу іонів натрію та кальцію
B. Втрата іонів калію
C. Збільшення іонів кальцію
D. @Накопичення іонів калію
E. Збільшення іонів натрію
9. У хворого, 75 рокiв, який довгий час страждав на атеросклероз церебральних судин, на аутопсiї
виявленi: тромбоз правої середньої мозкової артерiї, великий осередок неправильної форми сiрого кашоподiбного
розм’якшення мозкової тканини. Який патологiчний процес розвинувся в спинному мозку?
A. @Iшемiчний інфаркт
B. Геморагiчний інфаркт
C. Гума мозку
D. Кіста мозку
E. Гематома
10. У хворого з защемленою килою під час операції у мішку кили виявлено багряно-синюшного кольору
петлю кишки з тьмяною серозною оболонкою. Вкажіть вид розладу кровообігу:
А. Місцеве венозне повнокров'я
B. Місцеве артеріальне повнокров'я
С. Компресійна ішемія
D. Обтураційна ішемія
E. @Геморагічний інфаркт кишки
11. На секції в лівій легені виявлено ділянку щільної тканини червоного кольору. Ділянка має форму
конуса, чітко відмежована від здорової тканини, основою обернена до плеври. Тканина на розрізі зерниста,
темно-червона. Ваш діагноз:
A. Геморагічний інфаркт
B. Абсцес легені
C. Гангрена легені
D. Первинний туберкульозний афект
E. Крупозна пневмонія
12. Жінку, 68 років, яка страждає на атеросклероз, госпіталізовано до хірургічного відділення з приводу
розлитого гнійного перитоніту. Під час операції діагностовано тромбоз брижових артерій. Яка найбільш імовірна
причина перитоніту?
A. @Геморагічний інфаркт
B. Ішемія ангіоспастична
C. Стаз
D. Ішемія компресійна
E. -
13. У хворого на цироз печінки після видалення з черевної порожнини 10 літрів асцитичної рідини
розвинувся колапс і гіперемія очеревини. Визначте вид артеріальної гіперемії.
А. Колатеральна
B. @Гіперемія після анемії
С. Вакатна
D. Запальна
E. В результаті шунтування крові
14. Під час гри у волейбол спортсмен після стрибка приземлився на зовнішній край стопи. Виник гострий
біль у гомілковостопному суглобі, активні рухи в ньому обмежені, пасивні - в повному обсязі, але болючі. Потім
розвинулася припухлість в області зовнішньої щиколотки, шкіра почервоніла, стала теплішою на дотик. Який вид
розладу периферичного кровообігу розвинувся?
A. Стазі
B. Емболія
C. Тромбоз
D. Венозна гіперемія
E. @Артеріальная гіперемія
15. Чоловіку з переломом плечової кістки було накладено гіпсову пов'язку. Через 2 години кисть і видима
частина передпліччя стала синюшною, холодною на дотик, набряклою. Ваш діагноз:
А. Місцеве недокрів'я
B. Місцеве артеріальне повнокров'я
С. @Місцеве венозне повнокров'я
D. Стаз
E. Тромбоз
16. На розтинi виявлено збiльшену печiнку строкатого вигляду, з малюнком мускатного горiха на розрiзi.
У просвiтi печiнкових вен знайденi пристiнковi тромби. Назвiть вид порушення кровообiгу в печiнцi.
A. Загальне венозне повнокров'я
B. Кровотеча
C. Недокрiв'я
D. @Мiсцеве венозне повнокров'я
E. Крововилив
17. На розтині померлого, що хворів на ваду серця, виявлено збільшену печінку строкатого вигляду, з
малюнком мускатного горіха на розрізі. Назвіть вид порушення кровообігу.
A. Загальне артеріальне повнокров'я
B. @Загальне венозне повнокров'я
C. Недокрів'я
D. Крововилив
E. Кровотеча
18. На розтині померлого, 48 років, який страждав на хронічну серцеву недостатність, виявлено збільшену
печінку строкатого вигляду, з малюнком мускатного горіха на розрізі. Назвіть вид порушення кровообігу:
A. @Загальне венозне повнокров'я
B. Загальне артеріальне повнокров'я
C. Недокрів'я
D. Крововилив
E. Кровотеча
19. Чоловік, 50 років, хворів на ішемічну хворобу серця, помер від лівошлуночкової недостатності. При
розтині виявлено: набряк легень, повнокров’я печінки та нирок, дрібнокрапельні крововиливи у серозних та
слизових оболонках. Ваш діагноз:
A. Артеріальна гіперемія
B. @Гостре загальнне венозне повнокров’я
C. Хронічне загальне венозне повнокров’я
D. Гостре недокрів’я
E. Хронічне недокрів’я
20. Чоловік, 44 років, з інфарктом міокарда помер від лівошлуночкової недостатності. На аутопсії: набряк
легень, дрібні крововиливи в серозних і слизових оболонках. Мікроскопічно: дистрофічні і некробіотичні зміни
епітелію проксимальних канальців нирок, у печінці - центролобулярні крововиливи і осередки некрозу. Вкажіть
вид порушення кровообігу:
A. Хронічне недокрів'я
B. Хронічне загальне венозне повнокров'я
C. Гостре недокрів'я
D. Артеріальна гіперемія
E. @Гостре загальне венозне повнокров'я
21. На 3 добу у хворої, 62 років, з гострим повторним інфарктом міокарда передньо-бокової стінки лівого
шлуночка, з’явилася різко виражена задуха, кашель з виділенням великої кількості пінистої мокроти, ціаноз
обличчя. Хвора померла на фоні прогресуючої серцевої недостатності. На розтині виявлено великі, важкі легені,
сіро-рожевого кольору. З поверхні розрізу стікає світла піниста рідина. Який патологічний процес обумовив
смерть?
A. @Набряк
B. Інфаркт
C. Пневмосклероз
D. Пневмонія
E. Гідроторакс
22. На розтині померлого виявлено, що печінка збільшена, щільна, краї закруглені, на розрізі тканина
жовтувато-коричневого кольору з темно-червоними крапками та смужками, що нагадує малюнок мускатного
горіха. Який процес лежить в основі таких змін печінки?
A. Хронічна кровотеча
B. Гостре венозне повнокров'я
C. Артеріальне повнокров’я
D. @Хронічне венозне повнокров’я
E. Артеріальне недокрів’я
23. Хворий, 68 років, який страждав на цукровий діабет та переніс інфаркт міокарда, помер при явищах
прогресуючої серцево-судинної недостатності. На розтині – ціанотична індурація селезінки та нирок, бура
індурація легень та мускатна печінка. Який вид порушення кровообігу обумовив зміни внутрішніх органів?
A. Загальна артеріальна гіперемія після анемії
B. Загальна гостра венозна гіперемія
C. @Загальна хронічна венозна гіперемія
D. Артеріальна ішемія в результаті перерозподілу крові
E. Місцева хронічна венозна гіперемія
24. При розтині тіла померлого, 73 років, який тривало хворів на ішемічну хворобу серця, виявлено:
«мускатну» печінку, буру індурацію легень, ціанотичну індурацію нирок і селезінки. Вкажіть, який з видів
порушення кровообігу призвів до таких наслідків?
A. Гостре загальне венозне повнокров'я
B.Артеріальна гіперемія
C.Гостре недокрів'я
D. @Хронічне загальне венозне повнокров'я
E. Хронічне недокрів'я
25. При розтинi тiла померлого чоловiка, 73 рокiв, який тривало страждав на iшемiчну хворобу серця з
серцевою недостатнiстю, знайдено: ”мускатна” печiнка, бура iндурацiя легень, цiанотична iндурацiя нирок та
селезiнки. Який з видiв порушення кровообiгу найбiльш iмовiрний?
A. @Хронiчне загальне венозне повнокров’я
B. Артерiальна гіперемія
C. Гостре загальне венозне повнокров’я
D. Гостре недокрів’я
E. Хронiчне недокрів’я
26. При розтині померлого, 69 років, який тривало страждав на ІХС з серцевою недостатністю, виявлено:
“мускатна” печінка, бура індурація легень, ціанотична індурація нирок та селезінки. Вкажіть, який з видів
порушення кровообігу найбільш вірогідний?
A. Гостре недокрів’я
B. Артеріальна гіперемія
C. Гостре загальнне венозне повнокров’я
D. @Хронічне загальне венозне повнокров’я
E. Хронічне недокрів’я
27. На розтині померлого виявлено: печінка збільшена, щільна, краї закруглені, на розрізі тканина
жовтувато-коричневого кольору з темно-червоними крапками і смужками, що нагадує малюнок мускатного
горіха. Який патологічний процес лежить в основі таких змін печінки?
А. Артеріальне повнокров'я
B. Гостре венозне повнокров'я
С. @Хронічне венозне повнокров'я
D. Артеріальне недокрів'я
E. Хронічна кровотеча
28. У хворого на алкогольний цироз печінки зі скаргами на загальну слабкість, задишку виявлено: знижен
ня артеріального тиску, асцит, розширення поверхневих вен передньої черевної стінки, спленомегалію. Яке пору
шення гемодинаміки спостерігається у хворого?
A. Недостатність правого шлуночка серця
B. Тотальна серцева недостатність
C. Недостатність лівого шлуночка серця
D. Колапс
E. @Синдром портальної гіпертензії
29. При розтині померлого, 43 років, який страждав на ІХС з розвитком інфаркту міокарда,
патологоанатом виявив набряк легень. Які зміни могли зумовити набряк легень?
A. Ішемія малого кола
B. Гостре загальне недокрів'я
C. Гостра правошлуночкова недостатність
D. @Гостра лівошлуночкова недостатність
E. Стаз крові
30. У хворого, який страждав на ревматизм, виявлено стеноз мітрального клапана. Смерть настала від
серцево-легеневої недостатності. На розтині виявлено буру індурацію легень. Назвіть вид порушення кровообігу
при цих змінах в легенях.
A. Хронічна правошлуночкова недостатність
B. @Хронічна лівошлуночкова недостатність
C. Гостра лівошлуночкова недостатність
D. Гостра правошлуночкова недостатність
E. Портальна гіпертензія
31. Чоловік, 52 років, помер від хронічної серцевої недостатності в результаті ревматичної вади серця. На
розтині: легені бурого кольору, збільшені в розмірах, щільні. Як називаються такі зміни в легенях?
А. Хронічна емфізема
B. Мускатні легені
С. Стільникові легені
D. Хронічний бронхіт
E. @Бура індурація легень
32. Молодий чоловік з дитинства страждає на ревматизм, клінічно діагностовано стеноз мітрального
клапана. Останніми роками - часті епізоди серцево-судинної недостатності, майже постійний кашель з іржавим
мокротинням. Назвіть можливі зміни в легенях?
А. Пневмосклероз
B. Емфізема легень
С. Ателектаз легень
D. @Буре ущільнення легень
E. Бронхоектази
33. На розтині легені щільні, коричневого кольору за рахунок відкладання ендогенного пігменту. Відомо,
що за життя у хворого спостерігався хронічний венозний застій у малому колі кровообігу. Який патологічний
процес викликав таку картину?
A. Меланоз
B. Жовтяниця
C. @Гемосидероз
D. Порфірія
E. Кальциноз
34. Утворення "клітин серцевих вад" є проявом цитофізіологічної активності клітин за типом:
A. Резорбції
B. Макропіноцитозу
C. Апоптозу
D. @Фагоцитозу
E. Мікропіноцитозу
35. Чоловік, 63 років, страждав на емфізему легень, помер від прогресуючої серцевої недостатності. На
розтині виявлено: мускатний цироз печінки, ціанотична індурація нирок і селезінки, асцит, набряки нижніх
кінцівок. Для якого типу серцевої недостатності характерні дані зміни?
A. Загальна серцева недостатність
B.Гостра лівошлуночкова недостатність
C.Гостра правошлуночкова недостатність
D. @Хронічна правошлуночкова недостатність
E.Хронічна лівошлуночкова недостатність
36. Хворий помер при явищах серцево-судинної недостатності. Результати розтину: постінфарктний карді
осклероз, гіпертрофія міокарда і дилатація його порожнин, особливо правого шлуночка. Печінка збільшена, з гла
дкою поверхнею, на розрізі - повнокровна, з темно-червоним крапом на буруватому фоні поверхні тканини. Гісто
логічно: повнокров'я центральних відділів часточок; у периферичних відділах, навколо портальних трактів, гепат
оцити в стані жирової дистрофії. Як називаються описані зміни печінки?
A. Цироз печінки
B. Псевдомускатна печінка
C. Амілоїдоз
D. @Мускатна печінка (хронічне венозне повнокров'я)
E. Стеатоз печінки
37. При гістологічному дослідженні печінки виявлено: венозне повнокров'я центру часточок, дистрофія і
атрофія гепатоцитів в осередках венозного застою, жирова дистрофія гепатоцитів по периферії часточок з
наявністю розростання сполучної тканини в місцях атрофії гепатоцитів. Про який патологічний процес йдеться?
A. Жировий гепатоз
B. Біліарний цироз печінки
C. @Мускатна печінка з передциротичними явищами
D. Гепатит
E. Токсична дистрофія печінки
38. Хворий, 63 років, який страждав на ішемічну хворобу серця і переніс повторний інфаркт міокарда,
помер від прогресуючої серцево-судинної недостатності. На розтині виявлено: збільшену щільну селезінку,
темно-вишневого кольору на розрізі. При мікроскопічному дослідженні органа встановлено склероз пульпи і
атрофія фолікулів. Яким терміном визначають зміни селезінки?
A. Порфірова селезінка
B. “Сагова селезінка”
C. “Сальна селезінка”
D. @Ціанотична індурація селезінки
E. Септична селезінка
39. У хворого з варикозним розширенням вен пiд час огляду нижнiх кiнцiвок виявляється: цiаноз,
пастознiсть, зниження температури шкiри, поодинокi петехiї. Який розлад гемодинамiки має мiсце у хворого?
A. @Венозна гіперемія
B. Компресiйна ішемія
C. Обтурацiйна ішемія
D. Тромбоемболiя
E. Артерiальна гiперемiя
40. У хворого на пневмосклероз розвинулася легенева гіпертензія та правошлуночкова серцева
недостатність з асцитом та набряками. Який основний патогенетичний механізм розвитку набряків у цього
хворого?
A. Зменшення осмотичного тиску крові
B. Збільшення онкотичного тиску тканин
C. Зменшення онкотичного тиску крові
D. @Збільшення гідростатичного тиску у венах
E. Збільшення проникності стінок судин
41. На розтині тіла чоловіка, який помер від раку шлунка, виявлено множинні метастази в печінку,
лімфатичні вузли, карциноматоз очеревини. Встановлено кахексію, анасарку, гідроторакс, гідроперикард, асцит.
Визначити характер набряків:
A. Запальні
B.Застійні
C.@Дистрофічні
D. Серцеві
E. Ниркові
42. Хвора, 75 років, з тривалим флеботромбозом (близько 10 років) звернулась зі скаргами на значні
набряки нижніх кінцівок. При огляді стоп та гомілок вони збільшені в об’ємі, шкіра їх суха, напружена. Назвіть
вид набряку:
A. @Застійні
B. Запальні
C. Ниркові
D. Дистрофічні
E. Серцеві
43. У хворого з нефротичним синдромом спостерігаються масивні набряки обличчя та кінцівок. Який
патогенетичний механізм є провідним у розвитку набряків?
A. @Зниження онкотичного тиску крові
B. Підвищення судинної проникності
C. Підвищення гідродинамічного тиску крові
D. Лімфостаз
E. Підвищення лімфовідтоку
44. У хворого з церебральною формою перебiгу хронiчної малярiї спостерiгається коматозний стан.
Вкажiть, з чим пов'язаний його розвиток?
A. Депонуванням збудника малярiї в капiлярах
B. Виникненням тромбозу судин мозку
C. Розвитком загальної анемiї
D. Масивним крововиливом у тканину мозку
E. @Зі стазом еритроцитiв у капiлярних структурах мозку

ПОРУШЕННЯ КРОВООБІГУ 2
ВАРІАНТ №

1. У 68 рiчної жiнки, внаслiдок стенозуючого атеросклерозу вiнцевих артерій, розвинувся гострий iнфаркт
мiокарда. Пiд час секцiї виявлено: в просвiтi передньої мiжшлуночкової гiлки лiвої коронарної артерiї - крихкi
сiро-жовтi маси, що нагадують атероматозний детрит i повнiстю обтурують просвiт судини, в товщi передньої та
бiчної стiнок лiвого шлуночка - трансмуральну дiлянку неправильної форми, сiро-жовтого забарвлення, по
периферiї оточену геморагiчним вiнцем. Дiагностуйте причину iнфаркту.
A. Ретроградна емболія
B. Жирова емболія
C. @Тромбоз
D. Тканинна емболія
E. Емболiя стороннiми тiлами
2. При розтині померлого від набряку легень у міокарді виявлено великий осередок жовто-сірого кольору,
а в коронарній артерії – свіжий тромб. Уточніть діагноз:
A. @Інфаркт міокарда
B. Кардіосклероз
C. Міокардит
D. Амілоїдоз
E. Кардіоміопатія
3. Назвіть вид інфаркту за макроскопічними ознаками, який найчастіше зустрічається в міокарді.
A. Геморагічний
B. @Білий з геморагічним вінчиком
C. Білий
D. Змішаний
E. Червоний
4. У померлого, 75 років, який довгий час страждав на атеросклероз церебральних судин, на аутопсії у
правій тім'яно-скроневої ділянці головного мозку виявлено осередок неправильної форми, дряблої консистенції,
сірого кольору. Яка найбільш вірогідна причина розвитку цього процесу?
A. @Тромбоз правої середньої мозкової артерії
B. Тромбоз правої передньої мозкової артерії
C. Тромбоз правої задньої мозкової артерії
D. Тромбоз судини м'якої мозкової оболонки
E. Тромбоз базилярної артерії
5. При розтині померлого, хворого на атеросклероз, у головному мозку виявлено тромбоз гілки
внутрішньої сонної артерії та сірого кольору осередок вологого розм'якшення тканини. Який патологічний
процес виявлено в головному мозку?
A. @Ішемічний інфаркт
B. Геморагічна інфільтрація
C. Гематома
D. Енцефаліт
E. Пухлина мозку
6. У хворого на атеросклероз та гіпертонічну хворобу, 72 років, виникла гостра ішемія в басейні правої
середньо-мозкової артерії. Через 3 доби хворий помер. На розтині в головному мозку виявлені набряк та
набрякання, в ділянці підкоркових вузлів правої півкулі - осередок розм’якшення мозку сіро-червоного кольору,
неправильної форми, розмірами 4х4х3 см. Патологічний процес у головному мозку, що обумовив смерть:
A. @Білий інфаркт з геморагічним просоченням
B. Гематома
C. Абсцес
D. Червоний (геморагічний) інфаркт
E. Білий (ішемічний) інфаркт
7. Хворому з тромбофлебітом правої нижньої кінцівки проведено оперативне лікування гомілкових вен.
При дослідженні післяопераційного матеріалу виявлені тромби, що закупорюють просвіти судин, мікроскопічно
в тромботичних масах виявлені фібрин, поодинокі лейкоцити та велика кількість еритроцитів. Назвати вид
тромбу:
A. Змішаний
B. @Обтуруючий червоний
C. Обтуруючий білий
D. Пристінковий білий
E. Пристінковий червоний
8. У патоморфологiчну лабораторiю надiслано сполучну тканину з крупною артерiальною судиною, у
просвiтi якої - сiруватого кольору суха кришковата маса, щiльно з'єднана зі стiнкою судини. При гiстологiчному
дослiдженнi виявлено: в артерiальнiй судинi - маса облiтеруючого характеру, яка мiстить у собi фiбрин,
тромбоцити, малу кiлькiсть еритроцитiв з переважною кiлькiстю лейкоцитiв з явищем початкової органiзацiї.
Рiзновид якого процесу спостерiгаеться в просвiтi артерiальной судини?
A. Тромбоембол
B. Облітеруючий гiалiновий тромб
C. @Облiтеруючий бiлий тромб
D. Згорток кровi
E. Сполучна тканина
9. При мікроскопічному дослідженні згортка крові, який було виявлено в просвіті стегнової артерії,
патологоанатом побачив нерівномірне розташування фібрину і зруйновані формені елементи крові, зв'язок з
інтимою і повне закриття просвіту судини. Що це?
A. Тромбоембол
B. Посмертний згусток крові
C. @Змішаний тромб
D. Гіаліновий тромб
E. Червоний тромб
10. У померлого, 78 років, який впродовж 30 років страждав на цукровий діабет, на розтині встановлено:
виразковий атеросклероз аорти з аневризмою у черевному відділі діаметром 10 см, що нагадує форму мішка,
порожнина якої виповнена тромботичними масами. Визначити вид тромбу:
A. Пристінковий
B. Кулеподібний
C. Обтураційний
D. @Дилатаційний
E. Тромб-вершник
11. При мікроскопічному дослідженні пупкової вени новонародженого, померлого від інтоксикації: стінка
судини - з дифузною запальною інфільтрацією, у просвіті її – обтуруючий тромб з великою кількістю
нейтрофільних лейкоцитів з явищами каріорексису та колоніями бактерій. Який з наслідків тромбозу найбільш
вірогідний?
A. Асептичний лізис
B. @Септичний лізис
C. Організація та каналізація тромбу
D. Перетворення на тромбоембол
E. Петрифікація тромбу
12. Хворий, 55 років, якого було прооперовано з приводу гострого апендициту, на 5 добу, піднявшись з
ліжка, відчув недостатність повітря, різкий ціаноз обличчя, втратив свідомість. Реанімаційні заходи не дали
результату і хворий помер. На розтині виявлено тромбоемболію легеневого стовбура. Що є джерелом
тромбоемболії?
A. Тромбоз у лівому шлуночці серця
B. Тромбоз ворітної вени
C. Тромбоз брижової артерії
D. @Тромбоз вен нижніх кінцівок
E. Кулеподібний тромб передсердя
13. Хворого на атеросклероз, 65 рокiв, госпiталiзовано до хiрургiчного вiддiлення з приводу розлитого
гнiйного перитонiту. Пiд час операцiї дiагностовано тромбоз брижових артерiй. Яка найбiльш iмовiрна причина
перитонiту?
A. @Геморагiчний інфаркт
B. Iшемiя ангiоспастична
C. Iшемiчний інфаркт
D. Стаз
E. Компресiйна ішемiя
14. Хворий, 75 рокiв, був оперований з приводу раку передмiхурової залози, помер раптово на 4-ту добу
пiсля оперативного втручання. При розтинi тiла померлого у просвiтах головного стовбура i бiфуркацiї легеневої
артерiї були виявленi i легко видаленi крихкi маси темно-червоного кольору з тьмяною поверхнею. Такi ж маси
знаходились у порожнинi правого шлуночка серця. Який рiзновид порушення кровообiгу призвiв до раптової
смертi хворого?
A. @Тромбоемболiя легеневої артерiї
B. Тканинна емболiя
C. Тромбоз легеневої артерiї
D. Iнфаркт мiокарда
E. Парадоксальна емболiя
15. У хворого з тромбофлебітом нижніх кінцівок раптово після навантаження виникли: задишка, різкий
біль у грудях, ціаноз, набухання шийних вен. Яке найбільш імовірне порушення кровообігу виникло у хворого?
A. @Тромбоемболія легеневої артерії
B. Тромбоемболія вінцевих судин
C. Тромбоемболія судин головного мозку
D. Тромбоемболія мезентеріальних судин
E. Тромбоемболія ворітної вени
16. У хворого, 72 років, який страждав на тромбофлебіт глибоких вен гомілок, раптово виник біль за
грудниною, задуха, ціаноз обличчя і він помер. На розтині: в правому шлуночці і основному стовбурі легеневої
артерії виявлені щільні тромбоемболи, сіро-червоного кольору, з нерівною поверхнею. Причина смерті хворого:
A. Хронічне венозне повнокров'я
B. Гостре венозне повнокров'я
C. Больовий шок
D. Тромбоемболія легеневих артерій
E. @Пульмокоронарний рефлекс
17. У хворої, 65 років, яка страждала на тромбофлебіт глибоких вен гомілки, в поліклініці, в черзі на
прийом до лікаря, раптово настала смерть. На розтині трупа в загальному стовбурі та біфуркації легеневої артерії
знайдені вільно лежачі червоні пухкі маси з тьмяною, гофрованою поверхнею. Який патологічний процес у
легеневій артерії виявив патологоанатом?
A. Тканинну емболію
B. @Тромбоемболію
C. Тромбоз
D. Жирову емболію
E. Емболію сторонніми тілами
18. При розтині тіла чоловіка, 63 років, який помер від раптової зупинки серця, встановлено, що
можливим механізмом смерті є пульмокоронарний рефлекс. Який процес є причиною його виникнення?
A. Жирова емболія
B. @Тромбоемболія основного стовбура легеневої артерії
C. Тромбоз системи легеневих мікросудин
D. Повітряна емболія
E. Тромбоемболія дрібних гілок легеневої артерії
19. При розтині померлого від раптової зупинки серця, 63 років, встановлено, що можливим механізмом
смерті є пульмокоронарний рефлекс. Який патологічний процес є причиною його виникнення?
A. Жирова емболія
B. Тромбоз системи легеневих мікросудин
C. Повітряна емболія
D. @Тромбоемболія truncus pulmonalae
E. Тромбемболія arterie pulmonalae
20. У жiнки, 30 рокiв, з гiпернефроїдним раком правої нирки з проростанням пухлини у ниркову вену пiд
час операцiї нефректомiї, настала раптова зупинка серця. При аутопсiї в порожнинi правого шлуночка виявлено
пухлинний ембол великих розмiрiв. Вкажiть який з вказаних факторiв мав провiдне патогенетичне значення у
розвитку раптової смертi.
A. Спазм вiнцевих артерiй
B. Спазм бронхiального дерева
C. Великi розмiри ембола
D. @Пульмокоронарний рефлекс
E. Спазм гiлок легеневої артерiї
21. У хворого з тромбофлебiтом нижнiх кiнцiвок раптово пiсля навантаження виникли задишка, рiзкий
бiль у грудях, цiаноз, набухання шийних вен. Яке найбiльш iмовiрне порушення кровообiгу виникло у хворого?
A. @Тромбоемболiя легеневої артерії
B. Тромбоемболiя вiнцевих судин
C. Тромбоемболiя судин головного мозку
D. Тромбоемболiя мезентерiальних судин
E. Тромбоемболiя ворiтної вени
22. На секції виявлено: множинні геморагічні інфаркти легень, у деяких судинах легень - бурого кольору
щільні маси, які не прикріплені до стінки судин, варикозне розширення вен нижніх кінцівок, в яких наявні
тромби. Про який процес іде мова:
A. Застійний тромбоз судин легеневої артерії
B. Жирова емболія судин легеневої артерії
C. Тканинна емболія судин легеневої артерії
D. @Тромбоемболія судин легеневої артерії
E. Геморагічна бронхопневмонія
23. У хворого, який страждав на гнійний отит і тромбоз сигмоподібного синуса твердої мозкової
оболонки, відрив частини тромбу може загрожувати розвитком:
A. @Тромбоемболії гілок легеневої артерії
B. Тромбоемболії судин головного мозку
C. Розвитком сірого розм'якшення мозку
D. Тромбоемболії судин сітчастої оболонки ока і розвитком сліпоти
E. Тромбоемболії коронарних артерій
A. Місцевого недокрів'я
24. Чоловік помер після абдомінальної операції. На розтині у венах малого таза були виявлені численні
тромби. Клінічно був зафіксований пульмокоронарний рефлекс. Де слід шукати тромбоемболи?
А. Портальна вена
B. @Легеневий стовбур
С. Лівий шлуночок серця
D. Головний мозок
E. Вени нижніх кінцівок
25. У хворого, 27 років, з політравмою (закрита травма грудної клітини, закритий перелом правого стегна)
через дві години після проведення скелетного витягнення різко погіршився стан і, на фоні гострої легенево-
серцевої недостатності, настала смерть. При гістологічному дослідженні кровоносних судин легень та головного
мозку померлого, при забарвленні Суданом ІІІ, виявлені краплі помаранчевого кольору, які закупорюють просвіт
судин. Яке ускладнення політравми розвинулося у хворого?
A. @Жирова емболія
B. Газова емболія
C. Мікробна емболія
D. Тромбоемболія
E. Повітряна емболія
26. У хворого, 31 років, з діагнозом: «Перелом правого стегна з великою кількістю уламків», на 3 добу від
одержання травми з’явилися скарги на біль у грудній порожнині, утруднене дихання. Через добу на фоні
прогресуючої серцево-дихальної недостатності настала смерть. При гістологічному дослідженні у кровоносних
судинах легень та головного мозку виявлені суданофільні краплини оранжевого кольору, які повністю
перекривали просвіти судин мікроциркуляторного русла. З яким ускладненням пов’язана смерть хворого?
A. Медикаментозною емболією
B. Газовою емболією
C. @Жировою емболією
D. Мікробною емболією
E. Тромбоемболією
27. Хворий, 63 років, помер від гострої легенево-серцевої недостатності. Під час розтину було запідозрено
жирову емболію легеневої артерії. Який метод забарвлення мікропрепаратів потрібно застосувати для
підтвердження діагнозу?
А. Гематоксилін-еозін
B. Пікрофуксин
С. @Судан III
D. Конго-рот
E. Метиленовий синій
28. Під час дорожньо-транспортної пригоди водій отримав поранення в шию розбитим склом. Кровотеча
була невелика, але через декілька хвилин потерпілий помер при явищах гострої ядухи. При розтині серця
померлого у заповненій водою порожнині перикарда виділяються пухирці. Вкажіть імовіриний патологічний
процес.
A. @Повітряна емболія
B. Газова емболія
C. Жирова емболія
D. Тромбоемболія
E. Емболія чужорідними тілами
29. У хворого після відкритого перелому ключиці раптово настала смерть. На розтині в правому шлуночці
серця і легеневих артеріях виявлено пінисту кров. Що стало причиною смерті?
A. Жирова емболія
B. Бактеріальна емболія
C. Кровотеча
D. Тканинна емболія
E. @Повітряна емболія
30. До лікарні потрапив хворий з пораненням яремної вени. При обстеженні встановлено зниження
артеріального тиску, збільшення центрального венозного тиску, ціаноз. Яке порушення периферичного
кровообігу виникло у хворого?
A. Емболія портальної вени
B. Тромбоемболія
C. Ішемія
D. Емболія малого кола кровообігу
E. @Емболія великого кола кровообігу
31. Чоловік з кесонною хворобою помер з ознаками гострих порушень мозкового кровообігу в басейні а.
meningea media лівої півкулі головного мозку. На розтині виявлено осередок сірого розм'якшення мозку в
зазначеній ділянці розмірами 6х7х3,4 см. Що спричинило смерть людини?
A. @Газова емболiя
B. Жирова емболiя
C. Тромбоз
D. Тромбоемболiя
E. Атеросклероз судин
32. У льотчика, який загинув внаслідок розгерметизації кабіни літака, при розтині були виявлені: добре
виражене трупне задубіння, емфізема підшкірної клітковини тулуба та обличчя; у легенях спостерігався набряк
та периваскулярні крововиливи, у венах - піниста та рідка кров. При гістологічному дослідженні внутрішніх
органів у судинах виявлено велику кількість пухирців, у печінці - жирова дистрофія; в головному і спинному
мозку - множинні дрібні ішемічні осередки сірого розм'якшення. Вкажіть найбільш ймовірну причину таких змін.
A. Повітряна емболія
B. Тромбоемболія
C. Тканинна емболія
D. Жирова емболі
E. @Газова емболія
33. Робітник ртутного комбінату звернувся до лікаря зі скаргами на слабкість, зниження працездатності,
млявість. Об’єктивно: пожовтіння шкіри та слизових оболонок, окремі діапедезні крововиливи. В біоптаті
печінки - виражені дистрофічні зміни гепатоцитів, дрібні крововиливи та окремі гемосидеробласти. Назвати
механізм крововиливів:
A. Роз’їдання стінки протеолітичними ферментами
B. Розрив стінки судини
C. Підвищення проникності артерій середнього калібру
D. Підвищення проникності вен середнього калібру
E. @Підвищення проникності судин мікроциркуляторного русла
34. Хворого, 40 років, який страждає на хронічну виразкову хворобу шлунка, госпіталізовано зі скаргами
на гострий біль в епігастральній ділянці, одноразову блювоту у вигляді “кавової гущі”. В стаціонарі
зареєстровано мелену. Назвати патологічний процес, що обумовив дані симптоми захворювання:
А. Гематома
В. Геморагічна інфільтрація
С. Внутрішня кровотеча
D. Геморагічний синдром
Е. @Зовнішня кровотеча
35. На розтині померлого в правій півкулі головного мозку виявлено порожнину неправильної форми
4х2,5см, заповнену червоними згустками крові і розм'якшеною тканиною мозку. Назвіть відповідну патологію
головного мозку.
А. Кіста
B. Геморагічне просочення
С. Ішемічний інфаркт
D. @Гематома
E. Абсцес
36. У хворого похилого віку виникло гостре порушення мозкового кровообігу з коматозним станом і
смертельним наслідком. При розтині в правій півкулі головного мозку виявлено велику порожнину, заповнену
кров'ю. Яки патологічний процес виявлено в головному мозку?
A. Інфаркт мозку
B. Геморагічну інфільтрацію
C. @Гематому
D. Діапедезний крововилив
E. Пухлину мозку
37. При розтині померлого, що страждав на гіпертонічну хворобу, у речовині головного мозку виявлено
порожнину, стінки якої мають іржавий колір. Що передувало виникненню даних змін?
A. @Гематома
B. Діапедезні крововиливи
C. Ішемічний інфаркт
D. Плазморагії
E. Абсцес
38. Хворий, 35 років, поступив у лікарню зі скаргами на багаторазову блювоту, пронос, падіння
артеріального тиску, серцебиття. Свій стан пов'язує з прийомом недоброякісної їжі. Дослідження показало
інфікування сальмонелами. В аналізі крові - збільшення числа еритроцитів в одиниці об'єму. Вкажіть розлад
кровообігу:
A. @Згущення крові
B. Гемоліз еритроцитів і компенсаторне посилення гемопоезу
C. Загальне артеріальне повнокров'я
D. Поліцитемія
E. Гіперхромна анемія
39. При аутопсiї померлої вiд полiорганної недостатностi, виявленi численнi згортки кровi в мiкросудинах
легень, головного мозку нирок, печiнки, наднирникiв, а також осередковi геморагiї в слизових i серозних
оболонках органiв. Який процес виявлено в даному випадку?
A. Дисемiнований тромбоз
B. @Дисемiноване внутрiшньосудинне згортання кровi
C. Дисемiнований стаз
D. Дисемiноване гостре недокрiв'я
E. Дисемiнований садж-феномен
40. Хворого, 30 років, госпіталізовано до інфекційного відділення у тяжкому стані з гектичною
температурою, болем у горлі, інтоксикацією. Смерть настала на третю добу на фоні прогресуючої серцево-
судинної недостатності, що супроводжувалася симптомами шлунково-кишкової кровотечі, діапедезними
крововиливами у шкіру. На розтині виявлено: субарахноїдальні крововиливи. Гістологічно у внутрішніх органах
– фокуси некрозів та крововиливів, стаз та тромбоз судин мікроциркулярорного русла, сладж-феномен. Назвати
патологічний процес, що розвинувся:
A. Гіпертермія
B. Шок
C. @Синдром дисемінованого внутрішньосудинного згортання крові
D. Геморагічний синдром
E. Тромбоцитопенічний синдром
41. Хворого, 30 років, госпіталізовано до інфекційного відділення у тяжкому стані з гектичною
температурою, болем у горлі, інтоксикацією. Смерть настала на третю добу на фоні прогресуючої серцево-
судинної недостатності, що супроводжувалася симптомами шлунково-кишкової кровотечі, діапедезними
крововиливами у шкіру. На розтині виявлено: субарахноїдальні крововиливи. Гістологічно у внутрішніх органах
– фокуси некрозів та крововиливів, стаз та тромбоз судин мікроциркулярорного русла, сладж-феномен. Назвати
патологічний процес, що розвинувся:
A. Гіпертермія
B. Шок
C. Синдром згортання крові
D. @Тромбо-геморагічний синдром
E. Тромбоцитопенічний синдром
42. Хворий, 29 років, у результаті автотранспортної пригоди отримав множинні закриті переломи, помер
на фоні прогресуючої дихальної недостатності. На розтині: в легенях - множинні ателектази, під вісцеральною
плеврою - петехіальні крововиливи, з поверхні розрізу стікає рожева рідина пінистого вигляду. Мікроскопічно: в
судинах мікроциркуляторного русла - стаз, сладж-феномен, тромбоз; у просвітах альвеол - серозна рідина.
Назвати процес у легенях:
A. Гостра лівошлуночкова недостатність
B. Пневмонія
C. Хронічна лівошлуночкова недостатність
D. @Шок
E. Туберкульоз
43. Жінка, 59 років, поступила в лікарню з ознаками ексикозу в стані глибокого шоку, аутопсійне
мікробіологічне дослідження кишкового вмісту виявило наявність вібріону Ель Тор. Який вид шоку спричинив
смерть хворої?
A. Травматичний
B. @Гіповолемічний
C. Токсичний
D. Анафілактичний
E. Кардіогенний
44. Хвору, 42 років, через тиждень після вживання сирих домашніх яєць, госпіталізовано до інфекційного
відділення зі скаргами на слабкість, втому, гарячку, нудоту та діарею (до 15 разів на добу). У стаціонарі з’явилася
прогресуюча олігоанурія і настала смерть в результаті шоку. Назвати вид шоку у хворої:
A. @Інфекційно-токсичний
B. Кардіогенний
C. Септичний
D. Анафілактичний
E. Травматичний
45. У хворого на сепсис у фiналi захворювання вiдмiчено швидкопрогресуючу, наростаючу легеневу
недостатнiсть. На секцiї - помiтний набряк тканини легень. Гiстологiчно вiдмiчається потовщення та набряк
альвеолярних перегородок, наявнiсть гiалiнових мембран та некротизованих епiтелiальних клiтин в альвеолах.
Вказанi змiни характернi для:
A. Набряку легень гемодинамiчного характеру
B. Розвитку бронхоектатичної хвороби
C. Обструкцiї бронхiв
D. @Респiраторного дистрес-синдрому
E. Приєднання в фiналi захворювання пневмонiї
46. У хворого на алкогольний цироз печінки - скарги на загальну слабкість, задишку. Виявлено: зниження
артеріального тиску, асцит, розширення поверхневих вен передньої черевної стінки, спленомегалію. Ваш діагноз:
A. Недостатність правого шлуночка серця
B.Тотальна серцева недостатність
C.Недостатність лівого шлуночка серця
D. @Синдром портальної гіпертензії
E.Колапс
47. Жінка, 45 років, хворіє на рак лівої молочної залози. На лівій руці є ознаки недостатності лімфатичної
системи - набряк кінцівки, збільшення лімфовузлів. Яка форма недостатності лімфообігу спостерігається у
хворої?
A. @Механічна недостатність
B. Динамічна недостатність
C. Резорбційна недостатність
D. Змішана недостатність
E. -
48. У хворої, 52 років, через рік після правобічної мастектомії з приводу злоякісної пухлини поступово
збільшився об’єм правої руки. Поверхня шкіри стала грубою, в зморшках, з ціанозом. При пальпації рука з
підвищеною шорохуватістю, омозолінням. При гістологічному дослідженні біоптату встановлено потовщення
дерми в 4 рази, дифузне розростання волокнистої сполучної тканини з ділянками гіалінозу. Назвати вид
порушеного лімфообігу:
A. @Механічна
B. Резорбційна зі зміною дисперсності білків
C. Динамічна
D. Резорбційна зі збільшенням проникності лімфатичних капілярів
E. Функціональна
49. У 49 річної жінки після видалення лімфатичних вузлів правої аксилярної ділянки з приводу раку
грудної залози через півроку виявлено збільшення правої верхньої кінцівки в об’ємі та значне її ущільнення,
гладка та напружена шкіра, через щілиноподібні дефекти якої на поверхню витікає прозора рідина. Діагностуйте
вид порушення лімфообігу:
A. Хронічна набута місцева лімфедема
B. Хронічна вроджена місцева лімфедема
C. Гостра місцева лімфедема
D. Хронічна загальна лімфедема
E. Гостра загальна лімфедема
50. На розтині тіла жінки, 69 років, виявлено пухлину передньої стінки черевної порожнини. З черевної
порожнини вилучено 1500,0 мл рідини з сіруватим відтінком. В результаті лабораторних аналізів у рідині
виявлено велику кількість жиру. Назвати патологічний процес у черевній порожнині:
A. Марантичні набряки
B. Анасарка
C. @Хільозний асцит
D. Фібринозний перитоніт
E. Гнійний перитоніт
51. У хворого, 27 років, гострий кавернозний вторинний туберкульоз ускладнився правобічним
ексудативним плевритом. При пункції правої плевральної порожнини отримано 800,0 мл рідини білого кольору,
яка нагадує розведене молоко. Назвати патологічний процес у плевральній порожнині.
A. Гемоторакс
B. @Хілоторакс
C. Гідроторакс
D. Емпієма плеври
E. Фібринозний перикардит

ПРОЛІФЕРАТИВНЕ ЗАПАЛЕННЯ.
ВАРІАНТ №

1. До якого виду запалення відноситься запалення з утворенням поліпів та шпичастих кондилом?


A. Гранулематозне
B. Ексудативне
C. Фібринозне
D. Проміжне
E. @Продуктивне
2. У хворого на операції видалено з порожнини носа сосочкові розростання слизової оболонки. При
гістологічному дослідженні в них виявлено дифузну інфільтрацію лімфоцитами, плазмоцитами, макрофагами.
Назвіть вид запалення.
A. З утворенням шпилястих кондилом
B. Інтерстиціальне
C. Гранульоматозне
D. @З утворенням поліпів
E. Ексудативне
3. Мiкроскопiчно в стромi мiокарда виявлено запальний iнфiльтрат, який складається з гiстiоцитiв, фiбробластiв,
лiмфоцитiв i плазмоцитiв. Про який мiокардит йде мова?
A. @Промiжний продуктивний
B. Серозний
C. Гранулематозний
D. Альтеративний
E. Паренхiматозний проміжний
4. При мікроскопічному дослідженні міокарда чоловіка, померлого від серцевої декомпенсації, виявлено склероз
навколосудинної сполучної тканини та дифузну інфільтрацію її лімфоцитами, макрофагами, плазмоцитами та
поодинокими нейтрофілами. Вкажіть, який з перелічених видів запалення найбільш вірогідний?
A. @Інтерстиційне продуктивне
B. Гранульоматозне
C. Альтеративне
D. Ексудативне дифузне
E. Ексудативне осередкове
5. При мікроскопічному дослідженні міокарда, померлого від декомпенсації серцевої недостатності, виявлено
склероз навколосудинної сполучної тканини і дифузна інфільтрація її лімфоцитами, макрофагами, плазмоцитами
і поодинокі нейтрофілами. Вкажіть, який з перерахованих видів запалення найбільш вірогідний:
A. Гранульоматозне
B. Альтеративне
C. @Інтерстиціальне продуктивне
D. Ексудативне дифузне
E. Ексудативне осередкове
6. Хворий, 58 років, помер від прогресуючої серцевої недостатності. На розтині: серце розширене у поперечнику,
дрябле, м’яз на розрізі нерівномірного кровонаповнення, пістрявий. При гістологічному дослідженні: у міокарді -
повнокров’я, у стромі - лімфогістіоцитарні інфільтрати, що розсувають кардіоміоцити. Виявлені морфологічні
зміни свідчать про:
A. @Негнійний проміжний міокардит
B. Венозне повнокров’я
C. Жирову дистрофію міокарда
D. Кардіосклероз
E. Інфаркт міокарда
7. У хворого, 45 років, з країв виразкового дефекту гомілки хірург висік на біопсію шматочок. Клінічний діагноз:
«Трофічна виразка гомілки». Патолог на висновках виявленого морфологічного процесу країв виразки
діагностував вкриту виразками гуму. Вкажіть, на висновках якого з указаних патологічних процесів у краї
виразкового дефекту патолог поставив даний діагноз?
A. Серозне запалення, набряк, венозне повнокров'я, мікротромби
B. Сірозно-гнійне запалення, неспецифічна грануляційна тканина
C. Дифузне гнійне запалення, некроз
D. Фібринозно-гнійне запалення, некроз
E. @Лімфоплазматичний інфільтрат, продуктивні васкуліти
8. При мікроскопічному дослідженні біопсійного матеріалу патологоанатом виявив у тканині печінки
гранульоми, які складаються переважно з Т-лімфоцитів та епітеліоїдних клітин, серед яких поодинокі гігантські
багатоядерні клітини типу Пирогова-Лангханса. В центрі гранульоми розміщена невелика ділянка казеозного
некрозу. Який патологічний процес має місце?
A. @Проліферативне запалення
B. Коагуляційний некроз
C. Альтеративне запалення
D. Ексудативне запалення
E. Колікваційний некроз
9. У дiвчинки, 3,5 років, яка гостро захворiла дiагностовано черевний тиф. Смерть настала на 5 добу вiд початку
захворювання. На розтинi виявлено, що груповi фолiкули на протязi 1,5 м термiнального вiддiлу клубової кишки
збiльшенi, повнокровнi, набряклi, виступають над слизовою оболонкою. На їх поверхнi видно борозни i звивини,
якi нагадують поверхню мозку На розрiзi - сiро-червонi, соковитi. Гiстологiчно виявлено: повнокров'я, набряк,
тифознi вузлики, якi складаються з великих мононуклеарних клiтин, що мають свiтлу цитоплазму i мiстять
черевнотифознi палички. Про який патологiчний процес можна думати:
A. @Гостре пролiферативне запалення
B. Гіпертрофiю
C. Катаральне запалення
D. Гiперплазiю
E. Хронiчне пролiферативне запалення
10. При гістологічному дослідженні біоптату шкіри хворого, 24 років, виявлено: казеозниий некроз, оточений
клітинним інфільтратом з лімфоцитів, серед яких зустрічаються окремі гігантські клітини, має місце розростання
сполучної тканини, ендоваскуліти. Який характер патологічного процесу?
A. Продуктивне інтерстиціальне запалення
B. @Продуктивне гранульоматозне запалення
C. Абсцес
D. Катаральне запалення
E. Серозне запалення
11. Хворого госпіталізовано у лікарню з діагнозом «Трихінельоз». Які форма та вид запалення розвинулись у
м'язах хворого навколо тваринного паразиту?
A. Ексудативне серозне
B. @Продуктивне гранульоматозне
C. Ексудативне катаральне
D. Ексудативне фібринозне
E. Продуктивне інтерстиціальне
12. При мікроскопічному дослідженні біоптату нирки виявлені осередки, в центрі яких знаходяться зернисті
еозинофільні маси, оточені інфільтратом з лімфоцитів, епітеліоїдних клітин і поодиноких клітин Пирогова-
Лангханса. Назвіть патологічний процес.
A. @Гранульоматозне запалення
B. Коагуляційний некроз
C. Казеозний некроз
D. Альтеративне запалення
E. Проліферація і диференціювання макрофагів
13. При розтині померлого в легенях і печінці виявлено багато просоподібних вузликів. Мікроскопічно:
гранульоми з осередком некрозу в центрі, по периферії - епітеліоїдні, лімфоїдні, плазматичні клітини, а також
велику кількість клітин Пирогова-Лангханса, які переважають в інфільтратах. Яку гранульому знайдено в
легенях?
A. Макрофагальну
B. Фагоцитарну
C. Епітеліоїдно-клітинну
D. Чужорідних тіл
E. @Гігантоклітинну
14. При мiкроскопiчному дослiдженнi вилученого лiмфатичного вузла у хворого з пiдозрою на туберкульоз
виявленi гранульоми, у центрi яких - осередок казеозного некрозу, далi - вали з епiтелiоїдних клiтин i лiмфоцитiв,
мiж якими зустрiчаються поодинокi гiгантськi клiтини з великим числом ядер, розташованих пiдковоподiбно.
Назвiть гiгантськi клiтини за автором.
A. Вiрхова
B. Мiкулича
C. @Пирогова-Лангханса
D. Анiчкова
E. Рида-Березовського-Штернберга
15. При розтині померлого, 49 років, у легенях і печінці виявлено багато просоподібних вузликів. Мікроскопічно:
виявлено гранульоми з осередком некрозу в центрі, по периферії - клітини Пирогова-Лангханса, лімфоїдні,
плазматичні клітини, а також велику кількість епітеліоїдних, які переважають в інфільтратах. Яку гранульому
знайдену в легенях?
A. Макрофагальну
B. Фагоцитомну
C. Епітеліоїдноклітинну
D. Чужорідних тіл
E. @Гігантоклітинну
16. У чоловiка, 30 рокiв, при гiстологiчному дослiдженнi бiоптату з шийного лiмфатичного вузла виявленi
гранульоми, якi складаються з епiтелiоїдних, лiмфоїдних, багатоядерних гiгантських клiтин типу Пирогова-
Лангханса. У центрi гранульом визначається некроз. Який збудник потрiбно виявити в зонi некрозу для
пiдтвердження дiагнозу «Туберкульоз»?
A. Блiду трепонему
B. @Мiкобактерiю Коха
C. Бацили Волковича-Фрiша
D. Мікобактерію лепри
E. Сальмонели
17. При мiкроскопiчному дослiдженнi шийного лiмфатичного вузла виявлено скупчення епiтелiоїдних клiтин,
лімфоцитів i гiгантських клiтин Пирогова-Лангханса. В центрi - казеозний некроз. Вкажiть найбiльш iмовiрну
патологiю:
A. @Туберкульоз
B. Риносклерома
C. Саркоїдоз
D. Сап
E. Сифiлiс
18. У бiоптатi лiмфатичного вузла виявлено скупчення навколо осередків некрозу епiтелiоїдних клiтин i
лiмфоцитiв з наявнiстю гiгантських багатоядерних клiтин Лангханса. Для якого захворювання характернi
виявленi морфологiчнi змiни?
A. @Туберкульозу
B. Лiмфогранулематозу
C. Склероми
D. Прокази
E. Сифiлiсу
19. У жiнки, 30 рокiв, при гiстологiчному дослiдженнi шийного лiмфовузла виявлено порушення малюнка
внаслiдок розростання епiтелiоїдних, лiмфоїдних клiтин i макрофагiв з ядрами у виглядi пiдкови, в центрi деяких
скупчень клiтин - безструктурнi дiлянки блiдо-рожевого кольору з уламками ядер. Для якого захворювання
характернi такi змiни?
A. @Туберкульоз
B. Лiмфогранулематоз
C. Актиномiкоз
D. Метастаз пухлини
E. Сифiлiс
20. У хворого з субфібрильною температурою в біоптаті збільшеного лімфатичного вузла виявлені численні
гранульоми, які містять у центрі казеозний некроз, оточений епітеліоїдними клітинами, велетенськими
багатоядерними клітинами Пирогова-Лангханса і лімфоцитами. Для якого захворювання характерні такі
патогістологічні зміни?
A. @Туберкульоз
B. Лімфогранулематоз
C. Лепра
D. Лімфолейкоз
E. Банальний лімфаденіт
21. При мікроскопічному дослідженні тканини легень виявлено ділянку запалення, яка складається з осередку
некрозу, оточеного правильними рядами епітеліоїдних, лімфоїдних клітин, є плазматичні клітини, макрофаги і
гігантські багатоядерні клітини Пирогова-Лангханса. Ваш діагноз.
A. Банальне продуктивне запалення
B. Запалення при лепрі
C. @Запалення при туберкульозі
D. Ексудативне запалення
E. Альтеративне запалення
22. При мікроскопічному дослідженні легень хворого, 52 років, виявлені осередки некрозу, оточені валом
епітеліоїдних клітин і лімфоцитів. Між лімфоцитами й епітеліоїдними клітинами розміщені великі клітини
округлої форми з великою кількістю ядер, розташованних на периферії. Ваш діагноз:
A. @Туберкульозна гранульома
B. Саркоїдозна гранульома
C. Сифілітична гранульома
D. Лепрозна гранульома
E. Риносклерома
23. При мікроскопічному дослідженні некропсії легень хворого, 52 років, виявлено осередки некрозу, оточені
валом епітеліоїдних клітин та лімфоцитів з домішкою макрофагів та плазматичних клітин. Між лімфоцитами та
епітеліоїдними клітинами розміщені великі клітини округлої форми з великою кількістю ядер, розміщених на
периферії. Як називається виявлене утворення?
A. Гранульома навколо паразита
B. Осередкове інтерстиціальне продуктивне запалення
C. Сифілітична гранульома
D. @Туберкульозна гранульома
E. Гранульома навколо стороннього тіла
24. При мікроскопічному дослідженні біоптату шкіри виявляються гранульоми, які складаються з епітеліоїдних
клітин, оточених переважно Т-лімфоцитами. Серед епітеліоїдних клітин розташовуються поодинокі гігантські
багатоядерні клітини типу Пирогова-Лангханса. У центрі деяких гранульом - ділянки казеозного некрозу.
Кровоносні судини відсутні. Ваш діагноз:
A. @Туберкульоз
B. Сифіліс
C. Лепра
D. Риносклерома
E. Сап
25. У хворого на облiтеруючий перикардит в оперативно вилученому перикардi мiкроскопiчно виявленi
множиннi казеозно-епiтелiоїднi гранульоми з гiгантськими багатоядерними клiтинами Пирогова-Лангханса.
Уточнiть дiагноз.
A. Ревматичний перикардит
B. Некротичний перикардит
C. Фiбропластичний перикардит
D. Iдiопатичний перикардит
E. @Туберкульозний перикардит
26. На слизовій оболонці правого піднебінного мигдалика спостерігається безболісна виразка з гладким
лакованим дном та рівними хрящоподібної консистенції краями. Мікроскопічно - запальний інфільтрат
складається з лімфоцитів, плазмоцитів, невеликої кількості нейтрофілів та епітеліоїдних клітин. Яким збудником
викликаються такі зміни?
A. @Бліда трепонема
B. Стафілокок
C. Кандиди
D. Стрептокок
E. Актиноміцети
27. При мікроскопії біоптату з виразки порожнини рота виявлені осередки казеозного некрозу, оточені
плазмоцитами, епітеліоїдними і лімфоїдними клітинами, зрідка гігантськими багатоядерними клітинами типу
Пирогова-Лангханса, зустрічаються дрібні судини з ознаками ендо- і периваскуліту. Вкажіть захворювання.
A. Риносклерома
B. Туберкульоз
C. Лепра
D. @Сифіліс
E. Сап
28. При мікроскопії біоптату печінки виявлені гранульоми, які складаються з плазматичних, лімфоїдних,
гігантських багатоядерних клітин типу Пирогова-Лангханса, дрібних судин з ознаками ендо- та периваскуліту,
зустрічаються осередки казеозного некрозу. Про яке захворювання можна думати?
A. Риносклерому
B. Туберкульоз
C. Лепру
D. @Сифіліс
E. Сап
29. У тканині печінки виявлено округле утворення діаметром 0.5 см. Мікроскопічно воно має наступну будову: у
центрі – некротичні маси, їх оточує грануляційна тканина з наявністю у її складі плазматичних, лімфоїдних
клітин і кровоносних судин з явищами васкуліту. Який діагноз необхідно поставити на підставі даних
мікроскопії?
A. @Солітарна гума печінки
B. Солітарна аденома печінки
C. Солітарна лепрома печінки
D. Хронічний абсцес печінки
E. Рак печінки
30. На секцiї виявлено аневризму висхiдного вiддiлу аорти. Мiкроскопiчно в стiнцi аорти виявлено запальнi
iнфiльтрати з лiмфоцитiв, плазматичних клiтин, фiбробластiв, гiгантських клiтин Пирогова-Лангханса. Про яке
захворювання слiд думати?
A. Туберкульоз
B. Ревматизм
C. Гiпертонiчну хворобу
D. @Сифiлiс
E. Атеросклероз
31. Хвора, 22 років, постійно мешкає в Західній Україні, скаржиться на утруднене носове дихання. При огляді
носової порожнини слизова оболонка зі значною кількістю кірок, на носових раковинах виявлені інфільтрати
рожевого кольору, без виразок. Клінічний діагноз: склерома носа. Проведено біопсійне дослідження. Які
морфологічні зміни, характерні для риносклероми, могли бути виявлені в біоптаті?
A. Гранульоми з клітинами сторонніх тіл
B. Гранульоми з клітинами Вірхова
C. Інтерстиціальне запалення
D. Гранульоми з клітинами Пирогова-Лангханса
E. @Гранульоми з клітинами Мікулича
32. Хворий, 22 років, постійно мешкає в Західній Україні, скаржиться на утруднене носове дихання. При
морфологічному дослідженні біоптату слизистої оболонки носової порожнини виявлені лімфоїдні, епітеліоїдні,
плазматичні клітини і клітини Мікулича. Ваш діагноз:
A. Сап
B. Туберкульоз
C. Лепра
D. Сифіліс
E. @Риносклерома
33. Хворий, 46 років, скаржиться на утруднене дихання. У біоптаті з потовщеної слизової носа виявлено: клітини
Мікулича, скупчення епітеліоїдних клітин, плазмоцитів, лімфоцитів, гіалінових куль. Ваш діагноз?
A. Аденовірусний риніт
B. Алергічний риніт
C. @Склерома
D. Риновірусная інфекція
E. Менінгококовий назофарингіт
34. У біоптаті слизової носа виявлені епітеліоїдні клітини, плазмоцити, клітини Мікулича, еозинофілні тільця
Русселя. Ваш висновок:
A. Алергічний риніт
B. Сифіліс
C. Туберкульоз
D. Респіраторносинцитіальна інфекція
E. @Риносклерома
35. При гістологічному дослідженні біоптату перегородки носа хворого, який страждав на утруднене носове
дихання, у слизовій оболонці виявлено гранульоматозне запалення з наявністю в гранульомах клітин Мікулича і
бактерій. Ваш діагноз.
A. Сифіліс
B. @Риносклерома
C. Туберкульоз
D. Сап
E. Лепра
36. При дослідженні біоптату виявлено гранульому, що складається з лімфоцитів, плазматичних клітин,
макрофагів з пінистою цитоплазмою (клітини Мікулича), багато гіалінових куль. Про яке захворювання можна
думати?
A. Лепра
B. @Риносклерома
C. Сифіліс
D. Туберкульоз
E. Актиномікоз
37. Хворому зі скаргами на ядуху проведено біопсію слизової оболонки носової порожнини. Встановлено діагноз:
риносклерома. Які клітини типові для даного захворювання при мікроскопічному дослідженні?
A. Клітини Пирогова-Лангханса
B. Плазмоцити
C. Лімфоцити
D. @Клітини Мікулича
E. Тільця Шаумана
38. При гістологічному дослідженні біоптату перегородки носа хворого, який страждає на утруднене носове
дихання, в слизовій оболонці знайдено гранульоматозне запалення з наявністю в гранульомах клітин Мікулича і
бактерій Волковича-Фріша. Який найбільш вірогідний діагноз?
A. @Риносклерома
B. Сифіліс
C. Туберкульоз
D. Сап
E. Лепра
39. При гістологічному дослідженні біоптату шкіри виявлені гранульоми, які складаються з макрофагальних
вузликів з наявністю лімфоцитів і плазматичних клітин. Крім того, зустрічаються великі макрофаги з жировими
вакоулями у вигляді куль, які містять упаковані збудники захворювання (паличку Ганзена). Грануляційна
тканина добре васкуляризована. Для якого захворювання характерна описана картина?
A. Туберкульозу
B. @Лепри
C. Сифілісу
D. Риносклероми
E. Сапу
40. При гістологічному дослідженні біоптату шкіри виявлені гранульоми, які складаються з макрофагальних
вузликів з наявністю лімфоцитів і плазматичних клітин. Крім того, зустрічаються великі макрофаги з жировими
вакоулями у вигляді куль, які містять упаковані збудники захворювання (клітини Вірхова). Грануляційна тканина
добре васкуляризована. Назвіть захворювання:
A. Туберкульоз
B. @Лепра
C. Сифіліс
D. Риносклерома
E. Сап
41. У чоловіка, 58 років, померлого при наростаючих явищах хронічної серцевої недостатності, діагностовано
ревматичний гранульоматозний міокардит. Мікроскопічно: в міокарді спостерігаються гранульоми, що
складаються з гіперхромними ядрами і світлою цитоплазмою макрофагів, в центрі - осередок некрозу. Назвіть
характер некрозу в гранульомі?
A. @Фібриноїдний
B. Ценкерівський
C. Казеозний
D. Колікваційний
E. Жировий
42. У хворого, померлого від серцевої недостатності, при патоморфологічному дослідженні виявлено: стулки
мітрального клапана деформовані, потовщені, зрослися по краях; у сполучній тканині міокарда - дифузно
розкидані вузлики, які складаються з ділянок фібриноїдного некрозу, навколо яких скупчуються макрофагоцити,
що нагадують гігантські багатоядерні клітини. Подібні осередки оточені лімфоцитами та поодинокими
плазматичними клітинами. Яка з перерахованих гранульом має місце у даного хворого?
A. @Ревматична
B. Туберкульозна
C. Актиномікотична
D. Сифілітична
E. Лепрозна
43. Хворому проведено резекцію 1/3 печінки з багатокамерним ехінококом. Яку характерну реакцію оточуючих
тканин печінки на тваринний паразит виявив патологоанатом при гістологічному дослідженні операційного
матеріалу?
A. Альтеративну тканеву реакцію
B. Дряблу консистенцію печінки
C. Утворення специфічних гранульом
D. @Утворення неспецифічних гранульом
E. Ексудативну тканеву реакцію
44. При мiкроскопiчному дослiдженнi пiсляоперацiйного iнфiльтрату навколо шовного матерiалу виявленi
епiтелiоїдноклiтиннi гранульоми з гiгантськими багатоядерними макрофагами. До якого виду гранульом вони
вiдносяться:
A. Туберкульознi гранулеми
B. Ревматичнi гранульоми
C. Мiкотичнi гранульоми
D. @Гранульоми стороннiх тіл
E. Лепроматознi гранульоми
45. При мiкроскопiчному дослiдженнi шийного лiмфатичного вузла виявлено скупчення епiтелiоїдних клiтин,
лiмфоцитiв i гiгантських клiтин Пiрогова-Ланганса. Казеозний некроз вiдсутнiй. Вкажiть найбiльш вiрогiдну
патологiю.
A. @Саркоїдоз
B. Сап
C. Сифіліс
D. Туберкульоз
E. Риносклерома
ІМУННЕ ЗАПАЛЕННЯ

1. Хворий, 68 років, з діагнозом "Абсцес легені", помер при явищах наростаючої серцевої недостатності. На
розтині виявлено: лімфатичні вузли середостіння збільшені, соковиті, рожево-червоного кольору. Селезінка
збільшена, дрябла. Чим можна пояснити дані зміни?
A. Компенсаторною гіперплазією
B. Гормональною гіперплазією
C. Венозним застоєм
D. Гнійним запаленням в лімфовузлі і селезінці
E. @Захисною гіперплазією
2. Біля інфікованої рани збільшилися регіонарні лімфовузли. При гістологічному дослідженні в них виявлені
збільшення кількості макрофагів, лімфоцитів і лімфатичних фолікулів в кірковому шарі, а також велика кількість
плазматичних клітин. Який процес в лімфатичних вузлах відображають виявлені гістологічні зміни?
A. @Антигенну стимуляцію
B. Набуту недостатність лімфоїдної тканини
C. Природжену недостатність лімфоїдної тканини
D. Пухлинну трансформацію
E. Реакцію гіперчутливості
3. При гістологічному дослідженні лімфатичного вузла виявлено велику кількість збільшених
лімфоїдних фолікулів з крупними центрами розмноження і великою кількістю фігур мітозу. Про що свідчать
описані зміни?
A. @Антигенну стимуляцію з фолікулярною гіперплазією
B. Атрофію лімфоїдної тканини
C. Кандидоз
D. Лімфогранулематоз
E. Лімфолейкоз
4. При ультраструктурному вивченні біоптату тканини встановлено, що в цитоплазмі макрофага,
який входить до складу запального інфільтрату, виявляється значне збільшення кількості лізосом. Це свідчить
про:
A. Проліферативну активність клітин
B. Уповільнення фагоцитозу
C. Активацію апоптозу
D. @Активацію фагоцитозу
E. Уповільнення апоптозу
5. Під час розтину тіла померлої жінки, 54 років, з діагнозом "Вузликовий периартеріїт", мікроскопічно були
виявлені такі змін в артеріолах: мукоїдне і фібриноїдне набухання, фібриноїдний некроз стінок, дифузна
лімфогістіоцитарна інфільтрація, зміни ендотеліоцитів. Назвіть вид запалення в артеріолах:
A. @Гостре імунне запалення
B. Дифузне ексудативне запалення
C. Фібринозне запалення
D. Дифтеритичне запалення
E. Гостре неімунне запалення
6. Експериментальній тварині після попередньої сенсибілізації підшкірно введено дозу антигену. У місці ін'єкції
розвинулося фібинозне запалення з альтерацією стінок судин, основної речовини і волокнистих структур
сполучної тканини у вигляді мукоїдного і фібриноїдного набухань, фібриноїдного некрозу. Назвіть відповідну
реакцію.
A. @Гіперчутливість негайного типу
B. Гіперчутливість сповільненого типу
C. Реакція трансплантаційного імунітету
D. Нормергічна реакція
E. Гранульоматоз
7. Тваринi, сенсибiлiзованiй туберкулiном, внутрiшньоочеревинно введено туберкулiн. Через 24 години при
лапаратомiї виявлено венозну гiперемiю та набряк очеревини. У мазках-вiдбитках з очеревини велика кiлькiсть
лiмфоцитiв та моноцитiв. Яке запалення має мiсце у тварини?
A. @Алергiчне
B. Серозне
C. Гнiйне
D. Фiбринозне
E. Асептичне
8. Хворий, 38 років, помер під час нападу бронхіальної астми, що не знімався. При гістологічному дослідженні: в
просвіті бронхів виявлено скупчення слизу, в стінці бронхів - багато тучних клітин (лаброцитів), чисельні - в
стані дегрануляції, а також багато еозинофілів. Який патогенез змін у бронхах?
A. Цитотоксичний, цитолітична дія антигенів
B. @Атопія, анафілаксія
C. Імунокомплексний механізм
D. Клітинно обумовлений цитоліз
E. Гранульоматоз
9. У померлого від ядухи чоловіка, який багато років страждав на бронхіальну астму, при гістологічному
дослідженні легень виявлено: у просвітах бронхіол і дрібних бронхів багато слизу з домішкою еозинофілів,
склероз міжальвеолярних перегородок, розширення просвітів альвеол. Назвіть механізм розвитку реакції
гіперчутливості.
A. Цитотоксична реакція
B. Імунокомплексна реакція
C. @Реагінова реакція
D. Цитоліз, обумовлений лімфоцитами
E. Гранульоматоз
10. У хворої на бронхіальну астму вірусне інфікування спровокувало астматичний статус зі смертельним
наслідком. При гістологічному дослідженні легень виявлено: спазм і набряк бронхіол, в їх стінках - виражена
інфільтрація лімфоцитами, еозинофілами й іншими лейкоцитами, а також дегрануляція лаброцитів. Який
механізм гіперчутливості лежить в основі даного процесу?
A. @Реагінова реакція гіперчутливості
B. Запальний
C. Аутоіммунний
D. Імунокомплексний
E. Імунообумовлений клітинний цитоліз
11. У пацієнта з бронхіальною астмою за допомогою шкірних алергічних проб встановлено сенсибілізацію
алергеном тополевого пуху. Який фактор імунної системи відіграє вирішальну роль у розвитку цього
імунопатологічного стану?
A. @IgE
B. IgD
C. IgM
D. Т-лімфоцити
E. IgG
12. Хворому, 39 років, у зв'язку з лікуванням гнійного пульпіту було проведено інфільтраційну анестезію
розчином ультракаїну з адреналіном, після чого раптово виникли почервоніння, набряк шкіри, формування
пухирів і свербіння. Який тип гіперчутливості має місце?
A. Імунокомплексне пошкодження
B. Цитотоксичний
C. @Анафілактичний
D. Гіперчутливість сповільненого типу
E. Гранульоматозний
13. На гістологічному препараті в сполучній тканині знайдено великі клітини, заповнені базофільною
метахроматичною зернистістю; гістохімічно встановлено, що гранули містять гепарин та гістамін. Які клітини
найбільш вірогідно знайдено в препараті?
A. @Тучні
B. Фібробласти
C. Макрофаги
D. Плазмоцити
E. Адипоцити
14. Після проведення туберкулінової проби (проба Манту) у дитини через 48 годин на місці введення туберкуліну
утворилася папула розміром до 10 мм у діаметрі. Який механізм гіперчутливості лежить в основі описаних змін?
A. @Клітинна цитотоксичність
B. Анафілаксія
C. Антитілозалежна цитотоксичність
D. Імунокомплексна цитотоксичність
E. Гранульоматоз
15. 4-річній дитині зроблено реакцію Манту. Через 60 годин після введення в шкіру туберкуліну з'явилося
осередкове затвердіння і почервоніння шкіри, діаметром 15 мм, що було розцінене як позитивний тест. Який вид
реакції гіперчутливості лежить в основі цього тесту?
A. @Гіперчутливість сповільненого типу
B. Імунокомплекс-опосередкована гіперчутливість
C. Комплемент-опосредованна цитотоксична
D. Гіперчутливість негайного типу
E. Некроз
16. На 8-й день після введення протиправцевої сироватки з приводу брудної рани стопи у пацієнта підвищилася
температура тіла до 380С, з’явилися біль у суглобах, висипка, свербіж. У крові - лейкопенія і тромбоцитопенія.
Який тип алергічної реакції розвинувся?
A. @Імунокомплексна
B. Анафілактична
C. Цитотоксична
D. Гіперчутливість уповільненого типу
E. Стимулююча
17. При гістологічному дослідженні діагностичної біопсії лівого вушка серця виявлено ревматичну гранульому,
яка представлена осередком фібриноїдного некрозу, навколо якого розміщені базофільні макрофаги, лімфоцити,
фібробласти і поодинокі плазмоцити. Вкажіть вірогідний тип запальної реакції на імунній основі.
A. Нормергічний тип
B. Гіперчутливість негайного типу
C. Природжений імунодефіцит
D. @Гіперчутливість сповільненого типу
E. Набутий імунодефіцит
18. Хворому на аутоімунний гастрит проведено біопсію слизової шлунка. При дослідженні боптату в слизовій
оболонці виявлено лімфоцитарну і макрофагальну інфільтрацію. Це свідчить про розвиток у слизовій оболонці
гіперчутливості:
A. І типу (реагінового типу)
B. ІІ типу (антитільної цитотоксичності)
C. ІІІ типу (імунокомплексної цитотоксичності)
D. @ІV типу (клітинної цитотоксичності)
E. V типу (гранульоматозу)
19. 14-річна дівчинка раптово померла після перенесеної гострої респіраторної вірусної інфекції. Під час розтину
виявлені: різко збільшені в розмірах тимус, генералізована гіперплазія лімфатичних вузлів, гіперплазовані
яєчники. Діагностуйте захворювання.
A. @Тиміко-лімфатичний стан
B. Акцидентальна інволюція тимуса
C. Гіпоплазія тимуса
D. Атрофія тимуса
E. Тимома
20. При розтині померлої від внутрішньоутробної пневмонії новонародженої дитини, виявлено зменшення
розмірів і маси вилочкової залози. При мікроскопічному дослідженні в її часточках стерта межа між корковою та
мозковою речовиною внаслідок лімфоїдного спустошення обох зон; у мозковій речовині - численні, різних
розмірів тільця вилочкової залози (тільця Гассаля), з явищами кератозу і кальцинозу; міжчасточкова сполучна
тканина розвинена надмірно. Який з перерахованих патологічних процесів у тимусі найбільш вірогідний?
A. @Акцидентальна інволюція тимуса
B. Атрофія
C. Гіпотрофія
D. Аплазія
E. Тимомегалія
21. При дослідженні тимуса дитини, 5 років, яка померла від гострої деструктивної стафілококової пневмонії,
виявлено зменшення маси залози до 3,0 гр. При гістологічному дослідженні в тимусі визначається зменшення
часточок залози, значне зниження кількості лімфоцитів з колапсом строми, інверсія шарів, кістоподібне
збільшення тілець Гассаля. Який з перерахованих діагнозів вірогідний?
A. Агненезія тимуса
B. Тимомегалія
C. Гіпоплазія тимуса
D. Дисплазія тимуса
E. @Акцидентальна інволюція
22. У дитини, 2 років, - множинні порушення розвитку кісток лицьового відділу черепа. Причиною смерті став
сепсис, який розвинувся на тлі бронхопневмонії. У крові вміст імуноглобулінів в межах фізіологічної норми. На
розтині встановлено відсутність тимуса. Назвіть захворювання, що привело до розвитку сепсису.
A. Синдром комбінованого імунодефіциту
B. Вторинний імунодефіцитний синдром
C. Гострий лімфолейкоз
D. @Синдром недостатності клітинного імунітету
E. Синдром хронічної інтоксикації
23. При розтині дитини, 6 міс., померлої від сепсису, виявлено відсутність вилочкової залози, зменшення розмірів
і маси селезінки. При мікроскопічному дослідженні селезінки: відсутність периартеріальних Т-залежних зон
фолікулів зі спустошенням червоної пульпи; у лімфатичних вузлах – відсутність паракортикальної зони, яка в
основному представлена Т-лімфоцитами. В-зони в периферичних імунних органах розвинені нормально. Який
патологічний процес має місце?
A. Синдром Гланцмана-Рінкера (недостатність клітинного і гуморального імунітету)
B. Синдром Брутона (недостатність гуморального імунітету)
C. Акцидентальная інволюція тимуса
D. @Синдром Ді-Джорджі (недостатність клітинного імунітету)
E. ВІЛ-інфекція
24. При обстеженні вагітної жінки з резус-негативною групою крові визначено високий рівень
антиеритроцитарних антитіл, для зниження якого їй був підшитий шкірний клапоть її резус-позитивного
чоловіка. Через 2 тижні клапоть відторгнувся, мікроскопічно в ньому знайдені: порушення кровообігу, набряк,
клітинна інфільтрація в основному лімфоцитами, нейтрофілами і макрофагами. Який з перерахованих
патологічних процесів найбільш вірогідний?
A. Реакція гіперчутливості негайного типу
B. @Трансплантаційний імунітет
C. Реакція гіперчутливості сповільненого типу
D. Гранульоматозне запалення
E. Інтерстиціальне запалення
25. При пункційній біопсії в трансплантованій нирці виявлено дифузну інфільтрацію строми лімфоцитами,
плазмоцитами, лімфобластами, плазмобластами, а також некротичний артеріїт. Який патологічний процес
розвинувся у трансплантаті?
A. @Імунне відторгнення
B. Гломерулонефрит
C. Ішемічне пошкодження нирки
D. Тубулонекроз
E. Пієлонефрит

ЗАГАЛЬНЕ ВЧЕННЯ ПРО ЗАПАЛЕННЯ.


ЕКСУДАТИВНЕ ЗАПАЛЕННЯ
ВАРІАНТ №

1. Дитина, 5 років, скаржилася на біль у горлі при ковтанні, погане самопочуття. При огляді в зіві та
мигдаликах на тлі помірної набряклості і гіперемії виявлені сірувато-білі плівчасті нашарування, завтовшки до 1
мм, щільно пов'язані з прилеглими тканинами. Про який патологічний процес свідчать ці зміни?
A. Дистрофія
B. Некроз
C. @Запалення
D. Метаплазія
E. Організація
2. Хлопчика, 14 рокiв, турбувала сверблячка, пiдвищення температури в дiлянцi облямiвки нижньої губи,
пiзнiше з'явилося багато дрiбних пухирцiв, якi мiстять ледь каламутну, солом'яно-жовтого кольору, опалесцуючу
рiдину. При мiкроскопiчному дослiдженнi виявленi лiмфоцити, поодинокi нейтрофiли, злущенi епiтелiальнi
клiтини. Який патологiчний процес лежить в основi даного явища:
A. Альтерацiя
B. Набряк
C. Пролiферативне запалення
D. Бiлкова вакуольна дистрофія
E. @Ексудативне запалення
3. У жінки, 34-х років, після необережного поводження з праскою, у правому вказівному пальці з'явився
різкий біль, почервоніння, припухлість. Через декілька хвилин виник пухир, заповнений прозорою рідиною
солом'яно-жовтого кольору. Проявом якого патологічного процесу є описані зміни?
A. Травматичного набряку
B. Альтеративного запалення
C. @Ексудативного запалення
D. Проліферативного запалення
E. Вакуольної дистрофії
4. У хворого після термічного опіку на шкірі сформувалися болючі пухирі, заповнені каламутною
рідиною. Яка форма запалення виникла?
A. Продуктивне
B. Крупозне
C. Гранульоматозне
D. @Серозне
E. Дифтеритичне
5. У хворого, 43 років, опіки правої кисті. На долоні і зовнішній поверхні кисті розвинулося
відшаровування епідермісу і утворення пухирів, заповнених злегка каламутною рідиною. Яке запалення виникло
у хворого?
A. @Серозне
B. Гнійне
C. Катаральне
D. Гнильне
E. Фібринозне
6. У хворого, 6 рокiв, через кiлька хвилин пiсля укусу оси в дiлянцi правої кистi з’явився рiзкий бiль,
припухлiсть, почервонiння, локально пiдвищилася температура. Який морфологiчний процес лежить в основi
вказаних змiн:
A. @Серозне запалення
B. Пролiферативне запалення
C. Геморагiчне запалення
D. Транссудацiя
E. Дистрофiя
7. Чоловік скаржиться на свербіння і почервоніння шкіри в ділянці щоки, яке з'явилося після гоління.
Об'єктивно: на ділянці гіперемованої шкіри щоки виявлені пухирці, заповнені прозорою рідиною. Який характер
рідини в пухирцях?
A. Транссудат
B. Гнійний ексудат
C. Слизовий ексудат
D. Геморагічний ексудат
E. @Серозний ексудат
8. Жiнка, 38 рокiв, хворiє на декомпенсовану ваду мiтрального клапана ревматичної етiологiї. При
обстеженнi перед операцiєю кардiохiрург виявив у порожнинi перикарда бiля 60 мл солом'яно-жовтої,
опалесцуючої рiдини. Листки перикарда повнокровнi, тьмянi. При гiстологiчному дослiдженнi iнтраоперацiйної
бiопсiї виявлено помiрну лiмфоїдну iнфiльтрацiю з домiшкою поодиноких полiморфно-ядерних лейкоцитiв,
еритроцитiв. Вираженi повнокров'я i набряк епiкарда. До якого виду ексудативного запалення можна вiднести
описанi патологiчнi змiни:
A. Фiбринозне
B. @Серозне
C. Гнiйне
D. Катаральне
E. Геморагiчне
9. Захворювання дитини почалося з нежитю, нездужання. З'явилися висока температура, головний біль,
нудота, блювота. Дитину госпіталізовано в інфекційне відділення, проте, через 2 доби, настала смерть. На розтині
виявлено: різко потовщена, тьмяна, набрякла, повнокровна, жовтувато-зеленуватого кольору м'яка мозкова
оболонка, мозок різко набряклий, уклинення стовбура у великий потиличний отвір. Який вид запалення
розвинувся?
A. Катаральне
B. Геморагічне
C. Фібринозне
D. @Гнійне
E. Серозне
10. На розтині померлого від менінгіту, м'яка мозкова оболонка потовщена, різко повнокровна, тьмяна,
набрякла, просочена густим зеленувато-жовтим ексудатом. Про яку форму запалення йдеться?
A. @Гнійне
B. Серозне
C. Фібринозне крупозне
D. Фібринозне дифтеритичне
E. Геморагічне
11. У дитини, 3 років, розвинувся менінгококовий менінгіт. Макроскопічно: м'яка мозкова оболонка
жовто-зеленого кольору, набрякла. Назвіть форму запалення
A. @Гнійне
B. Серозне
C. Геморагічне
D. Катаральне
E. Гнильне
12. При розтині померлого патологоанатом виявив, що м'яка мозкова оболонка повнокровна, потовщена,
непрозора, жовтувато-зеленуватого кольору. Для якої форми запалення характерні такі зміни?
A. @Гнійного
B. Серозного
C. Геморагічного
D. Фібринозного
E. Катарального
13. У хворого з перитонітом виявлено в черевній порожнині 200 мл в'язкої жовто-зеленої рідини.
Визначте форму ексудативного запалення.
A. Серозне
B. Фібринозне
C. Геморагічне
D. @Гнійне
E. Гнилісне
14. Макроскопічно губчастий і кортикальний шари гомілкової кістки на окремих ділянках у стані розпаду;
порожнини, які сформувалися, заповнені вершкоподібними масами зеленувато-жовтого кольору. Вкажіть
найбільш вірогідну форму запалення:
A. Катаральне
B. @Гнійне
C. Серозне
D. Проліферативне
E. Змішане
15. На розтині у печінці померлого, 71 років, виявлено осередок розпаду тканини діаметром 3,7 см,
заповнений рідиною жовтувато-зеленого кольору. Найвірогідніший діагноз:
A. @Абсцес
B. Флегмона
C. Карбункул
D. Емпієма
E. Гранульома
16. На розтині померлого від легенево-серцевої недостатності чоловіка, 47 років, у лівій легені виявлено
порожнину розмірами 4х4см, заповнену гноєм, стінка - фестончаста, нерівна, представлена легеневою тканиною.
Найвірогідніше це :
A. @Гострий абсцес
B. Хронічний абсцес
C. Кавернозний туберкульоз
D. Ехінококоз
E. Фіброзуючий альвеоліт
17. Чоловiк, 40 рокiв, перебував у пульмонологiчному вiддiленнi з приводу рецидивуючої правобічної
пневмонiї. Помер вiд легенево-серцевої недостатностi. На розтинi в правiй легенi виявляється дiлянка круглої
форми 3х4 см. Вона являє собою порожнину з нерiвними шорсткими краями, заповнену каламутною
вершкоподiбною жовто-зеленою рiдиною. Мiкроскопiчно: стiнка порожнини утворена тканиною легенi з
дифузною iнфiльтрацiєю лейкоцитами. Визначте патологiчний процес у легенi:
A. @Гострий абсцес
B. Хронiчний абсцес
C. Емпiєма
D. Гангрена
E. Iнфаркт
18. Чоловiк, 65 рокiв, знаходився у ЛОР-вiддiленнi з приводу гнiйного, рецидивуючого отиту. На 9 добу
перебування в стацiонарi помер вiд набряку мозку з вклиненням мигдаликiв мозочка у великий потиличний отвiр.
На аутопсiї у скроневiй ділянцi лiвої пiвкулi виявлено осередк круглої форми, безструктурний, який являє собою
порожнину з нерiвними, шорсткими краями, заповнену каламутною вершкоподiбною жовтувато-зеленуватою
рiдиною. Стiнки порожнини вистеленi тканиною мозку. Про який патологiчний процес свiдчать описанi
морфологiчнi змiни у тканинi мозку:
A. Хронiчний абсцес
B. Флегмону
C. @Гострий абсцес
D. Колiквацiйний некроз
E. Емпiєму
19. Хворому хірург видалив порожнинне утворення печінки діаметром 2 см. Встановлено, що стінка
порожнини утворена щільною волокнистою сполучною тканиною, вміст являє собою каламутну, густу,
жовтувато-зеленуватого кольору рідину з неприємним запахом, яка мікроскопічно складається переважно з
поліморфноядерних лейкоцитів. Якому процесу відповідають такі зміни?
A. @Хронічному абсцесу
B. Гострому абсцесу
C. Емпіємі
D. Флегмоні
E. Холодному (натічному) абсцесу
20. У хворого після операції з приводу масивних опіків, в умовах різкого зниження реактивності
організму, виник і став прогресувати сепсис, що призвело до смертельного наслідку. В ділянці передньої стінки
черевної порожнини на розрізі мікроскопічно спостерігається набряк, дифузна інфільтрація сегментоядерними
лейкоцитами міжм'язових проміжків, лізис м'язових волокон. Визначити характер змін.
A. Дифтеритичне запалення
B. Абсцес
C. @Флегмона
D. Некроз
E. Катаральне запалення
21. У хворого на розтинi в правiй плевральнiй порожнинi було виявлено 380 мл жовтої медоподiбної
рiдини з неприємним запахом. При мiкроскопiї рiдини - багато нейтрофiльних гранулоцитiв. Ваш дiагноз?
A. Флегмона
B. Гнiйний катар плеври
C. @Емпiєма плеври
D. Абсцес легені
E. Гангрена легенi
22. При розтині хворого, який страждав на крупозну пневмонію і помер від пневмококового сепсису, в
правій плевральній порожнині містилося 900 мл каламутної зеленувато-жовтого кольору рідини. Листки плеври
тьмяні, повнокровні. Назвіть форму запалення в плевральній порожнині.
A. Фібринозне
B. Флегмона
C. Хронічний абсцес
D. Гострий абсцес
E. @Емпієма
23. У хворого висока температура, задуха, біль у правій половині грудної клітки. Під час плевральної
пункції видалено 700 мл вершкоподібної рідини жовтувато-зеленого кольору. Найвірогідніший діагноз:
A. @Емпієма плеври
B. Карциноматоз плеври
C. Серозний плеврит
D. Фібринозний плеврит
E. Геморагічний плеврит
24. Пiд час проведення операцiї з приводу гострого флегмонозного апендициту у хлопчика, 14 рокiв,
хiрург звернув увагу на те, що листки очеревини на всьому протязi дифузно потовщенi, повнокровнi, тьмянi,
шорсткi. В черевнiй порожнинi виявлено бiля 800 мл каламутної, густої, жовтувато-зеленуватого кольору рiдини
з неприємним запахом, яка мiкроскопiчно складається переважно з ПЯЛ, поодиноких еритроцитiв. Якому
патологiчному процесу вiдповiдають такi морфологiчнi змiни:
A. Гнiйно-геморагiчному запаленню
B. @Емпiємi
C. Абсцесу
D. Катарально-гнiйному запаленню
E. Флегмонi
25. Хворий, 34 років, звернувся до лікаря зі скаргами на локальний біль в зоні потилиці, підвищення
температури тіла в цій ділянці. Макроскопічно визначається інфільтрат конусоподібної форми багряно-
синюшного кольору з жовто-зеленою верхівкою, яка піднімається над поверхнею шкіри. Ваш діагноз.
A. @Фурункул
B. Флегмона
C. Абсцес
D. Карбункул
E. Емпієма
26. При мiкроскопiчному дослiдженнi запального iнфiльтрату шкiри виявлене накопичення гнiйно-
лейкоцитарного ексудату в декiлькох волосяних фолiкулах. Як називається таке запалення:
A. Фурункул
B. Флегмона
C. Натiчник
D. @Карбункул
E. Абсцес
27. У жінки, 38 років, хірург діагностував карбункул носо-губної складки. Яке небезпечне ускладнення
даного патологічного процесу може розвинутися?
A. Гнійний лімфаденіт
B. Гнійний фарингіт
C. @Гнійний менінгіт
D. Гнійний хейліт
E. Гнійний стоматит
28. Слизова оболонка трахеї тьмяна, повнокровна, з сіро-білими плівками. Вкажіть найбільш вірогідну
форму запалення:
A. Гнійне
B. @Фібринозне
C. Серозне
D. Проліферативне
E. Змішане
29. Дівчинка, 4 років, на 3 добу від початку захворювання на дифтерію померла від справжнього крупу.
На аутопсії слизова оболонка гортані, трахеї і бронхів потовщена, набрякла, тьмяна, вкрита сіруватою плівкою,
яка легко відділяється. Визначити вид ексудативного запалення гортані:
A. Серозне
B. @Фібринозне
C. Гнійне
D. Змішане
E. Катаральне
30. Дитину доставлено у відділення в стані асфіксії. При огляді в гортані виявлені білі плівки, які
обтурують просвіт і легко відділяються. Лікар запідозрив дифтерію. Про яку форму запалення гортані йдеться?
A. Катаральне
B. Геморагічне
C. Серозне
D. Гнійне
E. @Фібринозне
31. При розтині тіла померлого від крупозної пневмонії: в плевральній порожнині - непрозора рідина, на
вісцеральній плеврі - сіруватого кольору плівка. Визначити вид запалення на вісцеральній плеврі.
A. Катаральне
B. Гнійне
C. Гранулематозне
D. @Фібринозне
E. Геморагічне
32. Чоловік, 55 років, тривалий час хворів на хронічний пієлонефрит. Помер при явищах хронічної
ниркової недостатності. На поверхні епікарда і перикарда виявляються сірувато-білуваті ворсинчасті
нашарування. Який патологічний процес мав місце в перикарді?
А. @Фібринозне запалення
В. Організація
С. Проліферативне запалення
D. Геморагічне запалення
Е. Артеріальне повнокров’я
33. Чоловік, 55 років, хворів на хронічний гломерулонефрит. Смерть настала при явищах хронічної
ниркової недостатності. На аутопсії виявлено уринозний запах. Макроскопічно: на поверхні епікарда і перикарда
- сірувато-білуваті ворсинчасті нашарування, після зняття яких добре видно різко розширені і переповнені кров'ю
судини. Який процес мав місце в перикарді?
A. Організація
B. Геморагічне запалення
C. Проліферативне запалення
D. @Фібринозне запалення
E. Артеріальне повнокров'я
34. Хворий на хронічний пієлонефрит помер від хронічної ниркової недостатності. За життя
аускультативно був відмічений феномен "шуму тертя перикарда". На розтині епікард тьмяний, вкритий
"волосяним покривом". Який перикардит за характером запалення має місце?
A. @Фібринозний
B. Катаральний
C. Гнійний
D. Гнильний
E. Серозний
35. Хворий на хронічний пієлонефрит помер від хронічної ниркової недостатності. Прижитєво
аускультативно відмічено феномен “шум тертя перикарда”. На розтині виявлено, що епікард тьм’яний, шорсткий,
ніби покритий волосяним покривом. Який перикардит за характером запалення має місце?
A. @Крупозний
B. Дифтеритичний
C. Гнійний
D. Гнильний
E. Серозний
36. На розтині померлого від гострого трансмурального інфаркту міокарда на поверхні перикарда
виявлено: ниткоподібні нашарування біло-коричневого кольору, які сполучають парієтальний і вісцеральний
листки перикарда між собою. Вказати різновид запалення в перикарді?
A. @Фібринозне
B. Серозне
C. Гнійне
D. Гранульоматозне
E. Геморагічне
37. У померлого від гострого трансмурального інфаркту міокарда на розтині на поверхні перикарда
виявлено: ниткоподібні відкладення білувато-коричневого кольору, які з’єднували парієтальний та вісцеральний
листки перикарда між собою. Вказати різновид запалення в перикарді?
A. @Крупозне
B. Дифтеритичне
C. Серозне
D. Гнійне
E. Гранульоматозне
38. На розтині померлого від ревматизму, 34 років, - поверхня епікарда ворсиста, покрита плівками сірого
кольору, що легко відділяються. Після їх відділення визначається набрякла повнокровна поверхня епікарда.
Найвірогідніший діагноз:
A. @Фібринозний перикардит
B. Гнійний перикардит
C. Геморагічний перикардит
D. Проліферативний перикардит
E. Катаральний перикардит
39. Дівчинка, 6 років, захворіла на дифтерію. На третю добу померла від асфіксії внаслідок істиного
крупу. На розтині: слизова оболонка гортані, трахеї і бронхів потовщена, набрякла, тьмяна, вкрита сіруватими
плівками, які легко відділяються. Про який патологічний процес свідчать морфологічні зміни в гортані?
A. Серозне запалення
B. Гнійне запалення
C. @Крупозне запалення
D. Дифтеритичне запалення
E. Катаральне запалення
40. Дитину, 6 років, доставлено у приймальне відділення дитячої лікарні в стані асфіксії. У гортані
виявлені білі плівки, які легко відділяються та обтурують просвіт. Проведено трахеостомію. Який вид запалення
розвинувся?
A. Геморагічне
B. Дифтеритичне
C. Катаральне
D. Гнійне
E. @Крупозне
41. Дитину доставлено в санпропускник у станi асфiксiї. При оглядi в гортанi виявленi бiлуватi плiвки, що
обтурують просвiт та легко видаляються. Лiкар запiдозрив дифтерiю. Про яке запалення гортанi йдеться?
A. @Крупозне
B. Катаральне
C. Дифтеритичне
D. Серозне
E. Гнiйне
42. При розтині трупа дівчинки, яка померла від асфіксії, виявлено, що слизова оболонка трахеї і бронхів
вкрита блідо-сірими плівками, з’єднаними з підлеглими тканинами, які легко знімаються пінцетом. Просвіт
сегментарних бронхів заповнений масами сіро-білого кольору. Який трахеобронхіт за характером ексудату
відмічено при розтині?
A. Катаральний
B. Дифтерійний
C. Гнійний
D. @Крупозний
E. Гнильний
43. Під час розтину тіла дитини, яка померла при ознаках асфіксії, були виявлені в трахеї і головних
бронхах сіруватого кольору плівки, які вільно лежали у просвіті дихальних шляхів, нагадуючи їх зліпки. Вкажіть
вид запалення:
A. @Крупозне
B. Дифтеритичне
C. Катаральне
D. Серозне
E. Гнійне
44. При розтині померлого, 24 років, який хворів на підгострий гломерулонефрит і помер при явищах
прогресуючої уремії, на поверхні перикарда, вісцеральному і пристінковому листках плеври, очеревини
спостерігалися нашарування у вигляді білуватих ниток, які легко відділялися від підлеглої тканини. Вказати
різновид запалення.
A. @Крупозне фібринозне
B. Дифтеритичне фібринозне
C. Гнійне
D. Дифузне проліферативне
E. Серозне
45. У дитини з помірно вираженою загальною інтоксикацією виявлено дифтерію з ураженням великих
бронхів. Який вид запалення бронхів при дифтерії?
A. Дифтеритичне
B. Альтеративне
C. Геморагічне
D. @Крупозне
E. Продуктивне
46. На розтинi у правiй плевральнiй порожнинi виявлено 0,5 л жовтуватої прозорої рiдини з наявнiстю
бiлих пухких грудочок. Парiєтальна i вiсцеральна плевра покритi бiлим пухким налітом. Про який вид
ексудативного запалення йде мова?
A. Катаральне
B. Гнильне
C. @Крупозне
D. Серозне
E. Гнiйне
47. Дiвчинка, 4 рокiв, померла вiд дизентерiї. На розтинi слизова прямої кишки покрита бiло-сiрою
плiвкою, яка легко знiмається пiнцетом. Зустрiчаються поодинокi виразки. Який вид ексудативного запалення
має мiсце?
A. Альтеративне
B. Дифтеритичне
C. Гнiйне
D. Катаральне
E. @Крупозне
48. У чоловіка, 32 рокiв, було виявлено дифтеритичне запалення слизової оболонки зiва. Виберiть
морфологiчну ознаку, що зумовлює клiнiчний перебiг процесу.
A. @Щiльний зв'язок плiвки з пiдлягаючими тканинами
B. Щiльна консистенцiя ексудату
C. Глибокий некроз слизової та пiдлягаючих тканин
D. Наявнiсть нейтрофiльних лейкоцитiв у ексудатi
E. Наявнiсть полiмеризованого фiбрину у ексудатi
49. Дитина, поступила в інфекційне відділення зі скаргами на різкий біль у горлі, утруднення при
ковтанні, підвищення температури тіла до 39.0°С, набряк шиї. При огляді виявлено: мигдалики збільшені, їх
слизова оболонка повнокровна, вкрита великою кількістю плівок білувато-жовтого кольору, які щільно зв'язані зі
слизовою оболонкою. При спробі зняти плівку залишається глибокий дефект, який кровоточить. Який вид
запалення має місце?
A. @Дифтеритичне
B. Гнійне
C. Серозне
D. Крупозне
E. Геморагічне
50. У хворого, 28 років, мигдалики значно збільшені, повнокровні, болючі, з щільними брудно-сірими
плівками, які поширюються на тверде піднебіння і пов'язані з прилеглими тканинами; при спробі відділення
виникає кровотеча. Яке запалення зумовлює такі морфологічні зміни?
A. Крупозне
B. @Дифтеритичне
C. Катаральне
D. Гнійне
E. Геморагічне
51. У хворого, 28 років, мигдалики значно збільшені, повнокровні, болючі, на їх поверхні – щільні
брудно-сірі плівки, які поширюються на тверде піднебіння, щільно зв’язані з підлеглими тканинами, при спробі
відокремити їх – розвивається кровотеча. Який патологічний процес обумовлює дані морфологічні зміни?
A. @Дифтеритичне ексудативне запалення
B. Крупозне ексудативне запалення
C. Катаральне ексудативне запалення
D. Гнійне ексудативне запалення
E. Геморагічне ексудативне запалення
52. У дитини, 6 років, мигдалики збільшені, вкриті білястими плівками, що знімаються насилу. Виберіть
відповідь, яка характеризує зміни в зіві і мигдаликах.
A. @Дифтеритичне запалення
B. Казеозний некроз
C. Фібриноїдний некроз
D. Крупозне запалення
E. Гнійне запалення
53. У дитини підвищилася температура тіла, з'явилися болі при ковтанні. При огляді: піднебінні
мигдалики збільшені, темно-червоні, вкриті сіро-жовтими плівками, щільно спаяними з поверхнею мигдаликів.
Яке запалення розвинулося в мигдаликах?
A. @Дифтеритичне
B. Крупозне
C. Геморагічне
D. Гнійне
E. Катаральне
54. На слизовій оболонці мигдаликів та м’якого піднебіння виявляються білувато-сірого кольору плівки,
які щільно з’єднані з підлеглою тканиною, при спробі зняти плівку на її місці виникає глибокий дефект тканини.
Визначити патологічний процес, який виник на слизовій оболонці мигдаликів та м’якого піднебіння?
A. @Дифтеритичне запалення
B. Серозне запаленя
C. Крупозне запалення
D. Гнійне запалення
E. Змішане запалення
55. При розтині померлого від хронічної ниркової недостатності на слизовій оболонці товстої кишки
виявлені сіро-жовті, щільно спаяні з підлеглою тканиною плівки, які відділяються з утворенням виразок. Вид
запалення:
A. @Дифтеритичне
B. Серозне
C. Катаральне
D. Крупозне
E. Гнійне
56. При розтинi тiла жiнки, померлої вiд хронiчної ниркової недостатностi, в слизовiй оболонцi товстої
кишки виявленi сiро-жовтi плiвки, що щiльно з’єднанi з пiдлеглим шаром, при вiдокремленнi яких утворюються
виразки. Мiкроскопiчно: глибока ділянка некрозу пронизана нитками фібрину. Визначте вид запалення:
A. @Дифтеритичне
B. Крупозне
C. Катаральне
D. Гнильне
E. Гнiйне
57. У юнака, 18 років, який захворів гостро і помер від інфекційно-токсичного шоку, на аутопсії виявлено
збільшені мигдалики, вкриті сіро-білими плівками, що поширюються на піднебінні дужки, набряк м’яких тканин
шиї. При гістологічному дослідженні: некроз епітелію мигдаликів та дужок, підлеглі тканини просякнуті
фібринозним ексудатом, який утворює масивні нашарування на поверхні. Ваш діагноз:
A. @Дифтерія
B. Скарлатина
C. Аденовірусна інфекція
D. Інфекційний мононуклеоз
E. Стафілококова інфекція
58. У чоловіка, 50 років, діагностовано дизентерію. Смерть настала на 8-й день захворювання. Під час
розтину виявлено потовщену стінку сигмоподібної та початкових відділів прямої кишки, фібринозну плівку на
поверхні слизової оболонки. Гістологічно: глибокий некроз слизової оболонки з просяканням некротичних мас
фібрином. Який вид коліту мав місце у хворого?
A. Виразковий
B. Катаральний
C. @Дифтеритичний
D. Гангренозний
E. Фолікулярний
59. Чоловiк, 48 рокiв, захворів на сибірку і помер. На розтинi виявлено, що м'якi мозковi оболонки на
склепiннi та основi мозку набряклi, просякнутi кров'ю, темно-червоного кольору, нагадують "шапочку
кардинала". Гiстологiчно - альтерацiя стiнок дрiбних судин оболонок i тканини мозку, дрiбнi лiмфоцитарнi
інфільтрати. Якому патологічному процесу вiдповiдають описані зміни:
A. Субарахноїдальному крововиливу
B. Набряку мозку
C. Крововиливу у мозок
D. Венозному повнокров'ю мозку
E. @Геморагiчному запаленню
60. У хворого, 50 років, при фіброгастроскопії виявлено набухання та гіперемію слизової оболонки
шлунка, велику кількість каламутного, в'язкого ексудату на поверхні. Який морфологічний склад ексудату?
A. Фібрин
B. Глобуліни
C. @Слиз
D. Багато нейтрофілів
E. Лімфоцити
61. Дитина, 7 років, захворіла гостро, з підвищенням температури до 38,0°С, появою нежитю, кашлю,
сльозотечі і крупноплямистого висипу на шкірі. Слизова оболонка зіва набрякла, вкрита слизом. В ділянці
великих кореневих зубів, на слизовій оболонці щік - білясті плями. Яке запалення лежить в основі змін зіва?
A. @Катаральне
B. Гнійне
C. Фібринозне
D. Геморагічне
E. Серозне
62. Юнак, 17 років, захворів гостро, температура тіла підвищилася до 38,5°С, з’явилися кашель, нежить,
сльозотеча, виділення з носу. Яке запалення розвинулося у юнака?
A. @Катаральне
B. Серозне
C. Фібринозне
D. Гнійне
E. Геморагічне
63. У дитини, 5 років, розвинулось гостре респіраторне захворювання, яке супроводжувалось кашлем,
виділенням значної кількості слизу із носа. Який тип запалення у хворої дитини?
A. Катаральне
B. Фібринозне
C. Геморагічне
D. Гнійне
E. Гнилісне
64. При ректороманоскопії у дитини, 10 років, слизова оболонка прямої та сигмоподібної кишки набрякла,
червонуватого кольору, вкрита товстим шаром слизу. Визначити вид вказаних змін.
A. Гнійне запалення
B. Змішане запалення
C. @Катаральне запалення
D. Геморагічне запалення
E. Венозне повнокров’я
65. При гістологічному дослідженні легень померлого від серцевої недостатності, виявлені осередки
запалення з заповненням альвеол рідиною, забарвленою у блідо-рожевий колір, місцями з наявністю тонких
рожевих ниток, створюючих дрібносплетену мережу з невеликою кількістю лімфоцитів. Який характер ексудату
в легенях?
A. Геморагічний
B. Серозний
C. Гнійний
D. Фібринозний
E. @Серозно-фібринозний
66. На розтині у померлого в черевній порожнині виявлено близько 1,8 л гнійної рідини. Очеревина
тьмяна, з сіруватим відтінком. На серозній оболонці кишок - сіруватого кольору нашарування, що легко
знімаються. Вірогідно це:
A. Туберкульозний перитоніт
B. Геморагічний перитоніт
C. Серозний перитоніт
D. @Фібринозно-гнійний перитоніт
Е. Катаральний перитоніт
67. Дитині, 7,5 років, яка надійшла в інфекційне відділення з температурою до 38°С, дрібноплямистим
яскраво-червоним висипом на шкірі, був поставлений діагноз: «Скарлатина». Слизова оболонка зіва яскрава,
гіперемована, набрякла, мигдалики - різко збільшені, з тьмяними осередками жовтувато-сірого кольору і
ділянками чорного кольору. Назвіть вид запалення:
A. Фібринозне
B. Геморагічне
C. Серозне
D. Катаральне
E. @Гнійно-некротичне
68. При розтині тіла дитини, померлої від серцевої недостатності, виявлено: розширення порожнин
шлуночків серця. Мікроскопічно в стромі міокарда: повнокров’я, набряк, розповсюджені інфільтрати з
гістіоцитів, лімфоцитів, нейтрофілів, еозинофілів. Який найбільш імовірний діагноз?
A. @Дифузнопроміжний ексудативний міокардит
B. Осередковий проміжний ексудативний міокардит
C. Вузликовий продуктивний міокардит
D. Проміжний продуктивний міокардит
E. Альтеративний міокардит
69. При мікроскопічному дослідженні міокарда дівчинки, яка померла від серцевої недостатності при
дифтерії виявлені: жирова дистрофія і множинні осередки некрозу кардіоміоцитів, незначні осередкові клітинні
інфільтрати в інтерстиції. Про який міокардит йдеться?
A. Дифузний ексудативний
B. @Альтеративний
C. Осередковий ексудативний
D. Інтерстиціальний
E. Гранульоматозний
70. Жiнка, 33 років, знаходилася в гастроентерологiчному вiддiленнi з дiагнозом: «Гострий гастрит».
Ендоскопiчно - по малiй кривизнi шлунка виявляється безлiч виразок дiаметром до 1 см. Гiстологiчно в
гастробiоптатi: поверхневий дефект слизової оболонки, який покритий темно-сiрими некротичними масами; у
глибинi цього дефекту - повнокров'я i набряк тканин, скупчення дрiбних клiтин з великим гiперхромним ядром.
Про який вид запалення свiдчать виявленi змiни в стiнцi шлунка:
A. Пролiферативне
B. @Альтеративне
C. Гранульоматозне
D. Ексудативне
E. Слизове

АНЕМІЇ.
ВАРІАНТ №
1. Хворий, 17 років, з таласемією (хвороба Кулі) помер від прогресуючої серцевої недостатності. На
розтині виявлені: деформація черепа (“башенний” череп), гепатоспленомегалія, бура індурація легень,
гемосидероз серця, печінки, селезінки. В аналізах крові - хронічна гемолітична гіпохромна анемія з наявністю
мішенеподібних еритроцитів (гіпохромія еритроцитів), анізоцитоз, овалоцитоз. Вказати причину даної анемії:
A. Зниження активності піруваткінази еритроцитів
B. Аномалія структури мембрани еритроцитів
C. Дефіцит глюкозо-6-фосфатдегідрогенази еритроцитів
D. @Аномалія структури гемоглобіну
E. Дефіцит заліза

2. До гематологічного відділення госпіталізовано дитину, 5 місяців, з діагнозом «Гостра вірусно-


респіраторна інфекція, жовтяниця». При обстеженні виявлені: спленомегалія, в аналізі крові - гіпербілірубінемія з
підвищенням непрямої фракції, мікросфероцитарна анемія, ретикулоцитоз, підвищений вміст заліза в плазмі. В
аналізі сечі: реакція на білірубін слабо позитивна. Заключний діагноз: хвороба Шоффара-Мінковського
(спадковий мікросфероцитоз). Визначити патогенетичну ланку гемолітичної анемії:
A. Порушення активності ферментів еритроцитів
B. Синтез аномального гемоглобіну
C. Зміна антигенної структури еритроцита
D. @Дефект структури мембрани еритроцитів
E. Пошкодження еритроцитів антитілами

3. У хворого, 68 років, діагностовано рак голівки підшлункової залози з проростанням у стінку 12-палої
кишки, що спричинило масивну кишкову кровотечу. Проведено видалення частини головки залози з сегментом
тонкої кишки з формуванням анастомозів між її відділами. Згодом у хворого поступово розвинулась і наростала
слабкість. В аналізі периферичної крові: гіпохромна анемія (кольоровий показник 0,6), кількість лейкоцитів,
тромбоцитів та ретикулоцитів у межах норми, анізо- та пойкілоцитоз, швидкість зсідання еритроцитів 10 мм/год.
У плазмі крові кількість білірубіну в нормі, гемокоагулограма не змінена, вміст заліза знижений. Визначити
патогенетичну ланку залізодефіцитної анемії:
A. @Резорбційна недостатність
B. Аліментарна недостатність заліза
C. Підвищений попит заліза
D. Пригнічення гемопоезу
E. Аномалія в структурі гемоглобіну

4. У хворої, з групою крові А (II) та негативним резус-фактором на 36 тижні вагітності народився хлопчик
з групою крові А (II) та позитивним резус-фактором. В аналізі крові: гемоглобін 60 г/л, еритроцитів 1·1012/л,
ретикулоцити складають 100%. На 3 добу після народження дитина померла. На розтині виявлені: жовтяниця,
набряк шкіри, підшкірної клітковини та оболонок головного мозку; кардіо-, гепато- та спленомегалія, асцит,
гідроторакс, гідроперикард; об’єм легень зменшений; множинні дрібні крововиливи у внутрішніх органах, на
серозних та слизових оболонках. Плацента - губчаста, набрякла, маса та об’єм збільшені. Встановити діагноз:
A. Геморагічна хвороба новонароджених
B. @Гемолітична хвороба новонароджених
C. Анемія Фанконі
D. Хвороба Маркіафави-Мікелі
E. Аутоімунна гемолітична анемія

5. У хворого, 20 років, з жовтяницею встановлено: підвищення у плазмі крові вмісту загального білірубіну
за рахунок непрямого (вільного), в калі та сечі - високий вміст стеркобіліну, рівень прямого (зв’язаного)
білірубіну в плазмі крові в межах норми. Про який вид жовтяниці можна думати?
A. Гемолітична
B. Паренхіматозна (печінкова)
C. Механічна
D. Жовтяниця немовлят
E. Хвороба Жильбера

6. Хвора, 37 років, яка страждає на системний червоний вовчак, звернулась до лікаря зі скаргами на
слабкість, задуху, серцебиття, біль у суглобах. При обстеженні виявлені: гепатоспленомегалія, жовтяниця
(гіпербілірубінемія 25 мкмоль/л з підвищенням непрямої фракції), нормохромна анемія (гемоглобін 90 г/л,
кольоровий показник 1,0), швидкість зсідання еритроцитів 25 мм/год, гіперглобулінемія, гемоглобінурія. Назвіть
зміни внутрішніх органів, характерні для хронічного перебігу гемолітичної анемії.
A. Ліпофусциноз
B. Жирова дистрофія
C. Інфаркти
D. Продуктивне інтерстиційне запалення
E. @Гемосидероз

7. Хворий, 9 років, який страждав на хворобу Мінковського-Шоффара, помер від чергового гемолітичного
кризу. На розтині виявлені: жовтяниця, остеопороз, гепатоспленомегалія, паренхіматозна дистрофія. Внутрішні
органи з бурим відтінком (легені, нирки, печінка, селезінка). Назвіть зміни, що спричинили спленомегалію при
даному захворюванні.
A. Амілоїдоз
B. @Гемосидероз
C. Інфаркти
D. Мієлоїдна метаплазія
E. Ціанотична індурація

8. Хворого, 52 років, госпіталізовано до хірургічного відділення з ознаками механічної жовтяниці


(загальний білірубін крові 80 мкмоль/л, прямий 60 мкмоль/л). У загальному аналізі крові виявлені патологічні
форми еритроцитів: зменшені за розмірами, без характерного просвітлення в центрі. Під час операції в загальній
жовчній протоці виявлено пігментний камінець, що перекривав її просвіт на рівні papilla Fateri, та спленомегалію.
Проведено спленектомію та літотомію з дренуванням жовчної протоки. У післяопераційному періоді у хворого
розвинулась виражена анемія з відсутністю ретикулоцитів у периферичній крові. Визначити вид анемії у даного
хворого:
A. Гостра постгеморагічна
B. Хронічна постгеморагічна
C. @Спадкова мікросфероцитарна гемолітична
D. Аутоімунна
E. Дефіцитна

9. Хворого, 35 років, госпіталізовано у тяжкому стані з опіками шкіри 2-3 ступеня. На третю добу сеча
стала темною, з червоним відтінком. В аналізі: крові нормохромна анемія (еритроцити 2,3х1012/л, гемоглобін 50
г/л, кольоровий показник 1,0), гіпербілірубінемія з підвищенням некон’югованої фракції, в сечі - гемоглобінурія.
Визначити вид анемії:
A. @Гемолітична внутрішньосудинна
B. Гемолітична внутрішньоклітинна
C. Залізодефіцитна
D. Перніціозна
E. Постгеморагічна

10. Хворий, який переніс малярію, звернувся до лікаря зі скаргами на блідість шкіри з жовтим відтінком,
болі в кістках. При обстеженні виявлені: гепатоспленомегалія, тромбофлебіт нижніх кінцівок. В аналізі крові:
гіпохромна анемія, наявність еритробластів та еритроцитів серпоподібної форми, помірний лейкоцитоз. Вказати
можливу причину даної анемії:
A. Дефіцит вітаміну В12
B. Дефект структури мембрани еритроцитів
C. Дефект структури ферментів еритроцитів
D. @Дефект структури гемоглобіну
E. Дефіцит заліза

11. Хворий, 20 років, скаржиться на загальну слабкість, запаморочення, швидку втомлюваність. У крові:
Hb- 80 г/л. Мікроскопічно: еритроцити зміненої форми. Причиною цього стану може бути:
A. Серпоподібноклітинна анемія
B. Паренхіматозна жовтяниця
C. Гостра переміжна порфірія
D. Обтураційна жовтяниця
E. Хвороба Аддісона

12. Хвора, 18 років, звернулась до лікаря зі скаргами на появу жовтяниці під час менструацій. При
обстеженні виявлені: помірна спленомегалія, гіпербілірубінемія з підвищенням непрямої фракції, нормохромна
анемія макросфероцитарного типу з незначним ретикулоцитозом, анізопойкілоцитозом, зниження активності
піруваткінази. Активність глюкозо-6-фосфатдегідрогенази еритроцитів та структура гемоглобіну не змінені.
Вказати вид анемії:
A. @Внутрішньоклітинна, еритроцитоферментопатична
B. Внутрішньоклітинна, еритроцитопатична
C. Внутрішньосудинна, еритроцитопатична
D. Внутрішньосудинна, аутоімунна
E. Внутрішньосудинна, гаптенова
13. При ультразвуковому обстеженні вагітної в порожнині матки виявлено 2 живих плода. На 24 тижні
вагітності з’явилися ознаки порушення смаку, підвищений попит до вживання в їжу сирих макаронних та
круп’яних виробів, крейди. В аналізі крові: гіпохромна анемія, еритроцити не змінені, вміст заліза в плазмі крові
знижений. Хронічні екстрагенітальні захворювання не виявлені. Визначити патогенетичну ланку у розвитку
залізодефіцитної анемії:
A. Резорбційна недостатність
B. @Підвищений попит на залізо
C. Аліментарна недостатність
D. Еритроцитоферментопатія
E. Гемоглобінопатія

14. Пацієнтка, 58 років, скаржиться на підвищену втомлюваність, зниження працездатності, сонливість,


задишку під час швидкої ходи. У крові: ер.- 4,6 • 1012/л, Hb- 92 г/л, КП- 0,6. У мазку крові - велика кількість
гранулоцитів та мікроцитів. Для якої анемії це характерно?
A. @Залізодефіцитна
B. Постгеморагічна
C. Гемолітична
D. Перніціозна
E. Серповидноклітинна

15. На розтині померлої дитини, яку тривалий час лікували цитостатиками з приводу гострого
лімфобластного лейкозу (стадія рецидиву) і яка померла від сепсису, встановлені: анемія, множинні петехіальні
крововиливи у слизові та серозні оболонки, гемосидероз, жирова дистрофія паренхіматозних органів, виразково-
некротичний гастроентероколіт. Червоний кістковий мозок плоских і трубчастих кісток майже повністю
заміщений жировим. В аналізі периферичної крові: панцитопенія, нормохромна анемія (гемоглобін 20 г/л),
абсолютна кількість лімфоцитів 1,5х109/л, швидкість зсідання еритроцитів 50 мм/год. Визначити вид анемії:
A. Гемолітична
B. Постгеморагічна
C. Мегалобластична
D. @Апластична
E. Залізодефіцитна

16. Хворого, 45 років, прооперовано з приводу перфорації хронічної виразки шлунка. З черевної
порожнини вилучено близько 1000,0 мл кров’янистої рідини з домішками їжі та згортків крові. На 2-3 добу
післяопераційного періоду хворий став блідим, кволим, спостерігалась мелена. В аналізі крові – еритроцити
3,8·1012/л, гемоглобін 90 г/л, кольоровий показник 1,0, лейкоцити 9,6·109/л, тромбоцити 450·109/л, поодинокі
еритрокаріоцити, ШЗЕ 15 мм/год, гіперазотемія при нормальних показниках сечовини та концентрації заліза в
плазмі крові. Визначити вид анемії:
A. @Гостра постгеморагічна
B. Хронічна постгеморагічна
C. Аліментарна, залізодефіцитна
D. Гостра гемолітична
E. Хронічна гемолітична

17. Хворий, 10 років, з вадами розвитку, який страждав на сімейну апластичну анемію Фанконі, помер від
сепсису. На розтині: пігментація шкіри, множинні крововиливи в серозні та слизові оболонки, гнійна пневмонія,
септична селезінка, гострий поліпозно-виразковий ендокардит, абсцеси печінки, нирок. В аналізі крові:
панцитопенія, гіперхромна анемія, анізоцитоз, підвищення швидкості зсідання еритроцитів. Зміни у кістковому
мозку пласких кісток, які спричинили розвиток даної анемії:
A. Остеосклероз
B. Мієломатоз
C. @Панмієлофтиз
D. Гіперплазія
E. Остеомієліт

18. У хворого, 54 років, здійснено субтотальну резекцію шлунка з приводу злоякісного новоутворення.
Через 5 років виявлено: блідість шкірних та слизових покровів, атрофія сосочків язика, помірна
гепатоспленомегалія. В аналізі крові – гіперхромна анемія, анізо- та пойкілоцитоз з наявністю мегалобластів і
мегалоцитів, лейкопенія, нейтропенія з полісегментацією ядер, тромбоцитопенія, гіпербілірубінемія за рахунок
непрямого білірубіну. Визначити вид анемії:
A. Гемолітична
B. Постгеморагічна
C. Залізодефіцитна
D. @Перніціозоподібна В12 -дефіцитна
E. Перніциозна первинна

19. Чоловік, 56 років, потрапив до клініки зі скаргами на загальну слабкість, біль і печіння в язиці,
відчуття оніміння в кінцівках. У минулому переніс резекцію кардіального відділу шлунка. У крові: Hb- 80 г/л;
ер.- 2,0 • 1012/Л; КП- 1,2, лейк.- 3,5 • 109/Л. Який вид анемії у цього хворого?
A. @В12-фолієводефіцитна
B. Гемолітична
C. Постгеморагічна
D. Апластична
E. Залізодефіцитна

20. При якому виді анемії зустрічається дефіцит внутрішнього фактора Касла?
A. @В-12-фолієводефіцитна анемія
B. Постгеморагічна анемія
C. Залізодефіцитна анемія
D. Гіпопластична анемія
E. Гемолітична анемія

21. У хворого на анемію при аналізі крові виявили велику кількість мегалобластів. Назвіть цей вид анемії.
A. @В-12 – фолієво дефіцитна
B. Залізодефіцитна
C. Гемолітична
D. Постгеморагічна
E. Апластична

22. У хворого на анемію при дослідженні крові виявлено переважання еритробластів, нормобластів та
мегалобластів. Такі ж клітини знайдено в червоному кістковому мозку. Для якого виду анемії характерна така
картина крові?
A. @B-12-фолієводефіцитна
B. Гемолітична
C. Залізодефіцитна
D. Післягеморагічна
E. Апластична

23. У хворого при огляді порожнини рота визначається атрофія слизової оболонки язика з червоними
плямами (Гунтеровський глосит). Склери жовтуваті. В аналізі крові – кольоровий показник більше одиниці. Для
якої анемії характерні ці зміни?
A. @В-12-фолієводефіцитна
B. Залізодефіцитна
C. Гостра постгеморагічна
D. Хронічна постгеморагічна
E. Гемолітична

24. При розтині чоловіка, який страждав на хронічну анемію, знайдено лимонно-жовтий відтінок шкіри,
водянисту кров, гунтеровський глосит, атрофічний гастрит. Печінка, селезінка та нирки - щільні, бурого кольору,
кістковий мозок малиново-червоний, соковитий, має вигляд малинового желе. Який з перелічених діагнозів
найбільш вірогідний?
A. @Перніціозна анемія
B. Анемія Фанконі
C. Анемія Ерліха
D. Спадкова гемолітична анемія
E. Залізодефіцітна анемія

25. У хворого, який перенiс 5 рокiв тому субтотальну резекцiю шлунка, розвинулась В12-
фолiєводефiцитна анемiя. Який механiзм є провiдним у розвитку такої анемiї?
A. @Вiдсутнiсть внутрiшнього фактора Касла
B. Вiдсутнiсть зовнiшнього фактора Касла
C. Порушення всмоктування вiтамiну В12 в тонкiй кишцi
D. Дефiцит фолiєвої кислоти
E. Дефiцит транскобаламiну

26. Хворий, 45 років, звернувся до лікаря зі скаргами на поганий апетит, млявість, біль в епігастральній
ділянці, нудоту та здуття живота після прийому їжі, схильність до закрепу. При обстеженні виявлені: глосит,
гіперхромна анемія мегалобластичного типу, гастринемія, ахлоргідрія після гістамінової стимуляції.
Патогістологічне заключення: «Хронічний атрофічний гастрит з відсутністю парієтальних клітин фундального
відділу шлунка». Визначити вид анемії:
A. Анемія Альберс-Шемберга
B. @Анемія Аддісона-Бірмера
C. Анемія Ерліха
D. Хвороба Мінковського-Шоффара
E. Хвороба Кулі (талассемія)

27. Швидкою медичною допомогою до клініки доставлено хвору, 46 років, зі скаргами на луночкову
кровотечу впродовж 6 годин після видалення зуба, загальну слабкість. В анамнезі - гіпертонічна хвороба.
Об’єктивно: блідість шкіри та видимих слизових оболонок, кількість гемоглобіну у крові 80 г/л, гематокрит -
30%, показникі часу кровотечі та зсідання крові знаходяться в межах норми. Яке ускладнення розвинулося в
хворої в зв’язку з кровотечею?
A. Гемолітична анемія
B. Залізодефіцитна анемія
C. Хронічна постгеморагічна анемія
D. Фолієводефіцитна анемія
E. @Гостра постгеморагічна анемія

ІХС
ВАРІАНТ №

1. Під час емоційно напруженої роботи раптово помер молодий чоловік. На розтині виявлено
нерівномірне кровонаповнення міокарда. Гістохімічно – зниження вмісту глікогену в кардіоміоцитах.
Електронномікроскопічно – деструкція мітохондрій, контрактури міофібрил. Вкажіть ймовірний розлад
кровообігу?
A. Хронічна ішемія
B. @Гостра ішемія
C. Вакатна артеріальна гіперемія
D. Гостра венозна гіперемія
E. Ангіоневротична артеріальна гіперемія

2. Хворий, 56 років, помер через 2 години після нападу стенокардії. На аутопсії виявлено: атеросклероз
вінцевих артерій, нерівномірне кровонаповнення міокарда. Гістохімічно – відсутність забарвлення телуритом
калію, зниження вмісту глікогену. Електронномікроскопічно – деструкція мітохондрій. Вкажіть ймовірний
патологічний процес у міокарді?
A. @Гостра ішемія
B. Хронічна ішемія
C. Гостра венозна гіперемія
D. Ангіоневротична артеріальна гіперемія
E. Інфаркт міокарда
3. В патологоанатомічній практиці для ранньої (до 6 год.) макроскопічної діагностики гострої ішемії
міокарда використовують барвник, ефект якого проявляється лише в м’язовій тканині з незруйнованою
ферментативною системою. Назвіть цей барвник?
A. @Солі тетразолію
B. Толуоїдиновий синій
C. Йодисті сполуки
D. 10% розчин сірчаної кислоти
E. Конго червоний

4. При розтині померлого від набряку легень в міокарді знайдено великий осередок жовто-сірого кольору,
а в коронарній артерії – свіжий тромб. Уточніть діагноз:
A. @Інфаркт міокарда
B. Кардіосклероз
C. Міокарди
D. Амілоїдоз
E. Кардіоміопатія

5. Під час гістологічного дослідження серця померлого від гострої серцевої недостатності у міокарді
лівого шлуночка виявлено ділянку омертвіння, відмежовану від неушкодженої тканини зоною повнокровних
судин, дрібних крововиливів та лейкоцитарною інфільтрацією. Який діагноз найбільш імовірний?
A. Ішемічна дистрофія міокарда
B. @Інфаркт міокарда
C. Дифузний ексудативний міокардит
D. Осередковий ексудативний міокардит
E. Продуктивний міокардит
6. Назвіть вид інфаркту за макроскопічними ознаками, який найчастіше зустрічається в міокарді.
A. @Білий з геморагічним вінчиком
B. Геморагічний
C. Білий
D. Змішаний
E. Червоний

7. Назвіть найбільш імовірну ранню (1-2 години) ознаку інфаркту міокарда:


A. @Зникнення глікогену в кардіоміоцитах
B. Вакуольна дистрофія кардіоміоцитів субендокардіальної зони
C. Жирова інфільтрація кардіоміоцитів
D. Падіння активності дегідрогеназ
E. Розслаблення міофібрил

8. У хворого о 8 годині ранку з’явилися болі за грудиною, о 9 годині ранку в приймальному відділенні за
даними ЕКГ встановлено інфаркт міокарда. Через 10 хвилин хворий помер. Яку найбільш імовірну морфологічну
ознаку інфаркту міокарда знайдуть при гістологічному дослідженні після розтину трупа?
A. Зникнення глікогену в кардіоміоцитах
B. Вакуольну дистрофію кардіоміоцитів субендокардіальної зони
C. Жирову інфільтрацію кардіоміоцитів
D. @Некроз міокардіоцитів
E. Розслаблення міофібрил кардіоміоцитів

9. Чоловік, 38 років, раптово помер. На розтині у задній стінці лівого шлуночка серця виявлено інфаркт
міокарда. Які найбільш вірогідні зміни у будові кардіоміоцитів можна побачити в осередку інфаркту
мікроскопічно?
A. Жирова дистрофія
B. Вуглеводна дистрофія
C. @Каріолізис
D. Звапнування
E. Білкова дистрофія

10. Хвора, 65 років, перенесла інфаркт міокарда, а через 8 тижнів померла від гострої серцевої
недостатності. На розтині виявлено інфаркт на стадії організації та свіжий інфаркт. Свіжий інфаркт міокарда
називається:
A. Повторний
B. @Рецидивний
C. Первинний
D. Гострий
E. Хронічний

11. На розтинi в серцi виявлено наступнi змiни: великий осередок некрозу бiлого кольору з червонною
облямiвкою, який захоплює всю товщу серцевого м’яза. На зовнiшнiй оболонцi серця - ознаки фiбринозного
перикардиту. Який найбiльш iмовiрний дiагноз?
A. @Трансмуральний iнфаркт мiокарда
B. Субендокардiальний iнфаркт мiокарда
C. Субепiкардiальний iнфаркт мiокарда
D. Iнтрамуральний iнфаркт міокарда
E. Мiокардит

12. Під час розтину померлого, 43 років, який страждав на ІХС з розвитком інфаркту міокарда,
патологоанатом виявив набряк легень. Які патологічні зміни могли зумовити набряк легень?
A. @Гостра лівошлуночкова недостатність
B. Гостре загальне недокрів’я
C. Гостра правошлуночкова недостатність
D. Ішемия малого кола кровообігу
E. Стаз крові
13. Внаслідок якого ускладнення гострого інфаркту міокарда може розвинутись тампонада серця?
A. @Міомаляція
B. Дрібноосередковий кардіосклероз
C. Крупноосередковий кардіосклероз
D. Кардіогенний шок
E. Фібриляція шлуночків

14. Хворий, 59 років, який страждав на трансмуральний інфаркт міокарда лівого шлуночка, помер від
справжнього розриву серця – гемотампонади перикарда. Який процес у зоні інфаркту міг сприяти розриву серця?
A. Заміщення сполучною тканиною зони інфаркту (організація)
B. Підвищення тиску в малому колі кровообігу
C. @Аутолітичне розплавлення тканини міокарда (міомаляція)
D. Формування рубця зі стоншенням стінки лівого шлуночка
E. Гіперплазія кардіоміоцитів
15. У хворого, який переніс інфаркт міокарда, після фізичного навантаження відмічаються ознаки
недостатності кровообігу: ціаноз та набряк підшкірної клітковини нижніх кінцівок. Які морфологічні зміни
розвиваються на місці інфаркту міокарда у особи, після одужання?
A. @Кардіосклероз
B. Внутрішньоклітинна регенерація
C. Мітотичне новоутворення кардіоміоцитів
D. Міокардит
E. Атрофія міокарда

16. У чоловіка, 63 років, який багато років страждав на атеросклероз і переніс раніше інфаркт міокарда,
розвинувся напад загрудинного болю. Хворого госпіталізували, через 3 дні, на фоні прогресуючої серцево-
судинної недостатності, він помер. Під час розтину тіла у ділянці задньої стінки лівого шлуночка і
міжшлуночкової перегородки виявлено ділянку білого кольору близько 3см у діаметрі, волокнисту, западаючу, з
чіткою межею. Прозектор трактував ці зміни як:
A. Дистрофія міокарда
B. Міокардит
C. Ішемія міокарда
D. Інфаркт міокарда
E. @Осередковий кардіосклероз

17. Під час розтину трупа чоловіка, 60 років, у міокарді передньої стінки лівого шлуночка виявлено сірий
неправильної форми щільний осередок 5х4 см з чіткими межами, волокнистої структури. Який діагноз найбільш
вірогідний?
A. Дифузний дрібноосередковий міокардіосклероз
B. Міокардит
C. @Післяінфарктний міокардіосклероз
D. Кардіоміопатія
E. Інфаркт

18. Хворий переніс повторний інтрамуральних інфаркт міокарда. Після лікування та реабілітації його
виписали у задовільному стані під нагляд дільничого терапевта. Через 2 роки він загинув в автомобільній
катастрофі. Встановити характер патологічного процесу в міокарді, який було встановлено на розтині.
A. @Крупноосередковий кардiосклероз
B. Дрiбноосередковий кардiосклероз
C. Атрофiя
D. Некроз
E. Гiперплазiя

19. Під час розтину в міокарді макроскопічно виявлено крупний щільний осередок сірого кольору, що
гістологічно складається з грубоволокнистої сполучної тканини, оточеної гіпертрофованими м’язовими
волокнами. Які зміни виникли у серці?
A. @Постінфарктний крупноосередковий кардіосклероз
B. Ішемічна стадія інфаркту міокарда
C. Некротична стадія інфаркту міокарда
D. Дрібноосередковий дифузний кардіосклероз
E. Міокардит

20. Хворий, 70 років, помер від гострого порушення мозкового кровообігу. На розтині: в правій півкулі
головного мозку - ішемічний інсульт та тромбоембол у просвіті правої середньо-мозкової артерії, атеросклероз
аорти та стегнових артерій, хронічна аневризма лівого шлуночка серця з пристінковими тромбами, флеботромбоз
та набряк лівої гомілки. Визначити можливе джерело тромбемболії артерій головного мозку:
A. Гомілкові вени
B. @Стінка хронічної аневризми серця
C. Черевний відділ аорти
D. Правий атріо-вентрикулярний клапан
E. Стегнові артерії

21. Під час дослідження коронарних артерій виявлені атеросклеротичні бляшки з кальцинозом, що
закривають просвіт судин на 1/3. У м’язі дрібні множинні білуваті прошарки сполучної тканини. Як називається
процес, виявлений у міокарді?
A. @Дифузний кардіосклероз
B. Тигрове серце
C. Післяінфарктний кардіосклероз
D. Міокардит
E. Інфаркт міокарда
22. Хворий, 58 рокiв, помер вiд прогресуючої серцевої недостатностi. На розтинi: серце розширене у
поперечнику, дрябле, м’яз на розрiзi - нерiвномiрного кровонаповнення, пiстрявий. При гiстологiчному
дослiдженнi: у мiокардi - повнокров’я, у стромi - лiмфогiстiоцитарнi iнфiльтрати, що розсувають кардiомiоцити.
Виявленi морфологiчнi змiни свiдчать про:
A. @Негнiйний промiжний мiокардит
B. Венозне повнокров’я
C. Жирову дистрофiю мiокарда
D. Кардiосклероз
E. Iнфаркт мiокарда
23. При розтинi тiла дитини, померлої вiд серцевої недостатностi, виявлено: розширення порожнин
шлуночкiв серця. Мiкроскопiчно в стромi мiокарда: повнокров’я, набряк, розповсюдженi iнфiльтрати з
гiстiоцитiв, лiмфоцитiв, нейтрофiлiв, еозинофiлiв. Який найбiльш iмовiрний дiагноз?
A. @Дифузно-промiжний ексудативний мiокардит
B. Осередковий промiжний ексудативний мiокардит
C. Вузликовий продуктивний мiокардит
D. Промiжний продуктивний мiокардит
E. Альтеративний мiокардит
24. Смерть хворого настала від серцево-судинної недостатності. На аутопсії: серце значно збільшене в
розмірах, дрябле, порожнини розширені з тромботичними накладаннями на ендокарді; міокард на розрізі
строкатий, клапани не змінені. Назвіть патологічний процес.
A. @Ізольований ідіопатичний міокардит
B. Ревматичний міокардит
C. Міокардіопатія
D. Гіпертрофія міокарда при гіпертонічній хворобі
E. Вади мітрального клапана

25. У померлого від раптової зупинки серця чоловіка, 45 років, знайдено симетричний тип ожиріння III
ступеня, розрив стінки правого шлуночка з гемоперикардом; під епікардом - надлишкові відкладення жиру.
Мікроскопічно: жирова тканина з епікарда розповсюджується у міокард з атрофією м'язових волокон. Який
процес найбільш вірогідний?
A. @Ожиріння серця
B. Жирова дистрофія міокарда
C. Гострий інфаркт міокарда
D. Iшемічна хвороба серця
E. Гіпертонічна хвороба

26. На розтині померлого, 48 років, виявлено обтурацію просвіту середньої мозкової артерії тромбом. У
тім`яно-скроневій області лівої півкулі головного мозку – осередок кашоподібної консистенції сірого кольору.
Найвірогідніше це:
A. Секвестр
B. Гангрена
C. Казеозний некроз
D. Фібриноїдний некроз
E. @Інфаркт
27. При розтині померлого хворого на атеросклероз у головному мозку виявлено тромбоз гілки
внутрішньої сонної артерії та сірого кольору осередок вологого роз'якшення тканини. Який патологічний процес
виявлено в головному мозку?
A. @Ішемічний інфаркт
B. Геморагічна інфільтрація
C. Гематома
D. Енцефаліт
E. Пухлина мозку

28. У хворого, 75 років, який довгий час страждав на атеросклероз церебральних судин, на аутопсії
виявлені: тромбоз правої середньої мозкової артерії, великий осередок неправильної форми сірого кашоподібного
розм’якшення мозкової тканини. Який патологічний процес розвинувся в спинному мозку?
A. @Ішемічний інфаркт
B. Геморагічний інфаркт
C. Коагуляційний некроз
D. Гума мозку
E. -
29. У померлого, 58 років, при розтині в правій скроневій частці головного мозку знайдено велику ділянку
розм’якшеної сірої речовини кашицеподібної консистенції, блідо-сірого кольору. В артеріях основи мозку -
чисельні білувато-жовті потовщення інтими, які різко звужують просвіт. Ваш діагноз?
A. @Ішемічний інфаркт головного мозку
B. Абсцесс головного мозку
C. Крововилив головного мозку
D. Геморагічний інфаркт головного мозку
E. Набряк головного мозку

30. У померлої з атеросклерозом судин головного мозку на аутопсії у лівій півкулі головного мозку
виявлено осередок, що представлений дряблою, безструктурною, сірувато-жовтуватою тканиною, з нечіткими
краями. Про наявність якого патологічного процессу йде мова?
A. Мультифокальний ріст пухлини з кістозним перетворенням
B. @Ішемічний інсульт
C. Безліч осередків свіжих і старих крововиливів у мозок
D. Осередковий енцефаліт
E. Стареча енцефалопатія

31. При розтині тіла померлого, 56 років, у правій скроневій частці головного мозку знайдено великий
осередок розм’якшеної сірої речовини кашицеподібної консистенції, блідо-сірого кольору. В артеріях основи
мозку - чисельні білувато-жовті потовщення інтими, які різко звужують просвіт. Який найбільш вірогідний
діагноз?
A. @Ішемічний інсульт
B. Абсцес мозку
C. Крововилив
D. Геморагічний інсульт
E. Набряк мозку

32. У хворого похилого віку виникло гостре порушення мозкового кровообігу з коматозним станом і
смертельним наслідком. На розтині: в правій півкулі головного мозку виявлено велику порожнину, заповнену
згортками крові. Який патологічний процес виявлено в головному мозку?
А. Геморагічну інфільтрацію
В. Інфаркт мозку
С. @Гематому
D. Діапедезний крововилив
E. Пухлину мозку
33. На розтині померлого в підкоркових ядрах правої півкулі головного мозку визначається порожнина
неправильної форми 5х3х4 см, заповнена червоними згортками крові і розмякшеною тканиною мозку. Назвіть
патологію, яка розвинулась у головному мозку.
A. @Гематома
B. Геморагічне просякнення
C. Ішемічний інфаркт
D. Кіста
E. Абсцес

34. На розтинi тiла померлого, який страждав на гiпертонiчну хворобу, у лiвiй гемiсферi мозку виявлено
порожнину округлої форми 4х5 см iз iржавою стiнкою, заповнену жовтуватою прозорою рiдиною. Назвiть
патологiю, яка розвинулась у головному мозку хворого:
A. @Кiста
B. Iшемiчний iнфаркт
C. Геморагiчне просякнення
D. Гематома
E. Абсцес

35. Літня людина перенесла інфаркт правої півкулі головного мозку. Через рік, враховуючі відсутність
рухомості в лівих кінцівках, проведено комп’ютерну томографію мозку, яка у правій півкулі виявила порожнину
з гладкими стінками, заповнену ліквором. Який патологічний процес знайдено в головному мозку?
A. @Постінфарктна кіста
B. Гідроцефалія
C. Сіре розм’якшення мозку
D. Інфаркт мозку
E. Гематома

36. Хворий, 68 років, з ожирінням ІІ-ІІІ ступеня, помер на 5 добу перебування у відділенні реанімації. На
розтині: набряк головного мозку, атеросклероз судин головного мозку з осередком некрозу в лівій скронево-
базальній ділянці, розмірами 2х1,5х1см, сіро-білого кольору з перифокальними крапчасто-зливними
крововиливами. Визначити можливий наслідок вказаного процесу:
A. Рубець
B. Осифікація
C. Каверна
D. @Кіста
E. Секвестр

ОСНОВИ ОНКОМОРФОЛОГІЇ.
ВАРІАНТ №

1. Хворий помер від раку шкіри. З анамнезу встановлено, що він 15 років працював у науковій лабораторії
з вивчення впливу іонізуючого опромінення на життєві об'єкти. Назвіть, яке іонізуюче опромінення привело до
цього захворювання.

A. α-промені
B. β-промені
C. Нейтрони
D. @Рентгенівське
E. γ-промені

2. У хворої, яка мешкає в Чорнобильськiй зонi, через 5 рокiв пiсля аварiї збiльшилась щитовидна залоза, в
бiоптатi дiагностовано папiлярний рак залози. Вкажiть провідний механiзм у розвитку цього захворювання.

A. Дефіцит йоду в їжi


B. Стрес
C. Травма
D. @Вплив радiонуклiдiв
E. Надлишок йоду в їжi

3. Мікроскопічне дослідження новоутворення шкіри показало: паренхіма складається з покривного


епітелію зі збільшеним числом шарів. Строма разом з розростаннями епітелію формує сосочки. Вкажіть вид
атипізму.
A. Клітинний
B. @Тканинний
C. Метаболічний
D. Функціональний
E. Гістохімічний
4. При гiстологiчному дослiдженнi новоутворення шкiри виявлено: паренхiма сформована з покривного
епiтелiю зі збiльшеним числом шарiв. В епiтелiї збережена полярнiсть клiтин, стратифiкацiя, цiлiснiсть власної
мембрани. Строма розташована в центрi сосочкiв. Вкажiть вид атипiзму.

A. Клiтинний.
B. Метаболiчний.
C. Функцiональний.
D. Морфологiчний.
E. @Тканинний

5. У хворого в прямій кишці виявлені дрібні вузли на тонкій ніжці. При мікроскопічному дослідженні
біоптату слизової оболонки встановлено розростання сполучної тканини і залоз. Про який процес йдеться?

A. @Гіперпластичний поліп
B. Гіпертрофія
C. Метаплазія
D. Атрофія
E. Склероз

6. При ендоскопiчному дослiдженнi сечового мiхура проведено бiопсiю пухлини, що складається з


тонких, розгалужених сосочкiв, вкритих декiлькома рядами клiтин перехiдного епiтелiю. Назвiть пухлину:

A. @Папiлома
B. Базалiома
C. Перехiдноклiтинна карцинома
D. Плоскоклiтинна карцинома
E. Фiброаденома

7. У жінки на шкірі шиї виявляється зморшкувате новоутворення кулястої форми, на ніжці. В біоптаті:
надмірна проліферація покривного епітелію і строми шкіри; полярність клітин, їх комплексність, власна
мембрана збережена, є тканинний атипізм. Ваш діагноз?

A. @Папілома
B. Рак
C. Фіброма
D. Невус
E. Саркома

8. Хворому видалено пухлинне утворення шкіри, яке має вигляд вузла з сосочковою поверхнею (нагадує
цвітну капусту), щільної консистенції. Мікроскопічно пухлина складається з багатьох сосочків. Паренхіма
сформована з покривного епітелію зі збільшеним числом шарів. В епітелії збережена полярність клітин, шари
стратифіковані, цілісність власної мембрани збережена. Строма пухлини розташована в центрі сосочків. Який
найвірогідніший діагноз?

A. @Папілома
B. Фіброма
C. Аденома
D. Фіброаденома
E. Цистаденома

9. У хворого, 34 років, з'явилася охриплість голосу, під час ларингоскопії виявлено пухлину гортані, сіро-
білого кольору з сосочковою поверхнею. Мікроскопічно: розростання сполучної тканини, яка вкрита
багатошаровим плоским епітелієм зі збільшеним зроговінням, клітинна атипія відсутня. Достовірніше всього це:

A. Фіброма
B. Поліп
C. Ангіома
D. Ангіофіброма
E. @Папілома
10. Гістологічне дослідження біопсії пухлини сечового міхура показало, що вона складається з тонких
сосочків, що гілкуються, вкритих декількома рядами клітин перехідного епітелію. Назвіть пухлину.

A. Базаліома
B. Перехідно-клітинна карцинома
C. Плоскоклітинна карцинома
D. @Перехідно-клітинна папілома
E. Фіброаденома

11. Вкажiть доброякiсну органонеспецифiчну пухлину з епiтелiю:

A. Фiброма
B. Семiнома
C. @Аденома
D. Лейомiома
E. Лiпома

12. У хворої з молочної залози оперативно видалено щiльний вузол дiаметром 2.0 см у капсулi. На розрізi
вiн бiло-рожевий, волокнистий. Мiкроскопiчно: пухлина представлена залозами без клiтинного атипiзму, що
здавленi сполучною тканиною. У пухлинi строма переважає над залозистою паренхiмою. Ваш дiагноз?

A. Пролiферативна мастопатiя
B. Аденокарцинома
C. Непролiферативна мастопатiя
D. Аденома
E. @Фiброаденома

13. У жінки, 20 років, під час медичного оляду при пальпації в молочній залозі виявлено щільний
інкапсульований вузол діаметром 1.0см. Результат післяопераційного біопсійного дослідження: розростання
сполучної тканини навколо проток молочної залози та залозисті елементи різного діаметра, що не утворюють
часточок, без ознак клітинного атипізму. Яка відповідь патологоанатома?

A. @Фіброаденома
B. Фіброма
C. Метастаз раку
D. Аденома
E. Фіброзний рак

14. При дослідженні біоптату вагінальної порції шийки матки у базальному відділі епітеліального пласта
виявлено збільшення кількості клітин, підвищення мітотичної активності і порушення полярності клітин. Вкажіть
характер виявленої патології.

A. Помірна дисплазія
B. Важка дисплазія
C. @Легка дисплазія
D. Акантоз
E. Спонгіоз

15. У 47-рiчного чоловiка пiд час колоноскопiї дiагностовано полiп прямої кишки, який був видалений.
При гiстологiчному дослiдженнi полiпа виявленi характернi морфологiчнi ознаки аденоматозного полiпа, а також
деякi ознаки малiгнiзацiї. Вкажiть найдостовiрнiшу морфологiчну ознаку малiгнiзацiї.

A. Епiтелiальнi клiтини у залозах мають гiперхромнi ядра


B. У залозистому епiтелiї спостерiгаються множиннi мiтози
C. @Поодинокi залозистi структури знаходяться у нiжцi полiпа
D. Пухлина має органоїдну будову
E. Полiп побудований з призматичного епiтелiю, який утворює залозистi структури

16. Гістологічне дослідження біоптату з ділянки пухлини слизової оболонки правого бронха, показало
наявність клітинного і тканинного атипізму, появу структур у вигляді «ракових перлин». Назвіть патологічний
процес.

A. Доброякісна пухлина
B. Гіперплазія
C. Метаплазія
D. Гіпоплазія
E. @Злоякісна пухлина

17. При гістологічному дослідженні біоптату ерозії шийки матки у хворої, 47 років, виявлені ознаки
клітинного атипізму, базальна мембрана - без змін. Поставити діагноз.

A. @Карцинома in situ
B. Ерозія
C. Аденокарцинома
D. Папілома
E. Ендометріоз

18. У шкiрi виявлено щiльну, рухому, чiтко вiдмежовану вiд оточуючих тканин пухлину. На розрiзi вона
бiлого кольору, представлена волокнистою тканиною. Мiкроскопiчно: хаотично переплетенi колагеновi волокна,
клiтин мало. Назвіть вид росту пухлини?

A. @Експансивний
B. Інвазивний
C. Інфільтруючий
D. На місці
E. Апозиційний

19. При лапаротомії у чоловіка, 49 років, виявлено пухлину в ділянці сигми, що проростає усі її шари та
обтурує просвіт кишки. Який характер росту у виявленої пухлини по відношенню до оточуючих тканин?

A. Ендофітний
B. Експансивний
C. Мультицентричний
D. @Інфільтруючий
E. Екзофітний

20. У чоловіка, 37 років, при гастроскопічному дослідженні в ділянці малої кривизни виявлене пухлинне
утворення діаметром 1,5 см на ніжці. Який характер росту пухлини?

A. Експансивний
B. Інфільтруючий
C. Апозиційний
D. @Екзофітний
E. Ендофітний

21. На розтині тіла померлого від ракової інтоксикації, виявлено: потовщення стінки шлунка до 1,2 см,
слизова оболонка нерухома, її складки не виявляються. На розрізі - тканина однорідна, біляста, хрящоподібної
щільності. Для якої форми пухлини характерні описані зміни?

A. Вузол
B. @Інфільтрат
C. Виразка
D. Виразково-інфільтративна
E. Кіста

22. У хворого на шкірі щоки поступово розвинулася бляшка з некрозом і виразкою в центрі. При
патогістологічному дослідженні біоптату виявлено розростання атипових епітеліальних клітин з великою
кількістю патологічних мітозів. Ваш діагноз.

A. @Рак шкіри
B. Саркома
C. Папілома
D. Трофічна виразка
E. Фіброма

23. У хворого на шкірі обличчя з'явилося пухлинне утворення бляшковидної форми з виразкою. Ваш
попередній діагноз до гістологічного дослідження.

A. Текома
B. @Базаліома
C. Карциноїд
D. Пінеалома
E. Тімома

24. Чоловік, 45 років, звернувся до лікаря з приводу бляшкоподібного утвору на шиї. В біоптаті шкіри
гістологічно виявлено пухлинні клітини, розташовані гніздами, які мають круглу та овальну форму з вузьким
обідком базофільної цитоплазми та схожі на клітини базального шару епідермісу. Як називається пухлина?

A. @Базаліома
B. Епідермальний рак
C. Гідраденома
D. Трихоепітеліома
E. Сирінгоаденома

25. У хворого, 69 рокiв, на шкiрi в дiлянцi нижньої повiки з’явилося невелике бляшкоподiбне утворення з
наступним виразкуванням, яке було оперативно видалене. При мiкроскопiчному дослiдженнi утворення: в дермi
шкiри - комплекси з атипових епiтелiальних клiтин; по периферiї клiтини розташованi перпендикулярно до
базальної мембрани. Клiтини темнi, призматичної полiгональної форми з гiперхромними ядрами з частими
мiтозами. Iнодi зустрiчаються утворення, подiбнi до волосяного фолiкула. Яка гiстологiчна форма раку у
хворого?

A. @Базально-клiтинний рак
B. Плоскоклiтинний рак з ороговiнням
C. Плоскоклiтинний рак без ороговiння
D. Аденокарцинома
E. Недиференцiйований рак

26. При гістологічному дослідженні ворсинчатої пухлини сечового міхура встановлено, що вона
побудована з клітин атипового перехідного епітелію з патологічними мітозами, які формують пухлинні сосочки і
проростають в м'язовий шар. Назвіть вид пухлини.

A. Ворсинчата аденома
B. @Перехідно-клітинний рак
C. Папілома
D. Аденокарцинома
E. Саркома

27. У чоловіка, 64 років, який тривалий час курив і вживав міцні спиртні напої, на бічній поверхні язика
виявили подібний на виразку утвір з білої, помірно щільної тканини розмірами 5х3 см. При гістологічному
дослідженні біоптату виявили, що він побудований з клітин, які формують солідні структури і тяжі, що
нагадують за будовою багатошаровий плоский епітелій, в якому клітини з вираженим поліморфізмом, з великими
атиповими ядрами з патологічними мітозами. Діагностуйте виявлене у чоловіка захворювання:

A. @Плоскоклітинний незроговілий рак


B. Плоскоклітинний зроговілий рак
C. Еритроплакія
D. Рак на місці
E. Лейкоплакія

28. Гістологічне дослідження біопсії бронха виявило пухлину, яка побудована з гніздових скупчень
атипових клітин багатошарового плоского епітелію, місцями з характерними “перлинами”. Ваш діагноз?
A. Плоскоклітинний рак без зроговіння
B. Солідний рак
C. Слизовий рак
D. Скір
E. @Плоскоклітинний рак зі зроговінням

29. У хворого, 52 років, з клінічним діагнозом «Хронічний бронхіт, пневмосклероз», взято біоптат з
підозрілої ділянки слизової оболонки лівого бронха. Гістологічно встановлено клітинний і тканинний атипізм,
поява структур у вигляді ”ракових перлин”. Якому патологічному процесу відповідають виявлені гістологічні
зміни?

A. Хронічний поліпозний бронхіт


B. @Плоскоклітинний рак бронха зі зроговінням
C. Бронхоектаз
D. Гострий бронхіт
E. Плоскоклітинна метаплазія слизової оболонки бронха

30. Чоловік, 37 років, який страждав на хронічний бронхіт, помер від виснаження. На розтині в просвіті
правого бронха виявлено ендофітне розростання світло-сірої м’якуватої тканини. Мікроскопічне дослідження:
серед розростань атипового плоского епітелію - «рогові перлини». Ваш діагноз.

A. Плоскоклітинний рак без зроговіння


B. Аденокарцинома
C. @Плоскоклітинний рак зі зроговінням
D. Недиференційований рак
E. Апудома

31. У хворого, який довго палив та страждав на хронiчний бронхiт виявлено пухлину легень. Пiд час
операцiї встановлено, що пухлина тiсно пов'язана зi стiнкою бронха i росте у виглядi полiпа. Мiкроскопiчно
виявлено групи полiморфних епiтелiальних клiтин, мiтози, помiж яких зустрiчають лускатi оксифiльнi без'ядернi
структури. Назвати гiстологiчний тип пухлини?

A. Рабдомiосаркома
B. Солiдна аденокарцинома
C. @Плоскоклiтинний рак зі зроговiнням
D. Слизовий рак
E. Щiльна папiлома

32. Чоловiкові, 56 рокiв, проведено бронхоскопiю, в ходi якої в лiвому головному бронсi виявлено
екзофiтну пухлину, яка звужує просвiт бронха на 2/3; гiстологiчно - пухлина побудована з полiморфних
епiтелiальних клiтин, з численними патологiчними мiтозами та гiперхромними ядрами; пухлиннi клiтини
формують тяжi та гнiзда, в центральних частинах яких виявляються еозинофiльнi, концентрично орiєнтованi
маси. Дiагностуйте пухлину.

A. Великоклiтинний рак
B. @Плоскоклiтинний рак з кератинізацією
C. Залозисто-плоскоклiтинний рак
D. Дрiбноклiтинний рак
E. Плоскоклiтинний рак без кератинізації

33. У хворого, 52 років, взято біопсію слизової оболонки бронха. При гістологічному дослідженні
виявлені тяжі атипових клітин епітелію, які вростають в підлягаючу тканину. В центрі тяжів виявлені
концентричні утворення рожевого кольору (“ракові перлини”). Назвіть вид пухлини.

A. Помірно диференційована аденокарцинома


B. Плоскоклітинна рак без ороговіння
C. Меланома
D. Перехідно-клітинний рак
E. Інвазивний плоскоклітинний рак зі зроговінням

34. У біоптаті шийки матки при гістологічному дослідженні виявлено клітинну і ядерну атипію
багатошарового плоского епітелію, патологічні мітози, а також рогові перлини в глибині епітеліальних шарів.
Ваш діагноз:

A. Перехідно-клітинний рак
B. Плоскоклітинний рак без зроговіння
C. Залозистий рак
D. Анапластичний рак
E. @Плоскоклітинний рак зі зроговінням
35. При мiкроскопiчному дослiдженнi бiоптату шийки матки хворої на хронiчний ендоцервiкоз виявлені
клiтинна i ядерна атипiї багатошарового плоского епiтелiю, патологiчнi мiтози, а також роговi перлини в глибинi
епiтелiальних шарiв. Ваш дiагноз:

A. @Плоскоклiтинний рак зi зроговiнням


B. Перехiдноклiтинний рак
C. Залозистий рак
D. Плоскоклiтинний рак без зроговiння
E. Анапластичний рак

36. У жінки, 63 років, почалася кровотеча в менопаузі. Гістологічно: у вишкребках слизової оболонки
шийки матки виявлено розростання атипового епітелію з утворенням так званих "ракових перлин". Ваш діагноз?

A. @Плоскоклітинний рак зі зроговінням


B. Аденокарцинома
C. Плоскоклітинний рак без зроговіння
D. Слизовий рак
E. Недиференційований рак

37. У літньої жінки з метрорагією в менопаузі при патогістологічному дослідженні у вишкребках слизової
оболонки шийки матки виявлені розростання атипового епітелію з утворенням так званих ракових перлин. Ваш
діагноз?

A. @Плоскоклітинний рак зі зроговінням


B. Аденокарцинома
C. Плоскоклітинний рак без зроговіння
D. Слизовий рак
E. Недиференційований рак

38. При гістологічному дослідженні вишкребків слизової оболонки матки у хворої, 48 років, з клінічним
діагнозом «Маткова кровотеча» виявлено: розростання атипових залозистих структур, які складаються з клітин з
гіперхромними ядрами, фігурами мітозів, атипією. Атипові залозисті структури вростають у міометрій. Виберіть
патологічний процес для якого характерні виявлені мікроскопічно зміни?

A. @Аденокарцинома матки
B. Залозиста гіперплазія ендометрію
C. Гострий ендометрит
D. Плацентарний поліп
E. Хоріонепітеліома матки

39. У хворої в клімактеричному періоді з'явилися рецидивуючі маткові кровотечі. Виконано діагностичне
вишкрібання матки. У вишкребках ендометрію серед елементів крові розташовані залозисті елементи різної
величини і форми, утворені атиповими клітинами з гіперхромними ядрами, з численними мітозами (у тому числі
патологічними). Назвіть процес.

A. Залозиста гіперплазія ендометрію


B. @Аденокарцинома
C. Хоріонепітеліома
D. Аденоматозний поліп
E. Ознаки перерваної вагітності

40. При мікроскопічному дослідженні вишкребків слизової оболонки матки у хворої, 50 років, з клінічним
діагнозом «Порушення оваріально-менструального циклу» виявлено розростання залозистих структур, які
складаються з поліморфних клітин з гіперхромними ядрами, фігурами мітозів. Ваш діагноз:

A. @Аденокарцинома матки
B. Плацентарний поліп
C. Гострий ендометрит
D. Залозиста гіперплазія ендометрію
E. Хоріонепітеліома матки

41. При мікроскопічному дослідженні біоптату легень виявлено атипові клітини, які формують множинні
ацинарні структури і продукують слиз. Яка гістологічна форма раку легень має місце у хворого?

A. @Аденокарцинома
B. Солідний
C. Недиференційований рак
D. Плоскоклітинний рак
E. Залозистоплоскоклітинний рак

42. При гістологічному дослідженні біопсії легені виявлені атипові клітини, які формують множинні
ацинарні структури і продукують слиз. Вкажіть форму раку легень.

A. @Високодиференційована аденокарцинома
B. Помірно-диференційована аденокарцинома
C. Недиференційований рак
D. Помірно-диференційований плоскоклітинний рак
E. Залозисто-плоскоклітинний рак

43. Гістологічне дослідження вузла у видаленій молочній залозі показало, що серед рясної строми
присутні різних розмірів і форми комплекси атипових поліморфних епітеліальних клітин з наявністю просвітів у
центрі комплексів. Клітини з крупними ядрами, наявністю атипових мітозів. Поставте діагноз:

A. Плоскоклітинний незроговілий рак


B. Солідний рак
C. @Аденокарцинома
D. Фіброаденома молочної залози
E. Недиференційований поліморфноклітинний рак

44. При мiкроскопiчному дослiдженнi бiоптату товстої кишки виявлено пухлину з призматичного
епiтелiю, що формує атиповi залозистi структури рiзної форми i розмiру. Клiтини полiморфнi, ядра гiперхромнi, є
патологiчнi мiтози. Базальна мембрана залоз зруйнована. Деякi залози вистеленi псевдобагаторядним епiтелiєм,
просвiт збережений. Ваш дiагноз.

A. @Аденокарцинома
B. Базально-клiтинний рак
C. Недиференцiйований рак
D. Солiдний рак
E. Слизовий рак
45. При гістологічному дослідженні біоптату з товстої кишки, виявлено пухлину з призматичного
епітелію, що формує атипові залозисті структури різної форми і розмірів. Клітини поліморфні, ядра гіперхромні,
є патологічні мітози. Базальна мембрана залоз зруйнована. Ваш діагноз.

A. Неінвазивна аденокарцинома
B. Солідний рак
C. @Інвазивна аденокарцинома
D. Слизовий рак
E. Недиференційований рак

46. У чоловіка, 50 років, при гастроскопії на малій кривизні в препілоричному відділі знайдене утворення
кратероподібної форми. З краю утворення взято біопсію. Дані гістологічного дослідження: пухлина з
залозистоподібними структурами різної форми і величини, які вростають в оточуючу тканину, з вираженим
атипізмом клітин. Назвіть вид даної пухлини.

A. Плоскоклітинний рак
B. Скір шлунка
C. Слизовий рак шлунка
D. @Аденокарцинома
E. Солідний рак шлунка

47. При мікроскопічному дослідженні пухлини шлунка виявлено багато перснеподібних клітин. Назвіть
гістологічний варіант раку.

A. Аденокарцинома
B. Саркома
C. Солідний
D. @Слизовий
E. Карциноїд

48. При ендоскопiї шлунка виявлено атрофiю слизової оболонки. Мiкроскопiчно в бiоптатi виявлено
пухлину, побудовану з ланцюжкiв атипових епiтелiальних клiтин, розташованих серед тяжiв грубоволокнистої
сполучної тканини; строма значно переважає над паренхiмою. Який найбiльш iмовiрний дiагноз?

A. @Скірозний рак
B. Солiдний рак
C. Медулярний рак
D. Дрiбноклiтинний рак
E. Аденокарцинома

49. При гiстологiчному дослiдженнi бiоптатiв, взятих з потовщених країв виразки шлунка, виявленi
невеликi гнiздовi скупчення рiзко атипових гiперхромних невеликих епiтелiальних клiтин, якi розташованi серед
дуже розвиненої строми. Визначте пухлину:

A. @Скiрозний недиференцiйований рак


B. Медулярний рак
C. Аденокарцинома
D. Недиференцiйована саркома
E. Аденома

50. При мікроскопічному дослідженні пухлини молочної залози виявлено, що пухлина побудована з
низькодиференційованих атипових клітин епітеліального походження, які утворюють трабекули, розділені між
собою прошарками сполучної тканини, співвідношення клітин і строми приблизно 1:1. Назвіть гістологічний
варіант раку.

A. @Солідний
B. Аденокарцинома
C. Медулярний
D. Скірозний
E. Недиференційований

51. При гістологічному дослідженні біоптату пухлини молочної залози визначаються солідні пласти,
побудовані з дрібних епітеліальних клітин з поліморфними ядрами, з великою кількістю патологічних мітозів.
Строми дуже мало, присутня лімфоцитарна інфільтрація. Ваш діагноз.

A. Скірозний рак
B. Хвороба Педжета
C. @Медулярний рак
D. Аденома
E. Фіброаденома

52. При бронхоскопічному дослідженні у хворого, 65 років, у початковому відділі верхньочасткового


бронха правої легені знайдено утворення у вигляді поліпа діаметром до 1,0 см. При гістологічному дослідженні
виявлено пухлину, яка складається з дрібних лімфоцитоподібних клітин з гіперхромними ядрами, клітини
ростуть пластами і тяжами. Вкажіть вид пухлини.

A. Недиференційований великоклітинний рак


B. @Недиференційований дрібноклітинний рак
C. Плоскоклітинний рак
D. Аденокарцинома
E. Залозисто-плоскоклітинний рак

53. У хворої, 24 років, видалено пухлину яєчника, яка представлена кістою, діаметром 8 см. Під час
патогістологічного дослідження було з’ясовано, що кіста має в своєму складі жирову, хрящову і нервову тканини,
волосся. До якого різновиду пухлин відноситься дане утворення?
А. Пухлина жирової тканини
В. Епітеліальна органоспецифічна
С. Епителіальна органонеспецифічна
D. @Тератома
Е. Пухлина сполучної тканини

54. У хворого, 60 років, при обстеженні був діагностований рак передміхурової залози з метастазами у
нижні відділи хребта, кістки таза та патологічний перелом стегна. Який патологічний процес лежить в основі
розвитку метастазів пухлини?

A. Некроз тканини
B. Мікробна емболія
C. Газова емболія
D. @Тканинна емболія
E. Емболія сторонніми тілами

55. У хворої, 36 років, амбулаторно було видалено темно-коричневого кольору "родимку" на щкірі правої
гомілки. Гістологічне дослідження не проводилось. Через 5 місяців у правій паховій області з'явився конгломерат
лімфатичних вузлів. При гістологічному дослідженні одного з вузлів були виявлені метастази меланоми. Який
шлях метастазування пухлини має місце у хворої?

A. Гематогенний
B. Анапластичний
C. Змішаний
D. Периневральний
E. @Лімфогенний

56. При розтині трупа чоловіка, 59 років, померлого від раку легень, знайдені множинні метастази. Які
метастази можна розцінити як імплантаційні (контактні) за механізмом розвитку?

A. Метастази в перібронхіальні, біфуркаційні, паратрахеальні лімфатичні вузли


B. @Дрібні множинні пухлинні вузлики на плеврі
C. Метастази в головний мозок
D. Метастази в надниркові залози
E. Проростання пухлини з бронха в стравохід

57. Під час операції з приводу пухлини яєчника у жінки в черевній порожнині виявлено 2 літри
геморагічної рідини, в очеревині – велика кількість вузликів діаметром до 0,5 см. При цитобіопсії у вузликах
виявлено розростання ракових клітин. Який процесс розвинувся у черевній порожнині?
А. Туберкульоз
В. Геморагічний перитоніт
С. Фібропластичний перитоніт
D. Асцит
Е. @Канцероматоз очеревини

58. У жінки, 39 років, під час паліативної операції з приводу раку шлунка виявлені метастази в яєчники
(“крукенбергський рак яєчників”). Який шлях метастазування призвів до ураження яєчників?

A. @Лімфогенний ретроградний
B. Лімфогенний ортоградний
C. Гематогенний
D. Імплантаційний
E. Каналікулярний

59. У хворої, 45 років, виявлені збільшені надключичні лімфатичні вузли. При дослідженні біопсійного
матеріалу лімфатичного вузла виявлено метастаз перстенедібно-клітинного раку. Оберіть найбільш ймовірну
локалізацію первинної пухлини.

A. @Рак шлунка
B. Рак стравоходу
C. Рак щитовидної залози
D. Рак легень
E. Рак шийки матки

60. Вкажіть локалізацію вірховського метастазу у випадку раку шлунка:


A. @Ліві надключичні лімфатичні вузли
B. Метастатичний рак яєчників
C. Карциноматоз очеревини
D. Лімфатичні вузли параректальної клітковини
E. Гематогенні метастази в печінку

61. У хворого, 52 років, виявлені збільшені надключичні лімфатичні вузли. При дослідженні біопсійного
матеріалу з лімфатичного вузла виявлено метастаз перснеподібно-клітинного раку. Локалізацію первинної
пухлини:

A. @Шлунок
B. Стравохід
C. Щитовидна залоза
D. Легені
E. Шийка матки

62. У померлої жінки, 41 років, оперованої у минулому з приводу пухлини шлунка, на розтині виявлено
різко збільшений, щільний яєчник білого кольору. Гістологічно: в препараті вкрай атипові перснеподібні
епітеліальні клітини, які розміщені серед сполучної тканини. Яке захворювання мало місце у хворої?

A. Серозна цистаденокарцинома
B. @Крукенбергський рак яєчника
C. Псевдомуцинозна цисткарцинома
D. Злоякісна текома
E. Злоякісна гранульозоклітинна пухлина

63. Вкажіть локалізацію вірховського метастазу у випадку раку шлунка:


A. @Ліві надключичні лімфатичні вузли
B. Метастатичний рак яєчників
C. Карциноматоз очеревини
D. Лімфатичні вузли параректальної клітковини
E. Гематогенні метастази в печінку

64. Локалізація крукенбергівського метастаза у випадку раку шлунка:

A. Карциноматоз очеревини
B. В ліві надключичні лімфатичні вузли
C. @Ретроградні лімфогенні метастази в яєчники
D. Гематогенні метастази в печінку
E. В лімфатичні вузли параректальної клітковини

65. Під час розтину померлого в печінці виявлені метастази раку. Вважаючи, що мова йде про первинні
метастази гематогенного походження, яка локалізація первинної пухлини?
А. Легені
В. @Шлунок
С. Молочна залоза
D. Гортань
Е. Нирки

66. Хворий на рак шлунка з множинними метастазами помер від ракової кахексії. Назвати характерні
зміни в серці, які були виявлені на ротині:

A. @Бурая атрофія міокарда


B. Амілоїдна кардіомегалія
C. Дилятаційна кардіоміопатія
D. Гіпертрофічна кардіоміопатія
E. "Тигрове" серце

67. У хворого, прооперованого з приводу масивної заочеревинної пухлини, розвинулася гостра ниркова
недостатність. Які морфологічні зміни лежать в її основі?

A. Тромбоемболія ниркової артерії


B. Апоптоз
C. @Некротичні зміни ептелію канальців
D. Гематурія
E. Некротичні зміни в клубочках нирки
ПУХЛИНИ ЕПІТЕЛІАЛЬНІ РІЗНИХ ОРГАНІВ
ВАРІАНТ №

1. Мікроскопічне дослідження новоутворення шкіри показало: паренхіма складається з покривного епітелію зі


збільшеним числом шарів. Строма разом з розростаннями епітелію формує сосочки. Вкажіть вид атипізму.
А. Гістохімічний
B. Клітинний
С. Метаболічний
D. Функціональний
Е. @Тканинний

2. У жінки на шкірі шиї утворилося кулястої форми, зморшкувате новоутворення на ніжці. У біоптаті:
надлишкова проліферація покривного епітелію і строми шкіри, полярність клітин, їх комплексність, власна
мембрана збережена, тканинний атипізм. Ваш діагноз?
A. Саркома
B. Рак
C. Фіброма
D. Невус
E. @Папілома

3. Хворому видалено пухлинне утворення на шкірі, яке має вигляд вузла з сосочковою поверхнею (нагадує
кольорову капусту), щільної консистенції. Мікроскопічно пухлина складається з багатьох сосочків. Паренхіма
сформована з покривного епітелію зі збільшеним числом шарів. В епітелії збережена полярність клітин,
стратифікація ядер, цілісність власної мембрани. Строма пухлини розташована в центрі сосочків. Який найбільш
ймовірний діагноз?
А. @Папілома
B. Фіброма
C. Аденома
D. Фіброаденома
E. Цистаденома

4. У хворого на шкірі обличчя з'явилося пухлинне утворення бляшкоподібної форми з виразкою. Ваш
припустимий діагноз?
A. Карциноїд
B. Текома
C. @Базаліома
D. Пінеалома
E. Тимома

5. Чоловік, 70 років, звернувся до хірурга з приводу глибокої виразки (ulcus rodens), яка локалізується на шкірі
обличчя. Виберіть форму пухлини для якої таке ускладнення є найбільш характерним.
A. Недиференційований рак
B. Плоскоклітинний рак зі зроговінням
C. Плоскоклітинний рак без зроговіння
D. @Базально-клітинний рак
E. Рак з потових залоз

6. У чоловіка, 64 років, на шкірі обличчя з’явилася пухлина. Мікроскопічно: розростання дрібних, витягнутих
епітеліальних клітин, які формують чітко відокремлені тяжі і альвеоли. Зустрічаються поодинокі мітози. Ваш
діагноз:
А. Трихоепітеліома
В. Сarcinoma in situ
С. Саркома
D. @Базаліома
Е. Плоскоклітинний рак

7. Чоловік, 48 років, звернувся до лікаря з приводу бляшкоподібного утворення на шиї. Гістологічно в біоптаті
шкіри виявлені пухлинні клітини, розташовані гніздами, мають круглу і овальну форму з вузьким обідком
базофільної цитоплазми і схожі на клітини базального шару епідермісу. Вкажіть назву пухлини.
A. Епідермальний рак
B. Гідраденома
C. @Базаліома
D. Трихоепітеліома
E. Сирінгоаденома
8. У хворої на шкірі щоки поступово розвинулася бляшка з некрозом і виразкою в центрі. При патогістологічному
дослідженні біоптату виявлено розростання атипових епітеліальних клітин з великою кількістю патологічних
мітозів. Який найбільш вірогідний діагноз?
А. @Рак шкіри
B. Саркома
C. Папілома
D. Трофічна виразка
E. Фіброма

9. При гістологічному дослідженні біоптату піхвової частини шийки матки у хворої, 47 років, з ерозією, яка
довгий час не гоїлась, були виявлені ознаки клітинного атипізму, базальная мембрана - без змін. Поставте
діагноз.
А. @Карцинома in situ
В. Ерозія
С. Аденокарцинома
D. Папілома
Е. Ендометріоз

10. При гістологічному дослідженні біоптату ерозії шийки матки у хворої, 47 років, виявлено ознаки клітинного
атипізму, базальна мембрана - без змін. Поставити діагноз.
А. Папілома
B. Ерозія
C. Аденокарцинома
D. @Неінвазивний рак
E. Ендометріоз

11. У жінки, 65 років, почалася кровотеча в менопаузі. При гістологічному дослідженні в слизовій оболонці
шийки матки виявлені розростання атипового епітелію з утворенням так званих ракових перлин. Ваш діагноз?
A. Слизовий рак
B. Аденокарцинома
C. Плоскоклітинний рак без зроговіння
D. @Плоскоклітинний рак зі зроговінням
E. Недиференційований рак

12. У біоптаті шийки матки під час гістологічного дослідження виявлено: клітинну і ядерну атипію
багатошарового плоского епітелію, патологічні мітози, а також рогові перлини в глибині епітеліальних шарів.
Ваш діагноз:
A. @Плоскоклітинний рак зі зроговінням
B.Перехідно-клітинний рак
C.Плоскоклітинний рак без зроговіння
D. Залозистий рак
E. Анапластичний рак
13. У хворої в клімактеричному періоді з'явилися рецидивуючі маткові кровотечі. Зроблено діагностичне
вишкрібання матки. В ендометрії серед елементів крові видно залозисті елементи різної величини і форми,
утворені атиповими клітинами з гіперхромними ядрами, з численними мітозами (в тому числі і патологічними).
Про який процес можна думати?
А. @Аденокарцинома
B. Залозиста гіперплазія ендометрію
C. Хоріонепітеліома
D. Аденоматозний поліп
E. Ознаки перерваної вагітності

14. Під час гістологічного дослідження вишкрібків слизової оболонки матки у хворої, 48 років, з клінічним
діагнозом «Маткова кровотеча» виявлено: розростання атипових залозистих структур, які складаються з клітин з
гіперхромними ядрами, фігурами мітозів, атипією. Атипові залозисті структури вростають у міометрій. Для якого
патологічного процесу характерні виявлені мікроскопічно зміни?
А. Залозиста гіперплазія ендометрію
B. @Аденокарцинома матки
C. Гострий ендометрит
D. Плацентарний поліп
E. Хоріонепітеліома матки

15. При мікроскопічному дослідженні вишкрібків слизової оболонки матки у хворої, 50 років, з клінічним
діагнозом «Порушення оваріально-менструального циклу» виявлено розростання залозистих структур, які
складаються з поліморфних клітин з гіперхромними ядрами, фігурами мітозів. Для якого патологічного процесу
характерні виявлені гістологічні зміни?
А. Залозиста гіперплазія ендометрію
B. Плацентарний поліп
C. Гострий ендометрит
D. @Аденокарцинома матки
E. Хоріонепітеліома матки

16. У жінки, 25 років, вагітність закінчилася смертю плода на ранніх строках. Під час гістологічного дослідження
вмісту матки виявлено гідропічне і кістозне переродження ворсин хоріона, конгломерати яких нагадують гроно
винограду. Який патологічний процесс розвинувся в плаценті?
А. Залозисто-кістозна гіперплазія
В. Плацентарний поліп
С. Хоріонепітеліома
D. Фолікулярні кісти
Е. @Міхуровий занос

17. Хвора, 45 років, оперується з приводу пухлини матки. Макроскопічно: губчастий, строкатий вузол у
міометрії. Гістологічно - великі світлі епітеліальні клітини, серед яких багато темних поліморфних клітин.
Строма відсутня, судини мають вигляд порожнин, які вистелені клітинами пухлини. Виявляються множинні
крововиливи. Як називається виявлена пухлина?
A. Деструюючий (злоякісний) міхуровий занос
B. Аденокарцинома
C. @Хоріонепітеліома
D. Кавернозна гемангіома
E. Медулярний рак

18. У молодої жінки, яка страждала на метрорагію, під час операції виявлено червону, м’яку, губчасту пухлину,
що кровоточить; побудовану з атипових синцитіальних клітин і клітин трофобласта, яка проростає з ендометрію
в міометрій. Ваш діагноз?
А. Рак матки
В. Фіброміома
С. Міосаркома
D. Гемангіома
Е. @Хоріонкарцинома

19. У 30-річної жінки з матковими кровотечами під час огляду в стінці матки виявлено пухлину. Мікроскопічно
виявлені світлі епітеліальні клітини Лангханса, серед яких багато гігантських і поліморфних клітин синцитію;
строма відсутня, судини мають вигляд порожнин висланих клітинами пухлини. Який патологічний процес
найбільш вірогідний?
А. Рак матки
В. @Хоріонепітеліома
С. Міхуровий занос
D. Ендометрит
Е. Ендометріоз

20. У жінки, 32 років, через 8 місяців після аборту розвинулася маткова кровотеча. При гінекологічному огляді
виявлено пухлину матки. Макроскопічно: тканина темно-червоного кольору з множинними порожнинами, що
нагадує “губку”. При мікроскопічному дослідженні пухлини в лакунах крові знайдені атипові світлі епітеліальні
клітини Лангханса і гігантські клітини синцитіотрофобласта. Назвіть пухлину.
A. @Хоріонепітеліома
B. Плоскоклітинний незроговілий рак
C. Аденокарцинома
D. Фіброміома
E. Міхуровий занос

21. У жінки, 25 років, через 4 тижні після аборту з’явився кашель, підвищилась температура тіла. На
рентгенограмі в легенях виявлені осередки затемнення з обох боків. Впродовж 10-ти днів хвора отримувала
антибіотики, але без ефекту. Було проведено крайову резекцію легені. В препараті знайдено пухлину, яка
складається з клітин трофобласта та поліморфних гігантських елементів синцитіотрофобласта з високою
мітотичною активністю; строма відсутня, множинні крововиливи. Заключення по гістологічній картині:
A. @Хоріонкарцинома
B. Світлоклітинний рак
C. Плоскоклітинний рак
D. Трофобластична пухлина плацентарного ложа
E. Аденокарцинома

22. При гістологічному дослідженні вмісту порожнини матки, який був отриматий шляхом діагностичного
вишкрібання, виявлено хоріонепітеліому. З яких тканинних елементів розвивається ця пухлина?
A. Тканини яєчника
B. Слизової оболонки тіла матки
C. @Тканинних елементів плаценти
D. Слизової оболонки цервікального каналу
E. -

23. У жінки, 36 років, під час пальпації молочної залози виявлено вузол 2 см в діаметрі. При гістологічному
дослідженні виявляється сполучна тканина з ділянками гіалінозу, в якій розташовуються атрофічні часточки і
кістозно-розширені протоки. Протоки і кісти вистелені атрофічним епітелієм. Яке захворювання молочної залози
найбільш вірогідне?
А. Мастит
В. Рак
С. Фіброаденома
D. @Доброякісна дисплазія
Е. Хвороба Педжета

24. У біопсійному матеріалі молочної залози виявлені ознаки тканинного атипізму, що виражаються в порушенні
співвідношення паренхіми і строми, з переважанням останньої, різних розмірів і форми залозистих структур, з
щілинно-подібними просвітами, які вистелені одношаровим проліферуючим епітелієм. Ваш припустимий
діагноз?
A. @Інтраканалікулярна фіброаденома
B. Папілома
C. Неінфільтруючий рак
D. Інфільтруючий рак
E. Мастит

25. У жінки, 25 років, під час медичного огляду при пальпації в молочній залозі виявлено щільний вузол
діаметром 1,0 см. Результат біопсійного дослідження: вузол у капсулі, у вузлі - розростання сполучної тканини
навкруги базальної мембрани проток молочної залози, залозисті елементи різного діаметра, які не утворюють
часточок, клітинний атипізм відсутній. Поставте діагноз.
A. @Периканалікулярна фіброаденома
B. Фіброма
C. Метастаз раку
D. Аденома
E. Фіброзний рак

26. При гістологічному дослідженні видаленого збільшеного пахвового лімфатичного вузла у ньому виявлено
пухлину органоїдної будови, яка складається з комплексів атипових епітеліальних клітин різної виличени і
форми з гіперхромними ядрами і великою кількістю мітозів. Дані клітини мають рецептори до естрогенів та
прогестерону. Яка найбільш вірогідна локалізація первинної пухлини?
A. @Рак молочної залози
B. Дрібноклітинний недиференційований рак легені
C. Аденокарцинома шлунка
D. Плоскоклітинний рак стравоходу зі зроговінням
E. Медулярна карцинома щитовидної залози

27. Гістологічне дослідження вузла молочної залози показало, що серед рясної строми присутні різних розмірів і
форми комплекси атипових поліморфних епітеліальних клітин з наявністю просвітів у центрі комплексів.
Клітини з великими ядрами, збільшеним числом ядерець і ядерцевих організаторів, наявністю атипових мітозів.
Поставте діагноз.
A. @Аденокарцинома
B. Фіброаденома молочної залози
C. Солідний рак
D. Плоскоклітинний незроговілий рак
E. Недиференційований поліморфноклітинний рак

28. При мікроскопічному дослідженні пухлини молочної залози виявлено, що вона побудована з
низькодиференційованих атипових клітин епітеліального походження, які утворюють трабекули, розділені між
собою прошарками сполучної тканини, співвідношення клітин і строми приблизно 1:1. Назвіть гістологічний
варіант раку.
А. Дрібноклітинний
B. Аденокарцинома
C. Плоскоклітинний
D. Скірозний
E. @Солідний

29. При гістологічному дослідженні біоптату пухлини молочної залози визначаються солідні пласти, побудовані з
дрібних епітеліальних клітин з поліморфними ядрами, з великою кількістю патологічних мітозів. Строми дуже
мало, присутня лімфоцитарна інфільтрація. Встановіть вид пухлини.
А. Скірозний рак
B. @Медулярний рак
C. Хвороба Педжета
D. Аденома
E. Аденофіброма

30. Об'єктивно: у жінки, 39 років, біля соска молочної залози визначається мокнуча ділянка, неглибока виразка із
запальною гіперемією і набряком шкіри. При гістологічному дослідженні біоптату цієї ділянки в базальному шарі
потовщеного епідермісу виявлені атипові великі клітини зі світлою і оптично порожньою цитоплазмою, з
відсутністю міжклітинних містків. Такі клітини знайдені і в гирлі великих проток залози. Встановіть діагноз.
A. Внутрішньопротоковий рак
B. Базально-клітинний рак
C. Плоскоклітинний рак
D. @Хвороба Педжета
E. Меланома

31. У жінки, 37 років, об'єктивно: сосок з ареолою молочної залози набряклий, пастозний, має вигляд "лимонної
шкірки". Уся молочна залоза збільшена, ущільнена. В біоптаті виявляються солідні пласти, побудовані з
шароподібних клітин з поліморфними ядрами, з великою кількістю патологічних мітозів. Строма дуже бідна, з
лімфоцитарною інфільтрацією. Встановити діагноз.
A. Медулярний рак
B. Аденофіброма
C. Аденома
D. @Хвороба Педжета
E. Скірозний рак

32. У жінки, 25 років, з’явилися ознаки гірсутизму (підвищене оволосіння), аменорея. Про пухлину якого органа
слід думати?
А. Нирки
В. @Яєчника
С. Печінки
D. Молочної залози
Е. Матки

33. У хворої, 24 років, видалено пухлину яєчника, яка представлена кістою, діаметром 8 см. Під час
патогістологічного дослідження було з’ясовано, що кіста має в своєму складі жирову, хрящову і нервову тканини,
волосся. До якого різновиду пухлин відноситься дане утворення?
А. Пухлина жирової тканини
В. Епітеліальна органоспецифічна
С. Епителіальна органонеспецифічна
D. @Тератома
Е. Пухлина сполучної тканини

34. У чоловіка, 37 років, при гастроскопічному дослідженні в ділянці малої кривизни виявлено пухлиноподібне
утворення діаметром 1,5 см на ніжці. Який характер росту пухлини?
А. Ендофітний
B. Експансивний
C. Інфільтруючий
D. Апозиційний
E. @Екзофітний

35. На розтині тіла чоловіка, 50 років, померлого від ракової інтоксикації, виявлено потовщення стінки шлунка до
1.2 см, слизова оболонка нерухома, її складки не визначаються. На розрізі тканина однорідна, білувата,
хрящоподібної щільності. Для якої макроскопічної форми пухлини характерні описані зміни?
А. Виразково-інфільтративна
B. Вузол
C. Виразка
D. @Інфільтрат
E. Кіста

36. У хворого, 42 років, виявлені збільшені надключичні лімфатичні вузли. При дослідженні матеріалу біопсії з
лімфатичного вузла виявлено метастаз персневидно-клітинного раку. Виберіть найбільш ймовірну локалізацію
первинної пухлини.
А. @Шлунок
B. Стравохід
C. Щитовидна залоза
D. Легені
E. Шийка матка

37. У чоловіка, 50 років, при гастроскопії на малій кривизні в препілоричному відділі виявлено утворення
кратероподібної форми. З крайової ділянки утворення взято біопсію. Дані гістологічного дослідження: пухлина з
залозистоподібними структурами різної форми і величини, з вираженим атипізмом клітин вростає в навколишню
тканину. Назвіть гістологічний варіант даної пухлини.
А. Плоскоклітинний рак
B. @Аденокарцинома
C. Скір
D. Слизовий рак
E. Солідний рак

38. При мікроскопічному дослідженні пухлини шлунка виявлено багато перснеподібних клітин. Назвіть
гістологічний варіант раку.
А. Саркома
B. Аденокарцинома
C. @Слизовий
D. Солідний
E. Карціноїд

39. При гістологічному дослідженні біоптату з товстої кишки, виявлено пухлину з призматичного епітелію, яка
формує атипові залозисті структури різної форми і розміру. Клітини поліморфні, ядра гіперхромні, є патологічні
мітози. Базальна мембрана залоз зруйнована. Ваш діагноз.
А. @Інвазивна аденокарцинома
B. Базально-клітинний рак
C. Солідний рак
D. Слизовий рак
E. Недиференційований рак

40. Гістологічне дослідження біопсії пухлини сечового міхура показало, що вона складається з тонких,
розгалужених сосочків вкритих декількома рядами клітин перехідного епітелію. Назвіть пухлину.
А. Плоскоклітинна карцинома
B. Базаліома
C. Перехідно-клітинна карцинома
D. @Перехідно-клітинна папілома
E. Фіброаденома

41. При мікроскопічному дослідженні біоптату нирки виявлено пухлину, яка складається зі світлих
полігональних, поліморфних клітин, які вміщують у собі ліпіди, характерні множинні мітози. Клітини утворюють
альвеоли і часточки, залозисті і сосочкові структури, розділені погано вираженою стромою і судинами. Ваш
діагноз?
А. Нефробластома
В. @Нирково–клітинний рак
С. Темноклітинна аденома
D. Світлоклітинна аденома
Е. Ацидофільна аденома

42. У видаленій нирці - чітко відмежований вузол більше 5 см в діаметрі, з крововиливами. При гістологічному
дослідженні вузла було з’ясовано, що він складається з клітин з широким вінчиком вакуолізованої цитоплазми,
невеликими, мономорфними ядрами, які утворюють великі комплекси. Виберіть найбільш вірогідний діагноз:
А. Солідний рак
В. @Світлоклітинний рак
С. Світлоклітинна аденома
D. Трабекулярна аденома
Е. Метастатичний вузол у нирці

43. У чоловіка, 60 років, видалено нирку з пухлиною у вигляді вузла діаметром 8 см. Тканина пухлини на розрізі
строката, з множинними крововиливами, некрозами. Гістологічно: пухлина складається зі світлих клітин, які
утворюють альвеолярні і сосочкові структури, помірно виражений інвазивний ріст. У багатьох клітинах пухлини
визначаються патологічні мітози, гіперхромні ядра. Діагностуйте виявлену пухлину нирки.
A. Світлоклітинна аденома
B. @Світлоклітинний рак
C. Аденокарцинома
D. Нефробластома
E. Ацидофільна аденома з малігнізацією

44. При гістологічному дослідженні матеріалу після трансуретральної резекції простати виявлено переважання
залоз різної форми і величини, які утворені атиповими, гіперхромними клітинами з патологічними мітозами.
Діагнозтуйте захворювання.
А. @Аденокарцинома
В. Аденома
С. Аденоматозна нодулярна гіперплазія
D. М’язово-фіброзна гіперплазія
Е. Хронічний простатит
45. У хворої, 34 років, з'явилася захриплість, під час ларингоскопії виявлено пухлину гортані, сіро-білого кольору
з сосочковой поверхнею на широкій основі. Мікроскопічно: розростання сполучної тканини, яка вкрита
багатошаровим пласким епітелієм зі збільшеним зроговінням, клітинна атипія відсутня. Найвірогідніше це:
А. @Папілома
B. Фіброма
C. Поліп
D. Ангіома
E. Ангіофіброма

46. У хворої, 29 років, з'явилася захриплість, під час лярінгоскопії виявлено пухлину гортані, сіро-білого кольору
з сосочковой поверхнею на ніжці. Мікроскопічно: розростання сполучної тканини, яка вкрита багатошаровим
пласким епітелієм без зроговіння, клітинна атипія відсутня. Найвірогідніше це:
А. @Папілома
B. Фіброма
C. Поліп
D. Ангіома
E. Ангіофіброма

47. Гістологічне дослідження біоптату з ділянки пухлини слизової оболонки правого бронха показало: наявність
клітинного і тканинного атипізму, появу структур у вигляді «ракових перлин». Визначити характер
патологічного процесу.
А. Гіпоплазія
B. Доброякісна пухлина
C. Гіперплазія
D. Метаплазія
E. @Злоякісна пухлина

48. При гістологічному дослідженні біоптату з бронха виявлено пухлину, яка побудована з гніздних скупчень
атипових клітин багатошарового плоского епітелію, місцями з характерними "перлинами". Ваш діагноз?
A. Плоскоклітинний рак без зроговіння
B. @Плоскоклітинний рак зі зроговінням
C. Солідний рак
D. Слизовий рак
E. Скір

49. У хворого, 52 років, взято біопсію слизової оболонки бронха. При гістологічному дослідженні виявлено тяжі
атипових клітин епітелію, які вростають у підлеглу тканину. У центрі тяжів виявлені концентричні утворення
рожевого кольору. Назвіть вид пухлини?
A. @Плоскоклітинний рак зі зроговінням
B. Помірно диференційована аденокарцинома
C. Плоскоклітинний рак без зроговіння
D. Меланома
E. Перехідноклітинний рак
50. Чоловік, 48 років, страждав на хронічний бронхіт, помер від виснаження. На розтині: в просвіті правого
бронха виявлено ендофітні розростання світло-сірої м'якої тканини. Мікроскопічно: серед розростань атипового
плоского епітелію виявлені «рогові перлини». Ваш діагноз.
A. Недиференційований рак
B. Плоскоклітинний рак без зроговіння
C. Аденокарцинома
D. @Плоскоклітинний рак зі зроговінням
E. Апудома

51. У хворої, 52 років, з клінічним діагнозом «Хронічний бронхіт, пневмосклероз» узято біоптат з підозрілої
ділянки слизової лівого бронха. Гістологічно: клітинний і тканинний атипізм, "ракові перлини". Якому
патологічного процесу відповідають ці гістологічні зміни?
A. Бронхоектазія
B. Хронічний поліпозний бронхіт
C. @Плоскоклітинний рак бронха зі зроговінням
D. Гострий бронхіт
E. Плоскоклітинна метаплазія слизової бронха

52. У чоловіка, 55 років, в результаті обтурації середньочасткового бронха м'яким вузлом утворився ателектаз
середньої частки правої легені. При бронхобіопсії в зоні обтурації виявлені розростання залозистоподібного
атипового епітелію з патологічними мітозами, який проростає в підлягаючі тканини і хрящ. Якому захворюванню
найдостовірніше відповідають ці дані?
A. Дисплазія епітелію бронха
B. @Бронхогенний рак легень
C. Запальний поліп
D. Деформуючий бронхіт
E. Саркома бронха

53. При бронхоскопічному дослідженні у хворого, 65 років, у початковому відділі верхньочасточкового бронха
правої легені знайдено утворення у вигляді поліпа діаметром до 1,0 см. При гістологічному дослідженні виявлено
пухлину, яка складається з дрібних лімфоцитоподібних клітин з гіперхромними ядрами, клітини ростуть
пластами і тяжами. Вкажіть, який з перерахованих видів пухлин найбільш вірогідний?
A. @Недиференційований дрібноклітинний рак
B. Недиференційований крупноклітиний рак
C. Плоскоклітинний рак
D. Аденокарцинома
E. Залозисто-плоскоклітинний рак

54. При гістологічному дослідженні біопсії легень виявлено атипові клітини, які формують множинні ацинарні
структури і продукують слиз. Яка гістологічна форма раку легень має місце у хворого?
A. Помірно диференційована аденокарцинома
B. Недиференційований рак
C. @Високодиференційована аденокарцинома
D. Помірно диференційований плоскоклітинний рак
E. Залозисто-плоскоклітинний рак

55. У жінки, 22 років, яка декілька років хворіла на міастенію, в передньому середостінні виявлено пухлину
великих розмірів, яка гістологічно складається з витягнутих (веретеноподібних) клітин з овальними темними
ядрами, які утворюють тяжі, присутні тільця Гассаля. Після хірургічного видалення пухлини ознаки міастенії
почали прогресивно зникати. Діагностуйте пухлину, яка має етіопатогенетичний зв'язок з міастенією.
A. @Тимома
B. Аденома щитовидної залози
C. Аденома паращитовидної залози
D. Парагангліома
E. Медуллобластома

56. У хворої при пальпації щитовидної залози виявлено вузол діаметром 1,5 см. При гістологічному дослідженні
виявлено різного розміру порожнини, які вислані атиповим епітелієм і заповнені сосочками, які вростають у
стінку і капсулу пухлини. Який найбільш вірогідний діагноз?
А. Солідна аденома
В. Фолікулярний рак
С. Папілярна аденома
D. @Папілярний рак
Е. Фолікулярна аденома

57. У хворого під час пальпації щитовидної залози було виявлено вузол діаметром 2 см. Під час гістологічного
дослідження були виявлені великі епітеліальні клітини зі світлою оксифільною цитоплазмою, які розростаються
серед заповнених колоїдом фолікулів. Ваш діагноз?
A. Папілярна аденома
B. Фолікулярна аденома
C. Папілярний рак
D. Фолікулярний рак
E. @Солідна аденома

58. У хворого з підвищеним рівнем паратгормону при гістологічному дослідженні ділянки патологічного
перелому кістки виявлені осередки лакунарного розсмоктування (лакунарної резорбції) остеоїдних балок кістки і
новоутворення фіброзної тканини. Ваш діагноз.
А. @Аденома паращитовидних залоз
В. Мієломна хвороба
С. Мінеральна дистрофія
D. Остеобластокластома
Е. Остеомієліт

59. У хворого з тривалою гіперкальційемією при рентгенологічному дослідженні в нирках виявлено множинні
камені, в кістках - остеопороз. Яка пухлина може призвести до таких змін?
A. @Аденома паращитовидних залоз
B. Пухлина задньої часточки гіпофіза
C. Аденома з базофільних клітин гіпофіза
D. Феохромоцитома
E. Альдостерома

60. У хворого, 28 років, знайдені множинні рецидивуючі пептичні виразки шлунка та дванадцятипалої кишки.
Диагноз «Синдром Золлінгера-Елісона». Вказати з якою пухлиною повязаний цей синдром?
A. Аденома кори наднирників
B. Карциноїд сліпої кишки
C. @Пухлина острівців підшлункової залози
D. Феохромоцитома
E. Аденокарцинома підшлункової залози

61. Хвора, 64 років, за 2 місяці до госпіталізації помітила перші симптоми захворювання у вигляді слабкості і
болей в правому підребер’ї. В стаціонарі відмічалась: кахексія, асцит, пожовтіння шкіри та склер. Хвора померла.
Під час розтину: по малій кривизні шлунка виявлено утворення у вигляді поліпа на товстій ніжці. Печінка дуже
щільна, поверхня дрібнозерниста, ліва часточка представлена осередковими білими розростаннями щільної
консистенції, які зливаються. У нижній третині стравоходу - варикозне розширення вен. Атеросклероз аорти.
Набряк мозку. Ваш діагноз?
A. Прогресуючий портальний цироз печінки
B. Рак шлунка з метастазами в печінку
C. Хронічний гепатит
D. Постнекротичний цироз печінки
E. @Рак печінки на фоні портального циррозу

62. При розтині тіла чоловіка, 59 років, померлого від раку легень, виявлені множинні метастази. Які з нижче
перерахованих метастазів можна розцінити як імплантаційні за механізмом розвитку?
A. Метастази в наднирники
B. Метастази у перибронхіальні, біфуркаційні, паратрахеальні лімфатичні вузли
C. Метастази в головний мозок
D. @Дрібні множинні пухлинні вузлики по плеврі
E. Проростання пухлини з бронха в стравохід

63. У померлої, 48 років, у минулому оперованої з приводу пухлини шлунка, на розтині виявлено: різко
збільшений, щільний яєчник білого кольору. Гістологічно: в препараті вкрай атипові епітеліальні клітини,
розміщені серед пластів і тяжів сполучної тканини. Яке захворювання мало місце у хворої?
A. @Крукенбергівський рак яєчника
B. Серозна цистаденокарцинома
C. Псевдомуцинозна цисткарцинома
D. Злоякісна текома
E. Злоякісна гранульозоклітинна пухлина

64. У жінки, 39 років, під час паліативної операції з приводу раку шлунка виявлені крукенбергівскі метастази в
яєчники. Який шлях метастазування призвів до ураження яєчників?
А. Каналікулярний
B. Лімфогенний ортоградний
C. Гематогенний
D. Імплантаційний
E. @Лімфогенний ретроградний

65. У жінки, 46 років, під час паліативної операції з приводу раку шлунка встановлено наявність
крукенбергівських метастазів в яєчники ("крукенбергівський рак яєчників"). Який з наведених шляхів
метастазування призвів до ураження яєчників?
А. Гематогенний
В. Лімфогенний ортоградний
С. @Лімфогенний ретроградний
D. Імплантаційний
Е. Каналікулярний

66. Локалізація крукенбергівського метастазу у випадку раку шлунка:


A. Карциноматоз очеревини
B. Ліві надключичні лімфатичні вузли
C. @Ретроградні лімфогенні метастази в яєчники
D. Гематогенні метастази в печінку
E. Лімфатичні вузли параректальної клітковини

67. Вкажіть локалізацію вірховського метастазу у випадку раку шлунка:


A. @Ліві надключичні лімфатичні вузли
B. Метастатичний рак яєчників
C. Карциноматоз очеревини
D. Лімфатичні вузли параректальної клітковини
E. Гематогенні метастази в печінку

68. Під час операції з приводу пухлини яєчника у жінки в черевній порожнині виявлено 2 літри геморагічної
рідини, в очеревині – велика кількість вузликів діаметром до 0,5 см. При цитобіопсії у вузликах виявлено
розростання ракових клітин. Який процесс розвинувся у черевній порожнині?
А. Туберкульоз
В. Геморагічний перитоніт
С. Фібропластичний перитоніт
D. Асцит
Е. @Канцероматоз очеревини

69. Під час розтину померлого в печінці виявлені метастази раку. Вважаючи, що мова йде про первинні метастази
гематогенного походження, яка локалізація первинної пухлини?
А. Легені
В. @Шлунок
С. Молочна залоза
D. Гортань
Е. Нирки

70. Хворий на рак шлунка з множинними метастазами помер від ракової кахексії. Назвати характерні зміни в
серці, які були виявлені на ротині:
А. @Бурая атрофія міокарда
В. Амілоїдна кардіомегалія
С. Дилатаційна кардіоміопатія
D. Гіпертрофічна кардіоміопатія
Е. "Тигрове" серце

ПУХЛИНИ МЕЗЕНХІМАЛЬНІ
ВАРІАНТ №

1. При макроскопічному дослідженні пухлини матки (післяопераційний матеріал), вона м'якої


консистенції, з крововиливом і ділянками некрозу, на розрізі нагадує «риб’яче м’ясо». Гістологічне дослідження
встановило ознаки вираженого клітинного і тканинного атипізму, зустрічаються клітини з патологічними
фігурами мітозів. Поставте діагноз.
A. Аденокарцинома
B. Ангіома
C. @Саркома
D. Фіброма
E. Ліпома
2. У чоловіка, 30 років, у шкірі виявлено пухлину - щільну, рухому, чітко відмежовану від навколишніх
тканин. На розрізі вона білого кольору, представлена волокнистою тканиною. Мікроскопічно: хаотично
переплетені колагенові волокна, клітин мало. Назвіть пухлину.
A. @Фіброма
B. Міома
C. Гістіоцитома
D. Дерматофіброма
E. Десмоїд
3. У молодого чоловіка в товщі шкіри визначається щільна пухлина, рухома. При мікроскопічному
дослідженні: хаотично розташовані пучки колагенових волокон з невеликою кількістю веретеноподібних клітин.
Яку пухлину видалено?
A. @Щільна фіброма
B. Лейоміома
C. Меланома
D. Ліпома
E. Гломус-ангіома

4. У патогiстологiчну лабораторiю надiслано шматочок шкiри з пiдшкiрною клiтковиною. Макроскопiчно


в пiдшкiрнiй клiтковинi знайдено щiльну пухлину добре вiдмежовану вiд навколишнiх тканин. При
мiкроскопiчному дослiдженнi пухлинне утворення представлене хаотично переплетеними пучками колагенових
волокон i невеликою кiлькiстю осередково розташованих сполучнотканинних клiтин. Яка пухлина видiлена?
A. Меланома
B. Гемангiома
C. Лейомiома
D. Лiпома
E. @Щiльна фiброма

5. У жінки, 33 років, на передній стінці по білій лінії живота знаходиться пухлиноподібне утворення, яке
під час вагітності почало збільшуватися. При гістологічному дослідженні виявлено, що пухлина побудована з
диференційованої сполучної тканини, в якій колагенові волокна переважають над клітинами. Яка пухлина в
даному випадку?
A. @Десмоїд
B. Щільна фіброма
C. Фібросаркома
D. М'яка фіброма
E. Дерматофіброма
6. У чоловіка, 35 років, в м'яких тканинах лівого стегна з'явилося безболісне новоутворення без чітких
меж. У біоптаті: тканина новоутворення нагадує «риб'яче м'ясо», складається з незрілих фібробластоподібних
клітин з численними мітозами, що проростають у м’язи. Ваш діагноз?
A. Міосаркома
B. Фіброма
C. Рак
D. @Фібросаркома
E. Міома

7. У чоловіка, 40 років, було видалено пухлину, яка росла під шкірою спини. Гістологічний діагноз:
«Ліпома». На яких принципах класифікації пухлин базувався патологоанатом коли давав таке заключення?
A. Макроструктури органа
B. @Гістогенезу
C. Фізико-хімічних особливостей
D. Біохімічних особливостей
E. Ультраструктурних особливостей

8. У чоловіка, 41 року, під шкірою нижньощелепної ділянки виявлено рухоме утворення розмірами
1,0x0,7 см з чіткими межами, тістоподібною консистенцією, яке повільно збільшується. При гістологічному
дослідженні утворення виявлені ліпоцити, які формують різної форми і розмірів часточки, розділені тонкими
прошарками сполучної тканини з судинами. Встановити діагноз.
A. Фіброма
B. @Ліпома
C. Ангіома
D. Ліпосаркома
E. Фібросаркома

9. У жiнки, 65 рокiв, видалено пухлиноподiбне утворення 1,0 х 1,0 х 0,8 см, що росло пiд шкiрою стегна.
Макроскопично: утворення представлене жировою тканиною у капсулi. Мiкроскопiчно: рiзнокалiбернi часточки з
жирових клiтин, що роздiленi прошарками зі сполучної тканини. Назвiть це утворення.
A. Десмоїд
B. Гiбернома
C. Лiпосаркома
D. @Лiпома
E. Фiброма

10. Жінці, 24 років, видалено пухлинне новоутворення. При гістологічному дослідженні виявлені різних
розмірів часточки з жирової тканини. Поставте діагноз.
A. Фіброма
B. Гігрома
C. Папілома
D. @Ліпома
E. Гемангіома

11. При гістологічному дослідженні пухлини шкіри виявлені різних розмірів часточки з жирової тканини,
відмежовані нерегулярними прошарками сполучної тканини. Поставити діагноз.
A. @Лiпома
B. Фiброма
C. Гiгрома
D. Папiлома
E. Гемангiома

12. У хворого, 66 років, з позаочеревинного простору видалено пухлину розмірами 12х6х7 см. При
мікроскопічному дослідженні: анаплазовані жирові клітини з ознаками клітинного атипізму, поліморфізму.
Зустрічаються величезні спотворені клітини, які мають у цитоплазмі жирові крапельки. Який найвірогідніший
діагноз?
A. Ліпома
B. @Ліпосаркома
C. Міосаркома
D. Фібросаркома
E. Мезотеліома

13. У хворого, 32 років, у заочеревинному просторі виявлено пухлиноподібне утворення розмірами 17х15
см з проростанням його в брижу, яке хірург цілком вилучити не зміг. Макроскопічно: тканина утворення на
розрізі жовтувата, м’яка, з осередками некрозу і ослизнення. При мікроскопічному дослідженні виявлено:
клітини з вираженим поліморфізмом ядер, з наявністю патологічних мітозів, цитоплазма клітин - світла,
дрібновакуолізована, при фарбуванні суданом вакуолі дають позитивну реакцію. Визначте пухлину:
A. @Ліпосаркома
B. Ліпома
C. Фібросаркома
D. Фіброма
E. Пбернома

14. Хворий, 50 років, звернувся до лікаря зі скаргами на наявність під шкірою в правій тім'яній ділянці
новоутворення кулястої форми, діаметром до 2 см, щільної консистенції, нерухоме. При гістологічному
дослідженні видаленого новоутворення виявлено: безладне розміщення кісткових балок, між якими
розростається волокниста сполучна тканина. Ваш діагноз?
A. @Губчата остеома
B. Компактна остеома
C. Остеопороз
D. Остеомаляція
E. Остеосаркома

15. У чоловіка, 25 років, через 2 роки після удару ліктя з'явилося пухлинне розростання в області епіфіза
плечової кістки, без чітких меж. У біоптаті: велика кількість поліморфних клітин остеобластичного типу з
великою кількістю патологічних мітозів. Поставте припустимий діагноз.
A. @Остеосаркома
B. Хондросаркома
C. Остеоїд-остеома
D. Фібросаркома
E. Синовіальна саркома

16. У молодої жінки в області дистального кінця стегнової кістки видалено пухлину, яка швидко росла,
пухлина строкатого вигляду – від біло-сірого до коричнево-червоного кольору, пухкої консистенції.
Мікроскопічно: основний тканинний компонент пухлини представлений кістковими і остеоїдними структурами,
вистеленими атиповими остеобластами, з наявністю безлічі тонкостінних судин і фігур патологічних мітозів. Ваш
діагноз?
A. @Остеосаркома
B. Хондрома
C. Остеома
D. Саркома Юїнга
E. Ангіосаркома

17. У хлопчика, 14 років, виявлено збільшення в об'ємі нижньої третини стегна, місцеву гіперемію,
посилення венозного малюнку в зоні патологічного процесу. Рентгенологічно - коровий шар стегнової кістки
зруйнований, структури колінного суглобу збережені. При дослідженні біоптату виявлено скупчення атипових
остеобластів з множинними мітозами та ділянками аномально сформованих кісткових балок, ознаки інвазії у
прилеглі тканини. Діагностуйте захворювання.
A. @Остеогенна саркома
B. Фіброзна дисплазія
C. Остеоїд-остеома
D. Хондросаркома
E. Остеобластокластома

18. У дитини, 14 років, в ділянці діафіза стегнової кістки з'явився болісний м'якуватий вузол, який швидко
росте з кістково-м’язової порожнини з руйнуванням губчастого шару кістки. Мікроскопічно: мономорфні круглі
клітини трохи більше зрілого лімфоцита, зі скудною світлою цитоплазмою, яка містить глікоген. Місцями вони
формують псевдо-розетки з нечисленними мітозами. Між клітинами - фіброзні септи. Встановити діагноз.
A. Лімфома
B. Рабдоміосаркома
C. Нейробластома
D. @Саркома Юїнга
E. Метастаз дрібноклітинного раку

19. У дитини, 6 рокiв, з дiагнозом «Пухлина дiафiза стегнової кiстки» пiд час обстеження вiдмiчено
наявнiсть кiлькох метастатичних відсівів iншої кiсткової локалiзацiї. У гiстологiчному описi вказується, що
первинна пухлина переважно складається з округлих клiтин з бiдною цитоплазмою, якi мають незначну
тенденцiю до утворення псевдорозеток, i проявляють себе поодинокими мiтозами. Вказанi змiни характернi для:
A. Фiбросаркоми
B. @Саркоми Юiнга
C. Хондроми
D. Остеосаркоми
E. Плазмоцитоми
20. При гістологічному дослідженні вузла у центральній ділянці кістки зап'ястка, який поволі
збільшувався впродовж 3 років, у хлопця, 15 років, виявлено: безладно розташовані в хрящових лакунах зрілі
хондроцити без мітозів. Хрящові лакуни різної форми і розмірів в результаті різної кількості в них хондроцитів,
між якими основна речовина з рідкими прошарками сполучної тканини. Який найвірогідніший діагноз?
A. Хондробластома
B. Хондросаркома
C. @Хондрома
D. Тератома
E. Хордома

21. При біопсії вузла з поперечнопосмугованого м'язу знайшли клітини, які нагадують ембріональні м'язи
та міобласти без клітинного атипізму. Ваш діагноз?
A. @Рабдоміома
B. Рабдоміосаркома
C. Гібернома
D. Лейоміома
E. Фіброміома

22. Хворій, 47 років, видалено матку. Під слизовою оболонкою визначаються численні округлої форми
вузли, які чітко відмежовані від навколишньої тканини. Мікроскопічно пухлина побудована з пучків гладкої
мускулатури з явищами тканинного атипізму. Ваш діагноз?
A. @Лейоміома
B. Рак матки
C. Фіброміома
D. Хоріонепітеліома
E. Лейоміосаркома

23. Хворій, 29 років, видалено матку. Під слизовою оболонкою визначаються численні округлої форми
вузли, які не чітко відмежовані від навколишньої тканини. Мікроскопічно: пухлина побудована з різних пучків
гладкої мускулатури з явищами тканинного та клітинного атипізму. Ваш діагноз.
A. @Лейоміосаркома
B. Рак матки
C. Фіброміома
D. Хоріонепітеліома
E. Лейоміома
24. У дитини, 4 років, на шкірі шиї розташований плоский вузол червоного кольору, при натисканні склом
вузол блідне. Який найвірогідніший діагноз?
A. Пігментний невус
B. Меланома
C. @Гемангіома
D. Лейоміома
E. Лімфангіома
25. При мікроскопічному дослідженні пухлини верхньої губи, виявлено, що вона побудована з числених
щілино-подібних порожнин, стінка яких вистелена сплощеним ендотелієм, заповнених рідкою кров`ю і
згортками. Поставити діагноз.
A. @Капілярна гемангіома
B. Венозна гемангіома
C. Кавернозна гемангіома
D. Гемангіоперицитома
E. Гломус-ангіома
26. Гістологічне дослідження пухлини верхньої губи показало, що вона складається з численних судин
капілярного типу, строма пухка. Поставте діагноз.
A. Венозна гемангіома
B. Кавернозна гемангіома
C. Гемангіоперицитома
D. Гломус-ангіома
E. @Капілярна гемангіома

27. У хворого, 17 років, інтраопераційно на нижній поверхні печінки виявлена пухлина розмірами
4,5х5,0х3,5см з субсерозною локалізацією, темночервоного кольору, на розрізі представлена порожнинами
значним вмістом крові. Поставити попередній діагноз.
A. @Кавернозна гемангiома
B. Капiлярна гемангіома
C. Гемангiоперицитома
D. Гемангiоендотелiома
E. Лiмфангiома

28. У хворого з вираженим імунодефіцитом, наявністю в крові лимфопенії із зміною співвідношення Т-


хелперов до Т-супрессоров, відзначено ураження шкіри нижніх кінцівок у вигляді множинних пухлинних
вузликів синюшно-червоного кольору, що зливаються і створюють поверхні виразки. При дослідженні біоптату
шкіри виявлено новоутворення кровоносних судин і розширення капілярів, що створюють порожнині різної
форми і величини. Діагностуйте найвірогідніший характер шкірної патології.
A. Лімфома шкіри
B. Запальний дерматит
C. Базаліома
D. Дерматомікоз
E. @Саркома Капоши

29. У хворого з ВIЛ-iнфекцiєю визначена ураження шкiри нижнiх кiнцiвок у виглядi множинних
пухлинних вузликiв синюшно-червоного кольору, що зливаються i створюють поверхневi виразки. При
дослiдженнi бiоптату шкiри виявлено новоутворення кровоносних судин, що створюють порожнини рiзної форми
i величини та побудоване з ендотелiю, багато пучкiв веретеноподiбних клiтин. Дiагностуйте найiмовiрнiший
характер шкiрної патологiї:
A. @Саркома Капошi
B. Базалiома
C. Дерматомiкоз
D. Лiмфома шкiри
E. Запальний дерматит

30. Жінка 31-го року хворіє на ВІЛ-інфекцію на стадії СНІД. На шкірі нижніх кінцівок, слизової оболонки
піднебіння з’явились рудувато-червоні плями, яскраво-червоні вузлики різних розмірів. Один з вузликів взято на
гістологічне дослідження. Виявлено багато хаотично розташованих тонкостінних судин, вистелених ендотелієм,
пучки веретеноподібних клітин з наявністю гемосидерину. Яка пухлина розвинулась у хворої?
A. @Саркома Капоші
B. Гемангіома
C. Лімфома Беркіта
D. Лімфангіома
E. Фібросаркома

31. У жінки на шкірі обличчя з'явилося пігментне утворення у вигляді вузлика, яке швидко збільшувалося.
Зроблено біопсію. Мікроскопічно в біоптаті визначаються поля веретеноподібних і поліморфних клітин, які
містять бурий пігмент. Дуже багато мітозів. Ваш діагноз
A. Пігментний невус
B. Рак
C. Папілома
D. @Меланома
E. Дерматофіброма

32. У чоловіка, 46 років, на шкірі визначалась темна пляма, яка вибухала та не спричиняла турбот. З часом
пляма почала збільшуватись, з’явився біль, колір став чорно-коричневим і почав пальпуватися вузлик. При
гістологічному дослідженні видаленої тканини визначалися веретеноподібні і поліморфні клітини з численними
мітозами, цитоплазма яких вміщувала пігмент бурого кольору. Про яку пухлину йдеться?
A. @Меланома
B. Базаліома
C. Гемангіома
D. Невус
E. -

33. У жінки, 50 років, на шкірі з'явилася ділянка темного кольору, безболісна, що видається над
поверхнею шкіри. З часом пляма почала збільшуватися, з'явився біль, колір став чорно-коричневим і почав
пальпуватися вузлик. При гістологічному дослідженні видаленої тканини визначалися веретеноподібні і
поліморфні клітини, цитоплазма яких вміщувала пігмент коричневого кольору. Про яку пухлину йдеться?
A. Базаліома
B. @Меланома
C. Гемангіома
D. Гематома
E. Карциноїд

34. На шкірі обличчя у жінки, 60 років, впродовж року з'явилася неправильної форми коричнева бляшка з
чорними вкрапленнями. При гістологічному дослідженні її в епідермісі і всій товщині дерми знайдені поліморфні
великі клітини з патологічними мітозами, з крупними ядерцями, жовто-коричневим пігментом у цитоплазмі
багатьох клітин. Вказані клітини ростуть у вигляді дрібних груп, або поодинці. Встановіть діагноз.
A. Пігментний невус
B. Пігментна папілома
C. @Меланома
D. Пігментна ксеродерма
E. Меланоз шкіри

35. У чоловіка, 52 років, видалено очне яблуко, в судинній оболонці якого виявлено пухлинне утворення
1х0,4 см чорного кольору. Мікроскопічне дослідження показало наявність в клітинах множинних патологічних
мітозів, у цитоплазмі багатьох клітин визначається пігмент жовто-бурого кольору. Ваш діагноз?
A. Неврінома
B. Ангіосаркома
C. @Меланома
D. Нейробластома
E. Гангліонейробластома

36. Для гістологічного дослідження доставлено очне яблуко: в сітківці ока на проекції очного нерва -
пухлина 1х1см, м'якої консистенції, коричневого кольору. Мікроскопічно пухлина складається з поліморфних
клітин з прозорою цитоплазмою, згрупованих в альвеолярні структури, подекуди з накопиченням бурого
пігменту, визначаються патологічні мітози. Достовірніше всього це:
A. Шванома
B. Гломусна пухлина
C. @Меланома
D. Парагангліома
E. Ангіосаркома

37. У літнього чоловіка з ампутованої кукси нижньої кінцівки оперативно видалено пухлину в капсулі,
діаметром 2 см; мікроскопічно вона складається з веретеноподібних клітин мономорфного вигляду з
паличкоподібними ядрами, які утворюють разом з волокнами “палісадні структури”. Який з перерахованих видів
пухлин найбільш вірогідний?
A. @Доброякісна неврилемома
B. Нейрофіброма
C. Злоякісна неврилемома
D. М'яка фіброма
E. Фібросаркома

38. Чоловік, 45 років, повністю втратив слух на праве вухо. Клінічне обстеження не виявило ніяких
патологічних змін у системі правого слухового аналізатора. Томографія головного мозку виявила новоутворення
в ділянці мосто-мозочкового кута діаметром 7 см, гомогенне, без чітких контурів. Назвати новоутворення:
A. @Неврилемома кінця слухового нерва
B. Гангліоневрома
C. Астроцитома
D. Арахноїдендотеліома
E. Нейрофіброма
39. За ходом слухового нерва у молодої жінки виявлено пухлину у вигляді вузла до 3 см в діаметрі, м’яко-
еластичної консистенції, рожево-білого кольору, однорідна. Мікроскопічно пухлина містить пучки клітин з
овальними ядрами. Клітинно-волокнисті пучки формують ритмічні структури, створені паралельними рядами,
правильно орієнтованими клітинами, розташованими у вигляді частоколу, поміж яких знаходиться безклітинна
гомогенна зона (тільця Верокаї). Що це за пухлина?
A. @Невринома
B. Злоякісна невринома
C. Гангліоневрома
D. Нейробластома
E. Гангліонейробластома

40. У хворого, 20 років, видалено пухлину лобної частки правої півкулі головного мозку діаметром 5 см,
яка була нечітко відмежована від оточуючої тканини. На розрізі - однорідного вигляду, гістологічно - складається
з зіркоподібних клітин, численні відростки яких утворюють густі сплетіння. Яка пухлина була у хворого?
A. @Астроцитома
B. Олігодендрогліома
C. Іангліоневрома
D. Епендімома
E. Хоріоїдпапілома

41. У хворого, 35 років, видалено пухлину тім'яної частки правої півкулі головного мозку діаметром 5 см,
яка була нечітко відокремлена від навколишньої тканини. На розрізі - однорідна, гістологічно: складається з
клітин, численні відростки яких утворюють густі сплетіння. Яка пухлина мала місце?
A. @Астроцитома
B. Олігодендрогліома
C. Гангліоневрома
D. Епендімома
E. Хоріоїдпапілома

42. У хворого з швидко наростаючою внутрішньомозковою гіпертензією діагностовано пухлину мозку.


Під час операції видалено пухлину тім’яно-скроневої частки, м’якої консистенції, на розрізі строкатого вигляду.
Гістологічно пухлина побудована з поліморфних гіперхромних клітин з утворенням псевдорозеток та великої
кількості судин, ділянками некрозів і крововиливами. Який найбільш імовірний діагноз?
A. @Гліобластома
B. Олігодендрогліома
C. Арахноїдендотеліома
D. Астроцитома
E. Менінгіома

43 При розтині тіла дитини, 8 років, в мозочку виявлено пухлину, яка не має чітких меж з навколишньою
тканиною, гістологічно побудована з атипових дрібних клітин з гіперхромними ядрами. Достовірніше всього це:
A. Медулосаркома
B. Метастаз раку
C. Метастаз саркоми
D. Гліобластома
E. @Медулобластома

44. У хворого з неврологічними порушеннями діагностовано пухлину головного мозку. Під час операції
видалено пухлину, яка має вигляд щільного вузла, пов'язаного з твердою мозковою оболонкою. Гістологічно
пухлина побудована з ендотеліоподібних клітин, які тісно прилягають одна до одної. Ваш діагноз:
A. Гліобластома
B. @Менінгіома
C. Менінгіальна саркома
D. Астроцитома
E. Нейробластома

ГОСТРІ ЛЕЙКОЗИ
ВАРІАНТ №

1. Під час роботи щодо ліквідації наслідків аварії на АЕС, робітник одержав дозу опромінення 500
рентген. Скаржиться на головний біль, нудоту, запаморочення. Які зміни кількості лейкоцитів можна очікувати в
хворого через 10 годин після опромінення?
A. @Нейтрофільний лейкоцитоз
B. Лімфоцитоз
C. Лейкопенія
D. Агранулоцитоз
E. Лейкемія

2. Ураження хворого одноразовою дозою iонiзуючого випромiнювання спричинило розвиток


кiстковомозкової форми променевої хвороби. Якi патологiчнi прояви з боку кровi будуть характерними в перiод
удаваного благополуччя?
A. @Наростаюча лiмфопенiя, лейкопенiя
B. Перерозподiльчий лейкоцитоз, лiмфоцитоз
C. Анемiя, лейкопенiя
D. Тромбоцитопенiя, анемiя
E. Тромбоцитопенiя, лейкоцитоз

3. При розтині померлого чоловіка, який служив на підводному атомному човні, виявили наступні
патології: спустошення в кістковому мозку (панмієлофтиз), анемію, лейкопенію, тромбоцитопенію, розпад
лімфоцитів у лімфатичних вузлах, селезінці, лімфатичному апараті шлунково-кишкового тракту, крововиливи в
наднирники. Яка хвороба розвинулась у даному випадку?
A. @Гостра променева хвороба
B. Кесонна хвороба
C. Гострий лейкоз
D. Гостра анемія
E. Вібраційна хвороба

4. На розтинi трупа дитини, 4 рокiв, знайденi збiльшенi в розмiрах лiмфатичнi вузли, вилочкова залоза i
селезiнка. Кiстковий мозок трубчастих кiсток соковитий, малинового кольору. У легенях - осередок
бронхопневмонiї. За життя в кровi дитини виявлено 56·109 лейкоцитiв, серед яких 70% лiмфобластiв. Яке основне
захворювання було у дитини?
A. Сепсис
B. @Лейкоз гострий
C. Бронхопневмонiя
D. Лiмфосаркома
E. Лейкоз хронiчний

5. До гематологічного відділення госпіталізовано хворого, 10 років, зі скаргами на підвищення


температури тіла до 390С, петехіальні крововиливи на шкірі. При обстеженні виявлено: гепатоспленомегалія,
дифузна гіперплазія лімфатичних вузлів. В аналізі периферичної крові: анемія, лейкоцитоз 10·10 9/л, еозинофілія,
моно- та тромбоцитопенія, лімфоцити складають 30%. Після цитохімічного дослідження пунктату кісткового
мозку (трепанобіопсія грудини) діагностовано гострий лімфобластний лейкоз. Визначити варіант лейкозу:
A. Лейкемічний
B. Сублейкемічний
C. Лейкопенічний
D. Лейкемоїдна реакція
E. @Алейкемічний

6. До гематологічного відділення госпіталізовано хворого, 16 років, зі скаргами на підвищення


температури тіла до 380С, млявість, блідість та крововиливи у шкіру нижніх кінцівок, біль у кістках. В аналізі
периферичної крові: нормохромна анемія; загальна кількість лейкоцитів 12х109/л, серед яких 70% складали
лімфоцити; по 0,5% мієлоцити та метамієлоцити; 1% паличкоядерні гранулоцити; 3% сегментоядерні; моноцити
та еозинофіли – відсутні; швидкість зсідання еритроцитів 20 мм/год. За результатами мієлограми кісткового
мозку діагностовано гострий лімфобластний лейкоз (розгорнута стадія). Визначити гематологічний варіант
гострого лейкозу:
A. Лейкемічний
B. Сублейкемічний
C. @Алейкемічний
D. Лейкопенічний
E. Лейкемоїдна реакція

7. Хворому, 17 років, який страждає на гострий лімфобластний лейкоз, проводилось лікування


хіміопрепаратами. У черговому аналізі периферичної крові загальна кількість лейкоцитів знизилась до 3,5х10 9/л
та бластів - до 3%. Визначити варіант гострого лейкозу
A. Лейкемічний
B. Сублейкемічний
C. @Лейкопенічний
D. Алейкемічний
E. Апластичний

8. У хворого, 8 років, який страждає на анемію та геморагічний синдром, в аналізі периферичної крові
виявлені: нормохромна анемія, збільшення загальної кількості лейкоцитів до 75х10 9/л, серед яких 90% складали
бласти, поодинокі нормоцити, мієлоцити, промієлоцити, 5% лімфоцитів, 0% моноцитів та еозинофілів,
тромбоцитів 5х109/л. За результатами дослідження пунктату кісткового мозку грудини (мієлограми)
діагностовано гострий лімфобластний лейкоз. Визначити гематологічний варіант гострого лейкозу:
A. @Лейкемічний
B. Сублейкемічний
C. Лейкопенічний
D. Алейкемічний
E. Апластичний

9. У хворого, 35 років, який страждає на гострий мієлобластний лейкоз, в аналізі периферичної крові:
нормохромна анемія, лейкоцитоз з переважанням молодих недиференційованих клітин, мієлобластів; кількість
паличкоядерних та сегментоядерних нейтрофілів, тромбоцитів знижена; перехідні форми гранулопоезу та
еозинофіли відсутні; швидкість зсідання еритроцитів підвищена. Вказати вид гематологічних змін, що
характеризують дане захворювання:
A. @Лейкемічний провал
B. Бластний криз
C. Лейкемоїдна реакція
D. Мієлотоксичний агранулоцитоз
E. Мієлоз

10. До гематологічного відділення госпіталізовано хворого, 5 років, зі скаргами на геморагічний висип на


шкірі нижніх кінцівок, підвищення температури до 37,8 0С, знижений апетит. При огляді: блідість шкіри,
лімфатичні вузли 10х15 мм, край печінки виступає на 2 см нижче краю правої реберної дуги, селезінка на рівні
пупка. В аналізі периферичної крові: нормохромна анемія, лейкоцитоз 11,2·10 9/л, бласти 49%, метамієлоцити 1%,
лімфоцити 34%, моноцити 2%, плазмоцити 1%, нейтрофіли паличкоядерні 4%, сегментоядерні 9%, швидкість
зсідання еритроцитів 10 мм/год, анізо- та пойкілоцитоз. У мієлограмі - бласти 84%. За результатами цитохімічних
досліджень діагностовано гострий лімфобластний лейкоз. Визначені гематологічні зміни характерні для даної
форми лейкозу:
A. @Лейкемічний провал
B. Бластний криз
C. Лейкемоїдна реакція
D. Псевдобластна реакція
E. Промієлоцитарна реакція

11. Хворий, 3 років, помер від сепсису. На розтині в кістках обличчя виявлені два щільні утворення
розмірами 2х3 см, на розрізі зеленуватого відтінку. При гістологічному дослідженні спостерігається деструкція
кісткової тканини з проліферацією атипових недиференційованих кровотворних клітин. Ваш діагноз:
A. @Хлоролейкоз
B. Остеомієліт
C. Мієломна хвороба
D. Хвороба Педжета кісток черепа
E. Плазмобластний лейкоз

12. Хворий, 2 років, страждав на лейкоз і помер від крововиливу у головний мозок, на розтині виявлено
незначну гепатоспленомегалію. В кістці нижньої щелепи - множинні пухлинні вузли зеленуватого відтінку, що
мікроскопічно складались з атипових недиференційованих кровотворних клітин мієлоїдного ростка. Поставити
діагноз:
A. @Хлоролейкоз
B. Гострий мієлобластний лейкоз
C. Мієломна хвороба
D. Гострий недиференційований лейкоз
E. Хронічний мієлолейкоз

13. У хворого, 5 років, який відвідував дошкільний дитячий заклад, поступово погіршився апетит,
знизилась рухова активність на дитячому майданчику, впродовж останнього місяця шкіра нижніх кінцівок стала
блідою, з’явились геморагічні висипання. Після обстеження в гематологічному відділенні в пунктаті кісткового
мозку: бластні клітини перевищували 30%. За цитохімічними маркерами (PAS-реакція на глікоген позитивна,
реакції на глікозаміноглікани, ліпіди та ферменти - негативні) встановлено гострий лейкоз. Визначити форму
лейкозу:
A. Недиференційований
B. Мієлобластний
C. @Лімфобластний
D. Хлоролейкоз
E. Монобластний

14. Хворий, 8 років, з вираженим геморагічним синдромом помер від двобічної бронхопневмонії. На
розтині: кістковий мозок та селезінка соковиті, червоні. Вилочкова залоза, лімфатичні вузли середостіння та
брижі збільшені за розмірами, на розрізі соковиті, біло-рожевого кольору. Біла речовина головного та спинного
мозку, його оболонки інфільтровані лейкозними клітинами (нейролейкоз). В аналізі крові: анемія, різке
збільшення числа бластів та поодинокі зрілі елементи крові (hiatus leucemicus). Визначити форму гострого
лейкозу:
A. Мієлобластний
B. @Лімфобластний
C. Монобластний
D. Плазмобластний
E. Еритромієлобластний

15. Хворому, 10 років, з підозрою на пневмонію проведено рентгенологічне дослідження органів грудної
порожнини, за результатами якого встановлено значне збільшення прикореневих лімфатичних вузлів та
вилочкової залози, що утворювали конгломерат. В аналізі периферичної крові: нормохромна анемія, лейкоцитоз
до 150х109/л. гранулоцитопенія, лімфоцитопенія, тромбоцитопенія, 87% складали бласти, в яких виявлено
гранули глікогену (позитивна PAS-реакція). Реакції на ферменти негативні. Ваш діагноз:
A. Недиференційований гострий лейкоз
B. @Лімфобластний лейкоз
C. Мієлобластний лейкоз
D. Еритромієлобластний лейкоз
E. Монобластний лейкоз

16. Хворий на лейкоз зі злоякісним перебігом, 47 років, помер від масивного крововиливу у головний
мозок. У мієлограмі - проліферація незрілих кровотворних клітин з ядерним та клітинним поліморфізмом, з
гранулами глікозаміногліканів, глікогену та ліпідів у цитоплазмі, проміжні форми гранулоцитів відсутні, зрілі
форми поодинокі (лейкемічний провал). Визначити форму гострого лейкозу:
A. Недиференційований
B. Мієлобластний
C. Лімфобластний
D. @Промієлоцитарний
E. Монобластний

17. У хворого, 5 років, перебіг гострого лейкозу супроводжувався вираженою тромбоцитопенією,


гіпофібриногенемією, частими геморагіями. Пацієнт помер від крововиливу у головний мозок. При
гістологічному дослідженні внутрішніх органів: лейкозні клітини інфільтратів характеризуються поліморфізмом,
наявністю у цитоплазмі гранул глікозаміногліканів, глікогену та ліпідів. Позитивні реакції на пероксидазу, кислу
фосфатазу та хлорацетатестеразу, реакція на α-нафтилестеразу негативна. Визначити форму гострого лейкозу:
A. Недиференційований
B. Лімфобластний
C. @Промієлоцитарний
D. Плазмобластний
E. Мієлобластний

18. Хворий, 21 років, помер від ускладнень гострого лейкозу. На розтині: печінка збільшена в 1,5 рази, на
поверхні розрізу виявляються прошарки сіруватого кольору. При гістологічному та гістохімічному дослідженнях
вздовж судин портального тракту та синусоїдів - розростання пухлинних кровотворних клітин, в цитоплазмі яких
виявлено глікоген, суданофільні включення та позитивні реакції на пероксидазу, α-нафтилестеразу та
хлорацетатестеразу. Визначити форму гострого лейкозу:
A. Недиференційований
B. @Мієлобластний
C. Промієлоцитарний
D. Лімфобластний
E. Еритромієлобластний
19. Хворий, 20 років, помер від лейкозного менінгіту. На розтині кістковий мозок трубчастих та плоских
кісток сірий, з зеленуватим відтінком. При гістологічному дослідженні внутрішніх органів (печінка, селезінка,
нирки та оболонки головного мозку) виявлені лейкемічні інфільтрати, більшість клітин яких містять глікоген та
суданофільні включення, дають позитивну реакцію на пероксидазу, кислу фосфатазу, α-нафтилестеразу та
хлорацетатестеразу. Визначити форму гострого лейкозу:
A. Недиференційований
B. @Мієлобластний
C. Лімфобластний
D. Плазмобластний
E. Монобластний

20. При диспансерному обстеженні хворого, 43 років, який у минулому лікувався від
лімфогрануломатозу, виявлено гепатоспленомегалію. В аналізі периферичної крові: гіперхромна анемія,
абсолютна кількість лейкоцитів 9х109/л, кількість еритроцитів та тромбоцитів знижена, зустрічаються
еритрокаріоцити, атипові еритробласти та мієлобласти. Форма лейкозу, для якої характерні дані зміни:
A. Гострий мієлобластний
B. @Гострий еритромієлобластний
C. Хронічний мієлолейкоз
D. Хронічний еритромієлоз
E. Еритремія

21. Хворий, 6 років, який страждав на гострий лейкоз, помер від масивного крововиливу у головний
мозок. На розтині виявлені характерні зміни: кістковий мозок червоний, соковитий, селезінка та печінка
збільшені у розмірах, у правій легені – осередкова пневмонія, на серозних та слизових оболонках - геморагічний
висип дифузного характеру, печінка, нирки та серце - анемічні, некротичний гінгівіт та тонзиліт. При
цитохімічному аналізі кісткового мозку реакції на глікоген, ліпіди, глікозаміноглікани та ферменти негативні.
Визначити форму гострого лейкозу:
A. @Недиференційований
B. Мієлобластний
C. Лімфобластний
D. Хлоролейкоз
E. Монобластний

22. Хворий на лейкоз, 10 років, помер від сепсису. На розтині виявлені різноманітні крововиливи у
внутрішні органи, шкіру, серозні та слизові оболонки. Печінка та селезінка збільшені у 1,5 рази, кістковий мозок
трубчастих та плоских кісток на розрізі червоний, соковитий. У шлунці та кишечнику - множинні виразки,
гіперплазія групових лімфатичних та солітарних фолікулів. При цитохімічному дослідженні кістковий мозок
складається з однорідних кровотворних клітин, в яких реакції на глікоген, глікозаміноглікани, ліпіди та ферменти
негативні. Визначити форму лейкозу:
A. @Недиференційований
B. Лімфобластний
C. Монобластний
D. Плазмобластний
E. Мієлобластний

23. Хвора, 14 років, знаходилась на диспансерному обліку з приводу гострого лімфобластного лейкозу,
була госпіталізована зі скаргами на підвищення температури до 39,5 0С, головний біль, нудоту, блювання, судоми.
Проведено діагностичну пункцію спинномозкового каналу. У спинномозковій рідині виявлено значне
підвищення вмісту білка з переважанням глобулінової фракції (позитивна реакція Панді), загальна кількість
лейкоцитів 6х109/л, з них бласти складали 80%. Визначити ускладнення лейкозу:
A. @Лейкозний менінгіт
B. Гнійний менінгіт
C. Серозний менінгіт
D. Абсцеси головного мозку
E. Крововилив у головний мозок

24. У хворого, 43 років, на фоні гострого мієлолейкозу з’явились менінгеальні симптоми (головний біль,
нудота, блювання). За результатами цитологічного дослідження спинномозкової рідини діагностовано лейкозний
менінгіт. Патоморфологічні зміни нервової системи, що характеризують нейролейкоз:
A. @Дифузна інфільтрація оболонок головного мозку мієлобластами
B. Дифузна інфільтрація оболонок мозку нейтрофілами
C. Емболія судин головного мозку мієлобластами
D. Субарахноїдальні крововиливи
E. Абсцеси головного мозку

25. У хворого, 12 років, на гострий лейкоз підвищилась температура тіла до 39,8 0С, з’явився гострий біль
у горлі. При огляді ротової порожнини виявлено: набряк мигдаликів, на їх поверхні - глибокі дефекти з
нерівними краями, множинні петехіальні крововиливи у слизовому шарі зіва та навколо мигдаликів. Визначити
вид ангіни, що ускладнила перебіг захворювання:
A. Катаральна
B. Фібринозна
C. Гнійна
D. @Некротична
E. Гангренозна

26. Хворий, 38 років, через рік після лікування з приводу гострого мієлобластного лейкозу звернувся до
лікаря зі скаргами на підвищення температури тіла до 38,9 0С, біль у горлі, нудоту. При огляді ротової порожнини
діагностовано ангіну. Вказати форму ангіни, типову для даного захворювання:
A. Катаральна
B. Флегмонозна
C. Фолікулярна
D. @Фібринозно-некротична
E. Хронічна

27. Новонароджена дитина з вродженою формою лейкозу померла на 2 тижні при явищах прогресуючої
анемії та геморагічного синдрому. На розтині – шкіра бліда, суха, з множинними плямистими крововиливами
діаметром 1-3 см, печінка, селезінка та лімфтичні вузли збільшені. При гістологічному дослідженні виявлено
дифузну лейкемічну інфільтрацію внутрішніх органів. Вказати локалізацію лейкемічних інфільтратів:
A. Головний мозок
B. Печінка
C. Шлунково-кишковий тракт
D. Легені
E. @Шкіра
ХРОНІЧНІ ЛЕЙКОЗИ

1. При розтині померлого хворого виявлені: гіперплазія кісткового мозку плоских і трубчастих кісток
(піоїдний кістковий мозок), спленомегалія (6 кг), гепатомегалія (5 кг), збільшення всіх груп лімфатичних вузлів.
Якому захворюванню відповідають виявлені зміни?
A. Мієломна хвороба
B. Хронічний лімфолейкоз
C. @Хронічний мієлолейкоз
D. Справжня поліцитемія
E. Лімфогранулематоз

2. При розтинi трупа чоловiка, 48 рокiв, виявлено, що кiстковий мозок пласких кiсток, дiафiзiв та епiфiзiв
трубчастих кiсток соковитий, сiро-червоний або сiро-жовтий гноєподiбний (пiоїдний кiстковий мозок). Селезiнка
масою - 7 кг. На розрiзi вона темно-червоного кольору, з iшемiчними iнфарктами. Всi лiмфатичнi вузли
збiльшенi, м’якi, сiро-червоного кольору. В печiнцi - жирова дистрофiя i лейкемiчнi iнфiльтрати. Який найбiльш
iмовiрний дiагноз?
A. @Хронiчний мiєлоїдний лейкоз
B. Мiєломна хвороба
C. Гострий мiєлоїдний лейкоз
D. Гострий лiмфоїдний лейкоз
E. Лiмфогранулематоз

3. На розтині померлого чоловіка, 30 років, виявлені збільшені: селезінка (900 г), печінка (4000 г),
лімфатичні вузли; кістковий мозок діафіза стегнової кістки соковитий, малиново-червоного кольору.
Мікроскопічно в печінці, переважно за ходом портальних трактів, розміщені щільні інфільтрати, що складаються
з незрілих кровотворних клітин з ядром округлої форми і вузькою облямівкою цитоплазми. Яке це
захворювання?
A. Хронічний мієлоїдний лейкоз
B. @Хронічний лімфолейкоз
C. Генералізована форма лімфогранулематозу
D. Гострий мієлобластний лейкоз
E. Гострий лімфобластний лейкоз

4. При мікроскопічному дослідженні тканини збільшеного шийного лімфатичного вузла визначається


стертість його структури, лімфоїдні фолікули відсутні, всі поля зору представлені клітинами з округлими ядрами
і вузькою облямівкою базофільної цитоплазми. З клінічних даних відомо, що збільшені й інші групи лімфовузлів,
а також селезінка та печінка. Про яке захворювання слід думати?
А. Мієломна хвороба
B. Лімфогранулематоз
C. Лімфома
D. Мієлоїдний лейкоз
E.@ Лімфоїдний лейкоз
5. При патогістологічному дослідженні біоптату печінки чоловіка, 70 років, у якого в крові - велика
кількість лімфоцитів і пролімфоцитів, виявлені: множинні скупчення зазначенних клітин переважно проміж
печінковими часточками. Для якого захворювання характерні ці зміни?
A. Лімфогранулематоз
B. Гострий лімфолейкоз
C. @Хронічний лімфолейкоз
D. Хронічний персистуючий гепатит
E. Гепатоцелюлярний рак печінки

6. У хворого в обох щелепах рентгенологічно виявлені числені дефекти у вигляді гладкостінних округлих
отворів. При гістологічному дослідженні – явища остеолізису і остеопорозу, слабке кісткоутворення. В сечі
хворого виявлено білок Бенс-Джонса. Назвіть захворювання.
A. Гострий мієлолейкоз
B. @Мієломна хвороба
C. Хронічний еритромієлоз
D. Хронічний мієлолейкоз
E. Гострий недиференційований лейкоз

7. У хворого в обох щелепах рентгенологічно виявлено числені дефекти у вигляді гладкостінних округлих
отворів. При гістологічному дослідженні: явища остеолізису і остеопорозу, слабке кісткоутворення. В сечі
хворого виявлено парапротеїн подібний до імуноглобуліну Е. Назвіть захворювання.
A. Гострий мієлолейкоз
B. Хронічний мієлолейкоз
C. Хронічний еритромієлоз
D. @Мієломна хвороба
E. Гострий недиференційований лейкоз

8. У хворого при рентгенологічному обстеженні в плоских кістках виявлені множинні осередки


остеопорозу та остеолізису. У трепанобіоптаті виявлено велику кількість пухлинних плазматичних клітин. Ваш
діагноз.
A. Гістіоцитоз
B. Гострий моноцитарний лейкоз
C. Хронічний мієлолейкоз
D. Лімфогранулематоз
E. @Мієломна хвороба

9. У жінки, 64 років, виник «патологічний» перелом плечової кістки. Результат біопсії: атипові
плазматичні клітини. Рентгенологічно у місці перелому - пухлиноподібні утворення. Можливе захворювання:
А. Хондросаркома
B. Хронічний остеомієліт.
C. @Мієломна хвороба
Д. Фіброзна дисплазія кістки.
E. Метастаз аденокарциноми.
ЛІМФОМИ
ВАРІАНТ №

1. У хворого на шкірі відзначені інфільтративно-бляшкові поліморфні висипання, різних контурів,


розмірів, застійно-синюшного кольору, схильні до периферичного зростання і злиття. При біопсійному
дослідженні шкіри виявлені масивні проліферати з лімфоїдних клітин, які займають всю дерму і
розповсюджуються в підшкірний жировий шар. Ваш діагноз.
A. @Лімфома шкіри
B. Грибовидний мікоз
C. Внутрішньодермальний невус
D. Шкірні прояви системного червоного вовчака
E. Флегмона підшкірної клітковини

2. Під час огляду стоматологом хлопчика, 16 років, були виявлені збільшені підщелепні та шийні
лімфатичні вузли. Було проведено біопсію. Мікроскопічно було знайдено: типова будова лімфатичних вузлів
стерта, клітинна популяція гетерогенна, наявні великі клітини з багатолопатевим ядром, множинні одноядерні
клітини великого розміру, еозинофільні та нейтрофільні лейкоцити, лімфоцити; осередки некрозу та склерозу.
Діагностуйте захворювання.
A. Гнійний лімфаденіт
B. Гіперплапзія лімфатичного вузла
C. Гранульоматозний лімфаденіт
D. @Лімфома Ходжкіна
E. Неходжкінська лімфома

3. У хворого на шиї виявлено пакет спаяних між собою лімфовузлів щільної консистенції. При
гістологічному обстеженні видаленого лімфовузла відмічається проліферація ретикулярних клітин, наявність
клітин Березовського-Штернберга. Про яке захворювання йдеться?
A. Лімфагранулематоз
B. Лімфобластний лейкоз
C. Мієлобластний лейкоз
D. Мієлоцитарний лейкоз
E. Лімфоцитарний лейкоз

4. У молодої чоловіка збільшилися шийні лімфатичні вузли. При біопсії в тканині лімфовузла виявлені
осередки проліферації лімфоїдної тканини, поміж якими розміщені велетенські клітини Рід-Березовського-
Штернберга, еозинофіли, осередки некрозу та склерозу. Яке захворювання найбільш вірогідно буде у даному
випадку?
A. Мієломна хвороба
B. Хронічний лімфолейкоз
C. @Лімфома Ходжкіна
D. Лімфома Беркіта
E. Гістіоцитоз
5. У молодої людини збільшилися шийні лімфатичні вузли. При біопсії в лімфовузлі виявлені осередки
проліферації лімфоїдної тканини з наявністю велетенських клітин Березовського-Штернберга і еозинофілів,
ділянки некрозу та фіброзу. Яке захворювання найбільш вірогідно буде діагностоване у хворого?
A. @Лімфагранулематоз
B. Хронічний лімфолейкоз
C. Мієломна хвороба
D. Лімфома Беркіта
E. Гістіоцитоз

6. У збільшеному шийному лімфатичному вузлі дівчинки, 14 років, мікроскопічно було виявлено


порушену будову тканини вузла: лімфоїдні фолікули відсутні, наявні осередки склерозу та казеозного некрозу;
клітинний склад поліморфний: наявні лімфоцити, еозинофіли, атипові клітини великих розмірів з
багатолопатевими ядрами (клітини Рід-Березовського-Штернберга), одноядерні великі та малі атипові клітини.
Ваш діагноз.
A. Хронічний лімфолейкоз
B. Гострий лімфолейкоз
C. @Лімфома Ходжкіна
D. Лімфома Беркіта
E. Грибовидний мікоз

7. У жінки, 22 років, виявлені збільшені лімфатичні вузли. При гістологічному дослідженні в тканині
лімфатичного вузла виявлені лімфоцити, гістіоцити, ретикулярні клітини, малі та великі клітини Ходжкіна,
багатоядерні клітини Рід-Березовського-Штернберга, поодинокі осередки казеозного некрозу. Для якого
захворювання характерні такі зміни?
A. Лімфома Беркіта
B. @Лімфома Ходжкіна
C. Хронічний лейкоз
D. Гострий лейкоз
E. Метастаз раку легень

8. Під час розтину померлого, 35 років, поряд зі збільшенням багатьох лімфатичних вузлів було виявлено
збільшену селезінку (600 г), що на розрізі виглядала неоднорідною, темно-червоного кольору, щільною, з
осередками некрозу сірувато-жовтуватого кольору діаметром до 1 см (порфірова селезінка). Про яке
захворювання можна думати?
А. Лімфома Беркіта
В. Хронічний лімфоїдний лейкоз
С. Хронічний мієлоїдний лейкоз
D. @Лімфома Ходжкіна
Е. Метастази раку

9. На розтині померлого знайдено системне збільшення лімфатичних вузлів з утворенням пухлинних


конгломератів. Селезінка збільшена, на розрізі строката: на червоному тлі пульпи - множинні дрібні осередки
жовтувато-білого та сіруватого кольору. Якому захворюванню найбільше відповідають описані зміни?
A. @Лімфагранулематоз
B. Саркоїдоз
C. Лімфома Беркіта
D. Рак легень
E. Лімфолейкоз

10. На розтині померлого хворого виявлені збільшені, спаяні між собою, щільні, з біло-жовтими
осередками некрозу лімфатичні вузли різної локалізації (шийні, пахвові та медіастинальні). Селезінка мала
"порфіровий" вигляд. При мікроскопічному дослідженні тканини лімфатичних вузлів і селезінки виявлено
проліферацію поліморфних клітин, атипові клітини, поміж якими розміщені багатоядерні гігантські клітини Рід-
Березовського-Штернберга. Ваш діагноз.
А. @Лімфома Ходжкіна
B. Хронічний лімфолейкоз
C. Саркоїдоз
D. Ретикулосаркома
E. Грибоподібний мікоз

11. На розтині померлого виявлене системне збільшення лімфатичних вузлів з утворенням пухлинних
конгломератів. Селезінка збільшена, на розрізі строката: на червоному тлі пульпи - множинні дрібні осередки
жовтувато-білого та сіруватого кольору. Якому захворюванню найбільш притаманні описані зміни?
A. Лімфома Беркіта
B. Саркоїдоз
C. @Лімфома Ходжкіна
D. Рак легень
E. Лімфолейкоз

12. При гістологічному дослідженні тканини лімфатичного вузла виявлені: дифузне розростання
лімфоїдних клітин, еозинофіли, атипові гістіоцити, поодинокі гігантські клітини з двома та більше ядрами,
осередки некрозу та склерозу. Найбільш вірогідний діагноз:
A. Саркоїдоз
B. @Лімфома Ходжкіна
C. Лімфома Беркіта
D. Лімфолейкоз
E. Мієлолейкоз

13. При гістологічному дослідженні тканини лімфовузла хворого, 18 років, з ділянки заднього трикутника
шиї, морфолог виявив скупчення клітин, серед яких: поодинокі багатоядерні клітини Рід-Березовського-
Штернберга, великі та малі клітини Ходжкіна і багато лімфоцитів, поодинокі плазматичні клітини, еозинофіли.
Яке захворювання спостерігається у даному випадку?
А. Пухлина Беркіта
B. Нодулярна лімфома
C. @Лімфома Ходжкіна
D. Лімфоцитарна лімфома
Е. Хронічний лімфоїдний лейкоз

14. У 23-рiчного чоловiка впродовж трьох-чотирьох мiсяцiв вiдзначається збiльшення шийних


лiмфатичних лiмфовузлiв вздовж заднього краю грудинно-ключично-сосцеподібного м'яза. Проведено
дiагностичну бiопсiю одного з цих лiмфовузлiв. При гiстологiчному дослiдженнi виявленi типовi ознаки хвороби
Ходжкiна, змiшано-клiтинного варiанту. Виберiть, на пiдставi якої морфологiчної ознаки став можливим такий
дiагноз:
A. У складi пухлини є множиннi еозинофiльнi лейкоцити
B. @Пухлина мiстить поодинокi клiтини Березовського-Штернберга
C. Серед цих клiтин спостерiгаються великi i малi клiтини Ходжкiна
D. Звичайна будова лiмфовузла вiдсутня
E. Пухлина складається з виражено полiморфних клiтин

15. При дослідженні біоптату лімфатичного вузла було діагностовано лімфому Ходжкіна. На підставі
наявності яких ознак був поставлений діагноз?
A. Клітин Вірхова
B. Клітин Тутона
C. Клітин Пирогова-Лангханса
D. @Клітин Рід-Штернберга
E. Клітин Мікулича
16. При гістологічному дослідженні збільшеного шийного лімфатичного вузла відмічено, що загальний
рисунок вузла нечіткий за рахунок розростання атипових гістіоцитарних клітин, з наявністю гігантських клітин
Березовського-Штернберга, наявні ділянки некрозу, склерозу, гіалінозу. Для якого захворювання характерні
виявлені морфологічні зміни в лімфатичному вузлі?
A. @Лімфагранулематозу
B. Гострого мієлолейкозу
C. Хронічного мієлолейкозу
D. Грибовидного мікозу
E. Туберкульозу

17. При дослідженні біоптату збільшеного шийного лімфатичного вузла було виявлено: стертість
малюнка; тканина лімфатичного вузла представлена великою кількістю проліферуючих лімфоцитів з
поодинокими клітинами Рід-Березовського-Штернберга. Ці зміни свідчать про:
A. Лімфома Ходжкіна, змішано-клітинний варіант
B. Лімфома Ходжкіна, нодулярний склероз
C. @Лімфома Ходжкіна, з переважанням лімфоїдної тканини
D. Лімфома Беркіта
E. Лімфома Ходжкіна, з пригніченням лімфоїдної тканини
18. У лiмфовузлi, направленому для гiстологiчного дослiдження, виявлено розростання фiброзної
тканини, яка оточує осередки полiморфних клiтин, серед яких велика кiлькiсть гiгантських багатоядерних,
окремих гiгантських одноядерних i лiмфоцити. Дiагностувати процес?
A. Склерома
B. Туберкульоз
C. @Лiмфогранулематоз (нодулярний склероз)
D. Токсоплазмоз
E. Хронiчний мiєлолейкоз
19. При томографії у хворого в середостінні знайдені збільшені лімфатичні вузли. При гістологічному
дослідженні в лімфовузлі виявлені циркулярні розростання сполучної тканини, яка оточувала гранульомоподібні
утворення з лімфоцитів, плазмоцитів і великих двоядерних клітин Березовського-Штернберга. Ваш діагноз?
A. @Лімфагранулематоз
B. Лімфома Беркіта
C. Туберкульоз
D. Саркоїдоз
E. Лімфолейкоз

20. При томографії у хворого в середостінні виявлені збільшені лімфатичні вузли. При гістологічному
дослідженні в тканині лімфовузла виявлені циркулярні розростання сполучної тканини, яка оточувала
гранульомоподібні утворення з лімфоцитів, плазмоцитів і великих двоядерних клітин. Якому варіанту
захворювання відповідають виявлені зміни?
A. Лімфома Ходжкіна, змішано-клітинний варіант
B. @Лімфома Ходжкіна, нодулярний склероз
C. Лімфома Ходжкіна, з переважанням лімфоїдної тканини
D. Лімфома Беркіта
E. Лімфома Ходжкіна, з пригніченим лімфоїдної тканини

21. У хворого, 60 років, при торакотомії в передньому середостінні знайдені збільшені в розмірах та
спаяні між собою лімфатичні вузли, з яких взято біоптат. Мікроскопічно виявлено атипові клітини, серед яких
переважають клітини Ходжкіна і гігантські клітини Рід-Березовського-Штернберга, присутні лімфоцити,
еозинофіли; склероз відсутній. Яке захворювання найбільш відповідає цим ознакам?
A. @Змішано-клітинний варіант Лімфагранулематозу
B. Лімфагранулематоз з переважанням нодулярного склерозу
C. Лімфагранулематоз з переважанням лімфоїдної тканини
D. Лімфома Беркіта
E. Лімфагранулематоз з пригніченим розвитком лімфоїдної тканини
22. На секції тіла дівчинки, 15 років, було знайдено: шийні, медіастинальні та мезентеріальні лімфатичні
вузли збільшені в розмірах, об’єднані в конгломерати, на розрізі тканина вузлів неоднорідна, містить ділянки
некрозу. Мікроскопово: будова лімфатичних вузлів стерта, зустрічаються ділянки склерозу та осередки некрозу,
клітинна популяція неоднорідна - включає одноядерні атипові клітини, великі атипові клітини з
багаточасточковими ядрами, значний домішок еозинофілів та нейтрофілів, кількість лімфоцитів незначна.
Діагностуйте вид лімфоми Ходжкіна – лімфагранулематозу (ЛГМ).
A. ЛГМ, варіант з пригніченням лімфоїної тканини
B. @ЛГМ, змішано-клітинний варіант
C. ЛГМ, нодулярний склероз
D. ЛГМ, лімфогістіоцитарний варіант
E. Саркома Ходжкіна

23. У хворого, 60 років, при торакотомії в передньому середостінні знайдені збільшені та спаяні між
собою лімфатичні вузли, з яких взято біоптат. Мікроскопічно: атипові клітини, серед яких переважали клітини
Ходжкіна і гігантські клітини Рід-Березовського-Штернберга; склероз відсутній. Яке захворювання найбільш
відповідає цим ознакам ?
A. Лімфома Ходжкіна, змішано-клітинний варіант
B. Лімфома Ходжкіна, нодулярний склероз
C. Лімфома Ходжкіна, з переважанням лімфоїдної тканини
D. Лімфома Беркіта
E. @Лімфома Ходжкіна, з пригніченим лімфоїдної тканини

АТЕРОСКЛЕРОЗ.
ГІПЕРТОНІЧНА ХВОРОБА.
ВАРІАНТ №

1. На розтині тіла померлого від серцевої недостатності, в інтимі аорти і коронарних судин виявлені:
жовтого кольору плями і смуги; сірувато-жовтуваті бляшки, що здіймаються над поверхнею інтими, а також
осередкове виразкування даних бляшок з наявністю крововиливів, тромбозів та осередків звапніння. Для якого
захворювання характерні вказані зміни судин?
A. Системний васкуліт
B. Вузликовий периартеріїт
C. Сифілітичний мезаортит
D. Гіпертонічна хвороба
E. @Атеросклероз

2. При розтині померлого, 78 років, заочеревинна клітковина просочена кров’ю, у черевному відділі аорти
виявлено мішкоподібне випинання, в якому визначається дефект з нерівним краєм, стінка аорти місцями
кам’янистої щільності. Ускладнення якого захворювання розвинулось у хворого?
A. @Атеросклерозу
B. Гіпертонічної хвороби
C. Системного васкуліту
D. Вісцерального сифілісу
E. Хвороби Морфана

3. При гістологічному дослідженні стінки аорти виявлено велику кількість ксантомних клітин, які
розташовані, переважно, в інтимі. Для якого захворювання можлива така картина?
A. @Атеросклероз
B. Гіпертонічна хвороба
C. Сифілітичний мезаортит
D. Неспецифічний аортоартеріїт
E. Нодозний периартеріїт

4. Під час мікроскопічного дослідження аорти виявлено: осередки інфільтрації інтими ліпідами, білками.
Ліпіди просочують інтиму та накопичуються у міоцитах середнього шару та макрофагах. Встановіть стадію
атеросклерозу.
A. @Ліпоїдоз
B. Ліпосклероз
C. Атероматоз
D. Атерокальциноз
E. Доліпідна

5. На аутопсії трупа чоловіка, 39 років, який раптово помер, в інтимі черевного відділу аорти виявляються
ділянки жовтуватого кольору, які не підвищуються над її поверхнею. Гістологічно – в цих ділянках серед
гладком’язових клітин і макрофагів наявні клітини з пінистою цитоплазмою. При забарвленні суданом ІІІ ці
клітини мають яскраве жовтогаряче забарвлення. Для якої стадії атеросклерозу це характерно?
A. @Ліпоїдозу
B. Ліпосклерозу
C. Атероматозу
D. Виразкування
E. Атерокальцинозу

6. При розтині трупа чоловіка, 47 років, померлого раптово, в інтимі черевного відділу аорти знайдені
осередки жовтого кольору у вигляді плям та смуг, що не вибухають над поверхнею інтими. При фарбуванні
суданом ІУ спостерігається чорне забарвлення. Для якої стадії атеросклерозу характерні такі зміни?
A. @Ліпоїдозу
B. Ліпосклерозу
C. Атероматозу
D. Атерокальцинозу
E. Стадія утворення атероматозної виразки
7. Під час розтину трупа жінки, 69 років, підвищеної вгодованості, яка померла від гострого інфаркту
міокарда, в інтимі коронарних артерій виявлені численні білуваті, щільні формування, що вип’ячуються різко
звужуючи просвіт судин. Для якої стадії атеросклерозу властиві такі зміни?
A. @Ліпосклероз
B. Утворення атероматозної виразки
C. Ліпоїдоз
D. Атероматоз
E. Атерокальциноз

8. Мікроскопічне дослідження коронарної артерії у померлого, 53 років, виявило звуження просвіту


судини за рахунок фіброзної бляшки з домішкою ліпідів. Назвіть стадію атеросклерозу:
A. Ліпоїдоз
B. @Ліпосклероз
C. Доліпідна
D. Атероматоз
E. Виразкування

9. При розтині тіла чоловіка, 70 років, який помер від серцево-судинної недостатності, а за життя
страждав на стенокардію, гіперхолестеринемію та ожиріння, виявлені: хронічне венозне повнокров’я органів,
гіпертрофія лівого шлуночка серця з дрібноосередковим кардіосклерозом, об'ємні жовтувато-білуваті бляшки в
інтимі аорти з дрібнозернистими масами у центрі, які заглиблюються у товщу стінки. Яку стадію атеросклерозу
виявлено на аутопсії?
A. Артеріолосклероз
B. Ліпоїдоз
C. Ліпосклероз
D. @Атероматоз
E. Кальциноз

10. На розтині тіла чоловіка, 74 років, який помер від серцевої недостатності, виявлені деформовані,
звужені коронарні артерії. На розрізі внутрішня поверхня артерій горбиста, стінка білувата, ламка, кам'янистої
щільності. Про яку стадію атеросклерозу йдеться?
A. Атероматоз
B. Ліпосклероз
C. Виразкування
D. Ліпоїдоз
E. @Атерокальциноз

11. Під час розтину трупа чоловіка, 70-ти років, який тривало хворів на атеросклероз і помер при явищах
серцево-судинної недостатності, в черевному відділі аорти при макроскопічному дослідженні спостерігаються
щільні, овальної форми фіброзні бляшки з відкладенням вапна у вигляді щільних, ламких пластин. Вкажіть
стадію морфогенезу атеросклерозу:
A. Атероматоз
B. @Атерокальциноз
C. Ліпоїдоз
D. Ліпосклероз
E. Виразкування

12. На розтині тіла померлого від серцевої недостатності, в інтимі аорти і коронарних судин виявлені
жовтого кольору плями і смуги, сірувато-жовтуваті бляшки, що здіймаються над поверхнею інтими, а також
осередкове виразкування даних бляшок з наявністю крововиливів, тромбів і осередків звапнування. Для якої
стадії атеросклерозу характерні вказані зміни судин?
A. Доліпідна
B. Ліпоїдоз
C. Ліпосклероз
D. Атерокальциноз
E. @Ускладнених уражень

13. Під час дослідження коронарних артерій виявлені атеросклеротичні бляшки з кальцинозом, що
закривають просвіт на 1/3. У м'язі серця - дрібні множинні білуваті прошарки сполучної тканини. Вкажіть
макроскопічну стадію атеросклерозу?
A. Плям
B. Смуг
C. Фіброзних бляшок
D. @Ускладнених уражень
E. Виразкування

14. При розтині померлого, 78 років, заочеревинна клітковина просочена кров’ю, у черевному відділі
аорти виявлено мішкоподібне випинання, в якому визначається дефект з нерівним краєм, стінка аорти місцями
кам’янистої щільності. Яке ускладнення атеросклерозу черевного відділу аорти виникло у пацієнта?
A. Істинна аневризма
B. Розшаровуюча аневризма
C. @Хибна аневризма
D. Коарктація аорти
E. Атерокальциноз

15. У померлого, 75 років, який довгий час страждав на атеросклероз церебральних судин, на аутопсії
виявлені: тромбоз правої середньої мозкової артерії, великий осередок неправильної форми сірого кашоподібного
розм’якшення мозкової тканини. Який патологічний процес розвинувся в головному мозку?
A. @Ішемічний інфаркт
B. Геморагічний інфаркт
C. Коагуляційний некроз
D. Гума мозку
E. -

16. При розтині тіла померлого, 56 років, у правій скроневій частці головного мозку знайдено великий
осередок розм’якшеної сірої речовини кашицеподібної консистенції, блідо-сірого кольору. В артеріях основи
мозку чисельні білувато-жовті потовщення інтими, які різко звужують просвіт. Який найбільш вірогідний
діагноз?
A. @Ішемічний інсульт
B. Абсцес мозку
C. Крововилив
D. Геморагічний інсульт
E. Набряк мозку

17. Під час макроскопічного дослідження судин основи головного мозку померлого від ішемічного
інсульту, виявлено, що інтима судин головного мозку нерівна з помірною кількістю жовтих плям та жовтувато-
білуватих бляшок, що звужують просвіт. Ваш діагноз:
A. @Атеросклероз
B. Вузликовий періартеріїт
C. Цукровий діабет
D. Гіпертонічна хвороба
E. Ревматизм

18. При розтині померлого чоловіка, 63 років, виявлено: атеросклероз судин головного мозку, тромбоз
гілки внутрішньої сонної артерії і осередок вологого розм’якшення тканин сірого кольору. Ваш діагноз:
А Гематома
B. Геморагічне просочення
C. Геморагічний інсульт
D. @Ішемічний інсульт
E. Транзиторна ішемія

19. У померлої жінки, 86 років, яка страждала на атеросклероз судин головного мозку, на розтині
виявлено атрофію кори головного мозку. Як називається ця атрофія відносно причини?
A. Від дії фізичних та хімічних факторів
B. Від тиску
C. @Від недостатнього кровопостачання
D. Нейротична
E. Дисфункціональна
20. Жінка, 69 років, довго хворіла на атеросклероз. Поступила в хірургічне відділення з симптомами
гострого живота. При лапаротомії виявлені: тромбоз мезентеріальної артерії, петлі тонкої кишки набряклі,
багряно-чорного кольору, на їх серозній оболонці - фібринозні нашарування. Який патологічний процес
розвинувся у кишці хворої?
A. @Волога гангрена
B. Суха гангрена
C. Ішемічний інфаркт
D. Секвестр
E. Коагуляційний некроз

21. У хворого на гіпертонічну хворобу виявлено значне збільшення маси міокарда лівого шлуночка. Це
сталося внаслідок:
А. Збільшення кількості кардіоміоцитів
B. @Збільшення об’єму кардіоміоцитів
C. Розростання сполучної тканини
D. Затримки води в міокарді
E. Жирової інфільтрації міокарда

22. На розтині в серці виявлено наступні зміни: великий осередок некрозу білого кольору з червоною
облямівкою, який захоплює всю товщу серцевого м’яза. На зовнішній оболонці серця - ознаки фібринозного
перикардиту. Який найбільш імовірний діагноз?
A. @Трансмуральний інфаркт міокарда
B. Субендокардіальний інфаркт міокарда
C. Субепікардіальний інфаркт міокарда
D. Інтрамуральний інфаркт міокарда
E. Міокардит

23. Під час розтину трупа чоловіка, 60 років, у міокарді передньої стінки лівого шлуночка виявлено сірий
неправильної форми щільний осередок 5х4 см з чіткими межами, волокнистої структури. Який діагноз найбільш
вірогідний?
A. @Післяінфарктний міокардіосклероз
B. Дифузний дрібноосередковий міокардіосклероз
C. Інфаркт
D. Міокардит
E. Кардіоміопатія

24. У хворого, 60 років, який багато років страждає на атеросклероз і переніс раніше інфаркт міокарда,
розвинувся напад загрудинного болю. Хворого госпіталізовано через 3 дні; на фоні прогресуючої серцево-
судинної недостатності він помер. Під час розтину тіла у ділянці задньої стінки лівого шлуночка і
міжшлуночкової перегородки виявлено ділянку білого кольору близько 3 см у діаметрі, волокнисту, западаючу, з
чіткою межею. Прозектор трактував ці зміни як:
A. @Осередковий кардіосклероз
B. Ішемія міокарда
C. Інфаркт міокарда
D. Міокардит
E. Дистрофія міокарда

25. Хворий перенiс повторний iнтрамуральний iнфаркт мiокарда. Пiсля лiкування та реабiлiтацiї був
виписаний у задовiльному станi пiд нагляд дiльничного терапевта. Через 2 роки загинув у автомобiльнiй
катастрофi. Який характер патологiчного процесу в мiокардi було встановлено на розтинi?
A. @Крупноосередковий кардiосклероз
B. Дрiбноосередковий кардiосклероз
C. Атрофiя
D. Некроз
E. Гiперплазiя

26. Під час дослідження коронарних артерій виявлені атеросклеротичні бляшки з кальцинозом, що
закривають просвіт судин на 1/3. У м’язі - дрібні множинні білуваті прошарки сполучної тканини. Як називається
процес, виявлений у міокарді?
A. @Дифузний кардіосклероз
B. Тигрове серце
C. Післяінфарктний кардіосклероз
D. Міокардит
E. Інфаркт міокарда

27. Пiд час розтину померлого, 43 рокiв, який страждав на IХС з розвитком iнфаркту мiокарда,
патологоанатом виявив набряк легень. Якi патологiчнi змiни могли зумовити набряк легень?
A. @Гостра лiвошлуночкова недостатнiсть
B. Гостре загальне малокрiв’я
C. Гостра правошлуночкова недостатнiсть
D. Iшемiя малого кола
E. Стаз кровi

28. Хворий помер від прогресуючої серцевої недостатності. На розтині: серце розширене у поперечнику,
дрябле, м’яз на розрізі нерівномірного кровонаповнення, пістрявий, при гістологічному дослідженні у міокарді -
повнокров’я, у стромі - лімфогістіоцитарні інфільтрати, що розсувають кардіоміоцити. Виявлені морфологічні
зміни свідчать про:
A. @Негнійний проміжний міокардит
B. Венозне повнокрів’я
C. Жирову дистрофії міокарда
D. Кардіосклероз
E. Інфаркт міокарда
29. У близнюків, хворих на есенціальну гіпертензію, встановлено зміни скоротливих властивостей
гладких м’язів судин, що пов’язують зі спільним для них дефектом гена, який відповідає за:
A. @Транспорт натрію і кальцію в клітинних мембранах
B. Екскрецію натрію ниркою
C. Кодування синтезу ангіотензиногену
D. Натрій-калієвий транспорт в гладких м’язах
E. Порушення синтезу аденозину

30. При розтині тіла померлого, 57 років, який страждав на гіпертонічну хворобу та помер від серцевої
декомпенсації, виявлено збільшене серце з розширеними порожнинами (550 г товщина стінки лівого шлуночка
2,5 см). Мікроскопічно кардіоміоцити значно збільшені, з жировою дистрофією та гіперхромними
діжкоподібними ядрами. Який з перелічених патологічних процесів найбільш вірогідний у серці?
A. Концентрична гіпертрофія
B. @Ексцентрична гіпертрофія
C. Міокардит
D. Ангіогенний кардіосклероз
E. Кардіоміопатія

31. У чоловіка, 60 років, який тривалий час хворів на гіпертонічну хворобу і помер від хронічної ниркової
недостатності, при аутопсії виявлено, що обидві нирки значно зменшені у розмірах, поверхня їх дрібно-зерниста;
гістологічно - більшість клубочків гіалінізована, частина склерозована, окремі гіперплазовані; в стромі - поля
склерозу; артеріоло- та артеріосклероз, еластофіброз великих гілок ниркових артерії. Для якої форми
гіпертонічної хвороби характерні дані зміни?
A. Злоякісна, мозкова форма
B. Злоякісна, ниркова форма
C. Доброякісна, кардіальна форма
D. Злоякісна, кардіальна форма
E. @Доброякісна, ниркова форма

32. У хворого, який впродовж 15 років хворів на гіпертонічну хворобу, з`явилися скарги на швидку
втомлюваність, поліурію. Хворий помер від хронічної ниркової недостатності при наростанні явищ уремії. На
розтині: нирки зменшені, з дрібногранулярною поверхнею, паренхіма атрофічна, особливо стоншена кіркова
речовина. Який патологічний процес розвинувся в нирках?
A. @Первинно-зморщені нирки
B. Вторинно-зморщені нирки
C. Інфаркт нирок
D. Артеріолонекроз нирок
E. Амілоїдоз нирок

33. При розтині трупа жінки, 69 років, яка тривалий час страждала на гіпертонічну хворобу,
патологоанатом виявив, що обидві нирки щільної консистенції, значно зменшені за розмірами, поверхня їх
дрібнозерниста. Ці зміни свідчать про
A. Гіпоплазію
B. @Атрофію від недостатнього кровопостачання
C. Старечу атрофію
D. Атрофію дисфункціональну
E. Атрофію від тиску

34. Хворий, 67 років, впродовж 20 років страждав на гіпертонічну хворобу. Помер від хронічної ниркової
недостатності. Який вигляд мали нирки при розтині трупа?
A. Великі білі
B. Великі строкаті
C. Великі червоні
D. @Маленькі, щільні, поверхня дрібнозерниста
E. Великі з множинними тонкостінними кістами
35. У хворого, який помер від ускладнень гіпертонічної хвороби, на розтині виявлені маленькі, щільні
нирки з дрібнозернистою поверхнею, паренхіма та кіркова речовина - атрофічні. Дайте назву таким ниркам.
A. @Первинно-зморщені нирки
B. Амілоїдно-зморщені нирки
C. Вторинно-зморщені нирки
D. Пієлонефритично-зморщені нирки
E. Інфаркт нирок

36. У померлої, 56 років, яка впродовж тривалого часу хворіла на гіпертонічну хворобу, на розтині: нирки
значно зменшені (вага обох нирок 80г), їх поверхня дрібнозерниста, на розрізі - кора рівномірно стоншена.
Назвіть зміни в нирках?
A. Вторинно-зморщені нирки
B. Пієлонефритично-зморщені нирки
C. @Первинно-зморщені нирки
D. Амілоїдно-зморщені нирки
E. Діабетичний нефросклероз

37. У 60-річної померлої, яка тривало хворіла на гіпертонічну хворобу, на розрізі нирки значно зменшені
(вага обох нирок 80г), поверхня дрібнозерниста, на розтині кора рівномірно стоншена. Як можна назвати зміни в
нирках?
A. Атеросклеротичний нефросклероз
B. @Артеріолосклеротичний нефросклероз
C. Вторинно- зморщені нирки
D. Амілоїдно-зморщені нирки
E. Діабетичний нефросклероз

38. У чоловіка, 60 років, який тривалий час хворів на гіпертонічну хворобу і помер від хронічної ниркової
недостатності, при аутопсії виявлено, що обидві нирки значно зменшені, поверхня їх дрібнозерниста;
гістологічно - більшість клубочків гіалінізована, частина склерозована, окремі нефрони - гіперплазовані; в стромі
- поля склерозу, артеріоло- та артеріосклероз, еластофіброз великих гілок ниркових артерії. Назвіть виявлені
зміни:
A. Атеросклеротичний нефросклероз
B. @Артеріолосклеротичний нефросклероз
C. Хронічний гломерулонефрит
D. Хронічний пієлонефрит
E. Вторинно зморщена нирка

39. На розтині померлого, обидві нирки зменшені за розмірами вагою близько 50,0 гр з дрібнозернистою
поверхнею та рівномірно стоншеною корковою речовиною. При мікроскопічному дослідженні: стінки артеріол
значно потовщені за рахунок відкладання в них гомогенних, безструктурних мас рожевого кольору, клубочки
зменшені, склерозовані, канальці атрофовані. Для якого захворювання характерні дані зміни?
A. Амілоїдоз
B. Пієлонефрит зі зморщуванням нирки
C. @Гіпертонічна хвороба
D. Гострий гломерулонефрит
E. Мембранозна нефропатія
40. Хворий тривалий час хворів на гіпертонічну хворобу і помер від крововиливу в мозок. На розтині
нирки зменшені, щільної консистенції, поверхня їх дрібнозерниста, кіркова речовина дещо стоншена. Які
мікроскопічні зміни характерні для нирки?
A. @Артеріологіаліноз
B. Поля склерозу
C. Амілоїдоз
D. Інтерстиціальне запалення
E. Інфаркт

41. У хворого на гіпертонічну хворобу діагностовано багатогодинний гіпертонічна криза. Які


патоморфологічні зміни в стінках артеріол виникають під час гіпертонічного кризу?
A. Склероз
B. Гіаліноз
C. @Фібриноїдний некроз
D. Амілоїдоз
E. Кальциноз

42. На розтинi тiла померлого, який страждав на гiпертонiчну хворобу, у лiвiй гемiсферi мозку виявлено
порожнину округлої форми 4х5 см з iржавою стiнкою, заповнену жовтуватою прозорою рiдиною. Назвiть
патологiю, яка розвинулась у головному мозку хворого:
A. @Кiста
B. Iшемiчний iнфаркт
C. Геморагiчне просякнення
D. Гематома
E. Абсцес

43. При розтині тіла померлого, який страждав на гіпертонічну хворобу, у речовині головного мозку
виявлено порожнину, стінки якої мають іржавий колір. Що передувало виникненню даних змін?
A. Плазморагії
B. @Гематома
C. Ішемічний інфаркт
D. Діапедезні крововиливи
E. Абсцес

44. У хворого зі швидко наростаючою внутрішньомозковою гіпертензією діагностовано пухлину мозку.


Під час операції видалено пухлину тім'яно-скроневої ділянки, м'якої консистенції, на розрізі строкатого вигляду.
Гістологічно пухлина побудована з поліморфних гіперхромних клітин з утворенням псевдорозеток та великої
кількості судин, з ділянками некрозів і крововиливами. Який найбільш імовірний діагноз
A. Нома
В. @Гліобластома
С. Фіброма
D. Склерома
Е. Сіре розплавлення мозку

РЕВМАТИЧНІ ЗАХВОРЮВАННЯ
ВАРІАНТ №

1. Яким збудником викликається ревматизм?


A. Стафілокок
В. @Стрептокок
С. Гонокок
D. Пневмокок
E. Клебсієла
2. У жінки, 27 років, сукупність клінічних та лабораторних ознак дозволяє запідозрити ревматичне
захворювання. Вкажіть, яке з наведених положень є найбільш вірогідним для діагностики ревматичних
захворювань:
A. Генетична схильність
B. Хронічний хвилеподібний перебіг
C. @Системна прогресуюча дезорганізація сполучної тканини
D. Системне ураження судин
E. Імунопатологічні реакції

3. У біоптаті з вушка серця хворого на ревматизм виявлені ділянки мукоїдного набухання, фібриноїдного
некрозу сполучної тканини. Яка імунна реакція розвинулась у тканинах вушка серця?
А. Нормергічна реакція
В. Гіперчутливість сповільненого типу
С. Реакція трансплантаційного імунітету
D. @Гіперчутливість негайного типу
Е. Ексудативна реакція

4. При гістологічному дослідженні діагностичної біопсії лівого вушка серця виявлено ревматичну
гранульому, яка побудована з осередку фібриноїдного некрозу, навколо якого розміщені базофільні макрофаги,
лімфоцити, фібробласти і поодинокі плазмоцити. Яка імунна реакція розвинулась у тканині лівого вушка серця?
A. Нормергічний тип
B. @Гіперчутливість сповільненого типу
C. Гіперчутливість негайного типу
D. Вроджений імунодефіцит
E. Набутий імунодефіцит

5. У хворої після переохолодження розвинулася деформація суглобів, болючість та обмеженість рухів у


пальцях кінцівок; поряд з суглобами з’явились маленькі щільні вузлики. При біопсії в вузликах знайдені осередки
фібриноїдного некрозу, оточені гістіоцитами. Ваш діагноз:
A. Дерматоміозит
B. Ревматоїдний артрит
C. @Ревматизм
D. Подагра
E. Деформуючий артроз

6. У дитини, 7 років, на шкірі розгинальних поверхонь ліктьових і колінних суглобів з'явились щільні,
безболісні вузлики розмірами 1-2 мм. У біоптаті вузликів - великий осередок фібриноїдного некрозу сполучної
тканини з лімфоцитами і макрофагами по периферії. При якому захворюванні спостерігаються такі вузлики?
A. Склеродермія
B. Ревматоїдний артрит
C. @Ревматизм
D. Вузликовий периартеріїт
E. Системний червоний вовчак
7. При гістологічному дослідженні вушка серця у хворого на стеноз мітрального отвору виявлені
гранульоми Ашоффа-Талалаєва. Про який генез вади серця свідчать гістологічні дослідження?
A. @Ревматичний
B. Атеросклеротичний
C. Сифілітичний
D. Природжений
E. Септичний
8. На розтинi тiла померлої виявлено морфологiчнi змiни: стеноз лiвого атрiо-вентрикулярного отвору,
недостатнiсть мiтрального клапана. Гiстологiчно в мiокардi - осередковий кардiосклероз, наявнiсть квiтучих
гранульом Ашофф-Талалаєва. Який з перерахованих нижче дiагнозiв найбiльш iмовiрний?
A. @Ревматизм
B. Склеродермiя
C. Дерматомiозит
D. Вузликовий периартерiїт
E. Системний червоний вовчак
9. На розтині померлої виявлено морфологічні прояви хронічної правошлуночкової серцевої
недостатності, стеноз лівого атріовентрикулярного отвору, недостатність мітрального клапана. Гістологічно в
міокарді – осередковий кардіосклероз, наявність в’янучих гранулем Ашоф-Талалаєва. Який з перерахованих
нижче діагнозів найбільш імовірний?
A. @Ревматизм
B. Склеродермія
C. Дерматоміозит
D. Вузликовий периартериїт
E. Системний червоний вовчак

10. У померлого від серцевої недостатності, при патоморфологічному дослідженні виявлено: стулки
мітрального клапана деформовані, потовщені, зрослися по краях; у сполучній тканині міокарда - дифузно
розкидані вузлики, що складаються з ділянок фібриноїдного некрозу, навколо яких скупчуються макрофаги, що
нагадують гігантські багатоядерні клітини. Подібні осередки оточені лімфоцитами та поодинокими
плазматичними клітинами. Яка з перерахованих гранульом має місце у даного хворого?
A. Лепрозна
B. Туберкульозна
C. Актиномікотична
D. Сифілітична
E. @Ревматична

11. У чоловіка, 58 років, померлого при наростаючих явищах хронічної серцевої недостатності,
діагностовано ревматичний гранульоматозний міокардит. Мікроскопічно в міокарді спостерігаються гранульоми,
що складаються з макрофагів з гіперхромними ядрами та світлою цитоплазмою, в центрі - осередок некрозу.
Який характер має некроз в середині гранульоми?
A. @Фібриноїдний
B. Ценкеровський
C. Казеозний
D. Колікваційний
E. Жировий

12. При гістологічному дослідженні стулки мітрального клапана серця виявлено: мукоїдне набухання,
пошкодження ендотелію, утворення по замикаючому краю фібринових тромбів. Яка форма ревматичного
ендокардиту спостерігається?
A. Зворотно-бородавчастий ендокардит
B. Дифузний ендокардит
C. Фібропластичний ендокардит
D. @Гострий бородавчастий ендокардит
E. Поліпозно-виразковий ендокардит

13. У жінки, 27 років, яка з дитинства страждає на ревматичну ваду серця, розвинулась декомпенсована
серцева недостатність. На розтині було виявлено: мітральний стеноз, різке потовщення, склероз стулок, зрощення
їх по лінії змикання. Яка форма ендокардиту зумовила ваду серця?
A. @Фібропластичний
B. Дифузний ендокардит
C. Зворотний ендокардит
D. Гострий бородавчастий
E. Поліпозно-виразковий

14. Під час гістологічного дослідження стулок мітрального клапана серця дівчинки, 14 років, було
встановлено, що ендотеліальні клітини осередково десквамовані, в цих ділянках на поверхні стулки розташовані
дрібні тромботичні нашарування, сполучна тканина стулки помірно дезорганізована з ділянками склерозу та
ангіоматозу. Діагностуйте вид клапанного ендокардиту.
A. Дифузний
B. @Зворотно-бородавчастий
C. Гострий бородавчастий
D. Фібропластичний
E. Поліпозно-виразковий

15. У хворого, 50 років, впродовж багатьох років була ревматична вада серця. При загостренні
захворювання розвинулась геміплегія і настала смерть. Гістологічно: в мітральному клапані виявлено виражений
склероз, осередкові клітинні інфільтрати, фібринозні нашарування. Для якої форми ендокардиту характерні
виявлені зміни?
A. Фібропластичного
B. Гострого бородавчастого
C. Дифузного
D. @Зворотно-бородавчастого
E. Виразково-поліпозного
16. Хвору, 27 років, з ревматичною вадою серця госпіталізовано зі скаргами на задишку, набряки на
ногах, асцит, збільшення печінки. Смерть настала від хронічної серцевої недостатності. На розтині було
діагностовано мітральний стеноз. Який з факторів морфогенезу є найбільш вірогідним?
A. Наявність дрібних тромбів на поверхні клапана
B. Склероз і ущільнення стулок
C. Склероз і вкорочення стулок
D. Вкорочення сухожильних ниток
E. @Зрощення стулок між собою
17. Яка з форм ураження міокарда при ревматизмі спостерігається частіше у дорослих людей?
A. @Вузликовий продуктивний (гранулематозний) міокардит
B. Дифузний проміжний ексудативний міокардит
C. Осередковий продуктивний ексудативний міокардит
D. Альтеративний міокардит
E. Еозинофільний міокардит

18. У 23-річного хворого, який помер від прогресуючої серцевої недостатності діагностовані типові
прояви ревматичного ураження серця: ендокардит мітрального клапана, міокардит і серозно-фібринозний
перикардит. Виберіть найтиповіші зміни в інтерстиції міокарда лівого шлуночка, характерні для ревматичного
міокардиту:
A. Мукоїдне набухання
B. Дифузний лімфо-макрофагальний інфільтрат
C. Переважно периваскулярні, дрібні скупчення великих гіперхромних макрофагів
D. Дрібно осередковий і дифузний склероз
E. @Осередковий лімфо-макрофагальний інфільтрат
19. На розтині померлого, 34 років, від ревматизму - поверхня епікарда ворсиста, покрита плівками сірого
кольору, що легко відділяються. Після їх відділення визначається набрякла повнокровна поверхня епікарда.
Найвірогідніший діагноз:
A. @Фібринозий перикардит
B. Гнійний перикардит
C. Геморагічний перикардит
D. Проліферативний перикардит
E. Катаральний перикардит

20. Дитина, 7 років, померла від швидкого прогресування ревматизму з вираженими алергічними
реакціями. На розтині: у порожнині перикарда було 100 мл напівпрозорої рідини, на поверхні епікарда –
фібринозні нашарування сіруватого кольору. Назвіть, який процес має місце у даному випадку:
А. Геморагічний перикардит
В. Серозний перикардит
С. @Фібринозний перикардит
D. Гнійний перикардит
Е. Фібропластичний перикардит
21. Який різновид ексудату з’являється в перикарді при ревматичному перикардиті?
A. @Серозно-фібринозний
B. Гнилісний
C. Геморагічний
D. Гнійний
E. Катаральний

22. Дитина, 7 років, померла від швидкого прогресування ревматизму з вираженими алергічними
реакціями. На розтині виявлено набряк мітрального клапана, мікроскопічно у ньому виявлено осередки
мукоїдного та фібриноїдного набряку. Міокард щільний, повнокровний, мікроскопічно – набряк і дифузна
інфільтрація інтерстицію лімфоцитами. На епікарді - сіруваті ниткоподібні фібринозні нашарування. Який процес
має місце у даному випадку?
A. @Панкардит
B. Ендокардит
C. Міокардит
D. Перикардит
E. Ревмокардит

23. Хвора, 45 років, звернулась зі скаргами на облисіння. При огляді: шкіра голови плямисто-коричневого
кольору, малорухлива, тоненька, щільна, вкрита роговими лусочками. При гістологічному дослідженні -
гіперкератоз, в дермі склероз, периваскулярні лімфо-макрофагальні інфільтрати, атрофія потових та сальних
залоз. У крові виявлені LE-клітини. Який Ваш діагноз?
A. Ревматизм
B. Ревматоїдний артрит
C. Вузликовий периартеріїт
D. @Системний червоний вовчак
E. Системна склеродермія
24. На розтині померлої жінки, 49 років, - серце збільшене, стулки аортального клапана ущільнені,
потовщені, з тромботичними нашаруваннями. В стінці аорти - еластоліз і дрібні рубчики в середній оболонці.
Нирки – збільшені, строкаті, з діляками крововиливів. Мікроскопічо: у ядрах епітелію канальців -
гематоксилінові тільця, потовщення базальних мембран капілярів клубочків, які мають вигляд дротяних петель,
подекуди в капілярах - гіалінові тромби та осередки фібриноїдного некрозу. Поставте діагноз.
A. @Системний червоний вовчак
B. Ревматизм
C. Артеріолосклеротичний нефросклероз
D. Амілоїдоз
E. Атеросклеротичний нефросклероз

25. На розтині померлої дівчини, 19 років, знайдено бородавчастий ендокардит усіх клапанів, осередкові
крововиливи під ендокард, виразковий стоматит, виразки стравоходу, двобічну пневмонію, нефрит, гіперплазію
селезінки з периваскулярним склерозом, васкуліт і некроз в ділянці зорового бугра, на шкірі – численні
фурункули. Яке більш ймовірне захворювання?
A. @Системний червоний вовчак
B. Ревматизм
C. Септичний ендокардит
D. Ішемічна хвороба серця
E. Гіпертонічна хвороба

26. При гістологічному дослідженні органів і тканин померлої від ниркової недостатності молодої жінки,
у якої прижиттєво виявлявся високий титр антинуклеарних антитіл, виявлені поширені фібриноїдні зміни в
стінках судин. Відмічається ядерна патологія з вакуолізацією ядер, каріорексисом, утворенням гематоксилінових
тілець. Який найбільш імовірний діагноз?
A. @Системний червоний вовчак
B. Атеросклероз
C. Вузликовий периартеріїт
D. Гіпертонічна хвороба
E. Облітеруючий ендартеріїт

27. У хворого з високим титром антинуклеарних антитіл смерть настала від наростаючої ниркової
недостатності. При патологоанатомічному дослідженні виявлено: мезангіопроліферативний гломерулонефрит,
абактеріальний бородавчастий ендокардит, у селезінці - периартеріальний цибулинний склероз, у шкірі –
продуктивно-проліферативний васкуліт. Ваш діагноз.
A. @Системний червоний вовчак
B. Нефротичний синдром
C. Ревматизм
D. Дерматоміозит
E. Вузликовий периартеріїт

28. На розтині тіла жінки, 49 років, яка померла від хронічної ниркової недостатності, виявлено: нирки
ущільнені, зменшені, строкаті, з ділянками крововиливів. Мікроскопічно: у ядрах епітелію канальців -
гематоксилінові тільця, потовщення базальних мембран капілярів клубочків, які мають вигляд дротяних петель,
подекуди в капілярах - гіалінові тромби та осередки фібриноїдного некрозу. Який найбільш вірогідний діагноз?
A. @Системний червоний вовчак
B. Ревматизм
C. Артеріолосклеротичний нефросклероз
D. Амілоїдоз
E. Атеросклеротичний нефросклероз

29. На розтині померлої від уремії, 25 років: нирки збільшені, пістряві, з осередками крововиливів.
Патогістологічно виявляються гематоксилінові тільця, капілярні мембрани клубочків у вигляді дротяних петель,
гіалінові тромби та осередки фібриноїдного некрозу, у судинах селезінки – “цибулинний” склероз. Який Ваш
діагноз?
A. Системна склеродермія
B. @Системний червоний вовчак
C. Ревматоїдний артрит
D. Ревматичний артрит
E. Вузликовий периартеріїт
30. При розтині тіла жінки, 40 років, померлої від уремії, знайдено: збільшені пістряві нирки, у нирках –
потовщення капілярних мембран клубочків у вигляді “дротяних петель”, осередки фібриноїдного некрозу їх
стінок та “гіалінові” тромби у просвітах, у ядрах – “гематоксилінові тільця”; у серці – ендокардит Лібмана-Сакса.
Яке ураження нирок найбільш вірогідне?
A. @Вовчаковий нефрит
B. Ревматичний гломерулонефрит
C. Холерний гломерулонефрит
D. Склеродермічна нирка
E. Термінальний гломерулонефрит

31. У хворого, 38 років, на 3-му році захворювання на системний червоний вовчак виявлене дифузне
ураження нирок, що супроводжується масивними набряками і вираженою протеїнурією. Що є найбільш
вірогідною причиною розвитку протеїнурії у пацієнта?
A. @Аутоімунне ушкодження нирок
B. Асептичне ураження нирок
C. Ішемічне ушкодження нирок
D. Запальне ураження сечового міхура
E. Запальне ураження сечовивідних шляхів

32. Жінка, 45 років, померла внаслідок уремії при системному червоному вовчаку. Утворення антитіл до
яких стуктур найбільш вірогідно в механізмі розвитку даного захворювання?
A. Ендотеліальних клітин
B. Міоцитів
C. Міозину
D. @Нативної ДНК
E. Ревматоїдного фактора

33. Жінка, 54 років, мала значну деформацію суглобів пальців рук і стоп. Гістологічно: у
навколосуглобовій сполучній тканині - мукоїдне набухання, ділянки фібриноїдного некрозу, скупчення
макрофагів, ділянки склерозу; в синовіальній оболонці - набрякі ворсини, з ознаками мукоїдного та
фібриноїдного набухання; в синовіальній порожнині зустрічаються поодинокі "рисові тільця". Діагностуйте
захворювання.
A. @Ревматоїдний артрит
B. Ревматизм
C. Хвороба Бехтєрева
D. Гематогенний туберкульоз
E. Подагра

34. Хворий, 70 років, помер від гострої коронарної недостатності. За життя виявились припухлість,
деформація і біль колінних суглобів. При патоморфологічному дослідженні деформованих суглобів і
синовіальних оболонок виявлено: гіперемія оболонок з множинними периваскулярними запальними
інфільтратами з лімфоцитів, плазмоцитів, макрофагів. Скупчення фібрину, які організуються, вкривають ділянки
синовіальної оболонки і визначаються в суглобовій рідині у вигляді рисових зерен. Ваш діагноз.
A. Деформуючий артроз
B. Вузликовий периартеріїт
C. Анкілозуючий спондилоартрит
D. Туберкульозний артрит
E. @Ревматоїдний артрит
35. Жінка, 56 років, скаржиться на болі в дрібних суглобах рук і ніг. Хворіє впродовж 12 років, під час
огляду рук відмічено підвивих у п’ястно-пальцевих суглобах з відхиленням пальців назовні (вигляд „плавників
моржа”). В крові виявлено високомолекулярні імунні комплекси. Назвати основне захворювання.
A. @Ревматоїдний артрит
B. Ревматичний поліартрит
C. Системний червоний вовчак
D. Дерматоміозит
E. Подагричний артрит

36. Чоловік, 45 років, впродовж останніх 10 років скаржився на біль у спині, обмеження повноцінних
рухів у хребті. Хворий помер від прогресуючої ниркової недостатності. При аутопсійному дослідженні виявлено
амілоїдоз нирок, у суглобах хребта – анкілоз, деструкція суглобового хряща, панус, склероз синовіальної
оболонки з дифузним хронічним запаленням. Діагностуйте захворювання.
A. Остеоартроз хребта
B. @Анкілозуючий спондилоартрит
C. Ревматоїдний артрит
D. Остеопороз хребта
E. Паратиреоїдна остеодистрофія

37. Хворий, 6 рокiв, скаржиться на значну слабкiсть. Захворiв гостро з пiдвищенням температури тiла,
нездужанням, болем у суглобах та за ходом м’язiв нiг. Об’єктивно: фiолетово-синюшна еритема навколо очей та
над колiнними суглобами. ЧСС- 120/хв., тони серця ослабленi. У кровi: лейк.-12х109, ШЗЕ-40 мм/год. Який
найбiльш вiрогiдний дiагноз?
A. @Дерматомiозит
B. Системний червоний вовчак
C. Ревматоїдний артрит
D. Атопiчний дерматит
E. Реактивний полiартрит

38. Жінці, 45 років, зі скаргами на прогресуючу м’язову слабкість, проведено біопсію м’яких тканин
гомілки. При гістологічному дослідженні біоптату знайдено: дрібноосередкову петрифікацію дерми і скелетних
м’язів, зменшення кількості глікогену і зниження поперечної посмугованості у м’язових волокнах, деякі волокна
некротизовані, строма інфільтрована лімфоцитами, макрофагами і плазматичними клітинами. Діагностуйте
захворювання.
А. @Дерматоміозит
В. Системна склеродермія
С. Системний червоний вовчак
D. Ревматизм
Е. Вузликовий периартеріїт

39. У хворої, 40 років, біль у суглобах та за ходом м'язів ніг, який турбує її впродовж 2 місяців після
перенесеної вірусної інфекції. Об'єктивно: фіолетово-синюшна еритема навколо очей та колінних суглобів. При
мікроскопічному дослідженні біоптатів із м'язів гомілки та шкіри: в їх сполучній тканині: набряк, навколосудинні
лімфоплазмоцитарні інфільтрати; ділянки мукоїдного, фібриноїдного набряку, гомогенізації, гіалінізації
колагенових волокон шкіри. Крім цього, в м'язових волокнах - зникнення поперечної смугастості, зменшення
вмісту глікогену. Який найбільш імовірний діагноз?
A. @Дерматоміозит
B. Системний червоний вовчак
C. Ревматизм
D. Склеродермія
E. Вузликовий периартеріїт

40. При гістологічному дослідженні біоптатів скелетних м’язів хворого з підозрою на колагеноз виявлено
дистрофічні зміни: відсутність поперечної посмугованості, багато некротизованих м’язових волокон, відклади
дрібних зерен вапна в осередках некрозу. Вкажіть найбільш ймовірне захворювання.
A. Поліміозит
B. @Дерматоміозит
C. Системна склеродермія
D. Міотонічна дистрофія
E. М’язова дистрофія Беккера

41. При гістологічному дослідженні біоптату з ураженої ділянки шкіри виявлено: незначно виражену
атрофію епідермісу, в дермі - гіаліноз колагенових волокон, скупі периваскулярні лімфоцитарні інфільтрати, у
підлеглих скелетних м’язах - виражений інтерстиційний набряк, втрата поперечної посмугованості,
дрібноосередкові некрози з петрифікацією. Діагностуйте виявлене захворювання.
A. @Дерматоміозит
B. Склеродермія
C. Системний червоний вовчак
D. Вузилковий периартериїт
E. Ревматизм

42. Вузликовий периартериїт характеризується розвитком запалення в кровоносних судинах наступного


типу:
A. @Артеріях середнього і дрібного калібру
B. Аорті
C. Артеріях великих по калібру
D. Артеріолах
E. Капілярах
43. На розтині тіла жінки, 27 років, що померла від хронічної ниркової недостатності, виявлені множинні
рубці та інфаркти в селезінці, нирках. Під час мікроскопічного дослідження виявлені зміни в артеріях середнього
та дрібного калібру, які характеризуються склерозом стінок, помірною проліферацією ендотелію і вираженою
лімфогістіоцитарною інфільтрацією навколо. На яке захворювання страждала померла?
A. @Вузликовий периартеріїт
B. Атеросклероз
C. Гіпертонічна хвороба
D. Хвороба Марфана
E. Вісцеральний сифіліс

ЗАХВОРЮВАННЯ ШЛУНКОВО-КИШКОВОГО ТРАКТУ

1. При огляді зіва у хворого на ангіну визначається: гіперемія слизової оболонки піднебіння, мигдалики
збільшені, червоні, на їх поверхні помітні дрібні біло-жовті ділянки. Який клініко-морфологічний варіант ангіни
найбільш імовірний у даному випадку?
A. @Лакунарна
B. Катаральна
C. Гнійна
D. Фібринозна
E. Некротична
2. У дитини підвищилась температура тіла, з’явились болі при ковтанні. При огляді: піднебінні мигдалики
збільшені, темно-червоні, вкриті сіро-жовтими плівками, щільно спаяними з поверхнею мигдаликів. Яке
запалення розвинулося в мигдаликах?
A. @Дифтеритичне
B. Крупозне
C. Геморагічне
D. Гнійне
E. Катаральне
3. На слизовій оболонці мигдаликів та м’якого піднебіння виявляються білувато-сірого кольору плівки,
які щільно з’єднані з підлеглою тканиною; при спробі зняти плівку на її місці виникає глибокий дефект тканини.
Визначити патологічний процес, який виник на слизовій оболонці мигдаликів та м’якого піднебіння?
A. @Дифтеритичне запалення
B. Серозне запалення
C. Крупозне запалення
D. Гнійне запалення
E. Змішане запалення
4. При ендоскопії шлунка, на гістологічне дослідження взято біоптат слизової оболонки. Його
дослідження виявило: слизова оболонка збережена, потовщена, набрякла, гіперемована, з множинними дрібними
крововиливами, щільно вкрита слизом. Визначити форму гострого гастриту.
A. @Катаральний (простий)
B. Ерозивний
C. Фібринозний
D. Гнійний
E. Некротичний

5. При гастроскопії на гістологічне дослідження узято біоптат слизової оболонки. При дослідженні
виявлено, що слизова оболонка збережена, потовщена, з набряком, гіперемією, з численними дрібними
крововиливами, рясно покрита слизом. Визначити форму гострого гастриту.
A. @Катаральний (простий)
B. Ерозивний
C. Фібринозний
D. Гнійний
E. Некротичний
6. При гастробіопсії хворого виявлено: потовщення слизової оболонки в пілоричному відділі, вона
набрякла, гіперемована, поверхня вкрита товстим шаром слизу, подекуди - дрібні крововиливи. Ваш діагноз.
A. @Катаральний (простий) гастрит
B. Флегмонозний гастрит
C. Фібринозний гастрит
D. Сифілітичний гастрит
E. Туберкульозний гастрит
7. Хвора, 38 років, поступила в клініку з температурою тіла 39,4°C, різкими розлитими болями в
епігастральній області. При операції прийнято рішення видалити весь шлунок. Гістопатологічне дослідження всіх
анатомічних відділів шлунка показало розлите гнійно-деструктивне запалення всіх шарів стінки з утворенням
численних дрібних абсцесів. Діагноз?
A. @Флегмонозний гастрит
B. Хронічний гастрит, тип А
C. Хронічний гастрит, тип В
D. Хронічний гастрит, тип С
E. Хелікобактер-асоційований гастрит
8. При дослідженні шлунка померлого, який страждав на ниркову недостатність, виявлено наявність
жовто-коричневої плівки на потовщеній слизовій оболонці, яка міцно утримується на її поверхні та досягає
значної товщини. Мікроскопічно: наявність гіперемії та некрозу слизового та підслизового шарів, фібрин. Ваш
діагноз:
A. @Дифтеритичний гастрит
B. Крупозний гастрит
C. Флегмонозний гастрит
D. Катаральний гастрит
E. Корозивний гастрит
9. Хворому, з тривалими болями в шлунці, виконано гастробіопсію. Гістологічно виявлено: покривно-
ямковий епітелій сплощений, залози - вкорочені, кількість їх зменшена. У власній пластинці слизової оболонки
виявляються ділянки склерозу і дифузний лімфоплазмоцитарний інфільтрат. Ваш діагноз.
A. @Атрофічний гастрит
B. Гіпертрофічний гастрит
C. Виразкова хвороба
D. Поверхневий гастрит
E. Ерозивний гастрит
10. При фiброгастроскопiї хворого, водiя, 48 рокiв, який скаржився на бiль в епiгастрiї пiсля прийому їжі,
слизова оболонка шлунка гiперемована, складки її зменшенi. Мiкроскопiчно в гастробiоптатi: слизова оболонка
стоншенена, кiлькiсть залоз зменшена, розростання сполучної тканини, iнфiльтрованої лiмфоцитами,
плазмоцитами. Вкажіть діагноз.
A. @Хронiчний атрофiчний гастрит
B. Гострий катаральний гастрит
C. Гострий гнiйний гастрит
D. Хронiчний поверхневий гастрит
E. Гiгантський гiпертрофічний гастрит
11. При гастробiопсiї у хворого встановлено метаплазiю поверхневого епiтелiю слизової оболонки, який
замiсть цилiндричного набув вигляду кишкового. Разом з тим, спостерiгається склероз на мiсцi залоз слизової
оболонки та лiмфогiстiоцитарна iнфiльтрацiя. Про яке захворювання шлунка можна думати?
A. @Хронiчний атрофiчний гастрит
B. Корозивний гастрит
C. Хронiчний гастрит з ураженням залоз без атрофiї
D. Ерозивний гастрит
E. Поверхневий хронiчний гастрит

12. При гістологічному дослідженні гастробіоптатів виявлено стоншення слизової оболонки шлунка зі
зменшенням кількості залоз і значним розростанням сполучної тканини, протоки залоз розширені; слизова
оболонка інфільтрована лімфоцитами і плазматичними клітинами. Який з перерахованих діагнозів найбільш
вірогідний?
A. Хронічний поверхневий гастрит
B. Хронічний виражений атрофічний гастрит з кишковою метаплазією
C. Хронічний помірний атрофічний гастрит
D. @Хронічний виражений атрофічний гастрит
E. Флегмона шлунка

13. При гістологічному дослідженні біоптату слизової оболонки шлунка жінки, 50 років, виявлено:
стоншення слизової оболонки, зменшення кількості залоз, осередки кишкової метаплазії, повнокров'я, набряк і
склероз строми, дифузна лімфоплазмоцитарна інфільтрація зі значними включеннями полінуклеарних
лейкоцитів. Встановіть діагноз.
A. Хронічний атрофічний гастрит в неактивній фазі
B. Хронічний поверхневий гастрит
C. @Хронічний атрофічний гастрит в активній фазі
D. Гострий катаральний гастрит
E. Гострий фібринозний гастрит
14. Виберіть вид хронічного гастриту, при якому відбувається перебудова епітелію.
A. @Гастрит з кишковою метаплазією
B. Поверхневий
C. Антральний
D. Фундальний
E. Глибокий

15. У хворого при фіброгастроскопії шлунка виявлено різке потовщення слизової оболонки. Гістологічно
в біоптаті виявлені: проліферація залозистих клітин, гіперплазія залоз, лімфо-плазмоцитарна інфільтрація
слизової оболонки. Ваш діагноз.
A. @Гіпертрофічний гастрит
B. Аутоімунний гастрит
C. Дифузний рак шлунка
D. Рефлюкс-гастрит
E. Гострий гастрит
16. У хворого, який помер від гострої серцевої недостатності, клінічно спостерігалась шлунково-кишкова
кровотеча. При досліджені на слизовій оболонці шлунка знайдено декілька дефектів слизової які досягали до
м’язового шару; краї та дно їх рівні, пухкі, в деяких знайдені згустки крові. Який патологічний процес виявлено в
шлунці?
A. Хронічні виразки
B. Ерозії
C. @Гострі виразки
D. Тромбоз
E. Запалення
17. У померлого хворого від гострої серцевої недостатності, клінічно спостерігалась haemotemesis, при
дослідженні слизової оболонки шлунка виявлено декілька дефектів, які сягають м’язового шару; краї і дно їх
переважно рівні та пухкі, в деяких виявлено темно-червону кров. Який патологічний процес виявлено у шлунці?
A. @Гострі виразки
B. Хронічні виразки
C. Ерозії
D. Тромбоз
E. Запалення
18. При морфологічному дослідженні шлунка виявлено глибокий дефект стінки з ураженням м’язової
болонки, проксимальний край якого підритий, дистальний - пологий. При мікроскопічному дослідженні: в дні
дефекту виявляється зона некрозу, під якою - грануляційна тканина та масивна ділянка рубцевої тканини на місці
м’язового шару. Встановити діагноз.
A. @Хронічна виразка у стадії загострення
B. Хронічна виразка з малігнізацією
C. Гостра виразка
D. Ерозія
E. Рак-виразка

19. У хворого виявлено глибокий дефект стінки з ураженням м'язової оболонки, проксимальний край
якого підритий, дистальний - пологий. При мікроскопічному дослідженні в дні дефекту виявлено: зона
фібриноїдного некрозу, під якою, - грануляційна тканина і масивна ділянка грубої волокнистої тканини на місці
м'язового шару. Встановіть діагноз.
A. Хронічна виразка з малігнізацією
B. Гостра виразка
C. Ерозія
D. Рак-виразка
E. @Хронічна виразка у стадії загострення
20. При гістологічному дослідженні вилученої виразки шлунка в її дні знайдено фібринозно-
лейкоцитарний ексудат, поширена зона фібриноїдного некрозу, нижче розташовані шари грануляційної та
фіброзної тканини. Ваш діагноз:
A. @Хронічна виразка
B. Гостра виразка
C. Гостра ерозія
D. Малігнізована виразка
E. Флегмона шлунка

21. У хворого з кривавою блювотою при операції виявлено виразку, проникаючу в м'язовий шар шлунка.
Край виразки щільний, на дні - судина, що кровоточить. При біопсії в краях і дні виразки виявлено рубцеву
тканину. Яка це виразка?
A. Пенетруюча виразка
B. Гостра виразка, що кровоточить
C. @Хронічна виразка, що кровоточить
D. Перфоративна виразка шлунка
E. Малігнізована виразка

22. Хворому з хронічною виразковою хворобою без загострення виконано фіброгастроскопію з біопсією
слизової оболонки. Які морфологічні зміни в біоптаті свідчитимуть про стадію ремісії виразкової хвороби?
A. Фібриноїдний набряк
B. Фібриноїдні зміни стінок судин
C. Наявність гнійно-некротичного детриту
D. @Розростання сполучної тканини
E. Зона фібриноїдного некрозу в дні виразки

23. При дослідженні видаленого шлунка, лікар виявив в антральному відділі по малій кривизні глибокий
дефект слизової оболонки, що доходить до м'язового шару, діаметром 1,5 см, округлої форми, з рівними краями.
У дні дефекту визначалася напівпрозора, щільнувата ділянка, що на вигляд нагадувала гіаліновий хрящ. Який
процес розвинувся в дні дефекту?
A. @Місцевий гіаліноз
B. Амілоїдоз
C. Мукоїдний набряк
D. Фібриноїдні зміни
E. Загальний гіаліноз

24. При розтині чоловіка, померлого в результаті прогресуючої анемії, в шлунці виявлено близько 1 л
рідкої крові і кров'яних згустків, а на малій кривизні розташовується виразка овальної форми з валикоподібно
підведеними щільними краями і гладким дном. Яке захворювання має місце?
A. Хронічний атрофічний гастрит
B. @Хронічна виразка шлунка
C. Гострий гастрит
D. Гостра виразка шлунка
E. Хронічний гіпертрофічний гастрит
25. У хворого з кривавою блювотою на операції в шлунці знайдено виразку, що проникає в м’язовий шар.
Краї виразки щільні, в дні – кровоточива судина. При цитобіопсії в краях і дні виразки виявлено рубцеву тканину.
Яке ускладнення виразкової хвороби у пацієнта?
A. @Виразково-деструктивне
B. Виразково-запальне
C. Рубцеве
D. Змішане
E. Малігнізація

26. У хворого, який страждав на виразкову хворобу шлунка, ускладнену шлунковою кровотечею,
блювотні маси забарвлені в темно-коричневий колір, які описуються як блювота “кавовою гущею”. Наявність
якого пігменту визначає таке забарвлення блювотних мас?
A. Гемоглобіну
B. Білірубіну
C. Гемомеланіну
D. @Солянокислого гематину
E. Сульфіду заліза

27. На аутопсії померлого, 56 років, який тривало страждав на виразкову хворобу шлунка виявлено
виразку на малій кривизні з численними розростаннями щільної рубцевої тканин та потовщення слизової
оболонки біля виразки. При гістологічному дослідженні виявляється різко виражений клітинний атипізм. Ваш
діагноз:
A. Пенетрація виразки шлунка
B. Виразковий рак шлунка
C. Метастаз раку легені у шлунок
D. Перфорація виразки шлунка
E. @Малігнізована хронічна виразка шлунка

28. Хворому з діагнозом «Рак шлунка» проведено гастректомію. При патогістологічному дослідженні
виявлено: стінка шлунка потовщена, щільна, нерухома. Пухлина росте ендофітно в слизовому, підслизовому і
м'язовому шарах за ходом сполучнотканинних прошарків. Назвіть клініко-анатомічну форму раку.
A. Інфільтративно-виразковий
B. Поліпозний
C. @Дифузний
D. Фунгозний
E. Блюдцеподібний

29. При розтині трупа чоловіка, померлого від ракової кахексії, при макроскопічному дослідженні стінка
шлунка потовщена до 1,2 см, слизова оболонка нерухома, складки не виявляються. На розрізі - однорідна,
біляста, хрящоподібної щільності. Для якої макроскопічної форми пухлини характерні описані зміни?
A. Вузловатої
B. @Інфільтративної
C. Виразкової
D. Виразково-інфільтративної
E. Кістозної
30. При розтині померлої, 33 років, знайдено: потовщення стінки шлунка в пілоричному відділі (на розрізі
шари стінки розрізняються) з розростанням щільної білуватої тканини в підслизовому шарі і дрібними тяжами її
в м'язовому шарі. Рельєф слизової оболонки збережений, складки ригідні, нерухомі. Яка макроскопічна форма
пухлини в даному випадку?
A. @Інфільтрат
B. Вузол
C. Виразка
D. Кіста
E. Інфільтративно-виразкова форма

31. При ендоскопії шлунка виявлено атрофію слизової оболонки. Мікроскопічно в біоптаті - пухлина,
побудована з ланцюжків атипових епітеліальних клітин, розташованих серед тяжів грубоволокнистої сполучної
тканини; строма значно переважає над паренхімою. Який найбільш імовірний діагноз?
A. @Скірозний рак
B. Солідний рак
C. Медулярний рак
D. Дрібноклітинний рак
E. Аденокарцинома

32. При гiстологiчному дослiдженнi бiоптатiв, взятих з потовщених країв виразки шлунка, виявленi
невеликi гнiздовi скупчення рiзко атипових гiперхромних невеликих епiтелiальних клiтин, якi розташованi серед
дуже розвиненої строми. Визначте пухлину:
A. @Скiрозний недиференцiйований рак
B. Медулярний рак
C. Аденокарцинома
D. Недиференцiйована саркома
E. Аденома
33. При рентгенологічному дослідженні шлунка, лікар виявив дефект накопичення контрастної речовини
на малій кривизні за типом “плюс тканина мінус тінь”. У біоптаті цієї ділянки знайдено розростання залоз з
атипових клітин, багаточисленні патологічні мітози, гіперхромність клітин зі збільшеними ядрами. Про який
діагноз можна думати?
A. @Аденокарцинома шлунка
B. Атрофічно-гіперпластичний гастрит
C. Поліп шлунка
D. Плоскоклітинний рак шлунка
E. Виразка - рак шлунка
34. При гістологічному дослідженні пухлини шлунка виявлено багато персневидних клітин. Назвіть
гістологічний варіант раку.
A. @Слизовий
B. Солідний
C. Аденокарцинома
D. Саркома
E. Карциноїд
35. У померлої, 42 років, оперованої у минулому з приводу пухлини шлунка при розтині: яєчник різко
збільшений в розмірах, щільної консистенції, білуватого кольору. Гістологічно: в препараті крайньо атипові
епітеліальні клітини, розміщені серед пластів і тяжів сполучної тканини. Яке захворювання мало місце у хворої?
A. @Крукенбергівський рак яєчника
B. Серозна цистаденокарцинома
C. Псевдомуцинозна цистаденокарцинома
D. Злоякісна текома
E. Злоякісна гранульозоклітинна пухлина
36. У жінки, 46 років, під час паліативної операції з приводу раку шлунка встановлено наявність
крукенбергівських метастазів в яєчники (“крукенбергівський рак яєчників”). Який з наведених шляхів
метастазування призвів до ураження яєчників?
A. @Лімфогенний ретроградний
B. Лімфогенний ортоградний
C. Гематогенний
D. Імплантаційний
E. Каналікулярний

37. У хворої, 45 років, збільшились надключичні лімфатичні вузли. Під час дослідження біопсійного
матеріалу з лімфатичного вузла виявлено метастатичне пошкодження персневидно-клітинного раку. Виберіть
найбільш вірогідну локалізацію первинної пухлини.
A. Рак стравоходу
B. Рак щитовидної залози
C. Рак легень
D. Рак шийки матки
E. @Рак шлунка

38. Жінка, віком 41 рік, скаржиться на біль в епігастрії через 2-2,5 год після приймання їжі. Харчова
непереносимість риби та яєць. При ФГДС виявлено хронічну виразку цибулини дванадцятипалої кишки. Діагноз
підтверджено гістологічним дослідженням біопсії слизової оболонки дванадцятипалої кишки. Позитивна уреазна
реакція. Який найбільш вірогідний ведучий механізм розвитку захворювання?
A. @Хелікобактерна інфекція
B. Продукція ауто антитіл
C. Кишкова інфекція
D. Зниження синтезу простагландинів
E. Харчова алергія
39. Під час розтину тіла чоловіка, 29 років, який тривалий час хворів на виразкову хворобу
дванадцятипалої кишки були виявлені ознаки перитоніту, множинні стеатонекрози заочеревеної жирової тканини
і головки підшлункової залози, а в ділянці дна виразки по задній стінці ДПК знайдено дефект діаметром 5 мм і
глибиною до 15 мм, краї якого містять некротичні маси. Діагностуйте ускладнення виразкової хвороби
дванадцятипалої кишки.
A. Кровотеча
B. Стеноз
C. Перфорація
D. @Пенетрація
E. Малігнізація
40. У хворого під час операції з приводу хронічного апендициту виявлені розростання слизоподібної
тканини на очеревині. Червоподібний відросток кістозно розширений, заповнений слизовими масами у вигляді
глобул. Вкажіть найбільш імовірне захворювання:
A. Флегмонозний апендицит
B. Апостематозний апендицит
C. @Мукоцеле червоподібного відростка
D. Первинний гангренозний апендицит
E. Простий апендицит

41. При операцiї у хлопчика, 12 рокiв, видалено апендикс, який надiслано патологу на дослiдження.
Макроскопiчно: апендикс у дистальному вiддiлi з булавоподiбним потовщенням дiаметром 3 см, при розрiзi
якого вилилася прозора жовтувата рiдина, стiнка апендикса стоншена. Мiкроскопiчно: атрофiя всiх шарiв
апендикса, ознак запалення немає. Ваш діагноз:
A. @Водянка аппендикса
B. Флегмонозний апендицит
C. Емпiєма апендикса
D. Хронiчний апендицит
E. Мiксоглобульоз аппендикса
42. Апендикс, надісланий у патоморфологічне відділення після апендектомії, потовщений і
збільшений за розмірами, серозна оболонка блискуча, судини повнокровні, з просвіту відростка виділяються
калові маси. При якій формі апендициту розвиваються такі зміни?
A. @Простий катаральний
B. Поверхневий катаральний
C. Флегмонозний
D. Гангренозний
E. Апостематозний
43. Апендикс довжиною 9 см, товщиною 0,9 см. Серозна оболонка тьмяна, повнокровна. Мікроскопічно –
стінка набрякла, стази у капілярах і венулах - дрібні крововиливи; у слизовій і підслизовій оболонках – осередки
некрозу с лейкоцитарною інфільтрацією навкруги їх. Який з перелічених діагнозів найбільш вірогідний?
A. @Гострий поверхневий апендицит
B. Гострий простий апендицит
C. Гострий флегмонозний апендицит
D. Гострий флегмонозно-виразковий апендицит
E. Гострий гангренозний апендицит

44. Гістологічно в апендиксі у всіх шарах виявлені осередкові накопичення поліморфноядерних


лейкоцитів з осередками некрозу, повнокров'я, гемостази. Така картина характерна для:
A. Гангренозного апендициту
B. Поверхневого апендициту
C. Простого апендициту
D. Хронічного апендициту
E. @Апостематозного апендициту
45. При дослідженні апендикса спостерігається дифузна лейкоцитарна інфільтрація всіх шарів стінки.
Назвіть форму апендициту.
A. @Гострий флегмонозний
B. Гострий простий
C. Некротичний
D. Гострий поверхневий
E. Гангренозний

46. До патогістологічої лабораторії доставлено червоподібний відросток товщиною до 2,0 см. Серозна
оболонка його тьмяна, потовщена, вкрита жовто-зеленими плівковими нашаруваннями. Стінка дрябяла, сіро-
червона. Просвіт відростка розширений, виповнений жовто-зеленими масами. При гістологічному дослідженні
виявлено, що стінка інфільтрована нейтрофілами. Визначте захворювання апендикса:
A. @Гострий флегмонозний аппендицит
B. Гострий гангренозний апендицит
C. Гострий поверхневий апендицит
D. Гострий простий апендицит
E. Хронічний апендицит
47. На гістологічне дослідження надіслано видалений червоподібний відросток. Розміри його збільшені,
серозна оболонка тьмяна, повнокровна, вкрита плівками фібрину, стінки потовщені, на розрізі із просвіту
виділяється гній. При мікроскопічному дослідженні спостерігається повнокров’я судин, набряк всіх шарів і
дифузна інфільтрація їх лейкоцитами. Назвіть форму гострого апендициту:
A. Флегмонозний
B. Апостематозний
C. Простий
D. Поверховий
E. Гангренозний
48. Оперативно видалений червоподібний відросток потовщений, серозна оболонка його тьмяна,
повнокровна, з білуватими рихлими плівчастими накладеннями, в просвіті - каламутна, білувато-жовта рідина.
При мікроскопічному дослідженні стінка відростка дифузно інфільтрована нейтрофілами. Про який варіант
апендициту можна думати в даному випадку?
A. @Флегмонозний
B. Гангренозний
C. Простий
D. Поверхневий
E. Хронічний

49. Апендикс, надісланий до патомор- фологічного відділення після апендектомії, потовщений і


збільшений за розмірами, серозна оболонка тьмяна, судини повнокровні, з просвіту відростка на розрізі
виділяється рідина жовто- зеленого кольору. При якій формі апендициту виникають такі зміни?
A. @Флегмонозний
B. Простий катаральний
C. Поверхневий катаральний
D. Гангренозний
E. Апостематозний
50. Гістологічно в апендиксі в усіх шарах знайдені в значній кількості поліморфноядерні лейкоцити,
повнокров’я, стази. Така картина є характерною для:
A. @Флегмонозного апендициту
B. Гангренозного апендициту
C. Поверхневого апендициту
D. Простого апендициту
E. Хронічного апендициту

51. При мікроскопічному дослідженні оперативно видаленого апендикса відмічався набряк, дифузна
нейтрофільна інфільтрація стінки з некрозом та наявністю дефекту слизової оболонки з ураженням її м’язової
пластинки. Яка форма апендициту розвинулась у хворого?
A. @Флегмонозно-виразкова
B. Флегмонозна
C. Гангренозна
D. Поверхнева
E. Апостематозна
52. Вилучений при апендектомії хробакоподібний відросток потовщений, покритий фібринозно-гнійним
нальотом. Всі шари відростка інфільтровані нейтрофілами, слизова оболонка місцями зруйнована. Ваш діагноз?
A. @Флегмонозно-виразковий апендицит
B. Простий апендицит
C. Флегмонозний апендицит
D. Гангренозний апендицит
E. Поверхневий апендицит

53. Видалений при апендектомії червоподібний відросток потовщений, покритий фібринозно-гнійним


нальотом. Всі шари відростка інфільтровані гнійним ексудатом, слизова оболонка зруйнована. Ваш діагноз?
A. Простий апендицит з осередковою деструкцією слизового і підслизового шару
B. Флегмонозний апендицит з осередковою деструкцією слизового і підслизового шару
C. Гангренозний апендицит з осередковою деструкцією слизового і підслизового шару
D. @Флегмонозно-виразковий апендицит з осередковою деструкцією слизового і підслизового шару
E. Поверхневий апендицит з осередковою деструкцією слизового і підслизового шару
54. При терміновій операції лапаротомії у хворого, 42 років, було виявлено різко потовщений, сірувато-
чорного кольору червоподібний відросток. У дистальному відділі його – дефект стінки, через який з просвіту
апендикса виділяються сірувато-бурі маси з неприємним запахом. При мікроскопічному дослідженні: стінка
апендикса впродовж всієї довжини некротизована з осередками крововиливів, просвіт брижової артерії
заповнений тромбом. Яка морфологічна форма апендициту у хворого?
A. @Гангренозний
B. Флегмонозний
C. Хронічний
D. Простий
E. Апостематозний

55. При огляді хворого, 27 років, виявлені чисельні параректальні нориці, з яких виділяється гнійний
ексудат. Спостерігаються явища парапроктиту. Ускладненням якого коліту можуть бути вищезазначені прояви.
A. @Гострого
B. Хронічного
C. Поліпозного
D. Гіпертрофічного
E. @Виразкового

56. Під час розтину тіла померлого в прямій та сигмоподібній кишках видно дефекти слизової оболонки
неправильної форми з нерівними контурами, вони зливаються між собою, залишаючи невеликі острівці
збереженої слизової оболонки. Про який різновид коліту йдеться?
A. @Виразковий
B. Фібринозний
C. Гнійний
D. Фолікулярний
E. Катаральний

57. У товстій кишці при колоноскопії виявлено дефект слизистої оболонки діаметром 3,5 см з нерівним
горбистим дном, з нерівними краями; межа росту нечітка. Тканина дна і країв дефекту щільна, білувата; шари
стінки кишки в цій ділянці не помітні. Встановіть макроскопічну форму пухлини.
A. Вузловата
B. Інфільтративна
C. @Виразкова
D. Кістозна
E. Інфільтративно-виразкова форма

58. На розтині тіла чоловіка похилого віку, який впродовж останніх 2-х тижнів страждав на гострий
розлад кишківника, виявлені зміни у прямій та сигмоподібній кишках: на поверхні слизової оболонки
спостерігається коричнево-зелена плівка. Стінка кишки потовщена, порожнина різко звужена. Мікроскопічно
виявляється некроз, що проникає на різну глибину слизової оболонки, некротичні маси пронизані нитками
фібрину, з лейкоцитарною інфільтрацією. Який з перерахованих діагнозів найбільш імовірний?
A. @Фібринозний коліт
B. Катаральний коліт
C. Виразковий коліт
D. Фолікулярний коліт
E. -

59. У біоптаті товстої кишки - поверхневий дефект слизової оболонки, зниження числа келихоподібних
клітин і кількості слизу в них, різка лімфоплазмоцитарна інфільтрація з наявністю сегментоядерних лейкоцитів,
зокрема еозинофілів. Виберіть найбільш вірогідний і точний діагноз.
A. @Неспецифічний виразковий коліт у фазі загострення
B. Хвороба Крона
C. Хронічний ішемічний коліт
D. Дизентерія в третій стадії
E. Амебіаз кишечника з формуванням виразок

60. 54-річній жінці проведено лівобічну геміколектомію з приводу перфорації кишки. При гістологічному
дослідженні операційного матеріалу виявлено множинні епітеліоїдно-клітинні гранульоми, щілиноподібні
виразки, виражений набряк підслизового шару. Діагностуйте захворювання.
A. @Хвороба Крона
B. Туберкульоз кишки
C. Неспецифічний виразковий коліт
D. Хвороба Уіпла
E. Лімфома кишки

61. У товстій кишці під час колоноскопії виявлено дефект слизової оболонки діаметром 3,5 см з нерівним
горбистим дном, нерівними краями, що підвищуються над дном на 1,7 см; межа цього підвищення нечітка.
Тканина дна і країв дефекту щільна, білувата, шари стінки кишки у цій ділянці не розрізняються. Встановіть
макроскопічну форму пухлини:
A. @Виразка
B. Вузол
C. Інфільтрат
D. Кіста
E. Інфільтративно-виразкова форма

62. При лапаротомії чоловіка, 49 років, виявлено пухлину в ділянці сигми, що проростає усі її шари та
обтурує просвіт кишки. Який характер росту у виявленої пухлини по відношенню до оточуючих тканин?
A. @Інфільтруючий
B. Експансивний
C. Ендофітний
D. Екзофітний
E. Мультицентричний

63. Під час мікроскопічного дослідження біоптату з низхідного відділу товстої кишки діагностовано
пухлину з призматичного епітелію, яка формує атипові залозисті структури різної форми та величини. Клітини
поліморфні, ядра гіперхромні, патологічні мітози. Базальна мембрана залоз порушена. Ваш діагноз.
A. Базально-клітинний рак
B. Солідний рак
C. @Аденокарцинома
D. Слизовий рак
E. Недиференційований рак

ХВОРОБИ ПЕЧІНКИ

1. Хворого, 33 років, доставлено у відділення інфекційної реанімації після вживання у їжу грибів.
Загальний стан важкий, артеріальний тиск знижений. При наростанні гепато-ренальної недостатності він помер.
На розтині печінка зменшена за розмірами, дрябла, жовтого кольору. Поставити діагноз:
A. @Гострий гепатоз
B. Хронічний жировий гепатоз
C. Гепатит
D. Цироз
E. Хронічне венозне повнокров’я

2. При розтині тіла чоловіка, 49 років, який поступив у стаціонар з проявами гепатотропної інтоксикації і
раптово помер: печінка збільшена, дрябла, жовто-коричневого кольору; на поверхні розрізу печінки і лезі ножа
помітні краплі жиру. Мікроскопічно: гепатоцити периферії істинних печінкових часточок містять масу дрібних
крапель жиру, які переповнюють цитоплазму і зрушують ядро на периферію. Який процес найімовірніше має
місце в печінці?
A. @Жирова дистрофія
B. Хвороба Гоше
C. Хвороба Німанна-Піка
D. Хвороба Тея-Сакса
E. Хвороба Нормана-Ландінга

3. Чоловік, 60 років, був госпіталізований до стаціонару з діагнозом «Отруєння грибами», де і помер на


12-й день з ознаками гострої печінкової недостатності. На секції - макроскопічно: печінка дрябла, жовто-сірого
кольору, глинистого вигляду як на поверхні, так і на розрізі; мікроскопічно: ділянки некрозу з аутолітичним
розпадом і утворенням жиро-білкового детриту в центрі та вузькою смужкою гепатоцитів у стані жирової
дистрофії на периферії печінкових часточок. Який найбільш імовірний діагноз?
A. @Токсична дистрофія печінки в стадії жовтої дистрофії
B. Токсична дистрофія печінки в стадії червоної дистрофії
C. Хронічна токсична дистрофія печінки
D. Хвороба Вільсона-Коновалова
E. Спадковий пігментний гепатоз
4. При розтині людини, яка померла від отруєння грибами встановлено, що печінка зменшена, капсула її
зморщена, на розрізі тканина печінки червоного кольору. Мікроскопічно: в паренхімі органа - жиро-білковий
детрит, синусоїди переповнені кров’ю. Який діагноз витікає з наведеного?
A. @Токсична дистрофія печінки (стадія червоної дистрофії)
B. Токсична дистрофія печінки (стадія жовтої дистрофії)
C. Постнекротичний цироз печінки
D. Хронічний активний гепатит
E. Портальний цироз печінки
5. Хворий, 55 років, довгий час зловживає алкоголем. Останнім часом з’явився біль у правому підребер’ї
розміри печінки збільшилися. При досліджені біоптату печінки виявлені каблучкоподібні гепатоцити, які
розміщені окремими ділянками. Який діагноз можна передбачити?
A. @Жировий гепатоз (стеатоз печінки)
B. Гострий гепатит
C. Токсична дистрофія печінки
D. Рак печінки
E. Біліарний цироз печінки

6. У чоловіка при обстеженні виявлено порушення синтезу жовчних кислот, зниження каталазної
активності печінки. При електронномікроскопічному дослідженні біоптату печінки виявлено: відсутність
пероксисом в гепатоцитах, зменьшення числа мітохондрій, редукцію ендоплазматичної сітки, накопичення в
цитоплазмі глікогену та ліпідів. Який з перелічених діагнозів найбільш вірогідний?
A. @Синдром Целвегера
B. Акаталаземія
C. Системна недостатність карнітину
D. Хвороба Вільсона-Коновалова
E. Синдром Картагенера
7. У пункційному біоптаті печінки виявлені: дистрофія гепатоцитів з некрозами, а також склероз з
порушеннями балкової та часточкової будови, з утворенням несправжніх часточок і регенераторних вузлів.
Оберіть найбільш вірогідний діагноз:
A. @Цироз печінки
B. Хронічний гепатоз
C. Хронічний гепатит
D. Прогресуючий масивний некроз печінки
E. Гострий гепатит
8. Біопсія печінки хворого з підозрою на гепатит показала наявність дистрофії та некрозу гепатоцитів,
спотворення їх регенерації, широкі смужки склерозу, порушення часточкової структури печінки. Про яке
захворювання можна думати?
A. @Цироз печінки
B. Хронічний персистуючий гепатит
C. Хронічний активний гепатит
D. Токсична дистрофія печінки
E. Гострий некротичний гепатит

9. Хворий, 76 років, страждав на бронхоектатичну хворобу та пневмосклероз, помер від прогресуючої


серцево-судинної недостатності. На розтині печінка збільшена за розмірами, щільна, поверхня її дрібнозерниста,
край нерівний. Визначити вид цирозу печінки залежно від етіології:
A. Інфекційний
B. Токсичний
C. Біліарний
D. Обмінно-аліментарний
E. @Циркуляторний
10. Чоловік, 59 років, мав ознаки паренхіматозної жовтяниці та портальної гіпертензії. Під час
гістологічного дослідження пункційного біоптату печінки було знайдено: балково-часточкова будова порушена,
частина гепатоцитів має ознаки жирової дистрофії, формуються порто-портальні сполучнотканинні септи,
перипортально розташований лімфо-макрофагальний інфільтрат. Діагностуйте захворювання печінки.
A. @Портальний цироз
B. Гострий гепатоз
C. Хронічний гепатоз
D. Біліарний цироз
E. Постнекротичний цироз

11. Хворий, який страждав на хронічний вірусний гепатит, помер від гострої постгеморагічної анемії, яка
виникла на тлі кровотечі з варикозно розширених вен стравоходу. На розтині - печінка різко зменшена за
розмірами, щільної консистенції, поверхня дрібногорбиста. Мікроскопічна картина однорідна – тонкопетлиста
сполучнотканинна мережа і дрібні, неправильної будови часточки. Який морфогенетичний тип цирозу має місце:
A. Постнекротичний
B. Змішаний
C. @Портальний
D. Вірусний
E. Біліарний

12. При розтині померлого чоловіка, який тривалий час зловживав алкоголем - печінка малих розмірів,
щільна, дрібногорбиста. Мікроскопічно: несправжні часточки, дрібні, розподілені вузькими пучками сполучної
тканини, інфільтрованої лімфоцитами та макрофагами; гепатоцити у стані великокрапельного ожиріння. Який з
перелічених діагнозів найбільш вірогідний?
A. @Портальний цироз печінки
B. Хронічний активний алкогольний гепатит
C. Хронічний персистуючий алкогольний гепатит
D. Постнекротичний цироз печінки
E. Жировой гепатоз

13. У хворого, 67 років, який тривало страждав на жовчно-кам'яну хворобу з ознаками холангіту,
розвинувся цироз печінки. До якого з нижчеперерахованих видів цирозу він відноситься?
A. Інфекційного
B. Токсичного і токсико-алергічного
C. Обмінно-аліментарного
D. Циркуляторного
E. @Біліарного
14. Пiд час розтину тiла жiнки, 52 рокiв, яка тривалий час хворiла на жовчно-кам’яну хворобу,
макроскопiчно було виявлено: печiнка помiрно збiльшена, деформована, поверхня органа горбиста, тканина
щiльна, на розрiзi тканина коричнева з зеленим вiдтiнком, складається з множинних вузликiв дiаметром 8-10 мм.
Мiкроскопiчно - гепатоцелюлярнi вузлики оточенi прошарками сполучної тканини, яка мiстить збiльшену
кiлькiсть дрiбних жовчних проток з холестазом. Дiагностуйте захворювання печінки.
A. @Бiлiарний цироз печiнки
B. Холелiтiаз
C. Токсична дистрофiя печiнки
D. Портальний цироз печiнки
E. Постнекротичний цироз печінки

15. Хворій, 70 років, з підозрою на гострий холецистит, проведено лапаротомію. При ревізії органів
черевної порожнини був виставлений клінічний діагноз: «Біліарний цироз печінки». Які макроскопічні зміни
характерні для цього цирозу?
A. @Печінка збільшена, щільна, зеленого кольору з дрібновузловою поверхнею
B. Печінка велика, червона
C. Печінка жовта, дрябла
D. Печінка зменшена, щільна, з великими вузлами
E. Печінка щільна, бліда

16. Хворий, 25 років, звернувся до лікаря зі скаргами на пожовтіння склер, схуднення, відсутність
апетиту, свербіж. При дослідженні біоптату печінки виявлено: колапс строми, широкі поля фіброзу, зближення
тріад (в одному полі зору більше трьох), білкова дистрофія та некроз гепатоцитів, структурна перебудова
печінки. Про яке захворювання печінки можна думати?
A. @Постнекротичний цироз печінки
B. Хронічний активний гепатит
C. Біліарний цироз печінки
D. Токсична дистрофія печінки
E. Рак печінки

17. При розтині тіла чоловіка, який тривалий час зловживав алкоголем - печінка малих розмірів, щільна,
дрібногорбриста. Мікроскопічно: псевдочасточки дрібні, розподілені вузькими прошарками сполучної тканини з
лімфо-макрофагальними інфільтратами; гепатоцити в стані великокрапельної жирової дистрофії. Який з
перелічених діагнозів найбільш вірогідний?
A. @Алкогольний цироз
B. Хронічний активний алкогольний гепатит
C. Хронічний персистуючий алкогольний гепатит
D. Токсична дистрофія печінки
E. Жировий гепатоз

18. У хворого, 50 рокiв, виявленi: жовтяниця, дефанс (напруження м’язiв) у правому верхньому квадрантi
живота, судиннi зiрочки на шкiрi та асцит. Медичнi препарати не приймає. Зловживає алкоголем. Що найбiльш
вiрогiдно виявлятиметься при обстеженнi пацiєнта?
A. @Тiльця Малорi при бiопсiї печiнки
B. Напруженi яремнi вени
C. АлАТ набагато вище за АсАТ
D. Швидке клiнiчне одужання після припинення прийому алкоголю
E. Спленомегалiя

19. Хворого, 48 років, який страждає на алкоголізм, госпіталізовано зі скаргами на жовтушність шкіри,
тупий біль у правому підребер’ї, нудоту, відрижку, схуднення та підвищення температури. В біохімічному аналізі
крові - підвищення активності трансаміназ, гіпербілірубінемія, гіпергамаглобулінемія. При пункційній біопсії
встановлено активний хронічний гепатит, що характеризувався наявністю в цитоплазмі гепатоцитів та
екстрацелюлярно великої кількості еозинофільних тілець (алкогольний гіалін). Визначити назву тілець при
алкогольному гепатиті:
A. Тільця Руселя
B. Тільця Каунсильмена
C. @Тільця Малорі
D. Тільця Ерліха
E. Тільця Негрі

20. Хворий поступив у стаціонар з клінікою печінкової недостатності, де і помер. Печінка на секції
щільна, дещо зменшена, з горбкуватою поверхнею. Спостерігаються однакових розмірів (здебільшого діаметром
до 0,3 см) вузлики-регенерати розділені вузькими септами сполучної тканини. Про яку макроскопічну форму
патології печінки іде мова?
A. @Дрібновузловий цироз
B. Фіброз печінки
C. Великовузловий цироз
D. Мускатний фіброз печінки
E. Змішаний (дрібно-великовузловий) цироз

21. Хворий, 18 років, через 2 місяці після вірусного гепатиту помер від прогресуючої гепатаргії. На
розтині - печінка щільна, зменшена за розмірами, поверхня вузлувата, нагадує бруківку, на поверхні розрізу -
вузли (5-20 мм) жовтувато-коричневого кольору, відмежовані прошарками сірого кольору. При гістологічному
дослідженні – у великих полях сполучної тканини розташовуються по декілька тріад (міжчасточкові вена, артерія
і жовчна протока) та хибні часточки з багатоядерними гепатоцитами. В збережених часточках - білкова дистрофія
та некробіоз гепатоцитів, осередкові прояви холестазу . Визначити основну причину смерті хворого:
A. Хронічний гепатит
B. Первинний біліарний цироз печінки
C. Портальний цироз печінки
D. Вторинний біліарний цироз
E. @Постнекротичний цироз печінки

22. Пацiєнт звернувся зi скаргами на гострий бiль у правому пiдребер’ї. При оглядi лiкар звернув увагу на
пожовтiння склер хворого. Лабораторно: пiдвищена активнiсть АлАТ та негативна реакцiя на стеркобiлiн у калi.
Для якого захворювання характернi такi симптоми?
A. @Гепатит
B. Гемолiтична жовтяниця
C. Хронiчний гастродуоденiт
D. Хронiчний колiт
E. Хронiчний гастрит

23. Хворого госпіталізовано у стаціонар зі скаргами на біль в правому підребір’ї, жовтяницю, нудоту.
Захворів після вживання надмірної кількості алкоголю. Після обстеження хворого був встановлений діагноз:
«Гострий алкогольний гепатит». Визначте типовий наслідок хвороби, якщо пацієнт припинить вживати алкоголь.
A. @Видужання
B. Цироз печінки
C. Розвиток раку печінки
D. Гемосидероз печінки
E. Гемохроматоз печінки

24. У померлого хворого, який прижиттєво отримував чисельні ін’єкції наркотиків, в печінці гістологічно
виявлено гідропічну дистрофію гепатоцитів, “матовосклоподібні гепатоцити”, ацидофільні тільця Каунсильмена,
лімфоцитарно-макрофагальну інфільтрацію в портальних трактах. Найбільш вірогідна етіологія захворювання:
A. @Вірусна
B. Бактеріальна
C. Токсична
D. Паразитарна
E. Грибкова
25. Проведено пункційну біопсію печінки хворому, 38 років, з вираженою жовтяницею, дрібними
крововиливами у шкірі, загальною слабкістю, втратою апетиту. Гістологічне дослідження виявило поширену
дистрофію, некроз гепатоцитів, наявність тілець Каунсильмена. По периферії часточок - значна інфільтрація
лімфоцитами, зустрічаються окремі багатоядерні гепатоцити. Діагностуйте захворювання.
A. @Гострий вірусний гепатит
B. Гострий алкогольний гепатит
C. Міліарний цироз печінки
D. Токсична дистрофія печінки
E. Хронічний гепатит

26. При пункційній біопсії печінки хворого з клінікою печінково-клітинної недостатності виявлені:
вакуольна, балонна дистрофія гепатоцитів, некроз окремих клітин, тільця Каунсильмена, інфільтрація портальної
та часточкової строми переважно лімфоцитами та макрофагами з незначною кількістю поліморфноядерних
лейкоцитів. Який найбільш вірогідний діагноз?
A. Гострий вірусний гепатит
B. Хронічний персистуючий гепатит
C. Хронічний активний гепатит
D. Аутоімунний гепатит
E. Алкогольний гепатит

27. Хворий, 18 років, наркоман, помер в інфекційному відділенні від септичної пневмонії. В аналізі крові -
позитивна реакція на антитіла (Ig M) до РНК вірусного гепатиту типу С. На розтині – жовтуха, набряк головного
мозку, діапедезні крововиливи у внутрішні органи та в місцях ін’єкцій. В печінці – ознаки гострого гепатиту, що
характеризувався утворенням еозинофільних тілець з пікнотичним ядром або без ядра (“муміфіковані”,
некротизовані гепатоцити). Визначити назву цих клітин:
A. Тільця Руселя
B. @Тільця Каунсильмена
C. Тільця Малорі
D. Тільця Ерліха
E. Тільця Негрі

28. Хворого, 22 років, госпіталізовано в клініку зі скаргами на невизначений біль в правому підребер’ї,
диспепсичні явища, втрату ваги. За даними анамнезу в 20 років перехворів вірусним гепатитом В. У біоптаті
печінкової тканини виявлено незворотну внутрішньоклітинну зміну, що є типовою для перенесеного вірусного
гепатиту В. Назвіть цю зміну ?
A. @Тільця Каунсильмена
B. Гідропічна дистрофія
C. Тьмяний набряк печінкових клітин
D. Внутрішньоклітинне ожиріння
E. Скупчення в ядрах клітин глікогену

29. При гістологічному дослідженні біоптату печінки чоловіка з проявами печінкової недостатності
виявлено порушення будови часточок, алкогольний кератогіалін та балонну дистрофію гепатоцитів, їх некроз на
периферії часточок. Крім того спостерігається дифузна гістіолімфоцитарна інфільтрація склерозованих
портальних трактів, яка проникає на периферію часточок, оточуючи і руйнуючи гепатоцити. Який з перелічених
діагнозів найбільш вірогідний?
A. @Хронічний активний алкольний гепатит
B. Гострий алкогольний гепатит
C. Гостра циклічна форма вірусного гепатиту
D. Холестатична форма вірусного гепатиту
E. Хронічний перситуючий алкогольний гепатит

30. Хвора, 25 років, наркоманка, інфікована вірусом імунодефіциту людини, померла від сепсису. На
розтині: печінка збільшена за розмірами, нерівномірної щільності, поверхня місцями дрібнозерниста. При
гістологічному дослідженні: часточкова структура та погранична пластинка збережені, в гепатоцитах - дистрофія,
між часточками - дифузна інфільтрація лімфоцитами, гістіоцитами, плазмоцитами та розростання волокнистої
сполучної тканини. В окремих часточках зустрічаються гістіолімфоцитарні інфільтрати, проліферація
ретикулоцитів та осередки склерозу ретикулярної строми. Визначити форму гепатиту:
A. Гострий ексудативний
B. Гострий проліферативний
C. Хронічний активний
D. @Хронічний персистуючий
E. Хронічний холестатичний

31. Хворий в інфекційному відділенні скаржився на слабкість, відсутність апетиту, підвищення


температури до 38°С. На 7 добу - різкий біль в правому підребер’ї та пожовтіння шкіри. При мікроскопії
біоптату печінки: порушення балочної будови, в гепатоцитах –гідропічна і балонна дистрофія, в деяких
гепатоцитах - некроз, тільця Каунсильмена, на периферії часточок - збільшена кількість багатоядерних
гепатоцитів. Яка форма вірусного гепатиту найбільш вірогідна?
A. @Циклічна, жовтянична
B. Злоякісна
C. Хронічна
D. Холестатична
E. Безжовтянична

32. Хворий, 39 років, страждав на вірусний гепатит типу В і помер від печінкової недостатності. На
розтині: печінка зменшена за розмірами, дрябла, жовтого кольору, капсула зморщена. При гістологічному
дослідженні - масивні некрози паренхіми чергуються з ділянками спадіння ретикулярної строми вздовж
синусоїдів, тільця Каунсильмена, накопичення зірчастих ретикулоцитів, лімфоцитів, макрофагів, нейтрофільних
лейкоцитів. На периферії часточок - гідропічна та балонна дистрофія гепатоцитів. Визначити клініко-
морфологічну форму гепатиту:
A. @Фульмінантна
B. Безжовтушна
C. Холестатична
D. Гостра циклічна
E. Хронічна активна

33. У хворого з підгострим септичним ендокардитом при огляді лікар виявив загальну блідість та
іктеричність шкіри, склер і видимих слизових оболонок. У крові виявлено збільшену кількість непрямого
білірубіну. Жовтий колір шкіри, склер та слизових оболонок - це прояв:
A. @Надпечінкової жовтяниці
B. Жирової дистрофії
C. Гемосидерозу
D. Печінкової жовтяниці
E. Підпечінкової жовтяниці

34. У хворого виявлені: асцит, у два рази збільшена селезінка, варикозне розширення вен стравоходу і
прямої кишки. При гістологічному дослідженні біоптату печінки виявлено мікронодулярний цироз. Який процес
ускладнив цироз печінки?
A. Серцева недостатність
B. Гепато-лієнальний синдром
C. @Синдром портальної гіпертензії
D. Печінково-клітинна недостатність
E. -

35. Швидкою допомогою до приймального відділення доставлено хворого з кривавим блюванням. В


анамнезі - цироз печінки. Пошкодження яких вен найбільш вірогідне у даному випадку?
A. @Стравохідні
B. Верхня брижова
C. Печінкові
D. Нижня брижова
E. Селезінкова

36. Хворий, 48 років, з гострим вірусним гепатитом типу В (блискавична форма) помер від печінкової
коми. На розтині: печінка зменшена за розмірами, капсула зморшкувата, консистенція дрябла, на розрізі
паренхіма з жовтуватим відтінком. Морфологічні зміни, що зумовили печінкову кому:
A. Холангіт
B. @Некроз паренхіми
C. Проліферація ретикулоендотеліоцитів
D. Дистрофія гепатоцитів
E. Склероз

37. Хворий помер при явищах серцево-судинної недостатності. Результати розтину: постінфарктний
кардіосклероз, гіпертрофія міокарда і дилатація його порожнин, особливо правого шлуночка. Печінка збільшена,
з гладкою поверхнею, на розрізі - повнокровна, з темно-червоним крапом на бурому фоні тканини. Гістологічно -
повнокров’я центральних відділів часточок; у периферичних відділах навколо портальних трактів - гепатоцити в
стані жирової дистрофії. Як називаються описані вище зміни печінки?
A. Псевдомускатна печінка
B. Амілоїдоз
C. Цироз печінки
D. @Мускатна печінка
E. Стеатоз печінки

38. У хворого, 46 років, з ревматичною вадою серця (стенозом лівого атріовентрикулярного отвору) -
задишка при незначному фізичному навантаженні, серцебиття, ціаноз губ, вологі хрипи в нижніх відділах легень,
набряки нижніх кінцівок. Які гістологічні зміни будуть характерні для печінки?
A. Некроз гепатоцитів у центрі часточок, гіаліново-крапельна дистрофія на периферії
B. Некроз гепатоцитів у центрі часточок, гідропічна дистрофія на периферії
C. Жирова дистрофія гепатоцитів у центрі часточок, некроз на периферії
D. @Некроз гепатоцитів у центрі часточок, жирова дистрофія на периферії
E. Гідропічна дистрофія гепатоцитів у центрі часточок, некроз на периферії

39. У печінці виявлене округле утворення діаметром 0,5 см. Мікроскопічно воно представлене
некротичними масами в центрі, оточеними плазматичними, лімфоїдними клітинами і кровоносними судинами, з
явищами васкуліту. Який діагноз необхідно поставити на підставі даних мікроскопії?
A. Солітарна аденома печінки
B. @Солітарна гума печінки
C. Солітарна лепрома печінки
D. Хронічний абсцес печінки
E. Рак печінки

40. Хвору, 55 років, прооперовано з приводу перитоніту і видалено жовчний міхур. При гістологічному
дослідженні його стінки виявлено: повнокров’я з діапедезними крововиливами, ворсинки набряклі, слизова
оболонка та підлеглі шари дифузно інфільтровані поліморфно-ядерними лейкоцитами, на поверхні серозної
оболонки - нашарування фібрину. Визначити форму холециститу:
A. Катаральний
B. Фібринозний
C. @Флегмонозний
D. Гангренозний
E. Калькульозний

41. Хвору, 50 років, прооперовано з приводу перитоніту, причиною якого встановлено перфорацію стінки
жовчного міхура. При гістологічному дослідженні його стінки виявлені деструкція ворсинок слизової оболонки,
дифузна лейкоцитарна інфільтрація, крововиливи та великі ділянки некрозу, просочені гемолізованими
еритроцитами, та лейкоцитарна інфільтрація брижі. Визначити форму холециститу:
A. Катаральний
B. Фібринозний
C. Гнійний
D. @Гангренозний
E. Калькульозний
ХВОРОБИ НИРОК
ВАРІАНТ №
1. У жінки відмічались: кровохаркання, гематурія, при імунофлюресцентному дослідженні біоптату
нирки лінійне світіння IgG в базальній мембрані капілярів клубочків. Можливий діагноз:
A. Хронічний мезангіопроліферативний гломерулонефрит
B. Системний червоний вовчак; вовчаковий гломерулонефрит
C. @Синдром Гудпасчера; екстракапілярний продуктивний гломерулонефрит
D. Мієломна хвороба
E. Періодична хвороба, амілоїдоз нирок

2. У хворої жінки спостерігаються: субфебрильна температура, кашель, відходження мокроти з


прожилками крові, задишка, підвищення АТ, зниження діурезу (менше 500 мл на добу), набряки кінцівок.
Клінічно виявлено хронічну ниркову недостатність II ступепя. Дані симптоми розвинулися поступово впродовж 6
тижнів. Для уточнення діагнозу виконано біопсію нирок, після чого поставлено діагноз «Синдром Гудпасчера».
Вкажіть який патологічний процес лежить в основі даного захворювання.
A. @Наявність аутоантитіл до базальної мембрани
B. Наявність аутоантитіл до мітохондрій
C. Наявність аутоантитіл до ДНК
D. Цитотоксична реакція проти епітелію канальців
E. Відкладення імунних комплексів у клубочках

3. Пiсля перенесеної стрептококової iнфекцiї у чоловiка дiагностовано гострий гломерулонефрит.


Найбiльш iмовiрно, що ураження базальної мембрани ниркових тiлець виникає внаслiдок алергiчної реакцiї
такого типу:
A. @Iмунокомплексна
B. Анафiлактична
C. Цитотоксична
D. Сповiльнена
E. Стимулююча

4. У чоловiка, 25 рокiв, дiагностовано гострий дифузний гломерулонефрит. З анамнезу: за 18 днiв до


прояву хвороби перенiс ангiну. Який механiзм ураження ниркових клубочкiв буде спостерiгатися у цьому
випадку?
A. @Iмунний
B. Гідронефроз
C. Нефротоксичний
D. Iшемiчний
E. Медикаментозний

5. У дитини, 15 років, через 14 днів після перенесеної ангіни з’явилися набряки на обличчі зранку,
підвищення артеріального тиску, сеча у вигляді “м’ясних помиїв”. Імуногістохімічне дослідження біоптату нирки
виявило відкладання імунних комплексів на базальних мембранах капілярів та у мезангії клубочків. Яке
захворювання розвинулось у хворого?
A. @Гострий гломерулонефрит
B. Гострий інтерстиційний нефрит
C. Ліпоїдний нефроз
D. Гострий пієлонефрит
E. Некротичний нефроз

6. У хворого, який переніс стрептококову ангіну, з`явилися набряки на обличчі вранці, сеча - кольору
м`ясних помиїв, головний біль. Макроскопічний вигляд нирок:
A. Первинно-зморщені
B. Великі білі
C. @Великі строкаті
D. Вторинно-зморщенні
E. Строкаті

7. На розтині померлого спостерігаємо: нирки збільшені, набряклі, піраміди темно-червоні, кора -


сірувато-коричнева з дрібними крововиливами на поверхні та розрізі. Ваш діагноз.
A. @Гострий гломерулонефрит
B. Підгострий гломерулонефрит
C. Мезангіальний гломерулонефрит
D. Фібропластичний гломерулонефрит
E. Мембранозний гломерулонефрит

8. У добовій сечі хворого знайдені вилужені еритроцити. Для якої патології нирок найбільш притаманний
виявлений симптом?
A. @Дифузний гломерулонефрит
B. Нефротичний синдром
C. Нирковокам’яна хвороба
D. Пієлонефрит
E. Гостра ниркова недостатність

9. Для під гострого гломерулонефриту характерно:


A. Нирки збільшені в розмірах, блідо-рожеві з гладкою поверхнею
B. Нирки зменшені, з дрібними западинами на поверхні, щільні, сіро-червоні
C. Нирки щільні, бліді з жовтими плямами в корковому шарі
D. @Нирки збільшені, дряблі, корковий шар широкий, набряклий, жовто-сірий, тьмяний, з червоним
крапом і добре відмежований від темно-червоної мозкової речовини
E. Нирки збільшені, набряклі, піраміди темно-червоні, кора сірувато-коричнева з дрібними
крововиливами на поверхні та розрізі

10. У хлопчика, 8 років, через 2 тижні після тонзиліту розвинувся нефротичний синдром. В аналізі сечі:
гематурія, циліндрурія, протеїнурія, лейкоцитурія. Діагностовано постстрептококовий гломерулонефрит.
Морфологічний еквівалент це:
A. @Інтракапілярний проліферативний гломерулонефрит
B. Екстракапілярний проліферативний гломерулонефрит
C. Мембранозний гломерулонефрит
D. Фібропластичний гломерулонефрит
E. Гломерулонефрит

11. У хворого, 28 років, виявлялися: підвищення артеріального тиску, гематурія та набряки на обличчі.
Незважаючи на лікування, наростали явища ниркової недостатності. Через 6 місяців хворий помер від уремії.
Мікроскопічно при дослідженні нирок у клубочках виявлені: проліферація нефротелію капсули, подоцитів з
утворенням "півмісяців", склероз та гіаліноз клубочків. Який найбільш вірогідний діагноз?
A. Гострий пієлонефрит
B. Нефротичний синдром
C. Хронічний гломерулонефрит
D. @Підгострий гломерулонефрит
E. Гострий гломерулонефрит

12. При мікроскопічному дослідженні нирок померлої від ниркової недостатності жінки, 36 років, в
клубочках виявлено: проліферацію нефротелію капсули, подоцитів та макрофагів з утворенням “півмісяців”,
некроз капілярних петель, фібринові тромби в їх просвітах, а також склероз та гіаліноз клубочків, атрофію
канальців та фіброз строми нирок. Який з перелічених діагнозів найбільш вірогідний?
A. @Підгострий гломерулонефрит
B. Гострий гломерулонефрит
C. Хронічний гломерулонефрит
D. Фокальний сегментарний склероз
E. Мембранозна нефропатія

13. На аутопсії померлого від ниркової недостатності відмічено, що нирки збільшені, дрялі, широка кора
жовто-сірого кольору з червоним крапом. При мікроскопічному дослідженні виявлено: епітелій капсули
клубочків проліферує з появою “напівмісяців”, капілярні петлі з ділянками некрозу та фібриновими тромбами у
просвітах. Ваш діагноз:
A. @Підгострий гломерулонефрит
B. Гострий гломерулонефрит
C. Ліпоїдний нефроз
D. Хронічний гломерулонефрит
E. Амілоїдоз нирок

14. При розтині померлого від ниркової недостатності спостерігались наступні зміни: нирки збільшені,
дряблі, корковий шар широкий, набряклий, жовто-сірий, тьмяний, з червоним крапом, добре відмежований від
темно-червоної мозкової речовини. Мікроскопічно: проліферація епітелію капсули клубочків, подоцитів та
макрофагів з формуванням „півмісяців”. Для якого захворювання характерні вищезазначені зміни?
A. @Підгострий гломерулонефрит
B. Гострий гломерулонефрит
C. Хронічний гломерулонефрит
D. Гострий пієлонефрит
E. Амілоїдоз нирок

15. Для підгострого гломерулонефриту характерно:


A. Нирки збільшені в розмірах, блідо-рожеві з гладкою поверхнею
B. Нирки зменшені, з дрібними западинами на поверхні, щільні, сіро-червоні
C. Нирки щільні, бліді з жовтими плямами в корковому шарі
D. @Нирки збільшені дряблі, корковий шар широкий, набряклий, жовто-сірий, тьмяний, з червоним
крапом і добре відмежований від темно-червоної мозкової речовини
E. Нирки збільшені, набряклі, піраміди темно-червоні, кора сірувато-коричнева з дрібними
крововиливами на поверхні та розрізі

16. Для якої з перерахованих патології нирок характерно утворення півмісяців в клубочках
A. Ліпоїдний нефроз
B. Фокальний сегментарний гломерулярний гіаліноз
C. Мембранозна нефропатія
D. Гострий постстрептококовий гломерулонефрит
E. @Підгострий гломерулонефрит

17. До підгострого гломерулонефриту відносяться:


A. Інтракапілярний продуктивний
B. @Екстракапілярний проліферативний
C. Мезангіопроліферативний
D. Мезангіокапілярний
E. Фібропластичний

18. Для екстракапілярного продуктивного гломерулонефриту характерні всі перераховані зміни, за


виключенням:
A. Проліферація нефротелію та подоцитів з утворенням півмісяців
B. @Вузли Кімельстила-Уілсона
C. Некроз капілярних петель клубочка
D. Білкова дистрофія канальцевого епітелію
E. Фібрин у просвіті капсули клубочка

19. На розтині прозектор знайшов зміни в нирках: збільшення в розмірах, корковий шар широкий, жовто-
сірий з червоним крапом, добре відмежований від мозкової речовини, яка має темно-червоний колір. При
гістологічному дослідженні - розростання нефротелію і подоцитів в капсулі клубочків з утворенням “півмісяців”,
явища склерозу і гіалінозу клубочків, фіброз строми. Про яке захворювання нирок йде мова?
A. @Екстракапілярний продуктивний гломерулонефрит
B. Інтракапілярний продуктивний гломерулонефрит
C. Екстракапілярний ексудативний гломерулонефрит
D. Інтракапілярний ексудативний гломерулонефрит
E. Інтерстиційний нефрит
20. У молодої особи впродовж року прогресувала ниркова недостатність зі смертельним наслідком. При
розтині виявлені великі строкаті нирки з червоними дрібними краплинами в жовто-сірому корковому шарі.
Гістологічно у клубочках виявлені "напівмісяці" з проліферуючого нефротелію. Ваш діагноз:
A. @Швидкопрогресуючий гломерулонефрит
B. Амілоїдоз
C. Крововилив у нирки
D. Гнійний нефрит
E. Рак нирки

21. Хворий, поступив до лікарні зі скаргами на головний біль, запаморочення. Впродовж останнього року
визначалися високі показники артеріального тиску. Лікарські засоби майже не допомагали. У біоптаті нирки
виявлено екстракапілярну проліферацію з утворенням “півмісяців” Для якого захворювання це характерно?
A. @Швидкопрогресуючий гломерулонефрит
B. Гострий гломерулонефрит
C. Вовчаковий гломерулонефрит
D. Гранулематоз Вегенера
E. Хронічний гломерулонефрит

22. При мікроскопічному дослідженні нирок спостерігається проліферація нефротелію капсули Боумена,
подоцитів і макрофагів з утворенням в її просвіті “півмісяцевих” структур, які стискують клубочок. Капіляри
клубочків зазнають некрозу, в їх просвіті спостерігаються фібринові тромби. Деякі клубочки склерозуються або
гіалінізуються. Спостерігається виражена дистрофія нефроцитів, набряк та інфільтрація строми нирок. Назвіть
патологію нирок.
A. @Швидкопрогресуючий гломерулонефрит
B. Післяінфекційний гломерулонефрит
C. Хронічний гломерулонефрит
D. Хронічний пієлонефрит
E. Амілоїдоз нирок

23. Чоловік, 62 років, помер від хронічної ниркової недостатності. На секції виявлено: нирки зменшені,
щільної консистенції, фіброзна капсула знімається важко, поверхня грубозерниста, кіркова речовина стоншена,
тканина тьмяна. Гістологічно: в частині клубочків - проліферація епітелію капсули Боумена з напівмісяцевими
утвореннями, інші клубочки гіалінізовані. В стромі - нечисленні лімфоїдні інфільтрати. Стінки судин
гіперплазовані, просвітии звужені. Поставте діагноз:
A. @Хронічний гломерулонефрит
B. Артеросклеротичний нефросклероз (первинно зморщена нирка)
C. Атеросклеротично зморщена нирка
D. Гострий екстракапілярний ексудактивний гломерулонефрит
E. Підгострий екстракапілярний продуктивний гломерулонефрит

24. Для мезангіального гломерулонефриту характерно:


A. Нирки збільшені в розмірах, блідо-рожеві з гладкою поверхнею
B. Нирки зменшені, з дрібними западинами на поверхні, щільні, сіро-червоні
C. @Нирки щільні, бліді з жовтими плямами в корковому шарі
D. Нирки збільшені дряблі корковий шар широкий, набряклий, жовто-сірий, тьмяний, з червоним крапом
і добре відмежований від темно-червоної мозкової речовини
E. Нирки збільшені, набряклі, піраміди темно-червоні, кора сірувато-коричнева з дрібними
крововиливами на поверхні та розрізі

25. На розтині нирки зменшені, з дрібними западинами на поверхні, щільні, сіро-червоні. Можливий
діагноз.
A. Гострий гломерулонефрит
B. Підгострий гломерулонефрит
C. Мезангіальний гломерулонефрит
D. @Фібропластичний гломерулонефрит
E. Мембранозний гломерулонефрит

26. У жінки, 30 років, виникли набряки обличчя. При обстеженні виявлені: протеїнурія (5,87 г/л),
гіпопротеїнемія, диспротеїнемія, гіперліпідемія. Для якого стану характерно таке поєднання симптомів?
A. @Нефротичний синдром
B. Нефритичний синдром
C. Хронічний пієлонефрит
D. Гостра ниркова недостатність
E. Хронічна ниркова недостатність

27. Після гострой респіраторно-вірусної інфекції, у дворічного хлопчика розвинулась анасарка. При
проведенні лабораторних досліджень виявлено: значна альбумінурія, гіпопротеїнемія, гіперліпідемія. Зміни
зникли на фоні кортикостероїдної терапії. Вкажіть вірогідне захворювання.
A. @Мінімальні зміни
B. Фокальний сегментарний гломерулярний гіаліноз
C. Мембранозна нефропатія
D. Гострий гломерулонефрит
E. Швидко прогресуючий гломерулонефрит

28. У трирічної дівчинки, після перенесеної гострої респіраторної вірусної інфекції з’явилися
розповсюджені набряки (анасарка), масивна протеїнурія, гіпоальбумінемія, гіперліпідемія. При дослідженні
біоптату нирки виявлено: відсутність малих відростків подоцитів судинних клубочків. Вкажіть вірогідне
захворювання нирок.
A. @Ліпоїдний нефроз
B. Післяінфекційний гломерулонефрит
C. Швидкопрогресуючий гломерулонефрит
D. Фокальний сегментарний гломерулосклероз
E. Мембранозний гломерулонефрит

29. Проявом якого патологічного процесу є зміни в нирках, що називаються «Великі білі нирки»:
A. Підгострий гломерулонефрит
B. @Ліпоїдний нефроз
C. Нефротична стадія амілоїдозу нирок
D. Протеїнурична стадія амілоїдозу нирок
E. Латентна стадія амілоїдозу нирок

30. На розтині: нирки різко збільшені, дряблі, капсула знімається легко, поверхня під капсулою їх
гладка, жовтувата. На розтині корковий шар широкий, жовто-білий, піраміди сіро-червоні. Можливий діагноз.
A. Мембранозний гломерулонефрит
B. @Ліпоїдний нефроз
C. Фокальний сегментарний склероз
D. Склерозуючий гломерулонефрит
E. Екстракапілярний продуктивний гломерулонефрит

31. На розтині нирки збільшені в розмірах, блідо-рожеві з гладкою поверхнею. Можливий діагноз.
A. Гострий гломерулонефрит
B. Підгострий гломерулонефрит
C. Мезангіальний гломерулонефрит
D. Фібропластичний гломерулонефрит
E. @Мембранозний гломерулонефрит

32. У наркозалежного розвинулась протеїнурія. У біоптаті нирки виявлено: облітерація частини


капілярних петель деяких клубочків. Можливий діагноз.
A. Амілоїдоз
B. Півмісяцевий гломеруглонефрит
C. @Фокальний сегментарний гломерулярний гіаліноз
D. Ліпоїдний нефроз
E. Вузликовий дифузний діабетичний гломерулосклероз

33. У результаті порушення техніки безпеки відбулося отруєння сулемою (хлористою ртуттю). Через 2 дні
добовий діурез склав 620 мл. У хворого з’явилися: головний біль, блювання, судоми, задишка, у легенях - вологі
хрипи. Ваш діагноз:
A. @Гостра ниркова недостатність
B. Хронічна ниркова недостатність
C. Уремічна кома
D. Гломерулонефрит
E. Пієлонефрит

34. На розтині тіла жінки, яка померла від значних опіків, виявлено венозну гіперемію інтермедіарної
зони та пірамід при осередковій ішемії коркового шару нирок. Мікроскопічно: капіляри клубочків у стані
спадіння, в епітеліїї канальців - гіаліново-крапельна, гідропічна та жирова дістрофія, в канальцях - циліндри.
Який з перелічених діагнозів найбільш вірогідний?
A. @Гостра ниркова недостатність (шокова стадія)
B. Гостра ниркова недостатність (олігоанурічна стадія)
C. Гострий гломерулонефрит
D. Гострий тубуло-інтерстиціальний нефрит
E. Хронічний гломерулонефрит

35. У чоловіка, 42 років, який хворів на важку форму черевного тифу, розвинулась гостра ниркова
недостатність, від якої він помер. При аутопсії нирки збільшені, набряклі, фіброзна капсула знімається легко; на
розрізі кора – блідо-сіра, піраміди темно-червоні. При гістологічному дослідженні: у більшості канальців просвіт
звужений; епітеліальні клітини збільшені, без ядер; клубочки - колабовані; у стромі - набряк, невелика
лейкоцитарна інфільтрація, дрібні геморагії. Про яку патологію нирок іде мова?
A. @Некротичний нефроз
B. Гострий пієлонефрит
C. Гострий гломерулонефрит
D. Піонефроз
E. Гідронефроз

36. У хворого з кровотечею розвинулась гостра ниркова недостатнiсть, що спричинила смерть. На аутопсiї
макроскопiчно: нирки збiльшенi з широким блiдо-рожевим кiрковим шаром, рiзко вiдмежованим вiд темно-
червоних пiрамiд. Мiкроскопiчно: вiдсутнiсть ядер епiтелiю звивистих канальцiв, тубулорексис, венозний застiй,
ядра клiтин судинних клубочкiв та прямих канальцiв збереженi. Яка патологiя нирок розвинулась ухворого?
A. @Некронефроз
B. Iнфаркт
C. Гломерулонефрит
D. Пiєлонефрит
E. Нефроз

37. При розтинi тiла чоловiка, померлого вiд опiкової хвороби, виявлено: набряк головного мозку,
збiльшення печiнки, а також нирок, кiрковий шар яких широкий, блiдо-сiрий, мозковий - повнокровний.
Мiкроскопiчно: некроз епiтелiю канальцiв головних вiддiлiв з деструкцiєю базальних мембран, набряк
iнтерстицiю з лейкоцитарною iнфiльтрацiєю та крововиливами. Який з перелiчених дiагнозiв найбiльш
iмовiрний?
A. @Некротичний нефроз
B. Тубулоiнтерстицiальний нефрит
C. Пiєлонефрит
D. Подагрична нирка
E. Мiєломна нирка

38. На аутопсії померлого від отруєння етиленгліколем: нирки дещо збільшені, набряклі, їх капсула
знімається дуже легко, кіркова речовина широка, блідо-сіра, мозкова речовина - темно-червона. Яка патологія
нирок розвинулась у хворого?
A. @Некротичний нефроз
B. Гострий пієлонефрит
C. Гострий гломерулонефрит
D. Гострий тубуло-інтерстиціальний нефрит
E. Ліпоїдний нефроз

39. У хворого, який знаходиться у клініці з ознаками отруєння ртуттю, у нирках відмічаються наступні
процеси: осередкові некротичні зміни канальців головних відділів, набряк, лейкоцитарна інфільтрація та
геморагії інтерстицію, венозний застій. Який стан розвинувся у хворого?
A. @Гострий некротичний нефроз
B. Гострий гломерулонефрит
C. Хронічна ниркова недостатність
D. Гострий пієлонефрит
E. Хронічний пієлонефрит

40. У працівника шкіряного виробництва спостерігалось гостре захворювання з явищами гострої ниркової
недостатності. На 5-ту добу хворий помер. При аутопсії: в нирках, крім некротичних змін канальців з
деструкцією базальних мембран і великою кількістю "циліндрів", виявлено масивні відкладення солей кальцію в
ділянках некрозу. Який з перелічених діагнозів найбільш вірогідний?
A. @"Сулемова нирка"
B. Гемолітична нирка
C. Ураження нирок сульфаніламідами
D. Сечокам’яна хвороба
E. Ураження антифризом

41. При мікроскопічному дослідженні нирок виявлено некроз епітелію звивистих канальців,
тубулорексис, набряк строми, ішемія коркового шару і повнокров’я мозкового. Назвіть цей патологічний процес.
A. @Некротичний нефроз
B. Гломерулонефрит
C. Пієлонефрит
D. Полікістоз
E. Сечокам’яна хвороба

42. На аутопсії померлої, яка тривалий час хворіла на цистіт і дискінезію сечоводів, виявлено
морфологічні ознаки уремії. Нирка була нерівномірно рубцево-зморщена. У просвіті мисок містилися дрібні
уратні камені і пісок. Гістологічно виявлено “щитовидну нирку”, осередки інтерстиційного запалення. Який з
нижчеперерахованих діагнозів є найбільш вірогідний?
A. @Хронічний пієлонефрит
B. Гострий пієлонефрит
C. Атеросклеротично зморщена нирка
D. Первинно зморщена нирка
E. Амілоїдно зморщена нирка

43. На розтині жінки, 56 років, яка померла від ниркової недостатності, нирки нерівномірно зменшені за
розмірами, поверхня їх великогорбиста; на розрізі - ділянки рубцевої тканини чергуються з незміненою
паренхімою; миски розширені, стінки їх потовщені. Мікроскопічно в стінках мисок, чашок та в інтерстиції –
явища склерозу та лімфо-плазмоцитарної інфільтрації. Який з перелічених діагнозів найбільш вірогідний?
A. @Хронічний пієлонефрит
B. Гострий пієлонефрит
C. Гострий гломерулонефрит
D. Тубулоінтерстиціальний нефрит
E. Хронічний гломерулонефрит

44. При розтині тіла жінки, яка померла від уремії: нирки різних розмірів, поверхня їх великогорбиста,
між нирковою поверхнею та капсулою нирок – щільні спайки. Мікроскопічно в нирковой тканині виявляются
інкапсульовані абсцеси, розростання сполучної тканини з лімфогістіоцитарною інфільтрацією, осередки
метаплазії перехідного епітелію в багатошаровий, дистрофія та атрофія канальців. Який з перелічених діагнозів
найбільш вірогідний?
A. @Хронічний пієлонефрит
B. Хронічний гломерулонефрит
C. Гострий пієлонефрит
D. Гострий гломерулонефрит
E. Тубулоінтерстиціальний нефрит

45. Хворому з артеріальною гіпертензією, змінами в аналізах сечі і періодичними болями в лівій
поперековій області з діагностичною метою зробили біопсію нирки. При гістологічному дослідженні біоптату
виявлено розширення більшості канальців, різного ступеня хронічне інтерстиціальне запалення і фіброз у
кірковому і мозковому шарі. Велика кількість клубочків зберігає звичайну гістоструктуру. Вкажіть найбільш
вірогідний діагноз.
A. @Хронічний пієлонефрит
B. Хронічний гломерулонефрит
C. Гострий гломерулонефрит
D. Артеріолосклеротичний нефросклероз
E. Нефротичний синдром

46. У бiоптатi нирки чоловiка з хронiчною хворобою нирок виявлено: склероз, лiмфо-плазмоцитарна
iнфiльтрацiя стiнок мисок та чашок, дистрофiя та атрофiя канальцiв. Збереженi канальцi розширенi, розтягненi
колоїдоподiбними масами, епiтелiй сплющений (”щитоподiбна” нирка). Який дiагноз найбiльш iмовiрний?
A. @Хронiчний пiєлонефрит
B. Тубуло-iнтерстицiальний нефрит
C. Гострий пiєлонефрит
D. Гломерулонефрит
E. Нефросклероз

47. У біоптаті нирки: склероз, лімфо-плазмоцитарна інфільтрація стінок мисок та чашок; дистрофія та
атрофія канальців. Збережені канальці розширені, розтягнені колоїдоподібними масами, епітелій сплюснутий
(“щитовидна” нирка). Який діагноз найбільш вірогідній?
A. @Хронічний пієлонефрит
B. Гострий пієлонефрит
C. Гломерулонефрит
D. Нефросклероз
E. Тубуло-інтерстиціальний нефрит

48. Під час розтину трупа жінки, 45 років, яка впродовж тривалого часу страждала на хронічний
гломерулонефрит, виявлено: нирки розмірами 7х3х2,5 см, масою 65 г, щільні, дрібнозернисті. У внутрішніх
органах спостерігаються дистрофічні зміни, у головному мозку – набряк, у серозних і слизових оболонках –
фібринозне запалення. Це:
A. @Вторинно зморщені нирки
B. Первинно зморщені нирки
C. Амілоїдно зморщені нирки
D. “Мієломна” нирка
E. “Подагрична” нирка

49. У хворого, який помер від хронічної ниркової недостатності на фоні хронічного гломерулонефриту,
нирки зменшені за розмірами, щільної консистенції, капсула знімається важко, оголяючи зернисту поверхню. На
розрізі: кірковий та мозковий шари тонкі, тканина нирок суха, недокрівна, сірого кольору. Ваш діагноз:
A. @Вторинно-зморщені нирки
B. Первинно-зморщені нирки
C. Атеросклеротично-зморщені нирки
D. Амілоїдно-зморщені нирки
E. Пієлонефротично-зморщені нирки

50. У хворої, 62 років, яка впродовж 14 років хворіла на хронічний гломерулонефрит, на розтині
виявлені: фібринозне запалення серозних та слизових оболононок, дистрофічні зміни міокарда та печінки, набряк
легень, маленькі, щільні, дрібнозернисті нирки. Причина смерті:
A. Дистрофія міокарда
B. Дифтеретичний коліт
C. Фібринозна пневмонія
D. Фібринозний перекардит
E. @Уремія

51. Хворий, який страждав на хронічний гнійний остеомієліт, помер від хронічної ниркової недостатності.
При розтині знайдені великі щільні нирки біло-жовтого кольору з сальним блиском на розрізі. Ваш діагноз:
A. @Амілоїдоз нирок
B. Хронічний гломерулонефрит
C. Підгострий гломерулонефрит
D. Септичний нефрит
E. Гострий некротичний нефроз

52. Хворий на фіброзно-кавернозний туберкульоз легень помер від хронічної легенево-серцевої


недостатності. Впродовж останніх місяців спостерігалась виражена протеїнурія. На розрізі: нирки збільшені,
щільні, восковидні поверхні і на розрізі. Які зміни в нирках при цій формі туберкульозу могли викликати
протеїнурію?
A. @Амілоїдоз нирок
B. Казеозний некроз
C. Гломерулонефрит
D. Нефролітіаз
E. Некротичний нефроз

53. Хворий на фіброзно-кавернозний туберкульоз помер при зростаючих явищах ниркової недостатності.
На розтині – запах сечі, гіпертрофія лівого шлуночка, фібринозний перикардит, фібринозно-геморагічний
ентероколіт. Нирки дещо зменшені за розмірами, дуже щільні, з множинними втягненнями. Гістологічно на
препаратах, забарвлених конго-рот - рожеві маси у клубочках та стінках судин, загибель і атрофія більшості
нефронів, нефросклероз. Дайте характеристику нирки при даній патології.
A. @Амілоїдно зморщені нирки
B. Первинно зморщені нирки
C. Вторинно зморщені нирки
D. Атеросклеротично зморщені нирки
E. Пієлонефритично зморщені нирки

54. Для латентної стадії амілоїдозу нирок мікроскопічно характерно:


A. Епітелій канальців головних відділів у стані гіаліновокрапельної та гідропічної дистрофії
B. В канальцях виявляються циліндри
C. @В клубочках спостерігається потовщення та двоконтурність капілярних мембран з аневризматичним
розширенням просвіту капілярів
D. В пірамідах та інтермедіарній зоні склероз і амілоїдоз набувають дифузного характеру
E. Канальні розширені, заповнені циліндрами

55. Проявом якого патологічного процесу є зміни в нирках, який зветься «Велика сальна нирка»:
A. Підгострий гломерулонефрит
B. Ліпоїдний нефроз
C. Нефротична стадія амілоїдозу нирок
D. @Протеїнурична стадія амілоїдозу нирок
E. Латентна стадія амілоїдозу нирок

56. Макроскопічні зміни в нирках при нефротичній стадії амілоїдозу нирок:


A. В інтермедіарній зоні та пірамідах строма просякнута білками плазми
B. Нирки збільшені, щільні, з блідо-сірою, або жовто-сірою поверхнею
C. На розрізі корковий шар широкий, матовий, мозкова речовина сіро-рожева, має «сальний» вигляд
D. @Нирки великі, щільні, воскоподібні
E. Помірно зменшені, дуже щільні, з рубцями в корковому шарі

57. Проявом якого патологічного процесу є зміни в нирках, який зветься «Велика біла амілоїдна
нирка»:
A. @Нефротична стадія амілоїдозу нирок
B. Протеїнурична стадія амілоїдозу нирок
C. Латентна стадія амілоїдозу нирок
D. Азотемічна стадія амілоїдозу нирок
E. Ліпоїдний нефроз

58. Проявом якого патологічного процесу є зміни в нирках, який зветься «Амілоїдно-зморщені
нирки»:
A. Нефротична стадія амілоїдозу нирок
B. Протеїнурична стадія амілоїдозу нирок
C. Латентна стадія амілоїдозу нирок
D. @Азотемічна стадія амілоїдозу нирок
E. Ліпоїдний нефроз

59. На розтині померлого хворого виявлено, що нирки збільшені, поверхня їх крупно-горбиста за рахунок
наявності численних порожнин з гладкою стінкою, заповнених прозорою рідиною. Назвіть патологічний процес,
виявлений в нирках.
A. @Полікістоз
B. Некротичний нефроз
C. Пієлонефрит
D. Гломерулонефрит
E. Інфаркт

60. Мікроскопічні зміни в нирках при спадковому нефротичному синдромі:


A. Дифузне потовщення стінок капілярів
B. Базальна мембрана потовщена внаслідок злиття з подоцитами
C. @Кістозне перетворення канальців у поєднанні з аномаліями розвитку нирок
D. Шипики на базальній мембрані
E. Мезангіоцити набувають вигляду пінистих клітин

61. Запис в акті судово-медичного експерта про обстеження трупа потерпілого з тупою травмою обох
нирок, який після травми помер від гострої ниркової недостатності: “В обох сечоводах виявлені червоні тромби
довжиною до 10 см., що обтурують простір сечоводів”. Начальник судово-медичного бюро прочитав і не
погодився з цим записом. Він поступив так тому, що:
A. @У сечоводах не буває тромбів
B. У сечоводах можуть бути лише білі тромби
C. Це тромбоемболи
D. Тромби в сечоводах не бувають обтуруючими
E. У сечоводах можуть бути лише змішані тромби

62. На розтині у померлого хворого виявлено аденому передміхурової залози і великі нирки з різко
збільшеними мисками і чашечками, заповненими прозорою рідиною. Назвіть процес.
A. @Гідронефроз
B. Гломерулонефрит
C. Амілоїдоз
D. Туберкульоз
E. Пієлонефрит

63. При морфологічному дослідженні видаленої нирки виявлено наявність у проксимальному відділі
сечовода конренкременту, який обтурує його просвіт, нирка різко збільшена, перенхіма атрофована, чашки та
миска значно розширені. Мікроскопічно: дифузний склероз, атрофія клубочків та канальців, збережені канальці
кістозно розширені. Назвіть ускладення хворого:
A. @Гідронефроз
B. Піонефроз
C. Пієлонефрит
D. Гломерулонефрит
E. Хронічний паранефрит

64. На розтині померлого від хронічної ниркової недостатності внаслідок нирково-кам’яної хвороби,
виявлено: права нирка представлена тонкостінним мішком, заповненим прозорою рідиною солом’яно-жовтого
кольору, в її просвіті знаходиться конкремент жовто-коричневого кольору, розмірами 6х4х5,6 см, що повторює
форму мисочки і щільно притиснутий до її стінок; ліва нирка дещо збільшена, на розрізі у її чашечках знайдено 3
конкременти жовто-коричневого кольору круглої форми, діаметром по 1,5 см. Вказати назву патологічних змін,
що розвинулися в правій нирці і ускладнили перебіг захворювання.
A. @Гідронефроз
B. Гідрокалікоз
C. Пієлоектазії
D. Піонефроз
E. Нефросклероз

65. На секції виявлено значне збільшення об’єму правої нирки. На розрізі в ній міститься камінь. Просвіт
ниркової миски розтягнутий сечею, яка накопичується. Паренхіма нирки різко стоншена. Який з діагнозів є
найбільш правильним?
A. @Гідронефроз
B. Пієлоектазія
C. Гідроуретронефроз
D. Кіста нирки
E. Пієлонефрит

66. На розтині померлого, віком 62 років, виявлено наступні зміни: шкіра сіро-землистого кольору з
дрібнокраплинними крововиливами, обличчя мов припудрене білуватим порошком, фібринозно-геморагічний
ларингіт, трахеїт, фібринозний перикардит, катаральний гастроентероколіт. Для якого синдрому характерний
даний комплекс морфологічних змін?
A. @Хронічної ниркової недостатності
B. Гострої печінкової недостатності
C. Гострої ниркової недостатності
D. Хронічної серцевої недостатності
E. Гострої надниркової недостатності
67. Жінка, 49 років, тривалий час хворіла на хронічний гломерулонефрит, який призвів до смерті. На
розтині встановлено, що нирки мають розміри 7х3х2.5 см, масу 65,0 г, щільні, дрібнозернисті. Мікроскопічно:
фібринозне запалення серозних і слизових оболонок, дистрофічні зміни паренхіматозних органів, набряк
головного мозку. Яке ускладнення обумовлює вказані зміни?
A. @Уремія
B. Анемія
C. Сепсис
D. ДВЗ-снндром
E. Тромбоцитопенія

68. Який характер має запальний ексудат при розвитку уремічного ураження дихальних шляхів і легень?
A. @Фібринозний
B. Серозний
C. Катаральний
D. Гнійний
E. Гнилісний

69. Чоловік, 55 років, хворів на хронічний гломерулонефрит. Смерть настала при явищах хронічної
ниркової недостатності. На аутопсії виявлено уринозний запах. Макроскопічно: на поверхні епікарда і перикарда
- сірувато-білуваті ворсинчасті нашарування, після зняття яких добре видно різко розширені і переповнені
кров’ю судини. Який процес мав місце в перикарді?
A. @Фібринозне запалення
B. Організація
C. Проліферативне запалення
D. Геморагічне запалення
E. Артеріальне повнокров’я

70. Хворий на хронічний пієлонефрит помер від хронічної ниркової недостатності. Прижиттєво
аускультативно відмічено феномен “шум тертя перикарда”. На розтині виявлено, що епікард тьмяний, шорсткий,
ніби покритий волосяним покривом. Який перикардит за характером запалення має місце?
A. @Крупозний
B. Дифтеритичний
C. Гнійний
D. Гнильний
E. Серозний

71. При розтині чоловіка, 28 років, який хворів на підгострий гломерулонефрит і помер при явищах
прогресуючої уремії, на поверхні перикарда, вісцеральному та пристінковому листках плеври, очеревини
спостерігалися нашарування у вигляді білуватих ниток, які легко відокремлювалися від підлеглої тканини.
Вказати різновид запалення в серозних оболонках.
A. @Крупозне фібринозне запалення
B. Дифтеритичне фібринозне запалення
C. Гнійне запалення
D. Дифузне проліферативне запалення
E. Серозне запалення

72. При пункційній біопсії в трансплантованій нирці виявлені: дифузна інфільтрація строми лімфоцитами,
плазмоцитами, лімфобластами, плазмобластами, а також некротичний артеріїт. Який патологічний процес
розвинувся у трансплантаті?
A. @Імунне відторгнення
B. Гломерулонефрит
C. Iшемічне пошкодження нирки
D. Тубулонекроз
E. Пієлонефрит

73. У чоловіка, 62 років, видалено нирку, у якій при макроскопічному дослідженні виявлено пухлину у
вигляді вузла діаметром до 8 см. Тканина пухлини на розрізі строката, з множинними крововиливами, некрозами.
Гістологічно: пухлина складається зі світлих клітин, які утворюють альвеолярні і сосочкові структури, помірно
виражений інвазивний ріст. У багатьох клітинах пухлини визначаються патологічні мітози, гіперхромні ядра.
Ваш діагноз:
A. @Світлоклітинний рак
B. Світлоклітинна аденома
C. Аденокарцинома
D. Нефробластома
E. Ацидофільна аденома з малігнізацією

ЗАХВОРЮВАННЯ СТАТЕВИХ ОРГАНІВ. ПАТОЛОГІЯ ВАГІТНОСТІ.

ВАРІАНТ №

1. При гістологічному дослідженні передміхурової залози, оперативно видаленої у чоловіка, 72 років,


який скаржився на утруднення сечовипускання, виявлено: збільшення кількості залозистих та м'язових елементів.
Часточкова будова залози порушена. Який процес у передміхуровій залозі найбільш вірогідний?
A. @Змішана форма простатопатії
B. Залозиста гіперплазія
C. М'язово-фіброзна гіперплазія
D. Простатит
E. Аденокарцинома

2. На розтині у померлого, 73 років, виявлено збільшену, м’яку, еластичну, злегка горбкувату


передміхурову залозу, яка на розрізі складається з окремих вузлів, розділених прошарками сполучної тканини.
При мікроскопії відмічено збільшення кількості залозистих елементів. Величина часточок і кількість залозистих
елементів в них - різні. Який процес має місце у передміхуровій залозі?
A. М’язово-фіброзна (стромальна) нодулярна гіперплазія
B. @Залозиста нодулярна гіперплазія
C. Змішана нодулярна гіперплазія
D. Аденокарцинома
E. Недиференційований рак

3. Під час гістологічного дослідження слизової оболонки матки знайдено подовжені, звивисті, пилко- та
штопороподібні залози, розростання строми з гіперплазією її клітин. Який найбільш вірогідний діагноз?
A. @Залозиста гіперплазія ендометрію
B. Гострий ендометрит
C. Лейоміома
D. Міхуровий занос
E. Плацентарний поліп

4. Жінка, 45 років, звернулася до гінеколога зі скаргами на ацикличні, надмірні, маткові кровотечі. При
досліджені біоптату виявлено: збільшення кількості залоз, кістозне її розширення. Для якого процесу характерні
ці зміни?
A. Атрофія ендометрію
B. @Гіперплазії ендометрію
C. Гіпертрофії ендометрію
D. Метаплазії
E. Організації

5. Піхвова частина шийки матки вкрита:


A. Мезотелієм
B. Залозистим епітелієм
C. @Багатошаровим плоским епітелієм без зроговіння
D. Перехідно-клітинним епітелієм
E. Багатошаровим плоским епітелієм зі зроговінням

6. Цервікальний канал шийки матки висланий:


A. Мезотелієм
B. Багатошаровим плоским незроговілим епітелієм
C. @Залозистим епітелієм
D. Перехідно-клітинним епітелієм
E. Багатошаровим плоским зроговілим епітелієм

7. Під час гістологічного дослідження біопсії шийки матки був встановлений діагноз: «Ендоцервікоз
(псевдоерозія)». Які характерні мікроскопічні зміни в біоптаті шийки матки виявив патологоанатом?
A. Зроговіння покривного епітелію слизової оболонки
B. Клітинний атипізм покрівного епітелію слизової оболонки
C. @Локальна зміна багатошарового плоского епітелію на одношаровий призматичний
D. Ракові перлини
E. Слизова дистрофія залоз

8. При гістологічному дослідженні піхвової порції шийки матки виявлено заміщення багатошарового
плоского епітелію на однорядний призматичний, під яким розташовано багато залоз без ознак їх новоутворення.
Який з перелічених діагнозів найбільш вірогідний?
А. Аденоз
В. Прогресуючий ендоцервікоз
С. Ендоцервікоз, що загоюється
D. Малігнізований ендоцервікоз
Е. @Простий ендоцервікоз

9. Для ендоцервікозу характерним є:


A. Метаплазія епітелію в перехідно-клітинний
B. Наявність плоского зроговілого епітелію
C. Наявність стоншеного плоского незроговілого епітелію
D. @Циліндричний епітелій за межами цервікального канала
E. Відсутність епітелію

10. Основна ознака слабо вираженої дисплазії багатошарового плоского епітелію шийки матки:
A. Гіперкератоз
B. @Базально-клітинна гіперактивність на 1/2 - 1/3 товщі епітеліального покриву
C. Базально-клітинна гіперактивність по всій товщі епітелію
D. Паракератоз
E. Клітинний атипізм

11. Основна ознака дисплазії багатошарового плоского епітелію шийки матки:


A. Акантоз
B. @Порушення типової структури з осередковою клітинною атипією
C. Паракератоз
D. Зроговіння
E. Лейкоплакія

12. При гістологічному дослідженні біоптату піхвової частини шийки матки у хворої, 47 років, з ерозією,
що тривало не загоювалась виявлені ознаки клітинного атипізму, базальна мембрана - без змін. Поставити
діагноз.
A. Ендометрiоз
B. Ерозiя
C. Аденокарцинома
D. Папiлома
E. @Карцинома in situ

13. У літньої жінки з метрорагією в менопаузі при патогістологічному дослідженні вишкребків слизової
оболонки шийки матки виявлені розростання атипового епітелію з утворенням так званих "ракових перлин". Ваш
діагноз?
A. Слизовий рак
B. Аденокарцинома
C. Плоскоклітинний рак без зроговіння
D. @Плоскоклітинний рак зі зроговінням
E. Недиференційований рак

14. Основна ознака інвазивного раку в епітелії шийки матки:


A. @Прорив базальної підепітеліальної мембрани атиповими епітеліальними клітинами
B. Клітинний поліморфізм
C. Гіперхроматоз ядер
D. Збереження базальної мембрани
E. -

15. Що таке ендометріоз?


A. Гіперплазія ендометрію
B. @Наявність елементів ендометрію в невластивих йому місцях
C. Атрофія ендометрію
D. Дистрофія ендометрію
E. -

16. Жiнка, 39 рокiв, в анамнезi 3 вагiтностi та 3 пологiв, скаржиться на маткові кровотечi. При опитуваннi
встановлено вторинний прогресуючий характер дисменореї. Пiд час вагiнального дослiдження визначається
щільна, дифузно збiльшена матка. Результати бiопсiї ендометрiю без патологiї. Який найбiльш вiрогiдний
дiагноз?
A. @Ендометріоз
B. Ендометрит
C. Аденомiоз
D. Саркома матки
E. Лейомiома

17. При гістологічному дослідженні вишкребків слизової оболонки матки у хворої, 54 років, з клінічним
діагнозом «Порушення оваріально-менструального циклу» виявлено – розростання атипових залозистих
структур, що складаються з клітин з гіперхромними ядрами, фігурами мітозів, атипією. Атипові залозисті
структури вростають у міометрій. Для якого патологічного процесу характерні виявлені мікроскопічно зміни?
A. @Аденокарцинома матки
B. Залозиста гіперплазія ендометрію
C. Гострий ендометрит
D. Плацентарний поліп
E. Хоріонепітеліома матки

18. Назвати основну гістологічну ознаку, яка характерна для маткової вагітності:
A. Децедуальна тканина
B. Гравідарні залози в спонгіозному прошарку
C. @Ворсини хоріона та елементи трофобласта
D. Атипові клітини
E. Лейкоплакія

19. Плацентарний поліп виникає в результаті:


A. Метапластичних змін гермінативного епітелію в ендометрії
B. Метапластичних змін ендометрію
C. Міхурового заносу
D. Підслизової лейоміоми
E. @Затримки плацентарної тканини в порожнині матки

20. В перший період пологів у жінки відмічалось підвищення артеріального тиску (220/110 мм.рт.ст.),
з’явилася легка жовтяниця шкіри, потім розвинулися судоми і втрата свідомості. Смерть настала від гострого
порушення мозкового кровообігу. На розтині: ДВЗ синдром, жовтяниця, набряк легень і головного мозку, дрібні
крововиливи в підкіркових ядрах головного мозку, легень, печінки; ішемія і повнокров'я нирок, дрібні
симетричні некрози кіркової речовини нирок. Ваш діагноз.
A. @Еклампсія
B. Геморагічний інсульт
C. Вірусний гепатит
D. Некротичний нефроз
E. Геморагічна пневмонія

21. Для яких термінів вагітності характерна еклампсія вагітних?


А. Ранніх
В. @Пізніх
С. Середніх
D. Усіх

22. У вагітної після кримінального аборту – сепсис. Смерть настала при явищах наростаючої ниркової
недостатності. При патоморфологічному дослідженні нирок біло виявлено розповсюджений тромбоз
гломерулярних капілярів, аферентної артеріоли, некроз епітелію звивистих канальців і клубочків. Які зміни в
нирках були причиною розвитку ниркової недостатності?
A. Амілоїдоз нирок
B. Папілярний некроз
C. Гострий тубулярний некроз
D. @Кортикальний некроз
E. Гострий гломерулонефрит

23. У молодої жінки раптово відбулося переривання вагітності на 20 тижні. При цьому з матки виділилося
все плідне яйце (плід і оболонки), згортки крові. При гістологічному дослідженні було виявлено оболонки плоду,
ворсини хоріона, децидуальна тканина. Назвати різновид патології.
A. Штучний аборт
B. Передчасні пологи
C. Міхуровий занос
D. Хоріонепітеліома
E. @Мимовільний повний аборт

24. У жінки, 39 років, під час операції були видалені збільшена маткова труба та частина яєчника з
великою кістою. При гістологічному дослідженні стінки труби виявлені децидуальні клітини, ворсини хоріона.
Який найбільш вірогідний діагноз було встановлено при дослідженні маткової труби?
A. Трубна вагітність
B. Плацентарний поліп
C. Хоріонкарцинома
D. "Паперовий" плід
E. Літопедіон

25. У молодої жінки в зв’язку з гострими болями в клубовій ділянці, вилучено маткову трубу з локальним
розширенням її середньої третини, що заповнено кров’ю. При гістологічному дослідженні в отворі труби
виявлені ворсини хоріона, великі поля еритроцитів з домішками лейкоцитів. Який найбільш імовірний діагноз?
A. Трубна вагітність
B. Гострий гнійний сальпінгіт
C. Крововилив у маткову трубу
D. Геморагічний сальпінгіт
E. Гнійний сальпінгіт

26. Під час розтину тіла жінки, 28 років, яка померла від геморагічного шоку, виявлено: гемоперитонеум,
права маткова труба збільшена, багряна, з наскрізним дефектом стінки, її дилатований просвіт виповнений
темно-червоними згортками крові. Гістологічно у слизовій оболонці труби та серед м’язових клітин виявляються
пласти великих світлих децидуальних клітин, у м’язовій оболонці та серед згортків крові у просвіті труби -
ворсинки хоріона. Діагностуйте патологію вагітності:
A. @Порушена трубна вагітність
B. Порушена маткова вагітність
C. Деструюючий міхуровий занос
D. Черевна вагітність
E. Інтерлігаментарна вагітність

27. У жінки з позаматковою вагітністю відбулось виділення плода в порожнину маткової труби з
кровотечею. Назвіть патологічний процес.
A. Повний трубний аборт
B. @Неповний трубний аборт
C. Самочинний аборт
D. Штучний аборт
E. Кримінальний аборт

28. У жінки, 23 років, після аборту з’явилася маткова кровотеча. Мікроскопічно у вишкребках з
порожнини матки виявлені ворсини хоріона, що нагадують грона винограду. Мікроскопічно: набряк строми
ворсин з утворенням безліч кіст, що супроводжуються проліферацією епітелію та синцитію ворсин, залишки
плоду і плодових оболонок. Який найбільш імовірний діагноз?
A. @Міхуровий занос
B. Вагітність
C. Хоріонепітеліома
D. Ендометрит
E. Плацентарний поліп

29. Патогістологічно у вишкребках з порожнини матки у жінки, 36 років, яка мала значну маткову
кровотечу, було виявлено велику кількість світлих епітеліальних клітин Лангханса та багатоядерних симпластів,
кількість фігур мітозу дуже значна, строма відсутня, судинні порожнини вистелені вказаними клітинами.
Діагностуйте пухлину матки.
A. Поліп ендометрію
B. @Хоріонепітеліома
C. Низькодиференційована аденокарцинома ендометрію
D. Частковий міхуровий занос
E. Повний міхуровий занос

30. У жінки після пологів з'явилися безперервні кровотечі з порожнини матки. Після оперативного
видалення матки в її порожнині виявлено вузол, м'якої консистенції, строкатого кольору. Мікроскопічно тканина
вузла складається з клітин цитотрофобласта і поліморфних елементів синцитіотрофобласта, визначаються
множинні некрози. Ваш діагноз?
A. Гострий ендометрит
B. @Хоріонкарцинома
C. Поліпозна гіперплазія ендометрію
D. Плацентарний поліп
E. Інвазівний міхурний занесок

31. Після проведенної операції у жінки, 24 років, видалено маткову трубу, яка була досліджена
патологоанатомом. Макроскопічно: значне потовщення труби, серозна оболонка її тьмяна, вкрита жовтувато-
сірими плівками, судини її повнокровні. На розрізі просвіт труби заповнений жовто-зеленою в'язкою масою,
стінка – пухка. Який попередній патологоанатомічний діагноз?
A. Гострий фібринозний сальпінгіт
B. Гострий катаральний сальпінгіт
C. Гострий гангренозний сальпінгіт
D. @Гострий гнійний сальпінгіт
E. Гострий геморагічний сальпінгіт

32. При інтраопераційній біопсії молочної залози виявлено концентричне розростання


внутрішньочасточкової сполучної тканини навкруги дрібних проток, які вкриті одношаровим епітелієм. Ваш
діагноз?
A. Мастит
B. Внутрішньопротокова папілома
C. Часточковий неінфільтративний рак
D. Внутрішньопротоковий неінфильтративний рак
E. @Фіброаденома

33. При гістологічному дослідженні видаленого збільшеного лімфатичного вузла, в ньому було виявлено
пухлину органоїдної будови, яка складається з комплексів атипових епітеліальних клітинн різної величини і
форми з гіперхромними ядрами і великою кількістю мітозів. Ці клітини мають рецептори до естрогену та
прогестерону. Яка найбільш вірогідна первинна локалізація пухлини?
A. Аденокарцинома шлунка
B. Дрібноклітинний недиференційований рак легень
C. @Рак молочної залози
D. Плоскоклітинний зроговілий рак стравоходу
E. Медулярная карцинома щитовидної залози

34. При гістологічному дослідженні вузла у видаленій молочній залозі серед добре вираженої строми
були виявлені різних розмірів і форми комплекси атипових поліморфних епітеліальних клітин з наявністю
просвітів у центрі цих комплексів. Клітини з великими ядрами, наявність атипових мітозів. Поставте діагноз
A. Плоскоклітинний незроговілий рак
B. @Аденокарцинома
C. Солідний рак
D. Фіброаденома молочної залози
E. Недиференційований поліморфноклітинний рак

35. При гістологічному дослідженні злоякісної пухлини молочної залози виявлено, що пухлина
побудована з низькодиференційованих атипових клітин епітеліального походження, які утворюють трабекули,
відокремлені між собою прошарками сполучної тканини, співвідношення клітин і строми приблизно 1:1.
Встановіть діагноз.
A. @Солідний рак
B. Аденокарцинома
C. Плоскоклітинний рак
D. Фіброзний рак
E. Дрібноклітинний рак
ПРЕ-, ІНТРА- ТА ПЕРИНАТАЛЬНА ПАТОЛОГІЯ

1. При ранній пренатальній діагностиці хромосомних аномалій плода оптимальним матеріалом для
дослідження являються клітини:
A. @Амніотичної рідини
B. Лімфоцити матері
C. Лімфоцити батька
D. Децидуальної тканини
E. Все вказане
2. При аутопсії плода виявлено епідуральну (між кістками склепіння черепа і твердої мозкової оболонки)
гематома, яка утворилася в результаті розриву серповидного синуса і намету мозочка. Ці зміни є проявом
патології:
A. @Інтранатального періоду
B. Перинатального періоду
C. Антенатального періоду
D. Постнатального періоду
E. Періоду прогенезу
3. У жінки, 27 років, народився мертвий плід з гестаційним віком понад 43 тижні. Шкіра плоду суха,
лупиться. Відзначається загальна гіпотрофія і наявність ядер, окостеніння проксимального епіфіза
великогомілкової і плечової кісток. Води, пуповина та оболонки посліду забарвлені меконієм. Для якого перiоду
розвитку плода характерні описані морфологічні зміни?
A. @Антенатального
B. Пренатального
C. Інтранатального
D. Постнатального
E. Прогенезу
4. У новонародженої дитини виявлено вродженi вади розвитку травної системи, що пов’язано з дiєю
тератогенних факторiв на початку вагiтностi. На який з зародкових листкiв подiяв тератоген?
A. @Ендодерма
B. Усi листки
C. Ендодерма i мезодерма
D. Ектодерма
E. Мезодерма
5. У мертвонародженої дитини знайдено повний дефект задньої стінки хребтового каналу, м’яких тканин,
шкіри та мозкових оболонок. Розпластаний спинний мозок лежить відкритим на передній стінці каналу. Який з
перелічених діагнозів найбільш вірогідний?
A. @Рахіосхіз
B. Менінгоцеле
C. Енцефалоцистоцеле
D. Мієлоцеле
E. Менінгомієлоцеле
6. Під час аутопсії мертвонародженого відмічено виражене укорочення та потовщення кінцівок з
утворенням в їх шкірі великих складок. Голівка збільшена, ніс сідлоподібний, ротова порожнина напіввідкрита,
язик товстий, шия коротка, тіла хребців потовщені, грудна клітка гіпоплазована. Вада поєднана з гіпоплазією
легень. Для якої з вроджених вад м’язово-скелетної системи це характерно ?
A. @Летальна мікромелія
B. Ахондроплазія
C. Недосконалий остеогенез
D. Вроджена мармурова хвороба
E. Вроджена міотонія Опенгейма
7. При патологоанатомічному дослідженні інтранатально загиблої дитини виявлено: укорочення та
потовщення кінцівок, великі шкіряні складки, збільшення голови, сідлоподібний ніс, вкорочення шиї,
потовщення тіл хребців, гіпоплазію грудини та легень. Який з перелічених діагнозів найбільш вірогідний?
A. @Летальна мікромелія
B. Ахондродисплазія
C. Недосконалий остеогенез
D. Мармурова хвороба
E. Фокомелія
8. Дитина з масою тіла 2100 г, довжиною тіла 43 см та з іншими ознаками недоношеності, померла на 3-й
день після народження. На секції виявлено: крововиливи в сегментах легень, плеврі, підкапсулярні гематоми
печінки, двобічні гематоми в надниркових залозах, великі плямисті крововиливи в мозковій речовині нирок. Ваш
діагноз?
A. @Геморагічна хвороба новонароджених
B. Менінгококцемія
C. Геморагічна пневмонія
D. Вроджений сифіліс
E. Гемолітична хвороба новонароджених
9. У дівчинки, 4 років, під час операції в заочеревинній ділянці виявлено, що права нирка на 1/3 менша від
лівої. Діаметр правої ниркової артерії 0,3 мм, лівої - 0,4 мм. Який патологічний процес має місце в даному
випадку?
A. @Гіпоплазія
B. Атрофія фізіологічна
C. Атрофія патологічна
D. Агенезія
E. Аплазія
10. У новонародженого малюка педiатр виявив, що отвiр крайньої плотi за величиною не перевищує
дiаметр сечовивiдного каналу i голiвка статевого члена не може виходити через такий отвiр. Як називається цей
стан?
A. @Фiмоз
B. Епiспадiя
C. Гiпоспадiя
D. Парафiмоз
E. Гермафродитизм
11. До лiкаря-педiатра звернулись батьки новонародженої дитини зi скаргами на видiлення рiдини (сечi) в
дiлянцi пупка. Яка вроджена вада у дитини?
A. @Незрощена сечова протока
B. Дивертикул Меккеля
C. Розщеплення сечiвника
D. Пупкова грижа
E. Пахвинна грижа
12. У новонародженої дитини мають місце значний дефект передньої черевної стінки, гіпоплазія черевної
порожнини, наявність неприкритих петель тонкої кишки, печінки, позачеревної порожнини. Який з перелічених
діагнозів найбільш вірогідний?
A. @Евентрація органов черевної порожнини
B. Грижа пупка
C. Мекелев дивертикул
D. Пупково-кишкова нориця
E. Ентерокистома
13. Жiнка, 30 рокiв, народила хлопчика з розщепленням верхньої губи ("заяча губа" i "вовча паща"). При
додатковому обстеженнi виявленi значнi порушення нервової, серцево-судинної систем та зору. При дослiдженнi
карiотипу дiагностовано трисомiю за 13-ю хромосомою. Який синдром наявний у хлопчика?
A. @Патау
B. Шерешевського-Тернера
C. Едвардса
D. Дауна
E. Клайнфельтера
14. При обстеженні хворого з клінікою білатеральної ектопії кришталика очей виникло припущення про
наявність синдрому Марфана. Вкажіть, які з виявлених змін можуть підтвердити це припущення?
A. @Всі вказані зміни
B. Недостатність аортального клапана і розширення висхідного відділу аорти
C. Пролапс стулок мітрального клапана
D. Доліхоцефалія
E. Жодна з вказаних змін
15. При розтині новонародженої дитини, яка померла на першому тижні життя від серцевої недостатності,
виявлено збільшення серця з наявністю гіпертрофії м’язових волокон переважно лівого шлуночка. Ендокард
лівого шлуночка, а також мітральний і аортальний клапани значно потовщені, біло-жовтого кольору. Який з
перелічених діагнозів найбільш вірогідний?
A. @Фіброеластоз ендокарда
B. Дифузний ендокардит
C. Фібропластичний ендокардит
D. Гіпертрофія міокарда
E. Кардіоміопатія
16. У хлопчика, 1,5 років, який помер від вірусної пневмонії, на аутопсії виявлено: виступаючі потиличні
та лобові горби (“квадратна голова”), сідлоподібний ніс, сплюснута потилиця, незарощене велике тім’ячко,
потовщення у ділянках кістково-суглобових з’єднань ребер та епіфізів довгих трубчастих кісток, Х-подібна
деформація нижніх кінцівок. При макроскопічному дослідженні: зона росту кісток - нерівна, розширена,
скловидна; гістологічно - розширення хрящової зони з неправильним чергуванням і розташуванням хрящових
клітин, відсутня чітка і рівна погранична лінія між хрящовою та остеоїдною тканинами. Діагностуйте
захворювання скелета.
A. @Рахіт
B. Вроджений сифіліс
C. Хондродисплазія
D. Недосконалий остеогенез
E. Фіброзна дисплазія

ЗАХВОРЮВАННЯ
ЕНДОКРИННОЇ СИСТЕМИ
ВАРІАНТ № 1
1. При ураженні яких клітин підшлункової залози розвивається цукровий діабет?
A. @Бета-клітин
B. Альфа-клітин
C. Гама-клітин
D. Дельта-клітин
E. С-клітин

2. На розтині чоловіка, 67 років, померлого при явищах гіпоглікемічної коми, в підшлунковій залозі
спостерігаються ділянки розростання сполучної тканини та некротичні осередки, атрофія острівків Лангенгарса.
Яке захворювання зумовило такі зміни в підшлунковій залозі?
A. @Цукровий діабет
B. Муковісцидоз
C. Гострий панкреатит
D. Рак головки підшлункової залози
E. Гіпоплазія підшлункової залози

3. У жінки, 67 років, яка впродовж 14 років хворіла на цукровий діабет, за останні 2 роки різко погіршився
зір. З якими характерними процесами пов’язане погіршення зору?
А. Макроангіопатія
В. @Мікроангіопатія
С. Кальциноз скловидного тіла
D. Помутніння рогівки
Е. Помутніння кришталика
4. Хворому з наростаючими явищами хронічної ниркової недостатності проведено біопсію нирки.
Гістологічно в біоптаті виявлено: дифузний інтракапілярний гломерулосклероз, розширення і ущільнення
мезангію, потовщення базальної мембрани клубочків. З урахуванням клінічних і лабораторних даних
діагностовано синдром Кімельстіла-Уілсона. З яким захворюванням пов'язаний розвиток даного синдрому?
A. Інфарктом нирки
B. Амілоїдозом нирок
C. @Цукровим діабетом
D. Пієлонефритом
E. Гломерулонефритом

5. У померлого з прижиттєвим діагнозом «Цукровий діабет», з високою протеїнурією, набряками і


артеріальною гіпертензією, на розтині: нирки дещо зменшені, з заглибленнями на поверхні до 0,6 см, щільні.
Мікроскопічно: дифузне розростання сполучної тканини, різко виражений гіаліноз мезангію і загибель клубочків.
Назвіть клінічний синдром в даному випадку.
A. Синдром Дресслера
B. Синдром Бадда-Кіарі
C. Синдром Уотерхауса-Фрідеріксена
D. Синдром Ляріша
E. @Синдром Кімельстіла-Уілсона
6. Хворий на цукровий діабет помер від хронічної ниркової недостатності з розвитком уремії. Під час
секції виявлені всі прояви уремії зі змінами в нирках. Які найімовірніші мікроскопічні зміни мають місце в
нирках при цьому?
A. Некроз епітелію канальців нефрону
B. Спазм приносних артеріол
C. Гіалінові циліндри в просвіті канальців нефрону
D. @Гіаліноз та склероз капілярів клубочків
E. Осередковий некроз коркової речовини нирок

7. У хворого на цукровий діабет з'явились ознаки наростаючої ниркової недостатності. Який процес у
нирках є причиною ниркової недостатності?
A. Інтерстиціальне запалення
B. Некроз епітелію звивистих канальців
C. Амілоїдоз
D. @Гломерулосклероз
E. Мембранозна нефропатія

8. Жінка, 27 років, з дитинства хворіла на цукровий діабет. Впродовж останніх років спостерігались
артеріальна гіпертензія, протеїнурія. Хвора померла при явищах уремії. Які характерні зміни знайдено
патологоанатомом при розтині?
A. A “Тигрове” серце
B. @“Волохате” серце
C. “Панцирне” серце
D. Просте ожиріння серця
E. Гнійний перикардит

9. При електронномікроскопічному дослідженні біоптату печінки хворого на цукровий діабет в ядрах


гепатоцитів виявлено пошкодження пор нуклеолеми та "дірчасті, порожні ядра". Який з перелічених
патологічних процесів найбільш вірогідний?
A. Ядерні цитоплазматичні включення
B. @Істинні ядерні включення
C. Вірусозумовлені ядерні включення
D. Каріолізис
E. Дисфункціональний набряк ядра

10. Хворій виконано субтотальну резекцію щитовидної залози з приводу дифузного токсичного волоу.
Післяопераційний період ускладнився нападами тонічних судом, виник ларингоспазм з асфіксією. Що могло
призвести до даних ускладнень?
A. @Видалення паращитовидних залоз
B. Трахеомаляція
C. Пошкодження n.reccurens
D. Пошкодження трахеї
E. Тиреотоксична криза

11. На розтині померлого від кахексії виявлено аденому паращитовидних залоз, деформацію кісток,
особливо кінцівок, хребетного стовпа, ребер. Кістки м’які, порозні, легко ріжуться ножем. Який з перелічених
діагнозів найбільш вірогідний?
A. @Паратиреоїдна остеодистрофія
B. Остеопетроз
C. Хондродисплазія
D. Остеомієліт
E. Фіброзна дисплазія

12. При розтині померлого, 28 років, виявлено деформацію кісток кінцівок, ребер; вони м’які легко
ріжуться ножем, нирки зморщені. Гістологічно виявляються осередки лакунарного розсмоктування кісткової
тканини, фіброзна тканина. Про яке захворювання треба думати?
A. @Паратиреоїдна остеодистрофія
B. Остеомієліт
C. Фіброзна дисплазія
D. Остеонекроз
E. Хвороба Педжета

13. У померлого хворого на розтині виявлено бронзове забарвлення шкіри та слизових оболонок
порожнини рота. В наднирниках виявлені казеозні маси. Яке захворювання ендокринної системи у померлого?
A. @Адісонова хвороба
B. Синдром Іценко-Кушинга
C. Базедова хвороба
D. Акромегалія
E. Феохромоцитома

14. У хворого, який раніше хворів на гематогенний туберкульоз, з'явилась гіперпігментація (меланоз)
шкіри та слизових оболонок, кахексія, недостатність серцево-судинної системи. Яке захворювання викликало
такі зміни?
A. @Адісонова хвороба
B. Феохромоцитома
C. Хвороба Симондса
D. Хвороба Іценко-Кушинга
E. Хвороба Грейвса

15. Під час патологоанатомічного розтину померлої хворої був встановлений діагноз: «Адісонова
хвороба». Виберіть найбільш імовірну безпосередню причину смерті цієї хворої:
A. Гостра ниркова недостатність
B. @Гостра недостатність надниркових залоз
C. Печінкова недостатність
D. Тромбоемболія легеневої артерії
E. Тромбоз ворітної вени

16. На розтині тіла жінки, 40 років, яка прижиттєво страждала на ожиріння за верхнім типом, стероїдний
цукровий діабет, артеріальну гіпертонію, вторинну дисфункцію яєчників виявлено: гіпертрихоз, гірсутизм, стрії
на шкірі стегон та живота. В передній частці гіпофіза - пухлина (мікроскопічно: базофільна аденома); у
надниркових залозах - гіперплазія коркового шару. Який з перелічених діагнозів найбільш вірогідний?
A. Гіпофізарний нанізм
B. Синдром Іценко-Кушинга
C. Хвороба Симондса
D. Адипозогенітальна дистрофія
E. @Хвороба Іценко-Кушинга

17. На розтині тіла жінки, віком 45 років, яка впродовж останніх 15 років страждала на артеріальну
гіпертензію, цукровий діабет та дисфункцію яєчників, виявлено: ожиріння за верхнім типом, базофільну аденому
у передній частці гіпофіза, гіперплазію кори надниркових залоз Який з перелічених діагнозів найбільш
вірогідний?
A. Адипозогенітальна дистрофія
B. Синдром Іценко-Кушинга
C. Гіпертонічна хвороба
D. Адісонова хороба
E. @Хвороба Іценко-Кушинг

18. У хворого з верхнім типом ожиріння клінічно тривало відмічалися артеріальна гіпертонія,
гіперглікемія, глюкозурія. Смерть настала від крововиливу у головний мозок. При патоморфологічному
дослідженні виявленні: базофільна аденома гіпофіза, гіперплазія кори надниркових залоз. Ваш можливий
діагноз?
A. @Хвороба Іценко-Кушинга
B. Цукровий діабет
C. Синдром Іценко-Кушинга
D. Хвороба Симондса
E. Адипозогенітальна дистрофія

19. У молодої людини виявлено надлишок соматотропного гормону, збільшені розміри носа, губ, вух,
нижньої щелепи, кистей і стоп. Ваш діагноз?
A. @Акромегалія
B. Гіпофізарний нанізм
C. Хвороба Іценко-Кушинга
D. Адісонова хвороба
E. Адипозогенітальна дистрофія

20. У хворого, 45 років, поступово почали пропорційно збільшуватись за розмірами стопи, права кисть,
ніс і губи. Виявлено аденому гипофіза. Яке захворювання можна запідозрити?
A. @Акромегалія
B. Нанізм
C. Хвороба Базедова
D. Адісонова хвороба
E. Цукровий діабет

21. У видаленій збільшеній щитовидній залозі фолікули різних розмірів і форми, з інтра- та
екстрафолікулярною проліферацією епітелію, прошарками сполучної тканини, яка частково відокремлює окремі
ділянки. Поставте найбільш правильний діагноз.
A. @Макрофолікулярне, мікрофолікулярне воло з утворенням вузлів
B. Дифузне воло
C. Аутоімунний тиреоїдит
D. Фолікулярний рак
E. Тиреоїдит Риделя

22. У хворого з гірського району Середньої Азії виявлене збільшення щитовидної залози, що утруднило
ковтання. Спостерігались: збільшення маси тіла, загальмованість, сонливість, набрякле обличчя. При
мікроскопічному дослідженні у щитовидній залозі - різних розмірів фолікули з гіпохромним колоїдом. Який з
діагнозів найбільш вірогідний?
A. Тиреоїдит Хасімото
B. Базедове воло
C. @Ендемічне воло
D. Спорадичне воло
E. Воло Риделя
23. У хворого, який мешкає у болотистій місцевості, виявлене збільшення щитовидної залози, що заважає
ковтанню. Спостерігається збільшення маси тіла, сонливість, загальмованість, набрякле обличчя. Ваш діагноз:
A. Воло Хасімото
B. Хвороба Грейвса (тиреотоксичне воло)
C. @Ендемічне вузлувате воло
D. Тиреоїдит де Кервена
E. Колоїдне воло

24. Хворій з підвищеною активністю тиреостимулюючих імуноглобулінів був встановлений діагноз:


«Базедова хвороба». Назвіть імунний механізм розвитку даного захворювання.
A. Антитілозалежна цитотоксичність
B. Комплемент залежна реакція
C. Системна анафілаксія
D. Цитотоксичність опосередкована Т-лімфоцитами
E. @Антитілоопосередкована клітинна дисфункція

25. Жінці, 40 років, проведено операцію тироїдектомії. При гістологічному дослідженні щитовидної
залози виявлено, що її фолікули різновеликі, містять пінистий колоїд, епітелій фолікулів високий, місцями
формує сосочки, в стромі – осередки лімфоцитарна інфільтрація. Діагностуйте захворювання щитовидної залози.
A. Вузлове воло
B. Тироїдит Хасімото
C. Тироїдит Риделя
D. Тироїдит де Кервена
E. @Воло Базедове

26. Жінка, 45 років, звернулася до ендокринолога, який встановив попередній діагноз: «Дифузний
токсичний зоб». Які характерні мікроскопічні зміни повинен знайти патологоанатом у пункційному біоптаті
залози?
A. @Циліндричний фолікулярний епітелій з утворенням сосочків, які гілкуються всередині фолікулів і
лімфоплазмоцитарну інфільтрацію строми
B. Плоский фолікулярний епітелій з нейтрофільною інфільтрацією строми
C. Некроз залози з лімфоплазмоцитарною інфільтрацією строми
D. Великі фолікули зі сплющеним епітелієм, які заповнені колоїдом
E. Клітинний атипізм фолікулярного епітелію

27. При розтині тіла жінки, 50-ти років, померлої від серцевої недостатності, знайдено дифузне
збільшення щитоподібної залози. Мікроскопічно: перетворення призматичного епітелію фолікулів у
циліндричний, проліферація епітелію з утворенням сосочків, розрідження колоїду та лімфоплазмоцитарна
інфільтрація строми; в серці - гіпертрофія лівого шлуночка, серозний набряк та лімфоїдна інфільтрація строми, в
печінці - серозний набряк. Який з перелічених діагнозів найбільш імовірний?
A. @Дифузний токсичний зоб
B. Ендемічний зоб
C. Зоб Хасімото
D. Зоб Риделя
E. Гострий тиреоїдит

28. Хвора померла після операції з приводу дифузного токсичного вола. На розтині виявлене
“тиреотоксичне серце”. Які характерні морфологічні зміни в серці знайшов патологоанатом?
A. Амілоїдоз строми міокарда
B. Фібринозний перикардит
C. Великоосередковий міокардіосклероз
D. @Гіпертрофію кардіоміоцитів з набряком та лімфоїдною інфільтрацією строми
E. Гіаліново-крапельну дистрофію кардіоміоцитів

29. При гістологічному дослідженні щитовидної залози, видаленої при операції, виявлені: деструкція та
атрофія фолікулів, дифузна лімфоцитарна інфільтрація з формуванням лімфоїдних фолікулів у стромі. До якої
групи захворювань належить цей тиреоїдит?
A. Бактеріальних
B. @Аутоімунних
C. Викликаних фізичними факторами
D. Інфекційно-алергічних
E. Вірусних

30. При гістологічному дослідженні щитовидної залози виявлено: помірну атрофію паренхіми, склероз,
дифузну інфільтрацію строми лімфоцитами і плазматичними клітинами з формуванням лімфоїдних фолікулів.
Ваш діагноз?
A. Тиреотоксичне воло
B. Паренхіматозне воло
C. @Аутоімунний тиреоїдит
D. Тиреоїдит
E. Воло Риделя

31. При гістологічному дослідженні щитовидної залози встановлено лімфоцитарну інфільтрацію


міжфолікулярного простору з формуванням лімфоїдних фолікулів, розростання сполучної тканини з атрофією
паренхими. Який з перелічених діагнозів найбільш вірогідний?
A. @Аутоімунний тиреоїдит
B. Тиреоїдит Риделя
C. Базедова хвороба
D. Колоїдний зоб
E. Ендемічний зоб

32. У хворого з клінічними симптомами гіпотиреозу, щитовидна залоза збільшена удвічі, при пальпації:
щільна, з горбистою поверхнею. При гістологічному дослідженні - поряд з атрофією фолікулів залози
відмічається дифузна інфільтрація паренхіми лімфоцитами, плазматичними клітинами з утворенням фолікулів і
посилене розростання сполучної тканини. Вкажіть найбільш вірогідний діагноз.
A. @Зоб Хасімото
B. Фіброзний зоб
C. Ендемічний зоб
D. Спорадичний зоб
E. Дифузний токсичний зоб

33. У хворого виникло збільшення щитовидної залози в 2 рази. При пальпації залоза щільна, поверхня
нерівномірно горбиста. При гістологічному дослідженні - дифузна інфільтрація тканини залози лімфоцитами,
плазматичними клітинами з утворенням фолікулів та посилене розростання сполучної тканини. Яке
захворювання має місце у хворого?
A. Дифузней токсичне воло
B. Ендемічней воло
C. Спорадичне воло
D. @Воло Хасімото
E. Воло Риделя

34. При гістологічному дослідженні щитовидної залози хворого, який помер від серцевої недостатності
при явищах гіпотиреозу виявлено дифузну інфільтрацію залози лімфоцитами, плазмоцитами, атрофію паренхіми
і розростання сполучної тканини. Поставте діагноз.
A. @Тиреоїдит Хасімото
B. Тиреоїдит Риделя
C. Тиреоїдит де Кервена
D. Воло тиреотоксичне
E. Воло Базедова

35. У бiоптатi щитовидної залози виявлено атрофію паренхіматозних елементiв, дифузну інфільтрацію
тканини залози лiмфоцитами та плазматичними клiтинами з утворенням в нiй лiмфоїдних фолiкулiв. Для якого
захворювання є характерними наведенi ознаки?
A. @Тиреоїдит Хасiмото
B. Тиреоїдит Рiделя
C. Ендемiчний зоб
D. Хвороба Базедова
E. Аденома щитовидної залози

36. При гiстологiчному дослiдженнi щитовидної залози виявляються: значна iнфiльтрацiя лiмфоцитами з
утворенням лiмфоїдних фолiкулiв, руйнування паренхiматозних елементiв, розростання волокон сполучної
тканини. Для якого захворювання характерна така картина?
A. @Зоб Хасімото
B. Колоїдний зоб
C. Ендемiчний зоб
D. Дифузний токсичний зоб
E. Паренхiматозний зоб

37. Під час гістологічного дослідження операційної біопсії щитовидної залози виявлено: серед фолікулів,
заповнених колоїдом, лімфоїдні структури з центрами росту. Яке захворювання у хворого?
A. @Зоб Хасімото
B. Ендемічний зоб
C. Спорадичний зоб
D. Базедов зоб

ТУБЕРКУЛЬОЗ
ВАРІАНТ №
1. У жінки, 20 років, при гістологічному дослідженні шийного лімфатичного вузла виявлені вузлики, які
складаються з валів епітеліоїдних, лімфоїдних клітин та багатоядерних гігантських клітин Пирогова-Лангханса,
розташованих між ними. У центрі вузликів виявляється казеозний некроз. Який збудник потрібно виявити у зоні
некрозу для встановлення остаточного діагнозу?
A. @Паличку Коха
B. Паличку Ганзена
C. Бліду трепонему
D. Рикетсії
E. Паличку Волковича-Фріша

2. У чоловіка, 30 років, при гістологічному дослідженні біоптату з шийного лімфатичного вузла виявлені
гранульоми, які складаються з епітеліоїдних, лімфоїдних, багатоядерних гігантських клітин типу Пірогова-
Лангханса. У центрі гранульом виявляється некроз. Який збудник потрібно виявити в зоні некрозу для
підтвердження діагнозу туберкульозу?
A. @Мікобактерію Коха
B. Бліду трепонему
C. Стафілокок
D. Бацили Волковича-Фріша
E. Сальмонели

3. У дитини після перенесеної ангіни різко збільшилися лімфатичні вузли: паратрахеальні, біфуркаційні,
шийні. При мікроскопічному дослідженні шийного лімфатичного вузла виявлені осередки некрозу, обмежені
лімфоцитами, епітеліоїдними клітинами, та клітинами Пирогова-Лангханса. Вкажіть найбільш вірогідну
патологію.
A. @Туберкульоз
B. Саркоїдоз
C. Риносклерома
D. Сап
E. Сифіліс

4. У хворого з субфібрильною температурою в біоптаті збільшеного лімфатичного вузла виявлені численні


гранульоми, які містять у центрі казеозний некроз, оточений епітеліоїдними клітинами, велетенськими багато-
ядерними клітинами Пирогова-Лангханса і лімфоцитами. Для якого захворювання характерні такі
патогістологічні зміни?
A. @Туберкульоз
B. Лімфогранулематоз
C. Лімфома
D. Лімфолейкоз
E. Банальний лімфаденіт

5. При мікроскопічному дослідженні шийного лімфатичного вузла виявлено скупчення епітеліоїдних


клітин, лімфоцитів і гігантських клітин Пирогова-Лангханса. В центрі - казеозний некроз. Вкажіть найбільш
імовірну патологію:
A. Туберкульоз
B. Риносклерома
C. Саркоїдоз
D. Сап
E. Сифіліс

6. У жінки, 30 років, при гістологічному дослідженні шийного лімфовузла виявлено порушення малюнка
внаслідок розростання епітеліоїдних, лімфоїдних клітин і макрофагів з ядрами у вигляді підкови, в центрі деяких
скупчень клітин - безструктурні ділянки блідо-рожевого кольору з уламками ядер. Для якого захворювання
характерні такі зміни?
A. Туберкульоз
B. Лімфогранулематоз
C. Актиномікоз
D. Метастаз пухлини
E. Сифіліс

7. У жінки, 22 років, при гістологічному дослідженні збільшених шийних лімфатичних вузлів, виявлені
вузлики, що складаються переважно з плоских, дещо витягнутих, з блідо забарвленим ядром, гігантських клітин
округло- овальної форми з блідо-рожевою цитоплазмою та з ядрами, розташованими на периферії (у вигляді
частоколу), а також дрібних округлої форми клітин з вузьким обідком цитоплазми; у центрі деяких вузликів -
безструктурні маси, забарвлені в блідо-рожевий колір. Для якого захворювання характерні такі зміни?
A. Туберкульоз
B. Лімфогранулематоз
C. Саркоїдоз
D. Бруцельоз
E. Неспецифічний гіперпластичний лімфаденіт
8. При розтині у дитини виявлено первинний кишечний туберкульозний комплекс: первинна аффект-
виразка порожньої кишки, лімфангіт і регіонарний казеозний лімфаденіт. Смерть настала від перфорації виразки і
розвитку розлитого перитоніту. Назвіть шлях зараження дитини туберкульозом.
A. @Аліментарний
B. Трансплацентарний
C. Аерогенний
D. Змішанний
E. Контактний

9. Дівчинка впродовж останніх 3 місяців вживала сире коров'яче молоко. В лікарню поступила з клінікою
"гострого живота". На операції - в сліпій кишці знайдено циркулярну виразку з перфорацією. При гістологічному
дослідженні в краях виразки - некротичні маси, лімфоцити, епітеліоїдні клітини, одинокі гігантські багатоядерні
клітини. Ваш діагноз.
A. Неспецифічний виразковий коліт
B. Амебіаз
C. @Первинний туберкульозний афект в кишці
D. Рак сліпої кишки
E. Дизентерія
10. У хворого на рентгенограмі легень виявлено затемнення. Під час діагностичної експрес-біопсії
лімфатичного вузла бронха виявлено: сирнистий некроз, навколо якого розташовані епітеліоїдні та лімфоїдні
пласти з домішками багатоядерних гігантських клітин. Вкажіть причину лімфаденіту.
A. @Туберкульоз
B. Пневмонія
C. Сифіліс
D. Метастази раку
E. Аденовірусна інфекція

11. Під час розтину тіла дівчинки, 9 років, у верхівці правої легені було знайдено осередок казеозного
некрозу, діаметром 15 мм, біфуркаційні лімфатичні вузли були збільшені, містили дрібні осередки некрозу
коагуляційного типу. Мікроскопічно - у легеневому осередку та в лімфатичних вузлах навколо некротичних мас
були розташовані епітеліоїдні клітини, лімфоцити та поодинокі багатоядерні гігантські клітини. Діагностуйте
захворювання.
A. @Первинний туберкульоз
B. Гематогенний туберкульоз з переважним ураженням легень
C. Гематогенний генералізований туберкульоз
D. Вторинний осередковий туберкульоз
E. Вторинний фіброзно-осередковий туберкульоз

12. При гістологічному дослідженні збільшених надключичних лімфатичних вузлів, у жінки, 40 років,
знайдено порушення їх структури з осередками скупчення злегка витягнутої форми клітин зі світлим ядром,
багатоядерних крупних овально-округлої форми клітин, багатих цитоплазмою і ядрами, розташованими на
периферії у вигляді частоколу, скупчення оточені лімфоїдними клітинами; в центрі - безструктурні маси,
забарвлені в рожевий колір. Описані зміни характерні для?
A. Метастазу пухлини
B. Лімфогранулематозу
C. Гіперпластичного лімфаденіту
D. Саркоїдозу
E. @Туберкульозного лімфаденіту

13. На розтині тіла чоловіка, 43 років, в ІІІ сегменті правої легені субплеврально виявлено ділянку
ущільнення діаметром 1,5 см з чіткими межами, оточену білою волокнистою тканиною, на розрізі - з ділянками
біло-жовтого кольору, що кришаться. Наявність даного осередку характерно для:
A. Периферичного раку
B. Хондроми
C. @Інкапсульованого первинного афекту
D. Фіброми
E. Інфаркту легені, що організується

14. Дитина, 8 років, раніше не хворіла, в лікарню надійшла з ознаками задухи. При рентгенівському
обстеженні в плевральній порожнині - повітря і рідина. Під час операції в правій легені виявлено субплевральну
порожнину діаметром 2,4 см, неправильної форми з залишками казеозних мас, яка сполучалася з плевральною
порожниною. Такі ж казеозні маси виявлені при розрізі збільшених прикореневих лімфатичних вузлів. При
гістологічному дослідженні стінки порожнини виявлені лімфоцити, епітеліоїдні клітини, багатоядерні гігантські
клітини. Як правильно назвати порожнину?
A. @Первинна легенева каверна при туберкульозі
B. Абсцес легені
C. Емпієма плеври
D. Каверна при гострому кавернозному туберкульозі
E. Каверна при фіброзно-кавернозному туберкульозі
15. На розтині тіла дівчини, 17 років, померлої внаслідок легеневої недостатності, виявлено: у нижній
частці правої легені зливні зони казеозного некрозу, в бронхопульмональних, бронхіальних та біфуркаційних
лімфатичних вузлах – ділянки казеозного некрозу. Який з перелічених діагнозів найбільш вірогідний?
A. @Ріст первинного афекту при первинному туберкульозі
B. Гематогенна форма прогресування первинного туберкульозу
C. Гематогенний туберкульоз з переважним ураженням легень
D. Туберкулема
E. Казеозна пневмонія при вторинному туберкульозі

16. На розтині тіла чоловіка, 34 років, діагностовано туберкульоз з наявністю у верхній частці правої
легені зони казеозної пневмонії, казеозного лімфаденіту у збільшених лімфатичних вузлах середостіння та
множинні міліарні осередки у багатьох органах. Назвіть виявлену форму туберкульозу.
A. @Первинний змішаної форми генералізaції
B. Первинний гематогенно-генералізований
C. Первинний лімфогенно-генералізований
D. Первинний з ростом первинного афекту
E. Гематогенний

17. Хворий, 27 років, який з дитинства страждав на туберкульоз, помер від легенево-серцевої
недостатності. На розтині у III сегменті правої легені субплеврально розташовано петрифікований фокус Гона,
лімфатичні вузли грудної та черевної порожнини збільшені, з чисельними петрифікатами та свіжими фокусами
казеозного некрозу. Виберіть правильний діагноз:
A. @Хронічно перебігаючий первинний туберкульоз
B. Вторинний туберкульоз
C. Туберкулема
D. Гематогенний туберкульоз
E. Туберкульозна інтоксикація

18. На розтині тіла чоловіка, 52 років, який тривалий час страждав на туберкульозний простатит і помер
від менінгоенцефаліту, в м'яких оболонках головного мозку, селезінці, нирках, печінці знайдено велику кількість
щільних сірого кольору вузликів діаметром 0,5-1мм; при гістологічному дослідженні вузлики складаються з
епітеліоїдних, лімфоїдних і нечисленних гігантських клітин з ядрами, розташованими на периферії клітин у
вигляді підкови. Знайдені зміни свідчать про:
A. Найгостріший туберкульозний сепсис
B. @Гострий міліарний туберкульоз
C. Хронічний міліарний туберкульоз
D. Великоосередковий дисемінований туберкульоз
E. Септикопіємію
19. На розтині трупа чоловіка, 44 років, який помер від легенево-серцевої недостатності, виявлено:
пневмосклероз, емфізему легень, гіпертрофію правого шлуночка серця. В обох легенях, переважно
субплеврально, множинні осередки діаметром до 1 см. Гістологічно: в центрі осередку – зона некрозу, на
периферії – вал з епітеліоїдних клітин і лімфоцитів з домішкою макрофагів і плазматичних клітин. Виявляються
гігантські клітини Пірогова-Лангханса. Невелике число кровоносних капілярів розміщена на периферії осередку.
Ваш діагноз:
A. @Гематогенний туберкульоз
B. Актиномікоз легень
C. Саркоїдоз
D. Сифіліс
E. Силікоз

20. При розтині тіла чоловіка, 48 років, померлого від туберкульозу легень, виявлені частково
випорожнені симетричні туберкульозні каверни у кортико-плевральних зонах з переважанням продуктивної
тканинної реакції; в обох легенях - ніжний сітчастий пневмосклероз та емфізема; “легеневе серце”; у
біфуркаційних лімфовузлах - туберкульозний казеозний лімфаденіт. У ІІІ сегменті правої легені - осередок Гона.
Вкажіть, який з перелічених діагнозів найбільш вірогідний?
A. @Гематогенно-дисемінований туберкульоз
B. Гематогенна генералізація первинного туберкульозу
C. Хронічний первинний туберкульоз
D. Хронічний міліарний туберкульоз
E. Циротичний туберкульоз

21. При мікроскопічному дослідженні біоптату шкіри виявляються гранульоми, які складаються з
епітеліоїдних клітин, оточених в основному Т-лімфоцитами. Серед епітеліоїдних клітин розташовуються
поодинокі гігантські багатоядерні клітини Пирогова-Лангханса. В центрі деяких гранульом виявляються ділянки
казеозного некрозу. Кровоносні судини відсутні. Для якого захворювання характерні описані гранульоми?
A. @Туберкульозу
B. Сифілісу
C. Лепри
D. Риносклероми
E. Сапу

22. У померлої дитини, 3 рокiв, за життя мала мiсце менiнгiальна симптоматика. На розтинi в м’якiй
мозковiй оболонцi макроскопiчно виявленi просоподiбнi вузлики, якi мiкроскопiчно представленi осередком
казеозного некрозу з валами епiтелiоїдних, лiмфоїдних клiтин, мiж якими зустрiчаються великi клiтини з ядрами
на периферiї у виглядi пiвмiсяця. Який найбiльш iмовiрний менiнгiт у дитини?
A. @Туберкульозний
B. Сифiлiтичний
C. Бруцельозний
D. Грипозний
E. Менiнгококовий
23. У померлого, 70 років, на секції виявлено збільшення та деформація правого колінного суглоба. При
гістологічному дослідженні тканин суглоба та прилягаючих ділянок виявлено: масивні осередки казеозного
некрозу, які оточені валами епітеліоїдних, лимфоїдних клітин з наявністю гігантських макрофагів. Ваш діагноз:
A. @Туберкульозний артрит
B. Сифілітичний артрит
C. Ревматоїдний артрит
D. Гонорейний артрит
E. Деформуючий остеоартроз

24. 10-річна дівчинка поступила в травматологічне відділення з симптомами патологічного перелому


правої стегнової кістки. В ході гістологічного дослідження операційного матеріалу виявлено, що в кістковому
мозку видаленого фрагменту стегнової кістки розташовані множинні осередки казеозного некрозу, оточені по
периферії епітеліоїдними клітинами та поодинокими клітинами Пирогова-Ланхганса; зони казеозного некрозу
поширюються і на прилягаючі ділянки кісткової тканини, руйнуючи її. Діагностуйте захворювання.
A. @Туберкульозний остеомієліт
B. Туберкульозний спондиліт
C. Туберкульозний коксит
D. Загальний крупноосередковий туберкульоз
E. Вторинний гострий осередковий туберкульоз

25. При УЗ-дослідженні у хлопця, 17 років, у правій нирці виявлені ознаки пієлоектазії з порушенням
відтоку сечі по сечоводу. Мікроскопічне дослідження біоптату нирки виявило на фоні дифузної інфільтрації
інтерстицію лімфоцитами і гістіоцитами в кірковій і мозковій речовині - осередки деструкції з утворенням
порожнин, оточені валом з епітеліоїдних, лімфоїдних клітин і домішкою гігантських клітин Пирогова-Лангханса.
Яке захворювання найбільш ймовірно?
A. Апостематозний нефрит
B. Неспецифічний тубуло-інтерстиціальний нефрит
C. Полікістоз нирки
D. @Туберкульоз нирки
E. Хронічний пієлонефрит

26. На розтині тіла чоловіка, 63 років, у II сегменті правої легені виявлено туберкульозний панбронхіт, з
осередком казеозної бронхопневмонії, навколо якого спостерігається вал епітеліоїдних клітин з домішкою
лімфоїдних і гігантських клітин Пирогова-Лангханса. Яка форма туберкульозу легень у даному випадку?
A. @Гострий осередковий туберкульоз
B. Первинний туберкульозний комплекс
C. Ріст первинного афекту
D. Гострий міліарний туберкульоз легенів
E. Інфільтративний туберкульоз

27. На розтині тіла чоловіка, 35 років, у другому сегменті правої легені виявлено осередок ущільнення
діаметром 5 см, оточений тонкою капсулою. Осередок представлений сухою крихкою тканиною з тьмяною
поверхнею. Для якого захворювання характерні такі морфологічні зміни?
A. Туберкулема
B. Рак легені
C. Хондрома
D. Туморозна форма силікозу
E. Післязапальний пневмосклероз

28. При розтині тіла померлого чоловіка, 48 років, у ділянці 1-го сегменту правої легені виявлено круглий
утвір діаметром 5 см з чіткими контурами, оточений тонким прошарком сполучної тканини, виповнений білими
крихкими масами. Діагностуйте форму вторинного туберкульозу:
A. Туберкулема
B. Казеозна пневмонія
C. Гострий кавернозний туберкульоз
D. Гострий осередковий туберкульоз
E. Фіброзно-кавернозний туберкульоз
29. У хворого, який помер від туберкульозу виявлено: в І сегменті правої легені осередок біло-сірого
кольору, оточений капсулою, діаметром 3 см. Мікроскопічно: ділянка некрозу з капсулою, відсутність
перифокального запалення. Назвіть форму туберкульозу.
A. @Туберкулема
B. Гострий кавернозний туберкульоз
C. Фіброзно-кавернозний туберкульоз
D. Циротичний туберкульоз
E. Казеозна пневмонія
30. У чоловіка, 50 років, який хворів на туберкульоз та помер при явищах легенево-серцевої
недостатності, при розтині виявлено лобарний характер ураження легень: верхня частка правої легені збільшена,
щільна, на розрізі жовтого кольору, крихкого вигляду, на плеврі – фібринозні нашарування. До якої з форм
вторинного туберкульозу відноситься вказана патологія?
A. @Казеозна пневмонія
B. Фіброзно-осередковий туберкульоз
C. Інфільтративний туберкульоз
D. Туберкулема
E. Гострий осередковий туберкульоз

31. У померлого, який прижиттєво страждав на прогресуючу форму вторинного туберкульозу, під час
аутопсії виявлено збільшену, щільну ліву легеню, на розрізі жовтого кольору з фібринозними накладеннями на
плеврі. Мікроскопічно виявлено переважання некротичних змін. Який з перелічених діагнозів найбільш
вірогідний?
A. @Казеозна пневмонія
B. Інфільтративний туберкульоз
C. Гострий осередковий туберкульоз
D. Фіброзно-осередковий туберкульоз
E. Фіброзно-кавернозний туберкульоз

32. Хворий, який страждав на туберкульоз помер від прогресуючої легенево-серцевої недостатності. На
розтині: в області верхівки правої легені визначається порожнина діаметром 5 см, яка сполучається з просвітом
сегментарного бронха. Стінки порожнини зсередини покриті сирнистими масами, під якими знаходяться
епітеліоїдні клітини і клітини Пирогова-Лангханса. Вкажіть морфологічну форму туберкульозу.
A. @Гострий кавернозний туберкульоз
B. Туберкулема
C. Казеозна пневмонія
D. Інфільтративний туберкульоз
E. Гострий осередковий туберкульоз

33. У чоловіка, 43 років, який тривалий час хворів на туберкульоз, розвинулась кровотеча з легень, що
призвела до смерті. На розтині в легенях виявлено декілька порожнин овальної або круглої форми, стінка яких
створена некротичними масами та тканиною легень. Для якої форми туберкульозу характерні такі зміни?
A. @Гострий кавернозний
B. Фіброзно-кавернозний
C. Туберкулема
D. Казеозна пневмонія
E. Гострий осередковий

34. Померлій жінці було виставлено клінічний діагноз: «Хронічний абсцес легень». При розтині виявлено
порожнину в ІІ-му сегменті правої легені розміром 5 см, округлої форми. Внутрішня поверхня порожнини була
сформована казеозними масами, зовнішня – щільною тканиною легень, піогенна мембрана відсутня. Як на вашу
думку, про який процес йде мова?
A. @Гострий кавернозний туберкульоз
B. Фіброзно-кавернозний туберкульоз
C. Гострий абсцес
D. Хронічний абсцес
E. Первинна легенева каверна

35. На розтині тіла чоловіка, 63 років, були виявлені множинні патологічні порожнини в обох легенях,
мікроскопічно у внутрішньому шарі стінки порожнини знайдені некротичні маси, які дифузно інфільтровані
нейтрофілами, середній - містить інфільтрат з епітеліоїдних клітин, лімфоцитів та багатоядерних гігантів,
зовнішній шар складається зі зрілої сполучної тканини. Діагностуйте форму туберкульозу.
A. @Фіброзно-кавернозний
B. Гострий осередковой
C. Фіброзно-осередковий
D. Гострий кавернозний
E. Циротичний

36. При розтині померлого, який хворів на туберкульоз, у верхній частці правої легені знайдено
порожнину розмірами 3х2 см, яка сполучається з бронхом. Стінка порожнини щільна, має три шари: внутрішній -
піогенний, середній - шар туберкульозної грануляційної тканини, зовнішній - сполучнотканинний. Який діагноз
найбільш імовірний?
A. @Фіброзно-кавернозний туберкульоз
B. Фіброзно-осередковий туберкульоз
C. Туберкулема
D. Гострий осередковий туберкульоз
E. Гострий кавернозний тубеокульоз

37. При розтині тіла чоловіка, 40 років, який 10 років хворів на туберкульоз, у І і II сегментах правої
легені виявлено порожнину з щільною стінкою. Внутрішня поверхня порожнини нерівна, з балками, які є
облітерованими бронхами і тромбованими судинами. Середня і нижня частки щільні, на розрізі з жовтими
осередками. Яку форму туберкульозу можна запідозрити в даному випадку?
A. @Фіброзно-кавернозний
B. Гострий кавернозний
C. Фіброзно-осередковий
D. Туберкулема
E. Циротичний
38. 40-річний ув’язнений помер у виправно-трудовій колонії від туберкульозу. При автопсійному
дослідженні тіла померлого встановлено деформацію і зменшення розмірів верхівок обох легень, множинні
порожнини з щільними стінками, товщиною 2-3 мм у верхніх частках обох легень, дисеміновані осередки
діаметром від 5 мм до 2 см казеозного некрозу у нижніх частках легень. Діагностуйте форму туберкульозу.
A. @Вторинний фіброзно-кавернозний
B. Вторинний фіброзно-осередковий
C. Гематогенний великоосередковий з ураженням легень
D. Первинний, ріст первинного афекту
E. Вторинний циротичний

39. Хворий на фіброзно-кавернозний туберкульоз легень помер від хронічної легенево-серцевої


недостатності. Впродовж останніх місяців спостерігалась виражена протеїнурія. На розтині: нирки збільшені,
щільні, восковидні з поверхні і на розрізі. Які зміни в нирках при цій формі туберкульозу могли викликати
протеїнурію?
A. @Амілоїдоз нирок
B. Казеозний некроз
C. Гломерулонефрит
D. Нефролітіаз
E. Некротичний нефроз

40. На рентгенограмі у легенях було виявлено фіброзно-осередковий туберкульоз, у верхівках – бульозну


емфізему. Раптово після фізичного напруження у хворого виникли сильний біль у правій половині грудної
клітини, задишка. Об’єктивно: ціаноз, правобічний тимпаніт, ослаблене дихання. Яке ускладнення туберкульозу
виникло у хворого?
A. @Спонтанний пневмоторакс
B. Арозивна кровотеча
C. Ателектаз легені
D. Гостре легеневе серце
E. Амілоїдоз

41. На слизовій оболонці правого піднебінного мигдалика спостерігається безболісна виразка з


гладеньким лакованим дном та рівними хрящоподібної консистенції краями. Мікроскопічно: запальний
інфільтрат, що складається з лімфоцитів, плазмоцитів, невеликої кількості нейтрофілів та епітеліоїдних клітин, та
наявність ендо- та периваскуліту. Про яке захворювання йдеться?
A. @Сифіліс
B. Актиномікоз
C. Туберкульоз
D. Дифтерія зіва
E. Виразково-некротична ангіна Венсана

42. У хворого на слизовiй оболонцi ясен - виразка овальної форми з припiднятими краями хрящоподiбної
щiльностi. Дно виразки м’ясисто-червоного забарвлення з нашаруваннями сiрого кольору. При мiкроскопiчному
дослiдженнi - пролiферацiя ендотелiю дрiбних судин, периваскулярна лiмфоплазмоцитарна iнфiльтрацiя. Про яке
захворювання йдеться?
A. @Сифiлiс
B. Травматична виразка
C. Виразка-рак
D. Ерозивно-виразкова лейкоплакiя
E. Виразково-некротичний гiнгiвiт

43. У хворої, 18 років, пахові лімфатичні вузли при пальпації збільшені, не болючі, ущільнені. У ділянці
слизової оболонки геніталій - невеликих розмірів виразка з ущільненими краями та "лакованим"дном сіруватого
кольору. Який найбільш вірогідний діагноз?
A. @Сифіліс
B. Туберкульоз
C. Лепра
D. Трофічна виразка
E. Гонорея

44. Чоловік, 35 років, помер у приймальному відділенні лікарні, куди був доставлений у шоковому стані.
На розтині тіла виявлено аневризму дуги аорти з її розривом та масивною кровотечею. Мікроскопічно: в медії
аорти скупчення лімфоцитів та плазмоцитів, поодинокі гігантські клітини Пирогова-Лангханса, фібробласти.
Спостерігаються також дрібні осередки некрозу та руйнування еластичних структур стінки аорти. Яке
захворювання зумовило такі зміни?
A. Сифіліс
B. Висипний тиф
C. Атеросклероз
D. Туберкульоз
E. -
45. У тканині печінки виявлено округле утворення діаметром 0,5 см. Мікроскопічно воно має наступну
будову: у центрі – некротичні маси, їх оточує грануляційна тканина з наявністю у її складі плазматичних,
лімфоїдних клітин і кровоносних судин з явищами васкуліту. Який діагноз необхідно поставити на підставі
даних мікроскопії?
A. @Солітарна гума печінки
B. Солітарна аденома печінки
C. Солітарна лепрома печінки
D. Хронічний абсцес печінки
E. Рак печінки

46. Під час розтину тіла чоловіка 61 року, було діагностовано аневризму висхідної частини та дуги аорти.
Гістологічно в стінці аорти виявлені значні плазмоклітинні інфільтрати та ділянки фрагментації еластичних
мембран та осередки еластолізису. Діагностуйте форму та період розвитку сифілісу.
A. @Набутий третинний
B. Набутий первинний
C. Набутий вторинний
D. Набутий нейросифіліс
E. Природжений пізній сифіліс

ГОСТРІ ПНЕВМОНІЇ. ХНЗЛ


ВАРІАНТ №

1. При електронномікроскопічному дослідженні покривного епітелію дихальних шляхів та слизової


оболонки середнього вуха в їх війках знайдені дефектні малорухомі мікротрубочки. Який з перелічених діагнозів
найбільш вірогідний?
A. @Синдром Картагенера
B. Хвороба Альцгеймера
C. Синдром Чедіака-Хігасі
D. Хвороба Вільсона-Коновалова
E. Синдром Целвегера

2. На секції в лівій легені виявлено ділянку зруйнованої тканини червоного кольору. Ділянка має форму
конуса, чітко відмежована від здорової тканини, основою ділянка обернена до плеври. Тканина ділянки щільна,
зерниста, темно-червона. Ваш діагноз.
A. @Геморагічний інфаркт
B. Абсцес легені
C. Гангрена легені
D. Первинний туберкульозний афект
E. Крупозна пневмонія

3. Під час розтину в верхній частці правої легені виявлено крупну клиноподібну темно-червону, ділянку
щільної тканини. При гістологічному дослідженні в ній виявлено: некроз стінок альвеол, просвіт альвеол щільно
заповнений еритроцитами. Який процес розвинувся у верхній частці правої легені?
A. @Геморагічний інфаркт легень
B. Карніфікація легень
C. Гангрена легень
D. Крововилив у легені
E. Ателектаз легень

4. У хворого, який страждав на хронічний гломерулонефрит, на тлі хронічної ниркової недостатності


з'явився кашель з відходженням слизового мокротиння. При бронхоскопії: слизова оболонка бронхів
повнокровна, набрякла, з дрібними крововиливами. У просвіті бронхів - багато слизу. Встановіть процес у
бронхах.
A. @Вторинний гострий катаральний бронхіт
B. Первинний гострий катаральний бронхіт
C. Хронічний катаральний бронхіт
D. Деструктивно-виразковий бронхіт
E. Ателектаз легень
5. На розтині померлого від легенево-серцевої недостатності хворого виявлено: в просвіті дрібних бронхів
- гнійний ексудат, гіпертрофія правих відділів серця. Гістологічно виявляються гіпертрофія слизових залоз,
лімфоплазмоцитарна запальна інфільтрація слизової оболонки бронхів, перибронхіальний склероз. Вкажіть
найбільш вірогідний діагноз.
A. @Хронічний бронхіт
B. Пневмонія
C. Пневмосклероз
D. Емфізема легень
E. Бронхіальна астма

6. У гістологічному висновку дослідження біоптату стінки бронха хворого на хронічний бронхіт: у


слизовому шарі - розростання грануляційної тканини, що виступають над його поверхнею та містять дифузний
запальний інфільтрат. Діагностуйте вид бронхіту:
A. Хронічний деформуючий
B. Хронічний слизово-гнійний
C. Хронічний слизовий
D. Хронічний гнійний
E. @Хронічний поліпозний

7. У біоптаті слизової оболонки бронха хворого, 50 років, який впродовж 20 років страждав на хронічний
бронхіт, виявлено: стоншення слизової оболонки, кістоподібне перетворення слизових залоз, осередки заміни
призматичного епітелію на багатошаровий плоский. Який з перелічених процесів найвірогідніший?
A. Гіперплазія
B. @Метаплазія
C. Гетеротопія
D. Гетероплазія
E. Дисплазія

8. Під час гістологічного дослідження легень хворого, який помер від серцевої недостатності, виявлені
осередки запалення з заповненням альвеол рідиною, забарвленою у блідо-рожевий колір, місцями з наявністю
тонких рожевих ниток, що утворюють дрібнопетлисту сітку з невеликою кількістю лімфоцитів. Який характер
ексудату у легенях?
A. @Серозно-фібринозний
B. Геморагічний
C. Серозний
D. Гнійний
E. Фібринозний

9. При розтині трупа чоловіка зі злоякісною пухлиною шлунка, померлого від ракової кахексії, у задніх
нижніх відділах легень виявлені щільні, сіро-червоного кольору, неправильної форми осередки, що виступають
над поверхнею розрізу. Мікроскопічно: в просвіті, стінках дрібних бронхів і альвеолах виявляється ексудат, в
якому багато нейтрофілів. Яке захворювання виявлено в легенях?
A. @Гостра гнійна бронхопневмонія
B. Гострий бронхіт
C. Крупозна пневмонія
D. Проміжна пневмонія
E. Гостра серозна бронхопневмонія

10. Хворий, 67 років, захворів на важку форму грипу, що завершилася летально. На розтині виявлені
“великі строкаті легені”. При мікроскопічному дослідженні виявлено різке повнокров'я судин, крововиливи,
набряк легеневої тканини, в просвітах бронхів і альвеол - ексудат, що містить переважно еритроцити. Про який
характер змін легень свідчать ці морфологічні ознаки?
A. Гнійна бронхопневмонія
B. Катаральна бронхопневмонія
C. @Геморагічна бронхопневмонія
D. Десквамативна бронхопневмонія
E. Фібринозна плевропневмонія

11. На розрізі легені виявлені множинні осередки до 2,5 см. в діаметрі неправильної форми, щільні,
безповітряні, без чітких меж, яскраво-червоного кольору. При мікроскопічному дослідженні в просвіті альвеол
виявлено велику кількість еритроцитів та невелику кількість лімфоцитів. Описані зміни характерні для:
A. @Грипозної пневмонії
B. Крупозної пневмонії
C. Геморагічного інфаркту легені
D. Корової пневмонії
E. Аспіраційної пневмонії

12. У 30 річного чоловіка, який хворів на гостре респіраторне захворювання та загинув при явищах
гострої легенево-серцевої недостатності, під час розтину виявлено: фібринозно-геморагічне запалення в слизовій
оболонці гортані та трахеї, деструктивний панбронхит, збільшені легені, яки мають пістрявий вигляд за рахунок
абсцесів, крововиливів, некрозу. Який з перелічених діагнозів найбільш вірогідний?
A. Респіраторно-синцитіальна інфекція
B. Парагрип
C. @Грип
D. Кір
E. Аденовірусна інфекція.

13. Чоловік, 42 років, помер при явищах вираженої інтоксикації і дихальної недостатності. На розтині:
тканина легень у всіх відділах строката, з множинними дрібноосередковими крововиливами та осередками
емфіземи. Гістологічно у легенях: геморагічна бронхопневмонія з абсцедуванням, у цитоплазмі клітин епітелію
бронхів - еозинофільні і базофільні включення. Діагностуйте виявлене на секції захворювання.
A. Парагрип
B. @Грип
C. Аденовірусна інфекція
D. Респіраторно-синцитіальна інфекція
E. Стафілококова бронхопневмонія
14. На розтині померлого виявлено, що вся нижня частка лівої легені брудно-сірого кольору, печінкової
консистенції, на плеврі - клаптик жовто-сірої тонкої плівки. Вкажіть етіологію захворювання:
A. @Стрептокок пневмонії
B. Тромбоз системи бронхіальних артерій
C. Мікобактерія туберкульозу
D. Бліда трепонема
E. Тромбоз системи легеневих артерій

15. При розтині померлого від пневмонії виявлено, що нижня частка правої легені щільна, безповітряна,
на розрізі - сірого кольору, на вісцеральній плеврі - накладення фібрину. Вкажіть найбільш вірогідну етіологію
пневмонії.
A. Стафілококова
B. @Пневмококова
C. Стрептококова
D. Мікоплазмова
E. Вірусна

16. 42-річний чоловік захворів гостро після охолодження. Захворювання супроводжувалось серцево-
дихальною недостатністю, від якої він помер. Під час розтину виявлено, що уся права легеня збільшена,
печінкової щільності, важка; на плеврі – значні фібринозні нашарування. На розрізі легеня сірого кольору,
зерниста, з поверхні стікає каламутна рідина. При гістологічному дослідженні – гостре запалення з наявністю у
просвітах альвеол гнійно-фібринозного ексудату. Який з наведених діагнозів найбільш вірогідний?
A. Осередкова пневмонія
B. @Крупозна пневмонія
C. Інтерстиціальна міжальвеолярна пневмонія
D. Стафілококова пневмонія
E. Ідіопатичний фіброзуючий альвеолі

17. У 45-річного хворого, який гостро захворів на пневмонію, на 6-й день хвороби розвинувся набряк
легень, який став причиною смерті. На розтині знайдено, що уражена вся верхня частка правої легені - збільшена,
щільна, на плеврі з фібринозними нашаруваннями, на розрізі сірого кольору, з поверхні розрізу стікає каламутна
рідина. Мікроскопічно: в просвіті альвеол - фібрин, нейтрофіли, макрофаги, гемолізовані еритроцити. Вкажіть,
яка пневмонія була у хворого?
A. @Крупозна пневмонія
B. Стафілококова бронхопневмонія.
C. Вірусна пневмонія
D. Гіпостатична пневмонія
E. Респіраторний дистрес-синдром дорослих

18. У хворого, померлого через 3 тижні від початку пневмонії, нижня частка правої легені різко
збільшена, щільна, безповітряна, сіра, з накладеннями фібрину на плеврі. При мікроскопії у всіх альвеолах цієї
частки виявлено фібрин та сегментоядерні лейкоцити. Ваш діагноз:
A. @Крупозна пневмонія
B. Осередкова бронхопневмонія
C. Грипозна пневмонія
D. Фібринозний плеврит
E. Інтерстициальна пневмонія

19. Верхня частка правої легені збільшена, сірого кольору, безповітряна, з поверхні розрізу стікає
каламутна рідина, на плеврі - багато фібринозних плівок; мікроскопічно в альвеолах виявляється ексудат з
присутністю нейтрофілів, десквамованих альвеолоцитів і ниток фібрину. Стінка бронха інтактна. Який найбільш
вірогідний діагноз?
A. @Крупозна пневмонія
B. Інтерстиціальна пневмонія
C. Абсцес легень
D. Осередкова пневмонія
E. Грипозна пневмонія
20. На секції хворого, який помер від наростаючої легеневої недостатності, виявлено наступні прояви:
легені червоного кольору, щільні, безповітряні, за щільністю нагадують печінку. Мікроскопічно: в альвеолах
значне накопичення еритроцитів і поліморфноядерних лейкоцитів, між клітинами - нитки фібрину. Ваш діагноз.
A. @Крупозна пневмонія
B. Вірусна геморагічна пневмонія
C. Гангрена легень
D. Периферичний рак легень
E. Циротичний туберкульоз

21. При розтинi тіла чоловiка, 50 рокiв, виявлено наступнi змiни: права легеня у всiх вiддiлах помiрно
щiльна, на розрiзi тканина безповiтряна, дрiбнозерниста, сухувата. Вiсцеральна плевра з нашаруванням фiбрину
сiро-коричневого кольору. Який найбiльш iмовiрний дiагноз?
A. @Крупозна пневмонiя
B. Туберкульоз
C. Бронхопневмонiя
D. Iнтерстицiальна пневмонiя
E. Пневмофiброз

22. На розтині виявлено, що вся права легеня збільшена, щільна, на плеврі - нашарування фібрину, на
розрізі тканина сірого кольору, з якої стікає каламутна рідина. Для якого захворювання легень характерна дані
зміни?
A. Інрерстиціальна пневмонія
B. Бронхопневмонія
C. @Крупозна пневмонія
D. Гангрена легені
E. Фіброзуючий альвеоліт
23. При розтині хворого, який помер від крупозної пневмонії, в плевральній порожнині - непрозора
рідина, на вісцеральній плеврі - сіруватого кольору плівка. Визначити вид запалення на вісцеральнiй плеврi.
A. @Фiбринозне
B. Катаральне
C. Гнiйне
D. Гранульоматозне
E. Геморагiчне

24. При розтині тіла померлого, 49 років, який хворів на крупозну пневмонію і помер від пневмококового
сепсису, в лівій плевральній порожнині містилось до 700 мл каламутної рідини зеленувато-жовтого кольору з
неприємним запахом. Листки плеври тьмяні, повнокровні. Назвіть клініко-морфологічну форму запалення в
плевральній порожнині:
A. @Емпієма
B. Хронічний абсцес
C. Гострий абсцес
D. Флегмона
E. Фібринозне запалення
25. У хворого висока температура, задуха, біль у правій частині грудної клітини. Плевральна пункція дала
700 мл в’язкої рідини жовто-зеленого кольору. Який патологічний процес розвився у плевральній порожнині?
A. @Емпієма плеври
B. Бронхопневмонія
C. Серозний плеврит
D. Геморагичний плеврит
E. Карциноматоз плеври

26. У хворого, віком 30 років, гіпертермія, задуха, біль у лівій половині грудної клітини. Під час
плевральної пункціїї видалено 500 мл вершкоподібної рідини жовтувато-зеленого кольору. Найвірогідніший
діагноз:
A. @Емпієма плеври
B. Карциноматоз плеври
C. Серозний плеврит
D. Фібринозний плеврит
E. Геморагічний плеврит

27. На розтині хворого, який помер від двобічної бронхопневмонії у нижній частці лівої легені виявлено
порожнину діаметром 5 см, заповнену рідкими жовтувато-білими масами. Яке ускладнення розвинулось при
даному виді пневмонії?
A. @Абсцес
B. Гангрена
C. Гранульома
D. Секвестр
E. Туберкулема

28. Чоловік, 52 років, на 10 добу лікування гострої нижньочасточкової правобічної стафілококової


пневмонії помер при явищах наростаючої серцево-легеневої недостатності. На аутопсії у нижній частці правої
легені виявлено порожнину неправильно округлої форми приблизно 5 см в діаметрі, що містить гній і що
сполучається з бронхом. Яке ускладнення гострої пневмонії має місце у хворого?
F. Тромбоемболія легеневого стовбура
G. Гангрена легені
H. @Абсцес легені
I. Бронхоектатична хвороба
J. Гострий бронхіт
29. У хворого з легеневою патологією на розтині виявлено порожнину з ущільненими стінками,
виповнену густою рідиною зеленуватого кольору, з непрємним запахом. Поставити діагноз.
A. @Абсцес легень
B. Гангрена легень
C. Iнфаркт легень
D. Каверна легень
E. Туберкулома легень

30. Чоловік, 60 років, хворів на пневмококову плевропневмонію і помер від серцево-судинної


недостатності та інтоксикації. При розтині: в середній частці правої легені виявлено порожнину розмірами 5х5х7
см неправильної форми з нерівними фестончатими краями, заповнену жовто-зеленою в'язкою масою. Навколо
порожнини легенева тканина всієї частки щільна, сірого кольору без пористої структури. Яке ускладнення
плевропневмонії виникло в середній частці легені у хворого?
A. @Гострий абсцес
B. Хронічний абсцес
C. Гостра каверна
D. Хронічна каверна
E. Кіста

31. У хворого діагностовано порожнисте утворення легені діаметром 2 см. Встановлено, що стінка
порожнини утворена щільною волокнистою сполучною тканиною, вміст являє собою каламутну, густу,
жовтувато-зеленуватого кольору рідину з неприємним запахом, яка мікроскопічно складається переважно з
поліморфноядерних лейкоцитів. Якому патологічному процесу відповідають такі зміни?
A. @Хронічному абсцесу
B. Гострому абсцесу
C. Емпіємі
D. Флегмоні
Е. Карбункулу

32. Хворий помер від наростаючої легенево-серцевої недостатності. При гістологічному дослідженні
виявлено: дифузне ураження легень з інтерстиціальним набряком, інфільтрацією інтерстиціальної тканини
лімфоцитами, макрофагами, плазмоцидами; пневмофіброз, панацинарна емфізема. Який найбільш вірогідний
діагноз?
A. @Фіброзуючий альвеоліт
B. Хронічний бронхіт
C. Бронхопневмонія
D. Ателектаз легенів
E. Бронхіальна астма

33. Хвора, 38 років, померла під час нападу бронхіальної астми, який не вдалося купірувати. Під час
гістологічного дослідження в просвіті бронхів виявлені скупчення слизу, в стінці бронхів багато тучних клітин
(лаброцитів), численні - у стані дегрануляції, а також багато еозинофілів. Який патогенез цих змін у бронхах?
A. @Атопія
B. Цитотоксична, цитолітична дія антитіл
C. Імунокомплексний механізм
D. Клітинно обумовлений цитоліз
E. Гранулематоз
34. У хворої на бронхіальну астму вірусне інфікування спровокувало астматичний статус зі смертельним
наслідком. При гістологічному дослідженні легень виявлено спазм і набрякання бронхіол, в їх стінках - виражена
інфільтрація лімфоцитами, еозинофілами та іншими лейкоцитами, а також дегрануляція лаброцитів. Який
механізм гіперчутливості лежить в основі описаних змін?
A. @Реагінова реакція гіперчутливості
B. Запальний
C. Аутоімунний
D. Імунокомплексний
E. Імунозумовлений клітинний цитоліз
35. Спадкова схильність до розвитку емфіземи легень пов’язана з генним порушенням, яке проявляється у
зменшенні продукції:
A. @α-1-антитрипсина
B. С-реактивного білка
C. Антистрептолізину-О
D. Гіалуронідази
E. Комплемента

36. Чоловiк, 50 рокiв, хворiє на хронiчний бронхiт, скаржиться на задишку пiд час фiзичного
навантаження, постiйний кашель з вiдходженням харкотиння. При обстеженнi дiагностовано ускладнення -
емфiзема легень. Чим вона зумовлена?
A. @Зниження еластичних властивостей легень
B. Зменшення альвеолярної вентиляцiї
C. Зменшення розтяжностi легень
D. Зменшення перфузiї легень
E. Порушення вентиляцiйно-перфузiйного спiввiдношення в легенях

37. На розтині прозектор виявив, що легені збільшені за розмірами, бліді, м'якої консистенції, не
спадаються, ріжуться з хрускотом. Мікроскопічно - розширення альвеолярних ходів, міжальвеолярні перегородки
тонкі, є ознаки інтракапілярного склерозу. Для якого захворювання легень характерна така морфологічна
картина?
A. @Емфізема
B. Пневмосклероз
C. Пневмоторакс
D. Ателектаз
E. Пневмонія
38. Хворий, який тривало страждав на захворювання легень помер від наростаючої легенево-серцевої
недостатності. На розтині легені збільшені в об’ємі, не спадаються. На розрізі: тканина нагадує великочарункову
губку, на тлі якої визначаються крупні бульозні утворення. Мікроскопічно відмічається значне розширення
альвеол і розпад міжальвеолярних перегородок. Вкажіть вид легеневої патології.
A. @Емфізема легень
B. Хронічний бронхіт
C. Пневмосклероз
D. Пневмонія
E. Бронхіальна астма

39. На розтині померлого від легенево-серцевої недостатності, обумовленої хронічним дифузним


бронхітом, виявлені великі, підвищеної повітряності, легені, що прикривають середостіння, блідо-сірого кольору,
ріжуться з хрускотом, при натисканні пальцем на поверхні легень залишається ямка. З просвіту бронхів
виділяється слизово-гнійний ексудат. Встановіть діагноз.
A. @Хронічна дифузна обструктивна емфізема легень
B. Хронічна осередкова емфізема легень
C. Інтерстиціальна проміжна емфізема легень
D. Первинна ідіопатична емфізема легень
E. Вікарна компенсаторна емфізема легень

40. При розтині померлого від хронічної ниркової недостатності, у зв'язку з амілоїдозом нирок, чоловіка,
34 років, патологоанатом в легенях виявив (переважно в нижніх частках): множинні дифузні розширення бронхів,
в просвітах яких - гнійний вміст. Поверхня розрізу легень має дрібнокомірковий вигляд, нагадує бджолині
стільники. Гістологічно в стінці розширених бронхів - хронічне запалення, м'язові волокна заміщені сполучною
тканиною. Ці зміни в легені лікар розцінив як:
А. Хронічний бронхіт
В. Бронхопневмонія
С. @Бронхоектази
D. Хронічна пневмонія
E. Абсцеси легень
41. У дитини, 8 років, виражені ознаки легенево-серцевої недостатності, задуха, пальці у вигляді
барабанних паличок, нігті у вигляді годинкових скелець, кашель зі значною кількістю слизово-гнійного
харкотиння (особливо вранці) При рентгенологічному дослідженні виявлено різке розширення бронхів.
Діагностуйте захворювання.
A. @Бронхоектатична хвороба
B. Бронхіальна астма
C. Хронічний бронхіт
D. Бронхопневмонія
E. Емфізема легень

42. При аутопсії померлого, який тривалий час хворів на бронхоектатичну хворобу, виявлені різко
збільшені нирки за рахунок коркової речовини. Вони щільні, бліді, мають сальний вигляд. Мікроскопічно за
ходом ретикулярної строми, в стінках судин – відкладання безструктурних, гомогенних, еозинофільних мас, які
конго-рот-позитивні. Ці зміни характерні для:
A. Гіалінозу
B. Мукоїдного набухання
C. Фібриноїдного набухання
D. Ліпідозу
E. @Амілоїдозу

43. Юнак, 19 років, з раннього дитинства страждав на бронхоектатичну хворобу. Помер від ниркової
недостатності. На розтині: окрім множинних бронхоектатичних каверн, заповнених гнійним ексудатом виявлено
збільшені нирки, щільної консистенції, корковий шар потовщений, білого кольору, щільний. Піраміди нирки -
анемічні, чіткі. Назвати процес, який розвинувся в нирках?
A. Вроджений полікістоз нирок
B. Гломерулонефрит
C. Хронічний пієлонефрит
D. @Вторинний амілоїдоз
E. Вторинний нефросклероз
44. У хворого внаслідок обтурації середньочасточкового бронха вузлом м'яких тканин утворився
ателектаз середньої частки правої легені. При бронхобіопсії в зоні обтурації виявлені розростання
залозоподібного атипового епітелію з патологічними мітозами, який проростає в підлеглі тканини і хрящ. Якому
захворюванню найбільш вірогідно відповідають ці дані?
A. @Бронхогенний рак легень
B. Деформуючий бронхіт
C. Запальний поліп
D. Дисплазія епітелію бронха
E. Саркома бронха

45. При мікроскопічному дослідженні біоптату легень виявлено атипові клітини, які формують множинні
ацинарні структури і продукують слиз. Яка гістологічна форма раку легень має місце у хворого?
A. @Високодиференційована аденокарцинома
B. Помірнодиференційована аденокарцинома
C. Недиференційований рак
D. Помірнодиференційований плоскоклітинний рак
E. Залозисто-плоскоклітинний рак

46. При мікроскопічному дослідженні бронхобіопсії виявлено пухлину, яка побудована з гніздових
скупчень атипових клітин багатошарового плоского епітелію, місцями з характерними “перлинами”. Ваш
діагноз?
A. @Плоскоклітинний рак із зроговінням
B. Плоскоклітинний рак без зроговіння
C. Солідний рак
D. Слизовий рак
E. Скір

47. У хворого, 67 років, з клінічним діагнозом «Хронічний бронхіт, пневмосклероз, серцево-легенева


недостатність» взято біоптат з підозрілої ділянки слизової правого бронха. Гістологічно встановлено: клітинний і
тканинний атипізм, появу структур у вигляді ”ракових перлин”. Якому патологічному процесу відповідають
зазначені гістологічні зміни?
A. @Плоскоклітинний рак бронха зі зроговінням
B. Хронічний поліпозний бронхіт
C. Бронхоектаз
D. Гострий бронхіт
E. Плоскоклітинна метаплазія слизової бронха

48. При бронхоскопії у хворого, 62 років, у початковому відділі верхньочасточкового бронха правої легені
виявлено поліпоподібне утворення діаметром до 1,0 см. При гістологічному дослідженні виявлено пухлину, яка
побудована з дрібних лімфоцитоподібних клітин з гіперхромними ядрами, клітини ростуть пластами та тяжами.
Вкажіть, який з перелічених видів пухлин найбільш вірогідний?
A. @Недиференційований дрібноклітинний рак
B. Недиференційований великоклітинний рак
C. Плоскоклітинний рак
D. Аденокарцинома
E. Залозисто-плоскоклітинний рак

47. У хворого з вираженим пневмосклерозом пiсля перенесеного iнфiльтративного туберкульозу легень


розвинулась дихальна недостатнiсть. До якого патогенетичного типу вона вiдноситься?
A. @Рестриктивний
B. Обструктивний
C. Дисрегуляцiйний
D. Рефлекторний
E. Апнеїстичний
50. Чоловік, віком 62 років, помер внаслідок декомпенсації хронічного легеневого серця, на секції було
виявлено, що легені збільшені, їх тканина містить щільні вузли, овальної форми і темно-сірого або чорного
кольору. Гістологічно у вузлах виявлялася сполучна тканина та значні скупчення коніофагів, у цитоплазмі яких
були скупчення екзогенного пігменту. Діагностуйте захворювання.
A. @Пневмоконіоз
B. Осередкова пневмонія
C. Крупозна пневмоія
D. Бура індурація легень
E. Пневмоцироз

51. У легенях пацієнта, який впродовж 9-ти років працював шліфувальником каменю, виявлені дрібні
округлої форми щільні вузлики, що складаються зі сполучної тканини. На периферії цих вузликів розташовані
макрофаги. Проявом якого захворювання є зміни в легенях?
A. @Силікоз
B. Гостра пневмонія
C. Бронхоектатична хвороба
D. Хронічний бронхіт
E. Бронхіальна астма
52. Пiд час розтину трупа шахтаря, який проробив у шахтi бiльше 10-ти рокiв, у легенi виявленi тяжi
бiлуватої волокнистої тканини i вузлики 0,2-0,3 см у дiметрi. При гiстологiчному дослiдженнi у вузликах
невелика кiлькiсть коричневого пилу, концентричнi розростання сполучної тканини, бiдної на клiтини, з
вираженим гiалiнозом. Про який пневмоконiоз слiд думати у данному випадку?
A. @Силiкоз
B. Талькоз
C. Азбестоз
D. Сидероз
E. Берилiоз
53. Мікроскопічно у видаленому сегменті легені шахтаря знайдені чисельні, округлі вузлики, побудовані з
концентричних гіалінізованих пучків сполучної тканини. Більша частина їх розташована периваскулярно та
перибронхіально. Найбільш вірогідний діагноз:
A. @Силікоз
B. Туберкульоз
C. Бронхіт
D. Фіброзуючий альвеоліт
E. Рак легень
54. На розтині померлого, що багато років працював на підприємстві з високим рівнем вільного двоокису
кремнію у повітрі, знайдені збільшені та ущільнені в об'ємі легені зі значною кількістю міліарних і більш великих
осередків склерозу округлої та овальної форми сірого або сіро-чорного кольору. Який з перелічених діагнозів
найбільш вірогідний?
A. @Вузлувата форма силікозу
B. Дифузно-склеротична форма силікозу
C. Антракосилікоз
D. Силікоантракоз
E. Азбестоз

55. При розтині тіла шахтаря та гістологічному дослідженні у легенях було виявлені численні тонкі тяжі
сполучної тканини з осередками склерозу навколо бронхів та судин легень; розростання сполучної тканини в
альвеолярних перегородках з розвитком бронхіоліту та бронхоектазів. У біфуркаційних лімфовузлах – поодинокі
лімфовузли зі склерозом їх тканини та великою кількістю частинок пилу та коніофагів. Вкажіть форму силікозу,
виявлену під час розтину.
A. @Дифузно-склеротична
B. Вузликова
C. Змішана
D. Силікотична кавернозна
E. Силікотуберкульоз

56. У померлого, що понад 20 років працював на шахті зі здобуття кам’яного вугілля, при розтині
знайдені ущільнені легені сіро-чорного кольору зі значними ділянками новоутвореної сполучної тканини та
наявністю великої кількості макрофагів з пігментом чорного кольору у цитоплазмі. Який з перелічених діагнозів
найбільш вірогідний?
A. @Антракоз
B. Антракосилікоз
C. Силікоантракоз
D. Талькоз
E. Сидероз

57. На розтині померлого від хронічної легенево-серцевої недостатності шахтяря, виявлено, що легені
малоповітряні, значно ущільнені, склерозовані, верхівки - емфізематозно змінені, поверхня - сіро-чорного
кольору, на розрізі тканина легень аспідно-чорного кольору. Від якої хвороби настала смерть хворого?
A. @Антракоз
B. Силікоз
C. Талькоз
D. Асбестоз
E. Алюміноз

58. Хворий з 30-рiчним стажем роботи у шахтi помер вiд легенево-серцевої недостатностi, що наростала.
На розтинi легенi збiльшенi за розмiрами, щiльної консистенцiї, на розрiзi у них велика кiлькiсть вузликiв
розмiрами з просянi зерна i бiльше, щiльних, сiруватого i сiрувато-чорного кольору, мiсцями вузлики зливаються
у бiльш великi дiлянки. Назвiть найбiльш iмовiрне захворювання:
A. @Антрако-силiкоз
B. Азбестоз
C. Алюмiноз
D. Берилiоз
E. Сидероз
59. У чоловіка, віком 48 років, шахтаря, який добував гематит, на розтині виявлено збільшені легені,
буро-червоного кольору. Мікроскопічно: помірний пневмосклероз, субміліарні та міліарні вузлики, які
складаються з пилових клітин з частинками, що дають позитивну реакцію на залізо. В лімфатичних вузлах багато
пилу і значний дифузний склероз. Якому з професійних захворювань відповідає описана картина?
A. @Червоний сидероз
B. Чорний сидероз
C. Алюміноз
D. Бериліоз
E. Антракоз

60. Чоловік, 47 років, поступив у діагностичне відділення профпатології для уточнення характеру
пневмоконіозу. При вивченні біоптату легені виявлено багато дрібних гранульом, які локалізуються
субплеврально в інтерстиціальній тканині навколо дрібних судин і бронхів. Гранульоми складаються з
епітеліоїдних, лімфоїдних, плазматичних клітин, а також клітин типу Лангханса або гігантських клітин сторонніх
тіл. У гранульомах і всередині гігантських клітин розташовуються голчасті кристалічні включення (спікули) від 3
до 10 нм у довжину, які мають подвійну променезаломлюваність у поляризованому світлі і складаються з
карбонату кальцію. Виявлення цих структур дозволило лікарю поставити діагноз:
A. @Хронічний бериліоз
B. Гострий бериліоз
C. Азбестоз
D. Антракоз
E. Силікоз

61. У хворого, який тривалий час страждав на хронічний бронхіт, розвинулась хронічна серцева
недостатність. На розтині в легенях: емфізема та пневмосклероз. Які зміни в серці розвинулися в даному
випадку?
A. Гіпертрофія лівого шлуночка
B. @Гіпертрофія правого шлуночка
C. Гіпертрофія всіх відділів серця
D. Дифузний кардіосклероз
E. Атрофія міокарда

62. Хворий на хронічну обструктивну емфізему легень помер від легенево-серцевої недостатності. Які
зміни можливо виявити в серці?
A. Гіпертрофію лівого шлуночка
B. @Гіпертрофію правого шлуночка
C. Амілоїдоз
D. Великоосередковий кардіосклероз
E. Розрив правого шлуночка
63. В анамнезі у хворого, віком 62 років, з дитинства спостерігались хронічні запальні хвороби легень.
Хворий потрапив до лікарні з ознаками легеневої та серцевої недостатності і при наростанні останньої помер. Які
зміни в серці було виявлено на розтині?
A. @Гіпертрофію та дилатацію правого шлуночка
B. Гіпертрофію та дилатацію лівого шлуночка
C. Гіпертрофію обох шлуночків
D. Дилатацію обох шлуночків
E. Серце без макроскопічних змін

64. Чоловік, 63 років, який впродовж 15 років страждав на хронічну дифузну обструктивну емфізему
легень, помер від прогресуючої серцевої недостатності. На розтині виявлені: мускатний цироз печінки,
ціанотична індурація нирок та селезінки, асцит, набряки нижніх кінцівок. Визначте тип серцевої недостатності
при даних змінах.
A. Гостра правошлуночкова
B. @Хронічна правошлуночкова
C. Хронічна лівошлуночкова
D. Гостра лівошлуночкова
E. Загальна серцева

ІНФЕКЦІЙНІ ЗАХВОРЮВАННЯ. СЕПСИС. БАКТЕРІАЛЬНІ КИШКОВІ ІНФЕКЦІЇ.


ВАРІАНТ №
1. При розтині трупа чоловіка, 40 років, виявлено різко розширений просвіт тонкої кишки, переповнений
рідиною, яка нагадує “рисовий відвар”. Стінка кишки набрякла, на слизовій - велика кількість дрібнокрапчастих
крововиливів. Для якого інфекційного захворювання характерний описаний ентерит?

A. @Холера
B. Чума
C. Бруцельоз
D. Туляремія
E. Сибірка

2. При розтині хворого, померлого через тиждень від початку профузної діареї, виявлено різко виражений
ексикоз - усі тканини сухі, кров густа. При бактеріологічному дослідженні вмісту тонкої кишки, що нагадує
рисовий відвар, знайдені вібріони. Яке захворювання призвело хворого до смерті?

A. @Холера
B. Дизентерія
C. Черевний тиф
D. Сальмонельоз
E. Харчова токсикоінфекція

3. У хворого впродовж двох днів відмічались профузна діарея, блювота; смерть настала від різкого
зневоднення. На розтині: стінка кишки набрякла, гіперемована з множинними крововиливами в слизовій
оболонці. В просвіті кишки міститься біляста рідина, що нагадує рисовий відвар. Вкажіть захворювання яке
розвилось у хворого.

A. @Холера
B. Дизентерія
C. Сальмонельоз
D. Черевний тиф
E. Ентероколіт

4. Чоловік, 36 років, знаходився в інфекційній лікарні з профузною діареєю, ознаками ексикозу, падінням
температури тіла. Помер від уремії. Під час розтину знайдено: у просвіті тонкої кишки безкольорову рідину у
вигляді рисового відвару; слизова оболонка набрякла. При мікроскопічному дослідженні тонкої кишки –
повнокров'я судин, осередкові крововиливи, десквамація ентероцитів, гіперсекреція келихоподібних клітин та
лімфо-лейкоцитарна інфільтрація строми слизової оболонки. Назвіть захворювання.

A. @Холера
B. Сальмонельоз
C. Дизентерія
D. Черевний тиф
E. Хвороба Крона
5. У хворого, 35 років, який перебував в Індії, захворювання почалося гостро з раптового проносу.
Випорожнення були водянистими. Пізніше приєдналася багаторазова блювота, шкіра стала зморшкуватою,
спостерігалися судоми, сильна спрага, задишка. При бактеріологічному дослідженні виділено збудника
захворювання – вібріон Ель-Тор. При гістологічному дослідженні ентеробіоптату - серозно-геморагічний
гастроентерит з десквамацією епітелію, з ділянками його набуханням, вакуолізації цитоплазми та деструкції
мікроворсинок. Яке ускладнення може розвинутися при холерній діареї?
A. @Гіповолемічний шок
B. Анемія
C. Тромбогеморагічний синдром
D. Кахексія
E. Уремія

6. Хворий, 58 років, помер від холерної коми, що супроводжувалася різким зниженням температури тіла і
артеріального тиску, прогресуючою олігурією. На розтині виявлені ознаки вираженого зневоднення організму:
“поза гладіатора”, “руки прачки”, “біла жовч”. Вказати період холери, що характеризується проявом ексикозу:

A. Холерний ентерит
B. Холерний гастроентерит
C. @Алгідний період
D. Холерний тифоїд
E. Посталгідний період

7. При розтині трупа чоловіка, який хворів на черевний тиф, виявлені зміни у тонкій кишці: групові
лімфоїдні фолікули збільшені, виступають над поверхнею слизової оболонки, вони сіро-червоні, соковиті, їх
поверхня має вигляд звивин та перегородок. При мікроскопічному дослідженні відмічається утворення
черевнотифозних гранульом. Вкажіть, яка з перелічених стадій черевного тифу найбільш вірогідна?

A. @Мозкоподібного набухання
B. Некрозу
C. Утворення виразок
D. Чистих виразок
E. Загоювання

8. На розтині тіла чоловіка, який помер на 5-у добу захворювання на черевний тиф, виявлені наступні
зміни: групові фолікули клубової кишки збільшені, повнокровні і виступають над слизовою оболонкою, на їх
поверхні видно борозни та звивини. Гістологічно: повнокров’я і набряк тканини, наявність великих клітин зі
світлою цитоплазмою, які містять черевнотифозні палички. Про який період місцевих змін при черевному тифі
можна думати?

A. @Стадія мозкоподібного набухання


B. Стадія некрозу
C. Стадія загоювання виразок
D. Стадія чистих виразок
E. Стадія утворення виразок
9. На розтині померлого було виявлено наявність крові в тонкій та товстій кишках (1,5-2 л). У нижніх
відрізках клубової кишки стінка мала "брудні" виразки на місці групових лімфатичних фолікулів. Решта
пейєрових бляшок була зеленуватого кольору з демаркаційним запаленням навколо. Було діагностовано черевний
тиф. Для якого періоду черевного тифу найбільш характерні описані морфологічні зміни кишки й ускладнення?

A. @Утворення виразок
B. Загоювання виразок
C. Чистих виразок
D. Некрозу
E. Мозкоподібного набухання

10. На розтині тіла чоловіка, померлого від внутрішньoкишкової кровотечі, в клубовій кишці
спостерігається некроз групових і солітарних фолікулів, з імбібіцією жовчю і кров’ю мертвих тканин; у
нижньому відрізку кишки – явища секвестрації та відторгнення некротичних мас з утворенням дефектів. Який
Ваш діагноз?

A. @Черевний тиф, стадія "брудних" виразок


B. Черевний тиф, стадія чистих виразок
C. Черевний тиф, стадія некрозу
D. Черевнотифозна форма сальмонельозу
E. Хвороба Крона

11. У хворого, 50 років, перебіг черевного тифу, на третьому тижні від початку захворювання,
ускладнився внутрішньокишковою кровотечею. Назвіть стадію захворювання, що призвела до кровотечі?

A. Мозкове набухання
B. Стадія некрозу черевнотифозних гранульом
C. @Стадія брудних виразок
D. Стадія чистих виразок
E. Стадія загоєння виразок

12. На розтині померлого від черевного тифу у здухвинній кишці виявлені розташовані повздовж неї
виразки з рівними краями, чистим дном, яке утворено м’язовим шаром або навіть серозною оболонкою кишки, а
також поодинокі перфорації. На якій стадії черевного тифу це спостерігається?

A. @Стадія “чистих” виразок


B. Стадія мозкоподібного набряку
C. Стадія некрозу
D. Стадія “брудних” виразок
E. Стадія загоєння виразок

13. При розтині тіла померлого в інфекційному відділенні виявлено: фібринозно-гнійний перитоніт; у
слизовій оболонці клубової кишки багаточислені виразкові дефекти овальної форми до 3-5 см, які розташовані
вздовж кишки і повторюють форму пейєрової бляшки, краї виразок рівні, заокруглені, дно чисте, представлене
м’язовою або серозною оболонкою. В дні двох виразок знайдені отвори до 0,3 см в діаметрі. Для якого
захворювання найбільш характерні ці зміни?

A. @Черевний тиф
B. Дизентерія
C. Неспецифічний виразковий коліт
D. Хвороба Крона
E. Паратиф

14. На розтині трупа чоловіка, 56 років, у термінальному відділі тонкої кишки виявлено декілька виразок
розмірами від 4 до 5 см. Краї виразок піднімаються над поверхнею слизової оболонки, стінки виразок покриті
сірувато-жовтуватими масами, які кришаться. Реакція Відаля позитивна. Поставте діагноз.

A. @Черевний тиф
B. Паратиф
C. Поворотний тиф
D. Дизентерія
E. Хвороба Крона

15. У чоловiка, 43 рокiв, який помер у хiрургiчному вiддiленнi при явищах розлитого гнiйного перитонiту,
на розтинi у дистальному вiддiлi тонкого кишечника виявленi пейєровi бляшки, що виступають у просвiт,
поверхня деяких з них вкрита струпом, коричнево-зеленуватого кольору, у центрi деяких бляшок наявнi глибокi
дефекти, що сягають серозного шару. Для якого захворювання характернi дані змiни?

A. @Черевний тиф
B. Дизентерiя
C. Туберкульоз кишечника
D. Хвороба Крона
E. Стафiлококовий ентерит

16. При розтині померлого хворого від розповсюдженого перитоніту в дистальних відділах тонкої кишки
виявлено численні виразки овальної форми, які розташовані вздовж кишки. Дно виразок чисте, гладеньке,
утворене м'язовою або серозною оболонкою, краї виразок рівні, закруглені. У двох виразках є перфоративні
отвори діаметром до 0,5 см. Яке захворювання можна запідозрити у померлого хворого?

A. @Черевний тиф
B. Дизентерія
C. Холера
D. Туберкульоз
E. Висипний тиф

17. Жінка, 28 років, захворіла гостро. Температура тіла підвищилася до 40ºС. Втратила свідомість, були
зорові та слухові галюцінації, марення. На 9-й день хвороби на шкірі грудей і живота з’являється екзантема -
незначні висипання блідо-рожевого кольору (розеоли), розміром проса, яке зникає при натискуванні. На 18-й
день хвороби виявлені ознаки “гострого живота’’, діагностовано перитоніт. Після чого хвора померла. На
аутопсії у клубовій кишці виявлені глибокі виразки в ділянці групових фолікулів, одна з яких перфорувала. У
черевній порожнині - фібринозно-гнійний ексудат. Поставте діагноз.

A. @Черевний тиф
B. Лямбліоз
C. Кампілобактерійний ентероколіт
D. Дизентерія
E. Амебіаз

18. На розтині, померлого від інтоксикації хворого в тонкій кишці знайден набряк групових лімфатичних
фолікулів, виступаючих над поверхнею слизової оболонки у вигляді м'яко-еластичних бляшок з нерівною
поверхнею у вигляді борозен і звивин, що нагадує поверхню мозку. Який діагноз найбільш можливий?

A. @Черевний тиф
B. Дизентерія
C. Сальмонельоз
D. Гострий ентерит
E. Холера

19. Смерть хворого, 16 років, настала від розлитого фібринозно-гнійного перитониту. На розтині: в
нижній ділянці тонкої кишки виявлено виразку, яка повторювала форму пейєрової бляшки, з перфорацією стінки
кишки. Мікроскопічне дослідження виявило: стертість малюнка лімфоїдної тканини, витіснення її
проліферуючими моноцитами, які формують гранульоми. Ускладненням якого захворювання є причина смерті
хворого:

A. @Черевного тифу
B. Дизентерії
C. Холери
D. Бруцельозу
E. Неспецифічного виразкового коліту

20. Хворий, 50 років, захворів гостро: з’явився гострий біль у животі, підвищилась температура тіла до
38,5°С. Був направлений у хірургічний стаціонар з діагнозом “Гострий перитоніт”. При лапаротомії в черевній
порожнині виявлено фібринозно-гнійний ексудат (розлитий перитоніт), у низхідному відділі тонкої кишки -
перфоративні виразки, значне збільшення мезентеріальних лімфатичних вузлів. При мікроскопічному
дослідженні видаленої ділянки кишки визначаються специфічні гранульоми з ділянками некрозу, в складі яких є
великі макрофагальні клітини зі світлою цитоплазмою. Визначити найбільш ймовірний тип гранульоми:
A. Туберкульозна
B. @Черевнотифозна
C. Сипнотифозна
D. Ревматична
E. Бруцельозна

21. Хворий, 27 років, який страждав на черевний тиф, помер від розлитого перитоніту. На розтині: в
нижніх часточках обох легень виявлені щільнуваті строкатого вигляду ділянки, що зливалися між собою, при
мікроскопічному дослідженні цих ділянок - міжальвеолярні перегородки повнокровні, альвеоли нерівномірно
заповнені серозним, серозно-гнійним ексудатом, в якому виявлялися колонії бактерій (за результатами
бактеріологічного дослідження - стафілокок). Визначити ускладнення черевного тифу:

A. Первинна бронхопневмонія
B. Реактивна плевропневмонія
C. @Вторинна бронхопневмонія
D. Абсцедуюча бронхопневмонія
E. Гангрена легень

22. Хворий помер від інтоксикації на 4-у добу після вживання сирих яєць. На розтині слизова оболонка
шлунка і тонкої кишки запалена, вкрита слизовим ексудатом; у легенях, головному мозку і печінці знайдені
абсцеси. Який діагноз найбільш вірогідний?

A. @Сальмонельоз (септична форма)


B. Дизентерія
C. Сальмонельоз (черевнотифозная форма)
D. Сальмонельоз (інтестинальна форма)
E. Черевний тиф

23. При розтині померлої дитини, яка страждала на діарею, виявлено ексикоз і поширений фібринозний
коліт. В мазку-відбитку слизової оболонки виявлені грам-негативні палички. Ваш попередній діагноз?

A. @Дизентерія
B. Холера
C. Стафілококова кишкова інфекція
D. Черевний тиф
E. Сальмонельоз

24. Який мікроорганізм спричиняє бактеріальну дизентерію?

A. @Шигела
B. Амеба
C. Стафілокок
D. Кишкова паличка
E. Мікобактерія

25. На розтині у померлого хворго в сигмоподібній та прямій кишці - множинні червоні виразки
неправильної форми, між якими слизова вкрита брудно-сірою плівкою. Вкажіть етіологію захворювання:

A. @Шигела
B. Амеба
C. Мікобактерія туберкульозу
D. Сальмонела
E. Стафілококовий ентеротоксин
26. На розтині тіла чоловіка, 46 років, на слизовій оболонці прямої та сигмоподібної кишок виявлено
множинні коричнево-зелені нашарування, крововиливи; у просвіті кишки слиз, невелика кількість крові;
гістологічно - фібринозний коліт. При бактеріологічному дослідженні вмісту кишки висіяна S.Sonne. Який
найбільш вірогідний діагноз?

A. @Дизентерія
B. Холера
C. Сальмонельоз
D. Ієрсиніоз
E. Хвороба Крона

27. Під час ректороманоскопії хворого зі скаргами на діарею виявлено, що слизова оболонка прямої і
сигмоподібної кишок різко гіперемована, набрякла, вкрита великою кількістю слизу, а у деяких ділянках вкрита
плівчастими накладаннями зеленуватого кольору. Про яке захворювання можна думати?

A. @Дизентерія
B. Сальмонельоз
C. Холера
D. Черевний тиф
E. Амебіаз

28. У хворого на 5-й день від початку діареї при колоноскопії в слизовій оболонці прямої кишки знайдені
сіро-зелені плівчасті нашарування, щільно фіксовані з підлеглою тканиною. Який діагноз найбільш вірогідний?

A. @Дизентерія
B. Черевний тиф
C. Неспецифічний виразковий коліт
D. Сальмонеллез
E. Хвороба Крона

29. Хворий помер на 3-й день після операції з приводу перфорації стінки товстої кишки з явищами
розлитого гнійного перитоніту. На розтині: слизова оболонка стінки товстої кишки потовщена, вкрита
фібриновою плівкою, поодинокі виразки проникають на різну глибину; гістологічно: некроз слизової, фібрин,
інфільтрація лейкоцитами з фокусами геморагій. Ускладнення якого захворювання стало причиною смерті
хворого?

A. @Дизентерія
B. Черевний тиф
C. Неспецифічний виразковий коліт
D. Хвороба Крона
E. Амебіаз

30. У померлого, 67 років, на розтині знайдені ознаки фібринозного запалення в товстому кишечнику.
Ваш діагноз:
A. @Дизентерія
B. Амебіаз
C. Черевний тиф
D. Холера
E. Балантидіаз

31. Пiд час аутопсiї тiла хворого, котрий помер вiд тяжкого ексикозу, на ґрунтi профузної дiареї, виявленi
такi змiни: слизова оболонка прямої та сигмоподiбної кишок на всій довжинi вкрита сiро-бiлими плiвчастими
нашаруваннями, що мiцно з’єднанi з пiдлеглими тканинами, мiж плiвками розташованi множиннi великi й
поверхневi виразки, вкритi згортками кровi. Мiкроскопiчно дiагностовано фiбринозно-виразковий колiт. Яке
захворювання проявляється такими змiнами?

A. Дизентерiя
B. Колi-iнфекцiя
C. Стафiлококова iнфекцiя
D. Iєрсiнiоз
E. Сальмонельози

32. На розтині чоловіка похилого віку, який впродовж останніх 2 тижнів страждав від гострого розладу
кишечника виявлені зміни у прямій та сигмоподібній кишках: на поверхні та між складками слизової оболонки
відмічається фібринозна коричнево-зелена плівка. Стінка кишки потовщена, порожнина різко звужена.
Мікроскопічно виявляється проникаючий на різну глибину некроз слизової оболонки, некротичні маси пронизані
нитками фібрину, фокуси геморагій та лейкоцитарної інфільтрації. Який з перелічених діагнозів найбільш
вірогідний?

A. @Фібринозний дизентерійний коліт


B. Катаральний дизентерійний коліт
C. Виразковий дизентерійний коліт
D. Черевний тиф, стадія некрозу
E. Черевнотифозна форма холери

33. У хворого на дизентерію при колоноскопії виявлено, що слизова оболонка товстої кишки
гіперемована, набрякла, її поверхня вкрита сіро-зеленими плівками. Назвіть морфологічну форму дизентерійного
коліту:

A. @Фібринозний
B. Катаральний
C. Виразковий
D. Гнійний
E. Некротичний

34. Слизова оболонка товстої кишки у померлого від дизентерії на розтині повнокровна, покрита плівкою
сірого кольору, що відривається із зусиллям. Який вид запалення розвинувся у кишці хворого?

A. @Дифтеритичне
B. Крупозне
C. Геморагічне
D. Серозне
E. Катаральне

35. У чоловіка, 50 років, який гостро захворів, діагностовано дизентерію. Смерть настала на 8-й день
захворювання. Під час розтину виявлено потовщену стінку сигмовидної та початкових відділів прямої кишки,
фібринозну плівку на поверхні слизової. Гістологічно: глибокий некроз слизової оболонки з просяканням
некротичних мас фібрином. Який вид коліту мав місце у хворого?

A. @Дифтеритичний
B. Катаральний
C. Виразковий
D. Постдизентерійний
E. Гангренозний

36. Хвора дитина, 5 років, яка страждала на гостру дизентерію, померла від гнійного перитоніту. На
розтині: солітарні фолікули товстої кишки збільшені, поверхня їх некротизована, вкрита гнійними масами,
слизова повнокровна, набухла, вкрита каламутним ексудатом. Визначити форму дизентерійного коліту:

A. Катаральний
B. Фібринозний
C. Фолікулярний
D. @Фолікулярно-виразковий
E. Гангренозний

37. Хворий, 58 років, помер від тяжкого перебігу дизентерії. На розтині причиною смерті встановлено
розлитий фібринозно-гнійний перитоніт. Сигмоподібна кишка здута, серозна оболонка тьмяна, її стінка тоненька,
дрябла, буро-чорного кольору з дрібними перфораціями. При бактеріологічному дослідженні виявлена анаеробна
мікрофлора. Назвати морфологічну форму дизентерійного коліту:

A. Катаральний
B. Фібринозний
C. Виразковий
D. @Гангренозний
E. Атрофічний

38. У хворого, 40 років, хронічна дизентерія супроводжувалася нирковою недостатністю з порушенням


мінерального обміну (гіперкальціемія). Помер від виснаження. При мікроскопічному дослідженні внутрішніх
органів виявлені відкладання вапна в легенях, слизовій оболонці шлунка, в нирках, міокарді та стінках артерій.
Вказані зміни є проявом:
A. Метаболічного звапніння
B. Фібриноїдного набрякання
C. Дистрофічного звапніння
D. Мукоїдного набрякання
E. @Метастатичного звапніння

39. У породіллі з пієлонефритом, 25 років, післяпологовий період ускладнився стафілококковим


ендометритом. Після суправагінальної ампутації матки з придатками розвинулися гемолітична жовтяниця,
синдром дисемінованого внутрішньосудинного згортання крові, гектична лихоманка, гепатоспленомегалія. На
третю добу хвора померла. При аутопсії виявлені: діапедезні крововиливи в слизових та серозних оболонках,
крововиливи в м’яких тканинах навколо ін’єкцій, в серці фібриноїдний некроз ендокарда з пристінковими
тромбами, в селезінці та лімфатичних вузлах - проліферація ретикулярних клітин, лімфоцитів з накопиченням
зрілих та незрілих кровотворних клітин. Визначити клініко-морфологічну форму сепсису:

A. @Септицемія
B. Септикопіємія
C. Хроніосепсис
D. Септичний ендокардит
E. Криптогенний сепсис

40. Хворий, 30 років, під час швидкого гоління обличчя пошкодив шкіру гострим лезом. Через 6 годин
різко підвищилася температура до 39°С, погіршився стан, свідомість затьмарилася. При госпіталізації виявлені:
жовтяниця, петехіальні висипання на шкірі та слизових, збільшення селезінки та підщелепних лімфатичних
вузлів. На фоні прогресуючої інтоксикації він помер. При гістологічному дослідженні внутрішніх органів
встановлені набряк строми з дифузною інфільтрацією її нейтрофілами, лімфоцитами та гістіоцитами, васкуліти з
фібриноїдним некрозом стінок судин, а також гіперплазія лімфоїдної та кровотворної тканини. Визначити
клініко-морфологічну форму сепсису:

A. @Септицемія
B. Септикопіємія
C. Первинний септичний ендокардит
D. Хроніосепсис
E. Вторинний септичний ендокардит

41. У померлого, 40 років, за життя довгий час була висока температура, затемнення свідомості,
бактеріємія. При розтині: склера і шкіра жовтого кольору, крововиливи у серозних і слизових оболонках,
збільшені лімфатині вузли та селезінка. Мікроскопічно: в селезінці та лімфовузлах - проліферація ретикулярних
клітин; в серці, печінці, нирках – проміжне запалення; в стінках судин - васкуліти, фібриноїдний набряк. Який з
перелічених діагнозів найбільш вірогідний?

A. @Септицемія
B. Септикопіємія
C. Гострий лімфолейкоз
D. Гострий недиференційований лейкоз
E. Хроніосепсис

42. Після позалікарняного аборту у жінки пргресував гнійний ендоміометрит зі смертельним наслідком.
При розтині померлої виявлені чисельні абсцеси легень, субкапсулярні гнійнички в нирках, гіперплазія селезінки.
Яка форма сепсису виникла у жінки?

A. @Септикопіємія
B. Септицемія
C. Хроніосепсис
D. Легеневий сепсис
E. Уросепсис

43. Хвора, 20 років, померла від інтоксикації через 8 днів після штучного позалікарняного аборту при
терміні вагітності 14 – 15 тижнів. На розтині тіла померлої жовтувате забарвлення склери очей, шкіри, гнійно-
некротичний ендометрит, багаточисленні гнійники в легенях, явна гіперплазія селезінки з великою кількістю
нейтрофілів в її синусах. Яке ускладненя після аборту розвинулось у хворої?

A. @Септикопіємія
B. Септицемія
C. Геморрагічний шок
D. Хроніосепсис
E. Вірусний гепатит А

44. Жінка, 32 років, померла в післяпологовому періоді. На розтині були знайдені ознаки гнійного
ендометриту, гнійний тромбофлебіт вен матки, множинні абсцеси легень, абсцеси нирок і селезінки,
апостематозний міокардит та гнійний менінгіт. Вкажіть ускладнення післяпологового періоду, яке призвело до
смерті.

A. @Септикопіємія
B. Септицемія
C. Хроніосепсис
D. Гострий септичний ендокардит
E. Затяжний септичний ендокардит

45. На розтині хворого, 48 років, який помер від уросепсису, виявлені: ендокардит, абсцеси печінки,
абсцедуюча двобічна пневмонія, гострий пієлонефрит, гнійний пери- та паранефрит з флегмоною м’яких тканин
поперекової ділянки спини. Селезінка збільшена, дає великий зішкребок пульпи, при гістологічному дослідженні
в пульпі багато лейкоцитів. Визначити клініко-морфологічну форму сепсису:

A. Септицемія
B. @Септикопіємія
C. Хроніосепсис
D. Септичний ендокардит
E. Криптогенний сепсис

46. При гістологічному дослідженні матки хворої, 30 років, яка померла від септикопіємії, в міометрії
виявлені: гнійний тромбофлебіт, в стромі набряк та дифузна інфільтрація нейтрофільними лейкоцитами,
лімфоцитами та гістіоцитами. Визначити складовий компонент тромбу, характерний для сепсису:
A. Гемолізовані еритроцити
B. @Колонії бактерій
C. Грануляційна тканина
D. Нейтрофільні лейкоцити
E. Фібрин

47. Молодий чоловік помер від наростаючої серцевої недостатності. При патоморфологічному
дослідженні виявлені: селезінка масою 500 гр., імунокомплексний гломерулонефрит, виразки півмісяцевих
клапанів аорти, які прикриті поліпоподібними тромбами з колоніями бактерій. Вкажіть найбільш вірогідний
діагноз.

A. @Септичний бактеріальний ендокардит


B. Септицемія
C. Ревматичний тромбоендокардит
D. Гострий ревматичний вальвуліт
E. Септикопіємія

48. На розтині тіла померлої молодої людини, яку госпіталізовано в стаціонар у стані мозкової коми і
через деякий час померла, виявлено: поширений тромбоемболічний інфаркт лівої півкулі мозку, а також велика
септична селезінка, імунокомплексний гломерулонефрит, виразки в стулках аортального клапана, прикриті
поліпоподібними тромбами з колоніями стафілококів. Яке захворювання призвело до церебральної
тромбоемболії?

A. @Септичний бактеріальний ендокардит


B. Септицемія
C. Гострий ревматичний вальвуліт
D. Септикопіємія
E. Ревматичний тромбоендокардит

49. Хвора, 27 років, яка вживала ін’єкції наркотиків впродовж шести років, померла при явищах
прогресуючої серцево-судинної недостатності. На розтині серце збільшено в об’ємі, 600 г, гіпертрофія стінок
лівого та правого шлуночків (відповідно 3 см та 1 см), камери розширені, міокард дряблий. Стулки аортального
клапана зруйновані на 2/3, вздовж країв кукси бородавчасті тромботичні нашарування. При мікроскопічному
дослідженні в тромбах виявлені колонії бактерій. Визначити причину серцево-судинної недостатності:

A. Ревматична вада
B. @Септичний ендокардит
C. Кардіоміопатія
D. Гіпертонічна хвороба
E. Ішемічна хвороба серця

50. У хворого, який довгий час вживав наркотики - лихоманка, явища інтоксикації, піурія, на
ехокардіографії виявлені масивні нашарування на клапанах серця. З крові висіяний стафілокок. Хворий помер від
тромбоемболії легеневої артерії. Вкажіть на зміни серця, виявлені у хворого під час розтину.
A. @Поліпозно-виразковий ендокардит
B. Ендокардит Лібмана-Сакса
C. Дифузний вальвуліт
D. Гострий бородавчатий ендокардит
E. Зворотно-бородавчастий ендокардит

51. У чоловіка, 40 років, померлого від одонтогенного сепсису, на розтині виявлено різке потовщення
півмісяцевих клапанів аорти, вони були білясті, непрозорі, малорухливі, на зовнішній поверхні розташовувалися
тромботичні накладення розмірами 1х1.5 см. Про яку форму ендокардиту йдеться?

A. Дифузний ендокардит
B. @Поліпозно-виразковий ендокардит
C. Гострий бородавчастий ендокардит
D. Фібропластичний ендокардит
E. Поворотно-бородавчастий ендокардит

52. Хвора, 22 років, страждала на септичний ендокардит, померла від ішемічного некрозу головного
мозку. На розтині: множинні інфаркти та рубці в нирках, селезінці; гангрена фрагменту тонкого кишечника з
перифокальним серозно-фібринозним перитонітом, на зруйнованих і склерозованих стулках аортального клапана
- різні за розмірами тромботичні нашарування. Вказати головне ускладнення септичного ендокардиту:

A. Геморагічний синдром
B. Ішемічний синдром
C. Тромбогеморагічний синдром
D. @Тромбоемболічний синдром
E. Недостатність аортального клапана

53. Під час розтину чоловіка, у якого після поранення кінцівки виникло тривале нагноєння рани, і який
помер при явищах інтоксикації, виявлено: загальне виснаження, зневоднення, бура атрофія печінки, міокарда,
селезінки, поперечносмугастої мускулатури та амілоїдоз нирок. Який з перелічених діагнозів найбільш
вірогідний?

A. Хроніосепсис
B. Септикопіємія
C. Септицемія
D. Хвороба Чорногубова
E. Бруцельоз

54. У хворого-інваліда, 65 років, з посттравматичним остеомієлітом лівої стегнової кістки перебіг


захворювання супроводжувався септично-резорбтивною лихоманкою. Помер при явищах прогресуючої
поліорганної недостатності. На розтині: виражена кахексія, в серці, печінці та скелетних м’язах - бура атрофія,
зневоднення тканин, зменшення селезінки, в нирках - амілоїдоз. Для якої форми сепсису найбільш характерні
вказані зміни?

A. Септицемія
B. Септикопіємія
C. Первинний септичний ендокардит
D. Вторинний септичний ендокардит
E. @Хроніосепсис

55. На розтині померлого, 18 років, селезінка вагою 580 грамів, темно-червоного кольору на розрізі, дає
рясні вишкребки пульпи. Гістологічно: виражена проліферація ретикулярних клітин, наявність великої кількості
зрілих нейрофілів у сінусоїдних капілярах. Назвіть цю селезінку.

A. @“Септична”
B. “Сагова”
C. “Порфірова”
D. “Ціанотична”
E. “Лейкозна”

56. Хворий, 38 років, наркоман, помер від гнійного менінгіту. На розтині: селезінка збільшена, дає значні
вишкребки пульпи (септична селезінка). У внутрішніх органах - метастатичні гнійники. Найбільш характерним в
селезінці є:

A. Проліферація лімфоцитів
B. Проліферація ретикулоцитів
C. Інфільтрація мієлобластами
D. @Інфільтрація лейкоцитами
E. Гемоліз еритроцитів

57. Хвора, 18 років, померла від сепсису на 710 добу після патологічних пологів. На розтині встановлені
ознаки геморагічного синдрому: петехіальні крововиливи на шкірі, у внутрішніх органах, в серозних та слизових
оболонках. При гістологічному дослідженні строма внутрішніх органів повнокровна, з набряком, інфільтрована
нейтрофілами, лімфоцитами, гістіоцитами, в кровоносних судинах - білкова дистрофія, фібриноїдний некроз,
інфільтрація стінок поліморфноядерними лейкоцитами та макрофагами. Визначити причину геморагічного
синдрому при септицемії:

A. Осередковий некротичний васкуліт


B. @Дифузний некротичний васкуліт
C. Дифузний продуктивний васкуліт
D. Гранульоматозний васкуліт
E. Деструктивно-продуктивний васкуліт

58. На розтині померлого від шигельозу в печінці виявлені багаточисленні дрібні осередки діаметром до
1,5 см. При мікроскопічному дослідженні в їх центрі - гнійно-некротичні маси, обмежені нейтрофільними
лейкоцитами, в стінках венул та навколо них - виражена лейкоцитарна інфільтрація, в просвітах - тромби.
Визначити ускладнення, що розвинулося в печінці:

A. Кісти з нагноєнням
B. Гострий рективний гепатит з некрозами
C. Ехінококоз
D. @Пілефлебітичні абсцеси
E. Гострий фульмінантний гепатит

59. У хворого на підгострий септичний ендокардит при огляді лікар відзначив загальну слабкість і
іктеричність шкіри, склер і видимих слизових оболонок. У крові виявлено збільшену кількість непрямого
білірубіну. Що зумовлює жовтяничність шкіри і слизових?

A. @Надпечінкова жовтяниця
B. Жирова дистрофія
C. Гемосидероз
D. Печінкова жовтяниця
E. Підпечінкова жовтяниця

60. На розтині померлого від сепсису в стегновій кістці нижньої кінцівки виявлено флегмонозне
запалення, що охоплює кістковий мозок, гаверсові канали та періост. Під періостом - множинні абсцеси, в
навколишніх м’яких тканинах стегна - також флегмонозне запалення. Який патологічний процес має місце?

A. @Гострий гематогенний остеомієліт


B. Остеопороз
C. Хронічний гематогенний остеомієліт
D. Остеопетроз
E. -

61. У хворого, 40 років, при прогресуванні стафілококового гнійного періодонтиту виникло гнійне
запалення кістково-мозкових просторів альвеолярного відростка, а потім і тіла нижньої щелепи. Мікроскопічно
кісткові балки стоншені, осередки некрозу, кісткові секвестри, оточені сполучнотканинною капсулою. Який
найбільш імовірний діагноз?

A. @Хронічний остеомієліт
B. Гострий остеомієліт
C. Пародонтома
D. Хронічний фіброзний періостит
E. Гнійний періостит

ВІРУСНІ ІНФЕКЦІЇ.
ДИТЯЧІ ІНФЕКЦІЙНІ ЗАХВОРЮВАННЯ
ВАРІАНТ №

1. У дитини при огляді зіва виявлені жовто-білі плівки фібрину, які вкривають мигдалики, піднебінні
дужки і важко знімаються шпателем. Встановлено діагноз інфекційного захворювання. Назвіть його збудника.
A. Вібріон холери
B. Мікобактерія туберкульозу
C. Мікобактерія лепри
D. Паличка сибірки
E. @Коринебактерія дифтерії

2. Під час огляду ротової порожнини дівчинки, 9 років, було виявлено, що поверхня стінок зіва та
мигдаликів вкрита біло-жовтими плівками, регіонарні лімфатичні вузли збільшені. Гістологічно у видалених
мигдаликах встановлено, що поверхневий епітелій частково некротизований та вкритий плівчастими
нашаруваннями фібрину з домішкою нейтрофілів. У післяопераційному періоді з’явилися клінічні ознаки
міокардиту та ниркової недостатності. Діагностуйте захворювання.
A. @Дифтерія
B. Скарлатина
C. Важка форма грипу
D. Первинна фолікулярна ангіна
E. Первинна апостематозна ангіна

3. На розтині тіла дівчинки, 16 років, було виявлено, що поверхня зіва та мигдаликів вкрита білувато-
жовтими плівками, які міцно пов’язані з підлеглою сполучною тканиною, серцеві камери дилатовані, міокард
тьмяний та дряблий, помірно строкатий. Мікроскопічно в міокарді виявлені множинні осередки некрозу
кардіоміоцитів та ознаки їх жирової дистрофії, незначні осередкові інфільтрати інтерстицію. Діагностуйте
захворювання.
A. Грип
B. Кір
C. Скарлатина
D. @Дифтерія
E. Коклюш
4. У юнака, 18 років, який захворів гостро і помер від інфекційно-токсичного шоку, на аутопсії виявлено
збільшені мигдалики, вкриті сіро-білими плівками, що поширюються на піднебінні дужки, набряк м’яких тканин
шиї. При гістологічному дослідженні: некроз епітелію мигдаликів та дужок, підлеглі тканини просякнуті
фібринозним ексудатом, який утворює масивні нашарування на поверхні. Діагностуйте захворювання
A. @Дифтерія
B. Скарлатина
C. Аденовірусна інфекція
D. Інфекційний мононуклеоз
E. Стафілококова інфекція

5. Дівчинка, 4 років, на 3 добу від початку захворювання на дифтерію померла від справжнього крупу. На
аутопсії слизова оболонка гортані, трахеї та бронхів потовщена, набрякла, тьмяна, вкрита сіруватою плівкою, що
легко відокремлюється. Визначити вид ексудативного запалення гортані
A. @Фібринозне
B. Серозне
C. Гнійне
D. Змішане
E. Катаральне

6. У дитини з помірно вираженою загальною інтоксикацією виявлено дифтерію з ураженням крупних


бронхів. Який вид запалення характерний для цього виду дифтерії?
A. Продуктивнее
B. Дифтеритичне
C. Альтеративне
D. Геморагічне
E. @Крупозне
7. Дівчинка, 5 років, захворіла на дифтерію. На третю добу померла від асфіксії внаслідок справжнього
крупу. На розтині встановлено, що слизова оболонка гортані, трахеї та бронхів потовщена, набрякла, тьмяна,
покрита сіруватими плівками, які легко відокремлюються. Про який патологічний процес свідчать морфологічні
зміни у гортані:
A. @Крупозне запалення
B. Серозне запалення
C. Змішане запалення
D. Дифтеритичне запалення
E. Катаральне запалення

8. Основна клінічна тріада синдрому крупу:


A. Дрібно-плямистий висип, нежить, біль у горлі
B. Велико-плямистий висип, кашель, кон’юктивіт
C. @Грубий, “гавкаючий” кашель, осиплість голосу, задишка з утрудненим вдихом
D. Сухі, свистячі хрипи, задишка з утрудненим видихом, кашель
E. -

9. Оглядаючи дитину, 6 років, лікар помітив на глоткових мигдаликах сірувату плівку, при спробі
видалення якої виникла помірна кровотеча. Бактеріоскопія мазків з мигдаликів показала наявність
грампозитивних бактерій була- воподібної форми. Які симптоми можуть виникнути у дитини у найближчі дні,
якщо не буде проведене специфічне лікування?
A. @Токсичні ураження серцевого м’яза, печінки, нирок
B. Набряк легень
C. Дуже сильний нападоподібний кашель
D. Папульозні висипи на шкірі
E. Хвилеподібна лихоманка

10. При мікроскопічному дослідженні міокарда дівчинки, яка померла від дифтерії внаслідок серцевої
недостатності виявлені: жирова дистрофія та множинні осередки некрозу кардіоміоцитів, незначні осередкові
клітинні інфільтрати інтерстицію. Про який міокардит йде мова?
A. Осередковий ексудативний
B. Дифузний ексудативний
C. @Альтеративний
D. Інтерстиціальний
E. Гранульоматозний

11. На розтині тіла хлопчика 8 років, що хворів на дифтерію зіва та мигдаликів і помер на другий тиждень
від початку захворювання, виявлено зміни в міокарді у вигляді дрібноосередкових некрозів міокардиоцитів,
набряку строми з незначною лімфоцитарною інфільтрацією. Діагностуйте вид міокардиту.
A. @Альтеративний
B. Септичний
C. Гранульоматозний
D. Інтерстиціальний
E. Осередковий проміжний ексудативний

12. Під час огляду ротової порожнини хлопчика, 14 років, виявлено картину “палаючого зіва”, мигдалики
збільшені, яскраво-червоні. Мікроскопічно у мигдаликах - гіперемія, дрібні осередки некрозу з незначним
лейкоцитарним інфільтратом та ланцюжками стрептококів. Підщелепові лімфатичні вузли збільшені. Через 3
тижні після видужання з’явилися ознаки гострого гломерулонефриту. Діагностуйте захворювання.
A. Дифтерія
B. @Скарлатина
C. Важка форма грипу
D. Первинна фолікулярна ангіна
E. Первинна апостематозна ангіна
13. У дитини, 10 років, з’явилися різкий біль при ковтанні, набряк шиї, температура тіла підвищилась до
39ºС, з’явились яскраво-червоні дрібнокрапчасті висипання по всьому тілі. У зіві і мигдаликах – різке
повнокров’я (“палаючий зів”), “малиновий язик”. На поверхні мигдаликів – поодинокі сіруваті осередки некрозів.
Про яке захворювання може йти мова?
A. @Скарлатина
B. Менінгококовий назофарингіт
C. Дифтерія
D. Грип
E. Кір

14. У дитини, 7 років, з`явився біль у горлі, підвищилась температура тіла. На 2-ий день від початку
захворювання виявлені висипання червоного кольору у вигляді дрібних, густо розташованих плям, величиною з
макове зерно. Вони покривають усе тіло, за виключенням носо-губного трикутника. При огляді порожнини рота –
в зіві яскраве почервоніння, мигдалики збільшені, язик малиново-червоний. Ваш діагноз?
A. @Скарлатина
B. Аденовірусна інфекція
C. Дифтерія зіва
D. Кір
E. Стрептококова ангіна

15. Дитині, 8 років, яка надійшла до інфекційного відділення з високою температурою (до 38°C),
дрібнокрапчастим яскраво-червоним висипом, було встановлено діагноз «Скарлатина». Об’єктивно: слизова
оболонка зіва яскраво гіперемована, набрякла, мигдалики різко збільшені, з тьмяними осередками жовтувато-
сірого кольору і ділянками чорного кольору. Яке запалення лежить в основі змін у зіві?
A. @Гнійно-некротичне
B. Фібринозне
C. Геморагічне
D. Серозне
E. Катаральне
16. На розтинi тіла дитини виявлено: некротична ангiна, флегмона шиї, гнiйний отит, гнiйний менiнгiт. Цi
змiни найбiльш характернi для:
A. @Септичної скарлатини
B. Менiнгококової iнфекцiї
C. Токсичної скарлатини
D. Дифтерiї зiва
E. Отогенного сепсису

17. У людини, яка померла від інфекційного менінгіту, на розтині виявлено некрози та крововиливи в
надниркові залози, що при житті супроводжувалось гострою наднирковою недостатністю (синдром Уотерхауса-
Фридериксена). Назвіть збудника цього захворювання.
A. Пневмокок
B. Стафілокок
C. Стрептокок
D. @Менінгокок
E. Гонококк
18. Дитина, 8 років, захворіла гостро. Клінічно спостерігалася нудота, блювота, головний біль. Через дві
доби від початку захворювання настала смерть. На аутопсії виявлено, що м’які мозкові оболонки різко
повнокровні, просякнуті густим каламутним жовтувато-зеленуватим ексудатом на базальній поверхні головного
мозку. Тканина мозку набрякла. Мигдалики мозочка збільшені в об’ємі. виражена странгуляційна борозна.
Поставте діагноз.
A. @Менінгококова інфекція
B. Скарлатина
C. Коклюш
D. Дифтерія
E. Кір

19. Дитина, 3 років, померла при явищах глибокої церебральної коми. На розтині виявлено гнійні
нашарування на поверхні м’якої мозкової оболонки в передніх відділах головного мозку, набряк мозку.
Мікроскопічно – повнокров’я і нейтрофільна інфільтрація м’яких мозкових оболонок. Вкажіть ймовірне
захворювання?
A. @Менінгококовий менінгіт
B. Менінгіт при сибірці
C. Грипозний менінгіт
D. Туберкульозний менінгіт
E. Паротитний менінгіт

20. На розтині тіла хворого виявлено: м’яка мозкова оболонка верхніх відділів півкуль головного мозку
різко повнокровна, жовто-зеленого кольору, просочена гнійним та фібринозним ексудатом, що нагадує чіпець.
Для якого захворювання характерна така картина?
A. @Менінгококового менінгіту
B. Туберкульозного менінгіту
C. Грипозного менінгіту
D. Менінгіту при сибірці
E. Менінгіту при висипному тифі

21. Дитина, 4 років, захворіла гостро. Клінічні прояви: нудота, блювання, головний біль. Через 2 доби від
початку захворювання настала смерть. На розтині виявлено: набряк головного мозку, мозкові оболонки різко
повнокровні та просочені густим каламутним жовто-зеленуватим гнійним ексудатом. Виберіть правильний
діагноз:
A. @Менінгококовий лептоменінгіт
B. Скарлатина
C. Дифтерія
D. Кір
E. Менінгококовий назофарингіт

22. На розтині померлого від набряку головного мозку з дислокацією стовбура, виявлені зміни м'якої
мозкової оболонки: тьмяна, сірувато-зеленого кольору, потовщена. Який патологічний процес мав місце в м'якій
мозковій оболонці?
A. @Дифузний гнійний лептоменінгіт
B. Менінгококовий лептоменінгіт
C. Туберкульозний лептоменінгіт
D. Злоякісна арахноїдендотеліома
E. Гідроцефалія

23. Під час розтину трупа дівчинки, 12 років виявлено: множинні крововиливи у шкірі (переважно
сідниць, нижніх кінцівок), серозних та слизових оболонок, у головному мозку. У надниркових залозах -
осередковий некроз та масивні крововиливи, у нирках - некротичний нефроз, гнійний артрит, іридоцикліт,
васкуліт. Який найбільш вірогідний діагноз?
A. @Менінгококцемія
B. Висипний тиф
C. Вузликовий периартеріїт
D. Системний червоний вовчак
E. Променева хвороба
24. Дитина, 12 років, занедужала гостро з підвищенням температури до 38°C, з головним болем, дертям у
горлі, потім з'явився грубий кашель, на 3 день на шкірі обличчя, шиї і верхньої частини тулуба – рясна плямиста
папульозна висипка, яка не свербить. Слизова оболонка рота яскраво гіперемійована, з наявністю на щоках біля
нижніх корінних зубів дрібної білої енантеми. Який найбільш імовірний діагноз?
A. @Кір
B. Скарлатина
C. Краснуха
D. Алергічний дерматит
E. Дизентерія

25. У дитини, 2 років, катаральний кон’юктивіт, фарингіт, ларинготрахеобронхіт. На слизовій оболонці


щік, навпроти нижніх малих кореневих зубів, білі плями, а пізніше – великоплямистий і папульозний висип на
обличчі, тулубі, кінцівках. Вкажіть захворювання.
A. Висипний тиф
B. Скарлатина
C. Менінгококова інфекція
D. @Кір
E. Грип

26. У дитини, після перенесеного кору, при огляді виявлено у м’яких тканинах щік і промежини нечітко
відмежовані, набряклі, червоно-чорного кольору ділянки, які злегка флуктують. Яке ускладнення розвинулося у
дитини?
A. Пролежень
B. Суха гангрена
C. Газова гангрена
D. @Волога гангрена (нома)
E. Трофічна виразка

27. Хворого, 25 років, госпіталізовано до інфекційного відділення з діагнозом “Кір. Вторинний


міокардит” зі скаргами на гострий біль в області серця, що посилювався при кашлі, температуру 39,5 оС, задуху.
При аускультації вислуховувався характерний шум тертя перикарда та розширення меж серця при перкусії. На 3
добу при явищах прогресуючої серцево-легеневої недостатності хворий помер. На розтині під вісцеральним
листком перикарда - дрібні крововиливи, поверхня серозної оболонки тьмяна, дифузно вкрита нашаруваннями
сіро-білого кольору у вигляді сіточки, волосків та плівок, у просвіті перикарда 200 мл каламутної рідини. Який
вид запалення характерний для даної форми перикардиту?
A. Дифтеритичне фібринозне
B. @Крупозне фібринозне
C. Серозне
D. Гнійне
E. Катаральне
28. У дівчинки, 8 років, раптово підвищилась температура, з’явились явища катару дихальних шляхів. На
5-й день хвороби виник параліч м’язів нижніх кінцівок і приєднались дихальні розлади. В передніх рогах
спинного мозку виявлено проліферацію глії навколо загиблих нейронів. Яке захворювання розвинулось у
дівчинки?
A. @Поліомієліт
B. Кір
C. Дифтерія
D. Менінгококцемія
E. Скарлатина

29. У дитини, 3 років, тривале пiдвищення температури, збiльшенi лiмфовузли, у кровi - значне
пiдвищення кiлькостi лiмфоцитiв. Методом IФА виявлено антиген вiруса Епштейна-Бара. Який дiагноз можна
поставити на основi вказаного?
A. @Iнфекцiйний мононуклеоз
B. Лiмфома Беркіта
C. Герпетична аденопатiя
D. Генералiзована iнфекцiя, викликана Herpes-Zoster
E. Цитомегаловiрусна інфекція

30. На розтині у померлого хлопчика, віком 8 років, в обох легенях виявлено перибронхіальні ділянки
ущільнення неправильної форми різних розмірів, червоного кольору, які випинають або інколи западають
відносно поверхні розрізу легеневої тканини, з ділянок розрізу, що випинають, стікає червона рідина. Найбільш
вірогідна етіологія хвороби:
A. @Вірус грипа
B. Паличка Фридлендера
C. Стрептокок пневмонії
D. Повна обтурація окремих бронхів
E. Часткова обтурація окремих бронхів

31. Чоловiк, 42 рокiв, помер при явищах вираженої iнтоксикацiї i дихальної недостатностi. На розтинi:
тканина легень у всiх вiддiлах строката, з множинними дрiбноосередковими крововиливами та ділянками
емфiземи. Гiстологiчно у легенях: геморагiчна бронхопневмонiя з абсцедуванням, у цитоплазмi клiтин епiтелiю
бронхiв еозинофiльнi i базофiльнi включення. Дiагностуйте виявлене на секцiї захворювання:
A. @Грип
B. Крупозна пневмонiя
C. Плевропневмонiя
D. Часткова пневмонiя
E. Стафiлококова бронхопневмонiя

32. У хворого, 60 років, який скаржився на субфебрильну температуру, головний біль та кашель з
мокротинням впродовж 10 днів, задуху. Через 6 діб настала смерть. На розтині: слизові, серозні та мозкові
оболонки, просочені кров’ю, у легенях ознаки інтерстиціальної пневмонії, що викликана вірусом і
бронхопневмонії, що пов’язана з вторинною інфекцією; геморагії та ателектази. Ваш діагноз:
A. @Грип
B. Кір
C. Ентеровірусна інфекція
D. Натуральна віспа
E. ВІЛ-інфекція
33. У чоловіка, 30 років, який хворів на гостре респіраторне захворювання та помер при явищах гострої
легенево-серцевої недостатності, під час розтину виявлено: фібринозно-геморагічне запалення в слизовій
оболонці гортані та трахеї, деструктивний панбронхіт, збільшені легені, які мають пістрявий вигляд за рахунок
абсцесів, крововиливів, некрозу. Який з перелічених діагнозів найбільш вірогідний?
A. @Грип
B. Парагрип
C. Респіратрно-синцитіальна інфекція
D. Кір
E. Аденовірусна інфекція

34. Під час патологоанатомічного дослідження тіла хлопчика, 5 років, який хворів на ГРВІ, було
виявлено: серозно-геморагічний риноларинготрахеобронхіт з ділянками некрозу слизової оболонки трахеї і
бронхів; у легенях – множинні осередки серозно-геморагічної пневмонії з поодинокими гіаліновими мембранами
в просвітах альвеолярних ходів. Діагностуйте форму грипу.
A. @Важка
B. Легка
C. Середньої важкості
D. Грипозний токсикоз
E. Велика строката легеня

35. Хворий, 67 років, мав важку форму грипу з летальним наслідком. На секції зміни в легенях були
подібні змінам “великих строкатих легень”. При мікроскопічному дослідженні виявлено: різке повнокров’я
судин, крововиливи, набряк легеневої тканини, в просвіті бронхів і альвеол - ексудат, який містить переважно
еритроцити. Про який характер запалення легень свідчать ці морфологічні ознаки?
A. @Геморагічна бронхопневмонія
B. Катаральна бронхопневмонія
C. Гнійна бронхопневмонія
D. Десквамативна бронхопневмонія
E. Фібринозна плевропневмонія

36. У хворого з важким перебігом респіраторної вірусної інфекції з'явилися клінічні ознаки прогресуючої
серцевої недостатності, яка призвела до смерті на 2-му тижні захворювання. На розтині - серце зі значним
розширенням порожнин, дрябле. Гістологічно в міокарді виявляється повнокров'я мікросудин і дифузна
інфільтрація строми лімфоцитами і гістіоцитами. Вкажіть найбільш вірогідний діагноз.
A. @Міокардит
B. Стенокардія
C. Гостра коронарна недостатність
D. Інфаркт міокарда
E. Кардіоміопатія

37. На розтині в головному мозку виявлено набряк, повнокров’я, дрібні крововиливи в довгастому мозку.
Мікроскопічно: хроматоліз, гідропія і некроз нервових клітин; у цитоплазмі нервових клітин гіпокампа -
еозинофільні утворення (тільця Бебеша-Негрі). Який діагноз відповідає описаним морфологічним проявам?
A. Энцефаліт
B. Менінгококовий менінгіт
C. @Сказ
D. Енцефаломієліт
E. Бруцельоз

38. Для патоморфологічної картини енцефаліта, спричиненого вірусом простого герпесу, характерно:
A. Набряк нервових волокон
B. Гнійне запалення мозкових оболонок
C. @Ділянки некрозу речовини головного мозку
D. Периваскулярна інфільтрація
E. -

39. У хворого з наркоманією за результатами клініко-лабораторних досліджень поставлено діагноз


«СНІД». Назвіть збудника захворювання.
A. Реовіруси
B. Аденовіруси
C. @Ретровіруси
D. Риновіруси
E. Ентеровіруси
40. Який клон імунокомпетентних клітин уражується при інфікуванні вірусом імунодефіциту людини?
A. @Т-лімфоцити хелпери
B. В-лімфоцити
C. Т-лімфоцити кіллери
D. Макрофаги
E. Клітини пам’яті

41. Під час гістологічного дослідження головного мозку хворого, який помер від ВІЛ-інфекції, в
субкортикальній білій речовині, середньому мозку і стовбурі мозку були виявлені дрібні, периваскулярні
некрози, мікрогліальні вузлики з багатоядерними гігантськими клітинами, осередковий гліофіброз. Назвати
найбільш вірогідний характер ураження ЦНС.
A. Вакуолярна мієлопатія
B. Метаболічна енцефалопатія
C. Цитомегаловірусний енцефаліт
D. Первинна лімфома ЦНС
E. @Підгострий менінгоенцефаліт

42. У хворого з наркоманією довгий час була лихоманка, генералізована лімфаденопатія, зниження маси
тіла. Смерть настала від пневмоцистної пневмонії. Попередній діагноз:
A. Сепсис
B. Лімфома
C. Туберкульоз
D. @СНІД
E. Лімфолейкоз
43. У померлого, 44 років, громадянина України, за результатами розтину встановлено поєднання
пневмоцистної пневмонії, саркоми Капоші та В-клітинної лімфоми. В анамнезі невпорядковані статеві контакти.
Що є найбільш вірогідним?
A. @Інфекція вірусом імунодефіциту людини, стадія СНІДу
B. Інфекція вірусом імунодефіциту людини, стадія преСНІДу
C. Вторинний імунодефіцит внаслідок первинної В-клітинної лімфоми
D. Вторинний імунодефіцит внаслідок саркоми Капоші
E. -

44. У хворого з клініко-лабораторними проявами набутого імунодефіциту, зумовленого ВІЛ-інфекцією, на


шкірі дистальних відділів нижніх кінцівок розташовуються множинні темно-червоного кольору плями, бляшки.
Гістологічно в біоптаті шкіри з цих ділянок виявляється пухлинна тканина, яка складається з множинних
новоутворених хаотично розташованих тонкостінних судин і пучків веретеноподібних клітин. Вкажіть найбільш
вірогідний діагноз.
A. Дерматомікоз
B. Лімфома шкіри
C. @Саркома Капоши
D. Базальноклітинний рак
E. Запальний дерматит

45. Жiнка, 31 року, хворiє на ВIЛ-iнфекцiю на стадiї СНIД. На шкiрi нижнiх кiнцiвок, слизової оболонки
пiднебiння з’явились рудувато-червонi плями, яскраво-червонi вузлики рiзних розмiрiв. Один з вузликiв взято на
гiстологiчне дослiдження. Виявлено: багато хаотично розташованих тонкостiнних судин, вистелених ендотелiєм,
пучки веретеноподiбних клiтин з наявнiстю гемосидерину. Яка пухлина розвинулась у хворої?
A. @Саркома Капошi
B. Гемангiома
C. Лiмфома Беркiта
D. Лiмфангiома
E. Фiбросаркома

46. У хворого з вираженим імунодефіцитом, наявністю в крові лімфопенії зі зміною співвідношення Т-


хелперів до Т-супресорів, відмічається ураження шкіри нижніх кінцівок у вигляді множинних пухлиноподібних
вузликів синюшно-червоного кольору, які зливаються. При дослідженні біоптату шкіри виявлено новоутворення
кровоносних судин і розширення капілярів, які утворюють порожнини різної форми і розмірів. Діагностуйте
найбільш вірогідний характер шкірної патології.
A. Лімфома шкіри
B. Базаліома
C. Дерматомікоз
D. @Саркома Капоши
E. Запальний дерматит
47. Під час розтину тіла новонародженої дитини були виявлені ознаки гідроцефалії, привушна слина
залоза збільшена, ущільнена. Гістологічно в тканині залози - лімфо-макрофагальний інфльтрат строми, дрібні
ділянки склерозу, частина епітеліальних клітин проток та ацинусів збільшені за розмірами, містять
внутрішньоядерні еозинофільні включення великих розмірів. Діагностуйте вірусне інфекційне захворювання у
дитини.
A. @Цитомегаловірусна інфекція
B. Кір
C. Епідемічний паротит
D. Інфекційний мононуклеоз
E. Герпес

48. Хворий з пересадженою ниркою отримував імуносупресивну терапію, помер від інтоксикації. При
морфологічному дослідженні в легенях, нирках, підшлунковій залозі виявлені гігантські клітини з великими
ядрами та світлою зоною по периферії, які нагадують совине око. Назвати захворювання:
A. Туберкулез
B. @Цитомегалія
C. Сифиліс
D. Лепра
E. Чума
49. У померлого наркомана, 30 років, який страждав на ВІЛ-інфекцію при патоморфологічному
дослідженні виявлено, що обидві легені ущільнені, темно-бордово-сірі, мало повітряні, міжальвеолярні
перегородки густо інфільтровані лімфоцитами, частина альвеолоцитів трансформовані у великі клітини, з
центрально розташованим круглим ядром зі світлим обідком, і які нагадують “совине око”. Яка опортуністична
інфекція викликала пневмонію у даного хворого?
A. @Цитомегаловірус
B. Пневмоциста карінії
C. Атипова мікобактерія
D. Герпес-вірус
E. Токсоплазма

КАРАНТИННІ ІНФЕКЦІЇ
ВАРІАНТ №
1. У хлопчика, 7 років, з’явилися болі в ділянці пупка, незначне здуття живота, бурчання, нудота.
Впродовж тижня закрепи замінювалися проносами (жовтуваті випорожнення з незначними домішками слизу).
Випорожнення часті, рідкі, зі значною кількістю слизу, які мали зелене забарвлення і пінистий вигляд.
Мікроскопічне дослідження біопсії різних відділів кишки показало наявність у слизовій оболонці 12-ти палої
кишки серпоподібних трофозотів на поверхні мікроворсин епітеліальних клітин та ентероколіт з помірною
лімфогістіоцитарною інфільтрацією слизової оболонки. Ваш діагноз.
A. @Лямбліоз
B. Кампілобактерний ентероколіт
C. Дизентерія
D. Сальмонельоз
E. Амебіаз

2. Чоловiк, впродовж 3 рокiв працював в однiй з африканських країн. Через мiсяць пiсля повернення в
Україну звернувся до офтальмолога зi скаргами на бiль в очах, набряки повiк, сльозоточивiсть i тимчасове
послаблення зору. Пiд кон’юнктивою ока були виявленi гельмiнти розмiрами 30-50 мм, якi мали видовжене
ниткоподiбне тiло. Ваш діагноз.
A. @Фiлярiоз
B. Дифiлоботрiоз
C. Аскаридоз
D. Ентеробiоз
E. Трихоцефальоз

3. Турист нещодавно повернувся з країн Середньої Азiї, де є багато москiтiв. У нього на шкiрi з’явилися
невеликi виразки з нерiвними краями. В цьому випадку можна припустити наступне захворювання:
A. @Дерматотропний лейшманiоз
B. Токсоплазмоз
C. Короста
D. Демодекоз
E. Специфiчний мiаз

4. Пiд час операцiї в печiнцi хворого виявленi дрiбнi мiхурцi малих розмiрiв з незначною кiлькiстю
рiдини, якi щiльно прилягають один до одного. Який гельмiнтоз виявився у хворого?
A. @Альвеококоз
B. Фасцiольоз
C. Опiсторхоз
D. Клонорхоз
E. Дiкроцелiоз

5. Під час розтину в печінці померлого був виявлений утвір у вигляді міхура округлої форми з гладкою
поверхнею діаметром 5 см. В його порожнині знаходилася велика кількість дрібних міхурців з прозорим
безбарвним вмістом. Тканина печінки навколо міхура склерозована. Вкажіть ймовірний патологічний процес.
A. @Гідатидозний ехінококоз
B. Альвеококоз
C. Цистицеркоз
D. Опісторхоз
E. Шистосомоз

6. У чоловіка, 40 років, в ділянці шиї виникло почервоніння та набряк шкіри і з часом розвинувся
невеликий гнійник. На розрізі осередок щільний, жовто-зеленого забарвлення. В гнійних масах видно білі
крупинки. Гістологічно виявлено друзи грибка, плазматичні та ксантомні клітини, макрофаги. Вкажіть найбільш
ймовірний вид мікозу.
A. @Актиномікоз
B. Аспергільоз
C. Кандидоз
D. Споротрихоз
E. Кокцидіоїдомікоз

7. При гiстологiчному дослiдженнi бiоптату шкiри виявленi гранульоми, якi складаються з


макрофагальних вузликiв з наявнiстю лiмфоцитiв та плазматичних клiтин. Крiм того, зустрiчаються великi
макрофаги з жировими вакуолями, якi мiстять запакованих у виглядi куль збудникiв захворювання (клiтини
Вiрхова). Грануляцiйна тканина добре васкуляризована. Для якого захворювання характерна описана
гранульома?
A. @Лепра
B. Туберкульоз
C. Сифiлiс
D. Риносклерома
E. Сап
8. Чоловiк, 36 рокiв, лiсник за фахом, через тиждень пiсля тривалого перебування у весняному лiсi гостро
захворiв - гарячка, головний бiль, порушення свiдомостi, епiлептиформнi напади, смерть розвинулася на 3-й день
хвороби. На розтинi тiла: набряк головного мозку, множиннi крапчастi геморагiї; пiд час мiкроскопiчного
дослiдження - периваскулярний та перицелюлярний набряк, множиннi периваскулярнi, переважно лiмфоцитарнi,
iнфiльтрати. Дiагностуйте основне захворювання:
A. @Клiщовий енцефалiт
B. Менiнгококова iнфекцiя
C. Полiомiєлiт
D. Гнiйний енцефалiт
E. Церебро-васкулярна хвороба

9. Хворий скаржився на лихоманку, сильний головний біль, задишку, серцебиття. При огляді виявлено:
педикульоз, розеoли і петехії на шкірі грудної клітини, пролежні гомілок, гангрену стопи. Помер при ознаках
серцевої недостатності. Гістологічно в тканині довгастого мозку спостерігаються гіперемія, стази,
периваскулярні муфти з плазматичних клітин і осередки проліферації мікроглії (гранульоми Попова). Ваш
діагноз.
A. @Епідемічний висипний тиф
B. Сибірковий менінгоенцефаліт
C. Менінгококовий менінгіт
D. Черевний тиф
E. Краснуха

10. У померлого внаслідок серцевої недостатності на шкірі виявляються сліди висипу у вигляді плям і
крапок. У ділянці крижі, остистих відростків хребців - пролежні. При мікроскопічному дослідженні ЦНС, шкіри,
надниркових залоз, в судинах мікроциркуляторного русла і дрібних артеріях - деструктивно-проліферативний
ендотромбоваскуліт з наявністю гранульом Попова; в серці - інтерстиціальний міокардит. Який з перелічених
діагнозів навірогідний?
A. @Висипний тиф
B. Ку-гарячка
C. Черевний тиф
D. Вузликовий периартеріїт
E. ВІЛ-інфекція
11. На розтині хворого, який помер при наростанні явищ серцевої недостатності, виявлено: серозний
менінгіт, некроз і крововиливи в надниркових залозах, пролежні. Мікроскопічно: деструктивно-проліферативний
ендотромбоваскуліт (гранульоми Попова) у довгастому мозку, інтерстиціальний міокардит. В анамнезі: поява на
5-й день захворювання екзантеми на шкірі, гіпотонії; на 3-му тижні - бульбарних розладів: (порушення ковтання і
дихання). Яке захворювання має місце?
A. @Висипний тиф
B. Скарлатина
C. Кір
D. Черевний тиф
E. Грип

12. Хворого, 30 рокiв, турбують напади ознобу, гарячки, профузного поту, якi повторюються кожний
третiй день. Хворiє 2 тижнi. Склери та шкіра - яскраво жовтяничнi. Печiнка та селезiнка збiльшенi. Рiк тому
працював в Африцi. Який з дiагнозiв найбiльш вiрогiдний?
A. @Малярiя
B. Рак головки пiдшлункової залози
C. Сепсис
D. Вiрусний гепатит
E. Лептоспiроз
13. У хворого на важку плевропневмонію з мокротиння висіяли Yersinia pestis. Яке інфекційне
захворювання розвинулось?
A. @Чума
B. Сибірка
C. Бруцельоз
D. Туберкульоз
E. Лепра

14. При розтині, померлого від чуми хворого, на фоні геморагічного синдрому виявлено: геморагічний
некроз шкіри стегна, лимфангіт, паховий геморагічний лімфаденіт. Назвіть форму чуми.
A. @Шкірно-бубонна
B. Бубонна
C. Первинно-септична
D. Первинно-легенева
E. Геморагічна

15. Робітник тваринницької ферми гостро захворів і при наростаючих явищах інтоксикації помер. При
розтині тіла встановлено: селезінка збільшена, дрябла, на розрізі пульпа темно-вишневого кольору, з рясними
вишкребками. М’які мозкові оболонки на склепінні та основі мозку набряклі, просякнуті кров’ю, мають темно-
червоний колір ("шапочка кардинала"). Мікроскопічно: серозно-геморагічне запалення оболонок і тканин
головного мозку з руйнуванням стінок дрібних судин. Який найімовірний діагноз?
A. @Сибірка
B. Туляремія
C. Бруцельоз
D. Чума
E. Холера
16. Чоловік, 40 років, м’ясник, помер від сепсису. На правій щоці виявляється конусовидний, щільний
темно-червоний ільфільтрат 6 см з чорною корочкою у центрі. Права половина обличчя, шиї – різко набряклі,
щільні. При мікроскопічному дослідженні інфітрату – найгостріше серозно-геморагічне запалення, в центрі
інфільтрату – некроз епідермісу і підлеглих шарів. Ваш діагноз?
A. @Сибірська виразка
B. Чума
C. Туляремія
D. Флегмона шиї
E. Фурункул

17. Патологоанатом у біоптаті шкіри побачив гостре серозно-геморагічне запалення і ділянку некрозу. З
анамнезу: захворювання почалося з появи невеликої червоної плями, в центрі якої утворився міхур з серозно-
геморагічною рідиною. Згодом центральна частина стала чорною. Який діагноз найвірогідний?
A. @Сибірковий карбункул
PATHOMORPHOLOGY.

Introduction
#
1
A histological specimen presents the tissue that contains cells having no processes and a few tens of nuclei each. One of
cell surfaces has a corrugated zone that provides secretion of hydrolytic elements. What tissue is it?
Nerve tissue
Cartilaginous tissue
Muscular tissue
@Osseous tissue
Epithelial tissue
#
2
In course of a conditional experiment the development of mesenchyma cells was completely inhibited. Development of
the following muscular tissue will be disturbed:
Epidermal muscular tissue
Cardiac muscular tissue
@Smooth muscular tissue
Neural muscular tissue
Skeletal muscular tissue
#
3
Microspecimen of a child's finger skin reveals subnormal development of epidermis. What embryonic leaf was damaged
in course of development?
Mesenchyma
Entoderm
@Ectoderm
Ectomesenchyma
Mesoderm
#
4
At a vulgar pemphigus in a skin epidermis forms vesicles, which break out a wholeness of stratum spinosum and stratum
granulosum. What function of an epithelium is broken first of all?
@Barrier
Regenerative
Absorptive
Secretory
Endocrine
#
5
An embryon has signs of disturbed process of dorsal mesoderm segmentation and somite generation. What part of skin is
most likely to have developmental abnormalities?
Hair
Sebaceous glands
@Derma
Epidermis
Sudoriferous glands
#
6
A specimen of connective tissue of derma was stained with Sudan III and hematoxylin. There are clusters of big
polygonal cells that turned orange. Their nuclei are flattened and located on periphery. What tissue is it?
Reticular connective
Lamellar osseous
Brown adipose
@White adipose
Hyaline cartilaginous
#
7
A patient is complaning of dryness of head skin, itching, fragility and loss of hair. After examination he was diagnosed
with seborrhoea. Disturbed activity of which cells caued this condition?
@Cells of sebaceous glands
Cells of sudoriferous glands
Epithelial cells
Melanocytes
Adipocytes
#
8
During examination of a child's oral cavity a pediatrician found 8 incisors. The child's development corresponds to his
age. How old is the child?
12-15 months
7-8 months
6-7 months
16-20 months
@10-12 months
#
9
A mother consulted the doctor about her one year old child, who has got six teeth come out. How many teeth should the
child of such age have?
7
@8
12
6
10
#
10
There is the change of teeth at the 6-8-year-old children: deciduous are replaced by permanent. What embrionic tissues
are the sources of permanent teeth tissues formation?
I, II brachial arches
Mesodermal epithelium and mesenhime
Entodermal epithelium and mesoderm
Entodermal epithelium of a tooth plate and mesenhime
@Ectodermal epithelium of a tooth plate and mesenhime
#
11
A patient had his tooth extracted. The lingual surface of this tooth was smaller than the buccal one. Masticatoty surface
has oval form. Deep transverse sulcus separates buccal and lingual tubercles. The root is strongly compressed in mesio-
distal direction and has longitudinal sulci on its aproximal surfaces, it is bifurcated. What tooth was extracted?
Lower canine
@First upper premolar
First lower premolar
Second upper premolar
Upper canine
#
12
During examination a dentist revealed cervical caries of right inferior incisors as well as enlargement of a certain group of
lymph nodes. What lymph nodes are enlarged?
@Submental
Superficial cervical
Facial
Deep cervical
Occipital
#
13
A patient applied to a doctor with complaints about noise and painful sensations in his ear. Objectively: a patient is ill
with acute respiratory disease, rhinitis. The infection that caused inflammation of tympanic cavity could have penetrated
into it through the following pharynx opening:
Tympanic opening of auditory tube
Fauces
Choanae
Aperture of larynx
@Pharyngeal opening of auditory tube
#
14
A histological specimen of an oral cavity organ demonstrates that the organ's anterior surface is lined with multilayer
squamous nonkeratinous epithelium, and its posterior surface - with multiserial ciliated epithelium. What organ is it?
Cheek
Lip
Gingiva
Hard palate
@Soft palate
#
15
There is a specimen of soft palate where both oral and nasal surfaces can be seen. It was revealed that oral cavity had
damaged epithelium. What epithelium is damaged?
Multirowed ciliated epithelium
Multistratal prismatic nonkeratinizing
Multistratal squamous keratinizing
Multistratal cubical nonkeratinizing
@Multistratal squamous nonkeratinizing
#
16
While examining the oral cavity a stomatologist revealed inflammation of papillae on the border of the median and
posterior third of the back of tongue. What papillae are inflamed?
Papillae filiformes
@Papillae vallatae
Papillae conicae
Papillae fungiformes
Papillae foliatae
#
17
Histological study of an extirpated pulp revealed some cylindrical cells in its peripheral layer. What are these cells called?
@Odontoblasts
Fibroblasts
Monocytes
Ameloblasts
Myofibroblasts
#
18
A microspecimen of the submandibular salivary gland shows some basket-shaped cells concentrated around the acines
and excretory ducts. These cells surround bases of the serous cells and are called myoepitheliocytes. These cells relate to
the following tissue:
Special connective tissue
Epithelial tissue
Loose fibrous connective tissue
Neural tissue
@Muscular tissue
#
19
What substance makes saliva viscous and mucous, has protective function, protects mucous membrane of oral cavity from
mechanical damage?
Kallikrein
Lysozyme
Glucose
@Mucin
Amylase
#
20
In order to speed up healing of a wound of oral mucosa a patient was prescribed a drug that is a thermostable protein
occuring in tears, saliva, mother's milk as well as in a new-laid hen's egg. It is known that this protein is a factor of natural
resistance of an organism. What is it called?
@Lysozyme
Interferon
Interleukin
Complement
Imanine
#
21
During histological examination of the stomach it was found out that glands contain very small amount of pariental cells
or they are totally absent. Mucose membrane of what part of the stomach was studied?
Esophageal
Fundus of stomach
Body of stomach
Cardiak part
@Pyloric part
#
22
The reason of occurrence of some diseases of an oral cavity is connected with structural peculiarities of its mucous
membrane. What morphological attributes characterize these features?
Transitional epithelium, no muscularis mucosa
Well developed muscularis, no submucosa
Transitional epithelium, no submucosa
@No muscularis mucosa, stratified squamous epithelium
Simple columnar ciliated epithelium
#
23
Endoscopic examination of duodenum revealed a tumour of the major papilla. This pathological formation is localized in
the following part of duodenum:
@Descending part
Superior part
Horizontal part
Superior flexure
Ascending part
#
24
Premature infants have syndrom of respiratory failure. Failure of what aerohematic barriere component underlies this
pathology?
@Surfactant
Alveolocytes
Basal membrane of alveolocytes
Basal membrane of endothelium
Capillary endothelium
#
25
A child has inhaled a button. Where is it likely to be?
@In the right main bronchus
In the left main bronchus
In the trachea
In the larynx
In the esophagus
#
26
In the specimen of one of the parts of respiratory system a tubular organ was found. It has low epithelium, well developed
muscular tunic, glands and cartilage are absent. Name this organ:
Median bronchs
Larynx
Major bronchs
@Minor bronchs
Trachea
#
27
Examination of a tubular organ revealed that its middle membrane consisted of solid hyaline rings. What epithelium lines
mucous membrane of this organ?
Monolayer prismatic with a limbus
@Multinuclear prismatic ciliated
Multilayer squamous nonkeratinous
Monolayer prismatic glanduous
Monolayer cubic
#
28
In course of a small pelvis operation it became necessary to ligate an ovarian artery. What formation may be accidentally
ligated together with it?
Internal iliac vein
Uterine tube
Urethra
Round ligament of uterus
@Ureter
#
29
Examination of a newborn boy's genitalia revealed an urethral hiatus that opens on the undersite of his penis. What
malformation is it?
Monorchism
Hermaphroditism
Epispadia
@Hypospadias
Cryptorhidism
#
30
A young man complains about urination disorder. Examination of the external genitals revealed that the urethra was split
and urine could flow out of this orifice. What anomaly of the external genitals development is it?
@Epispadia
Hypospadia
Paraphimosis
Phimosis
Hermaphroditism
#
31
The electronic microphoto of kidney fragment has demonstrated the afferent glomerular arteriole, which under its
endothelium has giant cells, containing secretory granules. Name the type of these cells:
Smoothmuscular
Interstitial
@Juxtaglomerular
Mesangial
Juxtavascular
#
32
A histological specimen of a kidney shows a part of the distal tubule going between the afferent and efferent arteriol e.
The cells building the tubule wall have dense nuclei; basal membrane is absent. Such structural formation is called:
Juxtavascular cells
@Macula densa
-
Mesangial cells
Juxtaglomerular cells
#
33
Electron-microscope investigation of cortical substance of a kidney reveals some structures lined with prismatic
epithelium that normally has brush border and deep plicae of plasmolemma in its basal part. There is a big number of
mitochondrions between these plicae. These structures belong to the following part of a nephron:
Distal straight tubule
@Proximal tubule
Renal corpuscle
Distal convoluted tubule
Henle's loop
#
34
On autopsy of a still-born infant abnormalities have been revealed: ventricles are not separated, a single arterial trunk
originates from the right part. For what class of vertebrates is such heart construction characteristic?
Fishes
Birds
Mammals
@Amphibian
Reptiles
#
35
During ultrasound examination of the heart the doctor observes the leaves of the mitral valve. What happends to them
during the systole?
They turn inside the cavity of the atrium
They clasp with wall of the vessel
They close up covering the lumen of the orifice
@They turn inside the cavity of the ventricle
They clasp with the walls of the atrium
#
36
During the fetal period of the development in the vascular system of the fetus a large arterial (Botallo's) duct is
functioning which converts into lig.arteriosum after birth. What anatomical formations does this duct connect?
Aorta and superior vena cava
@Pulmonary trunk and aorta
Right and left auricle
Pulmonary trunk and superior vena cava
Aorta and inferior vena cava
#
37
A histological specimen shows a blood vessel. Its inner coat is composed by endothelium, subendothelium and internal
elastic membrane. The middle coat is enriched with smooth myocytes. Such morphological characteristics are typical for
the following vessel:
Capillary
@Muscular-type artery
Non-muscular vein
Elastic-type artery
Muscular-type vein
#
38
In the microspecimen of red bone marrow there were revealed multiple capillares through the walls of which mature
blood cells penetrated. What type of capillares is it?
Fenestrational
@Sinusoidal
Visceral
Lymphatic
Somatical
#
39
Intralobular capillaries of a liver specimen have wide irregular lumen. Basal membrane is absent in the major part of the
capillary. What type of capillaries is it?
Somatic
Postcapillaries
Precapillaries
@Sinusoid
Visceral
#
40
A 54-year-old man was admitted to the hospital with complaints of pain in the right subcostal region, vomiting with
blood. Objectively: enlarged liver, varicose veins in the stomach and esophagus. Disfunction of what vessel is likely to
have taken place?
Aorta abdominalis
Vena hepatica
Vena cava inferior
@Vena porta
Vena cava superior
#
41
Morphological examination revealed in histological specimen of biopsy material an irregular-shaped vessel. Its middle
membrane is formed by bundles of smooth myocytes and layers of connective tissue. What type of vessel is it?
Arteriole
Artery uf muscular type
@Vein of muscular type
Lymphatic vessel
Venule
#
42
A histological specimen of spleen shows a vessel with a wall consisting of endothelium and subendothelial layer, median
membrane is absent, exterior membrane inosculates with the layers of spleen connective tissu E. What vessel is it?
Artery of muscular type
Vein of muscular type
@Vein of non-muscular type
Arteriole
Capillary
#
43
Where should the cathetor for evacuation of the lymph from the thoracic lymph duct be inserted?
To the inferior vena cava
To the superior vena cava
To the left inguinal vein
@To the left venous corner
To the right venous corner
#
44
When a patient with traumatic impairment of the brain was examined, it was discovered that he had stopped to distinguish
displacement of an object on the skin. What part of the brain was damaged?
Parietal zone of the cortex
@Posterior central gyrus
Occipital zone of the cortex
Frontal central gyrus
Frontal zone
#
45
A 60-year-old man after cerebral hamorrhage felt asleep for a long time. Damage of what structure caused this state?
Nuclears of the cerebral nerves
@Reticular formation
Cortex of the large hemispheres
Black substances
Hippocampus
#
46
A patient becomes quickly tired during his work. In vertical position with closed eyes he is dizzying and loosing
equilibrium. Skeletal muscle tone is reduced. Which of the below mentioned brain structures is damaged?
@Cerebellum
Hypothalamus
Precentral gyrus of cerebral hemispheres cortex
Basal ganglia
Thalamus
#
47
As a result of craniocerebral trauma a patient reveals the following symptoms: intention tremor, dysmetry,
adiadochokinesis, dysarthria. What structure of the brain is injured?
Motor cortex
Pale sphere
@Cerebellum
Striatum
Black substance
#
48
A 50 year-old patien was injured on the occipital region of the head. The closed skull’s trauma was diagnosed. She was
taken to the hospital. The medical examination: deregulation of walking and balance, trembling of arms. What part of
brain was injured?
The inter-brain
The spinal cord
The medulla oblongata
The mind-brain
@The cerebellum
#
49
A patient with brain bloodstream disorder has got difficulties with swallowing, he can choke over while eating liquid
food. What part of the brain is damaged?
@Medulla
Midbrain
Cerebellum
Thalamencephalon
Cervical part of the spinal medulla
#
50
A sensory nerve ganglion consists of roundish neurocytes with one process that divides into axon and dendrite at a certain
distance from perikaryon. What are such cells called?
Unipolar
Apolar
Multipolar
Bipolar
@Pseudounipolar
#
51
During investigation of patient, it was found formation in the white substance of cerebral hemispheres with location in the
knee and frontal part of posterior crus of internal capsule. Fibres of what conductive tract of the brain will be disrupted?
Tr. frontothalamicus
Tr. parietooccipitopontinus
@Tr. pyramidalis
Tr. frontopontinus
Тr. thalamocorticalis
#
52
A patient was admitted to the hospital with a wound in the region of his neck. The examination revealed the damaged
nerve, situated in front of the anterior ladder muscle. What nerve is damaged?
Vagus
@Phrenic
Cervical part of sympathetic trunk
Sublingual
Glossopharyngeal
#
53
A histological spacemen presents parenchymal organ, which has cortex and medulla. Cortex consists of epitheliocytes
bars, between them there are blood capillaries; the bars form three zones. Medulla consists of chromaffinocytes and
venous sinusoids. What organ has these morphological features?
@ Adrenal gland
Kidney
Thyroid
Lymph node
Thymus
#
54
Examination of a microspecimen made of an unknown organ revealed some acini that contained 10-15 cone cells with
basophilic cytoplasm, round nucleus and well developed granular endoplasmic reticulum. An acinus is surrounded by a
basal membrane with myoepithelial cells localized in its splitting. What organ is the slice made of?
Sublingual gland
@Parotid gland
Pancreas
Liver
Lungs
#
55
A microspecimen of parotid gland presents secretory acines with serous cells that synthesize mostly enzymes. According
to the chemical composition classification, the parotid gland relates to the following glands:
Seromucous
Enzymatic
@Serous
-
Mucous
#
56
Examination of an ovary specimen stained by hematoxylin-eosine revealed a follicle in which follicular epithelium
consisted of 1-2 layers of cubic cells. There was also a bright red membrane around the ovocyt E. What follicle is it?
@Primary
Secondary
Primordial
Atretic
Mature
#
57
In the pubertal period cells of the male sexual glands start producing the male sexual hormone testosterone that is
responsible for formation of the secondary sexual characters. What cells of the male sexual glands produce this hormone?
Spermatozoa
@Leidig cells
Sertoli's cells
Sustenocytes
Sustentacular cells
#
58
During embryogenesis trophoblast develops into an organ rudiment that has endocrinal function. What rudiment is it?
@Villous chorion
Yolk sac
Amnion
Umbillical cord
Allantois
#
59
Inflammatory process of modified subserous layer around cervix of the uterus caused an intensive pain syndrome. In what
region of genitals does the pathological process take place?
Myometrium
Perimetrium
Mesometrium
@Parametrium
Endometrium
#
60
The specimens present sections of haemopoetic and immunogenetic organs. Organ has lymphoid tissue forming different
structures (lymph nodes, lobules, bars). In what organ does antigen-independent proliferation and differantiation take
place?
@Thymus
Lymphatic nodes
Hemolymph nodes
Tonsil
Spleen
#
61
Medullary substance of a hemopoietic organ's lobule in a histological specimen is lighter coloured and contains epithelial
bodies. What organ are these morphological preperties typical for?
@Thymus
Kidney
Spleen
Lymph node
Liver
#
62
A 6-month-old child is suffocating while lying on his back. The palpation on the anterior wall of trachea up to jugular
notch of the sternum revealed tumor-like mass, stretching into the anterior mediastinum. What from the below mentioned
can squeeze trachea?
Parathyroid lymph nodes
Parathyroid glands
Thyroid gland
@Thymus gland
Peritracheal lymph nodes
#
63
In a histological specimen parenchyma of an organ is represented by lymphoid tissue that forms lymph nodes; the latter
are arranged in a diffuse manner and enclose a central artery. What anatomic formation has such morphological structure?
Thymus
@Spleen
Tonsil
Lymph node
Red bone marrow
#
64
A histological specimen presents an organ that has both cortical and medullary substance. Cortical substance consists of
an external zone that contains lymph nodules as well as of a paracortical zone. Medullary substance contains medullary
cords, sinuses and trabecules. What organ possesses these morphological signs?
Adrenal glands
Kidney
Thymus
Spleen
@Lymph node
#
65
An electronic microphotograph shows a macrophagic cell with erythrocytes at different stages of differentiation located
along its processes. This is the cell of the following organ:
@Red bone marrow
Spleen
Tonsil
Lymph node
Thymus
#
66
Following exposure to radiation a lot of mutant cells appeared in a patient. Some time later most of them were detected
and destroyed by the following cells of the immune system:
@T-lymphocytes-killers
B-lymphocyte
T-lymphocytes-supressors
Plasmoblasts
Stem cells
#
67
Karyotyping of healthy man cells is carried out. In the karyotype there was found out a fine acrocentric odd chromosome.
What chromosome is it?
Group C chromosome
Group B chromosome
@Y-chromosome
Group A chromosome
Х-chromosome
#
68
Epithelium regeneration of mucous membrane of oral cavity (cell reproduction) was accompanied by semiconservative
DNA replication (selfreproduction). Nucleotides of a new DNA chain are complementary to:
Introns
Sense codons
DNA-polymerase enzyme
RNA-polymerase enzyme
@Maternal chain
#
69
Life cycle of a cell includes a process of DNA autoreduplication. As a result of this process monochromatid
chromosomes become bichromatid. This phenomenon is observed within the following period of the cell cycle:
G2
G1
@S
G0
M
#
70
An electronic micrograph presents a cell that has no nucleole and nuclear membrane. Chromosomes has free position,
centrioles migrate to the poles. What phase of cell cycle is it typical for?
@Prophase
Telophase
Metaphase
Interphase
Anaphase
#
71
Moving of the daughter chromatids to the poles of the cell is observed in the mitotically dividing cell. On what stage of
the mitotic cycle is this cell?
Interfase
Metaphase
@Anaphase
Prophase
Telophase
#
72
The study of mitotic cycle phases of onion root revealed the cell, in which the chromosomes are situated in the equatorial
plane, forming a star. What stage of the cell mitosis is it?
Prophase
Anaphase
Telophase
Interphase
@Metaphase
#
73
While studying maximally spiralized chromosomes of human karyotype the process of cell division was stopped in the
following phase:
Telophase
Prophase
Anaphase
Interphase
@Metaphase
#
74
Chromosomes forming figures of "Stars" located in the centre of the cell are seen in the microslide. What phase of mitosis
is there in this cell?
@Metaphase
Anaphase
Telophase
Prophase
Interphase
#
75
As a result of expression of some genome components the embryo cells acquire typical morphological, biochemical and
functional properties. Name this process:
Induction
@Differentiation
Reception
Capacitation
Determination
#
Ultrastructure
#
1
Microscopic analysis of human heart cells revealed some oval oganellas, their tunic being formed by two membranes: the
external one is smooth, and the internal one forms crista. Biochemical analysis determined the presence of ATP-
synthetase enzyme. What organellas were analysed?
Centrosomes
Ribosomes
@Mitochondrions
Lysosomes
Endoplasmic reticulum
#
2
A 50-year-old woman with a history of unstable angina suffers an acute myocardial infarction. Thrombolytic therapy with
tissue plasminogen activator (tPA) is administered to restore coronary blood flow early in the course of this myocardial
infarction. In spite of this therapy, the degree of myocardial fiber injury may increase because of which of the following
cellular abnormalities?
@Cytoskeletal intermediate filament loss
A decreased intracellular pH from anaerobic glycolysis
Increase in toxic oxygen radicals
Mitochondrial swelling
Nuclear chromatin clumping and decreased protein synthesis
#
3
Ultrastructural investigation of a liver biopsy revealed that between the mitochondria there were numerous flat cisterns
and bubbles with secretory granules circumscribed with membrane. Name a cell structure with the hyperplasic
constituents?
@Golgi apparatus
Pinocytosis bubbles
Endoplasmic reticulum
Lysosoms
Microtubes
#
4
The cell synthesized its own compounds (proteins and carbohydrates) and has separated them as microdrops of a secret.
What organelle was included in the work on anending stage of this process connected with separation of asecret?
@Golgi body
Rough-surfaced ER
Ribosomes
Smooth-surfaced ER
Lysosomes
#
5
A 3-month- old infant dies and autopsy is requested. Electron microscopic examination of liver tissue revealed a great
amount of flat cisterns and vesicles with secretory granules, surrounded by membrane, scattered among numerous
mitochondria. Which of the following cell ultra structures has been shown to be hyperplastic?
@Golgi complex.
Pinocytic vesicles.
Endoplasmic reticulum.
Lysosomes.
Filaments.
#
6
During the electron microscopic examination of liver biopsy, a great amount of flat cistern and sacs with secretory
globules, surrounded by membrane were founded, among the numerous mitochondria. What ultra structure hyperplasia is
meant?
Filaments
Pinocytosis sacs
Endoplasmatic reticulum
Lysosomes
@The Golgi complex
#
7
A granular endoplasmic reticulum and Golgi apparatus are well developed in cells. What main function do these cells
perform?
@Release of protein secretion
Phagocytosis and digestion of entrapped particles
Production of energy
Transmission of nervous stimulation
Production of protein
#
8
Golgi complex exports substances from a cell due to the fusion of the membrane saccule with the cell membrane. The
saccule contents flows out. What process is it?
Facilitated diffusion
@Exocytosis
Active transport
All answers are false
Endocytosis
#
9
Ultramicroscopical examination of "dark" hepatocyte population in the cell cytoplasm detected a developed granular
endoplasmic reticulum. What function has this organella in these cells?
Deintoxicative function
Calcium ion depositing
@Synthesis of blood plasma proteins
Bile production
Carbohydrate synthesis
#
10
Low level of albumins and fibrinogen was detected in the patient's blood. Decreased activity of what organelle of the liver
hepatocytes can cause it?
Mitochondrions
Agranular endoplasmatic net
Lysosomes
Golgi complex
@Granular endoplasmatic net
#
11
A tissue sample of benign tumor was studied under the electron microscope. A lot of small (15-20 nm) spherical bodies,
consisting of 2 unequal subunits were detected. These are:
Microtubules
Golgi complex
Mitochondria
@Ribosomes
Smooth endoplasmic reticulum
#
12
Labeled aminoacids alanine and tryptophane were introducted to a mouse in order to study localization of protein
biosynthesis in its cells. Around what organellas the accumulation of labeled aminoacids will be observed?
Lysosomes
@Ribosomes
Agranular endoplasmic reticulum
Cell centre
Golgi apparatus
#
13
Formation of ribosome subunits in a cell was disturbed in course of an experiment (by means of activated mutagenic
factors). This will have an effect on the following metabolic process:
Carbohydrate biosynthesis
Photosynthesis
Biological oxidation
ATP synthesis
@Protein biosynthesis
#
14
In course of practical training students studied a stained blood smear of a mouse with bacteria phagocyted by leukocytes.
What cell organella completes digestion of these bacteria?
Golgi apparatus
@Lisosomes
Ribosomes
Mytochondrions
Granular endoplasmic reticulum
#
15
Electron microscopic study of a cell revealed roundish bubbles confined by a membrane and containing a lot of various
hydrolytic enzymes. It is known that these organellas provide intracellular digestion and protective functions. These
elements are:
Endoplasmic reticulum
Ribosomes
Mitochondria
Centrosomes
@Lysosomes
#
16
The cell of the laboratory animal was overdosed with Roentgen rays. As a result albuminous fragments formed in the
cytoplasm. What cell organoid will take part at their utilization?
@Lysosomes
Endoplasmic reticulum
Cells centre
Ribosome
Golgi complex
#
17
In a genetical laboratory in course of work with DNA molecules of white rats of Wistar's line a nucleotide was substituted
for another one. At that only one amino acid was substituded in the peptide. This result is caused by the following
mutation:
Translocation
Displacement of reading frame
@Transversion
Deletion
Duplication
#
18
It was found out that some compounds, for instance fungi toxins and some antibiotics can inhibit activity of RNA-
polymerase. What process will be disturbed in a cell in case of inhibition of this enzyme?
Processing
Replication
Translation
@Transcription
Reparation
#
19
It was proved that a molecule of immature mRNA (precursor mRNA) contained more triplets than amino acids found in
the synthesized protein. The reason for that is that translation is normally preceded by:
Reparation
Mutation
@Processing
Initiation
Replication
#
20
Examination of a patient revealed reduced contents of magnesium ions that are necessary for attachment of ribosomes to
the granular endoplasmatic reticulum. It is known that it causes disturbance of protein biosynthesis. What stage of protein
biosynthesis will be disturbed?
Termination
@Translation
Aminoacid activation
Replication
Transcription
#
21
A woman who was sick with rubella during the pregnancy gave birth to a deaf child with hare lip and cleft palate. This
congenital defect is an example of:
Genocopy
Down’s syndrome
Edward’s syndrome
Patau’s syndrome
@Phenocopy
#
22
In some regions of South Africa there is a spread sickle-shaped cell anemia, in which erythrocytes have shape of a sickle
as a result of substitution of glutamin by valine in the hemoglobin molecule. What is the cause of this disease?
@Gene mutation
Disturbance of mechanisms of genetic information realization
Genomic mutations
Crossingover
Transduction
#
23
46 chromosomes were revealed on karyotype examination of the 5-year-old girl. One of the 15th pair of chromosomes is
longer than usual due to connected chromosome from the 21 pair. What type of mutation does this girl have?
Deletion
Insufficiency
Inversion
Duplication
@Translocation
#
24
Part of the DNA chain turned about 180 degree due to gamma radiation. What type of mutation took place in the DNA
chain?
Replication
Translocation
@Inversion
Doubling
Deletion
#
25
During the postsynthetic period of mitotic cycle the synthesis of tubulin proteins was disturbe d. These proteins take part
in construction of division spindle. It can lead to the disturbance of:
@Chromosomes' disjunction
Mitosis duration
Cytokinesis
Spiralization of chromosomes
Despiralization of chromosomes
#
26
During the postsynthetic period of mitotic cycle the synthesis of proteins - tubulines, which take part in the mitosis
formation, was destroyed. It can cause the impairment of:
Chromosome despiralization
Cytokinesis
Duration of mitosis
@Chromosome separation
Chromosome spiralization
#
27
In the woman of 60 years the tumor uterus is removed. At the analyses of tumor cells multipole mitoses were found with
divergence of chromosomes to many poles. What organelles were distroyed?
@Centrosome
Secondary lisosomes
Peroxisomes
Ribosomes
Rough-surfaced ER
#
28
Hurtnup's disease is caused by point mutation of only one gene. This results in abnormal absorption of tryptophane in the
intestine as well as its abnormal reabsorption in renal tubules. This causes synchronous disorders in digestive and urinary
excretion systems. What genetic phenomenon is observed in this case?
Complementary interaction
Polymery
Semidominance
Codominance
@Pleiotropy
#
29
After prophylactic medical examination a 7 y.o. boy was diagnosed with Lesch-Nyhan syndrome (only boys fall ill). His
parents are healthy, but his grandfather by his mother's side has the same disease. What type of inheritance is it?
@Recessive, sex-linked
Autosomal and recessive
Semidominance
Dominant, sex-linked
Autosomal and dominant
#
30
A 32 y.o. man is tall, he has gynecomastia, adult woman pattern of hair distribution, high voice, mental deficiency,
sterility. Provisional diagnosis is Klinefelter's syndrome. In order to specify diagnosis it is necessary to analize:
@Caryotype
Blood group
Genealogy
Leukogram
Spermatogenesis
#
31
An 8 month old child has non-closed palate, a number of eye defects, microcephaly, disorder of cardiovascular system.
Cytogenetic analysis revealed 47 chromosomes with an additional 13th chromosome. What diagnosis can be made on the
basis of clinical observations and cytogenetic examinations?
Down's syndrome
Edwards' syndrome
@Patau's syndrome
Cat cry syndrome
Klinefelter's syndrome
#
32
Autopsy of a newborn boy revealed polydactylia, microcephalia, cheiloschisis and uranoschisis as well as hypertrophy of
parenchimatous organs. These defects correspond with the description of Patau's syndrome. What is the most probable
cause of this pathology?
@Trisomy of the 13th chromosome
Partial monosomy
Trisomy of the 21st chromosome
Nondisjunction of sex chromosomes
Trisomy of the 18th chromosome
#
33
Abnormal chromosome disjunction during meiosis resulted in formation of: an ovum with 22 autosomes and polar body
with 24 chromosomes. If such an ovum would be fertilized with a normal spermatozoon (22+X) the child might have the
following syndrome:
Klinefelter's syndrome
Trisomy X
@Turner's syndrome
Down's syndrome
Edwards' syndrome
#
34
A 28 year old woman consulted a doctor about sterility. Examination revealed underdeveloped ovaries and uterus,
irregular menstrual cycle. Study of sex chromatin revealed 2 Barr's bodies in most somatic cells. What chromosome
disease is the most probable in this case?
Turner's syndrome
Patau's syndrome
Edwards' syndrome
@Triplo-X syndrome
Klinefelter's syndrome
#
35
Examination of a 12 year old boy with developmental lag revealed achondroplasia: disproportional constitution with
evident shortening of upper and lower limbs as a result of growth disorder of epiphyseal cartilages of long tubal bones.
This disease is:
Congenital
Inherited, sex-linked
Inherited, recessive
@Inherited, dominant
Acquired
#
36
In course of prophylactic medical examination a 7-year-old boy was diagnosed to have daltonism. Parents are healthy,
color vision is normal. But grandfather from the mother's side has the same disorder. What is the type of inheriting of this
anomaly?
@Recessive, sex-linked
Autosomal-dominant
Autosomal-recessive
Incomplete domination
Dominant, sex-linked
#
37
A healthy woman has three sons affected by color blindness who were born after her two marriages. Children both of her
husbands are healthy. What is the most possible pattern of inheritance of this disease?
@X-linked recessive
Autosomal recessive
Y-linked
Autosomal dominant
X-linked dominant
#
38
While studing of the family tree with history of hypertrichosis (hyperhirsutism of the ear) this sign was founded only in
the men and it was inherited from father to the son. Define the type of hypertrichosis inheritance?
Autosomal-dominant
Autosomal- recessive
@Connected with Y-chromosome
Connected with Х-chromosome dominant
Connected with Х-chromosome recessive
#
39
The study of the genealogy of a family with hypertrichosis (helix excessive pilosis) has demonstrated that this symptom is
manifested in all generations only in men and is inherited by son from his father. What is the type of hypertrichosis
inheritance?
X-linked recessive chromosome
Autosome-recessive
X-linked dominant chromosome
@Y-linked chromosome
Autosome-dominant
#
40
An individual is characterized by rounded face, broad forehead, a mongolian type of eyelid fold, flattened nasal bridge,
permanently open mouth, projecting lower lip, protruding tongue, short neck, flat hands, and stubby fingers. What
diagnosis can be put to the patient?
Alkaptonuria
@Down's syndrome
Supermales
Turner's syndrome
Klinefelter's syndrome
#
41
Examination of a 7 year old child revealed the following symptoms: small height, broad roundish face, closely placed
eyes with narrow palpebral fissures, half-open mouth. Valvular defect has been also diagnosed. These clinical
presentations are most likely typical for Down's syndrome. Name the cause of such pathology:
Nondisjunction of sexual chromosomes
Partial monosomy
@Trisomy of the 21 chromosome
Trisomy of the 13 chromosome
X-chromosome trisonomy
#
42
A 3-month-old child died of pneumonia. A macroscopical investigation revealed upward slant of palpebral fissures of the
eyes, a sinking down dorsum of nose, dolichouranic, a low locating of small auricles. An autopsy revealed a defect of a
heart and the main vessels' development. A genetic study showed a chromosome 21 trisomy. What of the listed diagnoses
is the most likely?
@Down's syndrome
Patau's syndrome
Shereshevsky-Turner's syndrome (gonadal dysgenesis)
Edwards' syndrome
-
#
43
At the macroscopic exam of a 5-months baby, who died of pneumonia, were revealed: oblique incision of the eyes,
retraction of a bridge of the nose, high palate, low location of small auricles (of ear). A defect in a heart and magistral
vessels development was detected at the section. The genetic exam revealed a trisomy 21 autochromosome. What is the
most possible diagnosis?
Cherechewski-Turner syndrome
Patau’s syndrome
@Down syndrome
Edwards’ syndrome
Klinfelter’s syndrome
#
44
A 40-year-old pregnant woman underwent amniocentesis. The examination of fetus karyotype revealed 47ХУ+21. What
pathology of the fetus was found out?
Schereschevsky-Terner's disease
Patau's disease
Phenylketonuria
@Down's syndrome
Klinefelter's syndrome
#
Dystrophies.
Parenchyma dystrophies.
1
All following mechanisms underlie intracellular accumulation EXCEPT one:
The rate of production is increased
The rate of the breakdown is decreased
Genetic defect of metabolism
The absence of degrading or transporting enzymes
@Permanent chronic inflammation
#
2
During the examination of a newborn, some apparent skin differences are noted. The skin is dry, with an uneven surface
and with the presence of grey plates which can be removed layer by layer. These changes are related to which type of
dystrophy?
@Horny dystrophy
Hydropic dystrophy
Hyaline dystrophy
Fibrinoid swelling
Mucoid swelling
#
3
While examining newborn baby, there were several skin changes found: dryness, dull with rough surface. What kind of
dystrophy could cause such changes?
@Keratous
Hydropical
Hyalin-droppy
Fibrinoid swelling
Mucoid swelling
#
4
A stillborn child was found to have thickened skin resembling of the tortoise shell, underdeveloped auricles. Histological
examination of skin revealed hyperkeratosis, atrophy of the granular epidermis layer; inflammatory changes were not
present. What is the most likely diagnosis?
Xerodermia
Leukoplakia
Dermatomyositis
@Ichthyosis
Erythroplakia
#
5
A male patient had a prosthetic appliance on the lower jaw. The ventral surface of tongue revealed a dense, gray plaque
with a clear boundary. Histology revealed the thickened of the stratified squamous epithelium due granular and to basal
layers thickening, hyperkeratosis, acanthosis, lymphocyte's infiltration of connective tissue. Make a diagnosis.
@Leukoplakia
Erythroplakia
Papilloma
Cancer in sity (intra-epithelial neoplasia)
Condyloma
#
6
During the preventive examination of a worker employed in the coal resins production the areas of thickening and
keratinization of the nucous membrane in the oral cavity were found. This occured mainly on the cheeks areas, showing a
whitish color with a rough surface. They were not painful. Which pathology is this related to?
@Leukoplakia
Papillomatosis
Glossitis
Stomatitis
Calcification
#
7
A 77-year-old male with a dental prosthesis on his upper jaw is seen by his dentist because of a solid gray patch on his
tongue. A lesion has irregular contour, uneven surface, and clear borders. Microscopic investigation of its biopsy revealed
the thickening of stratified squamous epithelium, its hyperkeratosis, and acanthosis accompanied with lymphocytes and
macrophages infiltration of subjacent connective tissue. What is the most likely diagnosis?
@Leukoplakia.
Erythroplakia.
Papilloma.
Cancer in situ.
Condyloma.
#
8
A solid gray patch, irregularly shaped with uneven surface, but with clear borders was founded on a ventral surface of a
tong of a man with prosthesis of mandible. During the histological examination of the formation there were noticed the
saved structure of stratified flat epithelium, its thickening through prickle and basal cell layers, hyperkeratosis, acanthosis.
Lymphomacrophagal infiltration of underlied connective tissue was detected, too. Make a right diagnosis.
Condyloma
Erythroplakia
Papilloma
Cancer in situ
@Leucoplakia
#
9
While examining oral cavity, there was found a white lamella on mucous membrane, slightly bulging above its surface.
Histologically: nodosity of multilayer epithelia, hyperkeratosis, acanthosis. Which diagnosis from below is the most
suitable in this case?
@Leucoplaqia
Desquamative glossitis
Flat-cell cancer
Papilloma
Rhomboid glossitis
#
10
Mucous membrane of a patient's oral cavity has a greyish-white focus, the mass is dense and protrudes above the mucous
membrane. Histological examination revealed hyperkeratosis, parakeratosis and acanthosis of epithelium in this are A.
What pathological process was revealed in the mucous membrane?
@Leukoplakia
Hyalinosis
Local tumourous amyloidosis
Focal ichthyosis
Leukoderm
#
11
In an experiment, a series of immunohistochemical stains are employed to identify different cellular components. One
particular stain identifies the presence of intermediate filaments within cells. This cytokeratin stain is most likely to be
useful for which of the following diagnostic purposes?
Cytoskeletal alterations are seen with impending cell death
A neoplasm can be determined to be a carcinoma
Contractile properties of the cells can be determined
@A history of chronic alcoholism can be confirmed
The degree of metaplasia or dysplasia can be assessed
#
12
During the section of a 49-year-old man, that was hospitalized with symptoms of hepatotropic intoxication and suddenly
died, an enlarged liver of yellow-brown color, soft consistence was revealed. Drops of oil were noticed on the surface of
incision of the liver and on the scalpel. Microscopically: peripheral hepatocytes of a classic hepatic lobule are filled with
small drops that are placed in the cytoplasm and displaced the nucleus to the cell periphery. About what process in the
liver do the following changes testify?
Gangliosidelipidosis (Tay-Sachs disease)
Cerebrosidelipidosis (Gaucher’s disease)
Sphyngomyelinlipidosis (Niemann-Pick disease)
@Hepar adiposa dystrophia
Generalized gangliosidosis (Norman- Landing disease)
#
13
A 53 year old patient died with symptoms of liver insufficiency. A post-mortem examination revealed the enlarged,
flabby, yellow-brown liver. Gross examination of the liver's section showed drops of fat. Microscopically: hepatocytes on
the peripheries of the hepatic lobules contained masses of small drops within the cytoplasm. Which process most likely
took place in the liver?
@Fatty dystrophy of the liver
Glucosylceramide lipidosis (Gaucher's disease)
Sphingolopidosis (Niemann-Pick disease)
Gangliosidosis (Tay-Sachs disease)
Systemic lipoidoses
#
14
An autopsy of a 49-year man, who was hospitalized with liver intoxication and suddenly died afterwards, shows: liver is
enlarged, yellowish-brown coloured and of soft consistency. On section, there are lipid drops found. Microscopically:
peripheral hepatocytes contain many small drops, which completely fill the cytoplasm and push aside the nucleus. Name
the process.
@Liver lipid dystrophy
Cerebrosidelipidosis (Gaucher’s disease}
Sphyngomyelinlipidosis (Niemann-Pick disease)
Gangliosidelipidosis (Tay-Sachs disease)
Generalized gangliosidosis (Norman-Landing disease)
#
15
A 38-year-old patient, suffering from chronic alcoholism and cirrhosis of the liver, developed profuse bleeding due to
varicose veins of the esophagus which resulted in death. During the autopsy a liver was noted to be diminished in size
with micronodular tuberosity. The organ was dense and rather yellow in color. A histological evaluation of the cryostat
specimens of the liver, stained with hematoxylin and eosin, revealed hepatocytes with large, optically empty vacuoles.
These vacuoles were black when stained with osmium acid. These optically empty vacuoles hepatocytes indicate:
@Fatty dystrophy •
Inclusions of hyaline.
Alcoholic hyaline (Mallory bodies)
Vacuolar dystrophy.
Carbohydrates dystrophy.
#
16
A patient died from pulmonary-cardiac insufficiency. During the dissection a significantly enlarged anemic liver, with
yellow doughy consistencies was found. A liver specimen stained with hematoxylin and eosin exposed various sizes of
vacuoles in the cytoplasm of the hepatocytes. Which one of the following dystrophies occurred?
@Parenchymal fatty
Parenchymal carbohydrate
Hyaline
Mesenchymal fatty
Hydropic
#
17
A 44-year-old woman died from chronic alcoholic intoxication. During the autopsy a significamtly enlarged liver of
doughy consistency and rather yellowish color was found. Microscopically, after staining with hematoxylin and eosin,
cytoplasm of the hepatocytes contained optically empty vacuoles. Which type of dystrophy has taken place?
@Parenchymal fatty dystrophy
Carbohydrate parenchymal dystrophy
Hyaline dystrophy
Mesenchymal fatty dystrophy
Hydropic dystrophy
#
18
An examination of a woman having diabetes mellitus showed enlarged liver. Blood analysis showed increased level of
triglycerides, lipoproteins. What kind of liver dystrophy is this?
Amyloidosis
Hyalin-droppy dystrophy
@Lipid parenchymal dystrophy
Hyalinosis
Mucoid swelling
#
19
An autopsy of 45 year old woman, who died of chronic alcohol intoxication, has shown grossly enlarged liver of
yellowish colour and gummy consistency. Microscopically: haematoxylin-eosin staining showed various shaped,
translucent vacuoles. Name the type of dystrophy.
@Lipid parenchymal
Carbohydrate parenchymal
Hyalin-droppy
Lipid mesenchymal
Hydropical
#
20
A 36-year-old female develops liver failure followed with lethal outcome. Autopsy has shown an enlarged liver of
yellow-brown color and soft consistence. Drops of fat are noticed on the liver cut surface and on the scalpel.
Microscopically: hepatocytes at peripheral zone of a liver lobules contain small drops that fill cytoplasm and push the
nucleus to the periphery. What process in the liver do the following changes testify to?
@Fatty degeneration of liver.
Cerebrosidelipidosis (Gaucher's disease).
Sphingomyelinlipidosis (Niemann — Pick disease).
Gangliosidelipidosis (Tay —Sachs disease).
Generalized gangliosidosis (Norman—Landing disease).
#
21
A 48-year-old man has a history of chronic alcoholism. He is able to perform his work at his job. He has had no major
illnesses. On physical examination, there are no significant findings. Which of the following microscopic findings in his
liver is most likely to be present:
Cholestasis
@Fatty change
Hemochromatosis
Hypertrophy of smooth endoplasmic reticulum
Coagulative necrosis
#
22
A 55-year-old male alcoholic presents with symptoms of liver disease and is found to have mildly elevated liver enzymes.
A liver biopsy examined with a routine hematoxylin and eosin (H&E) stain reveals abnormal clear spaces in the
cytoplasm of most of the hepatocytes. Which of the following materials is most likely forming these cytoplasmic spaces?
Calcium
Cholesterol
Hemosiderin
Lipofuscin
@Triglyceride
#
23
A 45-year old male is found to have a severe intoxication. A diagnosis of sepsis is made. Several days later he dies. At
autopsy, his myocardium grossly had a 'tiger heart' pattern. Microscopically, lipids were detected in the cytoplasm of
cardiac myocytes. What morphogenetic mechanism prevails in the development of this dystrophy?
@Decomposition
Infiltration.
Transformation
Abnormal synthesis.
Colliquation.
#
24
A man died of chronic cardiac failure. Autopsy revealed a tiger heart. There is yellowish stripy endocardium,
myocardium is dull and of clay-yellow colour. What process caused this pathology?
@Lipid parenchymal dystrophy
Carbohydrate dystrophy
Hyalin-droppy
Lipid mesenchymal dystrophy
Amyloidosis
#
25
A 42-year-old female became ill with diphtheria and died from acute cardiac insufficiency. During dissection it was noted
that the heart cavities were extended and that the muscle of the heart was dim, motley and on a cut surface had yellow
areas. Which process was exposed in the myocardial cells?
@Fatty dystrophy
Carbohydrate dystrophy
Ballooning dystrophy
Hyaline dystrophy
Hydropic dystrophy
#
26
Autopsy of a menopausal woman with a ling history of a chronic ischemic heart disease revealed soft and enlarged heart.
Its chambers were extended; the myocardium sectional view was lack-luster with grey- yellowish coloring. An
endocardium presented with yellow-white banding, most evident in papillary muscles. What is the most likely
pathological process in woman's heart?
@Fatty degeneration of myocardium.
Fatty heart
Dilated cardiomyopathy.
Myomalation.
Cardiosclerosis.
#
27
Autopsy of a man who died from chronic cardiacvascular collapse revealed "tiger heart". Sidewards of endocardium a
yellowish-white banding can be seen; myocardium is dull, dark-yellow. What process caused this pathology?
@Fatty parenchymatous degeneration
Carbohydrate degeneration
Fatty vascular-stromal degeneration
Amyloidosis
Hyaline degeneration
#
28
A patient with leukemia died from severe chronic anemia. An autopsy revealed an enlarged heart, with flabby
myocardium. It had a dim pale-grey color, yellow spots and bars. Which pathological process was found in the heart at
post-mortem?
@Parenchymal fatty dystrophy
Vacuolar dystrophy.
Hydropic dystrophy.
Mesenchymal fatty dystrophy.
Mixed dystrophy.
#
29
A patient died from chronic cardiovascular insufficiency. At the post-mortem a 'tabby cat' heart was found. From the side
of the endocardium, a yellow-white striped pattern was noticeable. The myocardium was a dim with gray-yellow color.
Which process is most likely diagnosed?
@Fatty parenchymal dystrophy.
Carbohydrate dystrophy
Hydropic dystrophy.
Fatty mesenchymal dystrophy.
Amyloidosis.
#
30
An autopsy of 62-year old man, who died of increasing cardiac failure, showed enlarged heart, dilated cameras. Section of
myocardium shows dull, clay-yellow tissue. Endocard is stripy, especially on papillary muscles. What pathological
process is the most suitable in this case?
@Myocardial lipid dystrophy
Cardial adiposis
Dilatational cardiomyopathy
Myomalation
Cardiosclerosis
#
31
A 66-year-old male died from cardiac insufficiency. During the dissection an increase volume heart was found.
Observation of the heart revealed a flabby consistency with stretched chambers. The myocardium section had a dim, clay-
yellowish color. From the side of the endocardium a yellow-white striped pattern was present, which was especially noted
in the papillary muscles. Which pathological process is the most credible?
@Fatty dystrophy of the myocardium
Obesity of the heart
Dilatation cardiomyopathy
Myomalacia
Cardiosclerosis
#
32
A man died due to cardiac insufficiency. At autopsy revealed the heart increased volume and flabby. A myocardium was
a clay-yellow color and dim. From the side of the endocardium a yellow-white striped pattern was visible ('tabby cat').
Under the microscope the groups of myocardial cells lost their normal structure, their cytoplasm contained shallow drops
which were black when stained with Sudan-IV. Which one of the following is the correct diagnosis?
@Fatty dystrophy of myocardium
Cardiosclerosis
Rheumatic myocarditis
Obesity of the heart
Myomalacia
#
33
During the section of a 62-year-old man, who died at the background of facts of progressive heart insufficiency, an
enlarged heart of soft consistence was revealed. Its chambers are dilated; on incision the myocardium is dim, of clayish-
yellow color. Yellow-white strips can be seen from the endocardium side that is especially manifested in papillary
muscles. What pathological process can be diagnosed?
Fat heart
@Cor adiposa dystrophia
Dilated cardiomyopathy
Myomalation
Cardiosclerosis
#
34
During an autopsy a parenchymal fatty dystrophy of the myocardium was diagnosed. What is the common or descriptive
name of the heart due to this dystrophy?
@Tabby cat' heart ('Tiger's' heart)
Bovine heart
'Hairy' heart
Solder plaque (bony heart)
Cor pulmonale
#
35
A seven-year-old child presented with diphtheria of the pharynx. He subsequently died from acute cardiac insufficiency.
Post-mortem examination of the heart revealed that the cavities of the heart were extended horizontally. Muscles of the
heart were dim and flabby. Gross section showed motley appearance, with yellow areas. Microscopically in the cytoplasm
of some myocardial cells small vacuoles were determined. The frozen sections showed vacuoles within cells stained with
sudan-III in orange color. Which type of dystrophy was found in myocardial cells?
@Fatty dystrophy
Carbohydrate dystrophy
Vacuolar dystrophy
Hyaline dystrophy
Hydropic dystrophy
#
36
A 62-year-old male who has been ill with diabetes mellitus for 15 years died from a cerebral hemorrhage. Post-mortem
revealed kidneys diminished in size with a fine-grained surface. The epithelium of the canaliculi of distal nephron's
segment was high, with a light foamy cytoplasm. The Best's carmine staining demonstrated a bright red coloring of the
cytoplasm's accumulations. These changes in the epithelium resulted from the accumulation of:
@ Glycogen
Lipids
Hyaline
Proteins
Amyloid
#
37
Mother of 5-year old boy complaints that his ears and tip of nose became darker, urine is to darken upon standing. Color
of feces is as usual. The cause of such state can be:
@alkaptonuria
phenylketonuria
branched chain aminoaciduria (maple syrup disease)
Hartnup’s disease
podagra (gout)
#
38
In a year old child urine is to darken upon standing. In the blood and urine homohentisine acid is found. Metabolism of
what substance is disordered?
@Tyrosine
Purine nucleotides
Leicine
Tryptophan
Melanin
#
39
A 2-years-old boy was admitted to the hospital with malabsorption syndrome. Deficiencies of disaccharidases and
dipeptidases have been found in this patient. What from the following types of epithelium is providing deficiency of
enzymes?
@Simple columnar epithelium
Simple cuboidal epithelium
Simple squamous epithelium
Pseudostratified epithelium
Simple columnar ciliated epithelium
#
40
A 6-month-old boy is being evaluated for failure to thrive, along with persistent vomiting, seizures, and a low-grade
fever. Physical examination finds a protuberant abdomen due to enlargement of both the liver and spleen, along with a
“cherry-red spot” on his retina and diffuses enlarged lymph nodes. A bone marrow biopsy reveals an abnormal diffuse
proliferation of foamy macrophages filled with lipid (“foam cells”). Electron microscopy reveals cytoplasmic bodies that
resemble concentric lamellated myelin figures within the foamy macrophages. Rare parallel pallisading lamellas (“zebra
bodies”) are also seen in the cytoplasm of these cells with electron microscopy. Which of the following substances is
most likely to be found at abnormal high levels within these foamy macrophages?
Ceramide trihexose
Glucocerebroside
Heparan sulfate
@Sphingomyelin
Sulfatide
#
41
A 6 year old child was delivered to a hospital. Examination revealed that the child couldn't fix his eyes, didn't keep his
eyes on toys, eye ground had the cherry-red spot sign. Laboratory analyses showed that brain, liver and spleen had high
rate of ganglioside glycometid E. What congenital disease is the child ill with?
@Tay-Sachs disease
Turner's syndrome
Wilson's syndrome
Niemann-Pick disease
MacArdle disease
#
42
Examination of cell culture got from a patient with lysosomal pathology revealed accumulation of great quantity of lipids
in the lysosomes. What of the following diseases is this disturbance typical for?
@Tay-Sachs disease
Galactosemia
Wilson disease
Phenylketonuria
Gout
#
43
A 10-month-old baby is being evaluated for visual problems and motor incoordination. Examination of the child’s fundus
reveals a bright “cherry red spot” at the macula. Talking to the family of this visually impaired 10-month-old infant, you
find that they are Jewish and their family is from the eastern portion of Europe (Ashkenazi Jews). Based on this specific
family history, which one of the following enzymes is most likely to be deficient in this infant?
Aryl sulfatase
β-glucocerebrosidase
@Hexosaminidase A
Hexosaminidase B
Sphingomyelinase
#
44
A 6-year-old girl is being evaluated for recurrent episodes of light-headedness and sweating due to hypoglycemia. These
symptoms are not improved by subcutaneous injection of epinephrine. Physical examination reveals an enlarged liver and
a single subcutaneous xanthoma. An abdominal CT scan reveals enlargement of the liver along with bilateral enlargement
of the kidneys. Laboratory examination reveals increased serum uric acid and cholesterol with decreased serum glucose
levels. Following oral administration of fructose, there is no increased in blood glucose levels. A liver biopsy specimen
reveals increased amounts of glycogen in hepatocytes, which also have decreased levels of glucosese-6-phosphatase.
Which of the following is the most likely diagnosis?
Andersen’s syndrome (type IV glycogen storage disease)
Cori’s disease (type III glycogen storage disease)
McArdle’s syndrome (type V glycogen storage disease)
Pompe’s disease (type II glycogen storage disease)
@von Gierke’s disease (type I glycogen storage disease)
#
45
A 9-year-old boy is being evaluated for deafness. Physical examination reveals a child with short stature, coarse facial
features (low, flat nose; thick lips; widely spaced teeth; facial fullness), a large tongue, and clear corneas. Laboratory
examination reveals increased urinary levels of heparan sulfate and dermatan sulfate. Metachromatic granules (Reilly
bodies) are found in leukocytes from a bone marrow biopsy. These leukocytes are also found to be deficient in
iduronosulfate sulfatase. Which of the following is the most likely diagnosis?
@Hunter’s disease
Hurler’s disease
I cell disease
Metachromatic leukodystrophy
Wolman’s disease
#
46
A child's blood presents high content of galactose, glucose concentration is low. There are such presentations as cataract,
mental deficiency, adipose degeneration of liver. What disease is it?
Steroid diabetes
Fructosemia
@Galactosemia
Diabetes mellitus
Lactosemia
#
47
In newborn the hypertonia of muscles and seizures are observed. On the forth day of life the specific ‘mousy’ odor was
felt from infant. After interaction of urine with FeCl3 the green-blue color appears. What disease is it possible to think
about?
@Phenylketonuria
Branched chain aminoaciduria (maple syrup disease)
Tyrosinosis
Hartnup’s disease
Familial cystinuria
#
48
In newborn the hypertonia of muscles and seizures are observed. On the forth day of life the specific ‘mousy’odor was
felt from infant. After interaction of urine with FeCl3 the green-blue color appears. The cause of such state can be
absence of
@phenylalaninhydroxylase
xantinoxydase
oxydase of homogentisic acid
phosphorilase
glucose-6-phosphatase
#
49
In newborn the hypertonia of muscles and seizures are observed. On the forth day of life the specific ‘mousy’ odor was
felt from infant. After interaction of urine with FeCl3 the green-blue color appears. What additional investigations are the
most important?
@Phenylalanine in blood serum and urine
Adrenalin, noradrenalin in blood serum and urine
Thyroxin, TSH in blood serum
Sodium chloride in sweat
Glucose in blood serum and urine
#
50
A 1,5-year-old child presents with both mental and physical lag, decolorizing of skin and hair, decrease in catecholamine
concentration in blood. When a few drops of 5% solution of trichloroacetic iron had been added to the child’s urine it
turned olive green. Such alteration are typical for the following pathology of the amino acid metabolism:
Tyrosinosis
Alkaptonuria
@Phenylketonuria
Albinism
Xanthinuria
#
51
Examination of a 6 days old infant revealed phenyl pyruvate and phenyl acetate excess in his urine. What aminoacid
metabolism is disturbed in the child's organism?
Arginine
@Phenylalanine
Methionine
Tryptophan
Histidine
#
Mesenchyma dystrophies.
#
1
On autopsy of the 58-year-old man it is revealed: mitral valve is deformed, thickened, not totally closed. Microscopically:
centers of collagen fibers are eosinophilic, have positive fibrin reaction. The most likely it is:
Hyalinosis
Mucoid swelling
@Fibrinoid swelling
Amyloidosis
Fibrinoid inflammation
#
2
The examination of skin biopsy of a patient with hemorrhage vasculitis revealed the next: the vessel wall was thickened,
homogenous, stained in yellow colour by means of picrofuchsin, PAS-positive. What pathologic process has developed in
the vessels walls?
Mucoid swelling
@Fibrinoid swelling
Amyloidosis
Hyalinosis
Lipidosis
#
3
The dissection of a 49 year old male reveals a deformed mitral valve, which is thickened and does not completely close.
Microscopically the foci of the collagen fibers are eosinophilic and give a positive reaction on a fibrin test? The most
credible explanation is:
@Fibrinoid swelling
Fibrinoid inflammation
Mucoid swelling
Hyalinosis
Amyloidosis
#
4
A skin biopsy of a patient with allergic vasculitis was submitted for examination. It is discovered that the vessel walls
were thickened and homogeneous. Picrofuxin stained a tissues a yellow color. They were Shiff-positive. Which
pathological process developed in the walls of the vessels?
@Fibrinoid swelling
Amyloidosis
Mucoid swelling
Hyalinosis
Lipidosis
#
5
Biopsy examination of skin of patient with allergic vasculitits showed: vessel side is thickened, homogenous, while
staining with picrofuxin becomes yellow in colour. Schiff-iodine reaction is positive. Name the pathological process.
Lipidosis
Amyloidosis
Mucoid swelling
Hyalinosis
@Fibrinoid swelling
#
6
A 66 years old patient, had peritonitis 10 years before the death. On the section, the capsules of the liver and the spleen
from place to place are much thickened, consolidated, and translucent. What is the most possible diagnosis?
Mucoid swellyng
Necrosis
@Hyalinosis
Fibrinoid swellyng
Amyloidosis
#
7
A 56 year old patient with a six year history of peritonitis has died. During dissection the capsule of the liver and the
spleen was markedly thickened in places and was noted as being dense and semi-lucent. The most credible explanation
for this is:
@Hyalinosis
Necrosis
Mucoid swelling
Fibrinoid swelling
Amyloidosis
#
8
During the post-mortem performed on a 72 year old man there are noted some diminished areas of the spleen with a
pinkish color. Microscopic examination revealed that the follicles are diminished in volume and the walls of the arterioles
and trabeculas are thickened as well as containing homogeneous eosinophilic, PAS-positive masses. Staining with
picrofuksin dye reveals the masses to be a red color. These changes indicate the presence of:
@Hyalinosis
Amyloidosis
Mucoid swelling
Fibrinoid swelling
Sclerosis
#
9
Macroscopic examination of a stomach delivered from surgery, revealed a round lesion 1.5cm in diameter which
extended by the muscle layer at the antral zone of a small curvature. A semilucent dense area on the bottom of the defect
was also determined. It resembled hyaline cartilage. Which process developed in the bottom of the stomach lesion?
@Localized hyalinosis
Amyloidosis
Mucoid swelling
Fibrinoid changes
Generalized hyalinosis
#
10
At autopsy a 76-year-old male, with a history of peritonitis 10 years ago, is found to have thickened and dense both liver
and spleen capsules. They were translucent on a sectional view. What is the most likely pathology of the described organs
capsules?
@Hyalinosis.
Necrosis.
Mucoid swelling.
Fibrinoid swelling.
Amyloidosis.
#
11
Macroscopically: liver is enlarged, thickened, its tissue greyish-yellow colour with greasy shine. Name the pathological
process.
@Amyloidosis
Hyalinosis
Lipid dystrophy
Mucoid swelling
Haemochromatosis
#
12
An 87-year-old man develops worsening heart failure. Workup reveals decreased left ventricular filling due to decreased
compliance of the left ventricle. Two months later the patients dies, and postmortem sections reveal deposits of
eosinophilic, Congo red-positive material in the interstitium of his heart. When viewed under polarized light, this material
displays an apple-green birefringence. Which of the following is the most likely diagnosis?
@Amyloidosis
Glycogenosis
Hemochromatosis
Sarcoidosis
Senile atrophy
#
13
A 55-year-old female, with a long history of rheumatoid arthritis, develops renal failure and dies. An autopsy revealed an
enlarged solid spleen. On the sectional view, its tissue had brown-reddish coloring with enlarged follicles that look like
translucent grayish-white grains. What is the most likely pathological process?
@Sago spleen.
Frosted spleen.
Lardaceous spleen.
Spleen hyalinosis.
Porphyry spleen.
#
14
The dissection of a 48 year old patient who suffered with rheumatoid arthritis reveals an enlarged, dense spleen. A
spleen's section demonstrates its brown-reddish color with enlarged follicles which have the appearance of semi-lucent,
grayish-white corns. What is the name of these lesions in the spleen?
@Sago-like spleen.
Glazed spleen.
Sebaceous spleen.
Hyalinosis of spleen.
Porphyry spleen.
#
15
On autopsy of the 40-year-old woman suffering from rheumatic arthritis, the enlarged solid spleen was revealed. On
section its tissue is of the mahogany color with enlarged follicles, which look like semi-transparent grayish-whitish
grains. What pathological process is the most likely?
Waxy spleen
Glaze spleen
Hyaline spleen
Porphyric spleen
@Sago spleen
#
16
During the section of the 40-year-old woman, who was ill with rheumatoid arthritis, an enlarged solid spleen was
revealed. Its tissue is of brown-red color with enlarged follicles that look like translucent grayish-white corn. On what
pathological process does the description indicate?
@Sago spleen
Shugar glazed spleen
Diffuse waxy spleen
Spleen hyalinosis
Porphyry spleen
#
17
During dissection of a 65 year old patient, who suffered from a fibrous-cavernous tuberculosis, an enlarged, dense spleen
was found. Spleen section grossly had brown-pinkish color, smooth, waxy-like surface. Which pathological process listed
below is the most credible?
@Sebaceous spleen.
Glazed spleen.
Porphyry spleen.
Sago spleen.
Cyanotic induration
#
18
A 66-year-old female, with a long history of post-traumatic osteomyelitis, is admitted to the hospital for treatment of
nephrotic syndrome. On the night pf admission she suddenly dies. Autopsy revealed dense, white kidneys with scars in
the cortical layer; they had a sebaceous glow on the cut surface. What is the most likely kidneys pathology?
@Secondary amyloidosis.
Primary amyloidosis.
Idiopathic amyloidosis.
Chronic glomerulonephritis.
Chronic pyelonephritis.
#
19
The post-mortem of a patient revealed feature of chronic kidney insufficiency. Grossly, kidneys were enlarged, dense, and
wax-like, with foci of irregular depressed scars on their surface. Microscopically, the mesangeal areas were expanded and
the glomerular capillaries obstructed by Congo red stain-positive amorphous acellular material. In some sections the
deposits took on nodular appearance. Which of the following diagnoses is most reliable?
@Amyloidosis of the kidneys (Amyloid nephropathy)
Acute glomerulonephritis
Chronic glomerulonephritis
Subacute glomerulonephritis
Lipoid nephrosis
#
20
An autopsy of a 45-year-old female revealed the kidneys were dense, yellow in color and appears to have a greasy
brilliance. Which pathological process is most likely?
@Amyloidosis
Hyalinosis
Fatty dystrophy
Mucoid swelling
Hemochromatosis
#
21
A 56 year bid female has been ill with chronic fibrocavernous tuberculosis of the lungs for the past 20 years. She entered
the nephrology department with a uremia syndrome. A test for the presence of amyloid in kidneys was positive. Which
form of amyloid is indicated in this case?
@Secondary
Primary
Localized
Familial congenital
Senile
#
22
An autopsy of a man, who suffered from bronchiectasia and died of chronic renal failure revealed enlarged kidneys of
thick consistency, with thickened cortex of white colour and greasy shine. Name the disease.
@Secondary amyloidosis
Glomerulonephritis
Chronic pyelonephritis
Necrotic nephrosis
Membranous nephropathy
#
23
A 40-year-old man during 10 years after the fracture of tibia had a chronic osteomyelitis. The patient had also nephritic
syndrome 3 years before the death. Uremia caused his death. On section: kidneys are consolidated, white, with scars in
cortex; they have waxy glow, on section. Determine the pathology that developed.
Idiopathic amyloidosis
Primary amyloidosis
@Secondary amyloidosis
Chronic glomerulonephritis
Chronic pyelonephritis
#
24
A man suffered from chronic mandibular osteomielitis. Recent blood analysis: hypoproteinaemia, dysproteinaemia.
Recent urine analysis: proteinuria, protein cylinders. Cause of death - chronic renal failure. What pathological process
was revealed on section?
@Amyloidosis
Chronic glomerulonephritis
Hydronephrosis
Chronic pyelonephritis
Interstitial nephritis
#
25
A 42 y.o. man who had been suffering from chronic granulomatous periodontitis and chronic purulent osteomyelitis of his
lower jaw for 8 years died under conditions of acute renal insufficiency. What complication of purulent osteomyelitis was
developed in kidneys?
Adipose degeneration
Atrophy
Necrosis of epithelium of convoluted tubules
@Amyloidosis
Hyalinosis
#
26
A man, who suffered from chronic mandibular osteomielitis for a long time, died of renal failure. Section revealed big
greasy kidneys. Name the process.
@Renal amyloidosis
Atherosclerotic nephrosclerosis
Glomerulonephritis
Necrotic nephrosis
Primarily shrunken kidney
#
27
A 52 year old male died from a heart attack. At the time of dissection a symmetric type of severe obesity discovered. The
rupture of the right ventriculum wall resulted in hemopericardium. Under epicardium an excessive fat tissue formation
discovered. A microscopy of the sample showed the excecive growth of fatty tissue accompanied with atrophy of
myocardial fibers. Which pathological process is most likely responsible for the patient's death?
@Simple obesity of the heart.
Fatty dystrophy of myocardium.
Ischemic heart disease.
Hypertension
Acute myocardium infarct
#
28
During a section of a 45-year-old man, who died of a sudden heart failure, were determined: symmetrical type of obesity
III stage, rupture of the right ventricle with haemopericardium; an excessive amount of fat under the epicardium.
Microscopically: fatty tissue penetrates the myocardium with the atrophy of muscle fibers. What pathological process is
the most possible?
@Simple fatty heart
Dystrophia adiposa cordis
Cardiac ischemia
Hypertonic disease
Acute myocardial infarction
#
29
A morbidly obese 51-year-old woman dies from complications of heart disease. At autopsy, her heart weighs 600 gm
(normal up to 300 gm) with all the chambers enlarged. Microscopically, there is increased fibrous connective tissue seen
in the interstitium between myocardial fibers. The fibers are increased in size. Beneath the epicardium can be seen
adipocytes interdigitating with the myocardial fibers. Which of the following terms best describes the presence of the
adipocytes in her myocardium?
Steatosis
Lipid degeneration
@Fatty infiltration
Cholesterolosis
Xanthomatosis
#
30
A post mortem performed on a 50 year old male who died of a heart attack indicated a symmetric type of obesity of the
III degree with rupture of the walls of the right ventricle and hemopericardium. Under the epicardium surplus deposits of
fat were found. Microscopically, fatty tissue from the epicardium was dispersed in the myocardium with an atrophy of the
muscle fibers. Which process listed below is the most reliable?
@Obesity the heart.
Fatty dystrophy of myocardium.
Acute infarct of myocardium.
Ischemic heart disease.
Hypertension.
#
31
A 55-year-old man, with a long history of a symmetrical type of severe obesity, developed acute heart insufficiency
followed with lethal outcome. An autopsy revealed right ventricle wall burst with hemopericardium and excessive amount
of fatty tissue under epicardium. Microscopically: adipose tissue from epicardium penetrates myocardium with muscle
fibers atrophy. Name the pathological process?
@Simple fatty heart.
Fatty degeneration of myocardium.
Ischemic disease.
Essential hypertension.
Acute myocardial infarction.
#
32
In 45-year-old patient died from sudden cardiac death the symmetrical type of adipose heart of third degree; the rupture of
right ventricle’s wall with hemopericardium and redundant accumulation of fat under epicardium were found out in
autopsy. Microscopically: the adipose tissue grows from epicardium into myocardium with atrophy of fibers of muscle.
What process is more probable?
Acute myocardial infarction
Ischemic heart disease
Fatty degeneration of myocardium
@Adipose heart
Hypertensive disease
#
33
All following statement is associated with cholesterol accumulation EXCEPT one:
Increase cholesterol blood level may leads to atherosclerosis
Cholesterol may accumulates within cells or extracellular space
Xantomas is the reflection of cholesterol-feeling cell aggregation within dermis
Crystals of cholesterol may encountered in necrotic debris
@With atherosclerosis cholesterol accumulates within endothelial cells of intima
#
34
Autopsy of a man ill with severe hypothyroidism revealed that connective tissue, organ stroma, adipose and cartilaginous
tissues were swollen, semitransparent, mucus-lik E. Microscopic examination of tissues revealed stellate cells having
processes with mucus between them. What type of dystrophy is it?
Parenchymatous adipose
@Stromal-vascular carbohydrate
Parenchymatous proteinaceous
Stromal-vascular adipose
Stromal-vascular proteinaceous
#
35
A child has physical and mental retardation, serious abnormalities in connective tissue of internal organs; urine contains
keratan sulfates. This is caused by metabolic disorder of the following substance:
Fibronectin
Hyaluronic acid
@Glycosaminoglycan
Elastin
Collagen
#
36
Decreased blood supply to the organs causes hypoxia that activates fibroblasts function. Volume of what elements is
increased in this case?
Lymphatic vessels
Parenchymatous elements of the organ
Nerve elements
Vessels of microcircular stream
@Intercellular substance
#
Mixed dystrophies.
#
1
Autopsy of a man who had been working as a miner for many years and died from cardiopulmonary decompensation
revealed that his lungs were airless, sclerosed, their apexex had emphysematous changes, the lung surface was greyish-
black, and the incised lung tissue was coal-black. What disease caused death?
@Anthracosis
Silicosis
Aluminosis
Asbestosis
Talcosis
#
2
The autopsy of the man's body, that was working for a long time as a miner and died beacuse of chronic pulmonary-
cardiac insufficiency, revealed that lungs were not in their ful volume, significantly thickened, sclerotic, the apexis were
emphysematously changed, the surface was gray-black in colour, on section the lung tissue was aspid-black in colour.
What disease caused the death?
Asbestosis
@ Anthracosis
Aluminosis
Silicosis
Talcosis
#
3
A man had worked in a coal mine for over 20 years. After his death autopsy revealed that his lungs were dense, grayish-
black and had large areas of neogenic connective tissue containing a lot of microphages with black pigment in the
cytoplasm. What is the most likely diagnosis?
@Anthracosis
Anthracosilicosis
Silicoanthracosis
Talcosis
Siderosis
#
4
While performing a pneumonectomy for a squamous cell carcinoma diagnosed in the right lung of a 54-year-old man, the
thoracic surgeon notes that the hilar lymph nodes are small, 0.5 to 1.0 cm in size, and jet black in color throughout. You
would describe these lymph nodes as having:
@Anthracotic pigmentation
Lipochrome deposition
Accumulation of melanin
Hemosiderosis
Metastatic carcinoma
#
5
A54-year-old man with a chronic cough has a squamous cell carcinoma diagnosed in his right lung. While performing a
pneumonectomy, the thoracic surgeon notes that the hilar lymph nodes are small, 0.5 to 1.0 cm in size, and jet black in
color throughout. Which of the following is the most likely cause for this appearance to the hilar nodes?
@Anthracotic pigmentation
Lipochrome deposition
Accumulation of melanin
Hemosiderosis
Metastatic carcinoma
#
6
An autopsy is performed on a 64-year-old man who died of congestive heart failure. Sections of the liver reveal yellow-
brown granules in the cytoplasm of most of the hepatocytes. Which of the following stains would be most useful to
demonstrate with positive staining that these yellow-brown cytoplasmic granules are in fact composed of hemosiderin
(iron)?
Oil red O stain
Periodic acid-Schiff stain
@Prussian blue stain
Sudan black B stain
Trichrome stain
#
7
Sputum analysis of a patient with mitral cardiac disease showed cells with brown pigment. Pearl's reaction is positive.
What pigment is it?
@Haemosiderin
Haematoidin
Melanin
Porphyrin
Bilirubin
#
8
A patient with a mitral valvular disease has a cough with a rusty phlegm. What pigment has determined the color?
Haemoglobin
Melanin
@Haemosiderin
Haemomelanin
Ferum sulphite
#
9
A patient with mitral cardiac disease suddenly occurred cough. Sputum has rusty colour. What pigment caused this?
@Haemosiderin
Melanin
Haemoglobin
Haemomelanin
Iron sulphide
#
10
The cells with a brown pigment were determined in patient’s phlegm, who had a mitral valve disease. The Perls’ test is
positive. What pigment was mentioned?
Bilirubin
Haematoidin
Melanin
Porphyrin
@Haemosiderin
#
11
While in a home improvement center warehouse buying paint, a 35-year-old man hears "Look out below!" and is then
struck on the leg by a falling pallet rack, which strikes him on his left leg in the region of his thigh. The skin is not
broken. The yellow-brown color of the 5 x 7-cm bruise to his thigh a couple of weeks after this injury are due to the
accumulation of:
Lipofuscin
Bilirubin
Melanin
@Hemosiderin
Glycogen
#
12
A 12-year-old boy with hemophilia A has decreased levels of coagulation factor VIII activity. He has pain about his
knees. Over time, there is an increase in size of the knee joints with deformity. Which of the following materials is most
likely to be seen within the joint space following episodes of pain:
Lipofuscin
Russell bodies
Neutrophils
Cholesterol crystals
@Hemosiderin
#
13
A 22-year-old woman has a congenital anemia. She has required multiple transfusions of red blood cells for many years.
She now has no significant findings on physical examination. Laboratory studies now show a serum AST of 74 U/L and
ALT 75 U/L with albumin 3.6 g/dL. Which of the following microscopic findings would most likely appear in a liver
biopsy?
Steatosis in hepatocytes
Bilirubin in canaliculi
@Hemosiderin in hepatocytes
Glycogen in hepatocytes
Amyloid in portal triads
#
14
A 36-year-old man presents because his skin has been darkening recently. You notice that his skin has dark, somewhat
bronze color. Work-up reveals signs of diabetes mellitus. His serum iron is found to be 1150mg/dL, and his transferring
saturation is 98%. A liver biopsy is performed and reveals extensive deposits of hemosiderin in the hepatocytes and
Kupffer cells. Which of the following basic abnormalities is the most likely cause of this constellation of clinical
findings?
Defective excretion of copper into the bile
Defective synthesis of α1 antitrypsin
Defective synthesis of glycogen
Excessive absorption of galactose from the small intestines
@Excessive reabsorption of iron from the small intestines
#
15
A man with insufficiency of the mitral valve complained of a cough and sputum with a brownish colouring. Which
pigment results in this color of the sputum?
@Hemosiderin
Melanin
Hemoglobin
Hemomelanin
Iron sulfate
#
16
The post-mortem of a man who presented in the hospital with a history of snakebite reveals expressed intravessels
hemolysis. During dissection it is noted that the spleen, bone marrow and lymphatic nodes had a brown colouring.
Microscopic examination showed that the cytoplasm of macrophages got a brown pigment. Which pigment accumulated
in the tissues?
@Hemosiderin
Hematoidin
Hematin
Lipofuscin
Bilirubin.
#
17
A 58-year-old male has been ill for many years with leukemia. A post-mortem exposed a brown color in the marrow,
spleen, liver, and lymphatic nodes. The Perls' histochemical reaction was conducted. It was determined that the reticular,
the endothelial cells and histiocytes of these organs contained granules of a dark blue color. Which pigment is responsible
for the colouring?
@Hemosiderin
Bilirubin
Hematoidin
Hemomelanin
Hematoporphyrin
#
18
A 56 year old patient died from chronic cardiac insufficiency as a result of rheumatic heart-disease. A post-mortem
revealed that lungs were enlarged, dense with red-brownish coloring. What is the most likely diagnosis?
@Brown induration lungs
Acute bronchitis
Honey-comb lungs
Chronic bronchitis
Chronic emphysema
#
19
A 65-year man died of chronic cardiac failure. Autopsy: brown-coloured due to rheumatic cardiac disease, enlarged and
thickened lungs. What's the name for such lung changes?
@Brown lung induration
Nutmeg lungs
Cell lungs
Chronic bronchitis
Chronic
#
20
A patient with mitral valve insufficiency presents in his sputum cells, filled with brown pigment. The Perls' reaction is
positive. Name this pigment.
@Hemosiderin
Hematoidin
Melanin
Porphyrin
Bilirubin
#
21
True statement regarding hemachromatosis include all the following EXCEPT
It characteristically causes a micronodular pigment cirrhosis
It is complicated by carcinoma of the liver in 15 to 30 percent of cases with cirrhosis
It causes diabetes with severity unrelated to the degree of pancreatic iron deposition
It can be diagnosed by liver biopsy showing raised amounts of hemosiderin
@It is associated with skin pigmentation due entirely to deposition of hemosiderin
#
22
A patient developed a cyst in the cerebrum following a hemorrhagic stroke. Two years later the patient died from
pneumonia due to a complication of influenza. During examination of the brain cyst it is noted that the walls have a rusty
tint. Perls' reaction is positive. Name the process occurring in the wall of the cyst?
@Localized hemosiderosis
General hemosiderosis
Local hemomelanosis
Infiltration of bilirubin
Primary haemochromatosis
#
23
A post-mortem of a 44-year-old patient with a history of mitral stenosis reveals dense lungs that are a brown color. Which
pathological process is most likely in the lungs?
@Hemosiderosis
Hemochromatosis
Icterus
Heme-melanosis
Lipofuscinosis
#
24
An autopsy of a man, who suffered from mitral stenosis, showed thickened lungs of brown colour. What pathological
process is described?
@Haemosiderosis
Haemochromatosis
Jaundice
Haemomelanosis
Lipofuscinosis
#
25
A 45-year-old male, with a long history of rheumatism and mitral valve insufficiency, develops a chronic cough with
rusty expectoration. What pigment colored sputum?
@Hemosiderin.
Melanin.
Hemoglobin.
Malarial pigment.
Iron sulfide.
#
26
A 67-year-old man, with a long history of mitral valve's insufficiency, has been experiencing a cough with red-brownish
coloring of sputum. Cells with brown pigmentation and positive Perls' test were detected in the sputum. Which pigment
responsible for the septum coloring?
@Hemosiderin.
Hematoidin.
Melanin.
Porphyrin.
Bilirubin.
#
27
In newborn on the 5th day of life the hypertermia and seizures are observed. Child flaccid, reflexes are lowering. Skin and
mucoses are yellow. Blood level of indirect bilirubin is 65.5 mcmol/l. What is more dangerous in this state?
@Kernicterus (nuclear jaundice)
Nephrotoxicity of bilirubin
Anemia
Accumulation of products of hemolysis
This is physiologic jaundice (not dangerous)
#
28
In newborn on the 3rd day of life skin and mucoses became slightly yellow. Blood level of direct bilirubin is 5.0 mcmol/l,
indirect bilirubin is 30.5 mcmol/l. What diagnosis is it possible to think about?
@Physiologic (neonatal) jaundice
Kernicterus (nuclear jaundice)
Toxic hepatitis
Parenchymatous jaundice
Obstructive jaundice
#
29
Blood analysis of a patient’s ill with jaundice revealed increase of total bilirubin by its indirect fraction. Urine and feces
are intensively stained. What is the most probable mechanism of these abnormalities?
Damage of liver parenchyma
Obstructed bile outflow from the liver
Disturbed conversion of urobilinogen in liver
@Increased erythrocyte hemolysis
Disturbed formation of direct bilirubin
#
30
In newborn on the 3rd day of life the skin and mucoses are yellow. Blood level of direct bilirubin is 4.5 mcmol/l, indirect
bilirubin – 65.5 mcmol/l. This state is the result of what process?
@Hemolysis of erythrocytes
Obstruction of bile ways
Fructose intolerance
Parenchymatose jaundice
Toxic hepatitis
#
31
A patient presents with icteritiousness of skin, scleras and mucous membranes. Blood plasma the total bilirubin is
increased, stercobilin is increased in feces, urobilin is increased in urine. What type of jaundice is it?
Gilbert's disease
Obturational
Cholestatic
Parenchymatous
@Haemolytic
#
32
Blood analysis of a patient’s suffering from jaundice revealed increase of total bilirubin by its indirect fraction. Urine and
feces have intense colouring. What is the most probable mechanism of these abnormalities?
Obstruction of bile outflow from the liver
Impaired transformation of urobilinogen in the liver
Damage of liver parenchyma
Impaired generation of direct bilirubin
@Increased haemolysis of erythrocytes
#
33
Enzymatic jaundices are characterized by disbalanced activity of UDP-glucuronyl transferase. What compound is
accumulated in the blood serum in case of these pathologies?
Biliverdin
@Indirect bilirubin
Verdoglobin
Direct bilirubin
Mesobilirubin
#
34
Young woman after snake’s bite has developed the picture of hemolysis. The jaundice of the skin, scleres and mucous
membranes are detected. These symptoms are caused by bilirubin accumulation. What type bilirubin is accumulated in
the blood?
@Indirect bilirubin
Direct bilirubin
Indirect and direct bilirubin
Ferritine
Haemomelanine
#
35
A 52-year-old male with a history of sub-acute septic endocarditis is examined by a physician. A doctor revealed marked
general pallor with icteric skin, sclera and visible mucous membranes. Blood test showed accumulation of indirect
reacting bilirubin (unconjugated bilirubin). The yellow staining of the skin, sclera and mucous membranes indicates
which one of the following?
@Prehepatic jaundice
Fatty dystrophy
Hemosiderosis
Hepatic jaundice
Posthepatic jaundice
#
36
All following disorders lead to the haemolytic jaundice EXCEPT one:
Sepsis
Isoimmune conflict
Talassemias
Malaria
@Arterial hypertension
#
37
A full-term normal male infant develops a slight yellow color to his skin on his second day of life. Laboratory
examination finds his serum bilirubin levels to be slightly elevated (due to increased indirect bilirubin), but the levels are
less than 6mg/dL. Additionally, serum hemoglobin levels are within normal limits. By the fifth day his bilirubin levels
have returned to normal levels and abnormal yellow skin color has disappreared. No therapy was given to this infant.
Which of the following is the most likely cause of these signs and symptoms?
Crigler-Najjar syndrome
Hemolytic disease of the newborn
Inspissated bile syndrome
Intrauterine varicella infection
@Physiologic jaundice of the newborn
#
38
Pre-hepatic (hemolytic) jaundice takes place in all the following cases besides the only one of them:
@Viral hepatitis
Sepsis
Malaria
Hemolytic disease of newborns
Relapsing fever
#
39
A patient has yellow skin colour, dark urine, dark-yellow feces. What substance will have strengthened concentration in
the blood serum?
Biliverdin
@Unconjugated bilirubin
Verdoglobin
Mesobilirubin
Conjugated bilirubin
#
40
A 12-year-old boy with sickle cell anemia presents with recurrent severe right upper quadrant colicky abdominal pain. At
the time of surgery, multiple dark black stone are found within the gallbladder. These stones are composed of which one
of the following substances?
@Bilirubin
Carbon
Cholesterol
Struvite
Urate
#
41
A 54-year-old man presents with a high fever, jaundice, and colicky abdominal pain in the right upper quadrant. The
gallbladder cannot be palpated on physical examination. Work-up reveals hemoglobin level of 15.3g/dL, unconjugated
bilirubin level of 0.9mg/dL, conjugated bilirubin level of 1.1 mg/dL, and alkaline phosphatase level of 180 IU/L. which of
the following is the most likely diagnosis?
Acute cholecystitis
Chronic cholecystitis
@Bile duct obstruction by a stone
Carcinoma of the gallbladder
Carcinoma of the head of the pancreas
#
42
According to Courvoisier’s law, a pancreatic cancer located in the head of the pancreas should be suspected in an
individual with which one of the following clinical signs?
Migratory thrombophlebitis
@Obstructive jaundice and a dilated gallbladder
Obstructive jaundice and a nonpalpable gallbladder
Steartorrhea and a nontender gallbladder
Steartorrhea and a tender gallbladder
#
43
The patient is admitted to a hospital with hyperthermia and complaints of weakness, loss of appetite, and yellow color of
skin, lighter than usually color of feces. Blood level of indirect is 28.5 mmol/l, direct bilirubin – 20.5mmol/l. What
additional investigation is the most important for diagnosis?
@Urobilinogen in urine
Tercobilin in feces
Hemoglobin in blood
Bilirubin in urine
Alkaline phosphatase in blood
#
44
A patient with apparent icteritiousness of skin, sclera and mucous membranes was admitted to the hospital. The patient's
urine was of brown ale colour, analysis revealed presence of direct bilirubin. Feces had low concentration of bile
pigments. What type of jaundice is it?
Parenchymatous
@Obturative
Haemolytic
Conjugated
Absorbtion
#
45
A patient ill with jaundice has increased content of conjugated bilirubin and bile acids in blood, no stercobilinogen in
urine. What jaundice are these symptomps typical for?
Cythemolytic
Hepatic
Hemolytic
Hepatocellular
@Obstructive
#
46
A 48-year-old patient was admitted to the hospital with complaints about weakness, irritability, sleep disturbance.
Objectively: skin and scleras are of yellow colour. In blood: increased concentration of total bilirubin with prevailing
direct bilirubin. The feces are acholi c. The urine is dark (contains bile pigments). What type of jaundice is it?
Gilbert's syndrome
Crigler-Najjar syndrome
@Mechanic
Parenchymatous
Haemolytic
#
47
On patient's gastroscopy was detected gastric ulcer of the mucous membrane with stamped edges and brown-black base,
and on gastrobiopsy acute ulcer is confirmed with brown-black pigment in its necrotic layer. What is the pigment?
@Hydrocloride hematin
Hemosiderine
Bilirubin
Ferritin
Porphirin
#
48
A fluid of coffee grounds color was endoscopically determined in a patient who had ulcerative disease of the stomach
with bleeding. What pigment does determine the color?
Haemosiderin
@Haematin chloride
Bilirubin
Ferritin
Porphyrin
#
49
A 46-year-old man has an acute stomach ulcer complicated by gastric bleeding and vomiting. Gastric masses had a brown
color and "coffee-like" appearance. Which pigment created such colouring?
@Hematin hydrochloride
Hemoglobin
Bilirubin
Hemomelanin
Iron sulfide
#
50
A patient with acute peptic ulcer and haemorrhage has vomit of brown, coffee-like colour. What pigment defines the
colour?
@Haematin chloride
Haemoglobin
Bilirubin
Haemomelanin
Iron sulphide
#
51
An endoscopy of a patient with peptic ulcer and haemorrhage showed coffee-like liquid. What pigment coloured the
gaster content?
@Haematin chloride
Haemosiderin
Bilirubin
Ferritin
Porphyrin
#
52
A 38-year-old female with chronic stomach ulcer complicated with bleeding examined endoscopically. Stomach masses
had coffee ground coloring. Which pigment responsible for this coloring?
@Hematin chloride.
Hemosiderin.
Bilirubin.
Ferritin.
Porphyrin.
#
53
An endoscopy was performed on a patient with a chronic stomach ulcer complicated with hemorrhage. This procedure
revealed a brownish (coffee-like) liquid in the stomach. Which pigment results in the color of the stomach contents?
@Hematin hydrochloride
Hemosiderin
Bilirubin
Ferritin
Porphyrin
#
54
During post-mortem of a patient arrived from a tropical country, it is discovered that there is a hemomelanosis of a liver,
spleen and elements of the reticuloendothelial stroma. These changes are characteristic for which disease?
@Malaria
Dysentery
Diabetes mellitus
Exanthematic typhus
Grippe
#
55
The post-mortem of a patient who suffered from malaria revealed jaundiced skin, sclera and mucous membranes. Also,
the spleen was enlarged and had dark-grey color. This colour of the spleen is due to the presence of:
@Hemomelanin
Hemosiderin
Lipofuscin
Melanin
Hemoporphyrin
#
56
A post-mortem was performed on a 55-year-old male, who over last eight years suffered from chronic form of malaria. At
the dissection both grey matter of the cerebrum and a spleen had the ash- grey color. Which pigment is responsible for
this discoloration?
@Hemomelanin
Lipofuscin
Hematoporphyrin
Melanin
Hemosiderin
#
57
An autopsy of 55-year old man, who suffered from chronic tropical malaria revealed grey brain substance and spleen of
greyish colour. What pigment caused this?
Haemosiderin
Lipofuscin
Haematoporphyrin
Melanin
@Haemomelanin
#
58
Patient experienced increased susceptibility of the skin to the sunlight. His urine after some time became dark-red. What
is the most likely cause of this?
Pellagra
Alkaptonuria
@Porphyria
Albinism
Hemolytic jaundice
#
59
A patient has high sunlight sensitivity of skin. During standing his urine turns dark-brown. What is the most probable
cause of this condition?
Pellagra
Albinism
Haemolytic jaundice
Alkaptonuria
@Porphyria
#
60
A patient, who suffers from congenital erythropoietic porphyria, has skin photosensitivity. The accumulation of which
compound in the skin can cause it?
Coproporphyrinogen 3
Protoporphyrin
@Uroporphyrinogen 1
Uroporphyrinogen 2
Heme
#
61
A mother consulted a doctor about her 5-year-old child who develops erythemas, vesicular rash and skin itch under the
influence of sun. Laboratory studies revealed decreased iron concentration in the blood serum, increased
uroporphyrinogen I excretion with the urine. What is the most likely inherited pathology in this child?
Methemoglobinemia
Coproporphyria
@Erythropoietic porphyria
Hepatic porphyria
Intermittent porphyria
#
62
A 29-year-old man goes on a snorkeling trip to Looe Key Marine Sanctuary and later spends time on the beach at Bahia
Honda State Park. The next day he has adarker complexion. His skin does not show warmth, erythema, or tenderness. His
skin tone fades to its original appearance within a month. Which of the following substances contributes the most to the
biochemical process leading to these skin changes?
Heme
Lipofuscin
@Tyrosine
Homogentisic acid
Glycogen
#
63
Which of the following pigments is derived from thyrosin:
Lipofuscin
@Melanin
Hemosiderin
Bilirubin
Ferritin
#
64
In the bit of the skin 1x2 cm, which was delivered for hystologic examination, brown-coloured formation 0,5 cm in
diameter is found, Microscopically formation consists of nevus cells streak- and nestlike, located in the derma with brown
pigment in the cytoplasm, which gives negative Perl's reaction. What pigment is the most probable?
@Melanin
Hemosiderin
Bilirubin
Hemomelanin
Hematoidine
#
65
The skin of a patient with bilateral affliction of adrenal glands became dark brown. The Perls’ test is negative. What
pigment determines the color of the skin?
@Melanin
Haemosiderin
Porphyrin
Lipofuscin
Biliverdin
#
66
A patient with bilateral adrenal gland failure gained dark-brown skin colour. Histochemical research of skin showed
Pearl's reaction negative. What pigment causes such skin colour?
@Melanin
Haemosiderin
Porphyrin
Lipofucsin
Biliverdin
#
67
Nappies of a newborn have dark spots that witness of formation of homogentisic acid. Metabolic imbalance of which
substance is it connected with?
Galactose
Cholesterine
Methionine
@Thyrosine
Tryptophane
#
68
A patient with a long history of tuberculosis was examined at the hospital. Physical examination revealed a grayish-brown
skin color, lowered arterial pressure, hypodynamia and a decline of the level of 17-oxycorticosteroids in the urine and
blood plasma. A problem with the metabolism of which pigment is indicated by the clinical signs of this patient?
@ Melanin
Bilirubin
Lipofuscin
Lipochrome
Hemosiderin
#
69
A fragment of skin (1x2 centimeters) delivered for histological research. Grossly a small (0,5 cm in diameter) slightly
elevated brown lesion, sharply demarcated from the surrounding normal skin, was recognized. Microscopically, a lesion
presented with nevus cells nests, rich with brown pigment. This pigment had negative Perls' reaction. Name the pigment.
@Melanin
Hematoidin
Hemosiderin
Bilirubin
Hemomelanin
#
70
A 55-year-old patient with bilateral adrenal glands lesions presented with dark brown colouring of the skin. During
histochemical examination of the skin the Perls' reaction was negative. Which pigment is responsible for this
discoloration of the skin?
@ Melanin
Hemosiderin
Porphyrin
Lipofuscin
Biliverdin
#
71
A 38-year old female is found to have bilateral adrenal adenopathy resulted in brownish coloring of her skin. Perls' test of
her skin's biopsy was negative. What pigment altered the color of the skin?
@Melanin.
Hemosiderin.
Porphyrin.
Lipofuscin.
Biliverdin.
#
72
Metabolism of which of the following pigments is disturbing due to Addison's disease:
Lipofuscin
@Melanin
Hemosiderin
Bilirubin
Ferritin
#
73
A 66-year-old male, with a history of hematogenic tuberculosis was examined. This revealed hyperpigmentation of skin
and mucous membranes, cachexia and insufficiency of the cardio-vascular system. Which disease caused such changes?
@Addison's disease
Phaeochromocytoma
Simmond's disease
Cushing disease
Grиves' disease
#
74
A 47-year-old male, with a history of secondary syphilis, has noticed foci of the skin depigmentation at his neck. Name
the pathological process of the skin.
@Leukoderma.
Metaplasia.
Leukoplakia.
Dysplasia.
Parakeratosis.
#
75
A patient with a secondary syphilis presents with skin focal depigmentation in the upper part of the back. Name the
pathological process of the skin.
Dysplasia
Metaplasia
Leucoplakia
@Leucoderma
Parakeratosis
#
76
The light brown perinuclear pigment seen on H&E staining of the cardiac muscle fibers in the grossly normal appearing
heart of an 80-year-old man at autopsy is:
Hemosiderin resulting from iron overload
@Lipochrome from "wear and tear"
Glycogen resulting from a storage disease
Cholesterol as a consequence of atherosclerosis
Calcium deposition following necrosis
#
77
An autopsy of a patient, who died of chronic sepsis, revealed atrophy of skeletal muscles, brown myocardial and liver
dystrophy. What pigment metabolism is failed?
@Lipofuscin
Lipochrome
Haemosiderin
Haemomelanin
Melanin
#
78
A man died from chronic sepsis. A post-mortem revealed an atrophy of the skeletal muscles and brown atrophy of both
myocardium and liver. Which one of the following pigments accumulated in tissues?
@Lipofuscin
Lipochrome
Hemosiderin
Hemomelanin
Melanin
#
79
A 62-year-old female with a history of stomach cancer with plural metastases died from a cachexia. Select the
characteristic changes of the heart expected to be revealed on dissection.
@Brown atrophy of myocardium.
Amyloid cardiomegaly.
Dilatation cardiomyopathy.
Hypertrophy cardiomyopathy.
"Tabby cat" ("Tiger's heart").
#
80
An 80-year-old man dies from complications of Alzheimer disease. At autopsy, his heart is small (250 gm) and dark
brown on sectioning. Microscopically, there is light brown perinuclear pigment with H&Estaining of the cardiac muscle
fibers. Which of the following substances is most likely increased in the myocardial fibers to produce this appearance of
his heart?
Hemosiderin resulting from iron overload
@Lipochrome from "wear and tear"
Glycogen resulting from astorage disease
Cholesterol as aconsequence of atherosclerosis
Calcium deposition following necrosis
#
81
A 90-year-old woman dies from pneumonia complicating Parkinson disease. At autopsy her heart is normal in size. On
microscopic examination, there is increased lipochrome (lipofuscin) seen adjacent to the nuclei within the myocardial
fibers. This microscopic finding is most likely to result from which of the following cellular mechanisms?
Nuclear pyknosis
Myocardial fiber hypertrophy
Coagulative necrosis
@Autophagocytosis
Anaerobic glycolysis
#
82
At autopsy 68-year-old male is found to have cancer of the esophagus, accompanied with cachexia. Grossly, fatty tissue
disappeared; both a liver and a heart were atrophic. Microscopy revealed brown- yellowish corn-like deposited next to
nuclei of myocardial cells. These accumulations had negative Perls' reaction. Name the material of accumulations.
@ Lipofuscin
Melanin
Hemosiderin
Ferritin
Hemomelanin
#
83
A 90-year-old woman dies from pneumonia complicating Parkinson disease. At autopsy her heart is normal in size. On
microscopic examination, there is increased lipochrome (lipofuscin) seen adjacent to the nuclei within the myocardial
fibers. This microscopic finding is most likely to result from which of the following cellular mechanisms?
Nuclear pyknosis
Myocardial fiber hypertrophy
Coagulative necrosis
@Autophagocytosis
Anaerobic glycolysis
#
84
A 70-year old man complains about pain in small joints both in upper and lower extremities. Joints are deformed, painful.
There is an increased level of urates in blood and urine. The metabolism of what substances is failed?
Calcium
@Nucleoproteides
Chromoproteins
Lipoproteins
Potassium
#
85
A patient has increased content of uric acid in his blood that is clinically presented by pain syndrome as a result of urate
deposition in the joints. What process does this acid result from?
Reutilization of purine bases
@Lysis of purine nucleotides
Heme catabolism
Proteolysis
Lysis of pyrimidine nucleotides
#
86
A 45-year-old man presents with severe pain in both knee joints. At the time of surgery, his cartilage is found to have a
dark blue-black color. Further evaluation reveals that the patient’s urine has darkened rapidly with time. Which of the
following is most likely diagnosis?
Hyperphenylalaninemia
Tyrosinemia
Tyrosinase-positive oculocutaneous albinism
@Alkaptonuria
Maple syrup urine disease
#
87
A 70-year-old man complains of the articular pains in his hands and feet. Joints are alterated, painful. An elevated level of
urates in blood and urine was determined. The metabolic disturbance of what substances is meant?
Potassium
Calcium
Chromoproteins
Lipoproteins
@Nucleoproteins
#
88
A patient has increased contents of uric acid in his blood, what is clinically manifested by pain syndrome due to
accumulation of urates in his joints. What process does the formation of this acid result from?
Purine bases re-using
Proteolysis
@Purine nucleotide decay
Heme catabolism
Pyrimidine nucleotide decay
#
89
A 65-year-old suffering from the gout man complains of the pain in the kidney's region. On ultrasonic examination the
renal calculi were revealed. As a result of what process were they formed?
Ornithine cycle
Protein catabolism
Heme decay
Restoration of cysteine
@Decay of purine nucleotides
#
90
A 66-year-old patient complained of pain in the hands and feet joints. Physical examination revealed a deformation and
painful of the joints. Laboratory tests showed the increased level of uric acid salts in the blood and urine. Which one of
following is not being fully metabolized?
@Nucleoprotein.
Calcium.
Chromoprotein.
Lipoprotein.
Potassium.
#
91
A 70-year-old man has noticed recent swelling of joints of his hands and feet. Joints are painful and stiff. Laboratory tests
revealed an increased level of urates in blood and urine. What is the most likely substance caused described pathology?
@Nucleoproteins.
Calcium.
Chromoproteids.
Lipoproteins.
Potassium.
#
92
A man 65 years old, who has suffered by gout, has been complaining on the pain in the kidney region.There are renal
calculus on the ultrasound diagnosis. What substance is the most probably cause of the calculus forming?
@Owing to uric acid
Owing to bilirubin
Owing to oxalate
Owing to phosphate
Owing to cholecterol
#
93
The patient, 55 years old, is admitted to a hospital with a joint pain syndrome. During examination the contents of uric
acid in the blood was 2.1 mmol/l, in the urine 0,066 g/l of protein and blood were found, deposits of lithates in joints and
kidney were detected. The cause of such state can be:
@podagra (gout)
phenylketonuria
branched chain aminoaciduria (maple syrup disease)
alkaptonuria
Hartnup’s disease
#
94
A 54-year-old man present with chronic knee pain. Resection of the patella reveals chalky white deposits on the surface
of intraarticular structures. Histologic sections reveal long, needle-shaped, negatively birefringent crystals. The
photomicrograph below was taken under polarized light. These finding are most consistent with a diagnosis of
Osteoarthritis
Rheumatoid arthritis
Ochronosis
@Gout
Pseudogout
#
95
Secondary gout may be seen in association with all the following EXCEPT
Polycytemia
Psoriasis
Hemolytic anemias
Myeloprolyferative disease
@Chonddrocalcinosis
#
96
An animal with aortic valve insufficiency got hypertrophy of its left heart ventricl E. Some of its parts have local
contractures. What substance accumulated in the myocardiocytes caused these contractures?
Sodium
Carbon dioxide
Lactic acid
@Calcium
Potassium
#
97
Periodic renal colics attacks are observed in a woman with primery hyperparathyroidizm. Ultrasonic examination
revealed small stones in the kidneys. What is the most plausible reason of the stones's formation?
Hyperphosphatemia
@Hypercalcemia
Hypercholesterinemia
Hyperuricemia
Hyperkalemia
#
98
All following states are associated with hypercalciemia EXCEPT one:
Tumour of parathyroid glands
@Tumour of thyroid gland
Multiple bones fractures
Chronic renal failure
Vitamin D intoxication
#
99
During the section of a died woman who had chronic shigellosis, there were found amorphous deposits of violet color in
myocardial, renal stroma and parenchyma, in the mucous membrane of the stomach and in the connective tissue of lungs.
The Koss’ reaction is positive. What pathological process had developed?
Metabolic calcification
Dystrophic calcification
@Metastatic calcification
Amyloidosis
Hyalinosis
#
100
A 58-year-old female with a long history of chronic dysentery died. At autopsy, the stroma and parenchyma of the
myocardium, kidneys, the mucous membrane of the stomach, and the connective tissue of lungs revealed violet color
amorphous masses, which had positive Koss' reaction. Which pathological process developed in the patient's organs?
@ Metastatic calcification
Dystrophy calcification
Metabolic calcification
Amyloidosis
Hyalinosis
#
101
The dissection of a patient who suffered from rheumatism and chronic rheumatic valvulitis revealed that mitral valve
leaflet was thickened with rough stony deposits. Name the pathology presented with stony appearance of the valves?
@Dystrophy calcification
Metastatic calcification
Metabolic calcification.
Fibrinoid
Amyloidosis.
#
102
A 38-year-old woman presents with intermittent pelvic pain. Physical examination reveals a 3-cm mass in the area of her
right ovary. Histologic sections from this ovarian mass reveal a papillary tumor with multiple, scattered small, round, and
laminated calcifications. Which of the following is the basic defect producing these abnormal structures?
Bacterial infection
@Dystrophic calcification
Enzymatic necrosis
Metastatic calcification
Viral infection
#
103
A 38-year-old man has a health screening examination. He has a routine chest x-ray that shows a 2 cm nodule in the right
lower lobe. The nodule has focal calcifications. A wedge resection of the nodule is done. On microscopic examination the
nodule shows caseous necrosis and calcification. Which of the following processes explains the appearance of the calcium
deposition:
@Dystrophic calcification
Apoptosis
Hypercalcemia
Metastatic calcification
Excessive ingestion of calcium
#
104
On the autopsy of the woman body 63 years old, who suffered from rheumatism, combine mitral valvular disease. Cusps
of the mitral valve are fibrotic, thick, irregular,calcified,adhere together, on the cut crunching is detected. What
pathological process is the cause of the stony density of the heart valves?
@Distrophic calcification
Amyloidosis
Hyalinosis
Metabolic calcification
Metastatic calcification
#
105
The best example of dystrophic calcification is seen in a (an):
55-year-old woman with metastases from breast carcinoma and hypercalcemia
@Healing granuloma in a 41-year-old man with pulmonary tuberculosis
Gangrenous lower extremity in a 50-year-old woman with diabetes mellitus
62-year-old woman with a recent cerebral infarction
Abscess of the left fallopian tube in a 19-year-old woman with Neisseria gonorrheae infection
#
106
A 38-year-old man has a chest x-ray that shows a 2-cm nodule in the right lower lobe. The nodule has focal calcifications.
A wedge resection is done and microscopically the lesion proves to be a granuloma with caseous necrosis and
calcification. Which of the following processes explains the appearance of the calcium deposition;
@Dystrophic calcification
Apoptosis
Hypercalcemia
Metastatic calcification
Excessive ingestion of calcium
#
107
A clinical study is performed in which the conditions leading to the apperance of calcification on chest and abdominal CT
scans are analyzed in patients from 60 to 90 years of age. One set of patients is found in which the finding of calcification
was incidental, with no serious health problems. In which of the following tissue locations was this incidental
calcification most likely to be noted?
Coronary artery
Renal cortex
Aortic valve
@Pulmonary parenchyma
Aortic arch
#
108
In which of the following patients would the presence of calcification is LEAST harmful:
A 55-year-old man undergoing coronary artery angioplasty
Nephrocalcinosis in a 60-year-old woman with hyperparathyroidism
Aortic valvular calcification in a 70-year-old man
@Alveolar wall calcification in a 41-year-old woman with breast cancer metastatic to bone
Aortic arch calcification in a 62-year-old man
#
109
A patient suffering from hepatocerebral degeneration has low cncentration of ceruloplasmin in blood serum. What
element accumulation will be observed in liver, cerebrum and kidneys of the patient?
Potassium
@Cuprum
Calcium
Sodium
Ferrum
#
110
Complaints and objective data allow to suggest that a patient has inflammation in his gallblader, bile colloidoclasia,
probability of gallstones formation. What can be the main cause of gallstones formation?
Oxolates
Chlorides
@Cholesterol
Urates
Phosphates
#
111
Man, 52 years old, is admitted to a hospital with a joint pain syndrome. During examination the contents of uric acid in
the blood was 3.0 mmol/l, in the urine protein and blood were found, deposits in joints were detected. What can be found
during X-ray examination (excretory urography) of kidney in this patient?
@Lithates
Oxalates
Cancer of kidney
Absence of kidney
Examination of kidney is not necessary in this patient
#
Necrosis
#
1
A 30-year-old woman present with malaise and increasing fatigue. Physical examination finds a yellow-tinge to her skin
and sclera, and laboratory evaluation finds elevated serum liver enzymes. After further work-up, the diagnosis of viral
hepatitis is made. This work-up included a liver biopsy, which revealed scattered, eosinophilic cells (Councilman
Bodies), but very little inflammation was present. These Councilman Bodies were formed by an active process that
characteristically lacks an inflammatory response and results from the activation of genes that forms new enzymes, such
as endonucleases. These enzymes subsequently destroy the cell itself. Which of the following terms best describes this
process?
@Apoptosis
Autophagy
Heterophagi
Metaplasia
Necrosis
#
2
The investigation of the liver's biopsy revealed that some separately arranged cells burn on small- sized pieces surrounded
by a membrane. In some of them there were organelles, other had the fragments of dissolved nuclei. The inflammatory
reaction around these cells missed. Name these changes:
@Apoptosis
Atrophy
Necrosis
Hypoplasia
Dystrophy
#
3
Histological investigation of liver's biopsy revealed that some cells burn on small pieces with separate organellas and
nuclei fragments surrounded by a membrane. The inflammatory reaction was missing. Select pathological process, the
described changes are characteristic for:
@Apoptosis
Necrosis
Karyorrhexis
Plasmolysis
Plasmorrhexis
#
4
An ultrastructural examination of a salivary gland revealed within cells pieces of the nuclei surrounded by a membrane.
Also condensate fragments of nuclear material and separate organelles were found. An inflammatory reaction around
these cells was missing. Which term most correctly defines these alterations?
@Apoptosis
Karyorrhexis
Coagulation necrosis
Karyopyknosis
Karyolysis
#
5
During embryogenesis the epithelial band also known as vestibular plate gives rise to development of vestibule of mouth.
What biological mechanism of the programmed death of cells provides growth of buccolabial sulcus from epithelial
plate?
Necrosis
@Apoptosis
Amitosis
Meiosis
Paranecrosis
#
6
All of the following are cellular responses to injury EXCEPT one:
Cell death
Cellular adaptation
Intracellular accumulation
@ Repair
Reversible changes
#
7
Which of the following cellular changes represents is the best evidence for irreversible cellular injury:
Epithelial dysplasia
Cytoplasmic fatty metamorphosis
@Nuclear pyknosis
Atrophy
Anaerobic glycolysis
#
8
A 45-year-old man has a traumatic injury to his forearm and incurs extensive blood loss. On physical examination in the
emergency department his blood pressure is 70/30 mm Hg. Which of the following cellular changes is most likely to
represent irreversible cellular injury as a result of this injury?
Epithelial dysplasia
Cytoplasmic fatty metamorphosis
@Nuclear pyknosis
Atrophy
Anaerobic glycolysis
#
9
A53-year-old man has experienced severe chest pain for the past 6 hours. On physical examination he is afebrile, but has
tachycardia. Laboratory studies show aserum troponin I of 10 ng/mL acoronary angiogram is performed emergently and
reveals occlusion of the left anterior descending artery. In this setting, an irreversible injury to myocardial fibers will have
occurred when which of the following cellular changes occurs?
Glycogen stores are depleted
Cytoplasmic sodium increases
@Nuclei undergo karyorrhexis
Intracellular pH diminishes
Blebs form on cell membranes
#
10
From a man suffering from tuberculosis a biopsy was taken. Histologically: in a nidus of caseous necrosis there are grains
of chromatin chaotically dispersed. These changes are the consequence of what?
Nuclear mitosis activity
Karyolysis
Karyopicnosis
@Karyorhexis
Apoptosis
#
11
A patient with tuberculosis has a kidney biopsy performed. Histological investigation revealed the caseous necrosis of the
tissue accompanied by disorderly arranged fine grains of a chromatin. Which term most correctly defines describe lesion?
@ Karyorrhexis
Karyolysis
Karyopyknosis
Mitotic activity of nuclei
Apoptosis
#
12
A 45-year-old male suddenly died with the following findings revealed during the postmortem. In the back wall of the left
ventricle of the heart a myocardial infarction was found. Which of the following microscopic changes in the
myocardiocytes can be seen in the locus of an infarct?
@Karyolysis
Fatty dystrophy
Carbohydrate dystrophy
Calcification
Protein dystrophy
#
13
Which of the following terms is defined correctly?
Heterolysis - changes induced by fixatives used in histopathologic tissue preparation
@Autolysis - tissue degradation by intracellular enzymes
Fibrinoid necrosis – ischemic necrosis resulting from arterial occlusion by fibrin clots
Apoptosis - ballooning of cell cytoplasm from lipid accumulation
-
#
14
Highly injured person gradually died. Please choose the indicator of biological death:
Absence of movements
@Autolysis and decay in the cells
Absence of palpitation
Loss of consciousness
Disarray of chemical processes
#
15
A 52 year old patient suffering from cancer of the lower jaw underwent a course of radiation therapy. The tumour has
remitted. Which mechanism of cell destruction ensures efficiency of radiation therapy most of all?
Mutagenesis
@Generation of free radicals
Hyperthermia
Lysis by natural killer cells
Vessel thrombosis
#
16
A 27-year-old woman who is actively training for a marathon presents with the new onset of a painful lump in the upper
outer quadrant of her right breast. A mammogram shows an irregular mass with focal areas of calcification. An excisional
biopsy reveals a localized area of granulation tissue and numerous lipid-laden macrophages surrounding necrotic
adipocytes. Which of the following is the most likely diagnosis?
Acute mastitis
Ectasia
Enzymatic fat necrosis
Foreign-body reaction
@Traumatic fat necrosis
#
17
Which of the following types of cell and tissue death is most often caused by sudden ischemia from vascular occlusion?
Apoptosis
Caseous necrosis
@Coagulative necrosis
Fat necrosis
Fibrinoid necrosis
#
18
Which type of necrosis is most characteristic of ischemia involving the heart or kidney?
@Coagulative
Liquefactive
Caseous
Fibrinoid
Enzymatic
#
19
A 49-year-old man develops an acute myocardial infarction because of the sudden occlusion of the left anterior
descending coronary artery. Which of the following types of necrosis should be present in these areas of infarction?
@Coagulative necrosis
Liquefactive necrosis
Fat necrosis
Caseous necrosis
Fibrinoid necrosis
#
20
A 48-year-old woman has a malignant lymphoma involving lymph nodes in the paraaortic region. She is treated with a
chemotherapeutic agent which results in the loss of individual neoplastic cells through fragmentation of individual cell
nuclei and cytoplasm. Over severaleeks, the size of the lymphoma decreases, as documented on abdominal CT scans. By
which of the following mechanisms has her neoplasm primarily responded to therapy?
@Coagulative necrosis
Mitochondrial poisoning
Phagocytosis
Acute inflammation
Apoptosis
#
21
In an experiment, a glass bead is embolized to a branch of the renal artery. A day later there is a focal area in which the
renal parenchymal cells in the distribution of the occluded artery show karyolysis and karyorrhexis. The outlines of the
cells are still visible, but the nuclei have lost basophilic staining and the cytoplasm is eosinophilic but pale. Which of the
following types of cellular necrosis is most likely present?
Caseous
@Coagulative
Fatty
Gangrenous
Liquefactive
#
22
A 59-year-old woman had the loss of consciousness that persisted for over an hour. When she became arousable, she
could not speak and she could not move her right arm or legacerebral angiogram revealed an occlusion to her left middle
cerebral artery. Months later, a computed tomographic (CT) scan shows a large 5 cm cystic areain her left parietal lobe
cortex. This CT finding most likely demonstrates a lesion that is the consequence of resolution from which of the
following events?
@Liquefactive necrosis
Atrophy
Coagulative necrosis
Caseous necrosis
Apoptosis
#
23
A 73-year-old man suffers a "stroke." On physical examination he cannot move his right arm. A cerebral angiogram
demonstrates occlusion of the left middle cerebral artery. An echocardiogram reveals a thrombus within a dilated left
atrium. Which of the following is the most likely pathologic alteration from this event that occurs in his brain?
@Cerebral softening from liquefactive necrosis
Pale infarction with coagulative necrosis
Predominantly the loss of glial cells
Recovery of damaged neurons if the vascular supply is reestablished
Wet gangrene with secondary bacterial infection
#
24
Liquefaction necrosis occurs in
@Brain
Heart
Liver
Kidneys
#
25
Coagulative necrosis is seen in all except:
Myocardial infarction
@Burns
Tuberculosis
Zenker's degeneration
#
26
A 29-year-old man hospitalized for AIDS is found to have pulmonary tuberculosis. Which type of necrosis is found in the
granulomatous lesions (clusters of modified macrophages) characteristic of this increasingly frequent complication of
AIDS?
Coagulative
Liquefactive
@Caseous
Fibrinoid
Enzymatic
#
27
A surgery on a patient, with a history of syphilis revealed a locus of flabby tissue. Grossly, this locus was yellowish, dry,
structures and gummy. The most likely diagnoses is:
@ Caseous necrosis
Infarct
Waxy necrosis
Fibrinoid a necrosis
Steatonecrosis
#
28
Coagulative necrosis is seen with
@Tuberculosis
Gangrene
Sarcoidosis
Zenkers degeneration
#
29
Caseous necrosis is seen in
Non-specific lymphadenitis
Metastatic lymph node
Filarial lymphadenitis
@Tuberculosis lymphadenitis
#
30
At histologic examination of tuberculoma resected from right lung a necrosis in center is found out. Call a kind of the
necrosis.
@Caseous
Fatty
Liquefactive
Coagulative
Gangrene
#
31
During the section of a man who died of abdominal typhoid, was revealed that striated muscles of anterior abdominal wall
were consolidated, white and looked like a stearin candle. What pathological process is meant?
@Zenker’s necrosis (degeneration)
Fibrinoid necrosis
Colliquative (liquefaction) necrosis
Caseation (cheesy) necrosis
Apoptosis
#
32
A 63-year-old male died of an endemic typhus. During the post-mortem it was revealed that the muscles of the abdominal
wall and legs were dense with whitish-yellowish colouring. They resemble a candle. Name the pathological process?
@Waxy necrosis (Zenker's necrosis)
Apoptosis
Fibrinoid necrosis
Colliquative necrosis
Caseous necrosis
#
33
During an autopsy of a man died of abdominal typhus: straight muscles of abdomen are thick, white-coloured, look like
stearine candle. What is the name for such pathological process?
Apoptosis
Fibrinoid necrosis
Colliquative necrosis
Curdled necrosis
@Wax necrosis
#
34
An autopsy of a 57-years old man, who died of abdominal typhus: front abdominal muscles and thigh muscles are thick,
white-yellow in colour, look like paraffin candle. Name the pathological process in muscles.
@Wax necrosis
Apoptosis
Fibrinoid necrosis
Colliquative necrosis
Curdled necrosis
#
35
A postmortem of a man, who died from typhoid revealed muscles on the abdominal wall and legs were dense, fragile,
whitish-yellowish colour, resembling a candle. Which term best characterizes the muscles changes?
@Zenker's necrosis
Fibrinoid necrosis
Caseous necrosis
Colliquative necrosis
Apoptosis
#
36
A postmortem of a 48-year-old male who had a history of typhoid fever revealed that the rectus abdominis at the wall was
dense, a whitish colour, and resembled a candle. Which is the most likely diagnosis?
@Waxy necrosis
Fibrinoid necrosis
Colliquative necrosis
Caseous necrosis
Apoptosis
#
37
At post-mortem, a 60-years-old man, with a history of typhoid fever, is found to have rectus muscles of the anterior
abdominal wall dense, whitish, and look like a stearic candle. What is the most likely diagnose?
@Zenker's necrosis.
Fibrinoid necrosis.
Colliquative necrosis.
Caseous necrosis.
Apoptosis.
#
38
A 44 year old ill patient died due to the severe chronic heart failure. Pathologist diagnosed rheumatic granulomatous
myocarditis. Microscopic evaluation of the myocardium indicated the presence of granulomas which consisted of
macrophages with hyperchromatic nuclei and clear cytoplasm. Also necrosis was seen in the center of a lesion. Which is
the most likely type of necrosis in the center of the lesion?
@Fibrinoid necrosis.
Waxy necrosis.
Caseous necrosis.
Colliquative necrosis.
Fatty dystrophy.
#
39
A postmortem was performed on a female who died due to the cystadenocarcinoma metastases. The postmortem revealed
large segments of a necrosis of the skin and soft tissues within cubitus area. Name the form of the necrosis.
@Decubitus ulcer
Infarct
Sequester
Caseous necrosis
Zenker's necrosis
#
40
A patient with circulatory deficiency got the skin and soft tissue darening above the sacrums after a long staying in the
bed. These tissues became swallowed. Sloughing off the epidermis in this area resulted in ulceration. Which complication
is most likely?
@Decubitus ulcer
Dry gangrene
Phlegmon
Infarct
Abscess
#
41
A male who had a long history of the intermittent claudication demonstrates the tissue of the foot fingers as being dry
with a black colour, resembling a mummy. On small interval from this place the dichromatic line (red colour is next to
unchanged tissues and white - yellow colour close to tered tissues). Which type of a necrosis occurred?
@Gangrene.
Infarct.
Sequester.
Decubitus ulcer.
Maceration.
#
42
The physical exam of a 6-year-old girl, who had measles, showed that soft tissues of cheeks had fuzzy outlined,
edematous, red-black territories. Name the complication of measles.
Gaseous gangrene
Dry gangrene
Dry gangrene
@Moist gangrene (noma)
Bedsore
Trophic ulcer
#
43
A 57-year-old patient has a long history of the type II diabetes mellitus. Physical examination revealed the alteration of
the right foot tissues. They are dense, black with precise boundaries from normal tissues. Which terms most correctly
describe the lesion?
@Dry gangrene.
Wet (moist) gangrene.
Gas gangrene.
Decubitus.
Trophic ulcer.
#
44
A 60 year old man suffered from diabetes mellitus. Tissues of his right foot are black, thickened, with sharp edges. What
is the diagnosis?
@Dry gangrene
Moist gangrene
Gaseous gangrene
Bedsore
Trophic ulcer
#
45
A 55-year-old man has a 30-year history of poorly controlled diabetes mellitus. He has had extensive black discoloration
of skin and soft tissue of his right foot, with areas of yellowish exudate, for the past 2 months. Staphylococcus aureus is
cultured from this exudate. A below-the-knee amputation is performed. The amputation specimen received in the surgical
pathology laboratory is most likely to demonstrate which of the following pathologic abnormalities?
Neoplasia
@Gangrenous necrosis
Coagulopathy
Hemosiderosis
Caseation
#
46
A patient with diabetes mellitus suddenly began having sharp pain in his right foot. The examination revealed black
hallux, swallen foot tissues, focuses of epidermis detachment, secreta with unpleasant smell. What clinico-morphological
form of necrosis developed in a patient?
Sequester
Bedsore
Dry gangrene
Infarction
@Moist gangrene
#
47
A patient with diabetes mellitus presents to his physician with an acute pain in the right foot. At inspection the toe of foot
had a black colour; the tissues of foot were edematous, with bed smell. Which form of necrosis was likely to be
diagnosed?
@ Moist (wet) gangrene
Decubitus ulcer
Sequester
Dry gangrene
Infarct
#
48
A patient with the history of diabetes mellitus presents has sharp pain in right foot. The thumb is black; the tissues are
edematous. Focal epidermal detachment and secret with wicked smell are also presented. What clinico-morphological
form of necrosis has developed?
Bedsore
@Moist gangrene
Sequestrum
Dry gangrene
Infarction
#
49
A 65-year-old female, with a long history of diabetes mellitus, presented her black, edematous and painful thumb of the
right foot. Gross inspection revealed a focal epidermal detachment and malodorous discharge. What is the most likely
clinicopathologic form of necrosis?
@ Moist (wet) gangrene.
Decubitus ulcer.
Sequester.
Dry gangrene.
Infarction.
#
50
A patient with diabetes mellitus gained pain in his right leg. Thumb tissues are black, swollen, epidermis is exfoliated,
exudate with repulsive smell appeared. Name the pathological process.
@Moist gangrene
Dry gangrene
Coagulative necrosis
Sequestrum
Infarction
#
51
A 5-year-old boy with measles presents to his pediatrician with necrotic changes of his cheeks. Gross inspection revealed
that the cheeks soft tissues were edematous with reddish black fluctuated indistinctly outlined foci. What is the most
likely complication of a measles?
@Moist gangrene.
Dry gangrene.
Gas gangrene.
Decubitus ulcer.
Trophic ulcer.
#
52
The examination of a child, who had a history of measles, revealed reddish-black, uneven, swollen, slightly fluctuated
lesions of cheaks and perineum area. Name the complication of measles?
@Moist gangrene (noma)
Dry gangrene
Gas gangrene
Decubitus ulcer
Trophic ulcer
#
53
A 6 year old child was delivered to the hospital because of measles pneumonia. On the mucous membrane of a cheek a
dentist revealed an ill-defined greish area 2х2,5 cm large. Soft tissues are edematic and foul-smelling. The most probable
diagnosis of the dentist should be:
Ulcerous stomatitis
Pustular stomatitis
@Noma
Phlegmonous stomatitis
Gangrenous stomatitis
#
54
Examination of a child who has recently recovered from measles revealed in the soft tissues of cheeks and perineum some
inaccurate, edematic, red-and-black, slightly fluctuating areas. What complication is it?
Trophic ulcer
Gas gangrene
Pressure sore
@Humid gangrene
Dry gangrene
#
55
A 62-year-old male got a surgery due to the inguinal hernia. Macroscopic examination reveals that the wall of the
intestine was a cyanotic, inflated, swallowed and coated with threads of a fibrin. Peristalsis was not heard. Which
pathological process occurred in the wall of the intestine?
@Moist gangrene.
Dry gangrene.
Coagulate necrosis
Colliquative necrosis
Decubitus ulcer
#
56
During a laparotomy of a 77 year old woman with strangulated abdominal hernia there is a cyanotic-coloured gut,
expanded, swollen, covered with fibrin filaments and without any peristaltic activity. What kind of pathological process
takes place in the gut?
@Moist gangrene
Dry gangrene
Coagulative necrosis
Colliquative necrosis
Bedsore
#
57
An ill elderly patient with a atherosclerosis, develops pain in the left foot. Grossly was found the foot enlargement, its
tissues were black, flabby and macerated. The demarcation zone was not expressed. Which term best characterized the
foot tissues changes?
@Moist (wet) gangrene.
Mummification.
Coagulate necrosis.
Dry gangrene.
Sequestrum.
#
58
A male had a surgery due to "acute abdomen". During the operation it was noted that the peritoneum was dull and in the
lumen of the mesenteric artery superior a thrombus was detected. About 80 centimeters of the ileac intestine had a black
colouring. Which process was diagnosed in the intestine?
@Gangrene
Decubitus ulcer
White infarct
White infarct with a hemorrhagic crown
Coagulative necrosis
#
59
A 70-year old woman was operated because of acute stomach. During operation there was found 80 cm of ileac intestine,
black-coloured. Peritoneum was dull, lumen of upper mesenterial artery is clogged by a thrombus. What's the name of the
process in the intestine?
@Gangrene
Bedsore
White infarction
White infarction with haemorrhagic halo
Coagulative necrosis
#
60
A patient with insufficient blood circulation, moveless after a stroke, has his skin and muscles above sacrum black and
swollen, epidermis is exfoliated and few ulcers appeared. Name the process.
@Bedsore
Dry gangrene
Phlegmona
Infarction
Abscess
#
61
A patient with cerebral haemorrhage has been lying on the bed for a long time. His skin along the vertebral column
became dark-brown colour, soft tissues became swollen, epidermis is desquamated. Name the form of necrosis.
@Bedsore
Dry gangrene
Moist gangrene
Coagulative necrosis
Sequestrum
#
62
During the exam of femur bone, a chronic purulent inflammation of compact substance and medulla and sequestra
formation were determined. There was a formation of sequestra, too. What disease causes such changes?
Osteoblastoclastoma
Reticulosarcoma
Myeloid disease (multiple myeloma)
@Osteomyelitis
Peruostitis
#
63
A physical examination of 67-year old lady, with a history of femoral bone fracture, revealed a sequester formation
accompanied with chronic inflammation of a bone marrow and adjacent tissues. What is the most likely disease caused
such lesions?
@Osteomyelitis.
Reticulosarcoma.
Multiple myeloma.
Osteoclastoma.
Periostitis.
#
64
A patient has exacerbation of chronic periodontitis: there is a fistula with purulent exudate. Radiogram shows: several
areas of rarefaction of bone, sequestri. What is the pathological process in the bone?
@Osteomielitis
Bedsore
Gangrene
Fibrous dysplasia
Osteoma
#
65
In 77-year-old patient suffered with atherosclerosis the pain has appeared in the right foot. The foot is enlarged in size, its
skin has black color and is macerated; the demarcation line is not clear. What pathological process takes place in a
patient?
@Sequestrum
Coagulative necrosis
Wet gangrene
Dry gangrene
Noma
#
66
A 45-year-old woman has had congestive heart failure for the past 4 years. She develops a fever that persists for over a
week. On physical examination, a heart murmur is present. Her temperature is 30.4 C. The spleen tip is palpable. On
echocardiography she has an abnormally thickened mitral valve. Laboratory studies show a blood culture positive for
Streptococcus, viridans group. Another echocardiogram reveals a 1 cm vegetation on the superior aspect of her mitral
valve. Which of the following findings would you most expect to appear in the kidney as a consequence of these events?
Marked passive congestion
@Ischemic thromboembolic infarct
Extensive edema
Granulomatous inflammation
Gangrenous necrosis
#
67
A patient presents high activity of LDH 1,2, aspartate aminotransferase, creatine phosphokinas e. In what organ (organs) is
the development of a pathological process the most probable?
In skeletal muscles (dystrophy, atrophy)
In connective tissue
@In the heart muscle (initial stage of myocardium infarction)
In kidneys and adrenals
In liver and kidneys
#
68
A patient died from acute cardiac insufficiency.The histological examination of his heart revealed in myocardium of the
left ventricle the necrotized section, which was separated from undamaged tissue by the zone of hyperimic vessels, small
hemorrhages and leukocytic infiltration. What is the most likely diagnosis?
@Myocardial infarction
Productive myocarditis
Myocardial ischemic dystrophy
Focal exudate myocarditis
Diffuse exudate myocarditis
#
69
At a post-mortem of the 46-year-old male a large yellow - grey lesion was found in the left ventricle of the heart. A fresh
thrombus was found in the coronary artery. What disease is he most likely to have?
@Infarct of the myocardium
Cardiosclerosis
Myocarditis
Amyloidosis
Cardiomyopathy
#
70
At autopsy a 60-year-old male is found to have ischemic heart disease and atherosclerosis of the coronary arteries of
heart. A section of the myocardium showed a white-yellowish focus, surrounded by the zone of hemorrhages in the apex,
anterior and lateral walls areas of a left ventricle, wnicn is me most likely diagnosis?
@Infarct of the myocardium
Post-infarction cardiosclerosis
Diffuse cardiosclerosis
Myocarditis
Fatty dystrophy of the myocardium
#
71
A 58-year-old female with the history of atherosclerosis dies suddenly due to acute heart failure. Gross inspection of the
left ventricle of the heart revealed a whitish-yellowish 6x5 cm, dense lesion with uneven boundaries and hemorrhagic
zone next to it. Which is most likely diagnosed?
@Infarct of the myocardium
Postinfarction fibrosis
Healed infarct
Myocarditis
Ischemic cardiomyopathy
#
72
A postmortem of on a previously ill 48-year-old patient found an obturation of the lumen of the middle cerebral artery
due to a thrombus. In the parietal-temporal area of the left hemisphere of the brain a locus of grey colour tissue with soft
texture is found. Which tern best characterizes the brain tissue changes?
@Infarct
Sequester
Gangrene
Caseous necrosis
Fibrinoid necrosis
#
73
A post-mortem revealed a thrombus in the left artery mesencephalicae and a large locus of grey softening in the tissues of
the left hemisphere of a brain. Which pathological process is most likely to be present in the brain?
@Ischemic infarct
Coagulative necrosis
Abscess
Moist gangrene
Sequestrum
#
74
A 48-year-old male with a history of hypertension (12 years) presents acute disturbance of the cerebral circulation. He
developed a headache and alteration of the motion in the right extremity. Following right-handed hemiplegia resulted in
fatal outcome. A postmortem revealed a systemic hyalinosis of the small arteries, thrombosis in the left arteria cerebri
media. In the left parietal-temporal area a lesion was found, which is called:
@Ischemic infarct
Hemorrhage
Abscess of a brain
Hemorrhagic infarct
Edema of the brain
#
75
A postmortem on an elderly male with atherosclerosis reveals a thrombus in a branch of the internal carotid artery as well
as a grey locus of a moist softening of the brain's tissue. Which pathological process was found in the brain?
@Ischemic infarct
Hemorrhagic infiltration
Hematoma
Encephalitis
Tumor of a brain
#
76
The postmortem of a 48-year-old male reveals in the right temporal lobe of the brain a large grey lesion with a softening,
porridge-like texture. The basal arteries of the brain had numerous white - yellow thickenings of an intima which
significantly decreased lumen. Which is the most likely diagnosis?
@Ischemic infarct
Abscess of a brain
Hematoma
Hemorrhagic infarct
Edema of the brain
#
77
Autopsy of a woman with cerebral atherosclerosis revealed in the left cerebral hemisphere a certain focus that is presented
by flabby, anhistic, greyish and yellowish tissue with indistinct edges. What pathological process is the case?
@Ischemic stroke
Focal encephalitis
Multiple foci of fresh and old cerebral hemorrhage
Senile encephalopathy
Multifocal tumor growth with cystic degeneration
#
78
An aged person had an infarction of a right hemisphere. In a year, as the person had immobility of left limbs, he was
made computed tomography. A cave was detected in the right hemisphere, which had plain walls and was filled with
liquor. What pathological process was detected in cerebrum?
Haematoma
Hydrocephalus
@Grey softening of the brain
Cerebral infarction
Post-MI (myocardial infarction) cyst
#
79
A postmortem on the upper lobe of the right lung reveals the large triangle-like locus of the
dark red dense tissue. Histological examination indicates necrosis of the walls of the alveolus's and the lumens filled with
erythrocytes. Which is the most likely associated finding?
@Hemorrhagic infarct
Carnification
Lung's gangrene
Hemorrhage
Atelectasis
#
80
A 62-year-old female with atherosclerosis was admitted to the hospitalized. At surgery gross examination revealed
purulent peritonitis. During the operation a thrombus in the mesenteric arterias was found. Which was the most likely
cause of the peritonitis?
@Hemorrhagic infarct.
Angiospastic ischemia
Ischemic infarct
Stasis
Compressive ischemia
#
81
A section of the left lung was found to have an area of dense red tissue. The area was cone-shaped, stood out distinctly
from the healthy tissue, with its base directed to the pleura. The dissected tissue was granular, dark-red. What is the most
likely diagnosis?
Lung abscess
Primary tuberculous affection
Lung gangrene
Croupous pneumonia
@Haemorrhagic infarction
#
82
An old man had infarction of the right cerebrum hemisphere. One year later, taking into consideration the absence of
movements of his left extremities, the patient underwent computer tomography of the cerebrum, which revealed a
smooth-walled cavity, full of liquor. What pathological process is found out in the cerebrum?
Brain infarction
Gray maceration of the brain
@Postinfarction cyst
Hematoma
Hydrocephaly
#
83
A 72-year-old-male had an infarct of the dextral hemisphere of the brain. One year later a computer tomography of the
right hemisphere of the brain reveals a cavity with smooth walls and filled with liquid. Which pathological process is he
most likely to have?
@Post-infarct cyst
Hydrocephalus
Grey softening of a brain
Infarct of a brain
Hematoma
#
84
An elderly woman with a history of a stroke one year ago complains of the left limbs immobility. A computer
tomography examination revealed a cavity filled with liquor, at right hemisphere of her brain. What is the most likely
diagnose?
@ Postinfarction cyst.
Hydrocephaly.
Grey encephalomalacia.
Cerebral infarction.
Hematoma.
#
85
During the autopsy it was found out the cavity 2,5x1,5 cm within the right frontal part of the brain. It was filled with
transparent fluid and its walls were smooth and of brownish colour. What process did develop in the brain?
@Cyst as an outcome of hemorrhage
Grey softening of brain
Abscess of brain
Birth defect of brain
Cyst as a outcome of gray softening
#
86
A postmortem examination of a dead body revealed a cloudy corneas, dry skin with yellowish - brown lesion. Which term
most correctly identifies describes a post-mortem alteration?
@ Cadaver desiccation
Clotting of blood
Livor mortis
Rigor mortis
Algor mortis
#
Circulatory disturbance №1.
#
1
All following features are associated with hyperemia EXCEPT one:
@Involved the sympathetic neurogenic mechanism
Depend on the releasing of vasoactive substances
Increased blood flow into capillary beds
Marked dilation of small venues
Caused intensified blue-red coloration
#
2
While playing volleyball a sportsman made a jump and landed on the outside edge of his foot. He felt acute pain in the
talocrural joint, active movements are limited, passive movements are unlimited but painful. A bit later there appeared a
swelling in the area of external ankle, the skin became red and warm. What type of peripheral circulation disturbance is
the case?
@Arterial hyperemia
Venous hyperemia
Stasis
Embolism
Thrombosis
#
3
A 22-year-old second-year medical student develops a “red” face after being asked a question during lecture. Which of
the following statements best describes this vascular reaction?
@Active hyperemia
Acute congestion
Non-palpable purpura
Passive hyperemia
Petechial hemorrhage
#
4
A 42-year-old woman with trigeminal nerve neuralgia complaints of redness of right part of her face and neck, feeling of
heat rush and increased dermal sensitiveness. What is the arterial hyperemia in this case according to pathophysiological
mechanism?
Neuroparalytic
Reactive
Working
@Neurotonic
Metabolic
#
5
A rabbit's nerve that innervates the right ear was cut and its right superior cervical ganglion was removed. Immediately
after operation the temperature of ear skin was measured. It was revealed that the temperature of the rabbit's ear skin on
the side of denervation was by 1,50C higher than on the opposite intact side. What of the following is the most probable
explanation of the above-mentioned effects?
@Atrerial hyperemia induced by metabolic factors
Arterial neuroparalytic hyperemia
Reactive arterial hyperemia
Arterial neurotopical hyperemia
Physiological arterial hyperemia
#
6
Upper neck node of sympathetic trunk was removed from the rabbit on experiment. Reddening and increased temperature
of the skin of head is observed. What form of peripheral circulation of the blood developed in the rabbit?
Venous hyperemia
Stasis
Neurotonic arterial hyperemia
Metabolic arterial hyperemia
@Neuroparalytic arterial hyperemia
#
7
A 65-year-old- patient presented with liver cirrhosis. The removal from his abdominal cavity of 10 liters of ascitic liquid
resulted a collapse. Grossly his peritoneum was hyperemic. Define the type of hyperemia in peritoneum.
@Post-anemic hyperemia
Collateral hyperemia
Complementary hyperemia
Inflammatory hyperemia
Caused by arteriovenous fistula
#
8
A male with a fracture of the shoulder bone carried on overlapped plaster bandage. Suddenly, an arm and the visible part
of the forearm became cyanotic, cold, and edematous. Which of the following is most likely happened?
@Local venous congestion
Local arterial hiperemia
Local anemia
Stasis
Thrombosis
#
9
A patient with closed fracture of humeral bone was bandaged with plaster. The next day the injured hand became swollen,
cyanotic and cold. What disorder of peripheral blood circulation are these symptoms typical for?
Thrombosis
Ischemia
Arterial hyperemia
Embolism
@Venous hyperemia
#
10
A 50-year-old male with a myocardial infarction died from the heart failure. A post-mortem revealed the edema of the
lungs and petechial hemorrhages at serous and mucus membranes. Microscopic examination indicated marked dystrophic
alterations of the nephron's epithelium at proximal canaliculi of kidneys. Also, the centrolobular hemorrhage and necrotic
zones were found in the liver. Name the type of the blood circulation disorder.
@Acute general venous congestion.
Arterial hyperemia.
Chronic general venous congestion.
Acute anemia.
Chronic anemia.
#
11
In patient with rheumatic mitral stenosis were revealed the dilatation of right chambers, tricuspid regurgitation, ascities,
hydrothorax, edema of low extremities and increased central venous pressure. This state is the finely correlated with:
Acute localized congestion
Acute localized hyperemia
Chronic localized congestion
@Chronic generalize congestion
Chronic generalize hyperemia
#
12
A post-mortem of an elderly man with a long history of the ischemic heart disease and heart failure revealed a nutmeg
liver, brown induration of lungs, cyanotic induration of kidney and spleen. Indicate, what type of the blood circulation
disorder is most likely?
@Chronic general venous congestion.
Arterial hyperemia.
Acute general venous congestion.
Acute anemia.
Chronic anemia.
#
13
During an autopsy of a man: liver is enlarged, thickened, has rounded edges. On incision the tissue of liver is yellowish-
brown in colour with red spots and stripes, looks like nutmeg. What pathological process causes such changes?
@Chronic venous hyperaemia
Acute venous hyperaemia
Arterial hyperaemia
Arterial ischemia
Chronic bleeding
#
14
During an autopsy of a 69 year old man, who died of chronic cardiac failure: liver is enlarged, thickened. On incision the
liver is stripy of greyish-yellow colour with dark-red spots. What is the figurative name for such liver?
Stripy
Yellow
Dotty
@Nutmeg
Red
#
15
The histological investigation of a liver detects venous plethora of center lobules, dystrophy and atrophy of hepatocytes in
the venous plethora's area, fatty a dystrophy of hepatocytes on the periphery of a lobe. The replacement fibrosis in places
of an atrophy of the hepatocytes was also revealed. Which pathological process does this refer to?
@'Nutmeg' liver with precirrhotic phenomena.
Biliary hepatic cirrhosis.
Fatty hepatosis.
Hepatitis.
Toxic dystrophy of a liver.
#
16
A male with a history of myocardial infarct died of cardiovascular failure. A post-mortem revealed a replacement fibrosis,
hypertrophy of the myocardium and dilatation of the cavities, especially the right ventricle. The liver was enlarged. Its
surface was smooth. Grossly, a motley pattern with dark red dots on the grey a background was revealed. Histologically,
central zones of the lobules were hyperemic. On the periphery, around of periportal tracts hepatocytes demonstrated fatty
dystrophy. Name these liver changes.
@"Nutmeg" liver (chronic venous congestion)
Pseudo 'nut-meg' liver
Amyloidosis
Cirrhosis of a liver
Steatosis of a liver
#
17
A 65-years-old patient, who presented with a 10 years history of ischemic heart disease, died due to heart failure. An
autopsy revealed the cyanotic induration of both spleen and kidneys, brown induration of lungs and "nutmeg" liver.
Which is the most likely type of the blood circulation disorder, resulted in such changes of internal organs?
@General chronic venous hyperemia
General acute venous hyperemia
General arterial hyperemia after an anemia
Arterial ischemia as a result of reallocating a blood
Local chronic venous hyperemia
#
18
A patient died under conditions of cardiovascular insufficiency. Autopsy results: postinfarction cardiosclerosis,
myocardium hypertrophy and dilatation of its cavities, especially of its right ventricle. Liver is enlarged, its surface is
smooth, and incision revealed that it was plethoric, with dark-red specks against the background of brownish tissue.
Histologically: plethora of central parts of lobules; peritheral parts around portal tracts contain hepatocytes in a state of
adipose degeneration. How are these liver changes called?
Liver cirrhosis
Liver steatosis
@Nutmeg liver
Pseudonutmeg liver
Amyloidosis
#
19
The postmortem of a 48-year-old male, with a long history of a chronic heart failure, revealed an enlarged liver. Grossly,
a liver had a motley pattern. Macroscopically, the sectional view looked like a nutmeg on incision. Which term most
correctly defines these alteration?
@General venous congestion
General arterial hyperemia
Anemia
Hemorrhage
Bleeding
#
20
A 77-years-old female with unstable angina presents to physician with slowly increasing heart failure symptoms. On the
night of admission to the hospital she dies. A post-mortem examination revealed the enlarged liver with dense texture and
the rounded edges. Sectional view had a nutmeg pattern with dark red dots on the yellowish background. Which
pathological process resulted in liver's alteration?
@Chronic venous congestion
Acute venous congestion
Arterial hiperemia
Arterial anemia
Chronic hemorrhage
#
21
The autopsy of the dead body revealed enlarged, solid with rounded margins liver, on section the tissue was yellow-
brown in colour with dark red spots and stripes, which resembled nutmeg. What pathological process led to such liver
changes?
Chronic hemorrhage
@Chronic venous congestion
Acute venous congestion
Arterial hyperaemia
Arterial anemia
#
22
A 54-years old male with a history of ischemic heart disease presents at hospital with recurrent myocardial infarction.
Few days later, he died due to cardiac failure. Post-mortem revealed an enlarged solid spleen of dark cherry color on the
cut surface. Microscopically, pulp sclerosis and follicles atrophy were found out. What is the most likely term to define
spleen's alterations?
@Cyanotic induration of spleen.
Sago spleen.
Lardaceous spleen.
Porphyry spleen.
Septic splenitis
#
23
A patient who had suffered from ischemic heart disease and has had recurrent myocardial infarction died in the
background of progressive cardiovascular failure. An autopsy showed enlarged, thickened spleen of cherry colour on
incision. Microscopically: follicle atrophy and pulp sclerosis. What term is used to describe such changes?
Porphyry spleen
Sago spleen
Fatty spleen
@Cyanotic spleen induration
Septic spleen
#
24
The patient, who had ischemic heart disease and myocardial infarction, died because of progressive cardiovascular
insufficiency. At section an enlarged solid spleen was detected, dark cherry colour when incised. Microscopically:
sclerosis of the pulp and the atrophy of follicles were examined. What term can we use to name these changes?
Porphyry spleen
Sago spleen
Tallow spleen
@Cyanotic induration of the spleen
Septic spleen
#
25
Examination of a patient who had been suffering from rheumatism for a long time revealed stenosis of mitral orifice,
death was caused by cardiac and pulmonary insufficiency. Autopsy has shown brown induration of lungs. What type of
circulation disturbance provokes such changes in lungs?
@Chronic left ventricular insufficiency
Chronic right ventricular insufficiency
Acute right ventricular insufficiency
Portal hypertension
Acute left ventricular insufficiency
#
26
Autopsy of a 73-year-old man who had been suffering from the coronary heart disease along with cardiac insufficiency
for a long time revealed: nutmeg liver, brown induration of lungs, cyanotic induration of kidneys and spleen. What kind
of circulation disorder was the cause of such effects?
Chronic anaemia
Arterial hyperaemia
Acute anaemia
@General chronic venous congestion
General acute venous congestion
#
27
A 63-year-old patient with long history of ischemic heart disease and repeated myocardial infarction died due to
progressive cardiovascular failure. A post-mortem revealed an enlarged, dense spleen with the dark red colour of the
sectional view. At the microscopic examination of spleen the pulp's sclerosis and atrophy of the follicles were found.
Which term most correcuy ueimca www changes?
@Cyanotic induration of a spleen
Sago spleen
Waxy spleen
Porphyry spleen
Septic spleen
Tissue embolism
#
28
A 52-year-old male with long history of rheumatic heart disease died of chronic heart failure. A post-mortem revealed
brown colour, enlarged dense lungs. Name the changes in lungs.
@Brown induration of lungs
Acute bronchitis
Horny-comb lungs
Chronic bronchitis
Chronic emphysema
#
29
An autopsy of a man, who died of chronic cardiac failure, showed: lungs are enlarged, thickened, rusty-coloured with
conjunctive tissue expansive growth around bronchi and vessels. What is the most probable diagnosis?
@Brown lung induration
Nidus pneumonia.
Croupous pneumonia
Interstitial pneumonia
Haemorrhagic pneumonia
#
30
A young male with a history of rheumatic heart disease since childhood, gradually develops mitral stenosis, accompanied
with episodes of heart failure. He presents to a hospital complaining of coughing with reddish-brown sputum. Name
probable changes at patient's lungs.
@Brown induration of lungs
Emphysema of lungs
Atelectasis of lungs
Pneumosclerosis
Bronchiectasis
#
31
A patient with a long history of rheumatic disease died of cardiopulmonary failure. A post-mortem revealed a stenosis of
the mitral orifice as well as a brown induration of lungs. Which term most correctly defines this blood circulation
disorder?
@Chronic left ventricular failure.
Chronic right ventricular failure.
Acute left ventricular failure.
Acute right ventricular failure.
Portal hypertension.
#
32
Histologic sections of lung tissue from a 68-year-old woman with congestive heart failure and progressive breathing
problems reveal numerous hemosiderin-laden cells within the alveoli. Which of the following is the cell of origin of these
“heart failure cells”?
Endothelial cells
Eosinophils
Lymphocytes
@Macrophages
Pneumocytes
#
33
During emotionally hard work suddenly died a young man. Autopsy showed uneven blood perfusion of myocardium.
Histochemically: decrease of glycogen content. Electronic microscopical research showed mitochondria destruction,
myofibrils contractures. Name the most probable blood circulation failure.
Chronic ischemia
@Acute ischemia
Vacate arterial hyperaemia
Acute venous hyperaemia
Angioneurotic arterial hyperaemia
#
34
On examination of a road accident victim a doctor revealed left clavicle fracture and disturbed blood circulation in an
extremity (no pulsing of radial artery). What cause of blood circulation disturbance is the most probable?
Compression of vertebral artery
Compression of subclavian vein
@Compression of subclavian artery
Compression of axillary vein
Compression of axillary artery
#
35
A patient, who had fast elimination of 10 liters of an ascitic liquid from abdomen, suddenly lost his consciousness. What
was the cause of this phenomenon?
@Ischimia of the brain
Thrombosis of the cerebral arteries
Brain hemorrhage
Arterial hyperemia
Thrombosis of the cerebral veins
#
36
During a section of an 86-year-old woman, who had an atherosclerosis of cerebral vessels, the atrophy of cerebral cortex
was determined. Name the atrophy due to the reason.
Pressure
@Insufficiency of blood supply
Physical and chemical factors
Neurotic
Dysfunctional
#
37
A post-mortem of 77-year-old male, with a long history of cerebral atherosclerosis, revealed an atrophy of his cerebral
cortex. What was the most likely cause of cortex alterations?
@Insufficiency of blood supply.
Pressure.
Physical and chemical factors.
Neurotic.
Dysfunctional.
#
38
During the autopsy of a 48-year-old man, the obturation of the middle cerebral artery’s lumen by clot was determined.
The parieto-temporal part of the left hemisphere had a focus of pappy consistence and of gray colour. Term the most
possible reason of such changes.
Fibrinoid necrosis
Sequestrum
Gangrene
Caseous necrosis
@Infarction
#
39
A 56-year-old male with a history of cerebral atherosclerosis suddenly dies. At a there is a thrombus within a middle
cerebral artery's lumen. Gross inspection revealed a grey flabby focus at the parietotemporal part of the left hemisphere of
his brain .What are the most likely alterations diagnosed in a brain tissues?
@Infarction.
Sequestrum.
Gangrene.
Caseous necrosis.
Fibrinoid necrosis.
#
40
Punctata hemorrhage was found out in the patient after application of a tourniquet. With disfunction of what blood cells is
it connected?
Monocytes
Eosinophiles
Neutrophiles
Lymphocytes
@Platelets
#
41
The main intermediate cause of diapedetic hemorrhage is:
@Increased vascular permeability
Increased arterial pressure
Small damages of vascular wall
Sclerosis of vascular wall
Hyalinosis of vascular wall
#
42
A 67-years old female with long state rheumatic mitral stenosis were revealed: marked dilatation of left atrial chamber,
the signs of lung circuit hypertension. Was usually associated with intra alveolar hemorrhage. What is the underlying
mechanism of this type of hemorrhage:
Hemorrhagia per rhexin
Hemorrhagia per diabrosim
@Hemorrhagia per diapedesis
All the answers are right
All the answers are fals
#
43
During postmortem examination of 34-deceased male, died by suicidal hanging, the multiple subpleural, subepi- and
subendocardial minute hemorrhages were established. What is the name of this type of hemorrhage;
@Petechia
Ecchimoses
Purpera
Hematoma
Hemothorax and hemopericardial
#
44
A 20-year-old man incurs blunt trauma to his upper outer arm. On physical examination, there is a 2 x 3 cm contusion.
The initial soft tissue bleeding stops in a few minutes and the size of the bruise does not increase. Name this kind of
haemorrhage:
Haematoma
@Ecchimoses
Petehii
Purpura
Cyst
#
45
A 42 y.o. patient complains of pain in the epigastral area, vomiting; vomit masses have the colour of "coffee-grounds",
the patient has also melena. Anamnesis records gastric ulcer. Blood formula: erythrocytes - 2,8*1012 /l, leukocytes –
8*109/l, Hb- 90 g/l. What complication is it?
Pyloric stenosis
Penetration
@Haemorrhage
Canceration
Perforation
#
46
During an autopsy in the upper lobe of right lung there was found a big, wedge-like nidus of dark-red, thick tissue.
Histologically: necrosis of alveolar walls, alveolar lumen is densely filled with erythrocytes. Name the process.
Gangrene
Carnification
@Haemorrhagic infarction
Bloodstroke
Atelectasis
#
47
An autopsy of a patient who died suddenly at emergency room revealed in his brain a cavity of irregular shape (5 x 3.5
cm) filled with blood clots and macerated cerebral tissue. An area of cavitary destruction had a rim of brown
discoloration. This lesion was found within the subcortex nuclei at the right hemisphere of brain. What is the most likely
definition of described pathology?
@Hematoma
Hemorrhagic impregnation
Ischemic infarction
Cyst
Abscess
#
48
An elderly female develops acute disorder of the cerebral blood circulation, followed with coma, resulted in fatal
outcome. A post-mortem revealed in the right hemisphere of the brain a large cavity, filled with blood. Which
pathological process took place in the brain?
@ Hematoma
Hemorrhagic infiltration
Infarct of the brain
Diapedesis
Edema of the brain
#
49
Autopsy of a man who suffered from essential hypertension revealed a cavity with rust-coloured walls in the cerebral
substunce. What preceded the appearance of these changes?
@Hematoma
Plasmorrhagias
Ischemic infarction
Abscess
Diapedetic hemorrhages
#
50
During an autopsy of a man, who suffered from chronic hypertension, there was found a cavity in cerebral tissues, it had
rusty walls. What preceded these changes?
Abscess
Diapedetic bleedings
Ischemic infarction
Plasmorrhagias
@Haematoma
#
51
Autopsy of a man who had been suffering from hypertension revealed in his brain a cavity with rubiginous walls. What
event preceded development of these changes?
Plasmorrhagias
Ischemic infarction
@Haematoma
Abscess
Diapedetic haemorrhages
#
52
A post-mortem of a 60-year-old female with a history of secondary hypertension revealed a cavity at the right hemisphere
of the brain 4x2,5 cm, filled with red clots of blood and softened brain tissue. What term best characterized the brain
changes?
@Hematoma
Hemorrhagic impregnation
Ischemic infarct
Cyst
Abscess
#
53
A post-mortem of a 59-year-old patient with long history of idiopathic hypertension revealed in brain a cavity with rusty
colour walls. Which process preceded these changes?
@ Hematoma
Diapedesis
Ischemic infarct
Plasmorrhagia
Abscess
#
54
During the right he bemisphere subcortex nuclei autopsy, a distorted cavity (5 x 35 cm) with red concentrated blood and
softened cerebral tissue was determined. Term the pathology that developed in this case.
Cyst
Hemorrhagic leakage
Ischemic infarction
@Haematoma
Abscess
#
55
A post-mortem of a 53-year-old male with a long history of hypertension revealed the cavity in the occipital lobe of the
brain. It measured 2x1 cm, had a brownish smooth walls and filled with transparent liquid. Which is the most likely
diagnosis?
@Cyst after hemorrhages
Softening of the brain
Abscess of a brain
Developmental defect of a brain
Cyst after softening of the brain
#
56
A 78-year-old male, with a history of stroke 2 years ago, died of pneumonia as a complication of severe influenza. At
post-mortem, gross investigation of his brain revealed a cerebral cyst with rusty color of its walls. Perls' test was positive.
Name the process reveled within a cystic wall.
@Local hemosiderosis.
Common hemosiderosis.
Local hemomelanosis.
Infiltration of bilirubin.
Primary hemochromatosis.
#
57
After having a hemorrhagic insult, a cerebral cyst developed in a patient. After 2 years he died of grippe pneumonia. At
section the cerebral cyst with the walls of whight-rusty color was detected. The Perls’ test is positive. What process in the
cystic wall is assigned?
@Local hemosiderosis
Common hemosiderosis
Local hemomelanosis
Infiltration of bilirubin
Primary hemochromatosis
#
58
After restoration of blood circulation in damaged tissue accumulation of lactate comes to a stop and speed of glucose
consumption slows down. These metabolic changes are caused by activation of the following process:
@Aerobic glycolysis
Gluconeogenesis
Lipolysis
Glycogen biosynthesis
Anaerobic glycolysis
#
59
A 45 year old patient was taken to the hospital by an emergency team with serious cranial trauma in shock condition.
Objectively: unconscious, skin is pale, body t 0- 35,00С, low muscular tonus, reflexes are absent, pulse is rapid and weak,
AP- 50/30 mm Hg. What clinical shock stage is it?
Erectile stage
Torpid stage
Excitement stage
Inhibition stage
@Terminal stage
#
60
Shock and signs of acute renal failure (ARF) developed in the patient due to permanent injury. What is the leading cause
of development of ARF in the case?
Urine excretion violation
Increased pressure in the renal arteries
@Decreased arterial pressure
Decreased oncotic BP
Increased pressure in the nephron capsule
#
61
Purulent endometritis developed in a woman after delivery. Treating with antibiotics inhibitors of murein synthesis was
ineffective. Wide spectrum bactericidal antibiotic was administered to her. In 6 hours temperature rapidly increased up to
400C with shiver. Muscle pains have appeared. BP dropped down to 70/40 mmHg. Oligura has developed. What is the
main reason for the development of this condition?
Toxic effect of preparation
Internal bleeding
@Endotoxic shock
Anaphylactic shock
Bacteremia
#
62
A 22-year-old woman presents with the sudden onset of a high fever, a diffuse erythematous skin rash, and shock. She
started menstruating at age 13 and for several years has used tampons. Which of the following is the most likely
diagnosis?
Erysipelas caused by Streptococcus pyogenes
Fifth disease caused by human parvovirus B19
Scarlet fever caused by S. pyogenes
Secondary syphilis caused by Treponema pallidum
@Toxic shock syndrome caused by Staphylococcus aureus
#
63
After two weeks in the hospital following a fall in which she incurred a fracture of her left femoral trochanter, a 76-year-
old woman now has a left leg that is swollen, particularly her lower leg below the knee. She experiences pain on
movement of this leg, and there is tenderness to palpation. Which of the following complications is most likely to occur
next after these events?
Gangrenous necrosis of the foot
@Hematoma of the thigh
Disseminated intravascular coagulation
Pulmonary thromboembolism
Soft tissue sarcoma
#
64
Within minutes following a bee sting, a 37-year-old man develops marked respiratory stridor with dyspnea and wheezing.
He also develops swelling and erythema seen in his arms and legs. What type of pathological processes has develops in
patient?
Hyperaemia
Oedema
Venous congestion
Serous inflammation
@Anafilactic shok
#
65
In order to provide infiltrative anaesthesia a man was injected a solution of ultracaine with adrenaline. Afterwards,
suddenly there appeared hyperaemias, swellings with vesicles and itching. What kind of hypersensitivity is this?
Cytotoxic
@Anaphylactic
Immune complex damage
Decelerated hypersensitivity
Granulomatous
#
66
For the purpose of anaesthetization a patient got injection of local anestheti C. A few minutes later the patient got dyspnea
and tachycardia; he lost consciousness. What type of shock is it?
@Anaphylactic
Cardiogenic
Burn
Haemorrhagic
Traumatic
#
Circulatory disturbance №2.
#
1
A 35-year-old patient complains of repeated vomiting, diarrhea, decreased arterial pressure and tachycardia. He supposes
this condition related to the food poisoning. Laboratory tests detected Salmonella's infection. Hematological examination
revealed an increased number of erythrocytes per unit volume. Which circulatory disturbance took place to create this
pathology?
@Clotting of a blood.
Hemolysis of erythrocytes and compensatory induction of a hemogenesis
General arterial plethora
Polycythemia
Hyperchromatic anemia
#
2
Examination of a patient who has recently had a hepatic disease revealed low concentration of prothrombin in blood. First
of all this will cause disturbance of:
Vasculothrombocytic haemostasis
First phase of coagulation haemostasis
Fibrinolysis
Anticoagulative blood properties
@Second phase of coagulation haemostasis
#
3
During the autopsy of a 46-year-old man who died when the motorcycle he was riding was hit by a truck, a 1.2-cm red
mass is found within a branch of the left pulmonary artery. Grossly this mass is rubbery, gelatinous, and has a “chicken
fat” appearance. Histologic sections reveal that this mass is not attached to the wall of the pulmonary artery, and
alternating lines of Zahn are not seen. Which of the following statements best describes this intravascular mass?
@Postmortem blood clot
Postmortem hematoma
Premorten embolic blood
Premorten non-embolic thrombus
Premorten non-thrombotic embolus
#
4
All following factors predispose to thrombosis EXCEPT one:
Injury of ultima
Turbulence blood flow
Stasis of blood
Hypercoagulability
@Fibrinolysis
#
5
What of the vascular wall components is a highly thrombogenic surface and plays a sufficient Jole in thrombogenesis?
Endothelium
@Subendothelium collagen
Muscular layer (media)
Adventitia
Elastic membranes
#
6
All following conditions contribute to the process of thrombosis EXCEPT one:
Stasis
Turbulence blood flow
Hypercoaculability states
Injury of vascular wall
@Hemodilution
#
7
At autopsy an elderly female is found to have a blood clot in the femoral artery, which grossly had a striped pattern.
Microscopy revealed congestions of fibrin fibers and broken down red and white cells. Which is the most likely type of
thrombus?
@Mixed thrombus
Postmortem convolutions of a blood
Thromboemboli
Hyaline thrombus
Red thrombus
#
8
A 62-year-old man has experienced substemal chest pain upon exertion with increasing frequency over the past 6 months.
At the moment he hospitalised with acute substemal pain with irradiation to left arm. An electrocardiogram shows
features of acute myocardium infarction. LDH level is increased. By angiography, there is complete occlusion of the left
anterior descending artery. Which of the following events is most likely to occur in this patient:
A systemic artery embolus from thrombosis in a peripheral vein.
A systemic artery embolus from a left atrial mural thrombus.
Pulmonary embolism from a left ventricular mural thrombus.
@Obstructive thrombosis of coronary artery superimposed to atherosclerotic plaque
Pulmonary embolism from thrombosis in a peripheral vein.
#
9
A neonate died of intoxication. A microscopic examination of an umbilical vein revealed a diffuse inflammatory
infiltration of the vessel's wall. Also, its lumen was filled with thrombus, rich with leucocytes and bacterial colonies.
Karyorrhexis was detected in many leucocytes. Which is most likely outcome of a thrombus in that case?
@Septic autolysis.
Aseptic autolysis.
Organization and vascularization of the thrombus.
Thromboembolism.
Calcification of a thrombus.
#
10
A 43-year-old patient has thrombopenia, reduction of fibrinogen, products of degradation of fibrin presented in the blood,
petechial haemorrhage along with septic shock. What is the most likely cause of the changes?
Autoimmune thrombocytopenia
@DIC-syndrom
Exogenous intoxication
Disorder of thrombocytes production
Haemorrhagic diathesis
#
11
A patient was ill with burn disease that was complicated by DIC syndrome. What stage of DIC syndrome can be
suspected if it is known that the patient's blood coagulates in less than 3 minutes?
Hypercoagulation
@Transition phase
Fibrinolysis
Terminal
Hypocoagulation
#
12
A patient underwent a surgery for excision of a cyst on pancreas. After this he developed haemorrhagic syndrome with
apparent disorder of blood coagulation. Development of this complication can be explained by:
Reduced number of thrombocytes
Activation of Christmas factor
Insufficient fibrin production
@Activation of fibrinolytic system
Activation of anticoagulation system
#
13
A 68-year-old female is admitted to the hospital for treatment of deep vein thromboses. The next day she suddenly died.
At autopsy a large plug of laminated blood clot is found to occlude the main pulmonary artery. Which is likely to be
found in her lungs?
@Thromboembolism
Thrombosis
Tissue embolism
Foreign body’s embolism
Fat embolism
#
14
A post-mortem of a 43-year-old female revealed multiple, hemorrhagic infarcts in lungs. Some lungs' vessels had reddish-
brown dense masses within lumens, which were not attached to the vessel walls. Varicose phlebectasia of the legs with
thromboses of some veins were also determined. Which pathological process occurred in this case?
@Thromboembolism of pulmonary artery.
Fat embolism of pulmonary artery.
Tissue embolism of pulmonary artery.
Foreign bodies embolism of pulmonary artery.
Gas embolism.
#
15
The gross appearance of thrombophlebities ofcatheterised subclavian vein is associated with:
Multiple lung infarction
@Systemic embolism
Isolated brain infarction
Myocardium infarction
Aneurysm formation
#
16
A 65-years-old patient suffered by thrombophlebitis of the deep veins of both legs has died suddenly. Autopsy was
showed free lying dry friable red masses with a dull crimped surface within the truncus pulmonalis and bifurcation of the
lung artery. What process within the vessels did pathologist find?
@Thromboembolism
Thrombosis
Tissue embolism
Postmortem clot
Hemangioma
#
17
Patient suffering from trombophlebitis of the deep veins suddenly died. Autopsy has shown freely lying red friable
masses with dim crimped surface in the trunk and bifurcation of the pulmonary artery. What pathologic process was
revealed by morbid anatomist?
Fat embolism
Thrombosis
Embolism with foreign body
@Tromboembolism
Tissue embolism
#
18
A patient suffering from trombophlebitis of deep veins suddenly died. The autopsy has shown freely lying red friable
masses with dim crimped surface in the trunk and bifurcation of the pulmonary artery. What pathologic process was
revealed by the morbid anatomist?
Thrombosis
Embolism with foreign body
@Tromboembolism
Fat embolism
Tissue embolism
#
19
A patient ill with thrombophlebitis of his lower limbs had chest pain, blood spitting, progressing respiratory insufficiency
that led to his death. Autopsy diagnosed multiple lung infarctions. What is the most probable cause of their development?
@Thromboembolism of pulmonary artery branches
Thrombosis of pulmonary veins
Thrombosis of bronchial arteries
Thrombosis of pulmonary artery branches
Thromboembolism of bronchial arteries
#
20
A patient with thrombophlebitis of lower extremities had got chest pains, blood spitting, growing respiratory failure that
caused his death. Autopsy revealed multiple pulmonary infarctions. What is the most probable reason of their
development?
@Pulmonary artery embolism
Pulmonary venous thrombosis
Bronchial artery thrombosis
Pulmonary artery thrombosis
Bronchial artery embolism
#
21
Following a fall down some stairs at work, a 64-year-old woman suffers a right hip fracture for which she is hospitalized.
She undergoes surgery for placement of a hip prosthesis. She is recuperating from the surgery when she suddenly
develops dyspnea and expires in only a few minutes despite resuscitative measures. Autopsy reveals a saddle pulmonary
embolus. A 1-cm infiltrating ductal carcinoma is found in the right breast, but there are no metastases. Which of the
following statements regarding death certification is most accurate in this case:
@The immediate cause of death is cardiopulmonary arrest
This is a therapeutic misadventure
The mode (manner) of death is accident
Breast carcinoma is a contributing cause of death
E. Pulmonary embolism is the underlying cause of death
#
22
A 62-year-old patient had a surgery due to acute appendicitis. She had been placed in a bed for 5 days. After she decided
to get out of bed she experienced a shortage of air, her face became cyanotic and she lost her consciousness. After
unsuccessful resuscitation the patient died. A post-mortem revealed a thromboembolism of the pulmonary artery. Which
of the following is most likely source of thromboembolism?
@Thrombosis of veins of the lower extremity
Thrombosis of a portal vein
Thrombosis of mesenteric arteries
Thrombosis in a left ventricle of heart
Ball-shaped thrombus of an auricle
#
23
A male patient died after a abdominal surgery. During the postmortem numerous thrombuses were found in the veins of
the pelvis. Thromboembolic syndrome was diagnosed. Where it is necessary to search for thromboembolics?
@Lungs artery
Portal vein
Left ventricle of heart
Brain
Veins of the lower extremity
#
24
A 72 year old man has congestive heart failure with decreased cardiac output due to chronic left ventricle aneurysm with
large mural thrombosis. All following statements concerning this state are true EXCEPT one:
This state may complicated by systemic embolism
The possible fatal result may associated with pulmonary embolism
@The possible fatal result may associated with brain infarction
The underlying cause of thrombosis is alteration of blood flow
#
25
During post mortem examination of the corpse 26 year old woman who died from infectious endocarditis of aortic valve
have been noted: multiple thrombotic vegetation on the semilunar aortic valves, infarction of the brain, spleen, kidney and
gangrene of the right foot. In the both lung there are prominent infarction with abscess (focal suppurative inflammation)
formation. Blood culture revealed Staphylococcus aureus. What condition is underlies all these complications:
Systemic embolism
@Systemic thrombobacterial embolism
Pulmonary embolism
Bacterial embolism
#
26
All following statements are associated with pulmonary embolism EXCEPT one.
The ultimate result depends on the calibre of Ihromboembolus
The possible cause of thrombosis is phlebothrombosis of deep veins
Prolonged thromboembolism leads to pulmonary hypertension
@The manifestation of thromboembolism oftruncus pulmonale are multiple infarction of lung
The tromboembolus obstruction of small branches of pulmonary artery results in haemorrhage infarctions
#
27
A patient has a purulent otitis and thrombosis of a sigmoid sinus of a dura mater. Name the complication if thrombus
breaks down and fragments circulate to distal vessels.
@Thromboembolism of branches of pulmonary artery
Thromboembolism of vessels of a brain with development of a grey softening of the brain
Thromboembolism of vessels of a retina of an eye and development of blindness
Thromboembolism of coronary arteries
Local anemia
#
28
A young male fractures his pelvis and femur in a motor vehicle accident. On the third day he dies after respiratory distress
and cerebral dysfunction. A histological examination revealed Sudan-positive orange globules scattered within cerebral
cortex, kidneys and lung microcirculatory vessels. Which are most likely diagnoses?
@Fat embolism
Gas embolism
Tissue embolism
Microbial embolism
Thromboembolism
#
29
A 43-year-old male, with a history of traffic accident and poly trauma, including multiple bone fractures, presented to the
hospital. On the second day after the trauma, he complained of a pain in the right half of the chest, heavy breathing. At
night, he died due to progressive heart and respiratory failure. Microscopic investigation revealed Sudan-positive orange
drops in pulmonary and cerebral vessels that completely occlude the lumens of microcirculatory vessels. What is the most
likely complication led to a patient's death?
@Fat embolism.
Gaseous embolism.
Medicamentous embolism.
Microbial embolism.
Thromboembolism.
#
30
The pulmonalis embolism has suddenly developed in a 40 year-old patient with opened fracture of the hip. Choose the
possible kind of embolism.
@Fat
Air
Thrombus-embolus
Tissue
Foreign body
#
31
A saddle- rider in a rodeo who was first crushed into the gate, then thrown from the horse, then stepped on and kicked by
the horse, then dropped from the stretcher by the paramedics carrying him out of the arena, sustained right femoral and
left humeral fractures. Two days later he developed respiratory difficulty, most likely due to:
Vasculitis
Pulmonary thromboembolism
Aspiration of gastric contents
@Fat embolism
Pulmonary contusion
#
32
A 19-year-old offensive tackle for a major university football team fractures his right femur during the first game of the
season. He is admitted to the hospital and over the next several days develops progressive respiratory problems. Despite
extensive medical intervention, he dies three days later. At the time of autopsy oil red O-positive material is seen in the
small blood vessels of the lungs and brain. Which of the following is the most likely diagnosis?
Air emboli
Amniotic fluid emboli
@Fat emboli
Paradoxical emboli
Saddle emboli
#
33
A 29-year-old man, who had a comminuted fracture of a right femur, presented on the third day after the event with
complains of a pain in his chest, hard breathing. In a day he died because of progressive heart insufficiency. Histological
examination showed sudanophylic orange drops that closed lumens of microcirculatory vessels in lungs and cerebrum.
What complication is associated with the death of the man?
@Fat embolism
Gaseous embolism
Medicine embolism
Bacillary embolism
Thromboembolism
#
34
2 hours after a skeletal extension was performed to a 27 year old patient with multiple traumas (closed injury of chest,
closed fracture of right thigh) his condition abruptly became worse and the patient died from acute cardiopulmonary
decompensation. Histological examination of pulmonary and cerebral vessels stained with Sudan III revealed orange
drops occluding the vessel lumen. What complication of polytrauma was developed?
@Fat embolism
Thromboembolism
Gaseous embolism
Microbal embolism
Air embolism
#
35
A patient with severe fracture of both femoral bones has died in a 4 hours after trauma. What additional staining of
microscopic sections is it necessary to make for exacter diagnosis of the cause of death?
@The lungs and brain by Sudan III
The lungs and kidney by Congo-red
The lungs and liver by Perl’s reaction
The brain and kidney by Congo-red
The kidney and heart by toluidin-blue
#
36
A male patient involved in the traffic accident received a wound in the neck due to broken glass. The bleeding was small,
but a short time after the accident he died of acute dyspnea. A post-mortem of the heart revealed bubbles excretion when
pericardium filled with water. Indicate which pathological process took place:
@Air embolism
Gas embolism
Fat embolism
Thromboembolism
Foreign body’s embolism
#
37
A male suddenly died after an open fracture of the clavicle. A post-mortem revealed in the right ventricle of the heart and
pulmonary arteries the foamy blood. Which one was the cause of death?
@Air embolism
Bacterial embolism
Hemorrhage
Tissue embolism
Fat embolism
#
38
A cosmonaut died due to the air-tightless of the space-shuttle cabin. Microscopic examination of the vessels of the
internal organs revealed multiple bubbles. Liver cells developed fatty dystrophy. In the brain and spinal cord multiple
small, grey, soft lesions were discovered. Indicate the most probable cause of such alterations.
@Gas embolism
Air embolism
Fat embolism
Thromboembolism
Tissue embolism
#
39
During a histological examination of a pilot, who died after depressurization of a plane’s cabin, a great amount of vesicles
were detected in the vessels of internal organs. There were also dystrophia adiposa hepatis and multiple, small, ischemic,
gray, softened focuses in cerebrum and spinal cord. Select the most possible reason of such changes.
Fat embolism
Aeroembolism
@Gaseous embolism
Thromboembolism
Tissue embolism
#
40
A pilot has died because of depressurization of the cabin of the airplane. In autopsy it was found out a gross marked rigor
mortis, emphysema of subcutaneous tissue of the body and face. There were also edema and perivascular hemorrhages
within the lungs. There was a foamy and fluid blood within the veins. The histological examination was showed a lot of
bubbles within the vessels of all inner organs, a fat dystrophy of the liver and a quantity of small gray softening areas in
the brain. Call the more probable cause of those changes.
@Gas embolism
Air embolism
Tissue embolism
Thromboembolism
Fat embolism
#
41
A diver who has been staying at the depth of 40 m for a long time fell ill with caisson disease as a result of
decompression. The main pathogenetic factor is the following embolism:
Paradoxical
@Gaseous
Tissue
Air
Fat
#
42
On a trip to the Caribbean, a 29-year-old male takes a scuba diving course. He goes on a wreck dive to a depth of 50
meters (170 feet). His diving partner signals to him that it is time to ascend, but he continues to explore the wreck,
noticing some time later than he is running low on air. He panics and ascends to the surface quickly- Within a couple of
hours he then develops a headache, tightness in his chest with difficulty breathing, and generalized pain in all extremities.
The mechanism for these findings is the result of:
Disseminated intravascular coagulopathy
Hypothermia
Free radical cellular injury
Apoptosis
@Nitrogen bubble formation
#
43
A male with a history of the decompression sickness developed symptoms of acute cerebral circulation disorder and died
soon. A post-mortem revealed in the left hemisphere of the brain the locus of a grey softening of the brain, which was
6x5x3 cm. Which one is most likely to cause the death of the patient?
@Gas embolism.
Fat embolism.
Thrombosis.
Thromboembolism.
Atherosclerosis of the vessels.
#
44
An inexperienced scuba diver ascends from a depth of 55 meters to the surface in about 5 minutes. Shortly after surfacing
he complains of severe muscle contraction and intense abdominal pain. What is most likely diagnosed?
@Gas embolism
Air embolism
Fat embolism
Thromboembolism
#
45
23-year old female during the course of labour suddenly feels chills, nausea, and vomiting and profound respiratory
difficulty. Medical examination revealed deep cyanosis and signs of cardiovascular shock. Few hours later the lung
edemas with respiratory failure have been developed. The hypercoagulability state was diagnosed. Later they follows by
hypocoagulability with massive bleeding from uterus.What are the name of this potentially fatal labour complication:
Pulmonary embolism due to thrombosis of parametric veins
Fat embolism due to damage of birth canal
@Amniotic fluid embolism due to tear of the placental membranes
Acute myocardial infarction due to systemic embolism
#
46
The pregnancy for a 27-year-old woman was uncomplicated, with an ultrasound at 19 weeks gestation demonstrating no
abnormalities. She goes into labour and 6 hours later has a routine vaginal delivery of a term male. An intact placenta is
delivered a few minutes later. However, within a few minutes she develops severe dyspnea and cyanosis, followed by
seizures and coma. The most likely explanation for this series of events is:
Disseminated intravascular coagulation
Metastatic choriocarcinoma
Cerebral infarction
@Amniotic fluid embolism
Pulmonary infarction
#
47
A 29 years old patient has obtained multifragmental thigh bone fracture. Three days after he gained itching in right part of
thorax and difficult breathing. One day after he died in the background of progressive cardio-respiratory failure.
Histological research revealed sudanophilic drops of orange colour, that clotted the lumen of microcirculatory vessels.
What complication caused death of the patient?
@Oil embolism
Gaseous embolism
Medicine embolism
Bacterial embolism
Thromboembolism
#
48
Inflammation is characterised by increasing penetration of vessels of microcirculation stream, increasing of their fluid
dynamic blood pressure. Increasing of the osmotic concentration and dispersity of protein structures present in the
intercellular fluid. What kind of oedema will appear in this case?
Hydrodynamic
Lymphogenic
Membranogenic
Colloid-osmotic
@Mixed
#
49
The concentration of albumins in human blood sample is lower than normal. This leads to edema of tissues. What blood
function is damaged?
Maintaining the body temperature
@Maintaining the oncotic blood pressure
Maintaining the blood sedimentation system
Maintaining the Ph level
All answers are correct
#
50
Laboratory rats that have been fed only with carbohydrate food for a long time display water accumulation in the tissues.
What pathogenetic mechanism is the main cause of edema in this case?
Disregulatory
@Hypooncotic
Hyperosmolar
Lymphogenic
Membranogenic
#
51
Chronic glomerulonephritis was diagnosed in a 34-year-old patient 3 years ago. Edema has developed in the last 6
monthes. What caused it?
Hyperaldosteronism
Hyperproduction of vasopressin
Hyperosmolarity of plasma
Disorder of albuminous kidneys function
@Proteinuria
#
52
A 9-year-old boy suddenly develops severe testicular pain. He is taken to the emergency room, where he is evaluated and
immediately taken to surgery. There his left testis is found to be markedly hemorrhagic due to testicular torsion. Which of
the following mechanisms is primarily involver in producing this type of testicular infarction?
Arterial occlusion
Septic implantation
Decreased collateral blood flow
Increased dual blood flow
@Venous occlusion
#
53
Toxic pulmonary edema was reproduced on a laboratory rat by means of ammonium chloride solution. What is the
leading pathogenetic factor of this edema?
Decrease of colloid osmotic pressure
Increase of lymph outflow
Disorder of neural and humoral regulation
Increase of venous outflow
@Increased permeability of capillars
#
54
A patient was stung by a bee. Examination results: his left hand is hot, pink and edematic, there is a big blister on the spot
of the sting. What is the leading mechanism of edema development?
Reduced blood filling of vessels
Vascular damage caused by the sting
@Increased vascular permeability
Reduction of osmotic pressure of tissue
Reduction of oncotic pressure of tissue
#
55
A 56-year-old patient with cardiac failure notices edematous feet and shins, the skin in the place of edema is pale and
cold. What is the leading pathogenesis of this patient's edema?
@Increased hydrostatic pressure in venulaes
Reduced oncotic pressure in capillaries
Lymph outflow impairment
Increased capillary penetration
Positive water balance
#
56
Which of the following changes best describes the pathophysiology involved in the production of pulmonary edema in
patients with congestive heart failure?
Decreased plasma oncotic pressure
Widespread endothelial damage
@Increased hydrostatic pressure
Increased vascular permeability
Acute lymphatic obstruction
#
57
The patient with acute miocardial infarction was given intravenously different solutions during 8 hours with medical
dropper 1500ml and oxygen intranasally. He died because of pulmonary edema. What caused the pulmonary edema?
Inhalation of the oxygen
Decreased oncotic pressure due to hemodilution
Neurogenic reaction
Allergic reaction
@Volume overload of the left ventricular
#
58
A patient aged 59 was hospitalised to cardiological department in a sever state, with diagnosis of acute myocardial
infarction of the posterior wall of the left ventricle and septum, and primary pulmonary edema. What is the primary
mechanism, which causes the development of pulmonary edema in the patient?
@Left ventricular failure
Pulmonary arterial hypertension
Pulmonary venous hypertension
Hypoxemia
Decrease of alveolocapillary diffusion of oxygen
#
59
Transmural myocardial infarction in the patient was complicated with progressive acute left ventricle insufficiency. What
is the most typical for this state?
Edema of the extremities
Cyanosis
@Edema of the lungs
Arterial hypertension
Ascites
#
60
A 45 year old woman is ill with breast cancer. Her left arm has symptoms of lymphatic system insufficiency - limb
edema, lymph node enlargement. What form of lymphatic circulation insufficiency is it?
Resorption insufficiency
Combined insufficiency
-
Dynamic insufficiency
@Mechanic insufficiency
#
61
A 55-year-old man with a history of ischemic heart disease has worsening congestive heart failure. He has noted
increasing dyspnea and orthopnea for the past 2 months. On physical examination there is dullness to percussion at lung
bases. A chest x-ray shows bilateral pleural effusions. A left thoracentesis is performed, and 500 mL of fluid is obtained.
Name this fluid?
Exudate
@Transsudate
Pus
Phlegm
Liquvor
#
62
A 52-year-old woman with no major medical problems takes a long airplane flight. Upon arrival at Sydney's Kingsford
Smith airport in New South Wales following the flight from Los Angeles, California, she cannot put her shoes back on.
Which of the following is the most likely explanation for this phenomenon?
Activation of Hageman factor has led to bradykinin production.
A lot of drinks were served in the first class section.
She didn't take any aspirin.
A cellulitis developed in her legs.
@Venous hydrostatic pressure was increased.
#
63
Within minutes following a bee sting, a 37-year-old man develops marked difficulty breathing with dyspnea and
wheezing. He also develops swelling and erythema seen in his arms and legs. An injection of epinephrine helps to reverse
these events and he recovers within minutes. Name the type of oedema:
Hydrostatic
Oncotic
Lymphatic
@Allergic
Kidneys’
#
64
A 19-year-old woman is in the early second trimester at 16 weeks gestation with her first pregnancy. On a routine prenatal
visit, the nurse cannot detect fetal heart tones and an ultrasound reveals no fetal cardiac activity or fetal movement. The
mother has noted no recent illness or other factors to account for this fetal demise. She is given a pharmacologic agent to
induce abortion. By which of the following mechanisms is this induction most likely to be accomplished?
Leukotriene-induced neutrophil aggregation
Complement-induced hemolysis
Histamine-induced edema
@Endotoxin-induced disseminated intravascular coagulation
Prostaglandin-induced muscular contraction
#
65
A pleural effusion is tapped in a 55-year-old man who has congestive heart failure. Upon examination of the fluid
obtained, which of the following is most characteristic of a transsudate:
Cloudy appearance
High protein content
@A few lymphocytes
The presence of fibrin
Larger size of the effusion
#
66
Lymphatics perform all of the following functions EXCEPT:
@Carry lymphocytes from lymph nodes into tissues
Drain off excess tissue fluid
Aid in resolution of inflammation
Serve as a route for dissemination of infection
Connect to lymph nodes
#
67
A 40-year-old man incurs a burn injury to his hands and arms while working on a propane furnace. Over the next 3
weeks, the burned skin heals without the need for skin grafting. The most critical factor in determining whether the skin
in the region of the burn will regenerate following this is the presence of which of the following factors?
Good cardiac output
Skin appendages
Underlying connective tissue
@Minimal oedema and erythema
Granulation tissue
#
68
A 60-year-old man who has a 90 pack year history of cigarette smoking has had a chronic cough for the past 10 years. He
has begun to lose weight during the past year. No abnormal findings are noted on physical examination. He has a chest
radiograph that reveals a right hilar mass. A sputum cytology shows atypical, hyperchromatic squamous cells. Name the
pathological process in pleural cavity.
Hydrothorax
Oedema
Pleuritis
Haemothorax
@Hilothorax
#
69
On sectioning of an organ from a 60-year-old man at the time of autopsy, a focal, wedge-shaped area that is firm is
accompanied by extensive hemorrhage, giving it a red appearance. The lesion has a base on the surface of the organ. In
which of the following situations will this lesion most likely occur?
@Lung with pulmonary thromboembolism
Heart with coronary thrombosis
Liver with hypovolemic shock
Kidney with septic embolus
Spleen with embolized mural thrombus
Brain with cerebral arterial aneurysm
#
70
A 59-year-old woman has the sudden onset of severe dyspnea and goes into cardiac arrest, from which she cannot be
resuscitated. At autopsy, she has the gross finding of a saddle pulmonary embolus. This event is most likely to be present
as a consequence of which of the following?
@Placement of a hip prosthesis
Marked thrombocytopenia
Chronic alcoholism
Infection with the human immunodeficiency virus
An autoimmune disease
#
71
Grandma falls down the steps leading to the entrance of the house of a relative hosting a family reunion, who is heard to
remark, "I've been meaning to get that loose step fixed." Grandma is hospitalized for surgery to replace the broken hip she
sustains and is then moved to a nursing home, but she is unable to ambulate until about a month later, when she dies
suddenly. Which of the following is most likely to be the immediate cause of death found at autopsy?
Squamous cell carcinoma of lung
Tuberculosis
@Pulmonary embolism
Pneumonia with pneumococcus
Congestive heart failure
#
72
A 68-year-old woman has survived multiple episodes of pulmonary thromboembolism during the past three months.
Which of the following is the most likely underlying condition leading to this patient's recurrent pulmonary
thromboembolism?
Micronodular cirrhosis of the liver
Adenocarcinoma of the pancreas
Familial hypercholesterolemia
@Mitral valve endocarditis
Type II diabetes mellitus
#
73
During hospitalization, a 40-year-old woman develops thrombophlebitis. She recovers and is discharged. She returns to
her job as an electrician. A couple of months later, which of the following terms would best describe the process seen in a
femoral vein after recovery from her thrombophlebitis:
Acute inflammation
Rupture
Embolization
@Organization
Propagation
#
74
A 52-year-old woman has a history of urinary tract infections. Recently, one of these episodes was complicated by acute
pyelonephritis involving her kidneys. She became septic, and a blood culture grew Escherichia coli. She developed severe
hypotension. She had purpuric areas on her skin. A stool for occult blood was positive. She had a prothrombin time of 50
sec (control 12), partial thromboplastin time of 100 sec (control 25), platelet count of 20,000/microliter, and D-dimer of 4
microgm/mL These findings are most characteristic for which of the following conditions:
Hemophilia A
Von Willebrand disease
@Disseminated intravascular coagulation
Antiphospholipid syndrome
Acute fulminant hepatitis
#
75
A patient who suffers from heart failure has enlarged liver, edemata of lower extremities, ascites. What is the leading
mechanism in the development of this edema?
Lymphogenous
-
@Hydrodynamic
Colloid osmotic
Membranogenic
#
76
Inflamation is characterised by increasing penetration of vessels of microcirculation stream, increasing of their fluid
dynamic blood pressure. Increasing of the osmotic concentration and dispersity of protein structures can be found in the
intercellular fluid. What kind of edema are to be observed in this case?
Membranogenic
Lymphogenic
@Mixed
Colloid-osmotic
Hydrodynamic
#
INFLAMMATION
Exudative inflammation
#
1
Choose the most correct definition of inflammation:
An adaptive process for restoration structural constituents of tissue
@Nonspecific complex reaction of vascularized connective tissue as a response on tissue damage
Specific reaction an organism directed on detection, killing and elimination of foreign substances
Pathologic process based on disturbance of tissue (cell) metabolism that leads to structural changes
#
2
Necrosis focus appeared in the area of hyperemia and skin edema in few hours after burn. What mechanism strengthens
destructive events in the inflammation area?
Proliferation of fibroblasts
@Secondary alteration
Primary alteration
Emigration of lymphocytes
Diapedesis of erythrocytes
#
3
Choose 2 leading mechanism of development of edema due to acute inflammation:
@Increased vascular permeability
Toxic injury
Lymphatic obstruction
Salt retention
Increase renin synthesis
Increase aldosterone synthesis
#
4
Choose 2 leading mechanism of development of edema due to acute inflammation:
Toxic injury
@Decrease oncotic pressure
Lymphatic obstruction
Salt retention
Increase renin synthesis
Increase aldosterone synthesis
#
5
A 34 year old woman due to incorrect usage of iron gained acute pain, plethora and swelling on her right index. Few
minutes later a bladder occurred, filled with transparent of yellow colour. What kind of pathological process is this?
@Exudative inflammation
Traumatic oedema
Alterative inflammation
Proliferative inflammation
Vacuole dystrophy
#
6
A 40-year-old woman, with a history of the right palm's burn, presented to her physician an acute pain, reddening and
swelling of a palm. In a few "minutes there was a bubble, filled with transparent yellowish liquid. The display of what
pathological process the described changes are?
@Exudative inflammation
Traumatic edema
Alterative inflammation
Proliferative inflammation
Vacuolar dystrophy
#
7
A man complains of itch and redness of the skin in the buccal area that appeared after shaving. Vesicles, which are filled
with bright fluid, can be observed on the hyperemia buccal area. What kind of fluid is in vesicles?
Haemorrhagic exudate
Transudate
Purulent exudate
Mucous exudate
@Serous exudate
#
8
A 15-year-old girl has had episodes of sneezing with watery eyes and runny nose for the past 2 weeks. On physical
examination she has red, swollen nasal mucosal surfaces. She has had similar episodes each Spring and Summer when the
amount of ragweed pollen in the air is high. To which kind of inflammation her symptoms are most likely to be?
@Serous
Fibrinous
Haemorrhagic
Putulent
Mixed
#
9
In some minutes after bite of the bee. There were swelling, redness, local increase of the temperature, pain. What
morfological process is the base of this changes?
@Serouse inflammation
Dystrophy and necrosis
Proliferative inflammation
Transsudation
Haemorrhagic inflammation
#
10
A 43-years-old patient has a burn of right hand. The exfoliation of epidermis and formation of bubbles filled by semi-
transparent fluid is manifested in the palm and the back surface of the hand. What kind of inflammation occurs in that
case?
@Serous
Purulent
Fibrinous
Putrefactive
Catarrhal
#
11
A woman, with a history of her hands' skin thermal burn, presented to physician painful blisters, filled with opaque liquid.
What is the most likely type of inflammation?
@ Serous inflammation
Productive inflammation
Croupous inflammation
Granulomatous inflammation
Diphtheritic inflammation
#
12
A patient presented to the hospital with combustion of his right hand. A physical examination revealed a desquamation of
epidermis with blisters formation. The blisters were filled with opaque liquid. What most probable inflammation
described in that case?
@ Serous
Purulent
Catarrhal
Putrefactive
Fibrinous
#
13
Autopsy of a man who died from influenza revealed that his heart was slightly enlarged, pastous, myocardium was dull
and had specks. Microscopical examination of myocardium revealed signs of parenchymatous adipose and hydropic
dystrophy; stroma was edematic with poor macrophagal and lymphocytic infiltration, vessels were plethoric; perivascular
analysis revealed petechial hemorrhages. What type of myocarditis was developed in this case?
Granulomatous
Purulent
Interstitial proliferative
@Serous diffuse
Serous focal
#
14
At study of a blood smear of the person with presence of inflammatory process it is possible to see a great number of
spherical cells with a segmented nucleus, acidophilic cytoplasm and shallow pink-violet granules in the cytoplasm. What
blood cells are these?
@Neutrophils
Erythrocytes
Eosinophils
Basophils
Lymphocytes
#
15
All following substances are leukocytes attractants (chemotactic agents) EXCEPT:
Bacterial products
Components of complement system
Antivirus antybody
IgG
@Arachidonic acid metabolites
#
16
Choose the best explanation of neutrophilic leukoeytes predominance in the composition of inflammatory exudate:
@Presence of effective systems of enzyme lysis
Ability to endocytobiosis
Speed of moving
Presence of certain receptors on the surface of membrane
Ability to create pseudopodia
#
17
Opsonins that aid in recognition and attachment of leukocytes to bacteria include all of the following features EXCEPT:
They may be derived from the C3b fragment of serum complement.
The leukocytes have an Fc receptor.
Binding of Opsonins triggers engulfinent
@Leukocyte adhesion molecules are needed.
Immunoglobulin G plays a role.
#
18
A small sliver of wood becomes embedded in the finger of a 25-year-old man. He does not remove it, and over the next 3
days the area around the sliver becomes red, swollen, and tender. Neutrophils migrate into the injured tissue. What kind
of inflammation is this one?
Serous
Fibrinous
Haemorrhagic
@Putulent
Mixed
#
19
An autopsy of a dead body revealed 200 ml of a viscid yellow-green liquid in the abdominal cavity. What is the most
likely form of exudate inflammation?
@ Purulent inflammation
Serous inflammation
Fibrinous inflammation.
Hemorrhagic inflammation.
Ichorous inflammation
#
20
A 45-year-old man has had a fever and dry cough for 3 days, and now has difficulty breathing and a cough productive of
sputum. On physical examination his temperature is 38.5 C. Diffuse rales are auscultated over lower lung fields. A chest
radiograph shows a right pleural effusion. A right thoracentesis is performed. The fluid obtained has a cloudy appearance
with a cell count showing 5500 leukocytes per microliter, 98% of which are neutrophils. Which of the following terms
best describes his pleural process?
Serous inflammation
@Purulent inflammation
Fibrinous inflammation
Chronic inflammation
Granulomatous inflammation
#
21
During the operation of revision of the abdominal cavity a surgeon noticed, that the leaves of the peritoneum on the
entire area are diffusely bulged, plethoric and dingy. 500 ml of muddy yellow-green liquid was detected in the abdominal
cavity. This morphology describes:
@Empyema
Phlegmon
Abscess
Incrust
Purulent-hemorrhagical inflammation
#
22
A 37-year-old male presents with a fever, dyspnea and pain in the right part of the chest. The pleurocentesis gave 700 mis
of thick yellow-greenish liquid. What pathological process was diagnosed in a pleural cavity?
@Empyema of pleura
Bronchopneumonia
Serous pleuritis
Hemorrhagic pleuritis
Carcinomatosis of pleura
#
23
A patient has high body temperature, breathlessness, pain in his right part of thorax. During pleural puncture there was
700ml of cream-like, yellowish-green coloured liquid extracted. What is the most probable diagnosis?
@Pleural empyema
Pleural carcinomatosis
Serous pleuritis
Fibrinous pleuritis
Haemorrhagic pleuritis
#
24
An autopsy of a 53-year-old male, with a history of crupous pneumonia, revealed in his dextral pleural cavity 900 ml of
cloudy, grey-yellowish colored liquid. Pleural membranes were found to be dim and plethoric. Name the clinical -
morphological form of the inflammation in the pleural cavity?
@ Empyema.
Fibrinous inflammation.
Phlegmon.
Chronic abscess.
Acute abscess.
#
25
A female patient presented to the hospital with a fever, asphyxia and pain in the right part of her chest. During a
pleurocentesis procedure 700 ml of a viscid yellow-green liquid was removed. What is the most likely diagnosis?
@Empyema of a pleura
Carcinomatosis of a pleura
Serous pleurisy
Fibrinous pleurisy
Hemorrhagic pleurisy
#
26
The patient complains of a high temperature, dyspnoae, and pain in the right part of the thorax. During the pleural
puncture, 700 ml of yellow-green colored creamy fluid was evacuated. Make the right diagnosis.
Hemorrhagic pleurisy
Carcinomatosis of the pleura
Serous pleurisy
Fibrinous pleurisy
@Pleural empyema
#
27
Autopsy of a patient who suffered from croupous pneumonia and died from pneumococcal sepsis revealed 900 ml of
turbid greenish-yellow liquid in the right pleural cavity. Pleural leaves are dull, plephori c. Name the clinicopathological
form of inflammation in the pleural cavity:
Fibrinous inflammation
Chronic abscess
Acute abscess
@Empyema
Phlegmon
#
28
Thoracentesis is performed on a 45-year-old man who has had a fever and cough for several days, and now has more
difficulty breathing and a cough productive of sputum. A right pleural effusion is seen by chest radiograph. The fluid
obtained has a cloudy appearance with a cell count showing 5500 leukocytes per microliter, 98% of which are
neutrophils. The best description of the pleural process is:
Serous inflammation
@Purulent inflammation
Fibrinous inflammation
Chronic inflammation
Granuloreatous inflammation
#
29
A 68-year-old man died from meningitis. An autopsy revealed thickened, congested, dim, edematous meninges (pia
maters), which were saturated by creamy greenish-yellow exudate. Define the type of inflammation?
@Purulent
Serous
Fibrinous croupous
Fibrinous diphtheritic
Hemorrhagic
#
30
A 4-year-old child presented to his physician with a cold, high temperature, nausea and vomiting. He was hospitalized
and died in two days. An autopsy revealed thickened, dim, edematous, congested, greenish-yellow pia mater of meninges.
What variety of exudative inflammation developed in pia maters?
@Purulent
Catarrhal
Hemorrhagic
Fibrinous
Serous
#
31
A post-mortem of young male revealed congested, thickened, opaque, yellowish-green meninges. What type of exudative
inflammation these changes are characteristic for?
@ Purulent inflammation
Serous inflammation
Hemorrhagic inflammation
Fibrinous inflammation
Catarrhal inflammation
#
32
A 9 year old child suddenly fell ill and died. An autopsy has revealed a cerebral swelling, cerebral covers are intensively
plethoric and saturated with viscous dull yellowish-green exudate. What kind of pathological process is this?
@Purulent inflammation
Serous inflammation
Fibrinous inflammation
Haemorrhagic inflammation
Mixed inflammation
#
33
A patient died with symptoms of the brain's edema and the dislocation of the stem. A post-mortem revealed thickened,
opaque, grayish-green color pia mater. Which pathological process took place in pia mater of meninges?
@Diffuse purulent leptomeningitis
Meningococcal leptomeningitis
Tuberculous leptomeningitis
Malignant arachnoendofhelioma
Hydrocephalus
#
34
An elderly man presented with a decomposition of some segments of spongy and cortical layers of an anticnemion bones.
The cavities are filled with creamy greenish-yellow masses. What is the type of inflammation?
@Purulent
Catarrhal
Serous
Proliferative
Mixed
#
35
Several areas of spongy and cortex layers of tibia are decaying. Cavities are filled with cream-like masses of greenish-
yellow colour. What is the most probable form of inflammation?
@Purulent
Catarrhal
Serous
Proliferative
Mixed
#
36
A patient with periodontitis has his subcutaneous fat cellulose of bottom of oral cavity is diffusely soaked with greenish-
yellow viscous liquid. What kind of exudative inflammation is this?
@Purulent
Serous
Catarrhal
Haemorrhagic
Fibrinous
#
37
Autopsy of a man who died from sepsis revealed a phlegmonous inflammation in the femoral bone of lower extremity.
The inflammation was seen in the bone marrow, haversian canals and periosteum. There were also multiple abscesses
underneath the periosteum; the surrounding soft tissues of the thigh were also affected by the phlegmonous inflammation.
What pathological process is it?
Osteopetrosis
Chronic haematogenous osteomyelitis
Osteoporosis
@Acute haematogenous osteomyelitis
-
#
38
The patient complains of local pain in the back of the head, increasing of the temperature in that area. Macroscopically:
there is a conical infiltrate of purple-blue colour with yellow-greenish apex, which protrudes under the surface of the skin.
Diagnose it:
@Furuncle
Phlegmon
Abscess
Fistula
Empyema
#
39
A 44-year-old man presented to the policlinic with a local pain in occipital area of a head and rise of a body temperature
there. Gross investigation of a lesion zone revealed the cone-shaped cyanotic infiltration with a yellow-greenish apex,
which rose above the surface of skin. What is the most likely diagnosis?
@Furuncle
Phlegmon
Abscess
Carbuncle
Empyema.
#
40
Appendix, sent to the pathological department after appendectomia, is intumescenced and enlarged in seizes, serous
mucous membrane is dingy, vessels are plethoric, yellow-green fluid discharges from the lumen of the appendix on the
cut. In which form of the appendicitis do such changes develop?
@Phlegmonous appendicitis
Apostematous appendicitis
Simple catarrhal appendicitis
Superficial catarrhal appendicitis
Gangrenous appendicitis
#
41
To a histological research there was sent a removed appendix vermiformis. Its size is enlarged, serous tunic is dull,
plethoric and is covered by films of fibrin; walls are thickened, on incision there is pus coming out of it. Name the form
of appendicitis.
@Phlegmonous
Apostematous
Simple
Superficial
Gangrenous
#
42
A pathology-histology laboratory received a vermiform appendix up to 2,0 cm thick. Its serous membrane was pale, thick
and covered with yellowish-green films. The wall was flaccid, of grayish-red colour. The appendix lumen was dilated and
filled with yellowish-green substanc e. Histological examination revealed that the appendix wall was infiltrated with
neutrophils. Specify the appendix disease:
Acute superficial appendicitis
Acute simple appendicitis
Acute gangrenous appendicitis
Chronic appendicitis
@Acute phlegmonous appendicitis
#
43
An operatively removed appendix was sent for histological examination. Macroscopical investigation revealed thickened
appendix. His serous membrane was dim, congested, with whitish, loose membranes; the lumen contained turbid, whitish-
yellow exudate. Histological study showed the diffuse neutrophil's infiltration of the appendix wall. Choose the most
likely type of appendicitis.
@Phlegmonous
Gangrenous
Simple
Superficial
Chronic
#
44
An operatively removed appendix was sent to pathology department. Macroscopical investigation revealed markedly
enlarged appendix. His serous membrane was dim, congested, covered by fibrin's membranes. The appendix' walls were
thickened. On a cut section pus in the lumen was detected. Microscopical investigation showed a plethora of vessels,
edema of all appendixes' layers and diffuses infiltration by leucocytes. What is the type of inflammation?
@Phlegmonous
Apostematous
Simple
Superficial
Gangrenous
#
45
A 22-year-old man develops marked right lower quadrant abdominal pain over the past day. On physical examination
there is rebound tenderness. Laparoscopic surgery is performed, and the appendix is swollen, erythematous, and partly
covered by a yellowish exudate. It is removed, and a microscopic section shows infiltration with numerous neutrophils.
Name the type of inflammation:
Serous
Fibrinous
@Purulent
Cataral
Mucous
#
46
A 16-year-old boy was performed an appendectomy. He has been hospitalized for right lower quadrant abdominal pain
within 18 hours. The surgical specimen is edematous and erythematous. Infiltration by what of the following cells is the
most typical for the process occuring here?
Basophils
Monocytes
Limphocytes
Eosinophils
@ Neutrophils
#
47
Microscopical examination of a removed appendix revealed an edema, diffuse neutrophilic infiltration of appendix wall
along with necrosis and defect of mucous membrane with affection of its muscle plate. What appendicitis form was
developed?
Phlegmonous
@Ulcerophlegmonous
Gangrenous
Superficial
Apostematous
#
48
An elderly man, with a history of an operation, presented with a sepsis, developed on a background of a decreased
immune reactivity of an organism. A disease resulted in fatal outcome. Microscopical investigation of the abdominal wall
revealed the diffuse segmentonuclear leukocyte's infiltration of intermuscular spaces, edema of a tissue and lysis of
muscle fibers. Define the type of inflammation?
@ Phlegmon.
Diphtheroid inflammation.
Abscess.
Necrosis.
Catarrhal inflammation.
#
49
A 13-years-old child was admitted into the hospital with complaints of fever, weakness, and intense pain in the right
thigh. From anamnesis it was defined that he has bitten by dog 3 days ago. During the visual examination it was found out
redness, swelling and acute painfulness along right thigh. After excision it was made the histological examination, which
was showed a considerable accumulations of polymorphonucleus leukocytes between the mussel fibers partly undergone
by purulent fusion. Diagnose that condition.
@Phlegmon
Abscess
Gangrene
Empyema
Croupous inflammation
#
50
After an operative treatment of a patient (burn disease), in a condition of severe the organism reactivity decreasing, a
progressive sepsis led to his death. Microscopically: in the area of the anterior abdominal wall the diffuse infiltration by
segmented leucocytes of the intramuscularly interval was observed. There were also tissue edema and lyses of muscle
fibrils. Define the kind of inflammation.
Diphtheritic
@Phlegmon
Abscess
Necrosis
Catarrhal
#
51
A 24-year-old primigravida is late in the second trimester of pregnancy. She experiences the sudden onset of some
cramping lower abdominal pain. This is immediately followed by passage of some clear fluid per vagina along with a
foul-smelling discharge. The fetus is stillborn two days later. Examination of the placenta demonstrates extensive
neutrophilic infiltrates in the chorion and amnion. Which kind of inflammation is presented in plasenta?
Serous
Fibrinous
Haemorrhagic
@Purulent
Mixed
#
52
A 22-year-old man develops marked right lower quadrant abdominal pain over the past day. On physical examination
there is rebound tenderness on palpation over the right lower quadrant. Laparoscopic surgery is performed, and the
appendix is swollen, erythematous, and partly covered by a yellowish exudate. It is removed, and a microscopic section
shows infiltration with numerous neutrophils. Name the tape of inflammation.
Serous
Diphteroid
Croupous
@Phlegmonous
Haemorrhagic
#
53
A 20-year-old man has experienced painful urination for 4 days following spring break. A urethritis is suspected, and
Neisseria gonorrheae is cultured. Numerous neutrophils are present in a smear of the exudate from the penile urethra.
Name the tape of inflammation.
Serous
Fibrinous
Haemorrhagic
@Purulent
Mixed
#
54
A clinical study is peformed of patients with pharyngeal infections. The most typical clinical course averages 3 days from
the time of onset until the patient sees the physician. Most of these patients experienced fever and chills. On physical
examination, the most common finding is a pharyngeal exudate with great amount of neutrophills. Which of the following
types of inflammation did these patients most likely have?
Granulomatous inflammation
@Acute purulent inflammation
Abscess formation
Resolution of inflammation
Chronic inflammation
#
55
A patient with burn disease died due to progressive septicaemia. The autopsy in the anterior abdominal wall by means of
microscopic examination revealed diffuse infiltration of intermuscular spaces by segmentonuclear leukocytes, tissues
edema and lysis of muscular fibres. Determine the nature of the pathological process.
Abscess
Catarrhal inflammation
Diphtheritic inflammation
@Phlegmon
Necrosis
#
56
A 40-year-old woman has had a chronic cough with fever and weight loss for the past month. A chest radiograph reveals
multiple nodules from 1 to 4 cm in size, some of which demonstrate cavitation in the upper lobes. A sputum sample
reveals the presence of acid fast bacilli. Which of the following cells is the most important in the development her lung
lesions?
Macrophage
Fibroblast
@Neutrophil
Mast cell
Platelet
#
57
The sponge and cortical layers of tibia are on different areas at the stage of destruction. Cavities, that have already
appeared, are filled with creamy mass of green-yellow color. What is the most possible form of inflammation?
@Suppurative inflammation
Catarrhal inflammation
Serous inflammation
Proliferous inflammation
Mixed inflammation
#
58
A 90-year-old woman has developed a fever and cough over the past 2 days. Staphylococcus aureus is cultured from her
sputum. She receives a course of antibiotic therapy. Two weeks later she no longer has a productive cough, but she still
has a fever. A chest radiograph reveals a 3 cm rounded density in the right lower lobe whose liquefied contents form a
central air-fluid level. There are no surrounding infiltrates. Which of the following is the best description for this outcome
of her pneumonia?
Hypertrophic scar
@Abscess formation
Regeneration
Bronchogenic carcinoma
Progression to chronic inflammation
#
59
A 24-year-old woman presents with increasing pain and swelling in the posterior region of her neck. Physical
examination finds a red, hot, swollen area measuring approximately 1 cm in greatest dimension. The skin is intact in this
area, but surgical exploration finds a cavity that is filled with purulent material. Cultures from this material grow
Staphylococcus aureus. Histologic section reveal liquefactive necrosis filled with numerous neutrophils and necrotic
tissue. These histologic finding best describe which one of the following pathological processes?
@Abscess formation
Epithelial erosion
Fibrinous inflammation
Serous inflammation
Ulcer formation
#
60
An autopsy of the 58-year-old man, revealed in his liver a focus of tissue destruction, 4 cm in diameter, filled with a
yellow-green liquid. What is the most likely diagnose?
@Abscess
Phlegmon
nthrax
Empyema
Granuloma
#
61
A 49-year-old man, with 14 days history of acute lobar staphylococcal pneumonia in the left lower lobe of a lung, died of
pulmonary and cardiac insufficiency. A post-mortem revealed in the upper lobe of a right lung an oval cavity,
approximately 5 cm in diameter, filled with pus and connected with bronchus. Which complication of acute pneumonia
took place in this case?
@Abscess of the lung
Gangrene of the lung
Thromboembolia of a pulmonary trunk
Bronchiectasis disease
Acute bronchitis
#
62
An autopsy of an elderly male revealed a cavity with compacted walls filled with a thick, greenish liquid, which had a
stinking odor. What is the most likely disease?
@ Abscess of the lung.
Gangrene of the lung.
Infarct of the lung.
Cavern of the lung.
Tuberculoma of the lung.
#
63
During the section of a 47-year-old man, who died of pulmonary-heart failure, there in the left lung was determined a
cavity (4x4 cm), filled with the pus. The wall was uneven, presented by the pulmonary tissue. Make a diagnosis.
Echinococcosis
Chronic abscess
Cavernous tuberculosis
@Acute abscess
Fibrous alveolitis
#
64
Patient with purulent recidiving othitis died from the brain’s swelling with pressing of cerebellum into foramen occipitalis
magnum. On the autopsy in the temporal lobe of the left hemisphere a cavity with rough edges is detected. It is filled with
muddy milk-like liquor of yellow-green colour. This morphology witnesses about the
@Acute abscess
Grey malacia
Phlegmon
Empyema
Chronic abscess
#
65
A 50-year-old male died of a pulmonary and cardiac insufficiency. An autopsy revealed in a left lung a 4 x 4 cm cavity,
filled with pus. The wall of a cavity had a rough pattern, presented by pulmonary tissue. What is the most likely
diagnose?
@ Acute abscess
Chronic abscess
Cavernous tuberculosis
Hydatidosis
Fibrosing alveolitis
#
66
A 40 year old male patient died from cerebral edema. In anamnesis the face carbuncle was registered. Autopsy revealed
hyperemia and edema of cerebral tissue. White matter of the left hemisphere had two cavities 6 х5,5 and 5х4,5 cm large
filled with yellowish-green cream-like fluid. Walls of the cavities were built up by nerve tissue with irregular rands.
What complication of carbuncle was it?
Chronic abscesses
Cysts
Empyema
Colliquative necroses
@Acute abscesses
#
67
After surgical removal of cavernous neoplasm of liver pathological research was executed. Its wall is made of thick
reticular conjunctive and granular tissue, the cavity itself contains viscous, dim liquid with an unpleasant smell. Most part
of its content is presented by polymorphous leucocytes. Name the pathological process.
Acute abscess
@Chronic abscess
Phlegmona of liver
Single-chamber echinococcus
Simple liver cyst
#
68
Surgeon removed cavity formation of the liver. The wall of the cavity was lined by uneven lager of septic necrotic
granulation tissue wich changes into fibrouse tissue at its periphery, midzone containing pus. Your diagnosis.
@Chronic abscess
Plegmon
Acute abscess
Catarral purulent inflammation
Hemorrhagic inflammation
#
69
A 40 year old man noticed a reddening and an edema of skin in the area of his neck that later developed into a small
abscess. The incised focus is dense, yellowish-green. The pus contains white granules. Histological examination revealed
drusen of a fungus, plasmatic and xanthome cells, macrophages. What type of mycosis is the most probable?
Aspergillosis
Sporotrichosis
Coccidioidomycosis
Candidosis
@Actinomycosis
#
70
Colonoscopy of a patient ill with dysentery revealed that mucous membrane of his large intestine is hyperemic, edematic;
its surface was covered with grey-and-green coats. Name the morphological form of dysenteric collitis:
Purulent
Catarrhal
Necrotic
@Fibrinous
Ulcerous
#
71
A macroscopic investigation of trachea revealed a dim, hyperemic mucosa, covered with grey - white membranes. Which
one is the most likely form of the inflammation?
@Fibrinous
Purulent
Serous
Proliferative
Mixed
Exudative focal.
#
72
Mucous tunic of trachea is dull, plethoric, with grey film coating. What is the most probable form of inflammation?
@Fibrinous
Purulent
Serous
Proliferative
Mixed
#
73
A young male presented to his physician with considerably enlarged, hyperemic, painful tonsils. A gross examination of
tonsils revealed dense, dirty-gray membranes on the surface. These membranes have spread to the hard palate and were
intimately attached to tissues underneath. An attempt to remove the membranes resulted in bleeding. Which pathological
process indicates these morphological changes?
@Diphtheroid exudative inflammation.
Croupous exudative inflammation.
Catarrhal exudative inflammation.
Purulent exudative inflammation.
Hemorrhagic exudative inflammation.
#
74
A physical examination of a 5-year-old boy reveals the fauces and the tonsils enlarged, plethoric, and coated with
irremovable whitish membranes. Which one of the following characterizes the changes in the fauces and tonsils?
@ Diphtheritic inflammation
Caseous necrosis
Fibrinous necrosis
Croupous inflammation
Purulent inflammation
#
75
A child with asphyxia was admitted to the hospital. On examination of the child whitish and spots difficult to separate
were revealed in his larynx. What type of inflammation was in that case?
@Diphtheritic inflammation
Croupous inflammaton
Purulent inflammation
Catarrhal inflammation
Serous inflammation
#
76
A man has died in 8-th day since beginning of the disease. It was diagnosed dysentery. During the autopsy it was found
out a thickened wall of the sigma and rectum, fibrinous membrane on the surface of mucous membrane. Histologically:
there is a deep necrosis of mucous membrane with infiltration of necrotic masses with fibrin. What kind of colitis does
correspond to those changes?
@Diphtheritic
Catarrhal
Ulcerative
Chronic
Gangrenous
#
77
A 28-year-old patient has considerable enlarged sanguineous painful tonsils. On itsr surface there are solid gray
membranes that spread on the hard palate and are firmly connected with underlied tissues. The effort to separate them
ends with the bleeding. What pathological process does determin these morphological changes?
@Diphtheritic exudative
Croupous exudative
Catarrhal exudative
Suppurative exudative
Hemorrhagic exudative
#
78
A 4-year-old boy presented to a hospital with pain in a throat at swallowing and malaise. A physical examination revealed
grayish- whitish membranes on a pharynx and tonsils on a background of moderate edema and hyperemia. These
membranes were intimately attached to tissues underneath. What pathology the described changes testify to?
@Inflammation
Dystrophy
Necrosis
Metaplasia
Reganerative process
#
79
A man who died of shigellosis has a sanguineous mucous membrane of the large intestine. Which was covered with a
gray membrane, which can be hardly dissociated. What kind of inflammation has developed in the intestine?
Catarrhal inflammation
Croupous inflammation
Hemorrhagic inflammation
Serous inflammation
@Diphtheritic inflammation
#
80
A man died 8 days after the beginning of the disease. He was diagnosed with dysentery. At the autopsy it was found out a
thickened wall of the sigma and rectum, fibrinous membrane on the surface of mucous membrane. Histologically: there is
a deep necrosis of mucous membrane with infiltration of necrotic masses with fibrin. What kind of colitis does
correspond to the changes?
Catarrhal
@Diphtheritic
Chronic
Gangrenous
Ulcerative
#
81
An autopsy of a 50-year-old male, who died of dysentery, revealed the hyperemic mucosa of the colon, coated with grey
membranes, which can be removed from tissues underneath with some effort. Which type of an inflammation described in
the intestine?
@Diphtheroid inflammation
Croupous inflammation
Hemorrhagic inflammation
Serous inflammation
Catarrhal inflammation
#
82
An autopsy of a 53-year-old male, who died from chronic renal failure, revealed the colon's mucosa, covered with grey -
yellow membranes, densely coherent to tissues underneath. The removal of the membranes resulted in ulcers formation.
Name a type of an inflammation?
@Diphtheroid
Serous
Catarrhal
Croupous
Purulent
#
83
A 5-year-old child has a fever and pain at swallowing. A physical examination revealed enlarged, dark-red palatine
tonsils. They were coated with a grey - yellow membrane which was intimately attached to the surface of the tonsils.
Which kind of inflammation described in the tonsils?
@Diphtheritic inflammation
Croupous inflammation
Hemorrhagic inflammation
Purulent inflammation
Catarrhal inflammation
#
84
A physical examination of tonsils and soft palate mucosa revealed white-grey color membranes, which are intimately
attached to tissues underneath. An attempt to take out membranes results in formation of a deep tissue defect. Diagnose
pathology on a mucosa of tonsils and a soft palate.
@Diphfheritic inflammation
Serous inflammation
Croupous inflammation
Purulent inflammation
Mixed inflammation
#
85
A 28 years old patient has enlarged, painful, tonsils. On the surface of tonsils there are thick, grey films that extend to soft
palate. Films are firmly connected to underneath tissues, while attempting to separate – a bleeding occurs. What
pathological process causes such changes?
@Diphtheritic exudative inflammation
Croupous exudative inflammation
Catarrhal exudative inflammation
Purulent exudative inflammation
Haemorrhagic exudative inflammation
#
86
A 6-year-old child presented to department of infectious diseases with acute pain in a throat, difficulty at swallowing, rise
of body temperature up to 39°C and edema of neck. A gross oral investigation revealed enlarged, hyperemic tonsils,
covered with plenty of yellow membranes, which intimately attached to the mucous membrane. An attempt to take off
membrane results in deep, bleeding defect formation. What type of inflammation takes place?
@ Diphtheritic
Purulent
Serous
Croupous
Hemorrhagic
#
87
During the section of a man, who died of croupous pneumonia, an opaque fluid was determined in pleural cavity. There
was also a membrane of gray color on the visceral pleura. What kind of inflammation has developed on the visceral
pleura?
Catarrhal
@Fibrinous
Suppurative
Granulomatous
Haemorrhagic
#
88
A 62-year-old man died of croupous pneumonia. A post-mortem revealed in the pleural cavity an opaque liquid and a
grayish membrane on visceral pleura. What is the most likely type of inflammation on the visceral pleura?
@ Fibrinous inflammation
Catarrhal inflammation
Purulent inflammation
Granulomatous inflammation
Hemorrhagic inflammation
#
89
A 27-years-old woman has been suffered by diabetes mellitus since she had been a child. During the last years the arterial
hypertension and proteinuria have occurred. She has died with signs of uremia. What typical changes were found out in
autopsy?
@“Cor villosum”
“Tiger heart”
“Armor heart”
Mesenchymal fat dystrophy of the heart
Purulent pericarditis
#
90
A 7-year-old child fell ill sharply. His temperature increased up to 38 oC; rhinorrhea, cough, lacrimation and large-
spot rash on skin appeared in him. Mucous membrane of patient’s pharynx is edematous and red; buccalmucous
membrane has whitish spots. What is the character of inflammation underlying changes of buccal mucous membrane?
@Fibrinous
Hemorrhagic
Serous
Purulent
Catarrhal
#
91
Autopsy revealed that right lung is enlarged, solid, there are fibrin layers on the pleura. Lung tissue is light grey color on
incision with muddy liqued exudates. What lung disease are these symptoms typical for?
Fibrosing alveolitis
@Croupous pneumonia
Interstitial pneumonia
Bronchopneumonia
Pulmonary gangrene
#
92
Autopsy of a 50-year-old man revealed the following changes: his right lung was moderately compact in all parts, the
dissected tissue was found to be airless, fine-grained, dryish. Visceral pleura had greyish-brown layers of fibrin. What is
the most likely diagnosis?
Interstitial pneumonia
Bronchopneumonia
Pneumofibrosis
@Croupous pneumonia
Tuberculosis
#
93
A patient has died after 3 weeks of pneumonia. Lower lobe of right lung is enlarged, thickened, airless, grey, with fibrin
coatings on pleura. Microscopically: every alveole of this lobe contain fibrin and segmented neutrophils. Your diagnosis?
@Croupous pneumonia
Nidus bronchopneumonia
Influenza pneumonia
Fibrinous pleuritis
Interstitial pneumonia
#
94
A forensic medical expert examines the body of a 58 y.o. man who had been consuming large amounts of alcochol for a
long time and died at hom e. Microscopicaly: the right lung is dense and enlarged, its incision revealed that the tissue is
greyish and homogenous, pleura is covered with greyish layers. Microscopically - alveolar cavities contain fibrin,
hemolyzed erythrocytes. Make a diagnosis:
Interstitial pneumonia
Caseous pneumonia
@Croupous pneumonia
Primary pulmonary tuberculosis
Focal pneumonia
#
95
A 5-years-old girl suffered from diphtheria. She has died in three days because of asphyxia caused by diphtheria’s croup.
During the autopsy it was found out, that mucous membranes of larynx, trachea and bronchi were thickening, swelling,
dull, covering by grayish membranes, which easily came off. What process did?
@Croupous inflammation.
Serous inflammation
Diphteritic inflammation
Proliferative inflammation
Suppurative inflammation
#
96
In autopsy the whole lower lobe of left lung had dirty-grey colour and liver-like consistence .There was yellow-grey fibrin
on a pleura. An etiology of disease is pneumococcus type IY. Call this disease:
@Croupous pneumonia
Clottage of bronchial arteries
Tuberculosis
Lues
Clottage of pulmonary arteries
#
97
On the third day of diphtheria a 4-year-old girl died of true diphtheria croup. During the autopsy there was revealed
thickened, edematous, dull mucous membrane of larynx, trachea and bronchial tubes; a grayish membrane that could be
easily dissociated covered the mucous membrane. Name the kind of inflammatory process.
Mixed
Serous
Suppurative
@Fibrinous
Catarrhal
#
98
During the section of girl, who died of asphyxia, there was determined, that the mucous membrane of trachea and bronchi
was covered with the white-gray membrane that was softly connected with underlied tissues and could be easily removed
by forceps. The lumen of the segmental bronchi was filled with loose gray-white masses. What kind of tracheobronchitis
does the exudation indicates on?
@Croupous
Catarrhal
Diphtheritic
Purulent
Fetid
#
99
A 5-year-old girl fell ill with diphtheria. In 3 days she died of asphyxia as the after-effect of true diphtheria croup. During
the section there was revealed thickened, edematous, dull mucous membrane of larynx, trachea and bronchial tubes. A
grayish membrane that could be easily dissociated covered the mucous membrane. What pathological process is indicated
by such morphological changes in larynx?
Suppurative inflammation
Serous inflammation
@Croupous inflammation
Diphtheroid inflammation
Catarrhal inflammation
#
100
At post-mortem a 5-year-old boy is found to have a severe tracheobronchitis complicated with asphyxia. Grossly, a thick,
gray, leathery membrane covered the mucous tunic of trachea and bronchi. The membrane loosely attached to underlying
tissues and easily removed with forceps. The lumen of the segmental bronchi was blocked with gray masses of tissue
debris, which microscopically consisted with necrotic tissues, neutrophils, fibrin and bacteria. What is the most likely
type of inflammation?
@Croupous.
Catarrhal.
Diphtheritic.
Purulent.
Serous
#
101
A 64-years-old male died of rheumatism. An autopsy revealed epicardium covered with villiferous grey color membranes,
which were easily separated from tissues underneath. The separation of membranes presented an edematous, hyperemic
surface of epicardium. What type of an inflammation revealed in pericardium?
@Fibrinous pericarditis
Purulent pericarditis
Hemorrhagic pericarditis
Proliferative pericarditis
Catarrhal pericarditis
#
102
A 38-year-old man, with a history of chronic pyelonephritis, died from the chronic renal failure. Medical record showed
that an auscultation in the hospital determined a patient's "pericardial frictions rub". An autopsy revealed a dim, rough,
and hairy - like epicardium. What type of pericarditis presented in that case?
@ Croupous
Diphtheroid
Purulent
Ichorous
Serous
#
103
A child presented to a hospital with symptoms of asphyxia. A physical examination revealed in the larynx whitish
membranes, which obstructed lumen and were easily separated from tissues underneath. Name the most probable type of
an inflammation in the larynx?
@ Croupous inflammation
Hemorrhagic inflammation
Diphtheroid inflammation
Catarrhal inflammation
Purulent inflammation
#
104
A 3-year-old child presented to a hospital with asphyxia. Examination of the larynx revealed whitish membranes, which
occluded a lumen and easily extracted. Diphtheria was suspected. Which form of inflammation took place in the larynx?
@Croupous inflammation
Catarrhal inflammation
Diphtheroid inflammation
Serous inflammation
Purulent inflammation
#
105
An autopsy of a 34-years old female, with a history of croupous pneumonia revealed opaque fluid in her pleural cavity.
There was also a grey membrane on the visceral pleura. What is the most likely type of the pleura's inflammation?
@Fibrinous.
Catarrhal.
Suppurative.
Granulomatous.
Hemorrhagic.
#
106
A 4-year-old girl with 3 days history of diphtheria presents to the emergency with croup symptoms. Intensive care was
unsuccessful and child died at the hospital. A post-mortem revealed thickened, edematous, dull mucous tunic of the
larynx, trachea, and bronchi covered by grayish membrane easily separated from tissues underneath. What is the most
likely type of inflammation?
@Fibrinous.
Serous.
Suppurative.
Mixed.
Catarrhal.
#
107
A man with chronic pyelonephritis died of chronic renal failure. However, there was a sound of pericardial rubbing
remarked, while the patient was alive. Autopsy showed: epicardium is dull, rough, seems like covered with hair. What
pericarditis is this?
@Croupous
Diphtheritic
Purulent
Putrefactive
Serous
#
108
A 65-year-old female with a long history of chronic glomerulonephritis died from chronic renal failure. A post-mortem
revealed on the surface of the epicardium and pericardium grey-whitish villous membranes. Which pathological process
took place in the pericardium?
@Fibrinous inflammation
Organization
Proliferative inflammation
Hemorrhagic inflammation
Arterial plethora
#
109
A 55 year old man had been suffering from chronic glomerulonephritis. He died from chronic renal failure.
Macroscopical examination revealed on the surface of epicardium and pericardium some greyish-white villous
depositions. After their removal dilated and plethoric vessels were uncovered. What process took place in the
pericardium?
Haemorrhagic inflammation
Proliferative inflammation
Organization
@Fibrinous inflammation
Arterial hyperemia
#
110
Autopsy of a 34 y.o. man who died from rheumatism revealed that epicardium surface was villous and covered with grey
films that can be easily remove D. After their removal the surface is edematic and plethori C. What is the most probable
diagnosis?
@Fibrinous pericarditis
Hemorrhagic pericarditis
Proliferative pericarditis
Catarrhal pericarditis
Purulent pericarditis
#
111
On the autopsy of the dead from the flu patient, muddy red fluid pours down from the cuts. About what inflammation
does this morphology witness?
@Serous-hemorrhagic
Catarrhal
Purulent
Productive
Croupous
#
112
During an autopsy of a man died of acute cardiac failure: under visceral pericardium there are small haemorrhages,
surface of serous tunic is dull and covered with grey-coloured hair-like coatings. In the cavity of pericardium there is
200ml of dim liquid. What form of inflammation is this?
@Serous-fibrinous
Diphtheritic fibrinous
Serous
Purulent
Catarrhal
#
113
During an autopsy of a man, who died of acute cardiac failure: underneath visceral pericardium there are lots of small
haemorrhages, the surface of serous tunic is dull and covered with a net of hair-like coatings. In the cavity of pericardium
there is 200ml of dim liquid. What kind of inflammation is this?
@Serous-fibrinous
Diphtheritic fibrinous
Serous
Purulent
Catarrhal
#
114
During the histologic lung analysis of a man who died from cardiac insufficiency the inflammation focuses were revealed.
Alveoles were full of light-pink fluid, here and there with pinkish fibers that formed a close-meshed reticulum with a
small number of lymphocytes. What type of exudate is present in lungs?
Purulent
Serous
@Serofibrinous
Hemorrhagic
Fibrinous
#
115
A 6-year-old child presented to infectious disease department with a body temperature 38°C, punctulated, and bright red
rash on a skin. The scarlet fever diagnosis is made. The fauces mucosa was brightly hyperemic, edematous; tonsils were
markedly enlarged, with the dim yellowish-grayish centers and sites of black color. What inflammation underlies changes
in fauces?
@Purulent-necrotic
Fibrinous
Hemorrhagic
Serous
Catarrhal
#
116
An 8-year-old child was admitted to the infectious department with fever (up to 38 oC) and punctuate bright-red skin rash.
The child was diagnosed as having scarlet fever. Objectively: mucous membrane of pharynx is apparently hyperaemic
and edematic, the tonsils are enlarged and have dull yellowish-grey foci with some black areas. What inflammation is the
reason for the pharynx alterations?
Haemorrhagic
Serous
@Purulent necrotic
Catarrhal
Fibrinous
#
117
A 3-year-old child died of a meningococcal fever. A macroscopic study revealed swallowed meninges which had yellow-
green colouring. Which pattern of inflammation was most likely in meninges?
@Fibrinous-purulent inflammation
Serous inflammation
Hemorrhagic inflammation
Catarrhal inflammation
Necrotic inflammation
#
118
A post-mortem of a man revealed in his abdominal cavity approximately 2.0 liters of a purulent exudate. A peritoneum
was found to be dull, with a grayish tint. The intestinal serous membranes were covered with grayish membranes, which
were removed easily. Which of the following is the most accurate diagnosis?
@Fibrinous-purulent peritonitis
Hemorrhagic peritonitis
Serous peritonitis
Tuberculous peritonitis
-
#
119
Opening of a patient's abdominal cavity revealed for about 2,0 L of purulent fluid. Peritoneum is dull, greyish, serous
tunic of intestines has grayish layers that can be easily removed. It is most likely to be:
Serous peritonitis
Hemorrhagic peritonitis
Tuberculous peritonitis
-
@Fibrinopurulent peritonitis
#
120
During an autopsy of a man, who died of gaster cancer in abdominal cavity there was revealed about 1 litre of dim green
liquid. On parietal and visceral peritoneum there are filament-like coatings of grey colour. What kind of inflammation is
this?
@Fibrinous-purulent
Catarrhal
Serous
Haemorrhagic
Mixed
#
121
A 19 year old patient gained painful haemorrhages from gingivae after she had been taking sulfadimezine for 2 days.
Objectively: hyperaemia and swelling of gingival edge and papillae, insignificant bleedings. What kind of inflammation
is this?
@Acute catarrhal
Chronic catarrhal
Haemorrhagic
Purulent
Fibrinous
#
122
A patient with an acute rhinitis has hyperemia and excessive mucus formation in nasal cavity. What epithelial cells of
mucous membrane have the intensified activity?
@Goblet cells
Endocrine cells
Microvillous cells
Basal cells
Ciliated cells
#
123
A 17 year old boy fell seriously ill, the body temperature rose up to 38,5 oC there appeared cough, rhinitis, lacrimation,
nasal discharges. What inflammation is it?
Serous
Purulent
Fibrinous
Hemorrhagic
@Catarrhal
#
124
During an endoscopy of gaster a biopsy of mucous membrane was taken. Following research showed that mucous
membrane is thickened, swollen, plethoric with numerous small haemorrhages and covered with thick layer of mucus.
Name the form of acute gastritis.
@Catarrhal
Erosive
Fibrinous
Purulent
Necrotic
#
125
In course of gastric endoscopy the biopsy material of mucous membrane was taken. Its histological examination revealed
the following: mucous membrane is intact, thickened, edematic, hyperemic, with small droplike hemorrhages, coated with
thick mucus. Name the form of acute gastritis:
Erosive
@Catarrhal
Fibrinous
Necrotic
Purulent
#
126
A 60 year old patient complains of tongue burning, excessive salivation and glossalgia effects that came 5 days after he
started using a metal dental bridge. Objectively: mucous membrane of oral cavity is edematic and hyperemic. What form
of stomatitis is it?
Gangrenous
@Catarrhal
Purulent
Fibrinous
Ulcerous
#
127
A 5-year-old child who often fells ill with respiratory diseases has eczematous appearances after consumption of some
food products, tendency to prolonged course of inflammatory processes. What kind of diathesis can be suspected in this
case?
Lymphohypoplastic
Hemmorhagic
@Exudative-catharral
Asthenic
Arthritism
#
128
A rectoromanoscopy revealed edematous, reddish color mucosa of rectum and sigmoid colon, covered with a thick layer
of mucus. Name the kind of inflammation?
@Catarrhal
Purulent
Mixed
Hemorrhagic
Fibrinous
#
129
A 5-year-old child presented to a hospital with acute fever (up to 38° C), rhinitis, cough, lacrimation, and a patchy rash on
the skin. His pharyngeal mucosa was edematous and hyperemic, with whitish maculae on the cheeks. What type of
inflammation is the background of described changes?
@Catarrhal
Purulent
Fibrinous
Hemorrhagic
Serous
#
130
A fragment of a stomach mucosa was taken for histology during endoscopy procedure. A microscopic investigation
revealed intact mucous membrane, covered by mucus, thickened, edematous and hyperemic with a numerous tiny
hemorrhages. Define the type of acute gastritis?
@Catarrhal (simple)
Erosive
Fibrinous
Purulent
Necrotic
#
131
Choose the steps of leukocytes emigration:
Dilation of interendothelial gaps
@Migration
@Chemotaxis
Phagocytosis
@Adhesion
Opsonisation
#
132
Choose 4 processes occurring during exudation:
@Reaction of microcirculation
@Chemotaxis
@Exiting of plasma components into extravascular space
Cell injury and necrosis
Phagocytosis
@Increasing of vascular permeability
Exiting of plasma component in connection of the changes of hydrostatic pressure in microcirculation ways
#
Proliferative inflammation
#
1
All following features are characteristics of chronic inflammation EXCEPT:
@The predominant cell type is neutrofill
The predominant cell type is macrophage
The predominant cell type is lymphocyte
Permanent tissue destruction
Sclerosis
#
2
In an experiment, a lung tissue preparation is exposed to Mycobacterium tuberculosis organisms. Over the next week, it is
observed that granuloms form in the lung. Within the granuloma are found inflammatory cells expressing class II MHC
antigens. These cells elaborate cytokines that promote fibroblastic production of collagen within the granulomas. From
which of the following peripheral blood leukocytes are these cells bearing class II antigen most likely to be derived?
Neutrophils
B cells
@Monocytes
NK cells
Basophils
#
3
A 37-year-old man presents with a cough, fever, night sweats, and weight loss. A chest x-ray reveals irregular densities in
the upper lobe of his right lung. Histologic sections from this area reveal caseous necrosis surrounded by rare acid-fast
bacilli and a few scattered giant cells. These giant cells result from the fusion together of what type of cell?
@Activated macrophages
Atypical lymphocytes
Endothelial cells
Epithelial cells
Reactive fibroblasts
#
4
During ultramicroscopic research of bioptate of tissue: a macrophage contains increased amount of lysosomes. What
process causes such changes?
@Phagocytosis activation
Cell proliferative activity
Phagocytosis slow down
Apoptosis activation
Apoptosis slow down
#
5
During the ultra structural examination of biopsy material, there was detected that the cytoplasm of the macrophage, that
comprises the inflammatory infiltration, had an increased amount of lysosomes. What does this mean?
@Activation of phagocytosis
Proliferative activity of cells
Slowness of phagocytosis
Activation of apoptosis
Slowness of apoptosis
#
6
A histological examination of the myocardium of a man, who died from heart failure, revealed a sclerosis of the
pericardial connective tissue and diffuse infiltration by lymphocytes, macrophages, plasmocytes and single neutrophils.
What is the most likely type of inflammation in a heart?
@ Interstitial productive.
Granulomatous.
Alterative.
Exudative diffuse.
Exudative focal.
#
7
A 43-year-old woman has had nausea with vomiting persisting for the past 5 weeks. On physical examination there are no
abnormal findings. She undergoes an upper Gl endoscopy and gastric biopsies are obtained. The microscopic appearance
of these biopsies shows mucosal infiltration by lymphocytes, macrophages, and plasma cells. Name the type of
inflammation?
Purulent
Diphteroid
Croupous
Granulomatous
@Interstitial proliferative
#
8
A 72-year-old woman did not get a "flu" shot in the fall as recommended for older persons. In the wintertime, she became
ill, as many people in her community did, with a respiratory illness that lasted for several weeks. During this illness, she
had a fever with a non-productive cough, mild chest pain, myalgias, and headache. What was her chest radiograph most
likely to have shown during this illness?
6 cm left hilar mass
@Bilateral interstitial infiltrates
Marked hilar lymphadenopathy
Left upper lobe consolidation
Extensive pleural thickening with pleural effusions
#
9
A young man had a removal of mucous membrane new growths in his nasal cavity. A histological investigation revealed
diffuse lymphocytes, plasma cells and macrophages infiltration. Name the type of inflammation.
@Inflammation with formation of polyps
Inflammation with formation pointed condyloma
Interstitial inflammation
Granulomatous inflammation
Exudative inflammation
#
10
A patient presented with small knots on a thin peduncle in his rectum. Microscopical investigation of the biopsy material
revealed a growth of connective tissue and glands. What is the most likely pathology?
@Hyperplastic polyp.
Hypertrophy.
Metaplasia.
Atrophy.
Sclerosis.
#
11
During the histological examination of skin biopsy of a 24-year-old patient, there was detected a caseous necrosis, that
was surrounded by an infiltration of lymphocytes with single huge cells. There was also an overgrowth of connective
tissue, endovasculitis. What kind of pathological process was it?
@Productive granulomatous inflammation
Productive inter inflammation
Abscess
Catarrhal inflammation
Putrefactive inflammation
#
12
A 44-year-old male had a skin biopsy procedure. Histological investigation of a tissue sample showed a caseous necrosis
surrounded by infiltrate, consisted from lymphocytes, some giant cells and the growth of connective tissue. In addition,
the endovasculitis was diagnosed. What is the most likely pathological process?
@ Productive granulomatous inflammation.
Productive interstitial inflammation.
Abscess.
Catarrhal inflammation.
#
13
A patient was admitted in the hospital with diagnosis of trichinellosis. What kind of inflammation develops within
the muscles around the parasites?
@Proliferative granulematous
Exudative fibrinous
Exudative purulent
Exudative serous
Proliferative interstitial
#
14
A microscopic examination of a kidney biopsy revealed foci with granular, eosinophilic masses in their centers. These
lesions were surrounded by an infiltrate, consisted of lymphocytes, epithelioid cells and giant Langhans' cells. What is the
most likely pathological process?
@ Granulomatous inflammation
Coagulative necrosis
Caseous necrosis
Alterative inflammation
Proliferation and derivation of macrophages

15
During the microscopic examination of biopsy material (kidney), there were detected many focuses with corny
eosinophilic mass in the middle that were surrounded by the infiltrate of lymphocytes, epithelioid cells and single
Pirogov-Langhans’ cells. Choose the pathologic process that best fits the description.
@Granulomatous inflammation
Coagulative necrosis
Caseous necrosis
Alterative inflammation
Proliferation and differentiation of macrophages
#
16
During a microscopical research of bioptate of kidney there were nidi of eosinophilic masses found. Those are surrounded
by lymphocytic and epithelioid-cell infiltrate and singular Pirogov-Langghans's cells. Name the pathological process.
@Granulomatous inflammation
Coagulative necrosis
Curdled necrosis
Alterative inflammation
Proliferation and differentiation of macrophages
#
17
All following diseases display granulomatous inflammation EXCEPT:
Tuberculosis
Leprosy
Syphilis
@Honorrhea
Sarcoidosis
#
18
During the section of a 60-year-old man, in his lungs and liver were detected a great amount of white nodules of corn
size. Microscopically: there were revealed many granulomas with a focus of necrosis in the middle. On the periphery of
granulomas there were epithelioid, lymphoid, plasmatic cells, macrophages and a great amount of Pirogov-Langhans
cells, which dominate in infiltrates. What kind of granuloma is meant?
Foreign-body
Macrophagal
Phagocytomal
Epithelioid
@Giant cell
#
19
A 40-year-old woman had bilateral silicone breast implants placed two years ago. During that time, there is increased
firmness with slight deformity on the left. The implants are removed, and there is evidence for leakage of the implant
contents on the left. Which of the following cell types would be most characteristic of the inflammatory response in this
situation:
Mast cell
Eosinophil
@Giant cell
Neutrophil
Plasma cell
#
20
An autopsy of a 53-year-old male showed many white, millet sized nodules in his lungs. A microscopic examination
revealed granulomas with a necrosis in their centers and epithelioid, lymphoid, plasma cells, macrophages and numerous
large multinucleated cells (Langhans' cells) on the periphery of a lesion.
What is the most likely form of granuloma?
@ Giant - cell granuloma
Macrophagal
Foreign body granuloma
Epithelioid cells
-
#
21
A 56-year-old man has had increasing dyspnea for 6 years. He has no cough or fever. He was inhaling silica dust for
many years in his job. A chest x-ray now shows increased intersitital markings and parenchymal 1 to 3 cm nodules. His
pulmonary problems are most likely to be caused by which of the following inflammatory processes?
Neutrophilic infiltration with release of leukotrienes
@Foreign body giant cell formation
Plasma cell production of immunoglobulin
Histamine release by mast cells
Release of growth factors by macrophages
#
22
A 40-year-old woman had bilateral silicone breast implants placed two years ago. Since that time, she has noted increased
firmness with slight deformity of the breast on the left. The implants are removed, and there is evidence for leakage of the
implant contents on the left. Which of the following cell types is most likely to be most characteristic of the inflammatory
response in this situation?
Mast cell
Eosinophil
@Giant cell
Neutrophil
Plasma cell
#
23
A histological investigation of enlarged neck lymphatic node revealed a congestion of epithelial cells, lymphocytes and
giant Pirogov-Langhan's cells. In the center of lesion a caseous necrosis was present. Which pattern of inflammation do
the lymphatic nodes display?
@ Tuberculosis
Rhinoscleroma
Sarcoidosis
Glanders.
Syphilis.
#
24
A 30-year-old male has a neck lymph node removed for examination. Histological investigation revealed granulomas,
which consisted of epithelioid, lymphoid and multinuclear Langhans giant cells. Granulomas' centers were presented with
necrosis. What causative agent needs to be revealed in the necrosis zone for support of the diagnosis of tuberculosis?
@Mycobacterium tuberculosis
Treponema pallidum
Staphylococcus
Frisch bacillus
Salmonella
#
25
A 28-year-old male presented to the hospital with a fever and weakness, (asthenia). A biopsy was taken from one of his
enlarged neck lymph nodes for histological examination. Microscopic investigation revealed the foci of necrosis,
surrounded by epithelioid cells, giant multinuclear Langhans' cells and also lymphocytes. What is the most likely
diagnosis?
@Tuberculosis
Hodgkin's disease
Lympholeukemia
Sarcoidosis
Syphilis
#
26
A 7-year-old child, with a history of angina, presented with enlarged paratracheal, bifurcation and neck lymph nodes. A
histological examination of his neck lymph node revealed the foci of necrosis, surrounded by lymphocytes, epithelioid
cells and Pirogov-Langhans' cells. Which one is the most likely pathology?
@Tuberculosis
Sarcoidosis
Rhinoscleroma
Malleus
Syphilis
#
27
The histological examination of a lung tissue revealed a lesion presented with a locus of necrosis, surrounded by regular
lines of epithelioid, lymphoid cells, plasma cells, macrophages and giant Pirogov-Langhans' cells.
Define a kind of inflammation?
@Tuberculosis inflammation
Banal productive inflammation
Inflammation at a lepra
Exudative inflammation
Alterative inflammation
#
28
A 47-year-old male presented with a sub-febrile temperature and enlarged lymph nodes. A biopsy investigation of a
lymph node revealed numerous granulomas with a caseous necrosis in the centers, surrounded by epithelioid cells, giant
multinuclear Langhans' cells and lymphocytes.
What is the most likely diagnosis?
@Tuberculosis
Lymphogranulomatosis
Lymphosarcoma
Lympholeukemia
Simple lymphadenitis
#
29
Histological investigation of a skin biopsy revealed granulomas, which consisted of epithelioid cells, surrounded mainly
by T-lymphocytes. In addition, few giant multinucleated cells (Langhans' cells) were determined among epithelioid cells.
The caseous necrosis was diagnosed within the centers of granulomas. There was lack of blood vessels. What is the most
likely disease?
@Tuberculosis
Syphilis
Lepra
Rinoskleroma
Glanders (malleus)
#
30
Microscopic investigation an elderly man's lungs revealed the foci of necrosis, surrounded by the bank of epithelioid cells
and lymphocytes. Between lymphocytes and epithelioid cells the rounded giant cells, with a considerable quantity of
nuclei, located on a periphery of a lesion were determined. Give the name for these changes:
@Tuberculous granuloma
Sarcoid granuloma
Syphilitic granuloma
Lepromatous granuloma
Scleromatous granuloma
#
31
A 18-year-old girl had a neck lymphatic node removal at the biopsy procedure. Histological investigation showed
nodules, consisted of banks of epithelioid, lymphoid cells and large multinucleated cells (Langhans' cells), located
between them. In the centers of nodes a caseous necrosis was determined. What causative agent needs to be revealed in
the necrosis zone for making final diagnosis?
@Koch's mycobacterium
Mycobacterium leprae
Treponema pallidum
Rickettsia
Fungi
#
32
An autopsy of 32-year-old male revealed slightly swollen and hyperemic lungs, covered with numerous millet-like
grayish nodules. Histological investigation determined lesions consisted of lymphoid, epithelioid and Langhans giant
cells. What is the most likely diagnosis?
@Tuberculosis
Croupous pneumonia
Bronchopneumonia
Interstitial pneumonia
Bronchiectatic disease
#
33
A sample of cervix uteri was taken for histological examination. A microscopical investigation revealed a tissue's
inflammatory infiltration with vascular walls involvement of small vessels, arteries and veins. In addition, there were also
determined a presence of plasma cells, lymphocytes, epithelioid cells in the infiltrate and foci of sclerosis and hyalinosis
of tissue. What is the most likely disease?
@Syphilis
Tuberculosis
Leukoplakia
Erosio colli uteri
Condyloma
#
34
A histological investigation of a liver biopsy revealed granulomas, consisted of plasma cells and lymphoid cells. Small
vessels were characterized by endo - and perivasculitis. Some foci of caseous necrosis were also determined. Such
granulomas are typical for:
@Syphilis
Tuberculosis
Lepra
Rinoskleroma
Glanders (malleus)
#
35
A young man was operated because of phimosis. Histological investigation of a removed prepuce revealed polymorphic
infiltrates, consisted of randomly located plasmocytes, epithelioid and lymphoid cells. The vasculitis was diagnosed in
vessels. What disease such changes are characteristic for?
@Syphilis
Tuberculosis
Actinomycosis
Periarteritis nodosa
Lepra
#
36
Biopsy of an oral cavity's ulcer presented for a histological research. A microscopical investigation revealed areas of
caseous necrosis, surrounded by plasmocytes, epithelioid, lymphoid cells and few giant multinuclear Pirogov-Langhans
cells. There were also small vessels with signs of endo- and perivasculitis. Choose the most likely diagnosis.
@ Syphilis
Tuberculosis
Lepra
Rinoskleroma
Glanders (malleus)
#
37
An inflammatory process that has continued for 3 months includes the transformation of tissue macrophages to epithelioid
cells. There are also lymphocytes present. Over time, fibroblasts lay down collagen as the focus of inflammation heals.
These events are most likely to occur as an inflammatory response to which of the following infectious agents?
@Mycobacterium leprae
Pseudomonas aeruginosa
Cytomegalovirus
Giardia lamblia
Treponema pallidum
#
38
Histological examination of a skin tissue sampling revealed granulomas consisting of macrophagal nodules with
lymphocytes and plasmatic cells. There are also some big macrophages with fatty vacuoles containing causative agents of
a disease packed up in form of spheres (Virchow's cells). Granulation tissue is well vascularize D. What disease is this
granuloma typical for?
Glanders
Syphilis
Rhinoscleroma
Tuberculosis
@Lepra
#
39
A 46-year-old woman living in western Texas presents with the slow development of numerous macules and nodules on
her face. Physical examination finds a peripheral neuropathy with enlarged palpable nerves. A biopsy from a one of the
skin nodules reveals aggregates of foam cells within the dermis. Special stains reveal rare acid-fast bacilli within
peripheral nerves. Which of the following is the most likely diagnosis?
@Leprosy
Nocardiosis
Sarcoidosis
Syphilis
Tuberculosis
#
40
A skin sample was taken for histological research. Microscopical investigation revealed granulomas, consisted of
macrophages' knots with adding of lymphocytes and plasma cells. There were also determined large macrophages with
fatty vacuoles, which contained packed layers of causative agents of a disease (Virchow's cells). For what disease the
given description is characteristic?
@Lepra
Tuberculosis
Syphilis
Rinoskleroma
Glanders (malleus)
#
41
As a result of a microscopic examination of the fragment of the skin the granulomas were found out; they composed of a
lot of macrophages with a few amount of lymphocytes and plasma cells. Besides large macrophages with fat vacuoles
occurred, the microorganisms were packed in those vacuoles as “cigars in the box” (Virchow’s cells). Granulation tissue
had a good vascularization. What disease are such granulomas
@Leprosy
Tuberculosis
Syphilis
Rhinoscleroma.
Actinomycosis.
#
42
An elderly man presented to his physician with the complicated nasal breathing. The histological investigation of a biopsy
from his nose septum revealed granulomatous inflammation in the mucosa with groups of large vacuolated histiocytes
containing bacteria (Mikulicz's cells). What is the most likely diagnosis?
@Rhinoscleroma
Syphilis
Tuberculosis
Malleus
Lepra
#
43
A histological examination of biopsy from the nose mucous revealed epithelioid cells, plasmocytes, Mikulicz cells, and
eosinophilic (acidophilic, Russell's) bodies. What is the most likely diagnosis?
@ Rhinoscleroma
Syphilis
Tuberculosis
Droplet infection
Allergic rhinitis
#
44
Histological investigation of a biopsy revealed a granuloma, consisted of lymphocytes, plasma cells, macrophages with
foamy cytoplasm (Mikulicz's cells) and numerous hyaline's sphere- like aggregates. About what disease it is possible to
think?
@Rinoskleroma
Lepra
Syphilis
Tuberculosis
Actinomycosis
#
45
A 43-year-old patient complains of hindered nasal breathing. Histological investigation of a biopsy of his nasal cavity
mucosa revealed lymphoid cells, epithelioid cells, plasma cells and Mikulicz's cells. What is the most likely diagnosis?
@Rinoskleroma
Glanders (malleus)
Tuberculosis
Lepra
Syphilis
#
46
During the histological examination of the biopsy it was found out the granulomas composing of lymphocytes, epithelioid
cells, plasma cells, macrophages with foamy pale cytoplasm (Mikulicz’s cells), and a lot of hyaline balls. What disease do
you think about?
@Rhinoscleroma.
Leprosy.
Syphilis.
Tuberculosis.
Actinomycosis.
#
47
At the biopsy material of the nasal mucous membrane epithelioid cells, plasmocytes, Mikulicz’s cells, eosinophilic
Russell’s corpuscles were detected. What is your diagnosis?
Allergic rhinitis
Syphilis
Tuberculosis
Respiratory-syncytial infection
@Rhinoscleroma
#
48
A 46-year-old patient complained of hard nasal breathing. The mucous membrane of the nose is thickened. At the biopsy
material were revealed Mikulicz’s cells, the aggregation of epithelioid cells, plasmocytes, lymphocytes, hyaline spheres.
What is your diagnosis?
Adenoviral rhinitis
@Scleroma
Allergic rhinitis
Rhino viral infection
Meningocococcal nasopharyngitis
#
49
A 46-year-old man complains of difficulties with nasal breathing. Mikulicz's cells, accumulation of epithelioid cells,
plasmocytes, lymphocytes, hyaline balls were discovered in the biopsy material of the thickened nasal mucosa. What is
the most likely diagnosis?
Rhinovirus infection
Meningococcal nasopharyngitis
Allergic rhinitis
@Scleroma
Virus rhinitis
#
50
A 54-year-old patient complains of complicated nasal breathing. A nasal mucosa was thickened. Histological
investigation of a biopsy revealed Mikulicz's cells, epithelioid cells, plasma cells, lymphoid cells, and hyaline sphere- like
aggregates. What is the most likely diagnosis?
@Skleroma
Adenoviral rhinitis
Allergic rhinitis
Rhinoviral infection
Meningococcal nasopharyngitis
#
51
A 22-year-old patient was admitted to the hospital with complaints of heavy nasal breathing. During the examination of
her nasal cavity the doctors found thickened mucous membrane, a lot of mucus and nodular infiltrates without erosions in
the nose.The nasal rhinoscleroma was diagnosed. The biopsy was taken. What typical morphological changes may be
found?
Granulomas with Langhan’s cells
@Granulomas with Mikulicz’s cells
Granulomas with Virchow’s cells
Granulomas with foreign body cells
Interstitial inflammation
#
52
A patient, who had complains of dyspnoae, was done a biopsy of the nasal mucous membrane. The diagnosis was made –
rhinoscleroma. What cells are typical for this illness?
Plasmocytes
Pirogov-Langhans’ cells
@Mikulicz’s cells
Lymphocytes
Schaumann’s corpuscles
#
53
At a 45-year-old man the biopsy procedure of his nasal cavity's mucosa was performed. The diagnosis of rhinoscleroma
made. What cells are typical for the disease?
@Mikulicz's cells
Pirogovs' - Langhans' cells
Plasma cells
Lymphocytes
Schaumann"s bodies
#
54
A 40-year-old man presented with hyperemia and edema of skin in the neck region. In course of time a small abscess
developed there. The focal was solid of yellow-green color. White grains could be seen in the purulent mass. Histological
examination showed there druses of fungi, plasmatic and xanthoma cells, macrophages. Indicate the most possible kind of
mycosis.
Sporotrichosis
Aspergillar
Candidiasis
@Actinomycosis
Coccidioidomycosis
#
55
A 40 year old man gained plethora and swelling on his neck. Some time after it developed into a small abscess. On
incision nidus is thick, yellowish-green coloured. In purulent masses small white grains can be seen. Histologically: fungi
mycelia, plasma cells, xantome cells, macrophages are revealed. What is the most probable type of mycosis?
@Actinomycosis
Aspergillosis
Candidosis
Sporotrichosis
Coccidiomycosis
#
56
Microscopical examination of an infiltrate removed from the submandibular skin area of a 30 y.o. man revealed foci of
purulent fluxing surrounded by maturing granulations and mature connective tissue, the pus contains druses consisting of
multiple short rod-like elements with one end attached to the homogenous centr E. What disease is it?
@Actinomycosis
Tuberculosis
Syphilis
Candidosis
-
#
57
Recovery of an organism from an infectious disease is accompanied by neutralization of antigens by specific antibodies.
What cells produce them?
Tissue basophils
Fibroblasts
@Plasmocytes
T-lymphocytes
Eosinophils
#
58
The woman of 37 years old during one year periodically had infectious diseases of a bacterial genesis, their current was
during the long term, the remissions - short-term. The hypogamaglobulinemia is found out during the inspection. Which
infringement of function of the cells can be its direct reason?
@Plasma cells
Phagocytes
Neutrophils
Macrophages
Lymphocytes
#
59
Live vaccine is injected into the human body. Increasing activity of what cells of connective tissue can be expected?
Fibroblasts and labrocytes
Adipocytes and adventitious cells
Macrophages and fibroblasts
@Plasmocytes and lymphocytes
Pigmentocytes and pericytes
#
60
The bacteria are phagcyted by macrophages after penetration into the organism. Which role is played the macrophages in
cooperation of immunocompetent cells at the first stage of the immune answer formation?
@Provide processing and presentation of antigen T-helpers.
Activate T-killers.
Activate NK-cells.
Produce immunoglobulins.
Provide processing and presentation of an antigen to T-killers.
#
61
A 4-year-old girl is being evaluated for multiple recurrent bacterial infections, especially with S. aureus. Pertinent clinical
history also includes a history of delayed separation of her umbilical cord following birth and severe pereodontitis.
Laboratory tests reveal her peripheral blood leukocyte count to be elevated, even when no infection is present. Further
work-up finds a familial deficiency of CD18. Which of following is the most likely diagnosis?
Cyclic neutropenia
DiGeorge syndrome
@Leukocyte adhesion deficiency
Myeloperoxidase deficiency
Shwachman-Diamond syndrome
#
62
A 4-year-old boy presents with progressive trouble breathing over the past couple of hours. Physical examination finds a
young boy in moderate respiratory distress with inflamed and edematous regions involving his epiglottis and uvula.
Material from his inflamed epiglottis is cultured and grows colonies of Hemophilus influenzae. This bacterium has
increased amounts of sialic acid, which can inhibit the alternate complement system. That is, organisms with increased
amounts of sialic acid, such as Streptococcus pneumonia, H. influenzae, and Neisseria meningitides, need to be killed by
the classic complement pathway. Which of the following substances is most likely to activate the classic complement
pathway?
Aggregated IgA
Aggregated IgD
C3 nephritic factor
IgE bound to antigen
@IgG bound to antigen
#
63
An acute pharyngitis is diagnosed in a previously healthy 12-year-old girl. A throat culture grows group A beta-hemolytic
streptococcus. The pharyngitis resolves, but three weeks later she develops chest pain along with fever and malaise. The
girl describes aching of her knees, hips, shoulders, and elbows. Multiple subcutaneous skin nodules are noted on physical
examination, and she exhibits slight shaking movements other head and neck. Her antistreptolysin-0 and anti-DNAse B
liters are elevated. Her creatine kinase MB fraction is elevated. These findings are most likely the result of which of the
following immunologic mechanisms:
Bypass of low-zone tolerance through cross-reactivity
@Deposition of antigen-antibody complexes in multiple tissue
Formation of an antibody to immune complexes
Exposure of a previously sequestered antigen
Interaction between genetic predisposition and environmental insults
#
64
The acquired defect of immune system - infringement of activation of system of a complement on a classical way on a
background of the sufficient contents of the components of the system is diagnosed at the patient. The presence of defect
antibodies formation is suspected. The decrease of the contents in which organism of antibodies is possible to expect
first of all?
@Ig M, IgG
Ig A, IgG
Ig D, IgG
Ig E, IgG
Ig M, Ig A
#
65
An appropriate, useful type 1 hypersensitivity response of the immune system, accompanied by eosinophilia, would be
directed against:
Amyloid protein
Spirochetes
Neoplasms
Inhaleddusts
@Liver flukes
#
66
The experiment animal with a history of previous sensibilization received the next subcutaneous dose of antigen. In the
place of injection a fibrinous inflammation with alteration of vessels walls, matrix and fiber structures of connective
tissue (mucoid and fibrinoid swelling) and fibrinoid necrosis developed. What immunological reaction takes place?
@ Hypersensitivity reaction immediate type
Hypersensitivity reaction slow type
Reaction of transplantation immunity
Normergic reaction
Granulomatosis
#
67
An experimental animal was first sensibilized whereupon an antigen dose was introduced subcutaneously. This injection
resulted in the development of a fibrinous inflammation with alteration of vessel walls, basal substance and fibrous
structures of connective tissue in form of mucoid and fibrinoid swelling and necrosis. What immunological reaction took
place?
@Immediate hypersensitivity
Reaction of transplantation immunity
Delayed-type hypersensitivity
Normergic reaction
Granulomatosis
#
68
An experimental animal after sensibilization was subcutaneously injected a doze of antigen. In the place of injection
fibrinous inflammation has developed, along with vessel wall alteration and mixomatous and fibrinoid swelling, fibrinoid
necrosis of conjunctive tissue. What kind of immunological reaction is this?
@Immediate hypersensitivity
Slow hypersensitivity
Transplantational immune reaction
Normoergic reaction
Granulematosis
#
69
A patient with paroxysmal attacks of asphyxia, which appear after inhalation of different aromatic substances has been
made a diagnosis of bronchial asthma. Ig E rate is elevated. What type of reaction is it the most typical for?
@Anaphylactic
Delayed type of hypersensitivity
Autoimmune
Cytotoxic
Immunocomplex
#
70
Atopical bronchial asthma is diagnosed at the patient with periodic attacks of a dyspnea which arise at inhalation of the
different aromatic substances. The augmentation Ig E is diagnosed. Of which type of reactions is it typical?
@Anaphylactic reactions
Cytotoxic reactions
Immunocomplex reactions
Delayed hypersensitivity
Autoimmune reactions
#
71
A surgeon used novocaine as an anaesthetic during surgical manipulations. 10 minutes after it the patient became pale, he
got dyspnea and hypotension. What type of allergic reaction is it?
@Anaphylactic
Cell-mediated
Cytotoxic
Stimulating
Immune complex
#
72
A child strung by a honeybee developes respiratory failure and vascular collapse within minutes and goes into shock. The
reaction is caused by which of the following factors:
@IgE
IgG
Macrophages
Sensitized T cells
IgM
#
73
A 27- year-old woman has dropped penicillin containing eye drops. In few minutes there appeared feeling of itching,
burning of the skin, lips and eyelids edema, whistling cough, decreasing of BP. What antibodies take part in the
development of this allergic reaction?
IgA and IgM
IgM and IgG
IgM and IgD
IgG and IgD
@IgE and IgG
#
74
The infiltration anesthesia by ultracain with adrenaline solution has been done to a young man. Then redness, edema of
the skin with itchy blisters suddenly developed. What type of hypersensitivity took place?
@ Anaphylaxis
Cytotoxicity
Immune complex damage
Hypersensitivity slow type
Granulomatosis
#
75
During the treatment of a 39-year-old patient with purulent pulpitis, he was made an infiltrative anesthesia with solution
of Ultracaine with Adrenaline. Suddenly hyperemia, skin edema with blisters and itching appeared. What kind of
hypersensivity does the patient have?
Cytotoxic
@Anaphylactic
Immunocomplex damage
Delayed hypersensivity
Granulomatous
#
76
A woman complaining of coryza, phonastenia, eyelids redness and lacrymation during spring period came to the doctor.
What type of allergic reaction by Gell ano Coombs classification develops in this case?
Delayed type of hypersensitivity
Immunocomplex
Stimulating
Cytotoxic
@Anaphylactic
#
77
During the histological examination of a man, who had asthma for many years and died of suffocation, a great amount of
mucus with eosinophiles was detected in the lumen of bronchioles and small bronchial tubes. There were also sclerosis of
interalveolar septa and dilatation of alveolar lumen. What mechanism of hypersensivity reaction is meant?
Granulomatous
Cytotoxic reaction
Immunocomplex reaction
Cytolysis, caused by lymphocytes
@Reaginic reaction
#
78
A man with a history of bronchial asthma died of asphyxia. Microscopical investigation of lungs revealed excess of
mucus with lots of eosinophils in the bronchial lumen, sclerosis of interalveolar septas and alveoli dilatation. What
mechanism of hypersensitivity reaction presented in that case?
@Reagin reaction
Cytotoxic reaction
Immune complex reaction
Cytolysis, conditioned by lymphocytes
Granulomatosis
#
79
A man died because of acute gasp. He has been suffering from bronchial asthma for a long time. Histological research of
lung tissue: in the lumen of small bronchi and bronchioles there is a large amount of mucus with eosinophilic infiltration,
intraalveolar walls sclerosis, alveolar dilatation. What is the mechanism of this hypersensitivity reaction?
@Reagine reaction
Cytotoxic reaction
Immune complex reaction
Lymphocyte mediated cytolysis
Granulematosis
#
80
A female patient suffering from bronchial asthma had got a viral infection that provoked status asthmaticus with fatal
outcome. Histological examination of lungs revealed spasm and edema of bronchioles, apparent infiltration of their walls
with lymphocytes, eosinophils and other leukocytes; labrocyte degranulation. What mechanism of hypersensitivity
underlies the described alterations?
Immune complex
@Reagin reaction
Autoimmune
Immune cytolysis
Inflammatory
#
81
A 38-year-old patient died during intractable attack of bronchial asthma. Histologic examination revealed mucus
accumulation in bronchial lumen, a lot of fat cells (labrocytes) in the wall of bronches, many of them are in the state of
degranulation, there are also a lot of eosinophils. What pathogenesis of bronchial changes is it?
@Atopy
Cytotoxic, cytolytic action of antibodies
Granulomatosis
Immunocomplex mechanism
Cellular cytolysis
#
82
A 38 year old patient died during intractable attack of bronchial asthma. Histological examination revealed mucus
accumulations in bronchi's lumen, a lot of mast cells (labrocytes) in bronchi's wall, some of these cells are degranulated,
there are also many eosinophils. Name pathogenesis of these changes in bronchi:
Granulematosis
Cytotoxic, cytolytic effect of antibodies
Immune complex mechanism
Cell-mediated cytolysis
@Atopy, anaphylaxis
#
83
The sensibilization by the allergen of poplar down had been established with the help of cutaneous allergic assay at the
patient with a bronchial asthma. Which factor of immune system plays a main role in development of it
immunopathological conditions?
@IgE
IgD
IgM.
Sensitized T-lymphocytes.
IgG.
#
84
A 48-year-old women with asthma presents with viral respiratory infection. The infection caused a status asthmaticus
with a fatal outcome. Histological investigation of lung tissue revealed a contracted bronchus and a luminal plug
containing mucus and cell debris. The submucosa was edematous and contained a mixed inflammatory infiltrate,
including a lot of eosinophils and degranulated must cells. What is the mechanism of hypersensitivity presented in this
case?
@Humoral immunity (Type I, anaphylactic reaction)
Inflammatory reaction
Autoimmune reaction
Immune complex reaction (Type III of reaction)
Cell-mediated immunity (Type IV of reaction)
#
85
A 4 year old child was performed Mantou test. After 60 hours on the place of injection a nidus thickening and plethora
appeared, and was interpreted as positive result. What kind of hypersensitivity reaction is the basis of the Mantou test?
@Slow hypersensitivity reaction
Immune complex-mediated
Complement-mediated cytotoxic
Immediate hypersensitivity reaction
Reagine reaction
#
86
During the histological examination of a biopsy material from a left auricle of the heart was detected a rheumatic
granuloma. It had developed from the focus of fibrinoid necrosis that was surrounded by basophile macrophages,
lymphocytes, fibroblasts and single plasmocytes. Name the possible type of inflammatory reaction on the immune base.
@Delayed hypersensivity reaction
Normergic type
Immediate hypersensivity reaction
Congenital immunodeficiency
Acquired immunodeficiency
#
87
Which type of hypersensitivity reactions is responsible for hemolytic anemia of newborns caused by Rh
incompatibility in Rh- negative mother and Rh-positive fetus?
@Cytotoxic type 2
Atopic local reaction
Immune complexes type 3
Anaphylactic type 1
Delayed hypersensitivity
#
88
Hemotransfusion stimulated development of intravascular erythrocyte hemolysis. The patient has the following type of
hypersensitivity:
IV type hypersensitivity (cellular cytotoxicity)
@II type hypersensitivity (antibody-dependent)
I type hypersensitivity (anaphylactic)
V type hypersensitivity (granulomatosis)
III type hypersensitivity (immune complex)
#
89
A patient was diagnosed with autoimmune hemolitic cytotoxic anemia. What substances are antigens in II type allergic
reactions?
Inflammation modulators
Hormones
Antibiotics
Serum proteins
@Modified receptors of cell membranes
#
90
A patient visited a dentist with complaints of redness and edema of his mouth mucous membrane in a month after dental
prosthesis. The patient was diagnosed with allergic stomatitis. What type of allergic reaction by Gell and Cumbs underlies
this disease?
Cytotoxic
Anaphylactic
Stimulating
@Delayed type hypersensitivity
Immunocomplex
#
91
Several weeks following a pharyngitis in which the throat culture grew group A lieinolytic streptococcus; a child is noted
to liavc hematuria on urmalysis. An immunofluoresence-staining pattern in a renal liiopsy shows granular deposition of
IgG and complement around glomerular capillary loops. Which of the following immune liypersensitivity mechanisms is
most likely responsible for this pattern of findings?
Type I
Type II
@Type III
Type IV
#
92
A woman has been applying a new cosmetic preparation for a week that resulted in eye-lid inflammation accompanied by
hyperemia, infiltration and painfulness. What type of allergic reaction was developed?
III
V
II
I
@ІV
#
93
A 30 year old woman has applied a lipstick with a fluorescent substance for a long tim E. Then she got a limited erythema
and slight peeling on her lip border, later there appeared transversal striae and cracks. Special methods of microscopic
examination of the affected area helped to reveal sensibilized lymphocytes and macrophages in the connective tissue;
cytolysis. What type of immunological hypersensitivity was developed?
@IV type (cellular cytotoxicity)
Granulomatosis
II type (antibody cytotoxicity)
I type (reaginic)
III type (immune complex cytotoxicity)
#
94
A 15 year old boy died of the heart failure. He had a history of rheumatic heart disease in his medical record. Histological
investigation of the left auricle of the heart revealed the rheumatic granuloma (the Aschoff-Talalayev body) with a central
fibrinous focus associated, with a perimeter plasma cells, macrophages and giant cells. Which of following best describe
the type of immune response?
@Cell mediated (Type IV, delayed hyper sensitivity reaction)
Anaphylactic (Type I, immediate)
Cytolytic (Type II, immediate)
Immune complex (Type III, immediate)
Reaction of transplant rejection.
#
95
A 48-year-old man has a chronic cough with lever that has persisted for several months. I lie chest radiograph reveals a
diffuse icliculonodular pattern. Microscopically on iransbronchial biopsy there are focal areas of inflammation containing
epithelioid inncrophages, Langhans giant cells, and lyinphocytes. These findings are most typical for which of the
following immunologic responses:
Type I hypersensitivity
Type II hypersensitivity
Graft versus host disease
Polyclonal B-cell activation
@Type IV hypersensitivity
#
96
At 46-year-old patient with autoimmune gastritis a biopsy was taken. Histologically, there was an infiltration by
lymphocytes and macrophages of a gastric mucous membrane. What type of hypersensitivity presented in that case?
@Type IY (cell-mediated hypersensitivity)
Type II (reagin type)
Type II (antibody-mediated cytotoxicity)
Type III (immune complex cytotoxicity)
Type V (granulomatous)
#
97
A child had a subcutaneous injection of tuberculin for a testing. Twenty-four hours later a physical examination revealed
the expressed hyperemia and induration of tissues in the place of injection. What was the mechanism of these changes?
@Cellular cytotoxicity
Reagin cytotoxicity
Antibody cytotoxicity
Granulomas formation
Immune complex
#
98
Tuberculine was injected intracutaneously to the child for tuberculin test. Marked hyperemia, tissue infiltration developed
on the place of injection in 24 hours. What mechanism caused these modifications?
@Cells cytotoxity
Reagin type cytotoxity
Granuloma formation
Immunocomplex cytotoxity
Antibody cytotoxity
#
99
Tuberculine was injected intraperitoneally to the animal sensibilized with tuberculine. Venous hyperemia and peritonial
edema were detected on the laparotomy in 24 hours. Increased amount of lymphocytes and monocytes were in the smear-
print from the peritonium. What pathological process is in the animal?
Suppurative inflammation
Fibrinous inflammation
Serous inflammation
Aseptic inflammation
@Allergic inflammation
#
100
The Mantoux test has been made to the sick child with the suspicion on a tuberculosis. The tumescence, a hyperemia and
morbidity has appeared in the place of insertion of the allergen in 24 hours. Which basic components are determine this
reaction of the organism?
@Mononuclears, T-lymphocytes and lymphokines
Granulocytes, T-lymphocytes and Ig G.
Plasma cells, T-lymphocytes and lymphokines
B-lymphocytes, Ig M
Macrophages, B-lymphocytes and monocytes
#
101
Skin tuberculin test was carried out to a patient with chronic lung tuberculosis. Restricted hyperemia and edema appeared
in the site of inracutaneous introduction of tuberculin preparation during 24-48 hours. What cells are primary effectors in
this mechanism reaction?
@T-lymphocytes
Neutrophils
B-lymphocytes
Endotheliocytes
Smooth muscle cells of microvessels
#
102
A patient develops induration in skin-test with purified protein derivative (PPD) to determine previous exposure to
Mycobacterium tuberculosis within 48 hours. Histologically, what kinds of cells can be found in the inoculation site?
@Helper T cells and macrophages
B cells
Basophils
Eosinophils
Neutrophils
#
103
48 hours after performing tuberculin test (Mantoux test) to a child a 10 mm papule appeared on the spot of tuberculin
introduction. What hypersensitivity mechanism underlies these changes?
Antibody-dependent cytotoxicity
Granulomatosis
Immune complex cytotoxicity
@Cellular cytotoxicity
Anaphylaxis
#
104
The Mantoux test (with a tuberculin) was put at the child of 10 years. The papule in the size up to 8 mm in diameter has
appeared in 48 hours on a place of injection of the tuberculin. Which type of hypersensitivity has developed after
introduction of a tuberculin?
@Hypersensitivity of IV type.
Artuse type phenomenon.
Serum disease.
Atopical reaction.
Hypersensitivity of II type.
#
105
A 4-year-old child was made a Mantoux reaction. In 60 hours after the administration of tuberculin subcutaneously a
focal callosity and hyperemia of skin (diameter 15 mm) appeared. This was marked as positive test. What kind of
hypersensivity reaction is the test based on?
Complement-mediate cytotoxic reaction
Immune complex-mediate hypersensivity reaction
@Delayed hypersensivity reaction
Immediate hypersensivity reaction
Reagin reaction
#
106
The first grade pupils went through a medical examination aimed at selection of children needing tuberculosis
revaccination. What test was applied?
Supracutaneous tularin test
Burne test
Anthracene test
Schick test
@Mantoux test
#
107
A positive tuberculin skin test, with a firm 15-mm diameter dark red firm area of induration on the forearm, which
appeared 60 hours following injection of the PPD, mainly takes place as a consequence of:
Type I hypersensitivity
Type II hypersensitivity
Type III hypersensitivity
@Type IV hypersensitivity
Complement activation
#
108
A 6-year-old child had the Mantoux test. In 3 days there was an inflammatory induration and a reddening of a skin 15 mm
in diameter. That was considered as the positive test. What type of hypersensitivity reaction presented in that case?
@Hypersensitivity of slow type
Immune complex hypersensitivity
Complement-mediated cytotoxicity
Hypersensitivity immediate type
-
#
109
Allergic diagnostic tests are used for the diagnosis of many infectious diseases (tuberculosis, brucelosis, tularemia
etc). Papula and redness in the place of the allergen injection appear, if diagnose was confirmed. Antigens interaction
reaction is conditioned by:
@T-lymphocytes and lymphokines
IgE and lymphokines
IgM and macrophages
IgE and T-lymphocytes
IgM and tissue basophiles
#
110
While registering the child to the school Mantu's test was made to define whether revaccination was needed test result is
negative. What does this result of the test mean?
Absence of antitoxic immunity to the tuberculosis
Presence of antibodies for tubercle bacillus
Absence of antibodies for tubercle bacillus
Presence of cell immunity to the tuberculosis
@Absence of cell immunity to the tuberculosis
#
111
The positive cutaneous Burne.s assay is observed at patients with the brucellosis. Which factor of immune system
plays a main role in development of inflammatory reaction in a place of insertion of the brucellin to these patients?
@Sensitized T-lymphocytes.
IgM
IgE
IgG.
IgD.
#
112
The boy of 1,5 years who did not receive scheduled inoculations, contacted to the patient with the measles. The donor
gammaglobulin has been inserted to the child with the purpose of emergency specific prophylaxis. What kind of
immunity has been created thus?
@Passive
Natural
Antitoxic
Postvaccinal
Local
#
113
A patient with a history of serious radioactive irradiation had bone marrow transplantation. In 2 months a rush on the skin
and diarrhea developed. Clinical and laboratory tests showed hepatic insufficiency, local necrosis of the skin's epithelial
cells, intestinal crypts and liver parenchymal cells. What was the reason for symptoms aggravation?
@"Graft against the owner" disease
Sepsis development
An acute graft rejection
A chronic graft rejection
-
#
114
The transplantation of the marrow is carried out to the liquidator of consequences of failure on the Chernobyl atomic
power station which has received the big exposure dose. The development of reaction " the transplant against the owner "
is diagnosed at the patient in few days after the carried out operation. Which antigens have served as a releaser of
occurrence of this reaction?
@Antigens of system HLA of cells of an organism of the liquidator.
Antigens of system Rh of erythrocytes of the liquidator.
Antigenes HBs, HBc, HBe
Antigens of system ABO of erythrocytes of the liquidator.
Antigens of system HLA of cells of an organism of the donor.
#
115
The patient.s condition has worsened in 2 months after operation of kidney transplantation. On the basis of
laboratory inspection it is ascertained, that reaction of the transplantant rejection began. Which factor of immune system
plays a main role in this reaction?
@T-killers.
Interleucin-1.
Natural killers.
B-lymphocytes.
T-helpers 2.
#
116
A 43-year-old male, with a history of kidney transplantation, followed by immunosuppressive therapy, died of
intoxication. A histological investigation of his lungs, kidneys and pancreas revealed strikingly enlarged (giant) cells with
basophilic cytoplasm and large nucleus. The violet intranuclear inclusions surround by a small clear halo, reminded an
eye of an owl. What is the most likely diagnosis?
@Cytomegaly
Tuberculosis
Syphilis
Lepra
Plague
#
117
Minutes after a donor kidney is connected to the recipient’s blood vessels, the transplanted kidney turns blue, becomes
flaccid, excretes a few drops of bloody urine, and has to be removed. Histologic examination of the kidney reveals
neutrophils within arterioles, glomeruli, and peritubular capillaries. Immunoglobulin and complement are found to be
deposited in blood vessel walls. Which of the following mechanisms is primarily involved in this type of transplant
rejection?
Donor cytotoxic T lymphocytes are directed against host antigens
Host cytotoxic T lymphocytes are directed against donor antigens
Donor natural killer cells are directed against host antigens
Preformed donor antibodies are directed against host antigens
@Preformed host antibodies are directed against donor antigens
#
118
The transplantation of a donor skin has been made to the patient with the large burns. But the transplant became swollen
at the 8th day, its colour changed and for 11 day started to torn away. What cells are take part in the tearing away?
@T-lymphocytes
Erythrocytes
Basophils
Eosinocytes
B-lymphocytes
#
119
Donor skin transplantation was performed to a patient with extensive burns. On the 8-th day the graft became swollen and
changed colour; on the 11-th day graft rejection started. What cells take part in this process?
B-lymphocytes
Erythrocytes
@T-lymphocytes
Basophils
Eosinophils
#
120
Physical examination of a pregnant woman with an Rh-negative blood revealed a high level of antierythrocyte antibodies.
The cutaneous flap of her Rh-positive husband applied for depression of antibodies. In 2 weeks a flap was rejected.
Microscopical investigation showed a disturbance of blood circulation, edema and cellular infiltration mainly by
lymphocytes, neutrophils and macrophages. What is the most probable pathology?
@Transplantation immunity
Hypersensitivity reaction immediate type
Hypersensitivity reaction slow type
Granulomatous inflammation
Interstitial inflammation
#
121
During puncture biopsy, a diffuse infiltration of stroma by lymphocytes, plasmocytes, lymphoblasts, plasmoblasts and a
necrotic arteriitis were revealed in the transplanted kidney. What pathologic process has developed in the transplant?
Tubulonecrosis
Glomerulonephritis
Ischemic damage of the kidney
@Immune rejection
Pyelonephritis
#
122
During operation of kidney transplantation in 15 minutes after vascular anastomoses application, transplant became
flabby, cyanotic and spotty; the pulsation was absent. Microscopical investigation of a kidney specimen revealed
periendothelial accumulation of neutrophils in glomeruli vessels. In addition there were also a thrombosis and necrosis of
some glomeruli. Specify the type of graft rejection.
@Hyperacute rejection
Accelerated rejection
Acute intersticial rejection
Acute vascular rejection
Chronic rejection
#
123
A morphological investigation of a kidney graft revealed a diffuse stromal infiltration by lymphocytes, plasmocytes,
lymphoblasts, plasmablasts, and also a necrotic arteriitis. What pathology has developed in a graft?
@Immunological graft rejection
Glomerulonephritis
Ischemic kidney damage
Tubular necroses
Pyelonephritis
#
124
Transplantation of kidney was performed between identical tweens. Graft between genetically identical individuals most
probably:
@Will be not rejected
Will be subjected to hypersensitivity type IV
Will cause lethal graft-versus-host disease
Will be characterized by type I hypersensitivity
Will be rejected slowly as a result of minor histocompartibility antigens
#
125
Local lymphonodules enlarged near the infected wound. Increased amount of macrophages, lymphocytes, lymphatic
follicles in the cortical layer and large amount of plasma cells were revealed on histological examination. What process
in the lymphatic nodules represent these histological changes?
Tumour transformation
@Antigen stimulation
Innate insufficiency of the lymphoid tissue
Hypersensibility reaction
Acquired insufficiency of the lymphoid tissue
#
126
A young man presents with the enlargement of the regional lymph nodes next to his inflamed wound. Histological
investigation of the lymph node biopsy revealed the increased number of macrophages, lymphocytes and lymphoid
follicules within cortex zone. Plasma cells number also increased. These alterations are most likely result of:
@Antigen stimulation
The acquired insufficiency of lymphoid tissue (secondary immunodeficiency state)
Hereditary immunodeficiency state
Neoplastic transformation
Hypersensitivity reaction
#
127
Histological investigation of a lymph node revealed a considerable quantity of the enlarged lymphoid follicles with big
germinal centers, and plenty of mitotic figures. What pathology these changes are characteristic for?
@Antigen stimulation with follicle hyperplasia
Atrophy of lymphoid tissue
Lymphosarcoma
Lymphogranulomatosis
Lympholeukemia •
#
128
During a histological research of bioptate of lymphoid node: hyperaemia and swelling of cortex layer, in light centre of
follicles and in the medulla there is a large amount of plasma cells, small amount of lymphocytes, sinus cell active
proliferation and great macrophage reaction. Name the changes in the lymphoid node.
Inherited peripheral lymphoid tissue insufficiency
Acute lymphadenitis
Lymphoid tissue suppression
Lypmphogranulematosis
@Lymphoid tissue antigen stimulation
#
129
A 20-year-old woman died of postnatal sepsis. A post-mortem revealed enlarged congested spleen with abundant
curettage from a cut section. Microscopical investigation showed hyperplasia and infiltration by plasmocytes of the red
pulp and spleen's follicles. The red pulp was also rich with macrophages. What pathology resulted in spleen's alteration?
@Antigenic stimulation of an organism
Hereditary insufficiency of a peripheric lymphoid tissue
Reaction of hypersensitivity of immediate type
Reaction of hypersensitivity of the slowed type *
Autoimmunization
#
130
Your patient under long-term antibiotic treatment makes autoantibodies against his own red blood cells, leading to
hemolysis. Which of the following mechanisms can explain the hemolysis development?
@Complement is activated, and membrane attack complexes lyse the red cells
Perforins and granzymes from cytotoxic T cells lyse the RBCs
Neutrophils release proteases that lyse the red cells
Apoptosis
Interleukin-2 binds to a receptor on the red cells, which results in lysis of the red cells
#
131
The renal biopsy from a 30-year-old woman with a positive antinuclear antibody test at a titer of 1:1024, acute renal
failure, myalgias, and bilateral serous pleural effusions shows a granular pattern of immunofluorescence staining with
antibody to complement component Clq. This pattern is most typically produced as a consequence of which of the
following immunologic mechanisms:
IgE coating mast cells
Antiglomerular basement membrane antibody
@Antigen-antibody complexes
Macrophage release of lymphokines
Release ofprostaglandins
#
132
A 33-year-old patient with a history of acute tonsillitis presented with the urinary syndrome (haematuria, proteinuria and
leukocyturia). Kidneys biopsy revealed an intracapillary proliferative glomerulonephritis. Electronic microscopy
determined large subepithelial deposits. What is the pathogenesis of this disease?
@Immune complex mechanism
Atopy
Cytotoxic, cytolytic action of antibodies
Cellularally caused cytolysis
Granulomatosis
#
133
The second pregnancy of a 23-year-old woman appears uncomplicated until ultrasound performed at 19 weeks shows
hydrops fetalis. The fetal organ development is consistent for 19 weeks, and no congenital anomalies are noted. Her first
pregnancy was uncomplicated and resulted in the birth of a normal girl at term. The current pregnancy yields a baby born
prematurely at 32 weeks gestation with marked icterus. The baby is also markedly anemic, and an exchange transfusion is
performed. Which of the following immunologic mechanisms best explains these findings?
Anti-receptor antibody
Loss of self tolerance
Immune complex formation
Delayed type hypersensitivity
@Complement mediated cell destruction
#
134
A patient with infectious mononucleosis had been taking glucocorticoids for two weeks. He was brought into remission,
but he fell ill with acute attack of chronic tonsillitis. What action of glucocorticoids caused this complication?
Antiallergic
@Immunosuppressive
Antitoxic
Antishock
Anti-inflammatory
#
135
A 57-year-old woman presents with difficulty swallowing, drooping eyelids, and double vision. Work-up finds a mass in
the anterior mediastinum. Biopsies from this mass reveal thymus tissue having scattered reactive lymphoid follicles with
germinal centers. Which of the following is the most likely diagnosis?
Malignant thymoma
Thymic carcinoma
@Thymic hyperplasia
Thymic hypoplasia
Thymic lymphoma
#
136
An infantile girl of 14 suddenly died after acute respiratory viral infection. During the section following changes were
observed: severe enlargement of thymus, generalized hyperplasia of lymphatic nodes and hypoplasia of the ovaries.
Diagnose the disease.
@Thymicolymphatic status
Accidental involution of the thymus
Thymic hypoplasia
Thymic atrophy
Thymoma
#
137
A 7-month-old child died of severe pneumonia complicated by sepsis. A post-mortem revealed an absence of thymus. In
lymph nodes there were no lymphoid follicles and cortical substance, in a spleen follicles were markedly reduced,
germinal centers were absent. What is the most likely diagnosis?
@ Thymus agenesia
Thymus aplasia
Thymus atrophy
Thymus hypoplasia
Acidental thymus involution
#
138
A newborn died after 3 days of life, as a result of a lethal outcome of intrauterine pneumonia. At autopsy a newborn is
sound have a great reduction of his thymic weight and its sizes. Histological investigation of thymus revealed the unclear
boundary between cortex and medulla zone as a result of lymphocytes amount reduction. A few Hassall corpuscles were
found. They were focally keratinized and had concentric aggregated of epithelial cells. The gland was also exhibited lobe
atrophy and sclerosis. What pattern of reaction did the thymus demonstrate?
@Accidental thymus transformation (involution)
Thymus atrophy
Thymus hypotrophy
Thymus aplasia (agenesis)
Thymus neoplasia
#
139
A 3-year-old child died of an acute destructive staphylococcal pneumonia. Macroscopically, thymus mass reduced to 3,0
gr. Microscopical investigation of thymus showed reduction of lobules, considerable decline of lymphocytes quantity,
stromal collapse, and inversion of layers with cystic enlargement of the Hassall's bodies. What is the most likely
diagnosis?
@Accidental thymus involution
Thymomegaly
Thymus hypoplasia
Thymus dysplasia
Thymus agnenesia
An exacerbation of a chronic hepatitis
#
140
A 5-year-old child died of acute destructive staphylococcus pneumonia. During the examination of the thymus was
revealed the decrease in its mass to 3,0g. The histological examination showed the regress of the gland’s lobules, greatly
decreased amount of lymphocytes with the collapse of the stroma, the inversion of coats, a cyst like increment of
Hassall’s corpuscles (bodies). What is the most possible diagnosis?
Thymic dysplasia
Thymomegaly
Thymic hypoplasia
@Accidental involution
Thymic agenesia
#
141
The influence of negative factors upon an organism results in the thymus change that is accompanied by mass loss of
thymocytes, their drive out to the peripheral organs, proliferation of epithelioreticulocytes. How is this phenomemon
called?
Thymus hypotrophy
Age thymus involition
Thymus atrophy
Thymus distrophy
@Accidental thymus involution
#
142
Influence of unfavourable factors upon the organism causes change of thymus accompanied by mass loss of thymocytes,
their displacement to the peripheral organs, proliferation of epithelioreticulocytes. What phenomenon is it?
Thymus atrophy
Thymus dystrophy
Age thymus involution
@Accidental thymus involution
Thymus hypotrophy
#
143
A 4-yers-old boy presents with multiple malformation of craniofacial bones. He died soon due to sepsis, resulted from
bronchopneumonia. Blood test revealed the immunoglobulins within normal range. At autopsy the thymical aplasia was
identified. Name the mane cause of death.
@ Cellular immunodeficiency
Combined immunodeficiency
Secondary immunodeficiency
Acute leukemia
Chronic intoxication
#
144
If a child has no detectable T and B cells, it results in severe immunodeficiency. The most probable defect is in:
@Stem cells originating in the bone marrow
The bursal equivalent
T cell-B cell interaction
The thymus
The spleen
#
145
Child B. 1 year old is frequently sick of viral-bacterial infections which are badly given in therapies. The absence of the
lymphocytes providing cellular immunity in the blood is revealed at carrying out of the immunological status research.
Which failure of immunity and on which system is revealed at the given sick child?
@Initial immunodeficiency, on T-system
Secondary immunodeficiency, on T-system
Initial immunodeficiency, on B-system
Initial immunodeficiency, on macrophagal system
Initial immunodeficiency, on microphages
#
146
A patient with clinical presentations of immunodeficiency went through immunological examinations. They revealed
significant loss of cells that form rosettes with erythrocytes of a ram. What conclusion can be made according to the
analysis data?
Decrease of complement system rate
Decrease of B-lymphocytes rate
Insufficiency of effector cells of humoral immunity
@Decrease of T-lymphocytes rate
Decrease of natural killer cell rate
#
147
The immunological researches are carried out at the patient with clinical attributes of an immunodeficiency. Significant
decrease of the cells amount which form rosettes with erythrocytes of the ram is revealed. What conclusion should be
made on the basis of the data of the analysis?
@Decrease of the level of T-lymphocytes
Decrease of the level of B-lymphocytes
Decrease of the level of natural killers (NK-cells)
Decrease of the level of complement system
Failure of cells - effectors of human immunity
#
148
During the induction of an immune response, which cell is thought to process the initiating antigen?
Eosinophil
Basophil
@Macrophage
T cell
B cell
#
149
The transgression of function antigenepresentation to immunocompetent cells is revealed at the patient with clinical
attributes of an initial immunodeficiency. What defect of cells structures is possible?
@Macrophages, monocytes
T-lymphocytes
B-lymphocytes
Fibroblasts
α-lymphocytes
#
150
The defect is found out in the patient with clinical attributes of an immunodeficiency with not changed amount and
functional activity T-and B-lymphocytes at inspection on the molecular level at which function of antigenepresentation to
immunocompetent cells is broken. Which defect of cells structures is possible?
@Macrophages, monocytes
T-lymphocytes, B-lymphocytes
NK-cells
Fibroblasts, T-lymphocytes, B-lymphocytes
0-lymphocytes
#
151
In a patient with clinical signs of immunodeficiency the number and functional activity of T and B lymphocytes are not
changed. Defect with dysfunction of antigen-presentation to the immunocompetent cells was found during investigation
on the molecule level. Defect of what cells is the most probable?
Т-lymphocytes, В-lymphocytes
0-lymphocytes
Fibroblasts, Т-lymphocytes, В-lymphocytes
NK-cells
@Macrophages, monocytes
#
152
A patient with clinical presentations of primary immunodeficiency displays disturbance of antigen-presenting function by
immunocompetent cells. What cells may have structure defect?
0-lymphocytes
B-lymphocytes
Fibroblasts
@Macrophages, monocytes
T-lymphocytes
#
153
A 1-year-old child died of a relapsing pneumonia. A post-mortem revealed hypoplasia of a thymus and peripheral
lymphoid tissue, atrophy of cerebellum cortex, angiotelectasis of bulbar conjunctiva. Immunohistochemistry revealed
decreased level of immunoglobulins. What is the most likely disease?
@ Louis-Bar's syndrome
Di-George syndrome
Nezelofsyndrome
West's syndrome
Brutone syndrome
#
154
A child was born with cleft palate. Examination revealed aorta defects and reduced number of T-lymphocytes in blood.
What immunodeficient syndrome is it?
Wiskott-Aldrich
Chediak-Higashi
Louis-Bar
Swiss-type
@DiGeorge
#
155
A 3-month-old child died because of sepsis. An autopsy revealed thymic hypoplasia, both sizes and mass of spleen
reduction. Histologically, in spleen there was lack of the periarterial follicular T-zone and remarkable absence of cells at a
red spleeny pulp. Lymph nodes investigation showed also absence of cells within paracortical (T-cells area). B-cells areas
in immune system organs looked normal. What term best characterizes described above changes?
@Di George's syndrome
Combined immunodeficiency syndrome
Bruton's disease (X-linked agammaglobulinemia)
Accidental thymus transformation
Acquired immunodeficiency syndrome (AIDS)
#
156
During the section of a 6-month-old baby, who died of sepsis, were revealed the absence of thymus gland, the decrease in
mass and size of spleen. Microscopically there were detected the absence of periarterial T-cell-mediate zones of follicles
with the depredation of red pulp, the absence of pericortical zone of lymphatic nodes which was presented mostly by T-
lymphocytes. B-zones of the peripheral immune organs were developed due to the norm. On what pathologic process do
the facts indicate?
Glanzmann-Riniker’s syndrome (insufficiency of the cell and humoral immunity)
@DiGeorge syndrome (insufficiency of the cell immunity)
Bruton’s syndrome (insufficiency of the humoral immunity)
Accidental involution
HIV-infection
#
157
A 10-monfh-old child has developed pneumonia. Clinical and laboratory tests revealed absence of mature B-lymphocytes
in peripheral blood and in B-zones of lymph nodes, acute reduction of immunoglobulins in serum. In peripheral blood a
total quantity of lymphocytes was normal. What kind of an immunodeficiency developed in that case?
@Brutone sindrome
Neselof syndrome
Severe combined immunodeficiency
Di-George syndrome
Viskotta-Oldrich syndrome
#
158
A 12-year-old boy often suffers from virus and bacterial infections and eczematous skin lesions. Enlargement of T-
lymphocytes and IgM with normal IgA and IgG was revealed on examination. What type of immune system pathology is
presented in the patient?
Bruton's hypogammaglobulinemia
Hereditary immundeficiency of the complement system
@Composite immunedefficiency
Hypoplasia of thymus
Turner's syndrome
#
159
A patient in three weeks after acute myocardial infarction has pain in the heart and joints and pneumonia. What is the
main mechanism of development of post-infarction Dressler’s syndrome?
@Autoimmune inflammation
Vessels ' thrombosis
Secondary infection
Ischemia of myocardium
Resorption of enzymes from necrotized area of myocardium
#
160
For the purpose of myocardium infarction treatment a patient was injected with embryonal stem cells derived from this
very patient by means of therapeutic cloning . What transplantation type is it?
Allotransplantation
Isotransplantation
Heterotransplantation
@Autotransplantation
Xenotransplantation
#
161
A patient has a skin defect as a result of an extensive burn. In order to mask this defect the surgeons transplanted a skin
flap from other body part of this patient. What type of transplantation is it?
Homotransplantation
@Autotransplantation
Xenotransplantation
Allotransplantation
Explantation
#
162
During a histological research of skin transplant there was revealed a lymphoid and histiocytic infiltration along with
swelling and haemorrhages with macrophages and neutrophils involved. What is the most probable diagnosis?
Interstitial inflammation
Slow hypersensitivity reaction
Immediate hypersensitivity reaction
@Transplant tear away reaction
Artuce's phenomenon
#
Compensatory and adjustment processes.
#
1
Life cycle of a cell includes the process of DNA autoreduplication. As a result of it monochromatid chromosomes turn
into bichromatid ones. What period of cell cycle does this phenomenon fall into?
G1
@S
G2
Go
M
2
A 23-years-old patient got a lesion of the liver because of trauma. In time a structure and functions of that organ was
restored completely. What kind of regeneration did occur in that case?
@Restitution
Pathologic regeneration
Physiologic regeneration
Metaplasia
Substitution
#
3
Some time after traumatic destruction of liver it has restored its structure and function. What is the name for such
regeneration?
Substitution
Incomplete
Pathological regeneration
Usual physiological regeneration
@Restitution
#
4
A 42-year-old man had a lower extremity amputation. After a while painful knots appeared in a stump. Microscopical
investigation revealed amputation neuromas. What pathology such changes are characteristic for?
@ Pathological regeneration
Complete reparative regeneration
Inflammation
Incomplete reparative regeneration
Metaplasia
#
5
A pathological process in bronchi resulted in epithelium desquamation. What cells will regenerate bronchial epithelium?
Ciliate
Intercalary
Goblet
Endocrinal
@Basal
#
6
Histological examination in the area of neck of fundus gland reveals small cells that have high nuclear-cytoplasmatic ratio
and basophilic cytoplasm. Name the function of these cells:
Secretion of chlorine ions
Endocrine
@Regeneration of glandular epithelium
Pepsinogen secretion
Protective
#
7
Histological examination in the area of cervix of a fundic gland reveals small cells that have high nuclear-cytoplasmatic
ratio and basophilic cytoplasm. What is the function of these cells?
Pepsinogen secretion
@Regeneration of glandular epithelium
Protective
Secretion of chlorine ions
Endocrinal
#
8
Patient with injured muscles of the lower extremities was admitted to the traumatological department. Due to what cells is
reparative regeneration of the muscle fibers and restoration of the muscle function possible?
Myofibroblasts
Myoblasts
@Satellite-cells
Fibroblasts
Myoepithelial cells
#
9
During road accident at the man of 30 years the femoral artery has been injured. In hospital the arteriorraphy was
performed. Regeneration of what muscle tissue is necessary for regeneration of a wholeness of the vessel`s tunica media?
@Smooth muscle tissue
Striated sceletal muscle tissue
Striated cardiac muscle tissue
Myoepithelial cells
Satellite cells
#
10
A patient underwent Caesarean section. During the operation a long incision was made in the uterus wall and the fetus
was extracted from uterus. Healing of the sutured myometrium will proceed in the following way:
Hypertrophy of smooth myocytes
@Formation of a fibrous cicatrix
Proliferation of myosatellitocytes
Formation of smooth muscular tissue
Formation of cross-striated muscle fibers
#
11
In course of an experiment a big number of stem cells of red bone marrow were in some way destructed. Regeneration of
which cell populations in the loose connective tissue will be inhibited?
Pericytes
Pigment cells
Lipocytes
Fibroblasts
@Macrophags
#
12
A routine H&E histologic section from an irregular white area within the anterior wall of the heart of a 71-year-old man
who died secondary to ischemic heart disease reveals the myocytes to be replaced by diffuse red material. This material
stains blue with a trichrome stain. Which of the following statements correctly describes this material?
It is secreted by endothelial cells and links macromolecules to integrins
@It is secreted by fibroblasts and has a high content of glycine and hydroxyproline
It is secreted by hepatocytes and is mainly responsible for intravascular oncotic pressure
It is secreted by monocytes and contains a core protein that is linked to mucopolysaccharides
It is secreted by plasma cells and is important in mediating humoral immunity
#
13
A 47-year-old man presents with pain in the midportion of his chest. The pain is associated with eating and swallowing
food. Endoscopic examination reveals an ulcerated area in the lower portion of his esophagus. Histologic section of tissue
taken from this area reveal an ulceration of the esophageal mucosa that is filled with blood, fibrin, proliferating blood
vessels, and proliferating fibroblasts. Mitoses are easily found, and most of the cells have prominent nucleoli. Which of
the following statements best describes this ulcerated area?
Caseating granulomatous inflammation
Dysplastic epithelium
@Granulation tissue
Squamous cell carcinoma
Noncaseating granulomatous inflammation
#
14
In an experiment, surgical wound sites are observed following suturing. An ingrowth of new capillaries is observed to
occur within the first week. A substance elaborated by macrophages is found at the wound site to stimulate this capillary
proliferation. Which of the following substances is most likely to have this function?
Platelet-derived growth factor
Phospholipase C-gamma
Fibronectin
@Basic fibroblast growth factor
Epidermal growth factor
#
15
A chemical burn of esophagus caused it's local constriction as a result of scar formation. What cells of loose connective
tissue take part in scar formation?
Immature nonspecialized fibroblasts
Fibroclasts
Mature specialized fibroblasts
@Fibrocytes
Miofibroblasts
#
16
At the time of a laparoscopic cholecystectomy, the small incisions are closed with sutures. Over the next several weeks,
the wounds regain tensile strength and there is re-epithelialization. Of the following substances, the one that functions
intracellularly in cells involved in this process is:
Fibronectin
Laminin
Tyrosine kinase
Hyaluronic acid
@Collagen
#
17
A 19-year-old man incurs a stab wound to the chest. The wound is treated in the emergency room. Two months later there
is a firm, 3x2 cm nodular mass with intact overlying epithelium in the region of the wound. On examination the scar is
firm, but not tender, with no erythema. This mass is excised and microscopically shows fibroblasts with abundant
collagen. Which of the following mechanisms has most likely produced this series of events?
@Hypertrophic scar (keloid) formation
Development of a fibrosarcoma
Poor wound healing from diabetes mellitus
Foreign body response from suturing
Staphylocccal wound infection
#
18
A 40-year-old man has a keloid scar after a treatment of the left arm burn. This process may be an example of:
@Pathologic regeneration
Hyperplasia
Metaplasia
Reparative regeneration
Vicarious hypertrophy
#
19
Microscopical investigation of tissue from the edge of a chronic gastric ulcer showed necrosis, granulation tissue,
abundant development of connective tissue and metaplasia of epithelium. What type of pathology takes place in that case?
@Pathological regeneration with disorder of phases
Hypertrophy
Physiology regeneration
Reparative regeneration (substitution)
Reparative regeneration (restitution)
#
20
A 45-year-old patient complains about pain and haemorrhage from carious cavity 36 during her meal. During an
examination of 36: on the chewing surface there is deep carious cavity, covered by red-coloured tissue. While probing,
pain and haemorrhage occur. What kind of tissue grew into the carious cavity?
Fibrous
@Granulative
Adiposal
Epithelial
Muscle
#
21
A 59-year-old man has signs of the parenchymatous jaundice and portal hypertension. On histological examination of the
puncture of the liver bioptate, it was revealed: beam-lobule structure is affected, part of hepatocytes has signs of fat
dystrophy, port-portal connective tissue septa with formation of pseudo-lobules,with periportal lympho-macrophage
infiltrations. What is the most probable diagnosis?
Viral hepatitis
Alcohol hepatitis
Chronic hepatosis
@Liver cirrhosis
Toxic dystrophy
#
22
A child was diagnosed to have fracture of humerus. The broken arm began retarding in growth. What part of the bone was
damaged?
Apophysis
Epiphysis
@Metaphysis
Marrow canal
Diaphysis
#
23
The symptoms of regeneration process (callus) on the place of fracture were revealed in the histologic specimen of
tubular bone. What tissue forms this structure?
Loose connective tissue
Reticular tissue
Epithelial tissue
Lamellar bone tissue
@Fibrous bone tissue
#
24
Examination of a histological specimen of tubular bone revealed signs of regeneration process (callus). What tissue is this
structured formed of?
Lamellar osseous
Loose connective
@Rough fibrous osseous
Reticular
Epithelial
#
25
45 year-old male several months ago have had accidental fracture of the backbone with spinal cord damage. All these
month he has lied supine. What change in his muscles will develops?
@Metaplasia
Atrophy
Hyperplasia
Hypertrophy
Cell injury
#
26
A 43-year-old man has complained of mild burning substernal or epigastric pain following meals for the past 3 years.
Upper Gl endoscopy is performed and biopsies are taken of an erythematous are a of the lower esophageal mucosa 3 cm
above the gastroesophageal junction. There is no mass lesion, no ulceration, and no haemorrhage noted. The biopsies
demonstrate the presence of columnar epithelium with goblet cells. Which of the following mucosal alterations is most
likely represented by these findings?
Dysplasia
Hyperplasia
Carcinoma
Ischemia
@Metaplasia
#
27
An elderly man with a long history of smoking presented with a cough accompanied by viscid mucous sputum, weakness
after the insignificant physical activity and pale color of the skin. He lost 12,0 kg of weight for the last two months.
Microscopical investigation of endoscopic biopsy showed squamous cell carcinoma. What pathology preceded the tumor
formation?
@ Metaplasia
Hypoplasia
Hyperplasia
Necrosis
Sclerosis
#
28
A man, who abused nicotine smoking obtained following changes of his lungs: microscopically mucous membrane of
bronchi is made of non-cornifying multilayer epithelia. What is the name for the process in the lungs?
Hyperplasia
Hypertrophy
Regeneration
@Metaplasia
Organisation
#
29
The biopsy of central bronchus mucosa of miner is taken. Microscopically, there is a transformation of a cylindrical
epithelium to the mature stratified one. What pathology such changes are characteristic for?
@Metaplasa
Hyperplasia
Leucoplakia
Dysphasia
Reparative regeneration
#
30
At a patient with chronic bronchitis biopsy of bronchus mucous membrane was taken. Microscopical investigation
revealed the areas of "stratified epithelium without keratinization. What pathology takes place in the mucous membrane
of bronchus?
@Metaplasia
Atrophy
Hyperplasia
Dysplasia
-
#
31
A 60-year-old man has a history of chronic bronchitis. Histological investigation of bronchus biopsy showed thinned
mucous membrane, cystic transformation of mucous glands, and replacement of prismatic epithelium on stratified one.
What of the listed pathological processes the most likely?
@ Metaplasia
Hyperplasia
Heterotopia
Heteroplasia
Displasia
#
32
Chronic inflammation and transformation of the one-layer ciliated epithelium into multiple-layers flat epithelium was
revealed in the thickened mucous membrane of the bronchus bioptate of the patient with smoke abuse. Which of the
processes is the most likely?
@Metaplasia
Epithelium hypertrophy
Squamous cancer
Hyperplasia of the epithelium
Leucoplacia
#
33
At the man with long history of smoking the bronchial biopsy is taken. Microscopical investigation showed the thickened
mucous membrane with chronic inflammation and transformation of ciliated epithelium into stratified one. What
pathology such changes are characteristic for?
@Metaplasia
Hyperplasia of epithelium
Planocellular cancer
Leukoplakia
Hypertrophy of epithelium
#
34
A patient is an evil-smoker. In the biopsy material from the left bronchus, in the thickened mucous membrane was
revealed a chronic inflammation and a transformation of simple ciliated epithelium into stratified squamous one. What
process is meant?
Hyperplasia
@Metaplasia
Squamous cell carcinoma
Leukoplakia
Hypertrophy
#
35
An autopsy study is performed to identify predisposing risks for tissue changes in which epithelial metaplasia occurred. In
which of the following situations is the process of epithelial metaplasia most likely to have occurred?
Tanning of the skin following sunlight exposure
Lactation following pregnancy
Vitamin adeficiency
Acute myocardial infarction
@Urinary obstruction from an enlarged prostate
#
36
During the histological examination of the fragment of mucous membrane of urine bladder of the patient suffered with
chronic cystitis it was found out: the transitional epithelium focally is replaced by stratified non-keratinized epithelium.
These changes can manifest development of:
@Metaplasia
Dysplasia
Leukoplakia
Hyperkeratosis
Hyperplasia
#
37
At a patient with a chronic cystitis biopsy of urinary bladder mucosa was taken. Microscopical investigation revealed a
transitional epithelium with foci of stratified epithelium without keratinization. What process underlies the described
changes in an epithelium?
@Metaplasia
Dystrophy
Hyperplasia
Dysplasia
Hyperkeratosis
#
38
The biopsy of a vaginal portion of uterus cervix is taken. There is an augmentation of quantity of cells, rising of mitotic
activity and alteration of polarity of cells in basal part of an epithelial layer. What pathology such changes are
characteristic for?
@Dysplasia
Metaplasia
Atrophy
Acantosis
Hyperplasia
#
39
In a clinical study, patients undergoing laparoscopic cholecystectomy are followed to document the post-surgical wound
healing process. The small incisions are closed with sutures. Over the 4 weeks following surgery, the wounds are
observed to regain tensile strength and there is re-epithelialization. Name this type of wound healing?
@By primary intention
By secondary intention
Under scab
Defects epithelial shutting
Scarring
#
40
A patient ill with diabetes mellitus went through an operation on account of abscess in the area of posterior part of his
neck. The wound healing lasted for a month and a half; the wound constantly discharged pus. On the site of the healed
wound there appeared an irregular scar. In what way did the wound healing take place?
@By secondary intention
Under the crust
By primary intention
By epithelization
By combined intention
#
41
A patient has deep lacerated wound with uneven margins, covered with pus. Marginal parts are filled with rich
granulating tissue that does not rise above the wound level. Name the form of the wound healing.
Organization
Healing by first intention
Healing under eschar
Immediate closing of the epithelial tissue’ defect
@Healing by second intention
#
42
A patient has deep torn wound with uncertain edges, covered with pus. In terminal areas there is juicy granular tissue,
which does not bulge above the wound. Name the wound healing process.
Primary tension healing
@Secondary tension healing
Healing underneath a slough
Direct epithelia defection closure
Wound organisation
#
43
A 20-year-old woman sustains an injury to her right calf in a mountain biking accident near Moab. On physical
examination she has a 5 cm long laceration on the right lateral aspect of her lower leg. This wound is closed with sutures.
Name possible wound healing?
By primary intention
@By secondary intention
Under scab
Defects epithelial shutting
Scarring
#
44
An injury is sustained to the right calf of a 20-year-old woman involved in a mountain biking accident near Moab. There
is a 5-cm long laceration noted on examination. This wound is closed with sutures. Wound healing over the next week
will be delayed the LEAST by:
@Wound infection
Poor tissue perfusion
Sutures
Corticosteroid therapy
Hypoalbuminemia
#
45
The microscopic examination of wound lavage of a patient with acute woundy process of his shin revealed big contents of
irregular extended-formed cells, with tough nucleus, the basophilic cytoplasm of which includes many lysosomes,
phagosomes and pinocytotic bubbles. What cells are found out in the wound?
Tissue basophils
@Connective tissue macrophages
Fibrocytes
Fibroblasts
Plasmocytes
#
46
A 31-year-old woman has a laparotomy performed for removal of an ovarian cyst. She recovers uneventfully, with no
complications. At the time of surgery, a 12 cm long midline abdominal incision was made. When you advise this patient
of the outcome of this healing process, you indicate that nearly all of the tensile strength to be obtained in wound healing
in the skin will be most likely be achieved within which of the following time periods?
One week
One month
@Three months
Six months
One year
#
47
A 20-year-old woman had Goodpasture syndrome which progressed to chronic renal failure. She was 165 cm tad and
weighed 55 kg. She had blood pressure measurements in the range of 150/90 to 180/110 mm Hg, but she did not regularly
take medications. Laboratory studies showed her blood urea nitrogen was over 100 mg/dL. She required chronic dialysis.
She died from heart failure. At autopsy, her heart weighed 540 gm. The size of her heart is most likely to be the result of
which of the following processes involving the myocardial fibers?
@Hypertrophy
Fatty infiltration
Hyperplasia
Fatty degeneration
Edema
#
48
In course of a preventive examination of a miner a doctor revealed changes of cardiovascular fitness which was indicative
of cardiac insufficiency at the compensation stage. What is the main proof of cardiac compensation?
Cyanosis
@Hypertrophy
Dyspnea
Tachycardia
Rise of arterial pressure
#
49
During the preventive examination of a miner a doctor revealed changes in cardiovascular fitness being evidence of
cardiac insuffisiency at a stage of compensation. What is the main evidence of compensation of cardiac activity?
@Myocardium hypertrophy
Dyspnea
Tachycardia
Cyanosis
Rise of arterial pressure
#
50
A 62-yer-old male died at the hospital after developing heart failure. At autopsy the weight of the heart was 500gm, with
thickened right ventricular wall and dilated chamber. Name the alteration at the right ventriculum.
@Hypertrophy
Hyperplasic alteration due to inflammation
Metaplasia
Atrophy
Sclerosis
#
51
A patient, who died of lung-cardiac failure, has his heart increased in size, right ventricle wall is thickened, and cavity is
dilated. Name the pathological process.
Sclerosis
Hyperplastic growth due to inflammation
Metaplasia
Atrophy
@Hypertrophy
#
52
Dystrophic changes of the heart muscle are accompanied with cardiac cavity enlargement, decrease of the strength of
heart contraction, increased amount of blood, which remains in the heart during systolic phase, overfilled veins. For what
state of heart is it characteristic?
Tonogenic dilatation
Emergency stage of hyperfunction and hypertrophy
@Myogenic dilatation
Cardiosclerosis
Tamponage of the heart
#
53
During the section of a 57-year old man, who had hypertensive disease and died of cardiac decompensation, an enlarged
heart with dilated cavities was diagnosed (mass – 550g, thickness of the left ventricle wall – 2,5cm). Microscopically:
cardiomyocytes were enlarged in size with lipoid dystrophy and hyperchromic barrel nuclei. What is the most possible
pathological process?
Myocarditis
@Eccentric hypertrophy
Concentric hypertrophy
Angiogenic cardiosclerosis
Cardiomyopathy
#
54
A 48-year-old patient with a history of a hypertensive disease died of heart failure. An autopsy revealed an enlarged heart
with dilated chambers. A thickness of the left ventricle wall was 2,5cm. Microscopically myocardial cells were
considerably enlarged, with fatty dystrophy and hyperchromic barrel-shaped nuclei. What is the most likely pathology
diagnosed in a heart?
@Excentric hypertrophy
Myocarditis
Concentric hypertrophy
Angiogenic cardiosclerosis
Cardiomyopathy
#
55
The lifestyle patterns of healthy persons from 20 to 30 years of age are studied. A subset of these persons have a lifestyle
characterized by consumption of a lot of pizza and very little physical exercise. Which of the following tissue changes is
most likely to develop in this subset of persons as a consequence of this lifestyle?
Fatty metamorphosis of liver
Pancreatic fat necrosis
Fatty degeneration of myocardium
@Hypertrophy of steatocytes
Metaplasia of muscle to adipose tissue
#
56
Young woman died after the labour in the case of sepsis.On the autopsy uterus has 22x17x10 sm in size, 1200 gramm in
mass,microscopically it is detected the enlargement of the volume of the smoth muscle fibers,hypertrophy of the nuclei.
@Hormonal hypertrophy
Working hypertrophy
Vicary hypertrophy
Pathologic hypertrophy
Protective hypertrophy
#
57
A 59-year-old patient with a history of prostate glandular hyperplasia operated in the urology. An operation revealed a
thickening of a urinary bladder wall up to 1 cm. It may be caused by:
@Work hypertrophy
Vicarious hypertrophy
Hormonal hypertrophy
Hormonal hyperplasia
Protective hyperplasia
#
58
An elderly man died of cardiac insufficiency. Twenty years ago he had the right lung pulmonectomy because of a cyst. A
post-mortem revealed an enlarged left lung. Diagnose pathology in the left lung.
@Vicarious hypertrophy
Neurohumoral hypertrophy
Dysfunctional atrophy
Dyscirculatory atrophy
Neurotic atrophy
#
59
A man had the right lung pulmonectomy because of a tumor 7 years ago. Then the volume of the left lung increased on
40%. What process developed in the left lung?
@Vicarious hypertrophy
Neurohumoral hypertrophy
False hypertrophy
Work hypertrophy
Hypertrophic vegetation
#
60
A patient had been performed a resection of his right lung because of neoplasm. Some time after volume of his left lung
increased by 50%. What's the name of the process in the left lung?
@Vicar hypertrophy
Neurohumoral hypertrophy
False hypertrophy
Working hypertrophy
Hypertrophic growth
#
61
A 42-year-old woman presented to the gynecologist with acyclic, profuse hemorrhagic discharge. A histological
investigation of a curettage material revealed increased quantity of glands and their cystic dilation. What is the most likely
diagnosis?
@ Endometrial hyperplasia
Endometrial atrophy
Endometrial hypertrophy
Metaplasia
Organization
#
62
Pathohistologic examination of endometrium of 58 year-old female who suffers to alternate bleeding and amenorrhea has
revealed: the amounts of endometrial glands as well as stroma cells are increased. What is the name of this condition of
endometrium?
Metaplasia
Neoplasia
@Hyperplasia
Hypertrophy
Inflammation
#
63
A histological investigation of an endometrium revealed coiled extended glands with 'saw-' and a 'spin-like' pattern. A
stromal proliferation with hyperplasia of its cells was also determined. What is the most likely diagnosis?
@ Glandular hyperplasia of endometrium
Acute endometritis
Leiomyoma
Hydatidiform mole
Placental polyp
#
64
A woman suffered with dysfunctional metrorrhagia was made a diagnostic abortion. Histologically in the scrape there
were a lot of small stamped glandulars covered by multirowed epithelium. The lumens of some glandulars were cystically
extended. Call the variant of general pathologic process in the endometrium.
@Glandular-cystic hyperplasia of endometrium
Atrophy of endometrium
Metaplasia of endometrium
Neoplasm of endometrium
Hypertrophic growth
#
65
A 38-year-old woman presented with complaints about the frequent uterine bleeding. The diagnostic curettage was
performed. A histological investigation of a curettage material revealed increased quantity of coiled glands, some of them
were with cystic dilation. What pathology these changes are characteristic for?
@ Glandular-cystic hyperplasia
Atrophy
Metaplasia
Dysplasia
Hypertrophic vegetations
#
66
Diagnostic scraping was performed to the woman with dysfunctional uterine bleeding. Multiple convoluted glands,
ganglially dilated lumens of some glands were revealed histologically in the scrape. Name the type of general
pathological process in endometry:
Hypertrophic excrescence
Displasia
Metaplasia
Atrophy
@Hyperplasia glandulocystica
#
67
At the histological examination of a smear from uterine mucosa of a 45-year-old woman who had a disordered ovario-
menstrual cycle, there was determined an increased amount of endometrial glands, some of them were saw like
anfractuous, others – cystic-widened. Diagnose the disease.
@Glandular-cystic hyperplasia of endometrium
Placental polyp
Atypical hyperplasia of endometrium
Glandular polyp of endometrium
Adenocarcinoma of endometrium
#
68
Diagnostic scraping was performed to the woman with dysfunctional uterine bleeding. Multiple convoluted glands,
ganglially dilated cavities of some glands were revealed histologically in the scrape. Name the type of general
pathological process.
Displasia
Hypertrophic excrescence
Metaplasia
@Glandulo-gangliac hyperplasia
Atrophy
#
69
78 year-old male has complained the retention of urine, intermittent oligouria and incomplete urination. After
examination the enlargement of prostate and compression of urethra were revealed. What process led to age-related
enlargement of prostate:
Metaplasia
Atrophy
@Hyperplasia
Hypertrophy
Neoplasia
#
70
A 19-year-old woman gives birth to her first child. She begins breast feeding the infant. She continues breast feeding for
almost a year with no difficulties and no complications. Which of the following cellular processes that occurred in the
breast during pregnancy allowed her to nurse the infant for this period of time?
Stromal hypertrophy
Epithelial dysplasia
Steatocyte atrophy
Ductal epithelial metaplasia
@Lobular hyperplasia
#
71
Histologic sections of an enlarged tonsil from a 9-year-old girl reveal an increased number of reactive follicles containing
germinal centers with proliferating B lymphocytes. Which of the following terms best describes this pathologic process?
B lymphocyte hypertrophy
Follicular dysplasia
@Follicular hyperplasia
Germinal center atrophy
Germinal center metaplasia
#
72
A patient with a history of frequent hemorrhoid bleeding died of acute myocardial infarction. A post-mortem revealed a
red, succulent bone marrow of a hip diaphysis. What pathology developed in a bone marrow?
@ Compensatory hyperplasia
Vicarious hypertrophy
Hypertrophy excrescences
Hormonal hyperplasia
Work hypertrophy
#
73
A 38-year-old man incurs a traumatic blow to his upper left arm. He continues to have pain and tenderness even after 3
months have passed. A plain film radiograph reveals a 4 cm circumscribed mass in the soft tissue adjacent to the humerus.
The mass contains areas of brightness on the x-ray. Over the next year this process gradually resolves. Which of the
following terms best describes this process?
Dysplasia
@Hyperplasia
Hypertrophy
Metaplasia
Neoplasia
#
74
Usually the patients with sepsis have spleen and liver enlargement. What process is underlying this state?
Metaplasia
Neoplasia
@Hyperplasia
Hypertrophy
Pigment accumulation
#
75
The left lobe of an adult donor liver is used as an orthotopic transplant in a recipient relative with severe liver disease. A
year later, the size of each liver is greater than at the time of transplantation. Which of the following mechanisms explains
this phenomenon?
Metaplasia
Dysplasia
@Hyperplasia
Anaplasia
Neoplasia
#
76
In a 40-year-old man, who has a stenosing (without metastases) cancer of the esophagus, the following changes can be
observed: the atrophy of the skeleton muscles and of subcutaneous fat. The skin is of ground black colour, epidermis is
thinned, and the heart is regressed in size. The myocardium and the liver are of brown colour. Make a diagnosis.
Addison’s disease
Myasthenia
Cachexy caused by cancer
Brown atrophy
@Alimentary cachexy
#
77
Examination of a 40 y.o. man ill with stenosing (without metastases) esophageal carcinoma revealed the folowing
changes: atrophy of skeletal muscles and fatty tissu E. His skin is sallow, epidermis is attenuated, heart has grown
smaller. Myocardium and liver are brown. What is the most probable diagnosis?
Brown atrophy
@Alimentary cachexia
Myasthenia
Cancerous cachexia
Addison's disease
#
78
A man with stenotic oesophageal cancer gained the following: atrophy of skeletal muscle and fat cellulose, skin is brown
colour, thin epidermis, heart is diminished in size, myocardium and liver are brown coloured. Name the diagnosis.
@Alimentary cachexia
Myasthenia
Cancer cachexia
Brown atrophy
Addison's disease
#
79
A patient, who had a cancer of the stomach, died of cancer cachexy. At the autopsy the characteristic changes were
detected. How can we call such heart?
Bony heart
Hairy heart
@Brown atrophic heart
Tiger heart
Bovine heart
#
80
During an autopsy of a man: heart size is diminished, fat cellulose under epicardium is absent, myocardium is thick,
brown coloured. Microscopically: cardiomyocytes are diminished, sarcoplasm is full of brown grains of lipofuscin. Name
the pathological process.
@Brown atrophy
Amyloidosis
Oil dystrophy
Haemochromatosis
Hypertrophy
#
81
During an autopsy of a man: heart size is diminished, fat cellulose under epicardium is absent, and myocardium is thick,
brown coloured. Microscopically: cardiomyocytes are diminished; sarcoplasm is full of brown grains of lipofuscin. Name
the pathological process.
Hypertrophy
Amyloidosis
Oil dystrophy
Haemochromatosis
@Lipofuscinosis
#
82
A 31-year-old primigravida has a difficult delivery of a term infant. There is loss of 1500 cc of blood. She has
hypotension for 6 hours. Over the next month, her ACTH level decreases. Within the next 3 months, her adrenal glands
become only about 2 grams each (normal 4 to 6 grams). This alteration of the adrenals is primarily due to which of the
following cellular processes?
Metaplasia of the cortex
Lipid depletion of the cortex
Atrophy of the medulla
@Atrophy of the cortex
Adrenal infarction
#
83
A 7-year-old child presented with a poliomyelitis. His somatic muscles are weak, their volume is reduced, and skin is dry
and pale. Choose the most likely pathology, which takes place in the soft tissues?
@ Atrophy
Hypertrophy
Hyperplasia
Metaplasia
Hypoplasia
#
84
There were adenoma of the prostate and big kidneys with enlarged pelves and calyces, filed with clear fluid on the
autopsy. Name the process in the kidneys.
@Atrophy
Inflammation
Hypertrophy
Tumor
Necrosis
#
85
78 year-old male has complained the retention of urine, intermittent oligouria and incomplete urination. After
examination the enlargement of prostate and compression of urethra were revealed. Urography shows the bilateral
dilation of ren pelvis and caps, as well as diminishing of kidney parenchyma. What process led to these kidney changes?
Metaplasia
@Atrophy
Hyperplasia
Hypertrophy
Cell injury
#
86
During a histological research of a removed tooth: sclerosis of conjunctive tissue of the pulp, decreased amount of
odontoblasts of reduced size and pulpocytes. Name the diagnosis.
Pulp dystrophy
Pulp necrosis
@Pulp atrophy
Pulp hyalinosis
Acute pulpitis
#
87
An autopsy of an elderly male revealed the atherosclerosis of the brain arteries accompanied with the thinning of the
brains cortex. Name the cause of the atrophy?
@Ischemia
Compression
Chemical and physical factors
Loss of innervations
Decreased workload
#
88
During an autopsy of a 69 year old woman, who has been suffering from chronic hypertension there was revealed, that
both kidneys are of thick consistency, shrunken and with granular surface. How these changes are called?
@Atrophy due to ischemia
Atrophy due to pressure
Elderly atrophy
Dysfunctional atrophy
Hypoplasia
#
89
A post-mortem of a 64-year-old woman with a long story of hypertensive disease revealed considerably diminished,
dense kidneys with a fine-grained surface. What pathology such changes are characteristic for?
@ Blood insufficiency atrophy
Atrophy from pressure
Senile atrophy
Dysfunctional atrophy
Hypoplasia
#
90
During an autopsy of a 86 year old woman, who suffered from atherosclerosis of cerebral vessels there was cerebral
cortex atrophy found. What is the name of such atrophy?
@Atrophy due to insufficient blood supply
Atrophy due to pressure
Atrophy due to chemical and physical factors
Neurotic atrophy
Dysfunctional atrophy
#
91
Pathohistologic examination of endometrium of 64 year-old menopausal female has revealed: the amount of endometrial
glands as well as stroma cells are decreased, stroma replaced by connective tissue, endometrial glands predominantly
have small size. What is the name of this condition of endometrium?
Inflammatory metaplasia
Virus-induced neoplasia
Age-related hyperplasia
@Hormonal atrophy
Work hypertrophy
#
92
A teenager complains of muscular atrophy and negative volume of the shank after a long healing up of the femur fracture
without nerve damage. How can we term such atrophy?
Of compression
Neurotic
Of short blood supply
@Dysfunctional
Of physical factors’ influence
#
93
After long-healing fracture of right thigh bone (nerves weren't damaged), a teenager complains on muscles getting thin
and decrease in volume of his right leg. What's the name for such atrophy?
Neurotic
@Dysfunctional
Caused by ischemia
Caused by pressure
Caused by physical factors
#
94
After removal of tooth patient gained an atrophy of edges of tooth pocket. What kind of atrophy is this?
Ischemic atrophy
@Dysfunctional atrophy
Atrophy due to pressure
Neurotic atrophy
Atrophy due to physical and chemical factors
#
95
A young male complains of diminishing leg muscles in size and volume as a result of the complicated femoral fracture.
The innervations of the muscles was not lost. Name the type of atrophy?
@Disuse atrophy
Degeneration atrophy
Ischemic atrophy
Atrophy due to compression
Atrophy due to chemical and physical influence
#
96
Histological examination of a 40 y.o. man's thymus revealed decreased share of parenchymatous gland elements,
increased share of adipose and loose connective tissue, and its enrichment with thymus bodies. The organ's mass was
unchanged. What phenomenon is it?
@Age involution
Hypotrophy
Atrophy
Dystrophy
Accidental involution
#
97
A 4-years-old girl was operated because of an acute appendicitis. During the operation in the retroperitoneal space it was
found that the right kidney is less by 1/3 in comparison with the left one. The diameter of the right renal artery was 0,3
cm, the left one was 0,4 cm. What pathologic process did occur in that case?
@Hypoplasia
Pathologic atrophy
Physiologic atrophy
Aplasia
Agenesia
#
TUMOURS
Epithelial tumour
#
1
Continuous taking of some drugs foregoing the pregnancy increase the risk of giving birth to a child with genetic defects.
What is this effect called?
Embryotoxic effect
Blastomogenic effect
@Mutagenic effect
Fetotoxical effect
Teratogenic effect
#
2
A 56-year-old man has had a chronic cough for the past year. He is a non smoker. He recently had an episode of
hemoptysis. A chest x-ray demonstrates a 6-cm perihilar mass. A sputum sample is collected, and the sputum cytology
report reads "Atypical cells present -suggestive of squamous cell carcinoma. Which of the following environmental
exposures is most likely to be associated with these findings:
@Asbestos
Radon
Silica
Benzene
Formaldehyde
#
3
Of the following characteristics of neoplasms, which implies the best prognosis:
Increased expression of laminin receptors
Increased cathepsin expression
Decreased apoptosis
@Decreased doubling time
Decreased nuelear/cytoplasmic ratio
#
4
A biopsy is performed on a patient with a mass lesion that proves to be a neoplasm. Of the following histopathologic
findings, the one that best indicates that a neoplasm is malignant is:
Pleomorphism
Atypia
Invasion
@Increased nuclear/cytoplasmic ratio
Necrosis
#
5
A 65-year-old woman has a firm mass with irregular borders felt in her left breast on a routine physical examination. A
fine needle aspiration is performed and microscopic examination shows an infiltrating ductal carcinoma. A left
mastectomy with axillary lymph node dissection is performed. A tissue sample of this neoplasm is submitted for analysis
by flow cytometry. Which of the following does flow cytometric analysis most likely?
@Determination of aneuploidy
Determination of the karyotype
Diagnosis of benign neoplasms
Distinguishing carcinoma from sarcoma
Quantitation of RNA content
#
6
An epidemiologic study is performed to determine risk factors for development of malignant neoplasms. A statistical
analysis of preexisting medical conditions is done. Some pre-existing conditions are observed to precede development of
malignant neoplasms, while others do not. Which of the following conditions is most likely to be statistically unrelated to
subsequent malignancy?
Endometrial atypical hyperplasia
Chronic alcoholism with hepatic cirrhosis
@Cervical squamous dysplasia
Chronic ulcerative colitis
Uterine leiomyomas
#
7
A 21-year-old woman has a routine Pap smear performed for a health screening examination. The pathology report
indicates that some cells are found cytologically to have larger, more irregular nuclei. A follow-up cervical biopsy
microscopically demonstrates disordered maturation of the squamaous epithelium, with hyperchromatic and pleomorphic
nuclei extending nearly the full thickness of the epithelial surface. No inflammatory ccells are present. Which of the
following descriptive terms is best applied to these Pap smear and biopsy findings?
@Dysplasia
Metaplasia
Anaplasia
Hyperplasia
Aplasia
#
8
At the histological examination of a formation was detected that parenchyma was formed of integumentary epithelium
with an increased amount of strata. Stroma together with the epithelial proliferation forms papillae. Term the form of
atypism.
Metabolic
Cell
Hystochemical
Functional
@Tissue
#
9
A teenager male presents with slowly enlarging, painless nodule on his right hand's skin. Microscopic examination of
removed lesion revealed an increased number of epithelial layers, with a stroma underneath, with developed a
papillomatous pattern. Identify most likely type of atypia?
@ Tissue
Cellular
Metabolic
Functional
Hystochemical
#
10
A 23-year-old female decided to have a surgeon's removal of a small nodule at her leg's skin. An operation material
delivered to pathology department. A histological examination of a new growth has shown that parenchyma has been
formed of integumentary epithelium with an increased amount of layers. Stroma together with epithelial proliferation
forms papillae. What is the most likely form of atypism?
@Tissue
Cellular
Histochemical
Functional
Metabolic
#
11
During a histological research of skin neoplasm: parenchyma is made of covering epithelia with increased amount of
layers in it. Stroma along with epithelia forms papillae. Name the type of atypism.
@Tissue
Cell
Histochemical
Functional
Metabolic
#
12
A study is performed to analyze characteristics of malignant neoplasms in biopsy specimens. The biopsies were
performed on patients who had palpable mass lesions on digital rectal examination. Of the following microscopic
findings, which is most likely to indicate that the neoplasm is malignant?
Pleomorphism
@Atypia
Invasion
Increased nuclear/cytoplasmic ratio
Necrosis
#
13
A 53-year-old woman feels a lump in her right breast. Her physician palpates an irregular 3 cm mass that is not movable
because it appears fixed to the overlying skin, which is retracted. A mastectomy is performed and the pathologist on
sectioning the breast finds a 3 x 3.5 cm ovoid mass that does not have discrete borders, but appears to infiltrate into the
surrounding fibrofatty breast stroma. The mass is firm, white, and has a fibrous consistency. Which of the following
features is most likely demonstrated by the gross appearance of this mass?
@Anaplasia
Aplasia
Desmoplasia
Dysplasia
Metaplasia
#
14
A 37-year-old woman complained of prolonged cramps, nausea, vomiting, diarrhea, and episodic flushing of the skin. At
autopsy, pearly-white, plaquelike deposits were found on the tricuspid valve leaflets. These cardiac lesion most likely
were due to
Rheumatic heart disease
Amyloidosis
Iron overload
Hypothyroidism
@Carcinoid heart disease
#
15
A 59-year-old man has had a worsening cough with chest pain for the past 6 months. On physical examination he has no
remarkable findings. A chest x-ray shows a 3 cm left lung mass. A sputum cytology specimen yields cells diagnosed as a
squamous cell carcinoma. A mediastinoscopy is performed and reveals metastases in a lymph node. He is given radiation
therapy, and the mass diminishes in size. Which of the following classifications best indicates the stage of his disease?
@T1N1M0
T1N0M1
T2N1M0
T3N0M0
T4N1M1
#
16
A 66-year-old man has a routine physical examination and a stool sample proves positive for the presence of occult blood.
He undergoes colonoscopy and a 5 cm sessile mass is present in the sigmoid colon. Biopsy of the mass yields a diagnosis
of adenocarcinoma. A chest x-ray shows multiple 1 to 3 cm nodules in both lungs. Presence of which of the following in
the neoplastic cells is most likely to explain the presence of lung nodules?
T1N1M0
@T1N0M1
T2N1M0
T3N0M0
T4N1M1
#
17
A lump in the right breast is discovered on physical examination of a 48-year-old woman. The mass is 4 cm in diameter
and appears fixed to the chest wall. Another 2-cm mass is palpable in the left axilla. A chest radiograph reveals multiple
0.5 to 2-cm nodules in both lungs. Which of the following classifications best indicates the stage of her disease:
T1 N1 MO
T1 NO Ml
T2 N1MO
T3 NO MO
@T4 N1 M1
#
18
A 44-year-old woman notes a lump in her left breast while taking a shower. Her physician notes a 3 cm firm, irregular,
non-movable mass located in the upper outer quadrant of her left breast on physical examination. A fine needle aspiration
of this mass is performed. Cells obtained from the mass are examined cytologically and are consistent with infiltrating
ductal carcinoma. The mass is removed with lumpectomy along with an axillary lymph node dissection. Which of the
following findings will best predict a better prognosis for this patient?
The tumor cells are strongly estrogen receptor positive
@No metastases are found in the sampled lymph nodes
Flow cytometric analysis demonstrates aneuploidy and a high S-phase
She has one relative who had a similar type of breast cancer
The tumor has a high grade
#
19
A 66-year-old woman has developed a chronic cough over the past 2 months. No abnormal physical examination findings
are noted. A chest x-ray shows a 3 cm mass in the right upper lobe. After fine needle aspiration biopsy, the mass is
resected. Gross and microscopic examination of the mass is performed and it is determined that a neoplasm is present.
Which of the following findings regarding this mass is most likely to be irrelevant for determining further therapy and
prognosis?
Is the neoplasm invading the margin of resection?
What is the size of the neoplasm?
@What is the degree of atypia and pleomorphism of the neoplastic cells?
How much inflammation is present in the neoplasm?
Is the neoplasm primary or metastatic?
#
20
A surgical pathology report indicates that a certain neoplasm is graded as grade I on a scale of I to IV. Clinically, this
neoplasm is found to be stage I. These findings imply that this neoplasm:
Is unlikely to be malignant
Has probably arisen from epithelium
May spread via lymphatics
Has an in situ component
@Is well-differentiated and localized
#
21
Preventive examination of a patient revealed an enlarged lymph node of metastatic origin on the medial wall of the left
axillary crease. Specify the most likely localization of the primary tumour:
Submandibular salivary gland
Stomach
Lung
Thyroid gland
@Mammary gland
#
22
A 55-year-old man dies after a year-long illness. At autopsy the liver contains multiple tumor masses from 2 to 5 cm in
size that are mostly firm and tan and that grossly exhibit umbilication with central necrosis. Which of the following
statements would best characterize the significance of such an appearance?
There is multicentric origin of a benign neoplasm.
The neoplasm has a high grade.
@The primary neoplasm is in the stomach.
A carcinogen was the underlying cause for the neoplasm.
The neoplasm has an advanced stage.
#
23
A 39-year-old woman presented in oncology with a stomach cancer. Physical investigation revealed metastatic tumors in
ovaries (Krukenberg tumors). What is most likely pattern of spread in that case?
@Lymphatic retrograde
Lymphatic orthograde
Gematogenous
Implantation
Along epithelium-lined surfaces
#
24
A 60-year-old man who has a 90-pack year history of cigarette smoking has had a chronic cough for years. He has
recently begun to lose weight. He has a chest radiograph that reveals a right hilar mass. A sputum cytology shows
atypical, hyperchromatic squamous cells. What is the most common initial pathway of spread of this lesion:
Bloodstream
Pleural cavity
Contiguous spread to chest wall
@Lymphatics
Bronchi
#
25
A post-mortem of a 48-year-old woman with a history of an operated stomach tumour in the past revealed markedly
enlarged, dense, whitish color ovary. Histological investigation of the ovarian tissue showedutterly atypical epithelial
cells, placed among layers and cords of a connective tissue. What if the most likely disease?
@Krukenberg's carcinoma of ovary
Serous cystadenocarcinoma
Pseudomucinous cystcarcinoma
Malignant thecoma
Malignant granular cell tumor
#
26
A physical examination of a 42-year-old patient revealed enlarged supraclavicular lymph nodes. A histological
investigation of a lymph node biopsy showed the metastasis of a signet-ring cancer. Choose the most probable
localization of a primary tumour.
@Carcinoma of stomach
Cancer of esophagus
Cancer of thyroid gland
Carcinoma of lungs
Carcinoma of uterine cervix
#
27
A post-mortem of a 59-year-old man, who died of a lung cancer, revealed plural metastases. What kind from the listed
below metastases it is possible to regard as implantation (contact) one according to a mechanism of development?
@Multiple tumorous nodules of pleura
Metastasis in prebronchial, paratracheal lymphatic knots
Metastasis in a brain
Metastases in an adrenal gland
Invasion of tumor from bronchus in an esophagus
#
28
A 60-year-old man who has a 90 pack year history of cigarette smoking has had a chronic cough for the past 10 years. He
has begun to lose weight during the past year. No abnormal findings are noted on physical examination. He has a chest
radiograph that reveals a right hilar mass. A sputum cytology shows atypical, hyperchromatic squamous cells. What is the
most common initial pathway of spread of this lesion?
Bloodstream
Pleural cavity
@Contiguous spread to chest wall
Lymphatics
Bronchi
#
29
A mass lesion in the right upper lobe of lung found by chest radiograph is removed by wedge resection and sent to
Surgical Pathology, where a neoplasm is diagnosed. Findings that will be important for determining further therapy and
prognosis include all of the following EXCEPT:
Is the neoplasm invading the margin of resection?
What is the size of the neoplasm?
What is the degree of atypia and pleornorphism of the neoplastic cells?
@How much inflammation is present in the neoplasm?
Is the neoplasm primary or metastatic?
#
30
An autopsy of a 50-year-old man, who died of cancer intoxication, revealed the thickening of a stomach's wall to 1, 2 cm.
The mucosa was fixed, with ill-defined gastric folds. On a cut view a tissue was homogeneous, whitish with chondroid
density. For what macroscopical form of a tumour the described changes are characteristic?
@Infiltrate
Node
Ulcer
Ulcerous-infiltration
Cyst
#
31
During an autopsy of man, who died of cancer intoxication: gaster wall is thickened (1,2cm), mucous membrane is
immovable and has no folds. On incision is homogenous, white coloured, has consistency of cartilage. What morphologic
kind of tumour is this?
@Infiltration
Node
Ulcer
Ulcer-infiltrative
Cyst
#
32
A 45 year old woman has a chest x-ray taken and is found to have a peripheral "coin lesion" that is a rounded,
circumscribed 2.5 cm mass in the right mid-lung field. She has no chest pain, cough, or fever. The biologic characteristic
that best distinguishes this lesion as a neoplasm, rather than a granuloma is:
Recurrence following excision
Rapid increase in size
Sensitivity to radiation or chemotherapy
@Uncontrolled (autonomous) growth
Necrosis
#
33
A 45-year old male with a family history of a gastric cancer inquires about screening. A gastroscopic examination of a
patient revealed a pedunculated tumor mass 1.5 cm in diameter in the area of the lesser curvature of stomach. What kind
of growth does the tumor have?
@Exophytic
Expansive
Infiltrating
Appositional
Endophytic
#
34
Pre-malignant conditions include all of the following conditions EXCEPT:
Endometrial hyperplasia following prolonged estrogen therapy
Chronic alcoholism leading to micronodular cirrhosis of the liver
Cervical squamous dysplasia
Chronic ulcerative colitis
@Multiple leiomyomas of the uterine myometrium
#
35
A 40-year-old woman has had a feeling of abdominal discomfort for the past 8 months. On pelvic examination, there is a
right adnexal mass. An abdominal CT scan demonstrates a 7 cm cystic mass involving the right ovary with small areas of
calcification. The uterus is normal in size. The right fallopian tube and ovary are removed surgically. Grossly, the mass
on sectioning is filled with abundant hair and sebum. Microscopically, the mass has glandular spaces lined by columnar
epithelium, squamous epithelium with hair follicles, cartilage, and dense connective tissue. Which of the following
statements regarding this type of neoplasm is most appropriate?
A sarcomatous element is usually present
A human papillomavirus infection preceded development of this mass
Metastases are unlikely to be present
@Tissues resembling those from an embryo can usually be seen
Such a mass is often seen to arise in the testis
#
36
A 40-year-old woman has had a feeling of abdominal discomfort for the past 8 months. On pelvic examination, there is
the right adnexal mass. Abdominal CT scan demonstrates a 7 cm cystic mass involving the right ovary with small areas of
calcification. The uterus is normal in size. The right fallopian tube and ovary have been removed surgically. Grossly, the
mass on sectioning is filled with abundant hair and sebum. Microscopically, the mass has glandular spaces lined by
columnar epithelium, squamous epithelium with hair follicles, cartilage, and dense connective tissue. What type of
tumour is it?
Melanoma
Squamous cell carcinoma of ovary
Metastase of cervical carcinoma
@Teratoma
Sarcoma of ovary
#
37
A 32-year-old woman has noted dull pelvic pain for the last two months. On physical examination there is a mass
palpated in the right lower quadrant. An abdominal ultrasound reveals an 8 cm mass involving the right ovary. The mass
is surgically excised. On gross inspection, the surface of the mass is smooth, is not adherent to surrounding pelvic
structure, and is cystic and filled with hair on sectioning. On microscopic examination there is squamous epithelium, tall
columnar glandular epithelium, cartilage, and fibrous connective tissue. Which of the following neoplasms is she most
likely to have?
@Teratoma
Choristoma
Hamartoma
Myxoma
Mesothelioma
#
38
An epidemiologic study is performed involving patients with long-standing Epstein-Barr virus infection. It is observed
that these patients have an increased risk for development of malignant neoplasms in adulthood. Which of the following
neoplasms is most likely to be found in these patients?
@Nasopharyngeal carcinoma
Kaposi sarcoma
Small cell anaplastic carcinoma of lung
Osteosarcoma
Endometrial carcinoma
#
39
A 34-year-old woman presented with a hoarseness of a voice. A laryngoscopy revealed a tumour of a larynx. The
neoplasm had a grey-white color and papillary surface. Microscopical investigation showed a new growth of a connecting
tissue, covered by a stratified epithelium with the marked keratinization. Cellular atypia was absent. Most likely the tumor
is:
@Papilloma
Fibroma
Polyp
Angioma
Angiofibroma
#
40
A man was performed a removal of a skin neoplasm, node with papillary surface that looks like cauliflower cabbage, of
thick consistency. Microscopically tumour consists of numerous papillae. Parenchyma is made of cover epithelia with
multiple layers. Polarity of cells, stratification, and basal membrane integrity are not changed. Stroma of the tumour is
situated in its centre. Name the diagnosis.
@Papilloma
Fibroma
Adenoma
Fibroadenoma
Cystadenoma
#
41
A 15-year-old boy is found to have a small, dense, nodule at his right forearm skin. Grossly it had a papillary surface,
which looked like a cauliflower. Microscopically, the tumor consists of many papillae. Parenchyma formed of
integumentary epithelium with an increased amount of layers. The polarity of epithelial cells, their stratification and
membrane wholeness are preserved. A connective tissue forms a stroma within a center of papillae. What is the most
likely diagnosis?
@Papilloma.
Fibroma.
Adenoma.
Fibroadenoma.
Cystadenoma.
#
42
A 50-year-old female is found to have a urinary bladder neoplasm. Biopsy investigation revealed that tumor consists of
thin, branching out papillae, covered with few layers of transitional cell epithelium. What is the most likely diagnose?
@Papilloma
Basal cell
Transitional cell carcinoma
Squamous cell carcinoma
Fibroadenoma
#
43
A new growth of the skin of solid consistency, that looks like a nodule with papillar surface and reminds cauliflower was
removed. Microscopically the tumour consists of many papillae. Parenchyma is formed of integumentary epithelium with
an increased amount of strata. The polarity of cells, stratification and the intact membrane are saved in the epithelium.
The tumour stroma is located in the center of papillae. What is the most possible diagnosis?
@Papilloma
Fibroma
Adenoma
Fibroadenoma
Cystadenoma
#
44
A previously healthy 66-year-old man has been feeling tired for 5 months. He goes to his physician, who performs a
complete physical examination, including stool guaiac, which is positive. A colonoscopy is performed, and a large, sessile
4.5 cm mass with surface ulceration is found in the cecum. A biopsy of this mass microscopically shows irregular glands
with crowded, tall columnar cells having marked nuclear hyperchromatism. Which of the following statements about his
condition is most accurate?
@His relatives have hereditary adenomatous polyposis
There is lack of apoptosis from a faulty Bcl-2 gene
A p53 mutation is present in the neoplastic cells
He has hereditary non-polyposis colon cancer
He has inherited a faulty Rb gene from one parent
#
45
A 23-year-old woman presents with a 0.4 cm nodule within the skin on the left side of her neck. The clinical removes the
lesion and sends it to the pathology lab, calling it a “sebaceous cyst”. Histologic sections reveal a cystic structure is the
dermis that is filled with keratin and lined by a stratified squamous epithelium, which has a granular cell layer. This cyst
is not ruptured, no adnexal structures are seen within the wall of the cyst, and no atypia is present. Which of the following
is the most likely diagnosis?
Acrochordon
Cystis hygroma
@Epithelial inclusion cyst
Intradermal nevus
Pilar cyst
#
46
A 38-year-old woman was diagnosed a tumour in the parotid gland. It consists of separate nodules that are separated by
strata of connective tissue. Pathohistologically: the cells are of epidermoid type and form solid structures. Mucous-
productive cells form tension bars that cover cavities with mucous. The third element of the tumour is small cells with
hyperchromic nucleus. What tumour is meant?
Papillary cystadenoma
Acinocellular tumour
@Mucoepidermoid tumour
Adenolymphoma
Adenocystic carcinoma
#
47
A healthy 22-year-old woman undergoes a routine physical examination. A discrete, firm, rubbery, movable mass is
found in the left breast. She has no axillary lymphadenopathy. The skin overlying the breast and the nipple appear normal.
Which of the following neoplasms is most likely to be present?
@Fibroadenoma
Leiomyoma
Malignant lymphoma
Intraductal carcinoma
Lipoma
#
48
A physical investigation of a 25-year-old-woman revealed in her breast a dense node 1,0 cm in diameter. A biopsy
research showed encapsulated growth of a connective tissue round a basal membrane of a mammary gland ducts.
Glandular elements had different diameter, did not form lobes. What is the most likely diagnosis?
@Fibroadenama
Fibroma
Metastasis of a cancer
Adenoma
Fibrous cancer
#
49
A 45-year-old woman presented to oncology with a breast tumor. A biopsy revealed a tissue atypia with stromal
predominance over neoplastic parenchyma. Breast ducts and ductules were variable, they had one or two layered
epithelium without atypical mitotic activity. The intralobular stroma was dence. What is the most likely diagnosis?
@Fibroadenoma
Papilloma
Non-invasive carcinoma
Invasive carcinoma
Mastitis
#
50
The intraoperational biopsy of mammal gland has revealed the signs of atypical tissue with disorder of parenchyma
stroma proportion with domination of the last, gland structures of the different size and shape, lined with single-layer
proliferative epithelium. What is the most appropriate diagnosis?
Mastitis
Papilloma
Noninfiltrative cancer
@Fibroadenoma
Infiltrative cancer
#
51
A histological investigation of the biopsy from cervix uteri of a 45-year-old woman revealed signs of cellular atypia with
intact basal membrane. What is the most likely diagnosis?
@Carcinoma in situ
Erosion
Adenocarcinoma
Papilloma
Endometriosis
#
52
A 57-year-old postmenopausal woman is found to have a chronic nonhealing lesion at her portio vaginalis uteri.A biopsy
was taken form this area. A histological examination of the biopsy material revealed a cellular atypism within epithelial
layer, but basic membrane was unchanged. What is the most likely diagnosis?
@Carcinoma in situ.
Erosion.
Adenocarcinoma.
Papilloma.
Endometriosis.
#
53
A 41-year-old woman has noted a foul-smelling vaginal discharge for 3 weeks. On physical examination there is an
exophytic 3 cm mass involving the ectocervix. Pap smear testing is performed; she has never had a previous Pap smear.
Cytologic changes are present on this Pap smear that are consistent with squamous cell carcinoma. She is found to have a
positive serologic test for syphilis. Her serum glucose is 157 mg/dL She has been a commercial sex worker in the past.
Which of the following is the most likely risk factor for her cervical carcinoma?
@Human papillomavirus infection
Diabetes mellitus, type II
Heavy cigarette smoking
Pelvic inflammatory disease
Previous cancer chemotherapy
#
54
At the histological examination of a bioptic material from a vaginalis part of the cervix of the uterus, that was taken from
a 47-year old woman, the attributes of the cell atypism, the basal membrane – without changes were detected. This part of
the uterus didn’t heal up for a long time. Make a diagnosis.
Erosion
@Carcinoma in situ
Adenocarcinoma
Papilloma
Endometriosis
#
55
A 62-year-old man has complained of pain on urination for the past week. He is afebrile. On cystoscopy, a slightly
erythematous 1 cm diameter area is seen on the bladder mucosa. This area is biopsied and on microscopic examination
shows cells with marked hyperchromatism and increased nuclear/cytoplasmic ratio involving the full thickness of the
epithelium. However, these changes are confined to the epithelium above the basement membrane. Which of the
following terms best describes these biopsy findings?
@Carcinoma in situ
Metaplasia
Minimal dysplasia
Microinvasion
Hyperplasia
#
56
A 67-year-old man present with a slowly growing lesion that involves the lower portion of his left lower eyelid. You
examine the lesion and find it to be a pearly papule with raised margins and a central ulcer (rodent ulcer). Which on the
following histologic features would most likely be seen when examining histologic section from this lesion?
Reactive epidermal cells surrounding a central superficial ulcer
@Infiltrating groups of basaloid cells with peritumoral clefting
Infiltrating groups of eosinophilic cells with keratin formation
Dermal aggregated of small cells histologically similar to oat cell carcinoma
An in situ lesion with full-thickness epidermal atypia
#
57
A 48-year-old man has presented to his physician a plaque-like formation on a neck. Histological investigation of a skin
biopsy revealed tumorous cells located as nests, having round and oval form with narrow rim of basophilic cytoplasm.
They reminded cells of a skin basal layer. Specify the tumor name.
@ Basal cell epithelioma
Epidermal cancer
Hidroadenoma
Trichoepithelioma
Syringadenoma
#
58
A 45 year old man consulted a doctor about a plaque-like formation on his neck. Histological examination of a skin
bioptate revealed clusters of round and oval tumour cells with a narrow border of basophilic cytoplasm resembling of
cells of basal epidermal layer. What tumour is it?
Epidermal cancer
Syringoadenoma
Trichoepithelioma
@Basal cell carcinoma
Hydroadenoma
#
59
The patient on a face skin had a tumorous formation plaque-like form with an ulcer. What is the most likely diagnosis?
@ Basal cell epithelioma
Carcinoid
Thecoma
Pinealoma
Thymoma
#
60
A 56-year-old woman feels a lump in her right breast. Her physician palpates an irregular 3-cm mass that is not movable
because it appears fixed to the overlying skin, which is retracted. A mastectomy is performed and the pathologist on
sectioning the breast finds a 3 x 3.5-cm ovoid mass that does not have discrete borders, but appears to infiltrate into the
surrounding fibrofatty breast stroma. The mass is firm, white, and has a fibrous consistency. Which of the following
features is demonstrated by the gross appearance of this mass:
Anaplasia
Aplasia
@Desmoplasia
Dysplasia
Metaplasia
#
61
Gradually, a patch with necrosis and ulcer in the middle has developed on the patient’s skin of the face. At the
pathohistological examination of a biopsy material an overgrowth of atypical epithelial cells with great amount of
pathological mitoses was detected. What is the most possible diagnosis?
@Skin carcinoma
Sarcoma
Papilloma
Trophic ulcer
Fibroma
#
62
A patient presented to her physician with gradually developed plaque on a skin of a cheek. The formation had necrosis
and an ulcer in the center. A histological investigation of a skin biopsy revealed a growth of atypical epithelial cells with a
lot of pathological mitoses. What is the most likely diagnosis?
@Carcinoma of skin
Sarcoma
Papilloma
Trophic ulcer
Fibroma
#
63
A 15-year-old boy has had lumps in the right neck for the past 5 months. On physical examination there is painless
lymphadenopathy in the right cervical region. One of the lymph nodes is biopsied and on microscopic examination shows
many lymphocytes that are large, with clumped chromatin and occasional mitoses. Define the changes of this population
of lymphocytes:?
@A neoplasm
Reactive hyperplasia
Lymphadenitis
Activating of immune response
Inflammation
#
64
A 29-year-old woman with a history of multiple sexual partners over the last 15 years has a routine physical examination
with no abnormal findings. On pelvic examination, the cervix shows no abnormalities, but a Pap smear is taken and
dysplastic cells are reported to be present. A cervical biopsy is performed and shows microscopic features of a minimal
dysplasia involving the cervical squamous epithelium. Which of the following processes could be the nextone:
Metaplasia
Benigne tumor
@Malignant tumor
Inflammation
Necrosis
#
65
A 62-year-old man has complained of pain on urination for the past week. He is afebrile. On cystoscopy, a slightly
erythematous 1 cm diameter area is seen on the bladder mucosa. This area is biopsied and on microscopic examination
shows cells with marked hyperchromatism and increased nuclear/cytoplasmic ratio involving the full thickness of the
epithelium. However, these changes are confined to the epithelium above the basement membrane. Which of the
following terms best describes these biopsy findings?
Metaplasia
Minimal dysplasia
Microinvasion
Hyperplasia
@Carcinoma in situ
#
66
A 17-year-old man presents with a lesion on his face that measures approximately 1.5 cm in its greatest dimension. He
has a history of numerous similar skin lesions that have occurred mainly in sun-exposed areas. The present lesion is
biopsied and reveals an invasive squamous cell carcinoma. This patient most probably has one type of a group of
inherited diseases associated with unstable DNA and increased incidence of carcinoma. Which of the following is the
most likely diagnosis?
@Xeroderma pigmentosa
Wiskott-Aldrich syndrome
Familial poliposis
Sturge-Weber syndrome
Multiple endocrine neoplasia type 1 (MEN1)
#
67
A 44-year-old woman who has had multiple sexual partners for the past 30 years has an abnormal Pap smear with
cytologic changes suggesting human papillomavirus infection. Without treatment, she is most likely to develop which of
the following lesions?
@Squamous cell carcinoma
Non-Hodgkin's lymphoma
Kaposi's sarcoma
Adenocarcinoma
Leiomyoma
#
68
Which of the following bronchogenic carcinomas is most frequently associated with production of parathormone-like
substances?
Acinar adenocarcinoma
Papillary adenocarcinoma
Bronchioloalveolar carcinoma
@Squamous cell carcinoma
Oat cell carcinoma
#
69
A 51-year-old man who is a long-term smoker and who has chewed tobacco for more than 20 year presents with a slowly
enlarging mass on the floor of his mouth. Physical examination finds the mass, which measure 1.3 cm in greatest
dimension, to be white in color and irregular in appearance. Biopsies from this lesion are most likely to reveal which one
of the following malignancies?
Acinic cell carcinoma
Adenocarcinoma
Basal cell carcinoma
Small-cell carcinoma
@Squamous cell carcinoma
#
70
A senior woman who had metrorrhagia in menopause was made a smear from the uterine cervix mucosa. There was
diagnosed the accretion of the atypical epithelium with the formation of cancer pearls. What is your diagnosis?
Mucous cancer
Adenocarcinoma
Squamous cell cancer without hornification
@Squamous cell cancer with hornification
Undifferentiate cancer
#
71
A 65-year-old woman presented to the hospital with the menopausal bleeding. At histological investigation revealed in
the curettage material from the mucosa of her cervix uteri revealed a new growth of atypical epithelium with formation
so-called «cancer pearls". What is the most likely diagnosis?
@ Squamous cell keratinous carcinoma
Adenocarcinoma
Squamous cell nonkeratinous carcinoma
Mucous cancer
Nondifferentiated carcinoma
#
72
Histological investigation of a bronchial biopsy revealed a tumour which is constructed from nests of stratified
epithelium's atypical cells with some characteristic "pearls". What is the most likely diagnosis?
@ Squamous cell keratinous carcinoma
Squamous cell nonkeratinous carcinoma
Solid carcinoma
Mucous carcinoma
Scirrhous carcinoma
#
73
A mucus membrane biopsy is taken from a bronchus of a 52-year-old patient. A histological investigation revealed cords
of atypical epithelial cells, which grew into the tissues underneath. In the cord's center a concentric pink color formations
were determined ("cancer pearls", "epithelial [epidermic] pearl, pearly body"). Name a kind of a tumor?
@Squamous cell keratinous carcinoma
Differentiated adenocarcinoma
Squamous cell nonkeratinous carcinoma
Melanoma
Transitional cell carcinoma
#
74
During a histological research of bronchi wall and adjacent lung areas there were found stains and lines of atypical
epithelia. Epithelia cells have moderate signs of atypism: nuclear polymorphism, nuclear hyperchromia, mitoses. In the
centre of these complexes there are concentric neoplasms of pink colour. Name the diagnosis.
@Flat-cell cancer with cornification
Flat-cell cancer without cornification
Adenocarcinoma
Fibrous cancer
Undifferentiated cancer
#
75
During a microscopical research of a bioptate of bronchi there was a tumour revealed. The tumour consists of nest-like
associations of atypical cells of multilayer epithelia, sometimes "pearls" can be found. Name the diagnosis.
@Flat-cell cancer with cornification
Flat-cell cancer without cornification
Solid cancer
Mucous cancer
Skirr
#
76
A 52-year-old female patient with a history of the chronic bronchitis and pneumosclerosis presented to the hospital for
biopsy diagnostics. A microscopical investigation of a left bronchial's mucous from the suspicious site revealed cellular
and tissue atypia. There were also found some structures in the form of cancer pearls". What is the most likely pathology
presented in that case?
@ Squamous cell keratinous carcinoma of bronchus
Chronic polypous bronchitis
Bronchiectasis
Sharp bronchitis
Squamous cell metaplasia of mucous of bronchus
#
77
A 48- year-old man with a history of a chronic bronchitis has died of a cachexy. A post-mortem revealed in a lumen of
the right bronchus an endophytic growth of a light- grey softish tissue. Microscopic investigation showed a development
of an atypical stratified epithelium with a presence of an « epithelial [epidermic] pearl, pearly body ». What is the most
likely diagnose.
@Squamous cell keratinous carcinoma
Squamous cell nonkeratinous carcinoma
Adenocarcinoma
Nondifferentiated carcinoma
Apudoma
#
78
A 60-year-old postmenopausal woman has been feel unhealthy and weak for about 3 month. A gross investigation of her
cervix uteri revealed a lesion and a biopsy from this area was obtained. Microscopically, a lesion composed of atypical
squamous cell, many of them displayed pathological mitoses. Keratin pearl formation was also observed histologically.
What is the most likely diagnose?
@Squamous cells carcinoma with keratinisation
Transitional cell carcinoma
Squamous cells without keratinisation
Adenocarcinoma
Anaplastic carcinoma
#
79
Histological investigation of a biopsy from a tumor of the right bronchus' mucous membrane revealed a cellular and tissue
atypia, appearance of structures in the form of' cancer pearls'. Define the pathological process.
@ Malignant tumor
Benign tumor
Hyperplasia
Metaplasia
Hypoplasia
#
80
A man has been performed a removal of lip tumour. The tumour is a node of soft consistency, invades adjacent tissues,
has a rough surface with few ulcers. Histologically the tumour is made of lines of atypical cells of multilayer epithelia,
that invade into adjacent tissue, destroying it, and make nest-like complexes. Name the type of tumour.
@Flat-cell cancer with cornification
Flat-cell cancer without cornification
Solid cancer
Carcinoma in situ
Adenocarcinoma
#
81
Microscopical examination of a surgical specimen (an ulcered part of a lip) revealed in the connective tissue of mucous
membrane near the borders and under the floor of the ulcer some epithelial complexes consisting of atypic multistratal
epithelium with accumulations of bright pink concentric formations. What pathology is it?
Transitional cell carcinoma
Papilloma
@Squamous cell keratinous carcinoma
Squamous cell nonkeratinous carcinoma
Basal cell carcinoma
#
82
A 70 year-old male is found to have a nodule at his right bronchus mucous membrane. A microscopic examination of the
biopsy material from the tumor revealed the cell and tissue atypism, keratin pearls formation. What is the most likely
pathologic process at biopsy material?
@ Malignant tumor.
Benign tumor.
Hyperplasia.
Metaplasia.
Hypoplasia.
#
83
A 64-year-old man presents with symptoms of anemia. On work-up, you discover that the patient has been losing blood
from the GI tract secondary to a tumor mass is his colon. The pathology report from a biopsy specimen indicates that this
mass is an invasive adenocarcinoma. Which of the following histologic appearances is most likely to be seen in a biopsy
specimen taken from this tumor mass?
A uniform proliferation of fibrous tissue
A disorganized mass of proliferating fibroblasts and blood vessels
A disorganized mass of cells forming keratin
A uniform proliferation of glandular structures
@A disorganized mass of cells forming glandular structures
#
84
A change in bowel habits prompts a 53-year-old woman to see her physician. On physical examination there are no
lesions noted on digital rectal examination, but her stool is positive for occult blood. A colonoscopy is performed and
reveals a 6 cm friable mass located in the cecum. A biopsy of this mass is performed and microscopic examination shows
a moderately differentiated adenocarcinoma. Which of the following tissues is most likely to be origin in this tumor?
Connective
Vessel
Muscule
@Epithelial
Nervous
#
85
A 66-year old male lost his appetite and has been loosing his weight for about 5 months. X-ray study revealed a stomach
neoplasm. Histological examination of a tumor's biopsy showed a great amount of signet-ring cells. Name the histological
variant of cancer.
@Adenocarcinoma.
Solid carcinoma.
Sarcoma.
Mucinous carcinoma.
Carcinoid.
#
86
A gastroscopic study of a 50-year-old patient revealed a crater-like lesion on small curvature in pre-pyloric zone of
stomach. From a regional site of formation a biopsy is taken, биопсия. A histological investigation showed a tumor with
glands-like structures of the various form and the sizes, growing into surrounding tissue. The cells had marked signs pf
atypia. Name a histological variant of the presented tumor.
@Adenocarcinoma
Squamous cell carcinoma
Scirrhous carcinoma of stomach
Mucous carcinoma of stomach
Solid carcinoma of stomach
#
87
On the microscopy of the infiltrate from the colon a tumor consisting of prismatic epithelium was discovered. The tissue
formed atypical glandular structures of various forms and seize. The cells are polymorphic, nuclei are hyperchromic, and
pathological mitoses take place. Glandules’ basal membrane is destroyed. Some glands are covered by pseudostratified
epithelium; the distance is not changed. Your diagnosis:
@Adenocarcinoma
Basal-celled carcinoma
Solid carcinoma
Mucous carcinoma
Undifferentiated carcinoma
88
A 35 year-old female with a family history of colon cancer inquires about screening. Colonoscopy revealed a tumor and a
tissue sample was taken for histology. Microscopic investigation has shown cells were arranged in glandular-like pattern.
They exhibit cell pleomorphism, atypia, invasive growth and pathologic mitosis's. What is the most likely diagnose?
@ Adenocarcinoma
Basal-cell carcinoma
Solid carcinoma
Mucous carcinoma
Undifferentiated carcinoma
#
89
A 45-year-old woman has noted a lump on her left shoulder that has enlarged over the past 4 months. On physical
examination there is an enlarged, non-tender supraclavicular lymph node. A biopsy of the node is done and on
microscopic examination there is a metastatic neoplasm. Which of the following is the most likely
@Adenocarcinoma of the colon
Cerebral glioma
Fibroadenoma of the breast
Liposarcoma of the retroperitoneum
Laryngeal papilloma
#
90
A histological investigation of a lung biopsy revealed atypical cells which form plural acinar structures and produce
mucus. What histological form of a cancer of lungs takes place at the patient?
@High differentiated adenocarcinoma
Low differentiated adenocarcinoma
Nondifferentiated carcinoma
Differentiated
Glandular squamous cell carcinoma
#
91
Histological investigation of a node in the removed mammary gland revealed complexes of atypical polymorphic
epithelial cells, which had various sizes and forms. There were clear spaces at the centers of complexes. The cells had
large nuclei, with presence of atypical mitoses. Diagnose the pathology.
@Adenocarcinoma
Squamous cell nonkeratinous carcinoma
Solid carcinoma
Fibroadenoma of breast
Nondifferentiated polymorphocellular carcinoma
#
92
Histological investigation of the removed breast node revealed different sizes and the form complexes of atypical
polymorphic epithelial cells among abundant stroma. The complexes had a clear space center. Cells were characterized by
large nuclei, the increased number of nucleoli, nucleoli organizers and presence of atypical mitoses. What is the most
likely diagnosis?
@Adenocarcinoma
Fibroadenoma of breast
Solid carcinoma
Squamous cell nonkeratinous carcinoma
Nondifferentiated polymorphic cells carcinoma
#
93
A microscopical investigation of a 50-year-old woman's endometrial curettage material with the clinical diagnosis «
ovarian - menstrual cycle's disorder » revealed growth of glandular structures. These glands consisted of polymorphic
cells with hyperchromic nuclei and mitoses figures. For what pathology the revealed histological changes are
characteristic?
@Adenocarcinoma of uterus
Placental polyps
Acute endometritis
Glandular hyperplasia of endometrium
Chorioepithelioma of uterus
#
94
A 48-year-old woman complains to her doctor of uterine bleeding. Endometrial biopsy investigation revealed a presence
of gland-forming cells with enlarged hyperchromic nuclei, cell atypia, and abnormal mitoses. These cells were also
determined at myometrium. What term most correctly identifies this pathological process?
@ Adenocarcinoma of uterus.
Adenomatous endometrial hyperplasia.
Acute endometritis.
Placental polyp.
Chorionepitelioma of uterus.
#
95
A 57-year-old patient gained periodical metrorrhagias. There was performed a diagnostical scrape of mucous membrane
of uterus. Microscopically: among blood elements there are glandular complexes of different shape and size. These
complexes consist of atypical cells with nuclear hyperchromia and numerous mitoses (including pathological ones).
Name the diagnosis.
@Adenocarcinoma
Fibromyoma of uterus
Chorionepithelioma
Hyperplasia of endometrium glands
Endometritis
#
96
A 57 year old patient has periodic uterine bleedings. Diagnostic endometrectomy was performed. Biopsy material
contains among the blood elements some glandular complexes of different sizes and forms that consist of atypic cells with
hyperchromic nuclei and multiple mitoses (including pathological ones). What is the most probable diagnosis?
Fibromyoma of uterus
@Adenocarcinoma
Chorioepithelioma
Glandular hyperplasia of endometrium
Endometritis
#
97
A 55-year-old woman had a relapse uterine bleeding. A diagnostic smearing from the uterine mucosa was made. There,
among blood elements, different form and size glandular elements were seen. They were formed of atypical cells with
hyperchromic nuclei and have many mitoses (including pathological). What process can we think of?
@Adenocarcinoma
Glandular hyperplasia of endometrium
Chorionepithelioma
Adenomatous polyp
Indications of breaked off pregnancy
#
98
A female patient in a climacteric period presented with relapsing uterine bleedings. The diagnostic curettage of a uterus is
executed. Microscopical investigation showed among blood glandular elements of different size and forms created by
atypical cells with hyperchromatic nuclei numerous mitoses (including pathological). What is the most likely pathology?
@Adenocarcinoma
Glandular hyperplasia of endometrium
Choriocarcinoma
Adenomatous polypus
Signs of the interrupted pregnancy
#
99
A 70-year-old healthy man is found to have a palpable firm nodule in the prostate gland on digital rectal examination. His
serum prostate specific antigen (PSA) is found to be 19 ng/mL. The microscopic appearance seen on biopsy of the
prostate will probably be most consistent with which of the following conditions:
Chronic prostatitis
@Adenoearcinoma
Infarction
Leiomyoma
Rhabdomyosarcoma
#
100
A 45-year-old woman has an enlarged, non-tender lymph node that on biopsy reveals a metastatic neoplasm. Which of the
following is the most likely primary neoplasm:
Cerebral glioma
@Adenocarcinoma of the colon
Fibroadenoma of the breast
Liposarcoma of the retroperitoneum
Laryngeal papilloma
#
101
A 59-year-old man went to his physician for a routine health maintenance examination. The only abnormal physical
examination finding is a positive stool guaiac test. Laboratory studies show a CBC with Hgb 10.0 g/dL, Hct 29.S&, MCV
73 fL, platelet count 300,000/microliter, and WBC count 8700/microliter. He is found on colonoscopy to have a 4 cm
mass lesion arising on the mucosal surface of the transverse colon. Which of the following statements is most appropriate
regarding these findings?
This is an uncommon site for a primary malignant neoplasm
@Microscopically, the mass is probably an adenocarcinoma
Persons who are smokers are at much greater risk to develop this lesion
Hypercalcemia is a frequent paraneoplastic complication
The predicted 5 year survival rate is less than 5%
#
102
A 60-year-old woman has a firm mass with irregular borders felt in her left breast on a routine physical examination. A
fine needle aspiration is performed and microscopic examination shows an infiltrating ductal carcinoma. A left
mastectomy with axillary lymph node dissection is performed. A tissue sample of this neoplasm is submitted for
microscopy. Which of the following does flow hystologicaly most likely provide?
Squamous cell carcinoma with keratinisation
Squamous cell carcinoma without keratinisation
Mucous carcinoma
@Adenocarcinoma
Fibrous cancer
#
103
A 64-year-old man has has noted a 10 kg weight loss along with increasing fatigue over the past year. He has experienced
dull abdominal pain for the past week. He has developed abdominal distention with lack of stools in the past two days. On
physical examination, bowel sounds are reduced. An abdominal CT scan reveals a mass involving the descending colon.
At laparotomy, a partial resection of the left colon is performed, with removal of an encircling mass in the descending
colon. Microscopically, the mass is found to be a high differentiated adenocarcinoma. Which of the following
morphological type is most likely to be present in this man?
@Tubular structures
Trabecular structures
Mucous
Cancer’s perles
Cellular atipism
#
104
Histologic section from a 3 cm mass found in the mandible of a 55-year-old woman reveal a tumor consisting of nests of
tumor cells that appear dark and crowded at the periphery of the nests and loose in the center (similar to the stellate
reticulum of a developing tooth). Grossly, the lesions consist of multiple cyst filled with a thick, “motor oil” – like fluid.
Which of the following is the most likely diagnosis?
Pleomorphic adenoma
Ameloblastoma
Mucoepidermoid carcinoma
@Adenoid cystic carcinoma
Acinic cell carcinoma
#
105
During the histological examination of the stomach tumour a great amount of signet-ring cells. Name the histological
variant of the cancer was diagnosed.
Sarcoma
Solid carcinoma
Adenocarcinoma
@Mucous carcinoma
Carcinoid
#
106
A 52-year-old woman feels a lump in her right breast and goes to her physician. On physical examination there is a 3 cm
right breast mass fixed to the chest. This mass is biopsied and on microscopic examination shows nests of cells with
marked hyperchromatism and pleomorphism. Which are surrounded by connective stroma tissue in a great a mount.
Name histological type of tumor:
Adenocarcinoma
@Skirrhous cancer
Trabecular cancer
Medullar cancer
Undifferentiated cancer
#
107
A 45-year-old woman has a small tumor of her left breast. Histological examination of a breast biopsy revealed poorly
differentiated atypical epithelial cells. They formed trabecules separated from one another by connective tissue. The cells
and stroma proportion was approximately 1:1 .What is the most likely histological variant of the cancer.
@ Solid carcinoma.
Adenocarcinoma.
Epidermoid carcinoma.
Sccirrhous fibrocarcinoma.
Small cell carcinoma.
#
108
A microscopical investigation of a breast tumour revealed that the neoplasm was constructed of undifferentiated atypical
epithelial cells. These cells formed trabecules, separated by layers of a connective tissue. A parity of cells and stroma was
approximately 1:1. Name a histological variant of a cancer.
@ Solid carcinoma
Adenocarcinoma
Squamous cell carcinoma
Fibrocarcinoma
Small cell carcinoma
#
109
At the histological examination of a mammary gland tumour was revealed. Thet it was built of low-differentiated atypical
cells of epithelial genesis. They formed trabeculae that were parted from one another by connective tissue. The correlation
between cells and stroma is approximately 1:1. Name the histological variant of the cancer.
Squamous cells
Adenocarcinoma
@Solid
Fibrous
Small-cell
#
110
A histological investigation of a breast tumor's biopsy revealed solid layers of fine epithelial cells with polymorphic
nuclei and a considerable quantity of pathological mitoses. In addition, the tumor had very little stroma and a lymphocytic
infiltration. Name the kind of presented tumor.
@Medullary carcinoma
Scirrhous carcinoma
Paget's disease
Adenoma
Adenofibroma
#
1101
"Fallout" from nuclear bomb testing and from nuclear power plant disasters may release radioactive isotopes into the
environment. Exposure to radiation via ingestion ofstrontium-90 that can contaminate milk products may result in an
increased risk for which of the following neoplasms:
Kaposi's sarcoma
@Small cell anaplastic carcinoma of lung
Osteosarcoma
Nasopharyngeal carcinoma
Endometrial carcinoma
#
112
A bronchoscopy of a 65-year-old patient revealed a polyp-like formation, 1,0 cm in diameter, in the proximal part of the
upper lobe bronchus of his right lung. A histological research revealed the tumour which consisted from fine lymphocyte-
like cells with hyperchromic nuclei. The cells grew like layers and cords. Specify, what of the listed below kinds of
tumors is the most authentic?
@Nondifferentiated small cell carcinoma
Nondifferentiated large cell carcinoma
Squamous cell carcinoma
Adenocarcinoma
Glandular squamous cell carcinoma
#
Mesenchymal tumours
#
1
There was determined a solid, moveable tumour, neatly isolated from surrounding tissues. It was of white color when
incised and was presented with fibrous tissue; microscopically there were seen chaotically set collagen fibres. There were
little of cells. Term the tumour.
Myoma
@Fibroma
Histiocytoma
Dermatofibroma
Desmoid
#
2
A 30-year-old man has a node on his left leg the skin. Physical investigation revealed dense, mobile tumor, circumscribed
by a connective tissue capsule. Grossly it cut section was presented with dense witish fibres. Microscopically the tumor
composed of mature fibroblasts and a collagenous stroma. What is the most likely diagnosis?
@Fibroma
Myoma
Histiocytoma
Dermatofibroma
Desmoid.
#
3
Examination of a patient revealed a dense, movable skin tumour that is standing out distinctly from the surrounding
tissues. Its section is found to be white and composed of fibrous tissu e. Microscopic examination revealed interlacing
collagen fibers and few cells. What tumour is it?
@Fibroma
Myoma
Histiocytoma
Dermatofibroma
Desmoid
#
4
There was a tumour of skin found. The tumour is thick, movable, separated from adjacent tissues. On incision the tumour
is white-coloured, and is made of reticular tissue. Microscopically: chaotically tangled colagen fibres, a few cells. Name
the tumour.
Dermatofibroma
Myoma
Histiocytoma
@Fibroma
Desmoid
#
5
In the skin macroscopically was detected a solid moveable tumour. Microscopically it is presented by chaotically set
fascicles of collagen fibres with a little amount of spindle cells. What tumour was removed?
Glomus-angioma
Leiomyoma
Melanoma
Lipoma
@Solid fibroma
#
6
At the young man in a skin depth the dense, mobile tumour, is defined. A microscopical research revealed chaotically
located fascicles of collagen fibers with a small amount of spindle-shaped cells. What tumour is removed?
@Dense fibroma.
Leiomyoma.
Melanoma.
Lipoma.
Glomus-angioma.
#
7
A 33-year-old woman presented with a tumor-like formation on a white line of her abdomen, which during pregnancy has
started to increase in sizes. A histological research revealed that a tumor is constructed of the differentiated connective
tissue, in which collagen fibers prevail of cells. What tumor presented in this case?
@ Desmoid
Dense fibroma
Fibrosarcoma
Soft fibroma
Dermatofibroma
#
8
Examination of the anterior abdominal wall of a pregnant woman revealed a tumour-like formation that arose on the spot
of a tumour that was removed two years ago. The neoplasm was well-defined, dense, 2х1 cm larg e. Histological
examination revealed that the tumour was composed of differentiated connective tissue with prevailing collagen fibres.
What tumour might be suspected?
Hibernoma
Lipoma
@Desmoid
Leiomyoma
Fibrosarcoma
#
9
A 23-year-old woman presents with a 0.4-cm firm brown lesion on her upper right thigh. Histologic sections from this
lesion reveal an irregular area in the upper dermis that is composed of a mixture of fibroblasts, hystiocytes, stromal cells,
and capillaries. The majority of cells of cells in this mixture are fibroblasts. The overlying epidermis reveals hyperplasia
of the basal layers. Which of the following is the most likely diagnosis?
@Dermatofibroma
Dermatofibrosarcoma protuberans
Fibroxanthoma
Pyogenic granuloma
Sclerosing hemangioma
#
10
A painless new growth without neat borders appeared in soft tissues of left thigh in a young man. At the biopsy material
the tissues of the formation look like fish meat, consist of embryonic fibroblast-like cells with many mitoses that grow
through the muscles. What is your diagnosis?
@Fibrosarcoma
Myosarcoma
Fibroma
Cancer
Myoma
#
11
The patient, a previously dealthy man of 25 years, presented with a painless neoplasm in a soft tissues of his left thigh.
Grossly, a tumor had uneven boundaries and on cut section it looked like a fish flesh. Microscopic investigation revealed
immature connective tissue cells with pleomorphism, numerous mitotic figures and lymphocyte infiltration at the edge of
the tumor. What is the most likely diagnosis?
@Fibrosarcoma
Myosarcoma
Fibroma
Carcinoma
Myoma
#
12
A young man incurs a stab wound to the chest. The wound is treated in the emergency room. Two months later there is a
firm, 3x2 cm nodular mass with intact overlying epithelium in the region of the wound. The scar is firm, but not tender,
with no erythema. The mass is excised and microscopically is found to he composed of fibroblasts with abundant
collagen. By which of the following mechanisms has this series of events most likely occurred:
Hypertrophic scar (keloid) formation
@Development of a fibrosarcoma
Poor wound healing from diabetes mellitus
Foreign body response from suturing
Staphylocccal wound infection
#
13
A 55-year-old woman has a moveable formation (1x0,7 cm) of a pasty consistence with neat borders and slow growth
under the skin of submandibular area. Histologically were detected the lipocytes, which formed lobules that differ in form
and size. Thin strata of connective tissue with vessels determined the lobules. Make a diagnosis.
Angioma
Fibroma
@Lipoma
Liposarcoma
Fibrosarcoma
#
14
A 48-year-old man presented to physician with mobile 1, 0 x 0, 7 cm formation under a skin of mandible. It had precise
borders, dough-like consistence and slow growth. A histologic research of formation revealed fat tissue cells (lipocytes),
which created lobules of different forms and the sizes, divided by thin layers of a connective tissue with vessels. What is
the most likely diagnosis?
@ Lipoma
Fibroma
Angioma
Liposarcoma
Fibrosarcoma
#
15
Examination of a 55 year old woman revealed under the skin of submandibular area a movable slowly growing pasty
formation with distinct borders 1,0x0,7 cm large. Histological examination revealed lipocytes that form segments of
diffrent forms and sizes separated from each other by thin layers of connective tissue with vessels. What is the most
probable diagnosis?
Angioma
Liposarcoma
Fibrosarcoma
@Lipoma
Fibroma
#
16
A 40-year-old woman presented with a very slowly enlarging subcutaneous mass at the right side of the chest wall.
Physical examination revealed a soft lobulated fluctuant swelling, not attached to the skin or underlying muscle.
Histologically a neoplasm was well-encapsulated and consisted of mature cells with clear cytoplasm that varied
considerably in size. What is the most likely diagnosis?
@ Lipoma.
Fibroma.
Hygroma.
Papilloma.
Hemangioma.
#
17
From the organism of the 65-year old woman tumor-like body 1,0x1,0x0,8 cm was removed from the underskin part of
the hip. Macroscopically: the body is represented by lipoid tissue inside capsule. Microscopically: different in seize lobuli
of lipoid tissue, separated by connective tissue. Name the body:
@Lipoma
Hybernoma
Liposarcoma
Fibroma
Desmoid
#
18
A a 27-year-old woman in excellent health has a routine health maintenance examination. A 2 cm firm, rounded mass is
palpable beneath the skin of the left forearm. She has no difficulty using the arm and there is no associated pain with the
mass, either in movement or on palpation. The overlying skin appears normal. The mass does not change in size over
the next year. Which of the following neoplasms is she most likely to have?
@Lipoma
Metastatic carcinoma
Melanoma
Rhabdomyosarcoma
Leiomyoma
#
19
A 50-year-old man has felt vague abdominal discomfort within past 4 months. Physical examination revealed no
lymphadenopathy, and no abdominal masses or organomegaly at palpation. Bowel sounds are heard. An abdominal CT
scan shows a 20 cm retroperitoneal soft tissue mass obscuring the left psoas muscle. A stool specimen tested for occult
blood is negative. Which of the following neoplasms is this man most likely to have?
Adenocarcinoma
Lymphoma
Melanoma
@Lipoma
Hamartoma
#
20
A 35-year-old man presents with the new onset of a “bulge” in his left inguinal area. After performing a physical
examination, you diagnose the bulge to be an inguinal hernia. You refer the patient to a surgeon, who repairs the hernia
and sends the resected hernia sac to the pathology laboratory along with some adipose tissue, which he calls a “lipoma of
the cord”. The pathology resident examines the tissue grossly and microscopically and decides that it is not a neoplastic
lipoma, but instead is nonneoplastic normal adipose tissue. Which one of the following features should be present to make
the diagnosis of lipoma rather than normal adipose tissue?
Anaplasia
Fibrous capsule
Numerous mitoses
@Prominent nucleoli
Uniform population of cells
#
21
A 75-year-old patient was removed a tumour (16 x 8 x 6cm) from extraabdominal cellular tissue. Microscopically were
detected the anaplastic fatty cells with the signs of cellular atypism and polymorphism. Giant deformed cells with the
drops of oil in the cytoplasm could be also seen. What is the most possible diagnosis?
Myosarcoma
Lipoma
Fibrosarcoma
@Liposarcoma
Mesotelioma
#
22
A 14x6x5 cm neoplasm excised from a retroperitoneum of a 66-year-old woman at surgery department. Microscopic
investigation revealed atypical anaplastic cells, which contained round cytoplasmic vacuoles of lipid that scallop the
nucleus. The majority of cells were pleomorphic, some of them were round with chromosomal abnormalities. What is the
most likely diagnosis?
@Liposarcoma
Lipoma
Myosarcoma
Fibrosarcoma
Mesothelioma
#
23
A 50-year-old man have felt vague abdominal discomfort for the past 4 months. A stool specimen tested for occult blood
is negative. He has no lymphadenopathy, and no abdominal masses or organomegaly can be palpated. Bowel sounds are
present. An abdominal CT scan shows a 20-cm retroperitoneal soft tissue mass obscuring the left psoas muscle. This
neoplasm is most likely to be a (n):
Melanoma
Hamartoma
Adenocarcinoma
Lymphoma
@Liposarcoma
#
24
The atypical rhabdomyoblasts illustrated in the photomicrograph below may be seen in all following lesions EXCEPT
Sarcoma botryoides
@Myositis ossificans
Mixed heterologous mьllerian tumor of the uterus
Adult pleomorphic rhabdomyosarcoma
Embryonal rhabdomyosarcoma
#
25
A 52-year-old man presents with symptoms of gastric pain after eating. During work-up a 3-cm mass is found in the wall
of the stomach. This mass is resected and histologic examination reveals a tumor composed of cells having elongated,
spindle-shaped nuclei. The tumor does not connect to the overlying epithelium and is found only in the wall of the
stomach. Which of the following is the cell of origin of this tumor?
Adipocyte
Endothelial cell
Glandular epithelial cell
@Smooth-muscle cell
Squamous epithelial cell
#
26
During a histological research of an uterus: under mucous membrane there are numerous round nodes, separated from
adjacent tissues. Microscopically the tumour is made of bunches of non-stratified muscles with tissue atypism. What is
the diagnosis?
Chorionepithelioma
Cancer of uterus
Fibromyoma
@Leiomyoma
Leiomyosarcoma
#
27
A physical examination of 47-year-old woman of gynecology department revealed that her uterus contained dicrete, firm,
white nodules. Histological examination excised lesion demonstrated a tissue atypia of a sample. It presented well-
defferentiated mature cells of smooth muscules. What is the most likely diagnosis?
@Leiomyoma.
Carcinoma of the uterus.
Fibromyoma.
Chorionepithelioma.
Leiomyosarcoma
#
28
A 35-year-old woman had a firm nodule palpable on the dome of the uterus six years ago recorded on routine physical
examination. The nodule has slowly increased in size and is now appears to be about twice the size it was when first
discovered. She remains asymptomatic. Which of the following neoplasms is she most likely to
@Leiomyoma
Adenocarcinoma
Leiomyosarcoma
Hematoma
Metastasis
#
29
A removed at the operation uterus was delivered for the histological examination. Multiple nodules of a round form,
neatly isolated from surrounding tissue, were revealed under the mucous membrane. Microscopically the tumour consists
of fascicles of nonstriated muscles with tissue atypism. What is your diagnosis?
@Leiomyoma
Uterine cancer
Fibromyoma
Chorionepithelioma
Leiomyosarcoma
#
30
An epidemiologic study is performed to determine risk factors for development of malignant neoplasms. A statistical
analysis of pre-existing medical conditions is done. Some pre-existing conditions are observed to precede development of
malignant neoplasms, while others do not. Which of the following conditions is most likely to be statistically unrelated to
subsequent malignancy?
@Uterine leiomyomas
Endometrial atypical hyperplasia
Chronic alcoholism with hepatic cirrhosis
Cervical squamous dysplasia
Chronic ulcerative colitis
#
31
A macroscopical investigation of operatively removed uterus revealed a tumour with a soft consistence, hemorrhages and
necroses. The tumor cut surface reminds ' the fish meat'. Histological research has found an expressed cellular and tissue
atypia; there were cells with pathological mitoses figures. What is the most likely diagnosis?
@Sarcoma.
Adenocarcinoma.
Angioma.
Fibroma.
Lipoma.
#
32
Macroscopically the uterine cancer (after operation material) is of soft consistence, with hemorrhages and areas of focal
necrosis. It looks like fish meat when incised. At the histological examination the signs of cell and tissue atypism are
traced. The cells with pathological mitoses can be also seen. What is your diagnosis?
Adenocarcinoma
@Sarcoma
Angioma
Fibroma
Lipoma
#
33
A 15-year-old young man presented with a tumorous formation in the central site of his wrist bone. The node grew slowly
within last 3 years. A histological research of a removed neoplasm revealed mature chondrocytes without mitoses, which
randomly located in chondral lacunas. Cartilages of a capsule had different form and the sizes due to variable quantity of
chondral cells, between which there were basic substance with liquid layers of a connective tissue. What is the most likely
diagnosis?
@Chondroma
Chondroblastoma
Chondrosarcoma
Teratoma
-
#
34
All the following statements about chondrosarcoma are true EXCEPT that
It is most frequent in middle age or later
The peripheral type can arise from enchondroma
It is common in the pelvic bones
Histologic analysis is of prognostic significance
@It is the second most common malignant bone tumor
#
35
The most common tumor that involves bone is
@A metastatic tumor from an extraosseous site
Osteogenic carcinoma
Multiple myeloma
Chondrosarcoma
A giant cell tumor
#
36
A 50-year-old patient presented to the doctor with a ball-shaped, dense, motionless neoplasm, 2 cm in diameter, under a
skin in the right parietal site of his head. A histological research of a removed neoplasm revealed a chaotic osteal beams
pattern with a connective tissue between. What is the most likely diagnosis?
@Cancellous osteoma (Osteoma spongiosum)
Compact osteoma (Osteoma durum)
Osteoporosis
Osteomalacia
Osteosarcoma
#
37
Osteosarcoma (osteogenic sarcoma) has an increased incidence in all the following conditions EXCEPT
Paget’s disease
Retinoblastoma
Fibrous dysplasia
@Plasmocytoma
Osteochondromatosis
#
38
At the young woman in'the area of her distal extremity of a femur the tumor, which quickly grew, is removed. Grossly, it
had a motley pattern - from white-sulphur to brown-red color and a quaggy consistence. Microscopical investigation
revealed the basic tissue component of a tumor presented with osteal and the ossiform structures covered by atypical
osteoblasts, with numerous thin-walled vessels and atypical mitoses figures. Make a diagnosis.
@Osteocarcoma
Chondroma
Osteoma
Ewing’s sarcoma
Angiosarcoma
#
39
A 28-year-old man with a history of an elbow bruise 3 years ago presented with a tumorous growth in the area of an
epiphysis of a humeral bone. The formation did not have accurate borders. A histological investigation of biopsy material
revealed a considerable quantity of polymorphic cells of osteoblastic type with numerous pathological mitoses. Make the
presumable diagnosis.
@ Osteosarcoma
Chondrosarcoma
Osteoid-osteoma
Fibrosarcoma
Sinovial sarcoma
#
40
Characteristics of Ewing’s sarcoma include all the following EXCEPT
Primary occurrence prior to age 20
@Primary occurrence in the epiphysis of long bones
Onion-skin pattern on x-ray
Basophilic cells resembling lymphocytes
PAS-positive intracytoplasmic glycogen
#
41
An 11-year-old boy presents with an enlarging, painful lesion that involvers medullary cavity of his left femur. X-rays
reveals an irregular, destructive lesion that produced an “onion-skin” periosteal reaction. The lesion is resected surgically,
and Histologic sections reveal sheets of uniform small, round, “blue” cells. Which of the following is the most likely
diagnosis?
Chondroblastoma
@Ewing’s sarcoma
Fibrosarcoma
Osteoblatoma
Osteogenic sarcoma
#
42
A 16-year-old child presented with a painful softish node in his femur diaphysis. The formation grows quickly with
destruction of a spongy layer of a bone. Microscopical investigation revealed monomorphic round cells little bit bigger
than mature lymphocytes, with jejunely light cytoplasm which contains glycogen. In some zones these cells form pseudo-
rosettes with few mitoses. Between cells there are fibrinous membranes. What is the most likely diagnosis?
@Ewing's sarcoma
Limphoma
Rhabdomyosarcoma
Neuroblastoma
Hemangioma
#
43
A 49-year-old man complains of pain in his left thigh for 3 months. On physical examination his thigh is increased in
size, compared to the right. A plain film radiograph reveals the presence of a 15 cm solid mass that does not appear to
arise from bone, but it does have infiltrative margins. A biopsy of this mass is taken, and on microscopic examination the
mass is composed of highly pleomorphic spindle cells. Name tumor?
Fibroma
Chondroma
Osteoma
@Osteosarcoma
Carcinoma
#
44
In an epidemiologic study, "fallout" from nuclear bomb testing and from nuclear power plant disasters is observed to have
released radioactive isotopes into the environment. One of these isotopes is found to be strontium-90. It is observed that
the strontium-90 in soils goes into the food chain and eventually contaminates milk products. This contamination is most
likely to increase the risk for which of the following neoplasms?
Kaposi's sarcoma of skin
Small cell anaplastic carcinoma of lung
@Osteosarcoma
Nasopharyngeal carcinoma
Endometrial carcinoma
#
45
During an examination of a 25-year-old patient: right half of mandible is spindle-shaped and enlarged, thickened, with a
smooth surface. A puncture revealed a liquid of brown colour. Histologically: mononuclear cells, presumably osteoblasts,
and giant multinuclear cells with nidi of lacunar resorbtion of bone girders. There are small haemorrhages and grains of
haemosiderin. What is the diagnosis?
Giant-cell epulis
Amyloblastoma
Fibrous osteodysplasia
Osteosarcoma
@Osteoblastoclastoma
#
46
4-year-old child presented with a flat red color knot on his neck skin, which turns pale at pressing by glass the knot. What
is the most probable diagnosis?
@Hemangioma
Pigmented nevus
Melanoma
Leiomyoma
Lymphangioma
#
47
A 14-year-old healthy girl has a 0.3-cm reddish, slightly raised nodule on the skin of the upper part of her chest found on
a routine physical examination. She states that this lesion has been present for years and has not appreciably changed in
size or color. Which of the following neoplasms is this nodule most likely to be:
@Hemangioma
Melanoma
Wilms tumor
Lymphoma
Glioma
#
48
A very painful lesion found under the nail of the second finger of the hand is most likely which one of the following
lesion of blood vessels or lymphatics?
Kaposi’s sarcoma
@Capillary hemangioma
Glomus tumor angiosarcoma
Cystic hydroma
#
49
Microscopically the tumour of the upper lip is built of multiple small chink-like cavities, the walls of which are covered
with endothelium and filled with liquid part of blood and its clots. Make a diagnosis.
Glomus-angioma
Venous hemangioma
Cavernous hemangioma
Haemangiopericytoma
@Capillary hemangioma
#
50
Microscopically: tumour of upper lip, made of numerous slot-like cavities, walls of which is made of flatted endothelia.
These cavities are filled with thin blood and clots. Name the diagnosis.
@Capillary haemangioma
Venous haemangioma
Cavernous hemangioma
Hemangiopericytoma
Glomus-angioma
#
51
A 20-year-old man has had a slowly growing reddish nodule on his upper lip. He finally decides to have a surgeon
remove it. Microscopically the nodule is composed of benign varying sized tiny blood vessels. What is the most likely
diagnosis?
@Capillary hemangioma.
Venous hemangioma.
Cavernous hemangioma.
Hemangiopericytoma.
Glomus-angioma.
#
52
A 17-year-old patient was diagnosed (at the operation) a tumour (4,5 x 5,0 x 3,5cm) on the lower surface of the liver. It
has sub serous localization and murrey color. On the incision it is presented by cavities with volumes of blood and thick
walls of connective tissue that are covered with one stratum of endothelium. Make a previous diagnosis.
@Cavernous haematoma
Capillary hemangioma
Haemangiopericytoma
Haemangioendothelioma
Lymphangioma
#
53
During surgery in a 17-year-old patient it was revealed the tumour of 4,5х5,0х3,5 sm in size on the lower surface of the
liver with subserose localization, of dark-red color. On the section tumour has cavities with marked amount of blood.
What is preliminary diagnosis?
Capillar hemangioma
Hemangiopericytoma
Lymphangioma
Hemangioendothelioma
@Cavernous hemangioma
#
54
A 2-year-old girl is being evaluated for progressive swelling of her neck. Physical examination finds a nontender, ill-
defined, loculated mass in the left side of her neck. During the workup of this patient, a karyotype reveals that she is
monosomic for the X chromosome. Which of the following is the cause of the swelling of this patient’s neck?
Bacillary angiomatosis
@Cystic hygroma
Glomus tumor
Nevus flammeus
Spider angioma
#
55
All the following are the result of melanocytic hyperplasia EXCEPT
@Freckle
Lentigo
Junctionals nevus
Blue nevus
Spitz tumor
#
56
A 35-year-old man present with 0.3-cm flat light brown lesion on his left forearm. The lesion is excised, and microscopy
reveal nests of round nevus cells within the lower epidermis is the dermal-epidermal junction. There is no “fusion”
present of adjacent nests of nervus cell. Cytologic atypia is not present, nor are nevus cell seen in the superficial or deep
dermis. Which of the following is the most likely diagnosis?
Compound nevus
Dysplastic nevus
Halo nevus
@Junctional nevus
Spitz nevus
#
57
The woman on her face skin had a pigmental formation in the form of a nodule which quickly grew. The biopsy is made.
Microscopical investigation of biopsy sample revealed fields of spindle- shaped and polymorphic cells which contain a
brown pigment. In addition, there were diagnosed numerous mitoses. What is the most likely diagnosis?
@ Melanoma
Nevus
Cancer
Papilloma
Dermatofibroma
#
58
A 46-year-old man gained a bulging black stain on his skin. The stain didn't heal for a long time. Afterwards, the stain
became a node, bigger and painful, changed its colour to dark-brown. Histologically: spindle-shaped and polymorphous
cells containing brown pigment. What kind of tumour is this?
Basalioma
@Melanoma
Hemangioma
Hematoma
Carcinoid
#
59
An eye of 53-year-old patient, excised at surgery due to neoplasm, presented in pathology department. Gross
investigation reveled a 1 x0,4 cm black lesion in the retina. Microscopic appearance of a tumor was characterized by nests
of immature cell with eccentric nuclei, prominent macronucleoli and cytoplasm brown pigment. What is the most likely
diagnosis?
@Melanoma
Neurinoma
Angiosarcoma
Neuroblastoma
Ganglioneuroblastoma
#
60
A physical investigation of a 60-year-old woman with one year history of a formation on a face revealed a brown skin
plaque with irregular form and black impregnations. A histological research of a skin biopsy showed in epidermis and
through all derma polymorphic big cells with pathological mitoses, large nucleoli and yellowy-brown pigment in a
cytoplasm of many cells. The specified cells grow in a kind of fine groups and also alone. Make a diagnosis.
@Melanoma
Nevus
Papilloma
Xeroderma
Melanosis
#
61
A 75 year old male patient consulted a surgeon about a brown nonhealing ulcer of shin. Examination of biopsy material
revealed diffuse growth of polymorphic atypic cells with brown pigment in their cytoplasm. Pearls reaction was negative.
There were also a lot of pathological mitoses and foci of tissue necrosis. What is the most probable diagnosis?
Skin cancer
Local hemosiderosis
Trophic ulcer
Intradermal nevus
@Melanoma
#
62
A patient complains on a brown ulcer of leg, that doesn't heal for a long time. During a biopsy: diffuse growth of
polymorphous atypic cells. In cytoplasm of those there is a brown pigment. Pearls' reaction is negative. There are a lot of
pathological mitoses and nidi of necrosis of the tissue. Name the diagnosis.
Skin cancer
Local haemosiderosis
Intradermal nevus
Trophic ulcer
@Melanoma
#
63
A dark convex macula appeared on the skin of a 46-year-old patient but didn’t disturb him. In course of time it began to
enlarge, the pain appeared, the color changed into light brown and a nodule could be palpated. At the histological
examination of the removed tissue, spindle and polymorphous cells were revealed, cytoplasm of which had a fulvous
pigment. What tumour is meant?
Haematoma
Basaloma
Haemangioma
@Melanoma
Carcinoid
#
64
A 50-year-old woman presented with a pigmented painful skin lesion above the ankle. It had been present for many years
but in recent months it had enlarged quite rapidly, its outline got irregularity. Microscopic investigation of a lesion biopsy
revealed nests of atypical cells and single cells with eccentric nuclei, prominent macronucleoli and cytoplasmic brown
pigment. What is the most likely diagnosis?
@Melanoma
Basal cell carcinoma
Hemangioma.
Haematoma.
Carcinoid.
#
65
A previously healthy 42-year-old man has a skin nodule on his right hand that has become larger and darker with more
irregular outlines over the past three months. On physical examination there is a 1.2 cm diameter darkly pigmented and
slightly raised nodule on the dorsum of his right hand. No other skin lesions are noted. Several nontender enlarged lymph
nodes are palpable in the right axilla. The lesion is excised and microscopic examination shows a neoplasm composed of
darkly pigmented polygonal and spindle cells. Which of the following neoplasm’s kind is this one?
Leiomyoma
Rabdomyoma
@Melanoma
Metastatic carcinoma
Basal-cell carcinoma
#
66
The incidence of malignant melanoma of the skin appears to be increasing in the United States. Which of the following is
most significant in predicting the clinical behavior following diagnosis?
The degree of pigmentation
@The level and depth
The amount of inflammation
The degree of pleomorphism
The state of nutrition
#
67
An eyeball with tumour like formation was delivered for the histological examination. The formation is located in the
vascular tract of the eye with the size 1 x 0,4cm and of black color. Multiple pathological mitoses can be seen in cells. A
fulvous pigment is also revealed in the cytoplasm of many cells. What is your diagnosis?
@Melanoma
Neurinoma
Angiosarcoma
Neuroblastoma
Ganglioneuroblastoma
#
68
A 66-year-old woman present with right eye bad vision, ophtalmoscopy revealed a neoplasm of retina which was soon
excised, at surgery together with an eye ball. Grossly, a neoplasm was soft, irregular in contour lxl cm in size and had a
brown coloring. Under microscope a lesion demonstrated nodular aggregates of infiltrating cells. There cells contained
large brown pigment nuclei with chromatin clumped at the periphery of nuclear membrane and prominent nucleoli.
Atypical mitoses figures were also revealed. What is most likely diagnosis?
@Melanoma
Schwannoma
Glomus tumor
Paraganglioma
Angiosarcoma
#
60
The most common primary tumor of the heart in adult is the
Rhabdomyoma
Rhabdomyosarcoma
Papillary fibroelastoma
Lipoma
@Myxoma
#
70
A 35-year-old man presents with weight loss, fever, and fatigue. Physical examination finds signs and symptoms of mitral
valve disease. Further work-up finds a pedunculated mass in the left atrium. The tumor is resected and histologic sections
reveal stellate cells in a loose myxoid background. Which of the following is the most likely diagnosis?
Chordoma
Fibroelastoma
Leiomyoma
@Myxoma
Rhabdomioma
#
Organ-specific tumor
#
1
The best example of viral oncogenesis in humans is seen with which of the following neoplasms:
Retinoblastoma
Small cell anaplastic carcinoma of lung
T-cell leukemia
Prostatic adenocarcinoma
@Hepatic carcinoma
#
2
A 55-year-old man presented to a hospital with the atelectasis of a right lung's middle lobe, resulted from the obturation
of a midlobar bronchus by a node of soft tissues. A bronchoscopy revealed a new growth within an obturation zone. A
microscopical investigation of a biopsy sample showed the growths of an atypical glandular epithelium with pathological
mitoses, which spread in tissues underneath and a cartilage. What is the most likely disease?
@Bronchogenic carcinoma of lungs
Dysplasia of epithelium of bronchus
Inflammatory polyps
Deforming bronchitis
Sarcoma of bronchus
#
3
All the following tumors may be found arising within the pineal gland EXCEPT
Teratoma
Pineoblastoma
Embryonal carcinoma
Choriocarcinoma
@Craniopharyngioma
#
4
A physical examination of a 22-year-old woman, with few years' history of myasthenia, revealed a big tumor at anterior
mediastinum. Histologically a tumor consisted of the oblong (spindle-shaped) cells with oval dark nuclei, which form
bands and nests and has little Hassall's bodies. After a surgical removal of a tumor symptoms of a myasthenia have started
to disappear progressively. Diagnose a tumor which has etiological and pathogenic connection with a myasthenia.
@ Thymoma
Adenoma of thyroid gland
Adenoma of parathyroid gland
Paraganglioma
Medulloblastoma
#
5
A 60-year-old man with a 90 pack year history of smoking experiences an episode of hemoptysis. A chest radiograph
reveals a 5-cm right upper lobe lung mass. A fine needle aspirate of this mass yields cells consistent with small cell
anaplastic ("oat cell") carcinoma. On physical examination he has puffiness of the face, some pedal edema, bruises of the
skin, and a blood pressure of 165/100-mm Hg. A bone scan shows no metastases. Immunohistochemical staining of the
tumor cells is likely to be positive for:
Parathormone related peptide
Erythropoietin
@ACTH
Insulin
Gastrin
#
6
A 32-year-old woman with a history of abortion 8 months earlier presented to the emergency department complaining of
uterine bleeding. Examination done by gynecologist revealed a neoplasm of the uterus. Grossly, tumor had a spongy
structure with multiple hemorrhages. Microscopically, atypical clear epithelial Langhan's cells and abnormal
syncytiotrophoblast cells arranged around maternal blood spaces were detected. What is the most likely diagnosis?
@ Choriocarcinoma
Squamous cell carcinoma without keratinisation
Adenocarcinoma
Fibromyoma
Hydatidiform mole
#
7
A 45-yer-old female examined by her gynecologist due to uterine bleeding. A neoplasm was diagnosed and then removed
at surgery department. The uterine lesion presented with huge necrotic and hemorrhagic spongy masses. Microscopic
investigation revealed large clear atypical epithelial cells and variety of abnormal dark cells, arranged around material
blood spaces. Stroma was not defined. Atypical cells resembled cytotrophoblast and syncytiotrophoblast cells. What is the
most likely diagnose?
@Choriocarcinoma
Invasive hydatidiform mole
Adenocarcinoma
Cavernous haemangioma.
Medullary carcinoma
#
8
6 months after delivery a woman had uterine bleeding. Gynecological examination revealed in the uterine cavity a dark-
red tissue with multiple cavities that resembled of "sponge". Microscopic examination of the tumour revealed some atypic
light epithelial Langhans cells and giant cells of cyncytiotrophoblast in blood lacunas. What tumour is it?
@Chorioepithelioma
Vesicular mole
Squamous cell nonkeratinous carcinoma
Fibromyoma
Adenocarcinoma
#
9
A 39-year-old woman has madescence in the region of mammilla, a small ulcer with inflammatory hyperemia and
cutaneous edema. Histologic examination of tissue sampling from this area revealed in the malpighian layer of thickened
epidermis atypical cells with light and optically empty cytoplasm, with no intracellular bridges. Such cells were also
found in the orifice of big mammal gland ducts. What is the most probable diagnosis?
@ Paget's disease
Intraductal cancer
Epidermoid cancer
Melanocarcinoma
Basal cell carcinoma
#
10
A physical examination of a 39-year-old woman revealed a soaking area by her breast nipple, a superficial ulcer with
inflammatory hyperemia and skin edema. A histological research of a biopsy from this area revealed in basal layer of
thickened epidermis atypical big cells with light and optically empty cytoplasm, with absence of intercellular bridges.
Such cells are found and in the ostium of the big ducts of a gland. What is the most likely diagnosis?
@Paget's desease
Intraductal cancer
Basal cell cancer
Epidermoid carcinoma
Melanoma
#
11
During a routine physical examination, a 49-year-old man is found to have a 2.5-cm “coin lesion” in the upper lobe of his
left lung. The lesion is removed surgically, and histologic sections reveal sheets of malignant cells with clear cytoplasm
(clear cell carcinoma). Which of the following is the most likely site of origin for this metastatic lung tumor?
Appendix
Breast
@Kidney
Pancreas
Stomach
#
12
A 60-year-old man noticed worsening urinary tract symptoms and thus inquires about screening tests for kidney
pathology. Investigation revealed a node 8 cm in diameter at right kidney's apex. After surgery, grossly tumor presented a
patchy pattern with hemorrhages and zones of necrosis. Histological study showed large anaplastic tumor cells with
abundant foamy cytoplasm and with small central densely staining nucleus. Abnormal mitoses were also found. What is
the most likely diagnose?
@ Clear -cell carcinoma
Clear -cell adenoma
Adenocarcinoma
Nephroblastoma
Acidophilic adenoma with malignization
#
13
A 62-year-old man was removed a kidney. At the microscopical examination there was revealed a tumour that looked like
a nodule with an approximate diameter 8cm. The tumour tissue is many-coloued with multiple haemorrhage bleedings
and necroses when incised. Histologically the tumour consists of light cells that form alveolar and papillar structures. The
invasive growth of the tumour is moderate. Many cells have pathological mitoses and hyperchromic nuclei. Diagnose the
tumour of the kidney.
@Clear cell cancer
Clear cell adenoma
Adenocarcinoma
Nephroblastoma
Acidophile adenoma with malignancy
#
14
Medical evaluation of a 55-year-old man finds the following laboratory data: increased hematocrit, increased RBC count,
increased total red cell mass, normal plasma volume, normal oxygen saturation of hemoglobin, and increased serum
erythropoietin. These findings are most characteristic of secondary polycythemia. Which of the following abnormalities is
most likely to be present in this individual?
Acute gastroenteritis
Chronic ulcerative colitis
Cyanotic heart disease
Polycythemia rubra vera
@ Renal cell carcinoma
#
15
A 5 5-year-old female is found to have asymptomatic microscopic hematuria. A plain radiograph of the abdomen revealed
a neoplasm at her right kidney. A gross investigation of removed kidney revealed a node 8cm in diameter at its upper
pole. The cut surface of a tumor presented with multiple hemorrhages and necroses. Histologically, it consists of light
cells that form alveolar and papillary structures. The invasive growth of the tumor is moderate. Many cells have
hyperchromic nuclei and atypical mitoses figues. What is the most likely diagnosis?
@Renal cell carcinoma.
Clear cell adenoma.
Adenocarcinoma.
Nephroblastoma.
Acidophilic adenoma with malignancy.
#
16
True statements about classic neurofibromatosis (von Recklinghausen’s disease) include all the following EXCEPT
Malignant degeneration may occur
Hamartomas of the iris are very common
@Hemangioblastomas of the brain are associated
Acoustic neuroma is associated
Pheochromacytomas and meningioma are associated
#
17
Examination of a young woman revealed a node-like, soft and elastic homogenous tumour of pinkish-white colour along
the acoustic nerve. The tumour contains cell bundles with oval nuclei. Cellular fibrous bundles form rhythmic structures
made up by parallel rows of regularly oriented cells arranged in form of a palisade with cell-free homogenous zone
(Verocay bodies) between them. What tumour is it?
@Neurinoma
Neuroblastoma
Ganglioneurinoma
Ganglioneuroblastoma
Malignant neurinoma
#
18
Examination of a young woman revealed a tumour up to 3 cm in diameter in form of a knot localized along the acoustic
nerve. The tumour is homogenous, soft and elastic, of pink-and-white colour. Microscopically the tumour contains
clusters of cells with oval nuclei. Fibrous cell clusters form regular structures made up by parallel rows of regularly
oriented cells arranged in form of a palisad e. Zones between the rows of cells are acellular and homogenous (Verocai
bodies). What tumour is it?
Ganglioneuroma
Neuroblastoma
Malignant neurinoma
Ganglioneuroblastoma
@Neurinoma
#
19
An elderly man with a 2-year history of right leg amputation presents with 2 cm encapsulated tumor at the area of a past
surgical trauma of soft tissue. Histology investigation revealed disordered orientation of mature nerve fiber bundles inter
mixed with connective tissue. Higher magnification showed bundles of axons, Schwann cells, fibroblasts and perineurial
cells within tumor mass. What is most likely diagnosis?
@Neurinoma
Neurofibroma
Malignant neurinoma
Soft fibroma
Fibrosarcoma
#
20
The man of 45 years has completely lost hearing on the right ear. Physical investigation has not revealed any pathological
changes in system of the right acoustic analyzer. The tomography of a brain has revealed a neoplasm, 7 cm in diameter,
homogeneous, without precise contours in a site of cerebellopontine angle. Name a neoplasm.
@Neurilemmoma
Gangliocytoma
Astrocytoma
Chorioidal papilloma
Neuroblastoma
#
21
A tumour in a capsule (diameter 2cm) was operative removed from the stump of the lower extremity. Microscopically it
consists of spindle monomorphic cells with stick-like nuclei that form palisade structures together with the fibres. What is
the most possible tumour?
Neurofibroma
@Benign neurilemmoma
Malignant neurilemmoma
Soft fibroma
Fibrosarcoma
#
22
All the following statements apply to ependymomas EXCEPT that
They are the most common type of intraspinal glioma
@They are most commonly located in the lateral ventricles
Patient may present with headache and papilledema
They may require differentiation from choroid plexus papilloma
Histologic section display rosettes
#
23
True statements about meningiomas include all the following EXCEPT
They usually presents clinically with headaches or seizures
They constitute about 20 percent of primary brain tumors
@They usually display rapid growth
They arise from arachnoid cap cells
They may be multiple in neurofibromatosis type 2
#
24
The tumor of a brain is diagnosed for the patient with neurologic disturbances. During operation the tumor, which looks
like the dense node bound to a firm cerebral membrane, is removed. Histological investigation revealed the neoplasm
constructed from endothelium-like cells closely adjoining to each other. What is the most likely diagnosis?
@ Meningioma
Glioblastoma
Anaplastic meningeoma
Astrocytoma
Neuroblastoma
#
25
A 63-year-old woman is hospitalized secondary to markedly decreased vision. She has no history of polydipsia or
nocturia. Physical examination finds bilateral sluggish light reflexes and a bitemporal hemianopsia. No papilledema is
present, and her urine specific gravity is within normal limits. A CT scan of the head finds a suprasellar mass with
calcification. Which of the following is the most likely diagnosis?
@Craniopharingioma
Germinoma
Juvenile pilocytic astrocytoma
Medulloblastoma
Meningioma
#
26
The most frequent of all the following intracranial tumors in adults is
Ependymoma
Medulloblastoma
Meningioma
@Glioma
Metastasis
#
27
A 55-year-old woman is suspected of having a brain tumor because on the onset of seizure activity. Computerized
tomograms (CT scan) and skull X-rays demonstrate a mass in the right cerebral hemisphere that is markedly calcific. A
high index of suspicion should exist for
@Oligodendroglioma
Astrocytoma
Cerebral lymphoma
Metastatic carcinoma
Brown tumor
#
28
A 44-year-old woman present with the new onset of seizures along with increasing frequency of severe headaches. Her
medical history is otherwise unremarkable. Physical examination finds bilateral neurologic defects. Work-up reveals a
large, ill-defined, necrotic mass that involves both the right and left cerebral cortex. Histologic sections from this lesion
reveal a hypercellular tumor with pseudopalisading of tumor cells around large areal of serpentine necrosis. Numerous
atypical nuclei and mitoses are seen. This tumor is best classified as what type of high-grade neoplasm?
@Astrocytoma
Lymphoma
Medulloblastoma
Olidodendraglioma
Schwannoma
#
29
A 38-year-old woman has a seizure while shopping and is taken to the hospital. A scan of a brain demonstrated a poorly
circumscribed 5 cm tumor at right parietal lobe. A biopsy of this area contains of increased number of glial cell nuclei and
an interening feltwork of file cell processes that give the background a fibrillary appearance. Abnormal mitoses and
atypical central nervous system cells were absent within biopsy sample. What is the most likely diagnosis?
@Astrocytoma
Oligodendroglioma
Ganglioneuroma
Ependimoma
Chorioid papilloma
#
30
An 8-year-old boy is noted to have a palpable, firm mass in the region of the right neck after he complains of difficulty
swallowing. A head CT scan reveals a 7-cm solid soft tissue mass lateral to the esophagus on the right. Further history
indicates that this boy was affected by fetal alcohol syndrome and lie does poorly in school. A biopsy of this mass
demonstrates pleomorphic spindle cells that are cytokeratin negative, CD45 negative, and vimentin positive. This mass is
most likely to be a (an):
Wilms tumor
Rhabdomyosarcoma
@Neuroblastoma
Ewing's sarcoma
Malignant lymphoma
#
31
A 9-year-old boy who had been suffering from a gait disturbance for several weeks was found to have a posterior fossa
mass on CT scan. The most likely cause for these findings is
A berry aneurysm
Astrocytoma
@Medulloblastoma
Oligodendroglioma
Pseudotumur cerebri
#
32
A 5-years-old boy presents with projective vomiting and progressive ataxia.
Work-up finds obstructive hydrocephalus due to an infiltrative tumor originating in the cerebellum. Which of the
following is the most likely histologic diagnosis for this cerebellar tumor?
Ependimoma
Glioblastoma multiform
@Meduloblastoma
Oligodendroglioma
Schwannoma
#
33
At autopsy a 8-year-old child is found to have a poorly circumscribed tumor of cerebellum. Histologically investigation
reveled crowds of small immature cells with hyperchromatic, round-oval nuclei and scant cytoplasm. A few rosettes were
also found by pathologist at slide examination. What is the most likely diagnosis in that case?
@Medulloblastoma
Astrocytoma
Metastasis of cancer
Metastasis of sarcoma
Glioblastoma
#
34
The tumour of a brain is diagnosed for the patient with quickly increasing intracranial hypertension. The removed tumor
of a parietal -temporal part of brain had a soft consistence and a motley pattern of a cut surface. Histological investigation
revealed neoplastic tissue constructed of polymorphic cells with sites of necroses and a hemorrhage. What is the most
likely diagnosis?
@Glioblastoma
Oligodendroglioma
Neuroblastoma
Astrocytoma
Meningioma
#
35
Which of the following tumors is characterized by pseudopalisading, necrosis, endoneureal proliferation, hypercellularity,
and atypical nuclei?
Schwannoma
Medulloblastoma
Oligodendraglioma
@Glioblastoma multiforme
Ependymoma
#
36
Retinoblastoma, the most common intraocular tumor of children, is associated with all the following EXCEPT
Occurrence in both familial and sporadic patterns
Unilateral and unifocal sporadic tumors
Inactivation of cancer suppressor genes
@Poor prognosis even with treatment
Frequent histologic occurrence of rosettes
#
37
A child is born with a single functional allele of a tumor suppressor gene. At the age of five the remaining normal allele is
lost through a point mutation. As a result, the ability to continue the transition from Gl to the S phase of cell cycle is lost.
Which of the following neoplasms is most likely to arise via this mechanism:
Infiltrating ductal carcinoma of breast
Small cell anaplastic carcinoma of the lung
@Retinoblastoma of eye
Cerebral astrocytoma
Chronic myeloid leukemia
#
Private pathology
Anemia. Acute leukaemia.
#
1
Intravascular hemolysis results in all the following EXCEPT
Elevated plasma hemoglobin (hemoglobinemia)
Hemoglobinuria
Hemosiderinuria
Jaundice
@Splenomegaly
#
2
A 32-year-old woman who has recently started taking alpha-methyl DOPA
develops dark, tea-colored urine. Physical examination reveals mild sclera icterus,
a low-grade fever, and mild hepatosplenomegaly. Examination of her peripheral
blood reveals many microspherocytes, while laboratory examination finds a
positive Coombs’ test. Which of the following is the basic pathomechanism that
caused this individual’s signs and symptoms?
@Autoimmune destruction of red cells in the spleen
Drug-induced destruction of red blood cell precursors in the bone marrow
Hyperimmune destruction of neutrophils in the liver
Immune complex depositive in the capillaries of the kidneys
Isoimmune destruction of red blood cells in the peripheral blood
#
3
A 25-year-old woman present with the new onset of severe intermittent pain in her
fingers that developed shortly after she recovered from mycoplasma pneumonia.
She states the pain occurs when she goes outside in the cold, at which time her
fingers turn white and then become numb. Laboratory evaluation finds the
presence of an IgM autoantibody that is directed against the I-antigen found on the
surface of her blood cells. Based on these clinical findings, the diagnosis of
Raynaud’s phenomenon is made. Which of the following disorders are most likely
presents in this individual?
@Cold autoimmune hemolytic anemia
Isoimmune hemolytic anemia
Paroxysmal cold hemoglobinuria
Paroxysmal nocturnal hemoglobinuria
Warm autoimmune hemolytic anemia
#
4
A woman who is 5 weeks post partum (normal delivery, healthy child) develops
bleeding episodes with oliguria and hematuria. No fever or neurologic
manifestations are present. The blood urea nitrogen level is 65 mg/dL; a peripheral
blood smear is presented in the photomicrograph below. This patient most likely
has
Thrombotic thrombocytopenic purpura
Isoimmune thrombocytopenic purpura
@Hemolytic uremic syndrome
Disseminated intravascular coagulopathy
Sickle cell crisis
#
5
A 5-year-old child develops the sudden onset of bloody diarrhea, vomiting of
blood, hematuria, and renal failure following a flulike gastrointestinal illness. The
blood urea nitrogen (BUN) level is markedly increased, but fibrin degradation
product and blood clotting times are within normal limits. A peripheral blood
smear reveals poikilocytes, schistocytes, and a decreased in the number of
platelets. No fever or neurologic symptoms are present. Which of the following is
the most likely diagnosis?
Autoimmune thrombocytopenic purpura (autoimunne ITP)
Disseminated intravascular coagulopathy (DIC)
@Hemolytic-uremic syndrome (HUS)
Isoimmune thrombocytopenic purpura (isoimunne ITP)
Thrombotic thrombocytopenic purpura (ITP)
#
6
Patient 54 year-old, 5th day after surgical operation. Blood count: Erythrocytes
3,6*101 2/ l, Hemoglobin 95 g/l, Erythrocyte’s hemoglobin content (color index)
0,78; Leukocytes 16*109 /l, Platelets 450*109 / l Blood picture: anizocytosis,
poikilocytosis, reticulocytes- 3,8%. What anemia does this patient have?
Chronic posthemorragic anemia
Acquired hemolytic anemia
@Acute posthemorragic anemia
Anemia from iron deficiency
Hypoplastic anemia
#
7
An anemic patient is found to have hypochromic, microcytic red cells. Additional
tests reveal the serum iron levels the total iron-binding capacity, and the
transferring saturation to be reduced. A bone marrow biopsy reveals the iron to be
present mainly within macrophages. Which of the following is the most likely
diagnosis?
Iron deficiency
Thalassemia trait
@Anemia of chronic disease
Sideroblastic anemia
Pernicious anemia
#
8
A 55 y.o. woman consulted a doctor about having continuous cyclic uterine
hemorrhages for a year, weakness, dizziness. Examination revealed skin pallor.
Hemogram: Hb- 70 g/l, erythrocytes - 3,2*101 2/ l, color index - 0,6, leukocytes -
6,0*10 9 /l, reticulocytes - 1%; erythrocyte hypochromi A. What anemia is it?
Iron-deficiency anemia
B 12 -folate-deficiency anemia
Hemolytic anemia
Aplastic anemia
@ Chronic posthemorrhagic anemia
#
9
A 58-year-old woman complaints of increased tiredness, decreased capasity for
work, somnolence and dyspnea during fast walking. Blood test revealed:
erythrocytes - 4,6*101 2/ l, hemoglobin - 92 g/l, colour index - 0,6. Blood smear
demonstrated high contents of microcytes and anulocytes. What anemia is it typical
for?
Acute posthemorrhagic
Hemolytic
@Iron deficiency
Penicious
Sickle cell
#
10
A patient with hypochromic anemia has splitting and loss of hair, increased nail
brittling and taste alteration. What is the mechanism of the symptoms
development?
Decreased production of parathyrin
Decreased production of thyroid hormones
Deficiency of vitamin А
@Deficiency of iron-containing enzymes
Deficiency of vitamin В 12
#
11
A 38-year-old man, with a history of an ulcer, resulted in a stomach's resection, in
his blood test had a normal quantity of erythrocytes, but reduced hemoglobin's
concentration and decreased color index. An autopsy revealed pale skin and visible
mucous membranes; bone marrow of long tubular bones was brightly red.
Erythrocytes in a smear had a normal form and the sizes. They look very pale
because of bad staining by dyes. What pathological process took place in this case?
@Hypochromic iron deficiency anemia
B12-folic acid deficiency anemia
Acute lymphoblast leukemia
Sicklemia sickle cell anemia
Aplastic anemia
#
12
2 years ago a patient underwent resection of pyloric part of stomach. He complains
of weakness, periodical dark shadows beneath his eyes, dyspne A. In blood: Hb -
70 g/l, erythrocytes - 3,0*101 2/ l, colour index - 0,7. What changes of erythrocytes
in blood smears are the most typical for this condition?
Megalocytes
Ovalocytes
Schizocytes
Macrocytes
@Microcytes
#
13
Hereditary microspherocytic hemolytic anemia (Mincovsky-Shoffar disease) was
diagnosed in a woman, aged 34. What mechanism caused hemolysis of
erythrocytes in the patient?
@ Membranopathy
Enzymopathy
Hemoglobinopathy
Autoimmune impairment
Hypoplasia of bone marrow
#
14
Substitution of the glutamic acid on valine was revealed while examining initial
molecular structure. For what inherited pathology is this symptom typical?
Favism
Thalassemia
Minkowsky-Shauffard disease
@Sickle-cell anemia
Hemoglobinosis
#
15
A 20 year old patient complains of general weakness, dizziness, quick fatigability.
Blood analysis results: Hb- 80 g/l. Microscopical examination results: erythrocytes
are of modified form. This condition might be caused by:
Obturative jaundice
Addison's disease
Hepatocellular jaundice
Acute intermittent porphyria
@Sickle-cell anemia
#
16
A 25-year-old man presents because of a recurrent rash on the sunexposed areas of
his face and arms. He has recently moved to the United States from South Africa,
where he has lives all of his life. He states that he has always been sensitive to the
light and he says that his face will break out in a rash if hi stays in the sun too long.
He notes that sometimes alcohol ingestion will make these episodes worse.
Pertinent medical history includes episodes of neuropsychiatric changes, including
hallucinations and manic-depressive episodes. Physical examination reveals
multiple fluid-filled vesicle and bullae on his face and forearms. Laboratory
examination reveals elevated levels of delta-aminolevulinic acid and
porphobilinogen in the urine. This individual’s disorder results from the abnormal
synthesis of which one of the following substances?
Globin
@Heme
Immunoglobulin
Spectrin
Trancferrin
#
17
A 63-year-old man, with 20 years history of working as the engineer for the service
of electronic microscopes, died of a sepsis. An autopsy revealed plural
hemorrhages in serous and mucous membranes, a general hemosiderosis, a fatty
dystrophy of a myocardium, liver and kidneys, ulcerative -necrotic and purulent
processes in a gastro -intestinal system. The red bone marrow was replaced by a
fatty. What is the most likely diagnosis?
@Hypoplastic anemia
Megaloblastic anemia
Posthemorrhagic anemia
Iron deficiency anemia
#
18
A man, with a history of getting a high doze of ionizing radiation, presented to his
physician with marked stomatorrhagia (gingival hemorrhage), spontaneous skin
and mucosas' haemorrhages. A blood test showed a normochromal anemia and
pancytopenia. The concentration of iron in blood's serum was normal. A
histological investigation of a bone marrow puncture sample revealed a
replacement of a hemopoietic tissue by the fatty tissue. What is the most likely
diagnosis?
@Aplastic anemia
B12-folic acid deficiency anemia
Hemolytic anemia
Myelodysplastic syndrome
Immune cytopenia
#
19
A 67-year-old man present with increasing fatigue and is found to be anemic.
Physical examination reveals a hard 1-cm nodule in the left lobe of the prostate.
The prostatic-specific antigen (PSA) level is found to be elevated. Examination of
the peripheral blood reveals an occasional myelocyte. The erythrocytes are mainly
normochromic and normocytic, and teardrop RBCs are not found. There are
however, about two nucleated red blood cells per 100 white cells. Which of the
following is most likely diagnosis?
Fanconi’s anemia
Microangiophatic hemolytic anemia
@Mmyelophthisic anemia
Autoummun hemolytic anemia
Aplastic anemia
#
20
Surgical removal of a part of stomach resulted in disturbed absorption of vitamin
B 12 , it is excreted with feces. The patient was diagnosed with anemi A. What factor
is necessary for absorption of this vitamin?
Pepsin
Folic acid
Hydrochloric acid
Gastrin
@Gastromucoprotein
#
21
A 43-year-old man suffers from chronic atrophic gastritis and megaloblastic
hyperchromic anemia. He also has methylmalonic aciduria. Insufficiency of what
vitamin led to the development of such complex of symptoms?
Vitamin В 3
@Vitamin В 12
Vitamin В 2
Vitamin В 1
Vitamin В 5
#
22
In patient with hyperchrome anemia during the blood investigation predominance
of erythroblasts, normoblasts and megaloblasts is observed. Same cells are found
in the red bone marrow. Deficiency of what vitamin can course this disease?
@B12
A
PP
B6
C
#
23
A 44-year-old man presented to a gastroenterologist with pains in his epigastrium.
A physical examination revealed an icteritiousness of his skin and scleras, an
alteration of a tongue's mucous membrane. A tongue grossly looked shining,
smooth, with red spots. In peripheral blood's smear there were found enlarged
erythrocytes (megaloblasts). A histological study of a gastrobiopsy from a body of
a stomach showed a thinning of mucosa, a reduction of glands quantity,
superfluous growth of a connective tissue. Specify, what of diagnoses is the most
probable in this case:
@B12-folic acid deficiency anemia
Chronic posthemorrhagic anemia
Hemolytic anemia.
Chronic myeloid leukemia.
Aplastic anemia
#
24
A 56 year old patient came to a hospital with complaints about general weakness,
tongue pain and burning, sensation of limb numbness. In the past he underwent
resection of forestomach. In blood: Hb- 80 g/l; erythrocytes - 2,0*101 2/ l; colour
index - 1,2, leukocytes - 3,5*10 9 /l. What anemia type is it?
Iron-deficient
Hemolytic
Aplastic
@ B 12 -folate deficient
Posthemorrhagic
#
25
During the revision of the mouth cavity of a patient there was detected the atrophy
of the mucous membrane of the tongue with red macules (Gunter’s glossitis).
Sclerae are yellowish. Color index of the blood passes 1. What anaemia do these
changes characterize?
@B 12 -folic acid deficiency anaemia
Iron deficiency anaemia
Acute posthemorrhagic anaemia
Chronic posthemorrhagic anaemia
Hemolytic anaemia
#
26
A physical examination of a young men's oral cavity revealed the atrophy of
mucous membrane and red spots on his tongue (atrophic; Hunter's; Moeller's
glossitis). Sclera had a yellow coloring. A blood test showed the color index above
one. For what anemia these changes are characteristic?
@Nutritional anemia due to vitamine B12 deficiency
Asiderotic anemia
Acute posthemorrhagic
Chronic posthemorrhagic
Hemolytic anemia
#
27
A female patient complains of malaise, weakness, breathlessness, rapid fatigability,
and dizziness. Her blood test data: erythrocytes-1.8x1012/L, Hb-80 g/L,
leukocytes-3.2x109/L, color index-1.5. Anisocytosis, poikilocytosis, megaloblasts,
megalocytes were found in smear. What is the possible diagnosis?
@ B12-deficiency anemia
Posthemorragic anemia
Acute leukemia
Iron deficiency anemia
Immunohemolytic anemia
#
28
The patient, 55 years old, had undergone gastrectomy last year. After examination
doctor supposed the development of one of vitamin deficiency. What investigation
can confirm the diagnosis in this case?
@Investigation of blood – megaloblastic anemia
Investigation of daily urinary excretion of vitamin
Determination of bile excretion of vitamin
Determination of patient’s daily requirenment of vitamin
Investigation of activity of vitamin-depended enzymes
#
29
In patient with anemia during the blood investigation predominance of
erythroblasts, normoblasts and megaloblasts is observed. Same cells are found in
the red bone marrow. For what type of anemia this state is characteristic?
@Megaloblastic, pernicious
Hypochrome, iron-deficient
Hyperchrome, iron-deficient
Vitamin C-deficient
For all types of anemia
#
30
A 45-year-old woman presents with increasing fatigue, weakness, and tingling of
her arms and legs. Physical examination finds numbness and loss of balance,
position, and vibratory sense in both of her lower extremities. Histologic
examination of a smear made from a bone marrow aspiration reveals asynchrony in
red blood cell precursors between the maturation of their nuclei and their
cytoplasm. Additional work-up discovers achlorhydria, and a biopsy of the anturm
of her stomach reveals chronic atrophic gastritis. Which of the following is the
most likely diagnosis?
Fanconi’s anemia
Leukoerythroblastic anemia
@Megaloblastic anemia
Myelophthisic anemia
Sideroblastic anemia
#
31
A gross examination of a dead body revealed the skin's pallor and a yellowness of
a sclera. Livores mortis were not defined. The volume of blood in a heart and large
vessels was reduced. A blood looked aqueous. In a skin, mucosa and serous
membranes there were petechial hemorrhages. The internal organs, especially a
spleen, a liver and kidneys had a rusty color on a cut. A bone marrow of flat bones
was a crimson-red and succulent. In cortical [tubular, cylindrical] bones it looked
like a crimson jelly. Name the disease, connected with a deficiency of vitamin B12.
@Pernicious anemia
Drepancytic (sickle-cell) anemia
Panmyelophthisis
Toxic anemia
Acute posthemorrhagic anemia
#
32
A post-mortem of a 56-year-old woman revealed the Hunter's (atrophic; Moeller's)
glossitis, atrophy of mucous membrane of a stomach and liver's hemosiderosis. A
bone marrow in all investigated bones was red. A microscopical study showed
hyper cellular infiltration in a lamina propria of a stomach with a presence of
lymphatic follicles. In a spinal cord there was a funicular myelosis and also
haemopoiesis foci detected in a spleen. What is the most likely diagnosis?
@Addison-Biermer anemia
Fanconi's (congenital aplastic) anemia
Hypoplastic anemia
Chronic gastritis (type A)
Chronic gastritis (type B)
#
33
A 59-year-old woman presents with increasing fatigue and left-side upper
abdominal pain. Physical examination reveals a markedly enlarged spleen. No
enlarged lymph nodes are found. A CBC reveals a normocytic normochromic
anemia. Myelocytes, nucleated red blood cells, and teardrop-shaped red blood cells
are seen in the peripheral smear. A bone marrow aspiration could not be performed
due to a “dry tap”. The bone marrow biopsy specimen revealed a hypocellular
marrow as shown in the picture below. Marrow reticulin was markedly increased
in amount. The peripheral leukocyte alkaline phosphatase score was within normal
limits. Which of the following clinical finding is most likely to be present in this
individual?
@Extramedullary hematopoiesis in the spleen
Increased total protein in the serum
Multiple black stones in the gallbladder
Multiple lytic lesions in the skull
Sequestration of neotrophils in the liver
#
34
Acute idiopathic thrombocytopenic purpura (ITP) is characterized by
An insidious onset
Being more common in females of childbearing age
@A history of recent viral infection
Megakaryocytic hypoplasia in the bone marrow
A high mortality
#
35
A 37-year-old woman who has a clinical picture of fever, splenomegaly, varying
neurologic manifestations and purplish ecchymoses of the skin is found to have a
hemoglobin level of 10.0 g/dL, a mean corpuscular hemoglobin concentration
(MCHC) of 48, peripheral blood polychromasia with stippled macrocytes, and
apherocytes, with a blood urea nitrogen level of 68 mg/dL. The findings of
coagulation studies and the patient’s fibrin-degradation products are not overtly
abnormal. Which of the following is the most likely diagnosis?
Idiopathic thrombocytopenic purpura
@Thrombotic thrombocytopenic purpura
Disseminated intravascular coagulopathy
Submassive hepatic necrosis
Waterhouse-Friderichsen syndrome
#
36
The bone marrow biopsy shown below was performed because of splenomegaly
and anemia in an adult. On the basic of the appearance of the bone marrow core,
choose the most likely diagnosis.
Chronic myeloid leukemia (CML)
Aplastic anemia
Acute leukemia
@Myeloid metaplasia with myelofibrosis
Microagiopathic hemolytic anemia
#
37
A patient's blood was analyzed and the decreased erythrocyte’s sedimentation rate
(ESR) was discovered. What disease from the listed below is accompanied with
decreased ESR?
Myocardial infarction
Hepatitis
Splenomegaly
@Polycytemia
Vitamin B deficiency
#
38
As a result of the damage of one of the Atomic Power Plant reactors the run-out of
radioelements took place. People in the high-radiation area were radiated with
approximately 250-300 r. They were immediately hospitalized. What changes in
the blood count would be typical for the victims?
Leukopenia
Thrombopenia
Neutropenia
Anemia
@Lymphopenia
#
39
After overdoses of x -rays destruction of B lymphocytes (B cells) in human
hematopoietic organs happened. Reduction of what blood plasma substancies will
be observe in this case?
@Immunoglobulins (antibodies)
Albumins
Cholesterol
Glucose
Fibrinogen
#
40
After the radiactive exposure a patient has stem cells disorder. The regeneration of
what cells of friable connective tissue will be damaged?
@Macrophages
Pericytes
Fibroblasts
Pigment cells
Adipocytes
#
41
A 23 y.o. patient complains of weakness, temperature rise up to $38-40^0C$.
Objectively: liver and spleen are enlarge d. Hemogram: Hb- 100 g/l, erythrocytes -
2,9*101 2/ l, leukocytes - 4,4*109/l, thrombocytes – 48*109 / l, segmentonuclear
neutrophils - 17%, lymphocytes - 15%, blast cells - 68%. All cytochemical
reactions are negativ e. Make a hematological conclusion:
Acute erythromyelosis
Acute myeloblastic leukosis
Chronic myeloleukosis
@ Undifferentiated leukosis
Acute lymphoblastic leukosis
#
42
A patient with acute myeloblast leucosis has developed liver and spleen
enlargement, anemia, myeloblasts in peripheral blood. What principal sign allows
to differ myeloblast leukosis from chronic one?
@Leukemic collapse
Pancytopenia
Anemia
Blast cells in peripheral blood
Thrombocytopenia
#
43
A 28-year-old man presents with widespread ecchymoses and bleeding gums.
Physical examination reveals enlargement of this spleen and liver. Laboratory
examination of his peripheral blood reveals a normochromic, normocytic anemia,
along with a decreased number of platelet and an increased number of white blood
cells. Coagulation studies reveal prolonged prothrombin and partial thromboblastin
time and increased fibrinogen degradation product. Examination of the patient’s
bone marrow reveals the presence of numerous granular-appearing blast cells with
numerous Auer rods. These immature cells make up about 38%of the nucleated
cells in the marrow. Which of the following is the most likely diagnosis?
Acute erythroid leukemia
Acute lymphoblastic leukemia
Acute monocytic leukemia
Acute myelomonocytic leukemia
@Acute promyelocytic leukemia
#
44
Acute lymphoblastic leukemia was diagnosed in a 10-year-old child. When this
child later developed a patchy pulmonary infiltrative and respiratory insufficiency,
a lung biopsy was performed. The material obtained by biopsy was they stained
with Gomori’s methenamine-silver stain and is shown in the photomicrograph
below. In consideration of the patient’s signs and microscopic evaluations, the
prognosis is now complicated by
Pseudomonas pneumonia
Aspergillus pneumonia
@ Pneumocystis carinii pneumonia
Pneumococcal pneumonia
Influenza pneumonia
#
45
A 4-year-old girl is being evaluated for the sudden onset of multiple petechiae and
bruises. She is found to have a peripheral leukocyte count of 55,000, 86% of which
are small, homogeneous cells that have nuclei with immature chromatin. Indistinct
nucleoli are also present. Initial tests on these immature cells are as follows: TdT,
positive; PAS, positive; acid phosphatase, positive; and myeloperoxidase, negative.
Based on these findings, which of the following in the cell of origin of these
immature cells?
Myeloblasts
Monoblasts
Megakaryoblasts
@Lymphobblasts
Erythroblasts
#
46
A post-mortem of a 4-year-old girl revealed plural petechial hemorrhages on her
skin, serous and mucous membranes, large focal hemorrhage in a brain and
necrotic tonsillitis. Microscopical study showed multiple cell infiltrates with
prevailing lymphocytes in a bone marrow, a liver, a spleen, a thymus, lymph
nodes, tonsils and a skin. What is the most likely diagnosis?
@Acute lymphatic leukemia
Chronic lymphatic leukemia
Hodgkin's lymphoma
Follicular non-Hodgkin's lymphoma
Mycosis fungoides
#
47
In a 45-year-old patient the ulcerative-necrotic damage of the mucosa of the oral
cavity takes place; also the spread lymphadenopathy, slight spleno- and
hepatomegaly, diffuse hemorrhages in the skin and mucous membranes were found
out. In blood analysis the increasing of leukocytes (to 100 10 per 1 ml) at the
account of lymphoblasts takes place. What diagnosis is more probable?
@Acute lymphoblastic leukemia
Chronic myelocytic leukemia
Chronic lymphocytic leukemia
Plasmocytosis
Acute promyelocytic leukemia
#
48
The death of a 7-year-old boy was by acute haemorrhagic anaemia, caused by
profuse bleeding from the digestive tract. During the pathoanatomic examination
following facts were revealed: macroscopically – less of blood in the internal
organs, the enlargement of different groups of lymphatic nodules, thymomegaly,
moderately signified hepatosplenomegaly, brightly red medulla; microscopically –
hypercellular medulla with monomorphic infiltration of blast cells, diffuse-focal
tumour infiltrates in a liver, a spleen, lymphatic nodules, brain tunics and
substance. Diagnose the disease.
@Acute lymphoblastic leucaemia
Acute myeloblastic leucaemia
Acute undifferentiated leucaemia
Acute monoblastic leucaemia
Acute plasmoblastic leucaemia
#
49
A 4-year-old girl died due to a post-hemorrhagic anemia, resulted from a
gastro-intestinal profuse bleeding. An autopsy revealed an anemia of her organs,
the enlargement of the different groups of lymph nodes, thymomegaly, mild
hepatomegaly, splenomegaly and bright red bone marrow. Microscopical study
showed hyper cellularity of a bone marrow with monomorphic blast cells infiltrate,
diffuse inflammatory tumor-like infiltrates in a liver, a spleen, lymph nodes,
meninges and substance of a brain. What is the most likely diagnosis?
@Acute lymphoblastic leucosis
Acute myeloblastic leucosis
Acute nondifferentiated leucosis
Acute monoblastic leucosis
Acute plazmoblastic leucosis
#
Chronic leukaemias.
Lymphomas.
#
1
All the following are known to cause splenomegaly EXCEPT
@Sickle cell disease
Hodgkin’s disease
Chronic lymphocytic leukemia (CLL)
Hairy cell leukemia
Polycythemia vera
#
2
Patient, 55 years old, is admitted to a hospital with spontaneous fracture of neck of
the femur. During the last year motiveless pains in the muscles and bones were
observed. Investigation result shown that concentration of protein in the
blood serum of the patient is 115 g/l. What more probably reason for such quantity
of blood serum
@Paraproteinemia
Diet rich of protein
Podagra (gout)
Myodistrophy
Kidney insufficiency
#
3
A 62 year woman complains of frequent pain attacks in the area of her chest and
backbone, rib fractures. Her doctor suspected myeloma (plasmocytoma). What of
the following laboratory characteristics will be of the greatest diagnostic
importance?
Hyperalbuminemia
Hypoproteinemia
Hypoglobulinemia
@ Paraproteinemia
Proteinuria
#
4
The presence in serum of a mu heavy-chain protein is associated with which of the
following disease?
@Chronic lymphocytic leukemia
Lymphoblastic lymphoma
Poorly-differentiated lymphocytic lymphoma
Plasma cell myeloma
Multiple myeloma
#
5
A 64-year-old man is being evaluated for pancytopenia. Clinical examination
reveals a marked increase in the size of his spleen. A bone marrow biopsy
histologically reveals replacement of the normal marrow cells by a diffuse
proliferation of the small mononuclear cells. There is a clear space around each of
these abnormal cells (“fried egg” apperance). These cells stained positively for
cytoplasmic acid phosphatase and this staining was not inhibited by treatment with
tartrate. A bone marrow did not reveal any marrow particles. Although his
peripheral leukocyte count was decreased, the majority of the cells were
lymphocytes. Which of the following is the expected histologic appearance of
these lymphocytes?
“Cerebriform” lymphocytes
“Chicken-footprint” lymphocytes
“Cloverleaf” lymphocytes
@”Hiry” lymphocytes
“Smudge” lymphocytes
#
6
A 54-year-old woman presents with headaches, viral abnormalities, bleeding, and
Raynaud’s phenomenon. Work-up reveals normal serum calcium, and no lytic
lesions are found within the skeleton by x-ray. Serum electrophoresis reveals a
single large M spike in the peripheral blood due to a monoclonal proliferation of
IgM. Sections from the patient’s bone marrow reveal numerous plasma cells,
lymphocytes, and plasmacytoid lymphocytes. Which of the following is the most
likely diagnosis?
IgM multiple myeloma
Monoclonal gammopathy of undetermined significance
Mu heavy-chain disease
Plasmacytoid small-cell lymphoma
@Waldenstrom’s macroglobulinemia
#
7
Clinical investigation of a patient revealed the enlarged lymph nodes, spleen and
liver. A microscopical study of the enlarged cervical (neck) lymph node showed
the blurring of its structures and absence of lymphatic follicles. All microscopical
views were presented by cells with round nuclei and narrow ring of a basophilic
cytoplasm. What is the most likely diagnosis?
@ Lymphatic leucosis
Lymphogranulomatosis
Lymphosarcoma
Myeloleukemia
Multiple myeloma
#
8
A 72-year-old man presents with increasing fatigue. Physical examination reveals
an elderly man in no apparent distress (NAD). He is found to have multiple
enlarged, nontender lymph nodes along with an enlarged liver and spleen.
Laboratory examination of his peripheral blood reveals a normocytic
normochromic anemia, a slightly decreased platelet count, and a leukocyte count of
72,000 cells/μL. An example of his peripheral blood is seen in the picture below.
Which of the following is the most likely diagnosis?
Acute lymphoblastice leukemia
Atypical lymphocytosis
@Chronic lymphocytic leukemia
Immunoblastic lymphoma
Prolymphocytic leukemia
#
9
An autopsy of a 35-year-old woman revealed the enlarged spleen, (weight 800gm),
liver (weight 4000 gm) and lymph nodes. A bone marrow of a femur diaphysis was
juicy (succulent), crimson-red color. Microscopical study of a liver defined dense
infiltrates within portal tracts, consisted of immature blood cells. These cells had a
round nuclei and narrow ring of a cytoplasm. What is the most likely diagnosis?
@Chronic lymphatic leukemia
Chronic myeloid leucosis
Generalizated form of lymphogranulomatosis
Acute myeloblasts leucosis
Acute lymphoblastic leucosis
#
10
A liver biopsy was taken from a 66-year-old man, with a history of increased
quantity of lymphocytes and pro-lymphocytes in his blood. A histological
investigation of a liver sample revealed plural accumulations of the mentioned
above cells, mainly between hepatic segments. For what disease above listed
changes are characteristic?
@Chronic lymphatic leukemia
Acute lymphatic leukemia
Lymphogranulomatosis
Chronic persistence hepatitis
Hepatocellular carcinoma of liver
#
11
A radiological investigation of a man's head revealed in his maxillary and
mandibular bones numerous round defects with smooth walls. A histological study
showed osteolysis and osteoporosis accompanied with insufficient bone repair. The
laboratory test of the urine detected the Bence Jones protein. What is the most
likely diagnosis?
@Multiple myeloma
Chronic myeloleukemia
Chronic erythromyelosis
Acute myeloleukemia
Acute nondifferentiated leucosis
Bone fibrous dysplasia
#
12
Patient, 60 years old, is admitted to a hospital with spontaneous fracture of neck of
the femur. During the last year motiveless pains in the muscles and bones were
observed. Result of biochemical investigation shown that concentration of protein
in the blood serum of the patient is 115 g/l, protein of Bence Jones is present. What
diagnosis is it possible to think about?
@Multiple myeloma
Hartnup’s disease
Podagra (gout)
Myodistrophy
Dermatomyositis
#
13
A bone marrow aspirate was obtained from a 70-year-old man whose symptoms
included weakness, weight loss, and recurrent infection. Laboratory finding
included proteinuria, anemia, and an abnormal component in serum proteins. A
photomicrograph of the bone marrow aspirate is shown below. The most probable
diagnosis is
Monomyelocytic leukemia
Hitiocytic leukemia
@Multiple myeloma
Gaucher’s disease
Leukemic reticuloendoteliosis
#
14
A 70-year-old man presents with increasing weakness and weight loss. Laboratory
finding include anemia, hypercalcemia, and increased serum protein. A
photomicrograph of the bone marrow aspirate is shown below. Which of the
following histologic finding is present in this photomicrograph?
Accumulation of glycogen in hystiocytes, as seen with Gaucher’s disease
Decreased numbers of myeloid precursors, as seen with aplastic anemia
Increased numbers of myeloblasts, as seen acute myelocitic leukemia
@Increased numbers of plasma cells, as seen multiple myeloma
Numerous inclusions in eritroid precursors, as seen with parvovirus infection
#
15
A 34-year-old male is found to have weakness confusion, other neurologic
manifestation and polyuria X-ray examination revealed multifocal destructive bone
lesions throughout the skeletal system. Electrophoretic analysis revealed increased
level of Ig in the blood and Bence Jones protein in the urine. What is the most
likely diagnosis?
@Myeloma
Acute monocytic leukemia
Chronic myeloleukemia
Lymphogranulomatosis
Histiocytosis
#
16
A 65-year-old female is noted to be anemic. He serum protein electrophoresis
demonstration a large monoclonal Ig G kappa protein. In her bone marrow are
increased numbers of atypical plasma cells. Her skull X-ray show multiple lytic
areas. What is the most likely diagnosis?
@Myeloma
Polycytemia vera (erythremia, Osier's disease)
Lymphocytic lymphoma
Chondrosarcoma
#
17
An elderly male is admitted to the hospital for treatment of humeral bone fracture.
His fracture's area X-ray showed a new growth and lytic zone within lesion.
Histological examination of a biopsy revealed abnormal plasma cells. What is the
most likely diagnosis?
@Myeloma
Chronic osteomyelitis
Chondrosarcoma.
Bone fibrous dysplasia
Metastasis of adenocarcinoma
#
18
Chronic myeloid leukemia is LEAST likely to be associated with
Splenomegaly
Basophilia
@Translocation (8; 14)
Thrombocytosis
Low leukocyte alkaline phosphatase (LAP)
#
19
At the roenngenological examination were detected multiple defects that looked
like round apertures with plain walls. Histologicaly were revealed the facts of
osteolysis and osteoporosis against the background of bad osteogenesis. Bence
Jones protein is presented in the urine. Name the disease.
Acute undifferentiated leucaemia
Chronic myeloleucaemia
Chronic erythromyelosis
Acute myeloleucaemia
@Myeloid disease
#
20
Multiple focuses of osteoporosis and osteolysis were visualized in plane bones of
the patient during the roenngenological examination. At the biopsy material from
bones was revealed an increased amount of tumour plasmatic cells. What is your
diagnosis?
Histiocytosis
Acute monocytic leucaemia
Chronic myeloleucaemia
Lymphogranulomatosis
@Myeloid disease
#
21
A 42-year-old female is admitted to the hospital for treatment a blood disorder. A
disease complicated with pneumonia led to lethal outcome. An autopsy revealed
hyperplasia of a bone marrow with "pus" - like appearance, splenomegaly (5 kg
weight), hepatomegaly (6 kg weight), systemic lymph nodes enlargement. What is
the most likely diagnosis?
@Chronic myeloleukemia
Chronic lymphatic leukemia
Myeloma
Polycytemia vera (erythremia, Osier's disease)
Lymphogranulomatosis
#
22
Hyperplasia of medulla in plane and tubular bones (pyoid medulla), splenomegaly
(6kg), hepatomegaly (5kg), an enlargement of all groups of lymphatic nodules
were revealed at the autopsy. What is the most possible disease?
Myeloid disease
Chronic lympholeucaemia
@Chronic myeloleucaemia
Real polycythaemia
Lymphogranulomatosis
#
23
A patient presented with an infiltrative, plaque-like polymorphic skin rash, which
had various contours, sizes and a congested -cyanotic color. The lesions tended to
peripheral growth and fusion. A microscopical investigation of a skin biopsy
revealed massive lymphocytes' proliferation, which occupied the entire derma and
a hypodermic fatty layer. What is the most likely diagnosis?
@Limphoma of skin
Systemic lupus erythematosus
Mycosis fungoi'des
Intradermal nevus
-
#
24
A 20-year-old man presents in the emergency room with respiratory distress
resulting from a lymphoma involving his mediastinum. These malignant
lymphocytes most likely have cell surface markers characteristic of what type of
cell?
B lymphocytes
@T lymphocytes
Macrophages
Dendritic reticulum cells
Langenhars cells
#
25
The non-Hodgkin’s lymphoma pictured in the photomicrograph below may be
characterized by which of the following?
Increased frequency in adolescents
Lymphoblastic lymphoma
@B lymphocytes
Tingible-body macrophages nodules
Well-differentiated lymphocytic lymphoma
#
26
A 60-year-old man presents with several enlarged lymph nodes involving his right
axilla. A biopsy from one in these enlarged lymph nodes reveals effacement of the
normal lymph node architecture by a diffuse proliferation of small,
normal-appearing lymphocytes. These same cells are present in the peripheral
blood, enough to cause a peripheral lynphocytosis. A bone marrow biopsy reveals
a diffuse proliferation of these small lymphocytes. Special stains reveal these cells
to be positive for CD5 and CD23. Which of the following is the classification for
this type of lymphoma?
A high grade non-Hodgkin’s lymphoma (using the Working Formulation)
@A low grade non-Hodgkin’s lymphoma (using the Working Formulation)
A precursor B cell neoplasm (using the REAL classification)
A peripheral T cell neoplasm (using the REAL classification)
A poorly differentiated lymphoma (using the Working Formulation)
#
27
A 65-year-old presents with several enlarged lymph nodes in his left
supraclavicular region. Physical examination reveals painless lymphadenopathy in
this region. No other abnormalities are found. A biopsy from one of these enlarged
lymph nodes, which is shown in the associated picture, reveals effacement of the
normal lymph node architecture by numerous nodules of uniform size that are
crowded within the cortex and medulla of the lymph node. Tangible-body
macrophages are not seen in these nodules. Which of the following in the most
likely diagnosis?
@Follicular non-Hodgkin’s lymphoma
Lymphocyte predominate Hodgkin’s disease
Metastatic adenocarcinoma
Reactive follicular hyperplasia
Small lymphocytic lymphoma
#
28
An 8-year-old African girl develops rapidly enlarging mass that involves a large
portion of the right side of her maxilla. A smear made from an incisional biopsy of
this mass reveals malignant cells of cytoplasmic vacuolis that stain positively with
oil red O. Histologic section from this biopsy reveal a diffuse, monotonous
proliferation of small, noncleavedlymphocytes. In the background are numerous
tangible-body macrophages that impart a “starry-sky” appearance to the slide.
Which of the following viruses is most closely associated with this malignancy?
Cytomegalovirus (CMV)
@Epstein-Barr virus (EBV)
Herpes simplex virus (HSV)
Human immunodifeciency virus (HIV)
Human pappilomavirus (HPV)
#
29
A 25-year-old patient presented with peripheral adenopathy, involving a single
cervical lymph node. A biopsy sample investigation revealed an unclear node
pattern with mixed cellular infiltration. Histologycal study under higher
magnification showed numerous variants of Berezovsky-Reed- Stenberg cells,
lymphocytes, necrotic area and mild diffuse fibrosis. What is the most likely
diagnosis?.
@ Lymphogranulomatosis
Nodular lymphoma
Burkitt's lymphoma
Lymphocytic lymphoma
Chronic lymphatic leukemia
#
30
The young man presented to his physician with enlarged neck lymph nodes. A
microscopical investigation of a lymph node biopsy sample revealed the lymphoid
tissue proliferation with presence of a giant Reed—Berezovsky - Sternberg cells,
eosinocytes, zones of necrosis and sclerosis. What is the most likely diagnosis?
@Lymphogranulomatosis
Chronic lymphatic leukemia
Multiple myeloma
Lymphosarcoma
Histiocytosis
#
31
A microscopic investigation of the enlarged neck lymph node biopsy revealed the
blurring of its structures, plenty of proliferating lymphocytes with adding of
solitary giant Reed—Berezovsky-Sternberg cells. What is the most likely
diagnosis?
@Lymphogranulomatosis, with predominance of lymphatic tissue
Lymphogranulomatosis, with exhaustion of lymphatic tissue
Mixed cell variant of lymphogranulomatosis
Lymphosarcoma
Nodular sclerotic variant of lymphogranulomatosis
#
32
A biopsy of the enlarged lymph node was taken. A histological investigation
revealed a diffuse growth of lymphoid cells with adding of eosinocytes, atypical
histiocytes, solitary giant Reed—Berezovsky-Sternberg cells with two and more
nuclei, cell's necrosis and sclerosis. What is the most likely diagnosis?
@Lymphogranulomatosis
Lymphatic leukemia
Burkett's lymphoma
Sarcoidosis
Myeloleukemia
#
33
At young men the increased cervical lymph node is removed. Microscopic
investigation revealed the altered node's structure, an absence of lymphoid
follicles, sites of a sclerosis and necrosis. Thecellular infiltrate is polymorphic with
a presence of lymphocytes, eosinocytes, and atypical one- nuclear cells and
multinuclear giant cells (Reed—Berezovsky - Sternberg cells). What is the most
likely diagnosis?
@Lymphogranulomatosis
Acute lymphatic leukemia
Chronic lymphatic leukemia
Burkett's lymphoma
Mycosis fungoides
#
34
A tomography revealed enlarged lymphatic nodes. A histological investigation of
lymph node's biopsy showed a circular growths of a connective tissue, which
surrounded a granuloma – like formation, made from lymphocytes, plasmocytes
and giant double- nuclear cells. What is the most likely diagnosis?
@Lymphogranulomatosis
Lymphosarcoma
Tuberculosis
Sarcoidosis
Lymphatic leukemia
#
35
A physical examination of a 42-year-old man revealed enlarged lymph nodes. A
histological investigation of a lymph node showed lymphocytes, histiocytes,
reticular cells, small and big Hodgkin's cells, multinuclear Berezovsky
-Reed-Sternberg cells (Sternberg-Reed cells) infiltration with solitary necrotic
areas. What disease such changes characteristic for?
@Lymphogranulomatosis
Lymphosarcoma
Chronic leucosis
Acute leucosis
Metastasis of carcinoma of lungs
#
36
A patient has a cluster of matted together dense lymph nodes on his neck.
Histological examination of a removed lymph node revealed proliferation of
reticular cells, presense of Reed-Sternberg cells. What disease is meant?
@Lymphogranulomatosis
Myeloblastic leukosis
Lymphocytic leukosis
Lymphoblastic leukosis
Myelocytic leukosis
#
37
At autopsy an elderly female is found to have enlarged groups of a neck, axillary
and mediastinal lymph nodes matted together. They were firm and rubbery. The
cut surface was gray-white, producing a "fish-flesh" appearance. Microscopy
revealed heterogeneous cellular infiltrate wich contained lymphocytes, classic and
mononuclear Reed-Berezovsky-Sternberg cells. What is the most likely diagnosis?
@Lymphogranulomatosis
Chronic lymphatic leukemia
Lymphosarcoma
Retikulosarcoma
Mycosis fungoides
#
38
An autopsy of a 67-year-old man revealed the systemic enlargement of lymph
nodes with formation of tumorous conglomerates. The spleen was also enlarged
with a motley pattern of a cut surface. There were plural, tiny, yellowish-white
spots on a red background of a spleen's pulp. What is the most likely diagnosis?
@Lymphogranulomatosis
Sarcoidosis
Lymphosarcoma
Carcinoma of lung
Lymphatic leukemia
#
39
A 67-year-old female presented with painless enlargement of lymph nodes.
Histological examination of a biopsy sample revealed that the whole lymph node
pattern was unclear, with heterogeneous cellular infiltrate. It included abnormal,
immature cells admixed with lymphocytes, eosinophils, plasma cells and
macrophages. Binucleated Berezovsky-Reed-Sternberg cells, where surrounded by
multiple cell types. What is the most likely diagnosis?
@Lymphogranulomatosis
Acute myeloleukemia
Chronic myeloleukemia
Mycosis fungoides
Tuberculosis
#
40
A 14-year-old boy presented to a hospital with enlarged submaxillary and cervical
lymph nodes. A biopsy procedure was performed. Microscopical investigation
revealed the disorder of a lymph node typical structure, a heterogeneous cellular
population with a presence of giant multinuclear cells and plural one-nuclear big
cells. There were also eosinocytes, neutrophils and lymphocytes in the cell
infiltrate. In addition, sites of necrosis and sclerosis were found. What is the most
likely diagnosis?
@Lymphogranulomatosis
Hyperplasia of lymphatic nodes
Granulomatous lymphadenitis
Purulent lymphadenitis
Non-Hodgkin's lymphoma
#
41
At the stomatological examination of a 16-year-old teenager, enlarged
submandibular and cervical lymphatic nodules were diagnosed. A biopsy was
made. Microscopically the typical structure is wiped and the cellular population is
heterogeneous. Giant cells with multi nucleus and multiple mononuclear cells of a
big size are presented. There are also eosinophilic and neutrophilic leucocytes,
lymphocytes, areas of necrosis and focuses of sclerosis. Diagnose the disease.
Hyperplasia of a lymphatic nodule
@Lymphogranulomatosis
Lymphogranulomatous lymphadenitis
Purulent lymphadenitis
Non-Hodgkin’s lymphoma
#
42
Microscopic examination of the enlarged neck gland of a 14-year-old girl revealed
destruction of the tissue structure of the node, absence of the lymph follicles,
sclerotic areas and necrosis foci, cell constitution of the node is polymorphous,
lymphocites, eosinophiles, big atypical cells with multilobular nuclei
(Beresovsky-Sternberg cells) and mononuclear cells of the large size are present.
What is the most likely diagnosis?
Fungoid mycosis
Acute lympholeucosis
Chronic lympholeucosis
Berkitt's lymphoma
@Lymphogranulomatosis
#
43
A 22-year-old woman has enlarged lymphatic nodules. Histologically there are
lymphocytes, histiocytes, reticular cells, small and big Hodgkin’s cells,
multinuclear Beresovski-Sternberg’s cells and single focuses of caseous necrosis.
What disease is meant?
Chronic leukosis
Lymphosarcoma
@Lymphogranulomatosis
Acute leukosis
Metastasis of pulmonary cancer
#
44
A thoracotomy of a 58-year-old woman revealed in her anterior mediastinum the
enlarged and soldered together lymph nodes. Microscopical investigation revealed
atypical cells with predominance of Hodgkin cells and giant Reed—Berezovsky -
Sternberg cells. A sclerosis was absent. What is the most likely diagnosis?
@Lymphogranulomatosis with low-spirited development of lymphoid tissue
Lymphogranulomatosis with predominance of nodularis sclerosis
Lymphogranulomatosis with predominance of lymphoid tissue
Lymphosarcoma
Mixed-cellular variant of lymphogranulomatosis
#
45
A post-mortem of a 15-year-old girl revealed enlarged neck, mediastinal and
mesenteric lymph nodes, which were integrated in conglomerates. On a cut section,
the tissue pattern of the nodes was non-uniform, with foci of necrosis.
Microscopical investigation showed the uneven structure of lymph nodes, foci of
sclerosis and necrosis. The cell population was also heterogeneous and included
uninuclear atypical cells, giant multinuclear atypical cells, a significant amount of
eosinocytes and neutrophils, and sparse lymphocytes. What is the most likely form
of Hodgkin lymphoma (lymphogranulomatosis)?
@Lymphogranulomatosis, mixed cell variant
Lymphogranulomatosis, lymphohistiocytic variant
Lymphogranulomatosis, nodular sclerosis
Lymphogranulomatosis, variant with low-spirited development of lymphoid tissue
Hodgkin's sarcoma
#
46
A patient is 60 years old. During the thoracotomy enlarged and knitted lymphatic
nodules were diagnosed in the mediastinum anterius, where a biopsy material was
taken. Microscopically atypical cells were revealed, mostly Hodgkin’s and
Beresovski-Sternberg’s giant cells; sclerosis was absent. What is the most possible
disease?
@Lymphogranulomatosis with a depressed development of lymphoid tissue
Lymphogranulomatosis with a domination of nodular sclerosis
Lymphogranulomatosis with a domination of lymphoid tissue
Lymphosarcoma
Mixed-cellular variant of lymphogranulomatosis
#
47
A 66-year-old man presents for his annual physical examination. Hi is
asymptomatic and physical examination is unremarkable. Examination of his
peripheral smear, however, reveal the presence of small mononuclear cells with
little cytoplasm and a mature nucleus with a prominent nuclear cleft. No “smudge
cells” are seen. The presence of these “buttock cells” in the peripheral blood
warrants further clinical work-up to search for which one of the following
malignancies?
Chronic lymphocytic leukemia
Follicular non-Hodgkin’s lymphoma
Multiple myeloma
@Nodular sclerosis Hodgkin’s disease
Small-cell carcinoma of the lung
#
48
A 22-year-old woman presents with fever, weing loss, night sweats, and painless
enlargement of several supraclaviculars lymph nodes. A biopsy from one of the
enlarged lymph nodes is shown in the photomicrograph below. The binucleate
giant cell with prominent acidofilic “owl-eye” nucleoli shown stain positively with
both CD15 and CD30 immunoperoxidase stains. Also present are atypical
mononucleare cells that are surrounded by clear spaces (lacunar cells). Which of
the following is the most likely diagnosis?
Anaplastic large cell lymphoma
Diffuse non-Hodgkin’s lymphoma
Lymphocyte predominate Hodgkin’s disease
@Nodular sclerosis Hodgkin’s disease
Reactive lymph node hyperplasia
#
CARDIO-VASCULAR SYSTEM
Aterosclerosis.
Hypertensive disease
#
1
A 25-year-old man experiences chest pain on exercise when he attempts to climb
three flights of stairs. This pain is relieved by sublingual nitroglycerin. Laboratory
studies show a total serum cholesterol of 550 mg/dL with an HDL cholesterol
component of 25 mg/dL. He is worried about these findings because his brother
died of a myocardial infarction at age 34. Which of the following conditions is this
man most likely to have?
Diabetes mellitus, type II
Malignant hypertension
Familial hypercholesterolemia
Gushing syndrome
Morbid obesity
#
2
A 70 year old man is ill with vascular atherosclerosis of lower extremities and
coronary heart disease. Examination revealed disturbance of lipidic blood
composition. The main factor of atherosclerosis pathogenesis is the excess of the
following lipoproteins:
@ Low-density lipoproteins
Intermediate density lipoproteins
Cholesterol
High-density lipoproteins
Chylomicrons
#
3
A 56-year-old has not received any medical care nor seen a physician for years. He
reports reduced exercise tolerance over the past 5 years. On occasion in the past
year he has noted chest pain after ascending a flight of stairs. He smokes 2 packs of
cigarettes per day. He is found to have a blood pressure of 155/95 mm Hg. His
body mass index is 30. Laboratory findings include a total serum cholesterol of 245
mg/dL with an HDL cholesterol that is 22 mg/dL. Which of the following vascular
abnormalities is most likely to be his most serious health risk?
Hyperplastic arteriolosclerosis
Lymphedema
Medial calcific sclerosis
@Atherosclerosis
Deep venous thrombosis
Plexiform arteriopathy
#
4
An autopsy study reveals that evidence for atheroma formation can begin even in
children. The gross appearances of the aortas are recorded and compared with
microscopic findings of atheroma formation. Which of the following is most likely
to be the first visible gross evidence for the formation of an atheroma?
Thrombus
@Fatty streak
Calcification
Hemorrhage
Ulceration
#
5
An aorta at autopsy that demonstrated a mild degree of atherosclerosis with only
lipid streake and just a few atheromatous plaques would be most consistent with
which of the following histories from patients:
A 62-year-old man with familial hypercholesterolemia
@A 48-year-old man who exercises regularly
A 45-year-old woman with nodular glomerulosclerosis
A l0-year-old child killer in a motor vehicle accident.
A 56-year-old man with angina
#
6
A 45-year-old male died in a traffic accident. A histological investigation of his
aorta revealed a lipid infiltration of its intima with fats accumulation in myocytes
and macrophages of aortal medial layer. Name the stage of atherosclerosis?
@Lipoidosis
Liposclerosis
Atheromatosis
Pre-lipoidosis
Atherocalcification
#
7
A post-mortem of a 65-year-old male revealed yellowish streaks and spots in the
abdominal aorta's intima, which did not rise above endothelial surface. These areas
were stained by Sudan III in orange color. What stage of atherosclerosis such
changes are characteristic for?
@Lipidosis
Liposclerosis
Atheromatosis
Ulceration
Atherocalcification
#
8
At autopsy of 52-year-old male revealed yellowish areas at his aorta's intima, this
did not bulge above its surface. Histological investigation showed the
accumulation of cells with foamy cytoplasm, stained by Sudan III in yellow color.
Name the most likely stage of atherosclerosis in aorta?
@Lipidosis
Liposclerosis
Atheromatosis
Pre-lipoidosis
Atherocalcification
#
9
A 46-year-old male suddenly died after developing a heart failure. An autopsy
revealed in abdominal aorta the yellow color areas, which did not rise over its
surface. Histological investigation of aorta showed the aggregations of cells with a
foamy cytoplasm among smooth mussels and macrophages. These cells had a
motley-orange coloring, when stained by a Sudan III. For what stage of an
atherosclerosis such picture is characteristic?
@Lipoidosis
Liposclerosis
Atheromatosis
Ulceration
Atherocalcification
#
10
At autopsy of an elderly female an aorta tissue sample collected for histology.
Microscopic investigation revealed in aorta's intima the accumulation of
xanthoma's cells. At what disease such morphological picture is possible?
@Atherosclerosis
Hypertension
Syphilitic mesaortitis
Nonspecific aortoarteritis
Nodular periarteritis
#
11
The first visible gross evidence for the formation of an atheroma in an artery such
as the aorta, even in a young person, is a(an):
Thrombus
@ Fatty streak
Calcification
Hemorrhage
Inflammatory infiitra.tr;
#
12
A 53-year-old female died from acute myocardial infarction. An autopsy revealed
multiple whitish dense plaques in her coronal arteries' intimae, which bulged in and
narrowed the vessel's lumen. Name the most likely stage of a coronary
atherosclerosis?
@Liposclerosis
Lipoidosis
Atheromatosis
Atherocalcification
Ulceration
#
13
During the section of a 69-year-old woman with overweight, who died of acute
myocardial infarction, were revealed multiple whitish solid formations in the
tunica intima of coronary arteries. They bulge into the lumen and constrict it. What
stage of atherosclerosis is described?
Stage of a creation of an atheromatous ulcer
Lipoidosis
Atheromatosis
Atherocalcinosis
@Liposclerosis
#
14
A post-mortem of an elderly male revealed microscopic alteration of his coronary
artery. There was narrowing of vessel's lumen due to a fibrous plaque with some
admixture of lipids. Name the stage of atherosclerosis:
@Liposclerosis
Lipoidosis
Pre-lipoidosis
Atheromatosis
Atherocalcification
#
15
A 54-year-old male died after developing a heart failure. An autopsy revealed a
chronic venous hyperemia of the internal organs, hypertrophy of the left ventricle
of a heart and focal cardiosclerosis. Gross investigation of aorta showed
yellow-white plaques in the intima with fine-grained masses in their centers, which
infiltrated the aortal wall. Give the name of this pathological process.
@Atheromatosis
Lipoidosis
Liposclerosis
Arteriolosclerosis
Calcinosis.
#
16
A 73-year-old woman who exercises regularly falls down the stairs and injures her
right hip. A radiograph is taken of the pelvis. There is no fracture but the
radiograph reveals calcification of the small muscular arteries in the region of the
pelvis. What is the probable vascular lesion which accounts for this calcification?
@Ulcerative atherosclerosis
Calcific medial sclerosis
Metastatic calcification
Trauma
Dystrophic calcification
#
17
If the coronary arteries on dissection at autopsy show severe atherosclerosis in a
patient who had clinically acute (60-h) myocardial infarction, you would expect to
find
Gross evidence of myocardial scarring
Coronary thrombosis in 90 percent of cases
Coronary thrombosis in 65 percent of cases
@A plaque with ulceration, fissure, or hemorrhage
Rupture of a papillary muscle
#
18
Which of the following complications is LEAST likely to occur in a 49-year-old
woman who has developed severe generalized atherosclerosis over the past 20
years:
Sudden death
Gangrene of the right big toe
@Pulmonary artery atherosclerosis
Remote (healed) myocardial infarction
Retinopathy with blindness
#
19
A 63-year-old man has had insulin dependent diabetes mellitus for over two
decades. The degree of control of his disease is characterized by the laboratory
finding of a hemoglobinAIC of 10.1%. He has noted episodes of abdominal pain
following meals, these episodes have worsened over the past year. On physical
examination, there are no masses and no organomegaly of the abdomen, and he has
no tenderness to palpation. Which of the following pathologic findings is most
likely to be present in this man?
Ruptured aortic aneurysm
Hepatic infarction
@Mesenteric artery occlusion
Acute pancreatitis
Chronic renal failure
#
20
A 70-year-old man has noted coldness and numbness of his lower left leg,
increasing over the past 4 months. He also experiences pain in this extremity when
he tries walking more than the distance of half a city block. On physical
examination, his dorsalis pedis, posterior tibial, and popliteal artery pulses are not
palpable. Which diagnosis is he most likely to have?
General atherosclerosis
Trombophlebitis
Atheromatosis
Atherocalcinosis
@Atherosclerosis of a. femoralis
#
21
A patient has tissue ischemia lower the knee joint, accompanied with "intermittent
claudication". Which artery is occluded in this case?
Posterior tibial
Proximal part of the femoral
Anterior tibial
@Popliteal
Fibial
#
22
The gross appearance an aorta with severe ulcerative atherosclerosis is associated
with each of the following additional findings EXCEPT:
Mural thrombosis
Diabetes mellitus, type I
Hypercholester-olemia
@Neopiasia
Aneurysm formation
#
23
A 66-year-old male suddenly died on his way to operation room. An autopsy
revealed a hemorrhage into retroperitoneal fat, a saccular dilatation of the
abdominal aorta wall with rupture in arterial wall. The aorta's defect had uneven
edges and stony hardening of surrounding tissues. Name the disease, resulted in
described complication?
@Atherosclerosis
Hypertension
Systemic vasculitis
Visceral Syphilis
-
#
24
A 56-year-old man present with the sudden onset of excruciating pain. He
describes the pain as beginning in the anterior chest, radiating to the back, and then
moving downward into the abdomen. His blood pressure is found to be 160/115.
Your differential diagnosis includes myocardial infarction; however, no changes
are seen on ECG, and you consider this to be less of a possibility. You obtain an
x-ray of this patient’s abdomen and discover a “double-barrel” aorta. Which of the
following is the basic cause of this abnormality?
A microbial infection
Loss of elastic tissue in the media
A congenital defect in the wall of the aorta
@Atherosclerosis of the abdominal aorta
Abnormal collagen synthesis
#
25
Under macroscopic (gross) investigation of aorta yellow patches and stripes,
fibrous plaques, ulcers were found out in its intima. What disease is characterized
with such changes?
@Atherosclerosis
Rheumatic fever
Tuberculosis
Diphtheria
Typhoid fever
#
26
A 77-year-old woman has experienced abdominal pain for the past month. On
physical examination, she has a pulsatile lower abdominal mass. An abdominal CT
scan reveals an abdominal aorta dilated to 7 cm in diameter, with calcification of
the aortic wall. While awaiting surgery, she has an episode of severe abdominal
pain and a repeat CT scan shows marked retroperitoneal hemorrhage. Which of the
following diagnosis is she most likely to have?
Atherosclerosis of aorta
General atherosclerosis
True aneurysm
False aneurysm
Stratified aneurysm
#
27
Hypertension is most closely related to the formation of which one of the following
types of aneurysms?
Berry aneurysm
Atherosclerotic aneurysm
Mycotic aneurysm
@Charcot-Bouchard aneurysm
Saccular aneurysm
#
28
An autopsy study is conducted involving the gross appearance of the aorta of
adults ranging in age from 60 to 90. In some of these patients, the aorta
demonstrates atheromatous plaques covering from 70 to 95% of the intimal surface
area, mainly in the abdominal portion, with ulceration and calcification. Which of
the following contributing causes of death are these patients most likely to have?
Hyperparathyroidis
@Hypertension
Marfan syndrome
Thrombophlebitis
Vasculitis
#
29
A 49-year-old woman has experienced marked pain in her lower extremities on
ambulation more than 300 meters for the past 5 months. On physical examination,
her lower extremities are cool and pale, without swelling or erythema. No dorsalis
pedis or posterior tibial pulses are palpable. Her body mass index is 32. She is a
smoker. Which of the following abnormalities of the vasculature is most likely to
account for these findings?
Lymphatic obstruction
@Arteriolosclerosis
Atherosclerosis
Medial calcific sclerosis
Venous thrombosis
#
30
An elderly woman, with a long history of bronchial asthma, ischemic heart disease
and hypertension, presented with hypertrophy of her heart's left ventricle. Name
the cause of the heart alteration?
@Arterial hypertension
Pulmonary insufficiency
Chronic ischemia of cardiac muscle
Bronchial asthma
Emphysema of lights
#
31
A 63-years-old patient, with a history of hypertensive disease, died from cardiac
insufficiency. At autopsy, the heart enlargement and dilated ventricular cavities
were revealed. Microscopical investigation showed marked hypertrophy of
cardiomyocytes, with their fatty dystrophy and hyperchromic barrel-like nucleuses.
What pathological process is the most likely in a heart?
@Eccentric hypertrophy
Myocarditis
Concentric hypertrophy
Angiogenic cardiosclerosis
Cardiomyopathy
#
32
An artery demonstrates concentric endothelial cell proliferation which markedly
narrows the lumen, resulting in focal ischemia and hemorrhage. This histologic
finding is likely to be found:
Most frequently in the small vessels of the liver
Most commonly in the enderly
@In association with progressive hypertension (diastolic >110 mm Hg)
Accompanying an acute myocardial infarction
As a result of hereditary hypercholesterolemia
#
33
The form of vascular disease responsible for malignant hypertension is
Medial calcific sclerosis
Arteriosclerosis obliterans
@Hyperplastic arteriolosclerosis
Hyaline arteriolosclerosis
Thromboangiitis obliterans
#
34
A 41-year-old woman present with recurrent severe headaches and increasing
visual problems. Physical examination reveals her blood pressure to be 220/150.
Her symptoms are most likely to be associated with which one of the following
abnormalities?
Medial calcific sclerosis
Arteriosclerosis obliterans
@Hyperplastic arteriolosclerosis
Hyaline arteriolosclerosis
Tromboangiitis obliterans
#
35
A 64-years-old male died from uremia. At autopsy revealed reduced in sizes
kidneys, with 50.0g weight. Their surfaces were regularly granulated; the cortexes
were thin. A microscopic study showed considerably thickened walls of glomerular
arterioles due to deposition of homogeneous unstructured pink masses. The lumens
of these vessels were markedly narrowed; nephrons were diminished with sclerotic
changes, tubules were atrophic. For what disease the described changes are
characteristic?
@Hypertension
Chronic glomerulonephritis
Amyloidosis of kidneys
Pyelonephritis with scarring of kidneys
Acute necrotic nephrosis
#
36
Malignant hypertension in a 44-year-old African-American man with a blood
pressure of 275/150 mm Hg is most closely associated with which of the following
pathologic lesions:
@Hyperplastic arteriolosclerosis with fibrinoid necrosis
Nodular glomerulonecrosis and renal hyaline arteriosclerosis
Monckeberg's medial calcific arterial sclerosis
Severe occlusive coronary artery atherosclerosis with recent thrombosis
Thrombophlebitis with pelvic vein thrombosis
#
37
A 52-year-old male had a long history of hypertension. He presented to emergency
care physician with complains of an acute, persisted for few hours rise of a blood
pressure. What is the most likely alteration of the arterioles' walls due to
hypertonic crisis?
@Fibrinous necrosis •
Hyalinosis
Sclerosis
Amyloidosis
Calcinosis
#
38
A patient with hypertensive disease was diagnosed a long hypertensive crisis.
Name the pathomorphologic changes in the arterioles walls during the hypertensive
crisis.
@Fibrinoid necrosis
Hyalinosis
Sclerosis
Amyloidosis
Calcinosis
#
39
At autopsy, the kidneys are of normal size. Their surfaces appear finely granular.
There are small hemorrhages noted. The cortices appear pale. Microscopically,
many small renal arteries and arterioles demonstrate concentric intimal thickening
with marked luminal narrowing. These finding are most likely to be present in
which of the following patients:
A 54-year-old man with chronic renal failure and amyloidosis
A 29- year-old woman with proteinuria and lupus nephritis
@A 44-year-old woman with marked hypertension and scleroderma
A 49-year-old man with acute cellular allograft rejection
A 19- year-old woman with graft versus host disease
#
40
An elderly male, with 15 years history of essential hypertension, died from renal
failure. What would be the most likely gross picture of his kidneys at autopsy?
@Small, dense, a surface is fine-grained.
Large pied and soft.
Large red
Large white and smooth
Large with the plural thin-walled cysts.
#
41
A 67-year-old patient had hypertensive disease for 20 years and died of chronic
renal insufficiency. How did the kidneys look at the autopsy?
Big, white
Big, mottled
Big, red
@Little, solid with fine-grained surface
Big with numerous thin-walled cysts
#
42
At autopsy the diminished kidneys with weight of 50.0 have been found out, the
surface has been closed-grained, cortex has been uniformly thinned. At
microscopic examination a wall of arterioles has been considerably thickened
because of deposition of homogeneous unstructured pink colored masses, the
lumen has sharply narrowed down; the glomeruli have been reduced, with sclerosis
and atrophy of tubules. What disease
@Hypertensive disease
Chronic glomerulonephritis
Amyloidosis of kidney
Pyelonephritis with shrinkage of kidneys
Acute necrotic nephrosis
#
43
An autopsy of a 61-year -old female revealed the thickening of her heart's left
ventricle up to 2.5 cm. Her kidneys were small, contracted and firm. The surface of
the kidneys had a granular appearance. On a cut, there was a thinning of a cortical
substance. Microscopical investigation showed arteriolosclerosis,
glomerulosclerosis and interstitial sclerosis. What is the most likely disease?
@Hypertension
Atherosclerosis
Ischemic heart disease
Rheumatism
Amyloidosis of kidneys
#
44
A 65-year-old male long time was ill by hypertensive disease and died from
chronic kidney insufficiency. The autopsy showed, that both kidneys are
considerably decrease in sizes, their surfaces are granulated; histologically - most
glomeruluses are containing hyaline, part of them in sclerotic condition, other in
hyperplasia; in stroma - the fields of sclerosis, arteriolo- and arteriosclerosis,
elastofibrosis of large kidney arteries branches. What is the name of the exposed
changes?
@Arteriolosclerotic nephrosclerosis
Atherosclerotic nephrosclerosis
Secondary-restricted kidney
Chronic pyelonephritis
Amyloidal- scarring kidney.
#
45
A 60-year-old man was ill with hypertensive disease for a long time and died of
chronic renal insufficiency. At the autopsy was revealed that both kidneys were
greatly decreased in size. Their surface was fine-grained. Histologically: the
majority of glomeruli were hyalinized, part of them was sclerosed, and some of
them were hyperplased. Stroma had the areas of necrosis, arteriolo- and
arteriosclerosis, elastofibrosis of large branches of renal arteries. Name these
changes.
Secondary wrinkled kidney
Aterosclerotic nephrosclerosis
Chronic glomerulonephritis
Chronic pyelonephritis
@Arteriolosclerotic nephrosclerosis
#
46
A 64-years-old patient died from the brain's hemorrhage. An autopsy revealed
markedly decreased kidneys (6x3x2cm in sizes and 60, 0 grams weight). They
were dense, anemic with an even, fine- grained surface. On a cut, there was a
constant thinning of the kidneys' cortex. Changes in kidneys are the implication of:
@Arteriolosclerotic nephrosclerosis
Atherosclerotic nephrosclerosis
Secondary- scarring kidney
Amyloidal- scarring kidney.
Gouty kidneys
#
47
A 57-year-old man has had blood pressure measurements in the range of 160/95 to
180/110 mm Hg for many years. He has taken no medications. A renal scan reveals
kidneys of normal size for age. These finding with benign nephrosclerosis are
most likely to occur with which of the following vascular change:
@Hyaline arteriolosclerosis
Monckeberg's medial calcific sclerosis
Complex calcified atherosclerosis
Arterial mural thrombosis
Hyperplastic arteriolosclerosis
#
48
An autopsy of 48-year-old patient, who died from the complications of
hypertensive disease, revealed small, dense kidneys with fine-grained surface.
Parenchyma and cortex matter were atrophied. Give the name for such kidneys.
@Primary-scarring kidneys
Amyloidal-scarring kidneys
Secondary-scarring kidneys
Pyelonephritic-scarring of kidneys
-
#
49
A 63-year-old man, with a long history of hypertensive disease, died from a
hemorrhage in the brain. An autopsy revealed reduced in sizes, firm kidneys with
fine-grained surface and thinned cortex matter. These changes in kidneys are
characteristic for:
@Primary-scarring kidneys
Secondary-scarring kidneys
Amyloidal-scarring kidneys
Pyelonephritic-scarring of kidneys
Infarct of the kidney
#
50
An autopsy of an elderly woman, with a long history of hypertensive disease,
revealed small size and weight (80 grams), firm kidneys. They had grey color and
granular surface. On a cut, there was uniform thinning of a cortex. How it is
possible to name the changes in kidneys?
@Primary- scarring kidneys
Pyelonephritic scarring of kidneys
Secondary - scarring kidneys
Amyloidal- scarring kidneys
Diabetic glomerulosclerosis
#
51
At the autopsy of a man who died of hypertensive disease’ complications, small
solid kidneys with fine-grained surface were revealed. They had atrophic
parenchyma and cortex. Name these kidneys.
Amyloid-wrinkled
@Primary wrinkled
Secondary wrinkled
Pyelonephritic-wrinkled
Atrophic
#
52
An elderly male, with 20 years history of hypertension, died from uremia. An
autopsy revealed a heart hypertrophy and diffuses cardiosclerosis. There were also
small, dense kidneys with granularity of their surface. Histological investigation of
kidneys tissue showed the collapse of glomerular's arterioles and sclerosis. Some
glomeruli were replaced with pinkish homogenous masses, negative to the
Kongo-red staining. Tubules were atrophic. Name the kidney pathology?
@Primary-scarring kidneys
Secondary - scarring kidneys
Amyloidosis
Chronic glomerulonephritis
Chronic pyelonephritis
#
53
A 53-year-old man presents with severe headaches, nausea, and vomiting. He also
relates seeing sport before his eyes and is found to have a diastolic blood pressure
of 160 mmHg. Microscopic examination of a renal biopsy demonstrates
hyperplastic arteriolitis. Gross examination of his kidneys is most likely to reveal
which one of the following changes?
A finely granular appearance to the surface
@Multiple small petechial hemorrhages on the surface
Diffuse, irregular cortical scars overlying dilated calyces
Cortical scars overlying dilated calyces in renal poles
Depressed cortical areas overlying necrotic papillae of varying stages
#
54
Choose correct features of atherosclerosis (3 points):
Acute disease
@Chronic disease
Lesions develop in the small-sized arteries
Lesions develop in the both small-sized and large-sized arteries
@Aorta have been damaged
Aorta and large veins have been damaged
Branches of pulmonary artery have been damaged
Develops due to lipid disorders
@Develops due to both lipid and protein disorders
#
55
Choose correct gross-view atherosclerotic changes ofarteria(4 points):
@Fatty streaks
Liposclerosis
Ulceration of intima of small-sized arteries with mural thrombosis
@Ulceration of intima of aorta with intramural hemorrhages
Increased permeability ofendothelial cells and basement membrane
@Fibrous plague
Intracellular accumulation of the lipids
@Fibrofatty atheroma
#
56
Choose correct morphological features characterized fatty streaks changes of
atherosclerosis (4 points):
@Have been revealed in all ages
@More often develop in the zones of branches of aorta
For the first time appear in the abdominal part of aorta
Are elevated upon the intima
@Are not be elevated upon the surface of the intima
Lead to stenosis of the lumen of the aorta
@Histologically are intimal aggregation of foam cells derived from the
macrophages
Histologically are intimal aggregation of lipid cells derived from adipose tissue
#
57
Choose correct features of the malignant form of hypertension disease (3 points):
Hyaline arteriolosclerosis
@Hyperplastic arteriolosclerosis
@Concentric, laminated thickening of the walls of arterioles
Excentric, laminated thickening of the walls of arterioles
Necrotizing phlebitis
Necrotizing arteritis
@Necrotizing arteriolitis
#
58
All features of hypertension disease are true EXEPT (3 points):
Infarctionofmyocardium
@Infarction of spleen
@Infarction of brain
Hemorrhages of brain
Hyaline arteriolosclerosis
Hyperplastic arteriolosclerosis
@Necrotizing arteritis
#
Cerebro-vascular disease.
Heart diseases.
#
1
A 45-year-old man dies suddenly and unexpectedly. The immediate cause of death
is found to be a hemorrhage in the right basal ganglia region. On microscopic
examination his renal artery branches have concentric endothelial cell proliferation
which markedly narrows the lumen, resulting in focal ischemia and hemorrhage of
the renal parenchyma. An elevation in which of the following substances in his
blood is most likely to be associated with these findings?
@Cholesterol
Renin
Troponin I
Triglyceride
C-reactive protein
#
2
A 66-year-old woman has the sudden loss of movement on part of the left side of
her body. She has smoked a pack of cigarettes a day for the past 45 years. She has
vital signs including T 37.1 C, P 80/minute, R 16/minute, and BP 160/100 mm Hg.
A cerebral angiogram reveals occlusion of a branch of her middle cerebral artery.
Laboratory findings include a hemoglobin A1C of 9%. Which of the following
components of blood lipids is most important in contributing to her disease?
Chylomicron
Lipoprotein
C lipase
VLDL
@HDL cholesterol
#
3
A 50 year-old patient had hemorrhage of the brain and was taken to the hospital.
The place of hemorrhage was revealed on the lateral hemispheres surfaces during
the medical examination. What artety was injured?
@The middle cerebral artery
The posterior cerebral artery
The anterior cerebral artery
The posterior communicating artery
The anterior communicating artery
#
4
The majority of cases of subarachnoid hemorrhage result from
Transaction of a branch of the middle meningeal artery
Bleeding from torn bridging veins
@Rupture of a preexisting aneurysm
Rupture of an arteriovenous malformation
Cortical bleeding occurring opposite the point of a traumatic injury
#
5
Subdural hematomas occur most frequently in the
Supracerebellar region
Infracerebellar region
Cerebellopontine angle
Pituitary region
@Cerebral hemisphere convexities
#
6
Which of the following conditions is the most frequent cause of intracerebral
hemorrhage?
Ruptured aneurysm
Trauma
Blood dyscrasias
Angoimas
@Hypertensive vascular disease
#
7
A 68-year-old woman can not move her upper and lower right extremities after
stroke. Muscle tone of these extremities and reflexes are increased. There are
pathological reflexes. What form of the paralysis is it?
Monoplegia
Paraplegia
Dissociation
Tetraplegia
@Hemiplegia
#
8
A patient after hypertension stroke does not have voluntary movements in his right
arm and leg with the increased muscle tone in these extremites. What type of
disfunction of nervous system is it?
Reflex paresis
Peripheral paresis
Central paresis
@Central paralysis
Peripheral paralysis
#
9
Cerebral embolism occurs frequently in association with all the following
conditions EXCEPT
Cardiac mural thrombi
Left-sided endocarditis
@Right-sided endocarditis
Cardiac catheterization
Prosthetic cardiac valves
IBS
#
10
The patient has come to the hospital from the smelting workshop in the condition
of hyperthermia. What is the direct cause of loss of consciousness at the heat
stroke?
@Decreased brain blood supply
Arterial pressure drop
Increased water loss through sweating
Dilatation of peripheral vessels
Decrease of heart output
#
11
A big pallid focus of a softened gray substance of pappy consistence was revealed
in the right temporal lobe at the autopsy of a 58-year-old man. Numerous
whity-yellow nodes of intima that constricted the lumen were detected in the
arteries of the cerebral basis. What is your diagnosis?
Abscess of the cerebrum
@Ischemic insult
Haemorrhage
Haemorrhagic infarction
Edema of the cerebrum
#
12
Autopsy of a 56 y.o. man revealed in the right temporal part of brain a big focus of
softened grey matter that was semi-liquid and light grey. Arteries of cerebral tela
contain multiple whitish-yellow thickenings of intima that abruptly narrow the
lumen. What is your diagnosis?
Brain edema
Hemorrhagic infarction
@ Ischemic stroke
Brain abscess
Hemorrhage
#
13
During the section of a dead man, who had atherosclerosis, a thrombosis of a
branch of internal carotid was revealed. There was also a gray focus of humid
softening of the brain tissue. What pathological process is diagnosed in the
cerebrum?
Encephalitis
Haemorrhagic infiltration
Haematoma
@Ischemic infarction
Braingrowth (the tumour of the brain)
#
14
At autopsy of 63-year-old male revealed an atherosclerosis of the brain's arteries
and a thrombosis of the internal carotid artery's branch. Gross investigation showed
a focus of moist softening in his brain's tissue. Define the pathological process in
the.brain.
@Ischemic infarction
Hemorrhagic infiltration
Hematoma
Encephalitis
Tumour of the brain.
#
15
Unexpected sudden cardiac death is a very rare complication of
Severe coronary artery disease
Myocarditis
Cardiac tamponade
@Mitral valve prolapsed
Dilated or hypertrophic cirdiomyopathy
#
16
A 32-year-old man suddenly died during the emotionally strained work. An
autopsy revealed uneven myocardium blood supply. Histochemical investigation
detected a decrease of the amount of glycogen. Electron-microscopical study
showed a destruction of mitochondria, the contractures of myofibrils. What is the
most likely disorder of a blood circulation?
@Acute ischemia
Chronic ischemia
Vacating arterial hyperemia
Acute vein hyperemia
Angioneurotic arterial hyperemia
#
17
A 48-year-old patient after severe psychoemotional exertion suddenly began
feeling sharp pain in the heart region, irradiating into left arm. Nitroglycerin
releaved pain 10 minutes later. What pathogenetic mechanism is responsible for
the development of pain in this case?
Compression of coronary vessels
@Spasm of coronary vessels
Dilation of peripheral vessels
Occlusion of coronary vessels
Increase of myocardial needs in oxygen
#
18
The appearance of coronary artery thrombosis in a 49-year-old man with sudden
onset of chest pain is LEAST likely to be associated with:
A long history of diabetes mellitus
Hypercholesterolemia
Acute myocardial infarction
@Recurrent pneumonia
A history of smoking
#
19
A 61-year-old man has the sudden onset of severe chest pain. Vital signs include T
37°C, P 101/minute, R 20/minute, and BP 80/40 mm Hg. An electrocardiogram
demonstrates changes that are consistent with myocardial ischemia involving the
left lateral ventricular free wall. He is given thrombolytic therapy with tissue
plasminogen activator (tPA). However his serum creatinine kinase is found to be
450 U/L 3 hours after this therapy. Which of the following cellular events has most
likely occurred?
Cellular regeneration
Drug-induced necrosis
@Reperfusion injury
Increased synthesis of creatine kinase
Myofiber atrophy
#
20
A 52-year-old man has the sudden onset of chest pain. He is found to have a serum
troponin I of 5 ng/mL. A year later he has reduced exercise tolerance. An
echocardiogram reveals an akinetic segment of left ventricle, and he has reduced
cardiac output, with an ejection fraction of 25%. He then experiences a transient
ischemic attack (TIA). His serum troponin I is now <0.5 ng/mL. Thrombosis
involving which of the following locations is most likely to have put him at
greatest risk for the TIA?
Saphenous vein
Vertebral artery
Superior vena cava
Left ventricle
@Coronary artery
#
21
Severe coronary atherosclerosis leads to an acute myocardial infarction in a 55 year
old woman. Which of the following factors is LEAST likely to have contributed to
this:
Endothelial injury occurred in the first 3 cm of the coronary artery frorn turbulent
flow
Platelets were adherent to a damaged intima following appearance of a lipid streak
Disruption of an intimal plaqua led to thrombus formation in the left anterior
descending artery
Macrophages released growth factors that led to smooth muscle migration and
proliferation
@A reduced amount of low density lipoproteins with LDL cholesterol were
present in the blood
#
22
12 hours after an accute attack of retrosternal pain a patient presented a jump of
aspartate aminotransferase activity in blood serum. What pathology is this
deviation typical for?
Collagenosis
Viral hepatitis
@ Myocardium infarction
Diabetes mellitus
Diabetes insipidus
#
23
Marked increase of activity of МВ-forms of CPK (creatinephosphokinase) and
LDH-1 were revealed on the examination of the patient's blood. What is the most
likely pathology?
@Miocardial infarction
Pancreatitis
Rheumatism
Cholecystitis
Hepatitis
#
24
A patient developed substernal pains in the 7 o'clock in the morning. He presented
at the Emergency Department in 8 o'clock in the morning, where
electrocardiograph investigation revealed a myocardial infarction. Ten minutes
later he died. What most reliable morphological sign of myocardium infarction
would be found at histological investigation after the autopsy?
@Disappearance of glycogen in myocardial cells
Vacuolar dystrophy of myocardial cells
Fatty infiltration of myocardial cells
Necrosis of myocardial cells
Weakening of myofibril of myocardial cells
#
25
A 58-year-old male, with a history of hypertensive disease, developed a long attack
of substernal pain. The infarction of myocardium was diagnosed. A patient died
soon. An autopsy revealed flabby myocardium with uneven blood filling.
Histological and histochemical examinations showed disappearance of glycogen
granules and decreased activity of oxidizing enzymes. What stage of myocardial
infarction presented in that case?
@Ischemic stage
Necrotic stage.
Organization
Recurrent myocardial infarction
Acute relapsing myocardial infarction
#
26
A patient has myocardial infarction in the region ofanterior wall of the left
ventricle. What artery basin has the circulatory impairment occured in?
Atrioventricular branch of left coronal artery
Circumflex branch of left coronal artery
Left marginal branch of left coronal artery
@Anterior interventricular branch of left coronal artery
Anterior ventricular branches of right coronal artery
#
27
The pathology evident in the photomicrograph below usually first appears after
which of the following lengths of time following a myocardial infarction?
12 h
@3 days
7 days
14 days
28 days
#
28
A patient died from acute cardiac insufficiency. The histological examination of
his heart revealed in myocardium of the left ventricle the necrotized section, which
was separated from undamaged tissue by the zone of hyperimic vessels, small
hemorrhages and leukocytic infiltration. What is the most likely diagnosis?
@Myocardial infarction
Productive myocarditis
Myocardial ischemic dystrophy
Focal exudate myocarditis
Diffuse exudate myocarditis
#
29
An autopsy of a 62-year-old male, with a history of ischemic heart disease,
revealed an atherosclerosis of coronal arteries and signs of hypertensive disease.
On a cut of the heart, in the area of the apex and left ventricle's frontal and lateral
walls, there was a well defined yellowish focus, surrounded by hemorrhages. What
is the most likely pathological process in the cardiac muscle?
@Myocardium infarction
Postinfarction cardiosclerosis
Diffuse cardiosclerosis
Myocarditis
Fatty dystrophy of myocardium
#
30
During the section of a man who died of pulmonary edema was revealed a big
yellow-gray focus in the myocardium. There was also a fresh clot in the coronary
artery. Specify the diagnosis.
Myocarditis
Cardiosclerosis
@Myocardial infarction
Amyloidosis
Cardiomyopathy
#
31
An autopsy of a 49-year-old patient, who died from lungs edema, revealed in
myocardium a yellow- grey, large focus and a fresh blood clot in a coronal artery.
What is the most likely diagnosis?
@Myocardium infarction
Cardiosclerosis
Myocarditis
Amyloidosis
Cardiomyopathy
#
32
An autopsy of a 56-year-old male, with a history of ischemic heart disease,
revealed the edema of lungs. What pathological changes could cause a pulmonary
disorder?
@Acute insufficiency of left ventricle
Acute general anemia
Acute insufficiency of right ventricle
Ischemia of small circle
Blood stasis
#
33
A patient aged 59 was hospitalised to cardiological department in a sever state,
with diagnosis of acute myocardial infarction of the posterior wall of the left
ventricle and septum, and primary pulmonary edema. What is the primary
mechanism, which causes the development of pulmonary edema in the patient?
* Left ventricular failure
Pulmonary arterial hypertension
Pulmonary venous hypertension
Hypoxemia
Decrease of alveolocapillary diffusion of oxygen
#
34
Histologic sections (routine H&E stain) of lung reveal the alveoli to be filled with
pale, nongranular pink fluid. Neither leukocytes nor erythrocytes are present within
this fluid. Which of the following is the most common cause of this abnormality?
Bacterial pneumonia
@Congestive heart failure
Lymphatic obstruction by tumor
Pulmonary embolus
Viral pneumonia
#
35
A 66-year-old patient presented in the hospital with the acute recurrent myocardial
infarction of front-lateral wall of the left ventricle. On the 4th day of disease, an
acute, marked difficulty in breathing, a cough with considerable quantity of a
foamy sputum discharge and facial cyanosis developed. A patient died from
progressive cardiac insufficiency. An autopsy revealed enlarged grey- pink color
lungs. A foamy liquid flowed down from the cut surface. What pathological
process in lungs caused a death?
@Edema of the lungs
Lungs infarction
Hydrothorax
Pneumonia
Pneumosclerosis
#
36
A 55-year-old woman has been treated in the hospital for pancreatitis for the past
three weeks. She is examined one morning on rounds and found to have a swollen
right leg. It is tender to palpation posteriorly but is not warm. This condition is
most likely to be the result of which of the following vascular complications?
Venous thrombosis
Septic embolization
@Congestive heart failure
Cellulitis
-
#
37
Laminar necrosis and watershed infarcts are most suggestive of
@Shock
Hypertension
Fat emboli
Vascular thrombosis
Verous sinus thrombosis
#
38
The mortality from myocardial infarction is most closely related to the occurrence
of
A pericardial effusion
Pulmonary edema
Coronary artery thrombosis
@An arrhythmia
Systemic hypotension
#
39
Several days following a myocardial infarction, a 51-year-old man develops the
sudden onset of a new pansystolic murmur along with diastolic flow murmur.
Work-up reveal increased left atrial pressure that develops late in systole and
extends into diastole. Which of the following is the most likely cause of the
abnormalities present is this individual?
Aneurysmal dilation of the left ventricle
Obstruction of the aortic valve
Rupture of the left ventricle wall
@Rupture of a papillary muscle
Thrombosis of the left atrial cavity
#
40
A 62-year-old man has experienced substernal chest pain upon exertion with
increasing frequency over the past 6 months. An electrocardiogram shows features
consistent with ischemic heart disease. He has a total serum cholesterol of 262
mg/dL. By angiography, there is 75% narrowing of the left anterior descending
artery. Which of the following vascular complications is most likely to occur in
this patient?
A systemic artery embolus from thrombosis in a peripheral vein.
A systemic artery embolus from a left atrial mural thrombus.
Pulmonary embolism from a left ventricular mural thrombus.
@A systemic artery embolus from a left ventricular mural thrombus.
Pulmonary embolism from thrombosis in a peripheral vein.
#
41
A 56-year-old patient presented to the hospital with symptoms of acute myocardial
infarction. A diagnosis was confirmed by the EKG and laboratory tests. For 5th
days the condition acutely worsened. A progressive cardiac insufficiency resulted
in patient's death. A dissection confirmed the diagnosis of myocardial infarction,
complicated by the heart's wall rupture and tamponade of pericardium. What
process developed in the area of myocardial infarction?
@Aseptic autolysis
Organization
Encapsulation
Septic disintegration
Petrification
#
42
A 48-year-old man, with a history of transmural heart infarction of the left
ventricle's myocardium, died from the veritable rupture of heart (the heart
tamponade). What process in an infarct zone could promote a heart's wall rupture?
@Autolysis with melting of myocardium tissue (myomalacia)
Substitution of connective tissue in area of infarction (organization)
Rising of blood pressure in the small circulatory circle
Scar formation with thinning of wall of the left ventricle
#
43
Following development of an acute myocardial infarction (MI), the LEAST likely
complication is
Cardiac arrhythmia
Cardiogenic shock
Sudden cardiac death
@Cardiac rupture
Thromboembolism
#
44
A 49-year-old man seven days being admitted to the hospital for an inferior wall,
transmural myocardial infarction suddenly becomes short of death. Physical
examination reveal hypotension, elevated jugular venous pressure, and muffled
heart sounds. His systemic blood pressure drops 13 mmHg with inspiration. Which
one of the following pathologic processes produced these clinical findings?
Acute inflammation of the pericardium due to an autoimmune reaction
Acute mitral regurgitation due to rupture of a papillary muscle
Acute suppurative inflammation of the pericardium due to bacterial infection
@Blood accumulation in the pericardial cavity due to rupture of the ventricular
wall
Serous fluid accumulation in the pericardial cavity due to congestive heart failure
#
45
In 45-year-old patient died from sudden cardiac death the symmetrical type of
adipose heart of third degree; the rupture of right ventricle’s wall with
hemopericardium and redundant accumulation of fat under epicardium were found
out in autopsy. Microscopically: the adipose tissue grows from epicardium into
myocardium with atrophy of fibers of muscle. What process is more probable?
@Acute myocardial infarction
Ischemic heart disease
Fatty degeneration of myocardium
Adipose heart
Hypertensive disease
#
46
Arrange the following numbered statement in the correct order of the expected
sequence of events that normally occur during healing of a myocardial infarction.
1 = Collagen is deposited, forming a fibrous scar; 2 = Flocculent densities form
within mitochondria; 3 = Granulation tissue begins to form; 4 = Macrophages
begin to arrive at the area of coagulative necrosis; 5 = Neutrophils begin to arrive
at the area of coagulative necrosis.
2, then 3, then 4, then 5, then 1
2, then 4, then 5, then 3, then 1
@2, then 5, then 4, then 3, then 1
4, then 5, then 3, then 2, then 1
5, then 4, then 3, then 2, then 1
#
47
A 72-year-old man suffered a myocardial infarction involving half the left
ventricular free wall 3 months ago. He now has has increasing dyspnea and
orthopnea. On examination he has poor capillary filling in hands and feet. A chest
x-ray shows pulmonary edema. Which morphological changes most likely to be
found in the heart?
Cougulative necrosis
Colliquative necrosis
Diffuse inflammation
@Connective tissue scar
Hyperplasia of cardiomiocytes
#
48
A patient, with a history of myocardial infarction, develops symptoms of blood
circulation insufficiency after physical exercises. There is marked cyanosis and
edema of subcutaneous tissue of his lower extremities. What changes have
developed on a place of a myocardial infarction at the recovered person?
@Cardiosclerosis
Intracellular regeneration
Myocarditis
Atrophy of myocardium
-
#
49
An autopsy of a 58-year-old female revealed in myocardium a large, dense, grey
focus, which histologically consisted of the rough connective tissue fibers. It was
surrounded by the hypertrophied muscular fibers. What changes arose up in a
heart?
@Postinfarction cardiosclerosis
Ischemic stage of myocardial infarction
Necrotic stage of myocardial infarction
Diffuse cardiosclerosis
Myocarditis
#
50
A 52-year-old male, after repeated intramural heart infarction of myocardium,
gradually returned to health and further supervision of a district internist. In 2 years
he died in a motor-car accident. Define a pathological process in myocardium,
which wound be revealed at the autopsy?
@Focal cardiosclerosis
Diffuse cardiosclerosis.
Atrophy.
Necrosis.
Hyperplasia
#
51
A 64-year-old patient, with long history of atherosclerosis and myocardial
infarction, developed the attack of substernal pain. A patient was hospitalized in 3
days and died soon from progressive cardiovascular insufficiency. An autopsy
revealed in the back wall of the left ventricle and interventricular septum of heart a
white color focus, about 3 cm in a diameter. It was fibred, falling back, with a clear
boundary. Give the name for these changes:
@Focal cardiosclerosis
Myocardial ischemia
Myocardial infarction
Myocarditis
Dystrophy of myocardium
#
52
66-year-old man took medical aid because of the sharp pain in the heart with
irradiation to the left upper extremity and short-wind. The arterial tension – 100/50
mm of Hg, pulse – 90 per 1 min; electrocardiographically: the hypoxia of
myocardium of the front wall of the left ventricle was found. He has died in 2
hours after taking medical aid. In autopsy, the congestion (venous hyperemia) of
the internal organs was found. There was also dilatation of the heart’s ventricle. In
the fore wall of left ventricle the myocardium was parti-colored with yellowish dim
areas of 3,5 to 4 sm. Coronary arteries had atherosclerotic plaques and narrow
lumen. In aortic intima, many yellowish-whitish plaques were found out also. The
lungs had pasty consistence.
@Multifocal (postinfarctional) cardiosclerosis
Smallfocal (atherosclerotic) cardiosclerosis
Acute myocardial infarction
Fatty degeneration of myocardium
Myocarditis
#
53
Examination of coronary arteries revealed atherosclerotic calcific plaques that
close vessel lumen by 1/3. The muscle has multiple whitish layers of connective
tissue. What process was revealed in myocardium?
Myocardium infarction
@ Diffuse cardiosclerosis
Myocarditis
Tiger heart
Postinfarction cardiosclerosis
#
54
A 57-year-old man, with a long history of an alcohol abuse, died at the increasing
phenomena of chronic heart failure. An autopsy revealed the weight of a heart 580
grams; a languid, clay color myocardium, with intensive diffuse interstitial fibrosis.
Coronal arteries were intact. A microscopical study of myocardium showed a
combination of hydropic and fatty dystrophy of cells; atrophy and hypertrophy of
cardiomyocites. There were also some foci of a cells lysis, accompanied with
sclerosis. What kind of cardiomyopathy described in that case?
@Alcoholic
Hypertrophic
Dilatative
Restrictive
Metabolic
#
55
A 59-year-old patient receiving chemotherapy with the anthracycline Adriamycin
develops severe heart failure. Sections from an endocardial biopsy specimen reveal
vacuolization of the endoplasmic reticulum of the myocites. Adriamycin therapy
most frequently causes what type of cardiomyopathy?
@Dilated cardiomiopathy
Hyperplastic cardiomyopathy
Hyperthrophic cardiomyopathy
Obliterative cardiomyopathy
Restrictive cardiomyopathy
#
56
A 3-month-old girl is being evaluated for feeding difficulty and failure to thrive.
Physical examination finds pallor, peripheral cyanosis, tachypnea, and fine
expiratory wheezing. Chest x-ray shows cardiac enlargement. She is admitted to
the hospital, quickly develops severe cardiac failure, and dies 3 days after
admission. At the time of autopsy the endocardium is found to have a “cream
cheese” gross appearance. Histologic sections from this area reveal thickening of
the endocardium due to a proliferation of fibrous and elastic tissue. Which of the
following is the most likely diagnosis?
Dilated cardiomyopathy
Hypertrophic cardiomyopathy
Infective endocarditis
Libman-Sachs endocarditis
@Restrictive cardiomyopathy
#
57
A synonym for nonbacterial thrombotic endocarditis is
Atypical verrucous endocarditis
@Marantic endocarditis
Limban-Sacks ebdocarditis
Viridians endocarditis
Rheumatic endocarditis
#
58
A 23-year-old woman develops the sudden onset of congestive heart failure. Her
condition rapidly deteriorates and she dies in heart failure. At autopsy, patchy
interstitial infiltrates composed mainly of lymphocytes are found, some of which
surround individual myocytes. The most likely cause of this patient’s heart failure
is
@Viral myocarditis
Bacterial myocarditis
Giant cell myocarditis
Hypersensitivity myocarditis
Beriberi
#
59
A 23-year-old woman develops the sudden onset of congestive heart failure. Her
condition rapidly deteriorates and she dies in heart failure. At autopsy, patchy
interstitial infiltrates composed mainly of lymphocytes are found, some of which
surround individual myocytes. Which of the following is the most likely cause of
this patient’s heart failure?
Autoimmune reaction ( to group A β-hemolytic streptococci)
Bacterial myocarditis ( due to S. aureus infection )
Hypersensitivity myocarditis ( due to an allergic reaction )
Nutritional deficiency ( due to thiamine deficiency )
@Viral myocarditis ( due to coxsakievirus infection)
#
60
A patient died from progressive cardiac insufficiency. An autopsy revealed a
flaccid, dilated in diameter heart. A cut surface investigation showed the irregular
blood filling of a myocardium, resulted in the patchy pattern of its tissue. A
histological study determined a hyperemia of myocardium and stromal
accumulations of small mononuclear cells. The described morphological changes
present:
@Nonpurulent interstitial myocarditis
Vein plethora
Fatty dystrophy of myocardium
Cardiosclerosis
Myocardial infarction
#
61
An autopsy of a child, who died of a heart failure, revealed the dilated heart
chambers. Microscopic investigations showed the hyperemia of the myocardial
stroma, edema, and diffuse interstitial infiltrates consists of hystiocytes,
lymphocytes, some scattered neutrophils and eosinophils. What is the most likely
diagnosis?
@"Diffuse interstitial exudative myocarditis
Focal interstitial exudative myocarditis
Nodular productive myocarditis
Interstitial productive myocarditis
Alternative myocarditis
#
62
Autopsy of a man who died from influenza revealed that his heart was slightly
enlarged, pastous, myocardium was dull and had specks. Microscopical
examination of myocardium revealed signs of parenchymatous adipose and
hydropic dystrophy; stroma was edematic with poor macrophagal and lymphocytic
infiltration, vessels were plethoric; perivascular analysis revealed petechial
hemorrhages. What type of myocarditis was developed in this case?
Granulomatous
Purulent
Interstitial proliferative
@ Serous diffuse
Serous focal
#
63
Severe mitral stenosis is frequently accompanied by all the following EXCEPT
Aortic valve disease
Antecedent rheumatic fever
Left arterial enlargement with atrial fibrillation
@Pulmonary valvular stenosis
Chronic passive pulmonary congestion
#
64
Which one of the following is the most common congenital heart defect to cause an
initial left-to-right shunt?
Tetralogy of Fallor
Coarctation of the aorta
@Ventricular septal defect
Atrial septal defect
Paten ductus arteriosus
#
65
Bacterial myocarditis constitutes the greatest to patients who have which of the
following forms of congenital heart disease?
Atrial septal defect
@Ventricular septal defect
Pulmonic stenosis
Tetralogy of Fallot
Patent ductus arteriosus
#
66
Which one of the following statements correctly describes the flow of blood in an
individual with an atrial septal defect who develops Eisenmenger’s syndrome?
Aorta to pulmonary artery to lungs to left atrium to left ventricle to aorta
Left atrium to right atrium to right ventricle to lungs to left atrium
Left ventricle to right ventricle to lungs to left atrium to right ventricle
@Right atrium to left atrium to left ventricle to aorta to right atrium
Right ventricle to left ventricle to aorta to right atrium to right ventricle
#
67
A 2-year-old girl is being evaluated for growth and developmental delay. She has
had several past episodes when she would suddenly have trouble breathing,
become blue, and then assume a squatting position to catch her breath. Work-up
finds a defect in the wall of the ventricular septum, increased thickness of the right
ventricle, and dextroposition of the aorta. Which of the following cardiovascular
abnormalities is most likely to be present in this child?
Coarctation of the aorta
Incompetence of the mitral valve
Patency of the foramen ovale
Persistence of the AV canal
@Stenosis of the pulmonic valve
#
68
A 2-month-old girl is being examined for a routine check-up. She was born at term,
and there were no problems or complications during the pregnancy. The baby
appeared normal at birth and has been asymptomatic. Physical examination at this
time finds a soft systolic murmur with a systolic thrill. No cyanosis is present, and
her peripheral pulses are thought to be within normal limits. An electrocardiogram
reveals slight left ventricular hypertrophy. Which of the following is the most
likely diagnosis?
Coarctation of the aorta
Patient ductus arteriosus
Persistent truncus arteriosus
Tetralogy of Fallot
Ventricular septal defect
#
Konec:
#end
69
A 60-year-old patient was diagnosed with hypothalamic lateral nuclei stroke. What
changes in patient’s behavior may be expected?
Thirst
Aggressive behaviour
Unsatisfied hunger
@The rejection of food
Depression
#
Rheumatic diseases
#
1
What organism causes a rheumatic fever?
@Streptococcus
Staphylococcus
Gonococcus
Pneumococcus
Clebsiella
#
2
A 38 year old patient suffers from rheumatism in its active phas E. What laboratory
characteristic of blood serum is of diagnostic importance in case of this pathology?
@C-reactive protein
Creatinine
Urea
Transferrin
Uric acid
#
3
What from the following clinico-morphological features does not typical for
rheumatic fever?
@The presence of focus of chronic infection
Generalized vasculitis
Disturbance of immune homeostasis
Systemic progressive disorganization of connective tissue
Acute flow
#
4
Manifestations of rheumatic fever that are of major diagnostic value include all the
following EXCEPT
Sudcutaneous nodules
Migratory arthritis of large joints
@Fever
Erythema marginatum
Chorea minor
#
5
A 7-year-old boy presents with the acute onset of fever, pain in several joints, and
a skin rash. Physical examination finds an enlarged heart, several sudcutaneous
nodules, and a skin rash on his back with a raised, erythematous margin.
Laboratory tests find an elevated erythrocyte sedimentation rate and elevated
anti-streptolysin O titers. Within the past month, this boy most likely had which
one of the following infection?
Pseudomonas aeruginosa infection of the aorta
Streptococcus pneumonia infection of the lungs
α-hemolytic streptococci infection of the oral cavity
@ β-hemolytic streptococci infection of the pharynx
Staphylococcus aureus infection of the skin
#
6
A 63-year-old man presents with signs of congestive heart failure, including
shortness of death, cough, and paroxysmal nocturnal dyspnea. Physical
examination reveals a hyperdynamic, bounding, “water-hammer” pulse and a
decrescendo diastolic murmur. His hyperdynamic pulse causes “bobbing” of his
head. Which of the following is the most frequent cause of the cardiac vascular
abnormality present in this individual?
Aortic dissection
Infective endocarditis
Latent syphilis
Marfan syndrome
@Rheumatic fever
#
7
The most characteristic feature of chronic rheumatic heart disease is
Endocarditis
Myocarditis
Pericarditis
@Mitral valvulitis
Pulmonic valvulitis
#
8
Autopsy of a 58 y.o. man revealed that bicuspid valve was deformed, thickened
and unclosed. Microscopically: foci of collagen fibrilla are eosinophilic, react
positively to fibrin. The most probably it is:
Amyloidosis
Fibrinous inflammation
Hyalinosis
Mucoid swelling
@ Fibrinoid swelling
#
9
A 30-year-old woman was hospitalized with symptoms of active rheumatic fever.
She has died because of the increasing cardio-vascular insufficiency. The signs of
decompensative chronic cardiac insufficiency were found during autopsy. Small
foci of diffuse cardisclerosis were found in myocardium. The leafs of mitral valve
were thickened and sclerotic with presence of thrombotic masses looked like
warties. What type of endocarditis was
@Diffuse
Acute wart-like (verrucous)
Fibroplastic
Recurrent wartilike
Polipous-ulcerative
#
10
A histology investigation of a mitral valve sample of a patient who died from
complications of rheumatism revealed a mucoid swelling, a damage of the
endothelial cells and also thrombi at the valve closure line. Name the type of the
rheumatic endocarditis?
@Acute warty endocarditis
Diffuse endocarditis
Fibroplastic endocarditis
Relapsing warty endocarditis
Polyps-ulcerated endocarditis
#
11
At the histological exam of the mitral valve were revealed: the mucoid edema, the
damage of the endothelium and the formation of fibrin thrombi on the covering
margin. What form of the rheumatic endocarditis is observed?
@Acute verrucous endocarditis
Diffuse endocarditis
Fibroplastic endocarditis
Recurrent-verrucous endocarditis
Polypous-ulcerative endocarditis
#
12
Histological investigation of the mitral valves of the heart revealed the focal
desquamation of endothelial cells replaced by thrombi. The connective tissue had
mucoid swelling areas and also zones of sclerosis and revascularization. Name the
type of valve's endocarditis?
@Relapsing warty endocarditis
Diffuse endocarditis
Acute warty endocarditis
Fibroplastic endocarditis
Polyps-ulcerated endocarditis
#
13
A 56-year old male with a long history of a rheumatic heart insufficiency died with
symptoms of hemiplegia shortly before death. Histological examination of his
mitral; valve revealed severe sclerosis, nodular collection of inflammatory cells
and vegetations composed mainly of platelets and fibrin. Name the most likely
type of endocarditis?
@Relapsing warty endocarditis
Acute warty endocarditis
Diffuse endocarditis
Fibroplastic endocarditis
Polyps-ulcerated endocarditis
#
14
Physical examination of an asymptomatic 29-year-old woman with a history of
rheumatic fever during childhood finds an early diastolic opening snap with a
rumbling late diastolic murmur. Which of the following is the most likely
diagnosis?
Aortic regurgitation
Aortic stenosis
Mitral regurgitation
@Mitral stenosis
Pulmonic stenosis
#
15
A 32-year-old female with a long history of rheumatic valve's defect presented at
the hospital with tachypnea and dyspnea, leg's edema, ascites and hepatomegaly.
She died from the chronic heart insufficiency. An autopsy revealed a mitral
stenosis. What was the most likely factor of the mitral stenosis morphogenesis?
@Leaves union between itself
Sclerosis and bulge of leaves
Sclerosis and shortening of leaves
Shortening of tendon filaments
Presence of small blood thromboses on the surface of valve
#
16
In 46-year-old patient with rheumatic heart disease (mitral stenosis) the short-wind
in low physical loading, tachycardia, cyanosis of lips, wet crepitation in the lower
parts of lung and edema of legs have appeared. What histological changes are
characteristic in the liver?
@Necrosis of hepatocytes in the center of lobule; fatty degeneration in outlying
districts
Necrosis of hepatocytes in the center of lobule; hyaline degeneration in the
outlying districts
Fatty degeneration of hepatocytes in the center of lobule; necrosis in the outlying
districts
Hydropic degeneration of hepatocytes in the center of lobule, necrosis in the
outlying districts
Necrosis of hepatocytes in the center of lobule; hydropic degeneration in the
outlying districts
#
17
The most frequent cause of aortic valve incompetence and regurgitation is
Latent syphilis
Infective endocarditis
@Rheumatic fever
Aortic dissection
Congenital
#
18
During the section of a child, who died of heart failure, the dilatated ventricular
cavities of the heart were revealed. Microscopically there were plethora, edema,
and diffuse infiltrates of histiocytes, lymphocytes, neutrophiles and eosinophiles in
the myocardial stroma. What is the most possible diagnosis?
Focal intermediate exudative myocarditis
@Diffuse intermediate exudative myocarditis
Nodular productive myocarditis
Intermediate productive myocarditis
Alternative myocarditis
#
19
Death of the patients in early stages of rheumatic fever because of acute cardiac
insufficiency is connected with one of the following manifestations of the
rheumatic fever:
@Myocarditis
Pericarditis
Endocarditis
Septic endocarditis
Cardiosclerosis
#
20
At the pathomorphologic exam of a man who died of heart insufficiency the
following facts were revealed: mitral valves were deformed, thickened, with
knitted margins; connective tissue is filled with diffuse nodules that consist of the
areas of fibrinoid necrosis. Macrophagocytes that look like giant multinuclear cells
aggregate around these areas. Lymphocytes and single plasmatic cells surround
similar focuses. What granuloma was defined?
Actinomycotic
Tuberculous
@Rheumatic
Syphilitic
Lepromatous
#
21
At the section of a dead woman were detected the morphological manifestations of
stenosis of the left atrioventricular aperture and the mitral valve insufficiency.
Histologically: there were presented a focal cardiosclerosis and Ashof-Talalaev
granulomas. What is the most possible diagnosis?
Systemic lupus erythematous
Scleroderma
Dermatomyositis
Nodular periarteritis
@Rheumatism
#
22
Microscopic investigation of the heart auricle from a patient with a history of
mitral stenosis revealed the Achoff- Talalayev's bodies (granulomas). What was
the most likely cause of the heart insufficiency supported by the histology results?
@Rheumatic
Atherosclerotic
Syphilitic
Innate
Septic
#
23
An autopsy of 48-year-old female revealed a mitral stenosis with valve's
incompetence. Histological investigation showed the post- inflammatory
cardiosclerosis and Aschoff-Talalayev nodes (granulomas). What is the most likely
diagnosis?
@Rheumatism
Systemic scleroderma
Dermatomyositis
Nodular periarteritis
Lupus erythematosus
#
24
Ashoff-Talalaev’s granulomas, which are foci of fibrinoid necrosis that are
surrounded by macrophages, are found out in perivascular tissue of myocardium in
the result of autopsy of 30 year woman who had the diagnosis: rheumatic fever and
who died of heart insufficiency. What type of granuloma is found?
@Blooming granuloma
Withered granuloma
Cicatrized granuloma
Epitheliod granuloma
Giant cell granuloma
#
25
Ashoff-Talalaev and fibroblasts granulomas, which are focuses of fibrinoid
necrosis that are surrounded by macrophages, are found in perivascular tissue of
myocardium in the result of autopsy of 30 year woman who had the diagnosis:
rheumatic fever and who died of heart insufficiency. What type of granuloma is
found?
Blooming granuloma
@Withered granuloma
Cicatrized granuloma
Epitheliod granuloma
Giant cell granuloma
#
26
An autopsy of a 34year-old patient with a long history of rheumatism, revealed the
epicardial surface of the heart with shaggy exudate formed by grey strands which
easily separate from underlying tissues and described as 'bread-and-butter'
pericarditis ('hairy heart'). What is the most likely diagnosis?
@Fibrinous pericarditis
Purulent pericarditis
Hemorrhagic pericarditis
Proliferated pericarditis
Catarrhal pericarditis
#
27
A 9-year-old boy presented with painless, firm 1-2 mm nodules at the skin around
ulnar and knee joints (at extensor's area). Biopsy investigation revealed a central
area of fibrinoid necrosis of the connective tissue surrounded by lymphocytes and
macrophages. What disease these nodules are characteristic for?
@Rheumatism.
Rheumatoid arthritis.
Systemic scleroderma.
Nodular periarteritis.
Lupus erythematosus.
#
28
In a patient with vasculitis, the finding of serum antineutrophil cytoplasmic
autoantibodies that react by immunofluorescence staining in a perinuclear pattern
is most suggestive of
Giant cell arteritis
@Classic polyarteritis nodosa
Wegener’s granulomatosis
Churg-Strauss syndrome
Microscopic polyangiitis
#
29
The necrotizing inflammation of the small gastrointestinal artery shown in the
photomicrograph below is most likely due to
Myasthenia gravis
@Polyarteritis nodosa
Atherosclerosis
Dissection aneurysm
Suphilis
#
30
Microscopical investigation of arterioles showed plasmatic saturation, mucoid and
fibrinoid swelling, fibrinoid necrosis of walls, diffuse infiltration by lymphocytes,
plasmocytes and monocytes. In addition, the focal proliferation of hystiocytes,
endotheliocytes and pericytes were determined. The final diagnosis was the
"periarteritis nodosa". What type of inflammation took place in arterioles?
@ Acute immune inflammation
Acute not immune inflammation
Focal exudative inflammation
Diffusive exudative inflammation
Fibrinous inflammation
#
31
A 33-year old female died from chronic kidney failure. A post-mortem revealed
multiple scars and infarcts in kidneys and spleen. Histological investigation
showed alteration of small and medium sized arteries presented with sclerosis and
mild endothelial proliferation. A severe lymphocytes and histiocytes infiltrates
were also recognized at the perivascular tissues. What is the most likely disease
caused these alterations?
@Nodular periarteritis
Atherosclerosis
Hypertonic disease
Morphine's Disease
Visceral syphilis
#
32
A 45-year-old woman has episodes of pain with swelling and warmth of her hands
and feet that make it difficult for her to walk or to prepare meals. She develops
painful subcutaneous nodules on the extensor surfaces other elbows. Over the
years, her hands become deformed so that it is difficult to perform tasks as simple
as opening a door o r buttoning her blouse. Which of the following laboratory test
findings is she most likely to have?
HLA B27
ANA positive at 1:256
@Elevated rheumatoid factor
Markedly decreased serum complement
Hypogammaglobulinemia
#
33
A 30-year-old woman is suffering from chronic inflammation of the hand joints. If
the doctor suggests rheumatoid arthritis, what will be the most probable
pathogenesis of the disease?
@It is caused by immune complexes of IgM antibody against human IgG
It is caused by release of histamine by mast cells, bound with IgE
It is caused by CD4 T cells and macrophages invading the joints
It is caused by the activity of superantigens that activate both humoral and cellular
immune responses
It is caused by the primary immune response to hepatitis B surface antigen
#
34
A 30 y.o. woman had been ill for a year when she felt pain in the area of joints for
the first time, they got swollen and skin above them became reddened. Provisional
diagnosis is rheumatoid arthritis. One of the most probable causes of this disease is
a structure alteration of a connective tissue protein:
Troponin
Ovoalbumin
Mucin
Myosin
@ Collagen
#
35
A 63-year-old woman developed symptoms of rheumatoid arthritis. Their increase
of which blood values indicators could be the most significant in proving the
diagnosis?
R-glycosidase
Acid phosphatase
Lipoproteids
General cholesterol
@Additive glycosaminoglycans
#
36
After the undercooling of the patient, the deformation of the joints, painfulness,
contraction of movements in the fingers of limbs has developed. Little solid
nodules appeared around the joints. At the biopsy there were revealed the focuses
of fibrinoid necrosis in the nodules that were surrounded by histiocytes. What is
your diagnosis?
@Rheumatoid arthritis
Dermatomyositis
Rheumatism
Podagra
Deforming arthrosis
#
37
A 48-year-old female after exposure to cold presents to her physician with painful
deformed fingers joints, which bones were restricted in their movements. Physical
examination revealed small firm nodules near the joints. Histological investigation
of the nodules biopsy showed the centrally located core of fibrinoid necrosis with
surrounding rim of macrophages and hystiocytes. What is the most likely
diagnosis?
@Rheumatoid arthritis.
Dermatomyositis.
Rheumatism.
Gout.
Deformed arthrosis.
#
38
A 62-year-old woman presents to her physician with considerable deformation of
metacarpal phalangeal and feet joints. Histological examination of the soft tissues
adjacent to the joints revealed a mucoid swelling of the connective tissue, areas of
a fibrinoid necrosis surrounded by palisading epithelioid macrophages and
sclerosis. Few 'rice bodies' were found within a synovial cavity. What is the most
likely diagnosis?
@Rheumatoid arthritis
Rheumatism
Behterev's Disease
Hematogenic tuberculosis
Gout
#
39
A 43-year-old woman presents to her physician with pain and immobility of the
bones in metacarpal phalangeal and feet joints. Similar complains in symmetric
pattern were about ulnar and knee joints, though not so severe. Physical
examination revealed pastous skin over the joints, a partial ankylosis of metacarpal
phalangeal and feet joints, and also ulnar deviation of the hands and flexionhypertension
('swan neck"or"walrus flipper") deformities of the fingers. In
addition, movable firm, rubbery and tender 1cm hypodermic nodules were found in
the phalangeal joints area. An aspirate of joint fluid showed increased turbidity and
presence of white "grains" ("rice bodies"). Immunofluorescence also revealed the
rhematoid factor. What is the most likely diagnosis?
@Rheumatoid poliartritis
Systemic disease of connective tissue
Gout
Osteoartrosis
-
#
40
A 44-year-old woman presents to her physician ulnar deviation of the hands and
flexion- hypertension ('swan neck" or "walrus flipper") deformities of the fingers.
Her metacarpal phalangeal joints are easily exposed to a dislocation and a
subluxation. Microscopical examination revealed nodular proliferations of
synovium, cartilage destruction and "pannus" formation. What is the most likely
diagnosis?
@Rheumatoid arthritis
Rheumatic arthritis
Osteoarthritis.
Lupus erythematosus.
-
#
41
A 36-year-old woman present because of increasing pain in her hands and knees,
which, she says, is worse in the morning. Physical examination finds her fingers to
be swollen and stiff, and there is ulnar deviation of her metacarpophalangeal joints.
A biopsy from her knee would likely show areas where hystiocytes were palisading
around irregular areas of necrosis, as seen in the picture below. The biopsy would
also likely show proliferation and hyperplasia of the synovium with destruction of
the articular cartilage. Which one of the following terms best describes these
pathologic changes?
Eburnation
Gumma
@Pannus
Spondylosis
Tophus
#
42
An autopsy of 61-year-old male with a history of rheumatoid arthritis revealed
enlarged dense kidneys, which had yellowish-whitish color and waxy appearance.
Grossly, foci of scars were recognized at kidneys surface. Microscopically, at the
slides stained by Congo red, homogeneous pink masses at capillaries of glomeruli
tufts, arterioles walls and arteries, basal membranes of tubuli and in s troma were
found. Name the described complication of rheumatoid arthritis?
@Secondary amyloidosis of kidneys.
Postinfective glomerulonephritis.
Quickly progressive glomerulonephritis.
Acute necrotic nephrosis.
Fibroplastic glomerulonephritis.
#
43
True statsments about SLE include all of the following except one:
Lupus is associated with autoantibodies to many nuclear components
Supressor T cells dysfunction is believed to plat a role in pathogenesis
An association with complement component dificiency (C2 and C4) has been
shown
@Organ damage in lupus is predominantly mediated by natural killer cells
-
#
44
A 38-year-old man has myalgias, erythematous skin rashes over his face, and
arthralgias without evidence for joint deformities. He has a positive antinuclear
antibody test with a speckled pattern. Additionally, he has high liters of
autoantibodies to ribonucleoprotein (KNP antibodies). A serum creatine kinase is
661 U/L. He has no evidence of renal disease. Which of the following conditions is
he most likely to have:
Mixed connective tissue disease
CREST syndrome
Dennatomyositis
Polymyositis
@Systemic lupus erythematosus
#
45
A post-mortem of a 19-year old female revealed multiple furuncles on her skin, a
warty endocarditis of the heart valves (Libman-Sacks endocarditis), a focal
hemorrhage under the endocardium, ulcerative stomatitis, esophagus ulcers,
pneumonia, nephritis, and a spleen hyperplasia with a perivascular sclerosis.
Morphological investigation of her brain showed areas of necrosis and signs of
vasculitis within thalamus. What is the most likely diagnosis?
@Lupus erythematosus
Rheumatism.
Septic endocarditis.
Ischemic heart diseas
Hypertensive disease.
#
46
At the section of a 19-year-old woman, there were revealed the verrucous
endocarditis of all valves, focal subendocardial hemorrhages, ulcerous stomatitis
and esophagitis, pneumonia, nephritis, hyperplasia of the spleen with perivascular
sclerosis, vasculitis, a necrosis in the area of the optic tuber and numerous
furuncles. What is the most possible disease?
Septic endocarditis
Rheumatism
@Systemic lupus erythematous
Ischemic heart disease
Hypertensive disease
#
47
A post-mortem of a 25-year old woman who died from chronic kidney failure
revealed a reddish malar rash ('butterfly rash') and small (up to 0,2 cm) pale tan
spreading vegetations over the mitral valve surface. Histological investigation of
kidneys showed foci of fibrinoid necrosis, eosinophilic deposits (hematoxylin
bodies), "wire loop" lesions in the basement membrane of the glomerular tuft and
karyorrhexis. What is the most likely diagnosis?
@Lupus erythematosus.
Nodular periarteritis.
Rheumatism.
Rheumatic arthritis.
-
#
48
Red-brownish stains are found on the symmetrical parts of both cheeks in the result
of autopsy of 45 year dead woman. Heart is increased, leaflets of aortic valve are
condensed and thickened, have thrombotic masses on its surface. Kidneys are
increased. Medulla is duck red, cortex is gray-brownish with red half-transparent
points. Histologically: “hemothoxilin bodies” are found in the nucleus of ductal
epithelium; thickening of basal membranes of glomerular capillaries, which look
like “wire-loops”, several places in capillaries have hyaline thrombus and focuses
of fibrinoid necrosis. Which diagnosis is more possible?
@Systemic lupus erythematosus
Rheumatic heart disease
Septic endocarditis
Glomerulonephritis
Atherosclerosis
#
49
A post-mortem of 40-year old female who died from uremia revealed enlarged
kidneys, which had a patchy pattern of their surface. Histological investigation of
kidneys showed eosinophilic deposits (hematoxylin bodies), "wire loop" lesions in
the basement membrane of the glomerular tuft, hyaline thrombi and foci of
fibrinoid necrosis. Besides these, Libman-Sacks endorcarditis was also determined.
What is the most likely pathology in kidneys?
@Lupus erythematosus nephritis.
Rheumatoid glomerulonephritis.
Choleric glomerulonephritis.
Sclerotic kidney.
Terminal glomerulonephritis
#
50
At the section of a 40-year-old woman, who died of uremia, there were detected
enlarged, mottled kidneys with thickened capillary membranes of glomeruli that
looked like wire stitches, focuses of fibrinoid necrosis of its walls and hyaline
thrombi in the lumens. Nuclei had haematoxylin corpuscles. The heart was affected
by Libman-Sacks endocarditis. What is the most possible renal affection?
Sclerotic kidney
Rheumatic glomerulonephritis
Choleric glomerulonephritis
@Lupoid nephritis
Terminal glomerulonephritis
#
51
At the section of a 25-year-old woman, who died of uremia, were detected
enlarged, mottled kidneys with the focuses of haemorrhages. Pathohistologically
there were revealed haematoxylin corpuscles, capillary membranes of glomeruli
that looked like wire stitches, hyaline thrombi, focuses of fibrinoid necrosis and
bulbous sclerosis in the renal vessels. What is the most possible diagnosis?
Nodular periarteritis
Systemic scleroderma
Rheumatoid arthritis
Rheumatic arthritis
@Systemic lupus erythematous
#
52
A 28-year old female died from uremia. A post-mortem revealed an enlarged
kidneys, which had a patchy pattern with hemorrhages on there surface.
Histological investigation showed eosinophilic deposits (hematoxilin bodies),
"wire loop" lesions in the basement membrane of the glomerular tuft, hyaline
thrombi and foci of fibrinoid necrosis and also 'onion skin' sclerosis at the spleen's
vessels. What is the most likely diagnosis?
@Lupus erythematosus.
Rheumatism.
Systemic scleroderma.
Rheumatoid arthritis.
Nodular periarteritis.
#
53
At the patient with suspicion on a systemic disease a biopsy from a site of the skin
tightening and restricted motility was taken. A histology investigation revealed all
kinds of disorganization of connective tissue fibers with mild cellular reaction and
also transition in excessive sclerosis and a hyalinosis. What is the most likely
diagnosis?
@Scleroderma
Nodular periarteritis
Lupus erythematosus
Psoriasis
Dermatomyositis
#
54
The woman of 45 years within several years has difficulties at swallowing. She
also notes the limitation of fingers movements at printing on the computer
keyboard, though joints are not painful. Physical investigation revealed her "stony
face" (no wrinkles) owing to tightening of the facial skin and restricted motion of
the mouth. The skin biopsy showed a widespread fibrosis of a derma without
inflammatory infiltration. Diagnose disease on the listed clinical and
morphological data.
@Scleroderma
Lupus erythematosus
Dermatomyositis
Amyloidosis
Rheumatic arthritis
#
55
An 8-year-old boy presents with weakness and pain over several of his proximal
muscle groups. Physical examination reveals periorbital edema along with a lilac
discoloration around his eyes and erythema over his knuckles. A muscle biopsy
reveals atrophic fibers located primarily at the periphery of muscle fiber fascicles.
Laboratory tests find the presence of antibodies directed against the
microvasculature of skeletal muscle. Which of the following is the most likely
diagnosis?
Ataxia-telangiectasia
Becker muscular dystrophy
Charcot-Marie-Tooth disease
@Dermatomyositis
McCardle’s disease
#
56
A 35- year- old female presented her physician intermittent episodes of ischemia of
her fingers, marked by pallor, paresthesias and pain, accompanied by tightening
and thickening of the skin and poliarthralgia. Histological investigation of the skin
biopsy from affected areas revealed mild epidermal atrophy, hyalinosis of collagen
fibers within derma, scattered perivascular lymphocytes' infiltrates. Underlying
skeletal muscles expressed interstitial edema, loss of cross-section striation, nidal
necroses followed with petrification. What is the most likely diagnosis?
@Dermatomyositis
Systemic scleroderma
Lupus erythematosus
Nodular periarteritis
Rheumatism
#
57
A perivascular inflammatory infiltrate in skeletal muscle is likely to be seen in all
the following EXCEPT
Hypersensitivity angiitis
Polymiositis
Polyarteritis nodosa
@Cystic medial necrosis
Systemic sclerosis
#
58
A post-mortem of the patient who died from uremia revealed deformation of a
spine column with severe restriction of its mobility. Articular cartilages of spine
joints were destructed with persistent chronic inflammation in tissues of joints. The
joints cavities filled with connective tissue, in some places leading to ossification
and ankylosis formation. In an aorta, heart, lungs a chronic inflammation and a
focal sclerosis were discovered. In kidneys an amyloidosis was recognized. What
diagnosis in this case is most probable?
@Ankiloid spondiloartritis (the Behterev's disease)
The Pedget's disease (deforming ostosis)
Rheumatoid arthritis.
Parathyroid osteodystrophy.
Osteopetrosis (marble disease).
#
Konec:
#end
Acute diseases of Respiratory system
#
1
Part of alveoles of a preterm infant didn't spread because of enhanced elastic recoil
of lungs. How can this recoil be reduced?
By artificial pulmonary ventilation
By glycose introduction
By pure oxygene inhalation
@By surfactant introduction
By fluid suction from the respiratory tracts
#
2
Lung of premature infant biopsy material presented. Collapse of the alveolar wall
caused by the deficiency of surfactant was revealed. Which cell type dysfunction
of the alveolar wall has been caused it?
@Pneumocyte type 2
Fibroblast
Pneumocyte type 1
Macrophage
Adipose cell
#
3
A lung of a premature infant biopsy material presented. Collapse of the alveolar
wall caused by the deficiency of surfactant was revealed. Disfunction of what cells
of the alveolar wall caused it?
Secretory cells
Alveolar macrophages
@Alveocytes type II
Alveocytes type I
Fibroblasts
#
4
After breathing with poisonous steams there is an increased quantity of slime in
respiratory passages of a chemical production worker. What of respiratory tract
epithelial cells participate in mucousa moistening?
@Goblet cells
Endocrine cells
Fibroblasts
Intercalated cells
Langergans cells
#
5
Decreased blood supply to the organs causes hypoxia that activates fibroblasts
function. Volume of what elements is increased in this case?
@Intercellular substance
Parenchymatous elements of the organ
Vessels of microcircular stream
Lymphatic vessels
Nerve elements
#
6
A patient after pathological process has a thickened alveolar membrane. The direct
consequence of the process will be the reduction of:
Reserve expiratiory capacity
Oxygen capacity of blood
Minute respiratory capacity
Alveolar lung ventilation
@Diffuse lung capacity
#
7
A woman with myasthenia developed respiratory disorders, which required
artificial ventilation. What type pf respiratory insufficiency did the woman have?
Centrogenic
Restrictive
@Neuromuscular
Thoracophrenic
Obstructive
#
8
While recovering in bed 1 week after an abdominal hysterectomy, a 42-year-old
woman develops acute shortness of breath with hemoptysis. Physical examination
finds the patient to be afebrile with moderate respiratory distress, calf tenderness,
and a widely split S 2 . Which of the following is the most likely diagnosis?
Atelectasis
Bacterisl pneumonia
@Pulmonary embolus
Pulmonary hypertension
Viral pneumonia
#
9
A 52-year old female, with a history of the chronic glomerulonephritis and chronic
renal failure, presented to the hospital with coughing spells accompanied by thick
phlegm and breathlessness. Bronchoscopy revealed congested, edematous,
bronchial mucus membrane with small hemorrhages. A bronchial lumen was
narrowed by thick mucus. Name the process in bronchi?
@Secondary acute catarrhal bronchitis
Primary acute catarrhal bronchitis
Chronic catarrhal bronchitis
Destructive - ulcerous bronchitis
Catarrhal - purulent bronchitis
#
10
A 19-year-old woman presents with sudden, severe right-sided chest pain that
developed shortly after she had been placing heavy boxes on shelves in her garage.
Physical examination reveals an afebrile woman in mild respiratory distress.
Breath sounds are markedly decreased on the right, and the right lung is
hyperresonant to percussion. Which of the following is most likely present in thia
individual?
Pneumoconiosis
Pneumocytic infection
Bacterial pneumonia
Viral pneumonia
@Pneumothorax
#
11
A 25-year-old woman presents with a 6-month history of increasing fatigue and
dyspnea. Physical examination finds that she is in moderate respiratory distress and
cyanosis is present. An echocardiogram of her heart finds the thickess of the right
ventricle to be increased, but the thickness of the left ventricle is within normal
limits. Histologic section from a lung biopsy reveal plexiform lesion within the
pulmonary arterioles. Hyaline membranes are not found nor are areas of lung
collapse present. Which of the following is the most likely diagnosis?
Adult respiratory distress syndrome
Churg-Strauss syndrome
Lymphomatoid granulomatosis
@Pulmonary hypertension
Wegener’s granulomatosis
#
12
All the following conditions can produce a histopathologic condition in the lung of
adult that is similar to the histopathology of respiratory distress syndrome of the
newborn (hyalinemembrane disease) EXCEPT
Viral pneumonia
Uremia
Pulmonary irradiation
Severe bacterial infection
@Tuberculosis
#
13
In course of an operation surgeon removed a part of a lung that was ventilated by a
tertiary bronchus accompanied by branches of pulmonary artery and other vessels.
What part of a lung was removed?
Pulmonary lobule
Middle lobe
@ Bronchopulmonary segment
Superior lobe
Inferior lobe
#
14
An 8 year-old child has died because of pulmonary damage. In autopsy the
pathologist has found out in the both lungs: edema, small foci of emphysema,
atelectasis, hemorrhages, inflammation without pus. The most probable disease
is…
@Virus pneumonia
Croupous pneumonia
Streptococcal pneumonia
Clebsiella’s pneumonia
Staphylococcal pneumonia
#
15
During the section of a man, who had the cancer of the stomach and died of cancer
intoxication, there were detected solid, gray-red, distorted focuses that elevated
above the incision surface in the posterior-inferior part of lungs. Microscopically,
the exudate with numerous neutrophiles was presented in the lumen and walls of
small bronchi and alveoli. Name the disease?
Intermediate pneumonia
Acute bronchitis
Croupous pneumonia
@Acute suppurative bronchopneumonia
Acute serous bronchopneumonia
#
16
A male patient with a medical history of the malignant neoplasm died from the
cancer intoxication. A post-mortem revealed the posterior segments of the inferior
pulmonary lobes firm clusters of solid, slightly elevated nodules, poorly delimited
at their margins, which had dark red and grey appearance. Histological
investigation showed the exudate within alveoli and small bronchi rich with
neutrophil polymorphs. What is the most likely diagnosis?
@Acute purulent bronchopneumonia
Acute bronchitis
Croupous pneumonia
Intestinal pneumonia
Acute serous bronchopneumonia
#
17
On autopsy it is revealed enlarged dense right lung, fibrin layers on the pleura.
Lung tissue is light green color on incision with muddy liqued exudates. What lung
disease are these symptoms typical for?
Interstitial pneumonia
Pulmonary gangreneі
Bronchopneumonia,
@Lung-fever
Fibrosing alveolitis
#
18
A 72-year old female died of severe grippe. A post-mortem revealed "the big
motley lungs". Histological investigation showed acute hyperemia, hemorrhages,
an edema of a pulmonary tissue, erythrocyte-rich exudate that filled the bronchi
and alveoli. What is the most likely kind of pneumonia?
@Hemorrhagic bronchopneumonia
Catarrhal bronchopneumonia
Purulent bronchopneumonia
Desquamative bronchopneumonia
Fibrinous bronchopneumonia
#
19
An autopsy of a patient who died from pneumonia revealed the inferior lobe of the
right lung firm, hypoventilated, with the fibrin fibers on the pleura. What is the
most likely etiology of the disease?
@Pneumococcal
Staphylococcal
Streptococcal
Mycoplasmal
Virus
#
20
A 46-year old female died of croupous pneumonia. A post-mortem revealed turbid
pleural effusions and visceral pleura membranous exudate formed by grey strands.
What is the most likely kind of inflammation on pleura?
@Fibrinous
Catarrhal
Purulent
Granulomatous
Hemorrhagic
#
21
A 37-year-old woman presents with the acute onset of a productive cough, fever,
chills, and pleuritic chest pain. A chest x-ray reveals consolidation of the entire
lower lobe of her right lung. She unexpectedly dies before treatment due to a
cardiac arrhythmia. Histologic examination of lung tissue taken at the time of
autopsy reveals multiple suppurative, neutrophil-rich exudates filling the bronchi,
bronchioles, and alveolar spaces. The majority of lung tissue from her ringht lower
lung is involved in this inflammatory process. Hyaline membranes are not found.
Which of the following is the most likely diagnosis?
Bronchiectasis
Bronchopneumonia
Interstinal pneumonitis
@Lobar pneumonia
Pulmonary abscess
#
22
A post-mortem revealed the left lung enlarged, firm, with fibrin fibers on the
pleura. On a cut section pulmonary tissue had a gray coloring. A turbid liquid
flowed down the surface. What is the most likely diagnosis?
@"Croupous pneumonia
Focal pneumonia
Intestinal pneumonia
Cancer of lungs
Bronchiectatic disease
#
23
In autopsy the whole lower lobe of left lung had dirty-grey colour and liver-like
consistence.There was yellow-gray fibrin on a pleura. An etiology of disease is
pneumococcus type 1Y. Call this disease:
@Croupous pneumonia
Clottage of a bronchial arteries
Tuberculosis
Syphilis
Clottage of pulmonary arteries
#
24
The upper lobe of the right lung is enlarged, grey and airless, the inscision surface
is dripping with turbid liquid, the pleura has many fibrinogenous films;
microscopical examination of alveoles revealed exudate containing neutrophils,
desquamated alveolocytes and fibrin fibers. The bronchus wall is intact. What is
the most probable diagnosis?
Interstitial pneumonia
@Croupous pneumonia
Pulmonary abscess
Focal pneumonia
Influenzal pneumonia
#
25
A patient died in 3 weeks of the pneumonia onset. Macroscopic investigation of
lungs revealed the whole of a left inferior lobe became enlarged, consolidated and
airless. Grossly lungs got the appearance of a grayish brown, dry surface with
fibrin fibers on the pleura. Histological investigation showed the collapsed and
bloodless alveolar capillaries, the exudate within alveoli consisted mainly of
neutrophil polymorphs. What is the most likely diagnosis?
@Croupous pneumonia
Focal bronchopneumonia
.Influenzal pneumonia гриппозная
Fibrinous pleuritis
Intestinal pneumonia
#
26
A 42-year old woman, with a history of common cold after acute cooling,
presented to his physician with symptoms of cough, chest pain, fever, and
difficulty in breathing. She died on the 5t h day staying at the hospital.A
post-mortem revealed the inferior lobe the right lung enlarged, firm, with
membranous fibrin fibers on the pleura. On a cut section pulmonary tissue had a
gray coloring and looked granular. What is the most likely diagnosis?
@"Croupous pneumonia
Influenzal pneumonia
Bronchopneumonia
Caseous pneumonia
Measles pneumonia
#
27
A 42-year old man presented to the hospital with symptoms of malaise,
complicated with a collapse one hour later. He died soon, not coming to
consciousness. A post-mortem revealed the lungs enlarged, congested, cut with a
sound of crunch. Whole of a right inferior lobe had a liver-like texture with some
fibrin fibers superposed on the pleura. On a cut section pulmonary tissue exhibited
a gray- brown coloring. What is the most likely diagnosis?
@Croupous pneumonia
Bronchopneumonia
Acute venous hyperemia in lungs
Toxic shock syndrome
Acute myocardial infarction
#
28
A 45-year-old man fell ill with pneumonia. On the 6 t h day of the disease a
pulmonary oedema developed that caused the death. At the section: the upper part
of the right lung is totally affected – enlarged, solid with fibrous strata on the
pleura. It is of gray color on the incision. A cloudy fluid flows down from the
surface of the incision. Microscopically: in the alveolar lumen there are fibrin,
neutrophiles, macrophages, haemolyzed erythrocytes. What type of the pneumonia
had the patient?
@Croupous pneumonia
Staphylococcal bronchopneumonia
Viral pneumonia
Hypostatic pneumonia
Interstitial pneumonia
#
29
An autopsy of the 47-year-old man revealed the right lung firm with dry
hypoinflated granular pulmonary tissue. Visceral pleura presented with
membranous gray-brown fibrin fibers on the pleura. What is the most likely
diagnosis? What is the most likely diagnosis?
@"Croupous pneumonia
Tuberculosis
Bronchopneumonia
Intestinal pneumonia
Pneumofibrosis
#
30
A 48-year old male, with a history of common cold after acute cooling, presented
to his physician with symptoms of lung a heart insufficiency. He died soon at the
hospital. A post-mortem revealed the right lung enlarged, firm, hypoventilated,
with a liver-like consistency and fibrin fibers on the pleura. On a cut section
pulmonary tissue had a gray coloring and looked granular. A turbid liquid flowed
down the surface. Histological investigation showed an acute inflammation with
the exudate within alveoli consisted mainly of neutrophil polymorphs. What is the
most likely diagnosis?
@Croupous pneumonia
Focal pneumonia
Intestinal interalveolar pneumonia
Staphylococcal pneumonia
Idiopathic fibrosing alveolitis
#
31
A 52-year old male presented to hospital with symptoms of acute pneumonia,
complicated on the 6t h day by pulmonary edema. The latter one resulted in the
patient's death. A post-mortem revealed the upper part of the right lung enlarged,
firm, with fibrin fibers on the pleura. On a cut section pulmonary tissue had a gray
coloring and looked granular. A turbid liquid flowed down the surface.
Histological investigation showed the exudate within alveoli consisted of fibrin,
neutrophil polymorphs, macrophages, and disintegrated red cells. What is the most
likely diagnosis?
@Croupous pneumonia
Staphylococcal bronchopneumonia
Virus pneumonia
Hypostatic pneumonia
Adult respiratory distress syndrome
#
32
At the section of a 58-year-old man that had alcohol abuse for a long time and died
at home were revealed the following facts: macroscopically – the right lung is
enlarged in size, its tissue is of grayish color, smooth, pleura is covered by grayish
membranous strata; microscopically – alveolar lumens contain fibres of fibrin,
haemolyzed erythrocytes. Diagnose the disease.
@Croupous pneumonia
Focal pneumonia
Interstitial pneumonia
Primary tuberculosis of the lungs
Caseous pneumonia
#
33
An autopsy of the 60-year-old man with a long history of over consumption of
alcohol revealed the right lung enlarged, firm, with fibrin fibers on the pleura. On a
cut section pulmonary tissue had a gray coloring. Histological investigation
showed the exudate within alveoli consisted of fibrin and disintegrated
erythrocytes. What is the most likely diagnosis?
@Croupous pneumonia
Focal pneumonia
Intestinal pneumonia
Primary tuberculosis of lung
Caseous pneumonia
#
34
A 50 year-old patient has died because of pulmonary- cardiac insufficiency. In
autopsy the pathologist has found out: the upper lobe of the right lung has been red
and dense as a liver, white fibrin has been found out on a pleura. Microscopic
investigation: exudate consists of fibrin, a lot of erythrocytes and a few leukocytes.
Call the stage
@Red hepatization, croupous pneumonia
Grey hepatization, croupous pneumonia
Interstitial pneumonia initial stage
Gangrene, initial stage
Lung abscess
#
35
A 44-year-old alcoholic man presents with fever and a productive cough with
copious amounts of foul-smelling purulent sputum. Physical examination finds that
changing the position of this individual produces paroxysms of coughing. Which of
the following is the most likely cause of this patient’s signs and symptoms?
Esophageal cancer
Esophageal reflux
Myocardial infarction
@Pulmonary abscess
Pulmonary infarction
#
36
A 56-year old male presented to hospital with symptoms of an acute
staphylococcal pneumonia of the inferior lobe of the right lung resulted in
cardiopulmonary insufficiency and lethal outcome of the disease. A post-mortem
revealed within the inferior lobe of the right lung a roundish cavity, 5 cm in
diameter containing pus and connected with bronchus. What complication of an
acute pneumonia takes place in that case?
@Abscesses of lungs
Gangrene of lungs
Pulmonary thromboembolism
Bronchiectatic disease
Acute bronchitis
#
37
A 45-year-old man has died because of pulmonary-cardiac insufficiency. In
autopsy the pathologist has found out croupous pneumonia in lower lobe of left
lung, 350ml of greenish-yellow fluid in the left pleural cavity. Microscopically it
contained many neutrophils. Call the complication of pneumonia
@Empyema of pleura
Fibrinous pleuritis
Pneumothorax
Hydrothorax
Hemothorax
#
38
A 49-year-old man presents with shaking chills and fever, pleuritic chest pain, and
increasing shortness of breath. A chest x-ray reveals a right-sided pleural effusion.
The lung is this area appears to be consolidated. The pleural fluid is tapped, and
laboratory examination reveals numerous acute inflammatory cells. Which of the
following is the basic defect that caused the accumulation of this pleural fluid?
Collagen-vascular disease involving the pleura
Obstruction of lymphatics by tumor
Right-sided congestive heart failure
Rupture of an aortic aneurysm
@Suppurative infection of adjacent lung tissue
#
Konec:
#end
Chronic Non-specific Lung Diseases
#
1
A 19-year-old woman presents with urticaria that developed after she took aspirin
for a headache. She has a history of chronic rhinitis, and physical examination
reveals the present of nasal polyps. This patient is at an increased risk of
developing which one of the following pulmonary disease following the ingestion
of aspirin?
@Asthma
Chronic bronchitis
Emphysema
Interstitial fibrosis
Pulmonary hypertension
#
2
A patient with bronchial asthma got viral infection that provoked status
asthmaticus with the death. At the histological examination of lungs spasm and
oedema of bronchioli were revealed. The walls were infiltrated with lymphocytes,
eosinophiles and other leucocytes; there was also a degranulation of labrocytes.
What mechanism of hypersensivity is meant?
Inflammatory
@Reaginic reaction of hypersensivity
Autoimmune
Immune complex
Immune-mediated cellular cytolysis
#
3
The major pathlogic injury in interstitial lung disease is generally accepted to be
Diffuse pneumonia
Bronchopneumonia
Bronchiolitis
Bronchitis
@Alveolitis
#
4
A patient died from cardiopulmonary decompensation. Histological examination
revealed diffused pulmonary lesion together with interstitial edema, infiltration of
tissue by limphocytes, macrophages, plasmocytes; pulmonary fibrosis, panacinar
emphysema. What disease corresponds with the described picture?
Pulmonary atelectasis
@Fibrosing alveolitis
Bronchial asthma
Bronchopneumonia
Chronic bronchitis
#
5
Examination of a miner revealed pulmonary fibrosis accompanied by disturbance
of alveolar ventilation. What is the main mechanism of this disturbance?
Constriction of superior respiratory tracts
Disturbance of neural respiration control
@Limitation of respiratory surface of lungs
Limitation of breast mobility
Bronchi spasm
#
6
From the childhood the patient had chronic inflammatory lung diseases. There was
a little of sputum that had white color. The patient was hospitalized with the signs
of pulmonary and heart insufficiency, the last one caused his death. What changes
were revealed at the section?
Macroscopically the heart is not changed
Hypertrophy and dilatation of the left ventricle
Hypertrophy of the both ventricles
Dilatation of the both ventricles
@Hypertrophy and dilatation of the right ventricle
#
7
A 68-year old patient, with a history of the chronic inflammatory disease of lungs
since his childhood, presented to the hospital with coughing accompanied by
scanty sputum (expectoration). The complications of pulmonary and a heart failure
resulted in his death. What changes in heart have been found at post-mortem?
@Right ventricular hypertrophy and dilatation
Left ventricular hypertrophy and dilatation
Ventricular hypertrophy
Ventricular dilatation
Heart without gross change
#
8
A female patient, with 10 years history of cough with a purulent sputum and
dyspnea, died from respiratory and heart failure. A post-mortem gross investigation
of a dead body revealed her fingers reminded drum sticks. A dissection of lugs
showed a bronchial alteration with a saccular deformity and purulent inflammation.
What is the most likely diagnosis?
@Bronchiectatic disease
Tuberculosis
Chronic bronchitis
Abscess
Acute bronchitis
#
9
A 48-year-old male with a long history of chronic bronchitis presents his physician
a cough with a mucopurulent sputum (especially in the morning), dyspnea, and
fingers deformity in a shape of the drum sticks. A pulmonal and heart insufficiency
was also diagnosed. Make the preliminary diagnosis.
@Bronchiectatic disease
Bronchial asthma
Chronic bronchitis
Bronchopneumonia
Emphysema of lungs
#
10
An autopsy of 55-year-old male with a 3 years history of a dry cough, a dyspnea,
pulmonary insufficiency and quick weight loss, revealed a pulmonary heart and
lungs fibrosis with a honey-comb pattern of the lungs tissue. Histological
investigation demonstrated an interstitial fibrosis with the apparent infiltration of a
stroma with lymphocytes, histiocytes and some neutrophils. What is the most
likely diagnose?
@Bronchiectatic disease
Postinflammatory pneumosclerosis
Chronic bullous emphysema
Fibrosing alveolitis
Dusty pneumosclerosis
#
11
A 27-year-old man presents with a chronic productive cough. Pertiment medical
history is that he has had recurrent sinusitis and numerouslower respiratory tract
infections since childhood. He has been married for 6 years and has no children,
although he and his wife do not use any form of birth control. His chest x-ray
reveals the apex of the heart to be directed toward the right (the cardiac apical
impulse is feltbest in the sixth intercostals space), and a CBC reveals a normocytic
normochromic anemia. Semen analysis reveals his spermatozoa to be immotile and
lack the normal ATPase-containingdynein arms. Without treatment, which of the
following changes is most likely to result from his disease?
@Abnormal permanent dilation of the bronch in the lungs
Destruction of the walls of the alveoli in the base of the lungs
Fatty metamorphosis of the hepatocytes in the liver
Hyperplasia of the smooth-muscle cells of the bronchioles in the lungs
Irreversible deposition of collagen in the subendothelial spacesin the liver
#
12
A 42-year old male, with a history of the amyloidosis, died of chronic renal failure.
A post- mortem revealed in the lower lung lobe multiple dilated bronchi filled with
abundant foul sputum. A gross investigation of a cut surface showed a honey-comb
pattern of a lungs tissue. A microscopic study demonstrated the chronic
inflammation within bronchial wall and also a replacement of elastic and muscular
fibers by a connecting tissue. These changes in a lung are regarded as:
@Bronchiectasis
Bronchopneumonia
Chronic bronchitis
Chronic pneumonia
Abscesses of lungs
#
13
An autopsy of the middle age man with a long history of bronchiectasis revealed
markedly enlarged adrenals volume at the expense of a cortical layer. Adrenals
were pale, dense, and sebaceous. Microscopic investigation showed the
unstructured, amorphous, eosinophilic, Congo red positive deposits by reticular
stroma and within vessels walls. These changes indicate a development of:
@Amyloidosis
Mucoid swelling
Fibrinoid swelling
Lipidosis
Hyalinosis
#
14
A 46-year old male, with a history of the chronic bronchiectasis, died from uremia.
An autopsy revealed enlarged, firm kidneys, with sebaceous (waxy) cut surface. To
what disease there correspond such changes?
@Amyloidosis of kidney
Glomerulonephritis
Acute tubular necrosis
Pyelonephritis
Arteriolosclerotic nephrosclerosis
#
15
A 54-year old male, with a history of destructive purulent bronchitis died of
multiple organ failure. An autopsy revealed cardiomegaly, a sebaceous (waxy)
kidney and a sago spleen. What process has complicated bronchitis?
@Secondary amyloidosis
Senile amyloidosis
Tumor-like amyloidosis
Primary amyloidosis
Generalized hyalinosis
#
16
A 7-year-old boy accidentally inhales a small peanut, which lodges in one of his
bronchi. A chest x-ray reveals the mediastinum to be shifted toward the side of the
obstruction. Which of the following pulmonary abnormalities is most likely present
in this boy?
@Absorptive atelectasis
Compression atelectasis
Contraction atelectasis
Patchy atelectasis
Hyaline membrane disease
#
17
X-ray examination discovered lungs emphysema in the patient. What is the reason
of short breath development in this case?
@Decreased lungs elasticity
Decreasing of alveoli receptors sensitivity
Increased lungs elasticity
Inhibition of respiratory center
Excitation of respiratory center
#
18
At the autopsy: lungs are enlarged in size, pale, of soft consistence, not collapsed,
don’t crepitate cutting. Microscopically, the widening of the alveolar paths and the
signs of intercapillary sclerosis can be seen. Also the interalveolar membranes are
thin. What lung disease are these signs typical for?
Pneumonia
Pneumosclerosis
Pneumothorax
Atelectasis
@Emphysema
#
19
An autopsy of the 47-year-old man revealed within lungs areas of increased
pulmonary volume, with some bullas, that bulged out from beneath the pleura.
Histological investigation showed thinning and rupture of interalveolar septas
resulted in large multiform cavities formation. What is the most likely diagnosis?
@Emphysema of lungs
Bronchiectatic disease
Cavernous tuberculosis
Chronic bronchitis
Fibrosing alveolitis
#
20
A 50-year-old man has died because of chronic pulmonary-cardiac insufficiency.
In autopsy the pathologist has found out in the lungs: areas of abnormal
enlargement with large subpleural blebs (diameter 0,5-1,5 cm). Microscopic
investigation: laceration of the airspace, thinning of alveolar walls with formation
of large cavities of the different form. What disease is revealed in lungs?
@Emphysema
Bronchoectatic disease
Cavernous tuberculosis
Chronic bronchitis
Hamman-Rich-syndrome
#
21
A 53-year old male, with a history of the chronic diffusive bronchitis, presented to
the hospital symptoms of cardiopulmonary insufficiency, then resulted in the lethal
outcome of the disease. An autopsy revealed increased volume and hyperinflation
of lungs, which covered a mediastinum with their edges and keep their form when
removed from the body and put at autopsy table. Gross investigation showed a pale
grey colour of lungs and a crunch sound while making an incision of a pulmonal
tissue. A pressing a finger on a lungs tissue creates a fossa. A mucopurulent
exudate was determined within bronchi's lumen. What is the most likely diagnosis?
@Chronic diffuse obstructive emphysema
Chronic focal emphysema
Intestinal emphysema
Primary idiopathic, emphysema
Vicarious compensatory emphysema
#
22
An autopsy of the 56-year-old man revealed increased pulmonary volume, pale
grey color, soft texture lungs that kept their form when removed from the body and
put at autopsy table. Gross investigation disclosed a crunch sound while making an
incision of a pulmonal tissue. Histological investigation showed dilated alveolar
ducts and alveoli, thinned interalveolar septas and capillary walls sclerosis. For
what disease of lungs such morphological picture is characteristic?
@Emphysema
Pneumosclerosis
Pneumothorax
Atelectasis
Pneumonia
#
23
An autopsy of the 69-year-old man with a long history of chronic bronchitis
revealed increased pulmonary volume and hyperinflation, pale grey color, soft
texture lungs that kept their form when removed from the body and put at autopsy
table. Histological investigation showed markedly dilated proximal acini, thinned
and somewhere ruptured interalveolar septas. For what disease of lungs such
morphological picture is characteristic?
@Emphysema of lungs
Chronic intestinal pneumonia
Atelectasis of lungs
Bronchiectatic disease
Pleural empyema
#
24
A patient, who had chronic obstructive pulmonary emphysema, died of
pulmonary-heart insufficiency. What changes can be revealed in the heart?
Amyloidosis
Left ventricle hypertrophy
@Right ventricle hypertrophy
Large focal cardiosclerosis
Cardiac rupture
#
25
A 63-year-old man who is a long-term smoker present with increasing shortness of
breath and dyspnea. He has smoked more than 2 packs of cigarettes per day for
more than 40 years. He denies havinga productive cough or any recent infection.
Physical examination reveals a thin elderly-appearing man in moderate respiratory
distress. While sitting him leans forward slightly and breathe quickly through
pursed lips. He is afebrile and his blood pressur is within normal limits.
Examination of his chest reveals an increased anteroposterior diameter and his lung
are hyper-resonant to percussion. His respiratory rate is increased, but no clubbing
or cyanosis is present. Chest x-ray reveals his heart to be of normal size, but there
is hyperinflation of his lung. Laboratory examination reveals that while breathing
room air, his arterial Po 2 is decreased but his arterial Pco 2 is normal. Which of the
following statements is a correct association concerning the pathogenesis of this
man’s pulmonary disease?
Destruction of entire acinus caused panlobular emphysema
Destruction of the cilia on the respiratory epithelial cells resulted in bronchiectasis
@ Destruction of proximalacinus caused centrilobular emphysema
Hyperplasia of the respiratory smooth-muscle cells resulted in intrinsic asthma
Hyperplasia of the respiratory mucus glands caused chronic bronchitis
#
26
An autopsy of a patient, who had been working within 20 years the coal worker,
revealed grayish- black dense, sclerotic lungs. The vast areas of collagen tissue
new growth and multiple macrophages, carrying black pigment within cytoplasm,
were recognized under microscope. What is the most likely disease?
@Anthracosis
Anthracosilicosis
Silicoanthracosis
Talcosis
Siderosis
#
27
A 49-year-old patient had been working as a coal miner many years and died of a
chronic pulmonary and cardiac insufficiency. A post-mortem revealed
hypoinflated, firm, sclerotic, lungs with emphysematous apexes. Pleura had a
grayish-black coloring. Gross investigation of the lungs sectional view also showed
deep black coloring of the pulmonary tissue. What is the most likely disease
resulted in patient's death?
@Anthracosis
Silicosis
Talcosis
Asbestosis
Aluminosis
#
28
A routine chest x-ray performed on an asymptomatic adult man who works at
sandblasting reveals a fine nodularity in the upper zones of the lungs and
“eggshell” calcification of the hilar lymph nodes. The patient’s serum calcium
level is 9.8 mg/dL, while his total protein is 7.2 g/dL. He denies any history of drug
use or cigarette smoking. A biopsy from his lung reveals birefringent particles
within macrophages. What is this material?
Asbestos
Beryllium
Carbon
@Silica
Talc
#
29
An autopsy of a male, who had been working within 9 years the stone polisher,
revealed multiple, roundish, dense, diffuse nodules. Histological investigation of
nodules showed concentric whorls of dense collagen fibers. At the periphery there
were aggregated of mononuclear cells, mostly lymphocytes, and fibroblasts. What
is the most likely disease?
@Silicosis
Acute pneumonia
Bronchiectatic disease
Chronic bronchitis
Bronchial asthma
#
30
An autopsy of the miner, who had worked in coal mine more of 10 years, revealed
in a lung whitish fibrous fibers and nodules 0,2-0,3 cm in diameter. Histological
investigation of nodules showed bundles of interacting concentric pink collagen
with apparent hyalinosis. There was a minimal inflammatory reaction and also
noticeable amount of a brownish dust. What is the most likely type of
pneumoconiosis in this case?
@Silicosis
Talcosis
Asbestosis
Siderosis
Berylliosis
#
31
Histological investigation of pulmonary segment, removed from a coal worker's
lung, revealed multiple roundish, nodules composed of concentric whorls of dense,
hyaline fibers of collagen. What is the most likely disease?
@Silicosis
Tuberculosis
Bronchitis
Fibrosing alveolitis
Cancer of lungs
#
32
An autopsy of a patient, who had a long history of exposure to free dioxide of
silica in the inhaled air, revealed enlarged, firm lungs with multiple miliary and
larger roundish areas of a sclerosis. Gross study showed grey and grey-black
colour of his lungs' tissue. What is the most likely diagnose?
@Nodular form of silicosis
Diffusive - sclerotic form of silicosis
Anthracosilicosis
Silicoanthracosis
Asbestosis
#
33
An autopsy of a patient, who had been working within 20 years the coal worker,
revealed enlarged, slightly firm and hypoinflated lungs. At the gross investigation
of the lungs sectional view multiple, round, grayish-black, diffuse nodules were
discovered. Histological study showed nodules composed of concentric whorls of
dense collagen fibers, some of them- with hyaline degeneration. What is the most
likely disease?
@Nodular form of silicosis
Miliary tuberculosis of lung
Bronchiectatic disease
Croupous pneumonia
Diffusive - sclerotic form of silicosis
#
34
An autopsy of a 54-year-old male, who died of a chronic pulmonary and cardiac
insufficiency, revealed enlarged, slightly firm and hypoinflated lungs. At the gross
investigation of the lungs sectional view multiple, whitish, 2-3 mm in diameter
nodules were discovered. Histological study showed nodules composed of
concentric whorls of dense, sparsely cellular collagen. What is the most likely
disease?
@"Nodular form of silicosis
Miliary tuberculosis of lung
Fibrosing alveolitis
Microfocal pneumonia
Metastasis tumor of lungs
#
35
After the histological examination of a biopsy material taken from the wall of the
bronchus in a patient with chronic bronchitis was detected the overgrowth of the
granulation tissue that elevated above the surface and contained a diffuse
inflammatory infiltration. What type of bronchitis does the patient have?
Chronic deforming
Chronic mucopurulent
Chronic mucous
Chronic purulent
@Chronic polypous
#
36
Histological investigations of the bronchus wall biopsy material, from the patient
with a history of chronic bronchitis, revealed granulation tissue and diffuse
inflammatory infiltrate. What kind of a bronchitis was diagnosed?
@Chronic polypous bronchitis
Chronic mucopurulent bronchitis
Chronic mucous bronchitis
Chronic purulent bronchitis
Chronic deforming bronchitis
#
37
A 54-year-old man presents with several roblems involving his face and pain in his
shoulder. He states that he has smoked 2 packs of cigarettes a day for almost 40
years. Physical examination reveals ptosis of his left upper eyelid, constriction of
his left pupil, and lack of sweating (anhidrosis) on the left side of his face. No other
neurologic abnormalities are found. Which of the following tumors is most likely
to be present in this individual?
A bronchioloalveolar carcinoma involving the left upper lobe
A small-cell carcinomainvolving the hilum of his left lung
A squamous cellcarcinoma involving the left mainstem bronchus
@An adenocarcinoma involving the apex of his left lung
An endobronchial carcinoid tumor involving the right mainstem bronchus
#
38
Histological investigation of a lung's biopsy revealed atypical cells which formed
multiple acinar structures and produced mucus. What is the most likely histological
diagnosis?
@High differentiated adenocarcinoma
Differentiated adenoarcinoma
Undifferentiated cancer
Differentiated squamous cell carcinoma
Glandular - squamous cell carcinoma?
#
39
Thereupon the obturation of the middle lobe bronchus by the nodule of soft tissues
the atelectasis of the middle lobe of the right lung had formed. At the
bronchobiopsy was revealed the overgrowth of the glandular atypical epithelium
with pathological mitoses that grew through the underlied tissues and the cartilage.
What disease is the most possible due to these signs?
Inflammatory polyp
Deforming bronchitis
@Bronchogenic cancer of the lung
Dysplasia of the bronchial epithelium
Sarcoma of the bronchus
#
40
A patient presents to his physician a middle lobar bronchial obstruction with a
node of intraluminal soft masses, resulted in the atelectasis of middle lobe of the
right lung. A bronchoscopy revealed in the obturation zone a new growth of
glandular type atypical epithelium with pathological mitosises which penetrated
the wall of the bronchus and infiltrated along the peribronchial tissue and a
cartilage. What is the most likely diagnosis?
@Bronchogenic carcinoma of lung
Deforming bronchitis
Inflammatory polyp
Dysplasia of bronchial epithelium
Bronchus sarcoma
#
41
A 39-year-old woman presents with a cough and increasing shortness of breath. A
chest x-ray is interpreted by the radiologist as showing right lower lobe (RLL)
pneumonia. No mass lesions are seen. The woman is treated with antibiotics, but
her symptoms do not improve. On her return visit, the area consolidation appears
to be increased. Bronchoscopy is performed. No bronchial masses are seen, but a
transbronchial biopsy is obtained in an area of mucosal erythema in the RLL. After
the diagnosis is made, the RLL is removed and section from this specimen reveals
well-differentiated mucus-secreting columnar epithelial cells that infiltrate from
alveolus to alveolus. Which of the following is the most diagnosis?
@Bronchioloalveolar carcinoma
Carcinoid
Large cell carcinoma
Small cell carcinoma
Squamous cell carcinoma
#
42
Histological investigation of a biopsy of a bronchus revealed a tumor which is
constructed from clusters of atypical cells of a laminated (stratified) squamous
epithelium, some areas with typical "pearls". What is the most likely diagnosis?
@Squamous cell keratinous carcinoma
Squamous cell nonkeratinous carcinoma
Solid carcinoma
Mucinous carcinoma
Scirrhous carcinoma
#
43
The biopsy is taken from a suspicious site at the mucous the right bronchus of a
58-year old male, with a history of the chronic bronchitis, pneumosclerosis and
cardiopulmonary insufficiency. Histological investigation revealed cellular and
tissue atypia, presence of a "cancer pearls" structures. What pathological process
associated with histological changes listed below?
@Squamous cell keratinous carcinoma
Chronic polypous bronchitis
Bronchiectasis
Acute bronchitis
Squamous metaplasia of the bronchial epithelium
#
44
A 67-year-old man who is a long-term smoker presents with weight loss, a
persistent cough, fever, chest pain, and hemoptysis. Physical examination reveals
cachectic man with clubbingof his fingers and dullness to percussion over his right
lower lobe. A chest x-ray reveals 3.5-cm hilar mass on the right and
postobstructive pneumonia of the right lower lobe. Sputum cytology is suspicious
for malignant cells. Histologic examination of transbronchial biopsy specimen
reveals infiltrating groups of cells with scant cytoplasm. No glandular structures or
keratin production are seen. The nuclei of these cells are about twice the size of
normal lymphocytes and do not appear to have nucleoli. Which of the following is
the most likely diagnosis?
Adenocarcinoma
Hamartoma
Large-cell undifferentiated carcinoma
@Small-cell undifferentiated carcinom
Squamous cell carcinoma
#
45
A 68-year-old man who worked for many years in the asbestos industry presents
with weight loss and increasing chest pain. Examination of his sputum finds very
rare asbestos bodies, while a CT scan shows a large mass involving the apical
surface of his left lung. Surgery is performed and gross examination finds a lesion
similar in appearance to that seen in the associated gross photograph of a sagittal
section of the lung. Sections from this mass examinated by electron microscopy
reveal tumor cells with long microvilli on their surface. Which of the following is
the most likely diagnosis?
@Malignant mesothelioma
Malignant thymoma
Metastatic malignant melanoma
Poorly differentiated adenocarcinoma
Small-cell carcinoma
#
46
During a routine physical examination, a 43-year-old man is found to have a
2.5-cm “coin” in the peripheral portion of his right upper lobe (RUL). Several
sputum samples sent for cytology are unremarkable, and a bronchoscopic
examination is also unremarkable. Surgery is performed and the mass is resected.
Histologic examination reveals lobulesof connective tissue that contains mature
hyaline cartilage. These lobules are separated by clefts that are lined by respiratory
epithelium. Which of the following is the most likely diagnosis?
Adenocarcinoma
Bronchioloalveolar carcinoma
Carcinoid
Fibroma
@Hamartoma
# The pathology of the digestive tract
Gastro-intestinal tract diseas
#
1
A physical examination of a fauces of a 12-year-old child with tonsillitis revealed the reddening and slight edema of
mucosa, with reactive enlargement of tonsils, dotted by pinpoints of exudates emanating from the tonsillar crypts.What
clinical-morphological variant of tonsillitis is the most probable in this case?
@Lacunar tonsillitis
Catarrhal tonsillitis
Purulant tonsillitis
Fibrinous tonsillitis
Necrotic tonsillitis
#
2
Physical exam of the patient’s fauces, which has quinsy, reveals hyperemia of the mucous membrane of the palate, and
enlarged red tonsils. Small white-yellow focuses can be seen on their surface. What clinico-morphological type of quinsy
is the most possible?
Fibrinous
Catarrhal
Suppurative
@Lacunar
Necrotic
#
3
A mother of a newborn complains of her baby's constant belching with undigested milk. Which developmental anomaly is
it an evidence of?
@Esophageal atresia
Anal atresia
Labium leporium
Faux lupinum
Esophageal fistula
#
4
All the following statements regarding carcinoma of the esophagus are true EXCEPT
most carcinoma arising in the body of the esophagus are squamous
squamous carcinoma begin as lesions in situ
patients with Barrett's esophagus have approximately a 10 persent risk of carcinoma
the most common morphological form is a polypoid fungating mass
@distant metastases are frequently present at the time of diagnosis
#
5
During the endoscopy the inflammation of a major papilla of the duodenum and the disturbances of bile secretion were
found. In which part of duodenum were the problems found?
Lower horizontal part
Ascendant part
Bulb
Upper horizontal part
@Descendent part
#
6
A patient underwent the surgical removal of pathologically disordered distal quarter of small intestine. How will it
influense the absorption of nutritional substances in case of patient's ordinary diet?
Carbohydrates absorption will be reduced
Proteins absorption will be reduced
Water absorption will be reduced
Lipids absorption will be reduced
@Absorption will not be altereded
#
7
A 62-year-old man presents after fainting at home approximalety 1h ago. He says that for the past day he has had
increasing left-sided abdominal pain, and also he as noted bright red blood in his stool. He denies any history of vomiting
or diarrhea. Physical examination finds a low-grade level and gross blood on rectal examination. Further work-up finds
abnormalities in his colon that are similar in appearance to the abnormalities seen in the gross picture of his colon below.
Which of the following is the most likely cause of his GI bleeding?
Angiodisplasia
Appendicitis
@Diverticulosis
Hemorrhoids
Instussusception
#
8
A 29-year-old woman presents with colicky lower abdominal pain and frequent bloody diarrhea with mucus. Physical
examination finds fever and peripheral leukocytosis, while multiple stool examinations fail to reveal any ova or parasites.
A colonoscopy reveals the rectum and sigmoid portions of her colon to have superficial mucosal ulcers with hemorrhage,
but regions more proximal are within normal limits. Which of the following histologic changes is most likely to be seen
in a biopsy specimen taken from her rectum?
@Crypt abscesses with crypt distortion
Dilated submucosal blood vessels with focal thrombosis
Increased thickness of the subepithelial collagen layer
Noncaseating granulomas with scattered giant cells
Numerous eosinophils within the lamina propria
#
9
A 58-year-old man presents with increasing fatigue. His past medical history is unremarcable. Physical examination is
also unremarcable exceptfor heme-positive stool. Laboratory examination finds his hematocrit to be slightly decreased
and hypochromic microcytic red cells are present in his peripheral smear. An upper GI is unremarcable, but a barium
enema study finds a 4-cm mass in the left side of his colon having an "apple core" appearance. Which of the following is
the most likely histologic diagnosis for this colon mass?
@Adenocarcinoma
Leiomyosarcoma
Medullary carcinoma
Small-cell carcinoma
Squamous cell carcinoma
#
10
A 45-year-old man presents with feve, chronic diarrhea, and weight loss. He is found to have multiple pain and swelling
of his joints (migratory polyartritis)and generalized lymphadenopathy. Physical examination reveals skin
hyperpigmentation. A biopsy from his small interstines reveals the presence of macrophagesin the lamina propria that
contain PAS-positive cytoplasm. Which of the following is the most likely diagnosis?
Abetalipoproteinemia
Crohn’s disease
Hartnup disease
Nontropical sprue
@Whipple’s disease
#
11
A microscopic investigation of a colon biopsy revealed a superficial defect of a mucosa, decrease of goblet cells number
with lack of mucus in them. In addition, considerable lympho- plasmocyte infiltration with presence of segmented
leucocytes and eosinocytes was defined. Choose the most probable and exact diagnosis.
@Nonspecific ulcerative colitis in exacerbation
Crohn's disease
Chronic ischemic colitis
Dysentery in the third stage
Amebiaz of intestine with forming of ulcers
#
12
A 39-year-old man present with bloody diarrhea. Multiple stool examinations fail to reveal any ova or parasites. A barium
examination of the patient's colon reveals a characteristic "string sign". A colonoscopy reveals the rectum and sigmoid
portion of the colon to be unremarceble inflamatory cells within the lamina propria. Worsening of the patient's symptoms
results in emergency resection of the distal small intestines. Gross examination of this resected bowel reveals deep, long
mucosal fissures extending deep into the muscle wall. Several transmural fistulas are also found. Which of the following
is the most likely diagnosis?
Ulcerative colitis
Lymphocytic colitis
Infectious colitis
Eosinophilic colitis
@Crohn's disease
#
13
Two subtotal colectomy spesiments are sent to the laboratory with both showing a hemorrhagic cobblestone appearance
of the mucosa. One, however, shows longitudinal grooving of the surface, which suggests
ischemic bowel disease
multiple polyposis syndrome
ulcerative colitis
@Crohn's disease
intestinal tuberculosis
#
14
A distinguishing feature when comparing ulcerative colitis with Crohn's disease is
colonic involvement
possible malignant transformation
artritis
@fistula formation
absence of granulomas
#
15
The colonic wall 30 cm from the anal margin in a man with severe bloody diarrhea. All the statements regarding this
condition are true EXCEPT
inflammation is usualy limited to the lamina propria
multiple crypt abscesses are comonly seen
atypical cytologic changes occur in the mucosa
@granulomas occur in the mucosa
"skip" lesions are not present
#
16
A macroscopic investigation of the appendix, 9 cm in length and 0.9 cm in thickness, revealed dull and hyperemic serous
membrane. Microscopic study determined edema, stases in capillaries and venules, fine hemorrhages throughout the
mucosa. Besides these, the necrotic foci with leukocyte infiltration around them were also verified. What is the most
likely diagnose?
@Superficial appendicitis.
Simple appendicitis.
Suppurative appendicitis
Flegmonous-ulcerative appendicitis
Gangrenous appendicitis.
#
17
A vermiform appendix is 9cm long and 0,9cm thick. Serous membrana is dark and sanguineous. Microscopically: the
wall is oedematous, capillary and venular stases, small hemorrhages. The focuses of necrosis that are surrounded by
leukocytic infiltration are located in the mucous and submucous membranae. What is the most possible diagnosis?
Acute phlegmonous-ulcerative appendicitis
Acute simple appendicitis
Acute phlegmonous appendicitis
@Acute superficial appendicitis
Acute gangrenous appendicitis
#
18
A 16-year-old boy was performed an appendectomy. He has been hospitalized for right lower quadrant abdominal pain
within 18 hours. The surgical specimen is edematous and erythematous. Infiltration by what of the following cells is the
most typical for the process occuring here?
Basophils
Eosinophils
Monocytes
@Neutrophils
Limphocytes
#
19
An 18-year-old woman presents with abdominal pain localized to the right lower quadrant, nausea and vomiting, mild
fever, and an elevation of the peripheral leukocyte count to 17,000/mL. An appendectomy is performed. Which of the
following statement best describes the expected microscopic appearance of her appendix?
An appendix with a normal appearance
@Neutrophils within the muscular wall
Lymphoid hyperplasia and multinucleated giant cells within the muscular wall
A dilated lumen folled with mucus
A yellow tumor nodule at the tip of the appendix
#
20
Microscopic study of appendix determined intensive leukocyte infiltration throughout all its layers. What is the most
likely type of inflammation?
@Suppurative appendicitis
Apostematous appendicitis
Gangrenous appendicitis.
Superficial appendicitis.
Flegmonous-ulcerative appendicitis
#
21
Microscopic study of appendix determined intensive leukocyte infiltration hyperemia, stases throughout all its layers.
What is the most likely type of appendicitis?
@Suppurative appendicitis
Gangrenous appendicitis
Superficial appendicitis
Simple appendicitis
Chronic appendicitis
#
22
A removed appendix delivered to pathology department. Macroscopic investigation revealed its thickening, enlargement,
dull and hyperemic serous membrane. A yellowish-green liquid lumen was visible on a cut section within its lumen. What
is the most likely type of the appendicitis?
@Suppurative appendicitis.
Catarrhal appendicitis.
Superficial appendicitis.
Gangrenous appendicitis.
Apostematous appendicitis
#
23
Macroscopic investigation of a removed appendix revealed its thickening, enlargement, dull and hyperemic serous
membrane with whitish membranaceous incrustation. A whitish-yellow, opaque, viscous liquid detected on a cut section
within its lumen. What is the most likely type of the appendicitis?
@Suppurative appendicitis.
Gangrenous appendicitis
Simple appendicitis
Superficial appendicitis
Chronic appendicitis.
#
24
A vermiform appendix, sent to the pathomorphologic department after the operation, is thickened and enlarged in size.
Serous membrana is dark and sanguineous. A yellow-green fluid excretes from the lumen, when the vermiform appendix
is incised. The wall is diffusely infiltrated with leucocytes. What type of appendicitis do the following changes develop?
Superficial catarrhal
Simple catarrhal
@Phlegmonous
Gangrenous
Abscessing
#
25
At the microscopical exam of the vermiform appendix that was operatively removed were noted the following facts:
oedema, diffuse neutrophilic infiltration of the wall with the necrosis and the presence of a mucous membrane defect with
the affection of the muscular one. What form of appendicitis has developed?
@Phlegmonous-ulcerative
Phlegmonous
Gangrenous
Superficial
Abscessing
#
26
Microscopic study determined edema, diffusive leukocyte infiltration of appendix wall and also a mucosal lesion with
alteration of its muscular layer. What is the most likely diagnose?
@Flegmonous-ulcerative appendicitis
Suppurative appendicitis
Gangrenous appendicitis
Superficial appendicitis
Apostematous appendicitis
#
27
Microscopical examination of a removed appendix revealed an edema, diffuse neutrophilic infiltration of appendix wall
along with necrosis and defect of mucous membrane with affection of its muscle plate. What appendicitis form was
developed?
Phlegmonous
@Ulcerophlegmonous
Gangrenous
Superficial
Apostematous
#
28
The removed appendix thickened and covered by fibrinopurulent incrustation. A purulent exudates infiltrated all appendix
layers; the destruction of mucosa membrane was evident. What is the most likely diagnosis?
@Flegmonous-ulcerative appendicitis
Simple appendicitis
Suppurative appendicitis
Gangrenous appendicitis
Superficial appendicitis
#
29
In the histological specimen an organ is seen. Wall of this organ consists of 4 layers: first is mucous layer forming pits
and glands; second is submucous layer of connective tissue; third is muscularis externa containing tree layers of smooth
muscles; fourth is serous layer. Which organ can be recognized in the specimen?
@Stomach
Jejunum
Duodenum
Esophagus
Colon
#
30
During histological examination of the stomach it was found out that glands contained very small amount of pariental
cells or they were totally absent. Mucose membrane of what part of the stomach was studied?
@Pyloric part
Cardia
-
Body of stomach
Fundus of stomach
#
31
A patient ill with chronic gastritis went for endogastric pH-metry that allowed to reveal decreased acidity of gastric juice.
It is indicative of diminished function of the following cells:
Chief exocrinocytes
@Parietal exocrinocytes
Endocrinocytes
Cervical cells
Accessory cells
#
32
Examination of the patient with pernicious anemia revealed antibodies against cell’s proteins of gastric glands cells
producing gastric extrinsic factor . What kind of cells in gastric glands was destroyed?
@Parietal cells
Mucous cells
Chief cells
DNES-cells
Surface- lining cells
#
33
A biopsy of the antrurn of the stomach of an adult who presents with epigastric pain reveals numerous lymphocytes and
plasms cells within the lamina propria, which is of normal thickness. There are also scattered neutrophil within the
glandular epithelial cells. A Steiner silver stain from this specimen is positive for a small, curved organism. These
histologic changes are moat consistent with infection by which one of the following organisms?
Enteroinvasive Escherichia coli
Enterotoxigenic E. coli
@Helicobacter pylori
Salmonella typhi
Shigella species
#
34
All the following statements regarding Helicobacter pylori gastritis as illustrated in the photomicrograph below are true
EXCEPT
it is present in the majority of patients with duodenal ulceration
@the inflammatory infiltrate is characteristically rich in eosinophils
tissue invasion by microorganisms is inconspicuous
organism is absent from arease of intestinal metaplasia
urease is an important virulence factor of H. pilory
#
35
A 49-year-old woman takin ibuprofen for increasing joint pain in her hands presents with increasing pain in her
midsternal area. Gastroscopy reveals multiple, scattered, punctuate hemorrhagic areas in her gastric mucosa. Biopsies
from one of these hemorrhagic lesions reveal mucosal erosions with edema and hemorrhage. No mucosal ulceration is
seen. Which of the following is the most likely diagnosis?
Active chronic gastritis
@Acute gastritis
Autoimmune gastritis
Chronic gastritis
Peptic ulcer disease
#
36
A gastroscopy of a 55-year old patient revealed a diffusive swelling, hyperemia and solitary small hemorrhage of the
stomach mucosa. These were accompanied with considerable quantity of a muddy, viscous grey exudate on a gastric
surface. What gastritis has developed at the patient?
@ Catarrhal gastritis
Hemorrhagic gastritis
Flegmonous gastritis
Fibrinous gastritis
Corrosive gastritis
#
37
A gastroscopy of a 44-year old patient, with a history of a pain in epigastrium after meal, revealed a hyperemia of the
stomach mucosa with the stomach folds reduction. Histological study showed a thinning of the mucosa, a reduction of the
glands quantity accompanied with the- growth of a connecting tissue, lymphocytes and plasmocytes infiltration. Specify,
what of the listed diagnoses is the most probable?
@Chronic atrophic gastritis.
Acute catarrhal gastritis.
Acute suppurative gastritis.
Chronic superficial gastritis.
Giant hypertrophic gastritis.
#
38
A histological investigation of a gastric biopsy revealed a thinning of a stomach mucosa with reduction of a glands
quantity. This was accompanied with a considerable growth of a connecting tissue, dilation of a glands lumen,
lymphocytes and plasmocytes infiltration of the mucosa. What is the most likely diagnosis?
@Chronic atrophic gastritis
Chronic superficial gastritis
Chronic atrophic gastritis with an intestinal metaplasia
Phlegmon of stomach
-
#
39
A histological investigation of a gastric biopsy of a 50-year old woman revealed a thinning of a stomach mucosa with
reduction of a glands quantity, foci of an intestinal metaplasia; a plethora, an edema and a stromal sclerosis. These were
accompanied with a diffuse leucocytes, lymphocytes and plasmocytes infiltration of the mucosa. What is the most likely
diagnosis?
@Chronic atrophic gastritis in an active phase •
Chronic atrophic gastritis in a nonactive phase
Chronic superficial gastritis
Acute catarrhal gastritis
Acute fibrinouse gastritis
#
40
A 37-year-old man has had nausea and vomiting for 5 weeks. He experienced an episode of hematemesis yesterday. On
physical examination he has no abnormal findings. Upper Gl endoscopy is performed, and there is a 1.5 cm diameter
lesion in the gastric antrum which appears to be an area with loss of the epithelial surface. These findings are most typical
for which of the following pathologic processes?
Abscess
Serositis
Granuloma
Gangrene
@Ulcer

#
41
A histological investigation of the removed stomach ulcer revealed in its floor the fibrinopurulent exudate, a zone of the
fibrinoid necrosis, a granulation tissue with a fibrous tissue underneath. What is the most likely diagnosis?
@Chronic ulcer
Acute ulcer
Acute erosion
Phlegmon of stomach
-
#
42
A post-mortem of a male with a history of anemia accompanied by vomiting by dark gastric contents reveled in a stomach
about 1 liter of liquid blood and bloody clots. A gross investigation showed an on small curvature of a stomach an oval
solitary ulcer with the elevated firm edgesand a smooth floor. What is the most likely diagnosis?
@Chronic gastric ulcer
Chronic atrophic gastritis
Acute gastritis
Acute gastric ulcer
Chronic hypertrophic gastritis
#
43
Morphological exam of a stomach reveals a deep defect of a wall with the affection of a muscular membrana; a proximal
margin of the last is excavated but a distal one is plain. Microscopically: an area of necrosis is detected at the bottom of
the defect. A granulation tissue and a massive area of a cicatricle tissue (at the place of muscular membrana) are located
under it. Make a diagnosis.
Chronic ulcer at the remission stage
Chronic ulcer with the malignancy
Acute ulcer
@Chronic ulcer at the stage of acute condition
Cancer-ulcer
#
44
A gross investigation of a stomach revealed a deep defect of a gastric wall with a lesion of a muscular layer. The proximal
edge was undermined and distal one was flat. A histological study of the removed stomach showed in its floor the zone of
the fibrinoid necrosis, a granulation tissue and massive fibrosis, which replaced the muscular layer. What is the most
likely diagnosis?
@Chronic ulcer in exacerbation
Chronic ulcer in remission
Acute ulcer
Erosion
Cancer-Ulcer
#
45
An operation of a male with a medical history of a hematemesis revealed a stomach ulcer penetrated into the muscular
layer. Ulcer edges were dense. A bleeding vessel was detected at the ulcer's bottom. What is the type of an ulcer?
@Chronic ulcer with hemorrhage
Chronic ulcer with penetration
Ecute ulcer with bleeding
Chronic ulcer with perforation
Chronic ulcer with malignisation
#
46
A 44-year old patient, with a history of the chronic duodenum peptic ulcer, died of peritonitis. An autopsy revealed
multiple steatonecroses of a retroperitoneal tissue and a pancreas. A gross investigation of a duodenum demonstrated an
ulcerative defect, which was 5 mm in diameter and 10 mm deep. The edges presented necrotic masses. Diagnose the
complication of a duodenal peptic ulcer?
@Penetration
Hemorrhage
Stenosis
Perforation
-
#
47
During the section of a 29-year-old man, that had duodenal ulcer for a long time, were revealed the signs of peritonitis,
numerous steatonecroses of extraabdominal fat and pancreas. In the area of the pancreas’ body, an ulcer-like defect was
located (diameter - 5 mm, depth - to 10 mm), the margins of which contained necrotic mass. Diagnose the complications
of the duodenal ulcer.
Malignancy
Hemorrhage
Stenosis
Perforation
@Penetration
#
48
During the palliative operation of a 46-year-old woman relatively the cancer of the stomach revealed the presence of the
Krukenberg’s metastases in the ovaries. What way of metastasis led to the affection of the ovaries?
@Lymphogenic retrograde
Lymphogenic orthograde
Haematogenic
Implantation
Canalicular
#
49
At the section of a 42-year-old woman, who had an operation relatively the tumour of the stomach, the ovary was hardly
enlarged in size, of solid consistence and of white color. Histologically: there were revealed atypical epithelial cells, that
were located between the strata and shaft brace of connective tissue. What is the most possible disease?
Serous cystadenoma
@Krukenberg’s carcinoma (metastatic ovarian carcinoma)
Pseudomucinous cystcarcinoma
Malignant tecoma
Malignant granulous-cellular tumour
#
50
An autopsy of a 48-year-old woman with a history of the stomach neoplasm revealed the markedly enlarged, firm, white
color ovary. Microscopic investigation showed markedly atypical epithelial cells placed among the layers and fibers of a
connecting tissue. What is the most likely diagnosis?
@Krukenberg tumor of ovary.
Serous cystadenocarcinoma
Pseudomucinous cystadenocarcinoma.
Malignant thecoma.
Malignant granulosa cell tumor.
#
51
The liver of a 55-year-old man at autopsy contains multiple tumor masses from 2 to 5 cm in size that are mostly firm and
tan and that grossly exhibit umbilication with central necrosis. Which of the following statements would best characterize
such an appearance:
There is multicentric origin of a benign neoplasm.
The neoplasm has a high grade.
@The primary neoplasm is in the stomach.
A carcinogen was the underlying cause for the neoplasm.
The neoplasm has an advanced stage.
#
52
The subclavicular lymphatic nodules are enlarged in the 45-year-old patient. A metastasis of a signet-ring cell cancer was
revealed at the biopsy material from the nodule. Choose the most possible localization of a primary tumour.
Pulmonary cancer
Esophageal cancer
Thyroid cancer
@Cancer of the stomach
Uterine neck cancer
#
53
A 48-year old female presents to her physician enlarged supraclavicular lymph nodes. Microscopic investigation of a
biopsy from a lymph node revealed the metastasis of the signet-ring cancer. Choose the most probable localization of a
primary tumor.
@Cancer of stomach.
Cancer of oesophagus.
Cancer of thyroid.
Cancer of lung.
Cancer of cervix uteri.
#
54
A 53-year-old man presents with increasing gastric pain and is found to have a 3-cm mass located in the anterior wall of
his stomach. This mass is resected and histologic examination reveals a tumor composed of cells having elongated,
spindle-shaped nuclei. The tumor does not connect to the overlying gastric epithelium and is instead found only in the
wall of the stomach. The tumor cells stain positively with CD117, but negatively with both desmin and S-100. Special
studies find that these tumor cells have abnormalities of the KIT gene. Which of the following is the most likely
diagnosis?
Ectopic islet cell adenoma (VIPoma)
@Gastrointerstinal stromal tumor (GIST)
Submucosal leiomyoma (“fibroid tumor”)
Lymphoma of mucosa-associated lymphoid tissue (MALToma)
Nonchromaffin paraganglioma (chemodectoma)
#
Liver diseases
#
1
Pre-hepatic (hemolytic) jaundice takes place in all the following cases besides the only one of them:
@Viral hepatitis
Sepsis
Malaria
Hemolytic disease of newborns
Relapsing fever
#
2
A 42-year-old previously healthy woman notes that over the past week her eyes have developed a jaundiced appearance.
She has had mild nausea and vomiting over the past week. On physical examination she has scleral icterus. She has no
other major physical examination findings except for mild right upper quadrant tenderness. Which of the following
underlying conditions is most likely to contribute to development of her icterus?
Hypercholesterolemia
Thrombocytopenia
Metastatic carcinoma
@Hepatitis
Diabetes mellitus
#
3
A 4-year-old boy presents with mild fatigue and malaise. Several other children in the day-care center he attends 5 days a
week have developed similar illnesses. Physical examination finds mild liver tenderness, but no lymphadenopathy is
noted. Laboratory examination finds mildly elevated serum levels of liver enzymes and bilirubin. The boy recovers from
his mild illness without incident. Which of the following organisms is the most likely cause of this child's illness?
Cytomegalovirus (CMV)
Epstein-Barr virus (EBV)
Group A b-hemolytic streptococcus
@Hepatitis A virus
Hepatitis B virus
#
4
The patent with the viral hepatitis A entered the infectious hospital. Which antibodies will synthesize first?
@IgM.
IgG.
IgA.
IgD.
IgE.
#
5
An autopsy of a male, with a history of a drug abuse, revealed vacuolar dystrophy of hepatocytes, a Councilman's bodies,
periportal inflammation consisted mainly of chronic inflammatory cells, lymphocytes, plasma cells and histiocytes. What
is the most likely etiology of the disease?
@Viral
Bacterial
Toxic
Parasite
Fungus
#
6
At the histological exam of a dead man, who was given numerous drug injections, were revealed the hydropic dystrophy
of hepatocytes, dim-glasslike hepatocytes, acidophilic Kaunsilmann corpuscles, and lymphocytic-macrophagocytic
conglomerations in portal tracts. What is the most possible etiology of the disease?
Bacterial
@Viral
Toxic
Parasitic
Fungus
#
7
In biopsy of the liver of a 40-year-old man, who received injections of narcotics the pathologist has found out acidophilic
Councilmen’s bodies, lymphocytes and macrophages surrounding portal tracts, "ground-glass” hepatocytes (an etiology
of disease is virus B). Call this disease
@Virus hepatitis B
Tuberculosis
Virus chirrosis
Parasitogenic hepatitis
Steatosis
#
8
A patient presented with vomiting, loss of appetite and jaundice. A histological investigation of his liver biopsy revealed
vacuolar and ballooning dystrophies of hepatocytes and necrosis of some cells. The Councilman's bodies were also
determined. Besides, a destruction of the liver's beam architecture within lobes and inflammation consisted mainly of
chronic inflammatory cells, lymphocytes, plasma cells and histiocytes were registered. What form of virus hepatitis is
most likely?
@Viral hepatitis
Purulent hepatitis
Cirrhosis of the liver
Toxic dystrophy of the liver
Malaria
#
9
A 48-year-old man presents with fatigue and slight malaise. Physical exammation is unremarkable except for slight
tenderness in the upper right quadrant of his abdomen. Laboratory examination reveals mild elevation of the liver
enzymes. He is followed over the next year and is found to have intermittent hyperbilirubinemia along with episodic
elevations in his serum transaminase levels (AST and ALT). During these episodes the AST/ALT ratio is less than 1. A
liver biopsy reveals chronic inflammation of U the portal triads that spills over into the hepatocytes and moderate fatty
change of the hepatocytes. No hepatocytes with ground-glass cytoplasm are found. Which of the following viruses is the
most likely cause of chronic hepatitis in this individual?
Hepatitis A virus
Hepatitis B virus
@Hepatitis C virus
Hepatitis D virus
Hepatitis E virus
#
10
During surgical operation a blood transfusion was made. The blood must be checked to find antigens of some disease.
What disease is expected to be found?
Adenovirus
Enterovirus
Virus of hepatitis E
Virus of hepatitis A
@Virus of hepatitis B
#
11
The donor who had not donated the blood for a long time was examined with IFA method. Anti-HBs antibodies were
revealed. What does positive result of IFA in this case mean?
Chronic hepatitis С
Chronic hepatitis В
Acute hepatitis B
@Previous hepatitis B
Acute hepatitis C
#
12
A 35-year-old man exhibits clinical symptoms of the virus hepatitis B. A puncture liver biopsy revealed: a destruction of
the liver's beam architecture with polymorphism of hepatocytes and plural mitosis figures. Hepatocytes were enlarged,
filled by vacuoles which contained a transparent liquid. What kind of a dystrophy is characteristic for the described case?
@Hydropic dystrophy
Fatty dystrophy
Hyalin dystrophy
Carbohydrate dystrophy
-
#
13
A patient being treated for viral hepatitis type B got symptoms of hepatic insufficiency. What blood changes indicative of
protein metabolism disorder will be observed in this case?
Absolute hyperglobulinemia
@Absolute hypoalbuminemia
Absolute hyperfibrinogenemia
Absolute hyperalbuminemia
Proteinic blood composition is unchanged
#
14
A 27-year-old woman presents with headaches, muscle pain (myalgia), anorexia, nausea, and vomiting. She denies any
history of drug or alkohol use, but upon further questioning she states that recently she has lost laste for coffee and
cigarettes. Physical examination reveals a slight yellow discoloration of her scleras, while laboratory results indicate a
serum bilirubin level of 1.8 mg/dL, and aminotransferases (AST and ALT) levels are increased. Which of the following is
the most likely diagnosis?
Gilbert's syndrome
Chronic hepatitis
Amebic liver abscess
@Acute viral hepatitis
Acute hepatic failure
#
15
A 38 year old patient with full-blown jaundice, small cutaneous hemorrhages, general weakness and loss of appetite
underwent puncture biopsy of liver. Histological examination revealed disseminated dystrophy, hepatocyte necrosis,
Councilman's bodies. Lobule periphery has signs of significant infiltration by lymphocytes, there are also individual
multinuclear hepatocytes. What is the most probable diagnosis?
Chronic hepatitis
@Acute viral hepatitis
Toxic degeneration of liver
Acute alcoholic hepatitis
Miliary hepatic cirrhosis
#
16
A mononuclear portal inflammatory infiltrate that disrupts the limiting plate and surrounds individual hepatocytes
(piecemeal necrosis), as shown in the photomicrograph below, is characteristic of
ascending cholangitis
@chronic active hepatitis
acute alcoholic hepatitis
cholestatic jaundice
nutritional cirrhosis
#
17
A 49-year-old woman presents with increasing fatigue and is found in have elevated liver enzymes (AST and ALT). You
follow her in your clinic ,ind find over the next 9 months that her liver enzymes have remained ele-vnieAll serologic tests
for viral markers are within normal limits. A liver Inopsy reveals chronic inflammation in the portal triads that focally
ilestroys the limiting plate and "spills over" into the adjacent hepatocytes. I'here are no granulomas present, and there is
no evidence of fibrosis surrounding any of the bile ducts within the portal triads. Anti-smooth-muscle antibodies and
antinudear antibodies are found in the patient's icrum. An LE cell test is positivWhich of the following is the most likely
diagnosis?
@Autoimmune hepatitis
Chronic persistent hepatitis
Primary biliary cirrhosis
Primary sclerosing cholangitis
Systemic lupus erythematosus
#
18
A 52-year-old patient presented with fatigue, loss of appetite, muscle and joint aches, an increase in body temperature to
38°C. On the 7th day a patient exhibited a yellow color in the skin, urine and around the whites of the eyes and also an
acute pain in right hypochondrium. A histological investigation of his liver biopsy revealed a destruction of the liver's
beam architecture, vacuolar and ballooning dystrophies of hepatocytes, some cells were necrotic. The Councilman's
bodies were also determined. On the periphery of lobes were registered the enlarged quantity of multinuclear hepatocytes.
What form of virus hepatitis is most likely?
@Icteric hepatitis
Malignant hepatitis
Chronic hepatitis
Cholestatic hepatitis
Anicteric hepatitis
#
19
After consumption of rich food a patient has nausea and heartburn, steatorrhe A. This condition might be caused by:
Disturbed phospholipase synthesis
Increased lipase secretion
Amylase deficiency
@Bile acid deficiency
Disturbed tripsin synthesis
#
20
Because of present gallstone in the common bile duct, a patient has no bile excretion into duodenum. What disorder can it
cause?
Carbohydrates digestion
Proteins absorption
Proteins digestion
Carbohydrates absorption
@Lipids digestion
#
21
A patient complains of frequent diarrheas, especially after consumption of fattening food, and of body weight loss.
Laboratory examination revealed steatorrhea; hypocholic feces. What can be the cause of this condition?
Mucous membrane inflammation of small intestine
@Obturation of biliary tracts
Lack of pancreatic phospholipase
Lack of pancreatic lipase
Unbalanced diet
#
22
A patient with cholelithiasis fell ill with mechanic jaundice. Examination revealed that the stone was in the common bile
duct. What bile-excreting ducts make up the obturated duct?
@Ductus hepaticus communis et ductus cysticus
Ductus hepaticus sinister et ductus cysticus
Ductus hepaticus dexter et ductus cysticus
Ductus hepaticus communis et ductus choledochus
Ductus hepaticus dexter et sinister
#
23
A 48 y.o. patient was admitted to the hospital with complaints about weakness, irritability, sleep disturbance. Objectively:
skin and scleras are yellow. In blood: conjugated bilirubin, cholalemia. Feces are acholic. Urine is of dark colour
(bilirubin). What jaundice is it?
@Mechanic
Gilbert's syndrome
Parenchymatous
Hemolytic
Crigler-Najjar syndrome
#
24
Which of the folowing cells found within the liver is the major source of the excess collagen deposited in cirrhosis?
Hepatocytes
Kupffer cells
@Ito cells
Endothelial cells
Bile duct epithelial cells
#
25
In chronically diseased liver some cells proliferate and acquire the feature of myofibroblasts with or without the lipid
droplets. These cells are found close to the damaged hepatocytes and play a major role in development of fibrosis. What
are these cells?
@Ito’s cells
Hepatocyte
Kupffer cells
Pit cells
Endothelium
#
26
A 55-yar-old man presents with increasing fatigue, weakness, anorexia and jaundice over the past several months.
Physical examination finds mild ascites and gynecomastia. A liver biopsy reveals regenerative nodules of hepatocytes
surrounded by fibrosis, as seen in the picture below. Which of the following is the source of the excess collagen deposited
in these fibrotic bands?
Hepatocytes
Kuppfer cells
@Ito cells
Endothelial cells
Bile duct epithelial cells
#
27
A patient suffers from hepatic cirrhosis. Examination of which of the following substances excreted by urine can
characterize the state of antitoxic function of liver?
Kreatinine
Uric acid
Ammonium salts
@Hippuric acid
Aminoacids
#
28
An autopsy of a 53-year old patient, with a history of a chronic alcoholism and repeated attacks of an alcoholic hepatitis,
revealed a firm, yellow color liver with pointed edge. A liver surface was coarsely scarred with multiple fine knots on a
cut surface. What is the most likely diagnosis?
@Cirrhosis of liver.
Cancer of liver.
Fatty dystrophy of liver.
Chronic hepatitis.
Acute hepatitis.
#
29
Analysis of a punction biopsy material of liver revealed hepatocyte dystrophy with necroses as well as sclerosis with
disorder of beam and lobulous structure, with formation of pseudolobules and regenerative nodes. What is the most
probable diagnosis:
Progressive massive liver necrosis
Chronic hepatosis
Acute hepatitis
@Liver cirrhosis
Chronic hepatitis
#
30
A microscopical investigation of a liver biopsy revealed a dystrophy, necroses of hepatocytes, a sclerosis, accompanied
with destruction of a hepatic beam and a lobular structure, a formation of false lobes and regenerative nodes. What is the
most likely diagnosis?
@Cirrhosis of liver
Chronic gepatoz
Chronic hepatitis
Massive diffuse necrosis of liver
Acute hepatitis
#
31
The liver biopsy is taken from the patient with symptoms of a parenchymatous icterus and a portal hypertension.
Histological investigation revealed a fatty dystrophy of hepatocytes, a destruction of a hepatic beam and a lobular
structure, a formation of false lobes and regenerative nodes. These were accompanied with a porto- portal fibrous septae
formation and periportal lympho- macrophageal infiltrates. What is the most likely diagnosis?
@Cirrhosis of liver
Alcoholic hepatitis
Chronic hepatosis
Viral hepatitis
Toxic dystrophy
#
32
A patient with the symptoms of acute alcoholic poisoning was brought to the hospital. What carbohydrates metabolism
changes are typical for this condition?
The anaerobic glucose metabolism predominates in muscles
The gluconeogenesis is increased in liver
The breakage of glycogen is increased in liver
@The gluconeogenesis velocity in liver is decreased
The anaerobic breakage of glucose is increased in muscles
#
33
A 45-year-old man presents with weight loss, steatorrhea, and mal absorption. A CT scan of the abdomen reveals a
questionable mass in the head of the pancreas. A biopsy specimen microscopically reveals chronic inflammation and
atrophy of the pancreatic acini with marked fibrosis. No malignancy is identifie. Which of the following is the most
common cause of this disorder in adults?
Abdominal trauma
@Chronic alcoholism
Cystic fibrosis
Gallstones
Hyperlipidemia
#
34
An autopsy of a male, with a history of a chronic alcohol abuse, revealed a small, firm, micronodular liver. Histological
investigation showed a fatty dystrophy of hepatocytes, a formation of small regenerative (false) nodes separated by
fibrous septae with lympho- macrophageal infiltration. What is the most likely diagnosis?
@Alcoholic cirrhosis.
Chronic active alcoholic hepatitis.
Chronic persistent alcoholic hepatitis.
Toxic dystrophy of liver.
Fatty hepatosis.
#
35
On autopsy of the man with alcohol abuse for a long time it was revealed: dense, small-knobby, small size liver.
Microscopically: small pseudo-lobules, divided with thin layers of connective tissue with lymphomacrophagial infiltrates;
hepatocytes in the state of globular fatty dystrophy. What is the most likely diagnosis?
Toxic liver dystrophy
Chronic persistent alcohol hepatitis
Chronic active alcohol hepatitis
Fatty hepatosis
@Alcohol cirrhosis
#
36
At the section of a man, who had alcohol abuse for a long time, the liver was small, solid, micro tuberous.
Microscopically: pseudo lobules were poky and were separated by thin strata of connective tissue with
lymphomacrophagal infiltrates. Hepatocytes were at the stage of a giant-drop adiposal dystrophy. What is the most
possible diagnosis?
@Alcoholic cirrhosis
Chronic active alcoholic hepatitis
Chronic persistent alcoholic hepatitis
Toxic dystrophy of the liver
Adiposal hepatosis
#
37
Finely nodular (micronodular) cirrhosis, as illustrated in the photomicrograph below, is seen in all the following
conditions EXCEPT
@postnecrotic cirrhosis
secondary biliary cirrhosis
primary biliary cirrhosis
alcoholic cirrhosis
hemachromatosis
#
38
A 42-year-old male, with a history of rheumatic stenosis of the left atrioventricular orifice resulted in chronic heart
insufficiency, presented at the hospital with dyspnea at small physical exercises, tachycardia, a cyanosis of his lips,
bubbling rales in the inferior segments of lungs, leg's edema. What histological changes will be characteristic for a liver?
@Necrosis of hepatocytes in the center of lobule, fatty dystrophy on periphery
Necrosis of hepatocytes in the center of lobule, hyalin dystrophy on periphery
Necrosis of hepatocytes in the center of lobule, hydropic dystrophy on periphery
Fatty dystrophy of hepatocytes in the center of lobule, necrosis on periphery
Hydropic dystrophy of hepatocytes in the center of lobule, necrosis on periphery
#
39
An autopsy of a female, with a history of a rheumatic disease with a mitral stenosis, resulted in decompensation stage,
revealed a general venous plethora, small, firm and micronodular liver. Gross investigation of a cut surface showed a
lobular structure and yellowish-brown color of hepatic tissue. Define the most probable pathology diagnosed in a liver.
@Portal cirrhosis
Fatty dystrophy
Billiary cirrhosis
Postnecrotic cirrhosis
Atrophy
#
40
A 62-year old patient, with a history of chronic virus hepatitis, died of an acute posthemorrhagic anaemia resulted from an
esophageal varicose veins bleeding. An autopsy revealed the reduced, dense liver with coarsely scarred surface. A
microscopical investigation showed regenerative false nodules separated by fibrous tissue, contained the remnants of
portal tracts. What morphogenetic type of cirrhosis took place in this case?
@Portal cirrhosis.
Postnekrotic cirrhosis.
Mixed cirrhosis.
Viral cirrhosis.
Biliary cirrhosis.
#
41
A patient with chronic viral hepatitis died of acute posthemorrhagic anaemia because of the haemorrhage from the
varicous dilation of esophagus veins. At the section the liver is hardly diminished, of solid consistence; its surface is
micro tuberous. Microscopically: thin-stitched net of connective tissue and small pseudo lobules. Name the
morphogenetic type of cirrhosis.
Postnecrotic
@Portal
Mixed
Viral
Biliary
#
42
A patient complains of frequent diarrheas, especially after consumption of rich food, weight loss. Laboratory examination
revealed steatorrhea; his feces were hypocholic. What might have caused such condition?
Unbalanced diet
@Obturation of biliary tracts
Lack of pancreatic lipase
Lack of pancreatic phospholipase
Inflammation of mucous membrane of small intestine
#
43
A 62-year old patient, with a history of a cholelithiasis accompanied with cholangitis and cholangiolitis, presented to the
hospital liver cirrhosis. What is the most likely type of cirrhosis presented?
@Biliary
Infection
Toxins
Nutritional deficiency
Circulatory
#
44
Am 38-year-old woman complains of fatigue and pruritus. she is found to have high serum alkalaine phosphatase and
slightly elevated serum bilirubin levels, and serum antimitochondrial antibodies are present. A liver biopsy reveals a
marked lymphocytic infiltrate in the portal tracts along with occasional granulomas. The most likely diagnosis is
inpacted gallstone
@Primary biliary cirrosis
primary sclerosing cholangitis
von Meyenburg's complex
Caroli's disease
#
45
A patient presented to the hospital an ascites, twice enlarged spleen, a varicose esophagogastric veins and veins around
and within the rectum. A histological investigation of liver biopsy revealed micronodular cirrhosis. What process has
complicated cirrhosis?
@Portal hypertension
Cardiac insufficiency
Hepatotenal syndrome
Hepatic-cellular insufficiency
-
#
46
A 45-year-old male alcoholic with a history of portal hypertension presents with vomiting of blood (hematemesis) and
hypotension. He denies any history of vomiting nonblood material or retching prior to vomiting blood. During work-up
he dies suddenly. Which of the following histologic changes is most likely to be seen in a biopsy specimen taken from his
esophagus?
Metaplastic columnar epithelium
Decreased ganglion cells in the myenteric plexus
@Dilated blood vessels in the submucosa
Mucosa outpouchings
Numerous intraepithelial neutrophils
#
47
A 62-year-old man with hepatic failure secondary to cirrhosis develops a pungent odor in his breath (fetor hepaticus). He
is also noted to have marked ascites, gynecomastia, asterixis, and palmar erythema. His serum ammonia levels are found
to be elevated. Which of the following is the basic defect that caused this patient's gynecomastia?
Decreased synthesis of albumin
Defective metabolism of the urea cycle
Deranged bilirubin metabolism
@Impaired estrogen metabolism
The formation of mercaptans in the gut
#
48
A 48-year-old man with fatigue is being evaluated for a 1-year history of elevated serum liver enzymes. A liver biopsy is
taken and the pathology report of this specimen states there is grade 2 inflammatory activity with piecemeal necrosis and
stage 1 fibrosis. The term "piecemeal necrosis" refers to which one of the following pathologic abnormalities?
Congo red-positive extracellular deposits surrounding necrotic hepatocytes in acinar zone 1
@Destruction of the limiting plate with necrosis of hepatocytes surrounding the portal triad
Fibrosis around the central hepatic veins with apoptosis of adjacent hepatocytes
Necrosis of hepatocytes extending from the portal area of one hepatic lobule to the central vein of an adjacent lobule
Random necrosis of individual or small clusters of hepatocytes in acinar zone 3
#
49
A 2-year-old girl is being evaluated for strikingly yellow skin and is found to have elevated serum levels of indirect
bilirubin. After appropriate work-up the diagnosis of type II Crigler-Najjar syndrome is made. She is then treated with
phenobarbital, which causes hyperplasia of the smooth endoplasmic reticulum in hepatocytes and decreases the serum
indirect bilirubin levels. Which of the following enzymes is most likely to be deficient in this child?
Aspartate aminotransferase
@Bilirubin-UDP-glucuronyl transferase
Galactosylceramide beta-galactosidase
Gamma-glutarnyl transpeptidase
L-iduronosulfate sulfatase
#
50
A 44-year-old man presents with the sudden onset of severe right i quadrant (RUQ) abdominal pain, ascites, tender
hepatomegaly, and hematemesis. These symptoms are suggestive of Budd-Chiari syndrom. Which of the following is the
most likely cause of this disorder?
Obctruction of the common bile duct
Obctructionof the intrahepatic sinusoids
Thrombosis of the hepatic artery
@Thrombosis of the hepatic vein
Thrombosis of the portal vein
#
51
A full-term normal male infant develops a slight yellow color to li skin on his second day of life. Laboratory examination
finds his serum bilirubin levels to be slightly elevated (due to increased indirect bilirubin), but the levels are less than 6
mg/dL. Additionally, serum hemoglobin levels are within normal limits. By the fifth day his bilirubin levels have returned
to normal levels and the abnormal yellow skin color has disappeared. No therapy was given to this infant. Which of the
following is the most likely cause of these signs and symptoms?
Crigler-Najjar syndrome
Hemolytic disease of the newbom
Inspissated bile syndrome
Inirauierine varicella infection
@Physiologic jaundice of the newbom
#
52
An autopsy is performed on a 19-year-old woman who died fruin oi overdose of acetaminophen. Which of the following
hisiologic changes is most likely to be seen in a biopsy specimen taken from her liver?
@Centrilobular necrosis
Focal scattered necrosis
Geographic necrosis
Midzonal necrosis
Periponal necrosis
#
53
A 51-year-old male alcoholic with a history of chronic liver disease presents with increasing weight loss and ascites.
Physical examination reveals a slightly enlarged, soft, nontender prostat. Examination of the scrotum is unremarkable,
and fecal occult blood tests are negative. A chest X-ray is unremarkable, but a CT scan of the abdomen reveak a single
mass in the left lobe of the liver. Work-up reveals elevated levels of a-fetoprotein in this patient's blood. Which of the
following is the most likely diagnosis?
Angiosarcoma
Cholangiocarcinoma
Hepatoblastoma
@Hepatocellular carcinoma
Metastatic colon cancer
#
54
A 36-year-old man presents with jaundice and pruritus. Physical examination finds a diffuse yellow discoloration to his
skin. Laboratory examination reveals markedly elevated serum levels of alkaline phosphatase, but neither antinuclear nor
antimitochondrial antibodies are present. A liver biopsy revealed reactive hepatocytes and fibrosis in the sinusoids. The
portal tracts showed marked fibrosis around the bile ducts, but no granulomas were seen. While waiting for a liver
transplant he developed a malignancy and dieWhich of the following tumors is most closely associated with his liver
disease?
@Cholangiocarcinoma
Gallbladder carcinoma
Gastric carcinoma
Hepatoblastoma
Pancreatic carcinoma
#
55
A 26-year-old presents with right upper quadrant abdominal pain and is found to have a large cyst in the right lobe of his
liver. X-rays reveal the cyst to have a calcified wall. The cyst is then surgically excised. Examination of this tissue
histologically reveals a thick, acellular, laminated eosinophilic wall. The fluid within the cyst is found to be granular and
contain numerous small larval capsules with scoleces ("brood capsules"). Which of the following is the most likely
diagnosis?
Pyogenic liver abscess
Amebic liver abscess
@Hydatid cyst
Schistosomiasis
Oriental cholangiohepatitis
#
56
A 40-year-old man was admitted to the surgical department with spleen rupture. What anatomic formation will
accumulate the blood?
@Bursa pregastrica
Right lateral canal
Hepatic bursa
Rectovesical excavation
Omental bursa
#
57
A 27-year-old man has purulent inflammation of gallblader. What region of the peritoneal cavity will the pus fall into if
gallblader ruptures in his typical position?
@Into hepatic bursa
Into epiploic bursa
Into the superior duodenal sinus
Into the left lateral duct
Into pancreatic bursa
#
58
Obturative jaundice developed in a 60-year-old patient because of malignant tumour of the big papillary of the duodenal.
Lumen of what anatomical structure is squeezed with tumour?
Left hepatic duct
Cystic duct
Right hepatic duct
Common hepatic duct
@Hepatopancreatic ampulla
#
59
A 54-year-old male alcoholic presents with the sudden onset of severe, constant epigastric pain that radiates to his
midback. Further evaluation finds fever, steatorrhea, and discoloration around his flank and umbilicus. Laboratory tests
find elevated seram levels of amylase and lipase.Which of the following is the most likely diagnosis?
Acute appendicitis
Acute cholangitis
Acute cholecystitis
Acute diverticulitis
@Acute pancreatitis
#
60
After clinical examination of the patient surgeon supposed an acute pancreatitis. What biochemical test can confirm this
supposition?
@Activity of alpha-amylase in blood and urine
Activity of aminotraspherases in blood and urine
Activity of creatin phosphokinase in blood
Activity of alkaline phospatase in blood
Activity of lactate dehydrogenase 1 in blood
#
61
A 12-year-old nonobese boy presents for evaluation after becoming sick at school. Pertinent recent medical history
includes weight loss with polyphagia, polydipsia, and polyuria. Laboratory examination finds hyperglycemia, while
urinary examination reveals increased glucose and trace ketones. Which of the following abnormalities is most likely to
be present in this boy?
Amyloid deposition in the pancreatic islets
Atrophy and destruction of the pancreatic acini
Decreased numbers of insulin receptors on adipocytes
@Lymphocytic infiltration in the pancreatic islets
Mutations in the gene that codes for hexokinase
#
62
A 48-year-old male alcoholic presents with malaise, fever, and mid abdominal pain that radiates to his back. Pertinent
medical history includf-i repeated bouts of pancreatitis that mainly occur after times of binge drink ing. Physical
examination finds a low-grade fever, and a mass is palpated in the epigastric area. An abdominal CT scan finds a fluid-
filled mass in thr pancreas. This mass is removed at celiotomy and has a similar appearance to the cystic mass shown in
the photograph below. It is filled with clear fluid, and histologic sections reveal a large cystic structure that lacks an
epithelial lining. Which of the following is the most likely diagnosis?
Cylindroma
Hydrocystoma
@Pseudocyst
Pseudomyxoma
Syringoma
#
63
A 44-year-old woman presents with repeated episodes of feeling "light-headed" that are associated with sweating and a
feeling like she is about to faint. She says that she feels better if she drinks some orange juice and eats a candy bar during
one of these episodes. Physical examination is unremarkable, but laboratory examination finds decreased serum levels of
glucose along with elevated levels of glucose along with elevated levels of insulin. The combination of hypoglycemia,
symptoms of hypoglycemia, and symptoms of hypoglycemia relieved by glucose is the definition of which of the
following clinical triads?
Beck's triad, as seen with acute tamponade
Charcot's triad, as seen with acute inflammation of the gallbladder
Marchiafava's triad, as seen with overwhelming sepsis
Virchow's triad, as seen with carcinoma of the exocrine pancreas
@Whipple's triad, as seen with a beta-cell tumor of the endocrine pancreas
#
Glomerulopathy
#
1
A 58-year-old patient with acute cardiac insufficiency has decreased volume of daily urine - oliguria. What is the
mechanism of this phenomenon?
Rise of hydrostatic blood pressure in capillars
@Decreased glomerular filtration
Reduced permeamility of renal filter
Decreased number of functioning glomerules
Drop of oncotic blood pressure
#
2
A 16 year-old patient got numerous traumas in automobile accident. Now the patient is haning a shock. АP - 80/60 mm
Hg. daily urine volume 60-80 ml. What pathogenic mechanism leads to kidneys function violation?
Increased vasopressin blood concentration
Increased osmotic pressure in glomerular capillaries
Increased pressure in Bowman’s capsule
Trauma of the urinary bladder
@Decreased hydrostatic pressure in glomerular capillaries
#
3
As a result of long-term starvation the glomerular filtration of a man was accelerated by 20%. The most probable cause of
filtration changes under such conditions is:
Rise of systemic arterial pressure
Growth of filtration coefficient
Increase of renal plasma flow
Increased permeability of renal filter
@Fall of oncotic pressure of blood plasma
#
4
A 24-years-old patient has edema of face and increasing of BP, which appeared in 1.5 weeks after severe
streptococcus tonsillitis. The patient has hematuria and proteinuria of 1.2 g/L. Anti-streptococcus antibodies and
decrease in content of components of compliment system were revealed in patient’s blood. Which microvessels are
deposits of immune complexes localized in that case of nephropathy?
@Glomerule
Proximal tubules
Descendent tubules
Loop of Henle
Pyramids
#
5
A girl of 5-years-old has admitted to nephrologic department of a municipal hospital. She complains of pain in low back,
urine contains constantly raised amount of red blood cells (erythrocytes), proteins (albumins and globulins). Which from
the following renal structures is destroyed?
@Glomerulus of nephron
Proximal convoluted tubule of nephron
Distal convoluted tubule of nephron
Mesangium of glomerulus
Collecting tubule of nephron
#
6
A 30 year old woman has face edemat A. Examination revealed proteinuria (5,87 g/l), hypoproteinemia, dysproteinemia,
hyperlipidemi A. What condition is the set of these symptoms typical for?
Chronic renal failure
Acute renal failure
@Nephrotic syndrome
Chronic pyelonephritis
Nephritic syndrome
#
7
Which of the following is the most likely cause of the clinical combination of generalized edema, hypoalbuminemia, and
hypercholesterolemia in an adult whose urinalysis demonstrated marked proteinuria, with fatty casts and oval fat bodies?
Nephritic syndrome
@Nephrotic syndrome
Acute renal failure
Renal tubular defect
Urinary tract infection
#
8
A physical examination of a young woman revealed marked edemas and high proteinuria. A histological investigation of
kidneys biopsy determined a disappearance of podocytes, a declining of heparansulfate in a basal membrane of
glomerular capillaries. What is the most likely disease?
@Idiopathic nephrotic syndrome
Postinfectious glomerulonephritis
Rapidly progressive glomerulonephritis
Chronic glomerulonephritis
Acute necrotic nephrosis
#
9
A 5-year-old girl with a history of an acute respiratory virus infection presented with widespread swelling, a massive
proteinuria, a hypoalbuminemia, a lipidemia. A microscopic investigation of the renal biopsy revealed an absence of
small podocytes processes within vascular glomerulus. What is the most probable diagnosis?
@Lipoid nephrosis
Postinfectious glomerulonephritis
Rapidly progressive glomerulonephritis
Focal segmentalglomerulosclerosis
Membranous glomerulonephritis
#
10
A 28-year-old man with AIDS presents with moderate proteinuria and hypertension. Histologic sections of the kidney
reveal the combination of normal-appearing glomeruli and occasional glomeruli that have deposits of hyaline material.
No increased cellularity or necrosis is noted in the abnormal, glomeruli. Additionally, there is cystic dilation of the renal
tubules, some of which are filled with proteinaceous material. Electron microscopy reveals focal fusion of podocytes, and
immunofluorescence examination finds granular IgM/C3 deposits. Which of the following is the most likely diagnosis?
Diffuse proliferative glomerulonephritis (DPGN)
Focal segmental glomerulonephritis (FSGN)
@Focal segmental glomerulosclerosis (FSGS)
Membranous glomerulopathy (MGN)
Minimal change disease (MCD)
#
11
A 2-year-old boy is being evaluated for peripheral edema that developed shortly after he recovered from an upper
respiratory infection. Physical examination finds that he is afebrile, and his blood pressure is within normal limits.
Laboratory examination finds decreased serum albumin, increased serum cholesterol, and normal BUN and creatinine
levels. Examination of his urine finds massive proteinuria and lipiduria, but no red blood cells are seen. The loss of
albumin in the urine is much greater than the loss of globulins. A histologic section from a renal biopsy examined with a
routine H&E stain is unremarkable. Which of the following pathologic changes is most likely to be seen with electron
microscopic examination of a renal biopsy from this child?
Diffuse thinning with fragmentation of the basement membrane
@Fusion of the foot processes of the podocytes
Large irregular subendothelial electron-dense deposits
Ribbon-like electron-dense deposits in the basement membrane
Small uniform subepithelial electron-dense deposits
#
12
In a 25 year-old female patient with severe edemas, hyperproteinuria, hyperlipidemia the biopsy of kidney established:
well-developed thickening of the glomerular capillary wall with the presence of electron-dense immunoglobulin-
containing deposits along the epithelial surface of the basement membrane. Epithelial cells lost their foot processes.
Described changes are characteristic for…
@Membranous glomerulonephritis
Acute Poststreptococcal glomerulonephritis
Crescentic glomerulonephritis
Chronic glomerulonephritis
Acute pyelonephritis
#
13
Marked thickening of the glomerular basement membrane, as shown in the photomicrograph below, may be seen in
lipoid nephrosis
@membranous glomerulonephritis
Goodpasture's syndrome
acute pyelonephritis
chronic glomerulonephritis
#
14
All the following renal disorders are assocated with the nephrotic syndrome EXCEPT
membranous glomerulonephritis
lipoid nephrosis
membranoproliferative glomerulonephritis
@acute tubular necrosis
focal segmental glomerulosclerosis
#
15
A 58-year-old patient with a history of purulent osteomyelitis died of chronic renal insufficiency. A post-mortem revealed
the enlarged firm kidneys of white-yellow color with a sebaceous (waxy) cut surface. What is the most likely diagnosis?
@Amyloidosis of kidneys
Chronic glomerulonephritis
Subacute glomerulonephritis
Septic nephrite
Acute necrotic nephrosis
#
16
A post-mortem of a patient with a history of chronic renal insufficiency revealed the enlarged,firm, sebaceous (waxy)
kidneys with multiple retractions on their surface. Histological investigation showed many glomeruli were replaced by the
Congo-Red positive masses. The same substance was found on the basal membranes of capillaries, in mesangium and
also within arterial walls and kidneys stroma. What of the listed diagnoses the most likely?
@Amyloidosis of kidneys
Acute glomerulonephritis
Chronic glomerulonephritis
Subacute glomerulonephritis
Lipoid nephrosis
#
17
The young man with a history of bronchiectasis disease since the early childhood died of renal insufficiency. A post-
mortem revealed in lungs plural dilated bronchi and bronchial tubes, filled with a purulent exudate. Besides, the enlarged
kidneys had a dense texture, their cortical layer was thickened, white color and dense. Kidney pyramids were anemic and
accurate. Name process which has developed in kidneys?
@Secondary amyloidosis
Glomerulonephritis
Chronic pyelonephritis
Congenital kidney cysts
Secondary nephrosclerosis
#
18
A 35-year-old woman recovering from hepatitis B develops hematuria, proteinuria, and red cell casts in the urine. Which
one of the following statements best describes the expected renal changes in this patient?
Plasma cell interstitial nephritis
IgG linear fluorescence along the glomerular basement membrane
@Granular deposits of antibodies in the glomerular basement membrane
Diffuse thickening of the glomerular basement membrane by subepithelialimmune deposits
Nodular hyaline glomerulosclerosis
#
19
An adult medical laboratory technician recovering from hepatitis B develops hematuria, proteinuria, and red cell casts in
the urine. Which of the following would best describe the changes within the kidney in this patient?
Plasma cell interstitial nephritis
IgG linear fluoresence along the glomerular basement membrane
@Granular deposits of antibodies in the glomerular basement membrane
Diffuse thickening of the glomerular basement membrane by subepithelial immune deposits
nodular hyaline glomerulosclerosis
#
20
A 54-year-old patient with a history of a diabetes mellitus died of chronic renal insufficiency. A post-mortem revealed all
uremia symptoms and kidneys alterations. What are most probable microscopical renal changes in that case?
@Hyalinosis and sclerosis of capillaries of glomerulus.
Spasm of afferent glomerular arteriole.
Hyaline cast in the lumen of renal tubules.
Necrosis of renal tubular epithelium.
Necrosis of renal cortex.
#
21
An 8-year-old girl, with a history of acute tonsillitis 3 weeks before the application, presented with nephrotic syndrome
(proteinuria, haematuria and cylindruria). These testify the glomerular basal membrane's lesion. What is the mechanism
of the basal membrane pathology?
@Immune complex mechanism
Granulomatous mechanism
Antibody-mediated mechanism
Reagin-mediated mechanism
Cytotoxical mechanism
#
22
A 12-year-old boy with a history of acute tonsillitis two weeks ago presented with edematous face in the mornings,
increasing of arterial pressure, urine in a kind of “meat slops” (“coca-cola” color). Immunohistochemistry of kidneys
biopsy revealed immune complexs on basal membranes of capillaries and within glomerular mesangium. What disease
has developed at the patient?
@Acute glomerulonephritis
Acute interstitial nephrite
Lipoid nephrosis
Acute pyelonephritis
Necrotic nephrosis
#
23
A 12-year-old child has developed nephritic syndrome (proteinuria, hematuria, cylindruria) after angina of 2 weeks ago.
In a biopsy of kidney it was established: diffuse proliferation of endothelial and mesangial, epithelial cells in glomeruli,
infiltration with leukocytes, both neutrophils and monocytes, interstitial edema. Described changes are characteristic
for…
@Acute poststreptococcal glomerulonephritis
Membranous glomerulonephritis
Chronic glomerulonephritis
Acute necrotic nephrosis
Crescentic glomerulonephritis
#
24
The photomicrograph below shows evidence of glomerular fibrin deposition. This histopathology is a supplemental
finding in a 2-year-old child who was a history of abdominal pain and bloody diarrhea, followed by acute
glomerulonephritis, Coombs'-negative severe hemolytic anemia, and renal failure. The likely diagnoses might include
lupus erythematosus
acute poststreptococcal glomerulonephritis
lipoid nephrosis
@hemolytic-uremic syndrome
bacterial endocrditis
#
25
Acute poststreptococcal glomerulonephritis usually does all the following EXCEPT
follow infection with group A beta-hemolytic streptococci
@follow a streptococcal infection in less than 5 days
have a better prognosis in children than in adults
show low serum complement levels (hypocomplementemia)
exhibit elevation of antistreptolysin O (ASO) titer
#
26
A 7-year-old boy presents with bilateral swelling around his eyes. His parents state that the child's eyes have become
"puffy" over the past several weeks, and his urine has become cocoa-colored. Physical examination reveals bilateral
periorbital edema, but peripheral edema is not found. The boy is afebrile and his blood pressure is slightly elevated. A
urinary dipstick reveals mild proteinuria, while microscopic examination of the boy's urine reveals hematuria with red
blood cell casts. Laboratory tests reveal increased ASO liters and decreased serum C3 levels, but C2 and C4 levels are
normal. A throat swab for streptococci is negativA microscopic section from the kidney reveals increased numbers of
cells within the glomeruli. An electron microscopic section of the kidney reveals large electron-dense deposits in the
glomeruli that are located between the basement membrane and the podocytes. The foot processes of the podocytes are
otherwise unremarkable. Which of the following is the most likely diagnosis?
@Acute poststreptococcal glomerulonephritis
Focal segmental glomerulonephritis
Focal segmental glomerulosclerosis
Membranous glomerulonephritis
Minimal change disease
#
27
A 43-year-old man with a history of microscopic polyarteritis acutely develops renal failure with oliguria and hematuria.
Laboratory examination reveals the presence of serum p-ANCA (antineutrophil cytoplasmic antibodies). A renal biopsy is
diagnostic of type III rapidly progressive glomerulonephritis. Which of the following histologic changes is most likely to
have been present in this biopsy specimen?
Eosinophilic masses were seen attached to the capsule of Bowman's space
Fibrinoid necrosis was present in many of the afferent arterioles
Large numbers of neutrophils were seen in the intersritium and tubules
@Numerous crescents were present in the glomeruli
The basement membrane was seen to be split by mesangial cells
#
28
A patient died from uremia. A post- mortem revealed enlarged flaccid kidneys with wide, swallowed with red specks
cortex. Medulla had a dark red coloring. Microscopic investigation showed epithelial crescents which compressed
capillaries. Nephrocytes exhibited a dystrophy; a stromal edema and infiltration were also recognized. What is the most
likely diagnosis?
@Glomerulonephritis
Pyelonephritis
Nephrolithiasis
Nephrotic syndrome
Amyloidosis of kidneys
#
29
A patient died with uremia. The autopsy demonstrated kidneys to be enlarged, of flabby consistence, cortex was broad,
edematous, with red spots; medulla was dark red in colour. Microscopic examination in the glomeruli capsule cavity
revealed "semilunar formations", squeezing capillaries, nephrocytes dystrophy, edema and infiltration of stroma. What
disease caused the death of the patient?
Renal amyloidosis
Pyelonephrosis
Nephrolithiasis
Nephrotic syndrome
@Glomerulonephritis
#
30
An autopsy of the 58-year-old patient revealed an enlarged kidneys, which had a well defined yellow-grey with red specs
cortical layer. Kidney's medulla colored in a dark red. Histological investigation revealed proliferation of capsular
epithelial cells and podocytes with crescents formation, sclerosis and hyalinosis of the glomerular tufts, stromal fibrosis.
What is the most likely kidneys disease?
@Extracapillary productive glomerulonephritis
Intracapillary productive glomerulonephritis
Extracapillary exudative glomerulonephritis
Intracapillary exudative glomerulonephritis
Interstitial nephrite
#
31
At the section were revealed the following changes in kidneys: kidneys were enlarged in size; the cortex was wide,
yellow-gray with red drops. It was separated from medulla in a good way. The medulla was of cherry color. Histological
exam revealed the overgrowth of nephrothelium and podocytes in the capsule of glomeruli with the formation of
crescents, the facts of sclerosis and hyalinosis of glomeruli, the fibrosis of the stroma. What disease is meant?
Interstitial nephritis
Intercapillary productive glomerulonephritis
Extracapillary exudative glomerulonephritis
Intercapillary exudative glomerulonephritis
@Extracapillary productive glomerulonephritis
#
32
Histological investigation revealed proliferation of Bowman’s capsul epithelial cells, podocytes and macrophages,
resulted in crescents formation, which compressed dlomeruli. Necrotic capillary loops had fibrinous thrombi within their
lumens. A sclerosis and hyalinosis of some glomeruli were detected. In addition, the expressed nephrocytes dystrophy, an
edema and stromal infiltration of kidneys was observed. What is the most likely kidneys pathology?
@Rapidly progressive glomerulonephritis
Postinfection glomerulonephritis
Chronic glomerulonephritis
Chronic pyelonephritis
Amyloidosis of kidneys
#
33
The young man presented in hospital with a headache and giddiness. Within last year he ofthen had high arterial pressure.
Medical drugs almost did not help. Kidneys biopsy investigation revealed the extra capillary epithelial proliferation with a
crescent formation. For what disease described pathology is characteristic?
@Rapidly progressive glomerulonephritis
Acute glomerulonephritis
Lupus glomerulonephritis
Wegener's granulomatosis
Chronic glomerulonephritis
#
34
At the microscopical exam of the kidneys was noted the proliferation of the nephrothelium in Bowman’s capsule, of the
podocytes and macrophages with the formation of crescent structures that compress the glomerulus.
Capillaries of glomeruli are affected with necrosis; fibrinous thrombi can be seen in the lumens of capillaries. Some of
glomeruli are affected with sclerosis or hyalinosis. The dystrophy of nephrocytes, oedema and infiltration of the nephral
stroma are observed, too. Name the pathology of kidneys.
@High-progressive glomerulonephritis
Postinjectional glomerulonephritis
Chronic glomerulonephritis
Chronic pyelonephritis
Renal amyloidosis
#
35
A 28 year old patient had high arterial pressure, hematuria and facial edemat A. In spite of treatment renal insufficiency
was progressing. 6 months later the patient died from uremia. Microscopic examination of his kidneys and their
glomerules revealed proliferation of capsule nephrothelium and of podocytes with "demilune" formation, sclerosis and
hyalinosis of glomerules. What disease corresponds with the described picture?
@Subacute glomerulonephritis
Acute pyelonephritis
Chronic glomerulonephritis
Nephrotic syndrome
Acute glomerulonephritis
#
36
Microscopical renal examination of a 36 y.o. woman who died from renal insufficiency revealed in the glomerules
proliferation of capsule nephrothelium as well as of podocytes and phagocytes accompanied by formation of "crescents",
capillary loop necrosis, fibrinous thrombs in their lumens; sclerosis and hyalinosis of glomerules, atrophy of tubules and
fibrosis of renal strom a. What is the most probable diagnosis?
Membranous nephropathy
Acute glomerulonephritis
Focal segmentary sclerosis
Chronic glomerulonephritis
@Subacute glomerulonephritis
#
37
A 36-year-old woman died of renal insufficiency. Microscopical exam revealed the proliferation of the nephrothelium in
the capsule, of the podocytes and macrophages with the formation of crescents. There was also necrosis of capillary
stitches with fibrinous thrombi in their lumens, sclerosis and hyalinosis of glomeruli, atrophy of the tubules and fibrosis
of the nephral stroma. What is the most possible diagnosis?
Membranous nephropathy
Acute glomerulonephritis
Chronic glomerulonephritis
Focal segmental sclerosis
@Subacute glomerulonephritis
#
38
A 42-year-old man died of renal insufficiency. Microscopic investigation revealed a proliferation of Bowman's capsule
epithelial cells, podocytes and macrophages, resulted in crescents formation, necrosis of capillary loops and fibrin
thrombi within their lumens. In addition, the histological study showed sclerosis and hyalinosis of glomeruli, tubular
atrophy and stromal fibrosis of kidneys. What of the listed diagnoses the most likely?
@Subacute glomerulonephritis
Acute glomerulonephritis
Chronic glomerulonephritis
Focal segmental sclerosis
Membranous nephropathy
#
39
A 48-year-old woman died of renal insufficiency. A post- mortem revealed enlarged flaccid kidneys with wide,
swallowed, dim cortical layer. A yellow-grey with red specks cortex was delimited from dark red cerebral substance.
Microscopic investigation showed a proliferation of Bowman's capsule epithelial cells, podocytes and macrophages,
resulted in crescents formation. For what disease described changes are characteristic?
@Subacute glomerulonephritis.
Acute glomerulonephritis.
Chronic glomerulonephritis.
Acute pyelonephritis.
Amyloidosis of kidneys.
#
40
A man died of renal insufficiency. A post- mortem revealed enlarged flaccid kidneys with wide, yellow-grey with red
specks cortex. Microscopic investigation showed a proliferation of Bowman's capsule epithelial cells, resulted in
crescents formation. Capillary loops exhibited necrotic alterations and fibrin thrombi within their lumens. What is the
most likely diagnosis?
@Subacute glomerulonephritis
Acute glomerulonephritis
Lipoid nephrosis
Chronic glomerulonephritis
Amyloidosis of kidneys
#
41
Nephrobiopsy is performed to 30-year-old women. Histological investigation revealed a proliferation of mesangial cells, a
swelling and proliferation of capillary endothelial cells, enlargement mesangial matrix, is diffusive a thickening of the
glomerular basal membrane, mild tubular-interstitial component. Electron-microscopic research showed interposition of
mesangium, a diffusive and irregular thickening of a glomerular basal membrane. What form of a glomerulonephritis has
developed in that case?
@Mesangiocapillary glomerulonephritis.
Mesangial proliferative glomerulonephritis
Membranous glomerulonephritis.
Rapidly progressive glomerulonephritis.
Postinfectious
42
The elderly man, with 15 years history of a membranous -proliferative glomerulonephritis, constantly received
hemodialysis therapy. The last half a year he did not receive a treatment. He presented to the hospital with extremely
severe condition, without consciousness, with a smell of urea from his body and edemas. A marked pleuritis, pericarditis
and peritonitis were also diagnosed. In a whole, all symptoms were regarded as a uremia. What kind of an inflammation
is the most probable?
@Fibrinous inflammation
Purulent inflammation
Catarrhal inflammation
Hemorrhagic inflammation
Serous inflammation
#
43
A 47-year-old man presents with increasing peripheral edema and dark, tea-colored urinLaboratory examination finds
decreased serum albumin, while examination of a 24-h urine specimen reveals marked proteinuria. Microscopic
examination of this patient's urine reveals numerous red cells along with rare red cell casts. Electron microscopic
examination of a renal biopsy from this patient reveals dense, ribbon-like deposits in the lamina densa of the glomerular
basement membrane. Which of the following is the most likely diagnosis?
Acute glomerulonephritis
IgA nephropathy
Lipoid nephrosis
@Membranoproliferative glomerulonephritis
Membranous glomerulopathy
#
44
A linear pattern of immunoglobulin deposition along the glomerular basement membrane that can be demonstrated by
immunofluorescence is typical of
lupus nephritis
diabetic glomerulopathy
@Goodpasture's syndrome
Goldblatt's fidney
renal vein trombosis
#
45
A 28-year-old man with a history of malaise and hemoptysis presents with the acute onset of renal failure. Laboratory
examination reveals increased serum creatinine and BUN, but no antineutrophil cytoplasmic antibodies (ANCA) nor
antinuclear (ANA) antibodies are present. Urinalysis reveals the micrpscopic presence of red blood cells and red blood
cell casts, while a renal biopsy reveals crescents within Bowman's space of many glomeruli. Immunofluorescence reveals
linear deposits of IgG and C3 along the glomerular basement membrane. Which of the following is the most likely
diagnosis?
Alpon syndrome
Diabetic glomerulopathy
@Goodpasture's syndrome
Henoch-Schonlein purpura
Wegener's granulomaiosis
#
46
A 26-year-old woman presents with increasing fatigue and malaisShe states that recently she develops a red facial rash
whenever she goes outside on a sunny day. Physical examination finds that she is afebrile, but her blood pressure is
slightly increased and slight peripheral edema is founLaboratory evaluation finds slightly elevated BUN and creatinine,
while dipstick examination of her urine reveals slight proteinuria with microscopic hemaiuria. Very rare granular and red
cell casts are seen. Laboratory examination is also positive for serum antinuclear antibodies, one of which is anti-double-
stranded DNA renal biopsy reveals changes ol diffuse proliferative glomerulonephritis, and the diagnosis of class IV
lupus nephritis is made. Which of the following histologic changes is most characteristic of class IV lupus nephritis?
Mesangial deposits form a "holly leaf" pattern
Positive immunofluorescence staining forms a "string of popcorn" pattern
Splitting of the basement membrane forms a "tram-track" pattern
Thickening of the basement membrane forms a "spike and dome" appearance
@Thickening of the glomerular capillaries forms a "wire-loop" appearance
#
47
Chronic glomerulonephritis was diagnosed in a 34-year-old patient 3 years ago. Edema has developed in the last 6
monthes. What caused it?
Hyperaldosteronism
Hyperproduction of vasopressin
Hyperosmolarity of plasma
Disorder of albuminous kidneys function
@Proteinuria
#
Tubulopathy
#
1
All the following renal disease cause hypertension EXCEPT
@small, bilateral renal infarction
renal artery arteriosclerosis
fibromuscular dysplasia of the renal artery
hydronephrosis
pyelonephritis
#
2
Physical examination of a 3-day-old male infant reveals urine leaking from the area of the umbilicus. Which of the
following is the most likely diagnosis?
Balanoposthitis
Meckel's cyst
Meckel's diveniculum
Omphalocele
@Urachal fistula
#
3
A sexually active man who has had a negative evaluation for gonococcal infection and who complains of persistent
dysuria but no other symptoms should be considered to have
prostatic hypertrophy
epididumitic
orchitis
@nonspecific uretritis
renal stones
#
4
A 35-year-old woman during her first pregnancy develops oligohy-dramnios. At 34 weeks of gestation she delivers a
stillborn infant with abnormal facial features consisting of wide-set eyes, low-set floppy ears, and a broad-flat nose.
Which of the following abnormalities is most likely to be present in this still-born infant?
Absence of the thymus
@Bilateral renal agenesis
Congenital biliary atresia
Cystic renal dysplasia
Urinary bladder exstrophy
#
5
During the complicated labour the symphysis pubis ruptured. What organ can be damaged the most?
Rectum
@Urinary blader
Uterus
Uterine tubes
Ovaria
#
6
Acute tubular necrosis (ATN) is a fairly common renal lesion and is associated with all the following EXCEPT
@red cell casts in the urine
proteinuria
proximal tubular damage in toxic ATN
oliguria
acute renal failure
#
7
Shock and signs of acute renal failure (ARF) developed in the patient due to severe injury. What is the leading cause of
development of ARF in the case?
Increased pressure in the renal arteries
@Decreased arterial pressure
Urine excretion violation
Decreased oncotic BP
Increased pressure in the nephron capsule
#
8
A 50-year-old man had been blocked up for 2 days because of earthquake. After he was rescued, the diagnosis of
compression syndrome was made. What further complication is the most likely to develop in this case?
Acute vascular failure
Acute cardiac failure
@Acute renal failure
Acute hepatic failure
Acute respiratory failure
#
9
Violation of safety rules resulted in calomel intoxication. Two days later the daily diuresis was 620 ml. A patient
experienced headache, vomiting, convulsions, dyspnea, moist rales in lungs. What pathology is it?
Chronic renal insufficiency
@Acute renal insufficiency
Glomerulonephritis
Uraemic coma
Pyelonephritis
#
10
A patient with the signs of mercuric poisoning had the following processes in kidneys: focal necrotic changes in tubules
of the main departments, oedema, leukocytic infiltration and haemorrhages of the interstice and venous congestion. What
patient’s condition have developed?
Acute glomerulonephritis
@Acute necrotic nephrosis
Chronic renal insufficiency
Acute pyelonephritis
Chronic pyelonephritis
#
11
An autopsy of a patient, who died from poisoning of ethylene glycol, revealed enlarged edematous kidneys. Their capsule
was taken out very easily. Gross investigation showed the wide pale-grey cortical layer and dark red medulla. What
disease has developed at the patient?
@Necrotic nephrosis
Acute pielonephritis
Acute glomerulonephritis
Acute tubulointerstitial nephritis
Lipoid nephrosis
#
12
The kidney biopsy is taken from the man who is in clinic with symptoms of a mercury poisoning. Histological
investigation revealed focal tubular necrosis, an edema, hyperemia, hemorrhages and leukocytic infiltration in a stroma.
What condition has developed at the patient?
@Acute necrotic nephrosis
Acute glomerulonephritis
Chronic renal insufficiency
Acute pyelonephritis
Chronic pyelonephritis
#
13
A 42-year-old man, who had a complicated form of enteric fiver, acute nephral insufficiency developed of which he died.
At the autopsy: kidneys are enlarged in size, oedematous, fibrous capsule can be easily removed; on the incision the
cortex is pale-gray, pyramids are cherry. Histological exam reveals mostly narrow lumen of the tubules; epithelial cells
are enlarged in size without nuclei; glomeruli are collapsed; stroma is oedematous with small leukocytic infiltration and
small haemorrhages. What pathology is meant?
Pyonephrosis
Acute pyelonephritis
Acute glomerulonephritis
@Necronephrosis
Hydronephrosis
#
14
At the patient operated concerning a widespread tumour of an abdominal cavity, the considerable bleeding and declining
of arterial pressure took place. After operation the acute renal insufficiency, resulted in the patient's death, developed. An
autopsy revealed the wide light pink cortical layer, which was clearly delimited from dark red pyramids. Histological
investigation showed the absence of nuclei in epithelium of convoluted tubules, tubulorrhexis, venous hyperemia. Nuclei
of the cells of vascular glomeruli and straight tubules were unchanged. What pathology of kidneys has developed at the
patient?
@Necronephrosis
Infarct
Glomerulonephritis
Pyelonephritis
Nephrosis
#
15
Histologic sections of a kidney reveal patchy necrosis of epithelial cells of both the proximal and distal tubules with
flattening of the epithelial cells, rupture of the basement membrane (tubulorrhexis), and marked interstitial edema. Acute
inflammatory cells are not seen. Which of the following is the most likely diagnosis?
Acute pyelonephritis
@Acute tubular necrosis
Chronic glomerulonephritis
Chronic pyelonephritis
Diffuse conical necrosis
#
16
A 49-year-old patient died of a massive bleeding from an operating wound. A post-mortem revealed enlarged kidneys
with strained fibrous capsule. A gross investigation of a cut surface determined wide, pale grey color cortical layer
accurately delimited from cyanotic, dark red pyramids. Histological study showed a necrosis and a marked dystrophy of
an epithelium convoluted tubules, tubulorrhexis.What condition has developed in kidneys?
@Acute necrotic nephrosis.
Postinfectious glomerulonephritis
Lipoid nephrosis.
Rapidly progressive glomerulonephritis.
Secondary amyloidosis of kidneys.
#
17
A young man died of a burn disease. A post-mortem revealed the brain edema, liver and kidneys enlargement. Gross
investigation showed the thickened pale-grey cortical layer and hyperemic medulla. Histological study demonstrated focal
tubular necrosis at different points along the nephron with a disruption of tubular basement membrane interstitial edema
with leukocytic infiltration and a hemorrhage. What of the listed diagnoses the most authentic?
@Necrotic nephrosis.
Tubulointerstitial nephritis.
Pyelonephritis.
Gouty kidney.
Myeloma kidney.
#
18
Histological study of a renal biopsy revealed a necrosis and a marked dystrophy of the convoluted tubules epithelium,
tubulorrhexis, stromal edema and hyperemic medulla. Name the pathological process?
@Necrotic nephrosis
Glomerulonephritis
Pyelonephritis
Cystic disease
Kidney stone disease
#
19
At the section of a man, who died of burn disease, the brain edema, the enlargement of the liver and kidneys were
revealed. The cortex of the last is wide and pale-gray, but the medulla is sanguineous. Microscopically: necrosis of the
tubules of main departments with the destruction of basal membranes, oedema of the interstitium with leukocytic
infiltration and haemorrhages. What is the most possible diagnosis?
Tubulointerstitial nephritis
@Necrotic nephrosis
Pyelonephritis
Podagra kidney
Myeloma kidney
#
20
On the 6th day of treatment a patient with acute renal insufficiency developed polyuria. Diuresis intensification at the
beginning of polyuria stage of acute renal insufficiency is caused by:
Growth of natriuretic factor
Volume expansion of circulating blood
Reduction of vasopressin content in plasma
Reduction of aldosteron content in plasma
@Renewal of filtration in nephrons
#
21
The patient treated for a cold by the big doses of paracetamol, presented symptoms of an oliguria and azotemia. In 5 days
he died of an acute renal insufficiency. Histological investigation of kidneys revealed the diffusive edema of an interstitial
tissue of a kidneys cortex, its infiltration by lymphocytes, the eosinophils, some neutrophils. Almost intact glomerules
were found along with a destruction of a tubular epithelium. Name the most probable type of a kidneys lesion?
@Tubulointerstitial nephritis
Acute glomerulonephritis
Nephrotic syndrome
Pyelonephritis
Necronephrosis
#
22
A 49-years-old- patient with a 10 days history of dysentery presented with raised body temperature, a pain in lumbar area,
a fever and a considerable quantity of leucocytes in urine. Histological investigation of a puncture renal biopsy revealed
hyperemia, interstitial leukocytic infiltration and numerous inflammatory cells (mainly neutrophiles) with a desquamated
epithelium, filling renal tubules. What complication has developed at the patient?
@Acute pyelonephritis
Chronic pyelonephritis
Pyelitis
Glomerulonephritis
Necrotic nephrosis
#
23
A macroscopic investigation of the removed kidney reveled renal swallowing, congestion and a capsule easily taken out.
Renal pelvis and calyxes were dilated, filled by turbid urine. Their mucosa was dim, with the areas of hemorrhages. The
cut surface of a kidney had a motley pattern; yellow-grey zones were surrounded by plethora and hemorrhages areas. To
what disease there corresponds such macroscopic picture of kidneys?
@Acute pyelonephritis
Acute glomerulonephritis
Amyloidosis of kidneys
Nephrolitiasis
Renal cystic disease
#
24
A 60-year-old woman died of a chronic renal failure. An autopsy revealed the irregularly contracted and markedly
lobulated kidneys with a fibrous tissue alternated with an intact parenchyma. Renal pelvises were dilated, their walls -
thickened. Histological investigation showed sclerosis, lymphocyte - plasmocyte infiltration of the renal pelvis walls, and
interstitium. What diagnosis is most probable?
@Chronic pyelonephritis
Acute pyelonephritis
Acute glomerulonephritis
Tubulointerstitial nephritis
Chronic glomerulonephritis
#
25
Histological investigation of a kidney biopsy showed sclerosis, lymphocyte – plasmocyte infiltration of the renal pelvis
walls and calyces; tubules dystrophy and atrophy. The intact tubules were dilated, stretched by colloid-like masses. The
epithelium was flattened. In whole, the microscopic picture of tissue sample had 'a thyroid kidney' pattern. What
diagnosis is most probable?
@Chronic pyelonephritis
Sharp pyelonephritis
Glomerulonephritis
Nephrosclerosis
Tubulointerstitial nephritis
#
26
A post-mortem of a patient with a long history of cystitis and a dyskinesia of ureters revealed morphological signs of
uremia. Kidneys were contracted, with irregularly scarred surface. In the renal pelvis fine urate stones and sand were
registered. Histological investigation showed 'the thyroid kidney' and areas of an interstitial inflammation. What is the
most likely diagnosis?
@Chronic pyelonephritis
Acute pyelonephritis
Atherosclerotic-contracted kidney
Primary- contracted kidney
Amyloidal- contracted kidney
#
27
At the section of a woman, who died of renal insufficiency, the kidneys are unevenly reduced in size and have surface
with big tubers; on the incision the areas of cicatricial tissue change with the unchanged parenchyma, renal pelves are
widened with thinned walls. Microscopically in the walls of renal pelves, calices and in the interstice are the facts of
sclerosis and lymphoplasmocytic infiltration. What is the most possible diagnosis?
Acute glomerulonephritis
Acute pyelonephritis
@Chronic pyelonephritis
Tubulointerstitial nephritis
Chronic glomerulonephritis
#
28
At the biopsy material from a kidney were detected: sclerosis, lymphoplasmocytic infiltration of renal pelves and calices;
dystrophy and atrophy of the tubules. Saved tubules were widened, outstretched by colloid-like masses; the epithelium
was squashed (disk kidney). What is the most possible diagnosis?
Nephrosclerosis
Acute pyelonephritis
Glomerulonephritis
@Chronic pyelonephritis
Tubulointerstitial nephritis
#
29
A postmortem of a 58-year-old male revealed in kidneys asymmetrical, coarse, corticomedullary scars. Histological
investigation of a kidney showed atrophy of cyst-like dilated tubules, filled with colloid eosinophilic masses. Interstitium
was characterized by pronounced periglomerular sclerosis. What diagnosis is most probable?
@Chronic pyelonephritis with scarring
Chronic glomerulonephritis with scarring of kidneys
Amyloidosis of kidneys
Tuberculosis of kidneys
Kidneys at hypertension
#
30
An autopsy of a 59- year-old man revealed the coral-like stone filled all renal pelvis of a right kidney. The kidney was
enlarged, hyperemic with the capsule, taken out hardly. The pelvis and calyxes cavities were dilated, filled by a turbid,
greenish- yellowish viscous liquid. The mucosal layer was dim with some hemorrhages. A cut surface investigation
showed motley pattern of the kidney's tissue, with yellow spots in up to 1cm in diameter. What complication to
nephrolithiasis has developed?
@Chronic pyelonephritis with exacerbation
Tumour of kidney
Postinfectious glomerulonephritis
Primary amyloidosis
Rapidly progressive glomerulonephritis.
#
31
A 67-year-old woman presents with signs of slowly progressive renal failure. Physical examination reveals mild
hypertension, while laboratory tests find increased serum creatinine and BUN with mild proteinuria. A few scattered
neutrophils and rare bacteria are seen microscopically in her urine. Grossly both of her kidneys are small and irregular
with dilation of the renal pelvis and clubbing of the calyces, these changes being similar in appearance to those seen in the
picture below. Histologic sections from her kidneys revealed chronic inflammation of the interstitium. Which one of the
following additional microscopic findings should be present in these histologic sections?
The renal papillae should have numerous activated macrophages
@The tubules should be dilated and filled with colloid casts
The afferent and efferent arterioles should demonstrate fibroelastic hyperplasia
The glomeruli should have multiple fibrin microthrombi
The interstitium should have needle-shaped crystals
#
32
The kidneys of a patient, who died of chronic renal failure caused by chronic glomerulonephritis, are reduced in size, of
solid consistence, the capsule can be hardly removed, exposing granular surface. On the incision cortex and medulla are
thinned; renal tissue is dry, anaemic, of gray color. How can we term such kidney?
Atherosclerous-wrinkled
Primary wrinkled
@Secondary wrinkled
Amyloid-wrinkled
Pyelonephritic-wrinkled
#
33
A 53-year-old patient with a history of a chronic glomerulonephritis died of chronic renal insufficiency. A post-morten
revealed smaller that normal, firm kidneys. Their capsule was taken out hardly, baring a granular renal surface. On a cut
section a cortical and cerebral layers were thin with dry, anemic, grey color kidneys tissue. How is called such kidneys?
@Secondary - scarring kidneys
Primary - scarring kidneys
Atherosclerotic - scarring kidneys
Amyloidal - scarring kidneys
Pyelonephrotis - scarring kidneys
#
34
An autopsy of the 68-year-old man revealed a sallow-grey colour of his skin with petechial hemorrhages. His face and
tongue had a powdery 'uremic frost'. Fibrinous hemorrhagic laryngitis, a tracheitis, a shaggy pericardium and a catarrhal
gastroenterocolitis were also diagnosed. For what syndrome the given complex of morphological changes is
characteristic?
@Chronic renal insufficiency
Acute hepatic insufficiency
Acute renal insufficiency
Chronic heart insufficiency
-
#
35
For a long time a 49-year-old woman had suffered from glomerulonephritis which caused death. The autopsy revealed
that the size of her kidneys was 7х3х2,5 sm, weight 65,0 g, they were dense and small-grained. Microscopically:
fibrinogenous inflammation of serous and mucous capsules, dystrophic changes of parenchymatous organs, brain edema.
What complication can cause such changes of serous capsules and inner organs?
Sepsis
DIC-syndrome
Anemia
@Uraemia
Thrombopenia
#
36
A post-mortem of a 52- year-old man, with a long history of chronic glomerulonephritis, resulted in his death, revealed
markedly reduced, firm, fine-grained kidneys. Microscopical investigation showed fibrinous inflammation of serous and
mucosas, dystrophic changes of parenchymatous organs and a brain edema. What complication has led to the serous and
mucosas lesions?
@Uremia
Anaemia
Sepsis
DIC-syndrome
Thrombocytopenia
#
37
A 55 year-old man has died after chronic glomerulonephritis, chronic renal insufficiency. In the autopsy the pathologist
has found out characteristic changes in kidneys for this disease, also fibrinous pericarditis, pleuritis, bronchitis. Call the
cause of the fibrinous inflammation in serosal and mucosal layers.
@Uremia
Hypolipidemia
Hyperlipidemia
Arterial hypertension
Arterial plethora
#
38
A 27-years-old woman has been suffered by diabetes mellitus since she had been a child. During the last years the arterial
hypertension and proteinuria have occurred. She has died with signs of uremia. What typical changes were found out in
autopsy?
@“Cor villosum”
“Tiger heart”
“Armor heart”
Mesenchymal fat dystrophy of the heart
Purulent pericarditis
#
39
All the following statements are true of urinary calculi EXCEPT
they are common in males
@they are bilateral in 40 percent of cases
they are radiopaque in about 90 percent of cases
they may be associated with Pseudomonas infection
the incidence is increased in leukemia
#
40
A 35-year-old woman presents with the sudden onset of severe, colicky pain on the right side other abdomen. She does
not relate the pain to food, but says that she cannot find a pain-free position. Physical examination finds marked
tenderness over the right costovenebral angle, but rebound tenderness is not present. A pelvic examination is
unremarkablMicroscopic examination of her urine reveals the presence of numerous red blood cells. The urine is negative
for esterase and nitrite, and no bacteria are seen. Which of the following is the most likely cause of her signs and
symptoms?
Choledocholithiasis
Cholelithiasis
Cystitis
@Nephrolithiasis
Pyelonephritis
#
41
A 65 year old man suffering from gout complains of kidney pain. Ultrasound examination revealed renal calculi. The
most probable cause of calculi formation is the strengthened concentration of the following substance:
Bilirubin
Cystine
Cholesterol
@Uric acid
Urea
#
42
A patient with urolithiasis after the examination was administered allopurinol - competitive inhibitor of xanthine oxidase.
It was influenced by chemical analysis of the calculuses, which consisted mainly of:
Calcium sulphate
Calcium oxolate monohydrate
Calcium oxolate dihydrate
Calcium phosphate
@Sodium urate
#
43
During the morphological examination of a removed kidney was detected the presence of a concrement that obturated the
lumen of the proximal part of ureter. The kidney is hardly enlarged with the parenchyma atrophy; renal pelves and calyx
are hardly widened, too. Microscopically: diffuse sclerosis, atrophy of glomeruli and tubules; saved tubules are cyst
widened. What complication of urolithiasis appeared in a patient?
@Hydronephrosis
Pyonephrosis
Pyelonephritis
Glomerulonephritis
Chronic paranephritis
#
44
A macroscopical investigation of a removed kidney revealed in proximal area of an ureter the concrement which
obturated its lumen. The kidney was markedly enlarged; a parenchyma was atrophic with considerably dilated pelvis and
calyces. Microscopical study of kidney's sample showed the diffusive sclerosis, an atrophy of glomeruli and tubules.
Survived tubules were cystically dilated. What complication of a nephrolithiasis developed at the patient?
@Hydronephrosis.
Pyonephrosis.
Pyelonephritis.
Glomerulonephritis.
Chronic paranephritis.
#
45
A postmortem of a 65-year-old patient with a history of the adenoma of a prostate revealed big kidneys with markedly
dilated pelvis and calyces filled with a transparent liquid. Name the kidneys pathology.
@Hydrohephrosis
Glomerulonephritis
Amyloidosis
Tuberculosis
Pyelonephritis
#
46
An autopsy revealed substantial enlargement of a right kidney. A gross investigation of a renal cut section showed a stone
in it with marked dilation of the pelvis and calyces by urine and thinning of the renal parenchyma. What of diagnoses is
the most likely?
@Hydronephrosis.
Pyelectasia
Pyelonephritis.
Cyst of kidney
-
#
47
An autopsy revealed marked enlargement of a kidney. Gross examination of a kidney's section presented with dilation of
both renal pelvis and calyces resulted from renal stones. What from diagnoses is most faithful?
@Hydronephrosis
Simple cysts
Pyelonephritis
Benign nephrosclerosis
Polycystic kidney disease
#
48
At autopsy a male is found to have big kidneys with marked dilatation of the renal pelvis and calyces filled with
transparent liquid. A thinning of the renal parenchyma accompanied above lesions. Which is the most likely finding?
@Hydronephrosis
Glomerulonephritis
Amyloidosis
Tuberculosis
Pyelonephritis
#
49
An 8-month-old male infant presents with progressive renal and hepatic failure. Despite intensive medical therapy, the
infant dies. At the time of autopsy, the external surfaces of his kidneys are found to be smooth, but cut section reveals
numerous cysts that are lined up in a row. Which of the following is the mode of inheritance of this renal abnormality?
Autosomal dominant
@Autosomal recessive
X-linked dominant
X-linked recessive
Mitochondrial
#
50
An autopsy of a 44-year-old female revealed a substantial enlargement of a right kidney reminding grape clusters. The cut
surface showed cavities varying in size from 0, 5 to 3 cm in diameter, which were filled by serous liquid and colloid
masses. Kidney parenchyma between cavities was thinned to 0,1 cm. What is the most likely diagnosis?
@Renal cystic disease
Chronic pyelonephritis
Acute pyelonephritis
Nephrolithiasis
Dysphasia of kidneys
#
51
A post-mortem revealed the enlarged kidneys with a lobulated surface due to plural cavities with the smooth wall, filled
with a transparent liquid. What is the most likely disease?
@Renal cystic disease
Necrotic nephrosis
Pyelonephritis
Glomerulonephritis
Infarct
#
52
On autopsy it is revealed that kidneys are enlarged, surface is large-granular because of multiple cavities with smooth
wall, which are filled with clear fluid. What kidney disease did the patient have?
@Polycystic kidney
Infarction
Necrotic nephrosis
Glomerulonephritis
Pyelonephritis
#
53
At the section: the kidneys are enlarged in size; their surface is with big tubers on account of the presence of numerous
cavities with plain walls and bright fluid. What disease is meant?
@Polycystosis
Necrotic nephrosis
Pyelonephritis
Glomerulonephritis
Infarction
#
54
A middle-aged man comes to you with the single presenting symptom of occasional hematuria of very recent oncet. The
most probable cause is
acute pyelonephritis
nephroblastoma
renal cell carcinoma
mesoblastic nephroma
@renal pelvic urothelial tumor
#
55
All the following statements are true regarding transitional cell carcinoma of the bladder EXCEPT that
it is more common in men than in woman
@it is associated with infection by Schistosoma haematobium
it is associated with cigarette smoking
it shows increased incidence in aniline dye workers
it tends to recur after excision, regardless of grade
#
56
A 49-year-old man who is long-term smoker present with frequency and hematuria. Histologic examination of section
taken from an exophytic lesion of the urinary bladder reveals group of atypical cells with frequent mitises forming finger-
like projections that have thin, fibrovascular cores. These groups of atypical cells do not extend into the lamina propria
and muscularis. No glands or keratin production are found. Which of the following is the most likely diagnosis?
Adenocarcinoma, noninvasive
Inverted papilloma, noninvasive
Transitional cell carcinoma in situ
Papillary transitional cell carcinoma, noninvasive
Squamous cell carcinoma in situ
#
57
All the following adnormalities are associated with an increased incidence of Wilms'tumor EXCEPT
aniridia
male pseudohermaphroditism
hemihypertrophy
renal medullary cysts
@hypoplasia of radii
#
Reproductive system pathology
#
1
A 32-year-old patient has been diagnosed with bartholinitis (inflammation of Bartholin's glands). In what part of the
female urogenital system are the Bartholin's glands located?
The labia minor
@The labia major
The uterus
The clitoris
The vagina
#
2
A 75-year-old woman presents with a pruritic vulvar lesion. Physical examination reveals an irregular white, rough area
involving her vulv. Which one of the following histologic changes is most consistent with the diagnosis of lichen
sclerosis?
@Atrophy of the epidermis with dermal fibrosis
Atypia of the epidermis with dysplasia
Hyperplasia of the epidermis with hyperkeratosis
Invasion of the epidermis by individual malignant cells
Loss of pigment in the basal layers of the epidermis
#
3
Primary malignant neoplasms of the vagina include all the following EXCEPT
sarcoma botryoides
clear cell carcinoma
squamous carcinoma
@vaginal adenosis
rhabdomyosarcoma
#
4
Vaginal adenosis precedes the development of which of the following?
Condyloma acuminatum
Cervical carcinoma
@Clear cell carcinoma
Carcinoma of the endometrium
Squamous carcinoma of the vagina
#
5
A 25-year-old woman being evaluated for infertility is found to have an abnormal ridge of red, moist granules located in
the upper third of her vagin. Pertinent medical history is that her mother was treated with diethylstilbestrol (DES) during
her pregnancy. A biopsy from the abnormal vaginal ridge reveals the presence of benign glands underneath stratified
squamous epithelium. Which of the following is the most serious long-term complication of this abnormality?
@Clear cell carcinoma
Condyloma acuminatum
Extramammary Paget's disease
Multiple papillary hidradenomas
Verrucous carcinoma
#
6
A 30-year-old woman, with a history of a chronic endocervicitis, passed routine inspection at the gynecologist with no
complaints. A histological investigation of a biopsy, taken from the pars vaginalis neck of the uteri, revealed the high
cylindrical epithelium, secreting mucus and branched out glands underneath. What is the most likely diagnose?
@Endocervicosis
Squamous metaplasia
Glandular metaplasia
Leukoplakia
Erythroplakia
#
7
A histological investigation of curettage of the cervical canal of the uteri revealed a prismatic epithelium and a significant
amount of gland-like structures. They developed from the prismatic epithelium cambial elements, originated from the
neck of a uterus. What is the most likely diagnose?
@Proliferative endocervicosis
Simple endocervicosis
Endocervicosis, stage of healing
Cervical adenomatosis
Cervical polyps
#
8
A gynecologic investigation of the 36-year-old woman revealed bright red brilliant maculae on a uterus neck. They bleed
easily at a touch. Biopsy investigation showed that a tissue sample was covered by a cylindrical epithelium with papillary
growths. There was also a growth of glands within the thickness of a uterus neck tissue. What pathology of a uterus neck
was diagnosed?
@Pseudoerosion
Cervical erosion
Endocervicitis
Glandular hyperplasia
Leukoplakia
#
9
Which of the following describes multiple small mucinous cysts of the endocervix that result from blockage of the
endocervical glands by overlying squamous metaplastic epithelium?
Bartholin's cysts
Chocolate cysts
Follicular cysts
Gartner's duct cysts
@Nabothian cysts
#
10
The pathologist has found out in histological examination of a remote uterus a lot of glandular formations with syngl
cycts in myometrium., endometrium had usual structure. Described changes are characteristic for
@Endometriosis.
Glandular hyperplasia of endometrium.
Adenocarcinoma of uterus
Adenomatosis of a mucosa of a uterus
Leyomyosarcoma of uterus.
#
11
A 25-year-old woman presents to your office for workup of infertility. In giving a history she describes severe pain during
menses, and she also tells you that in the past another doctor told her that she had "choco late in her cysts." Which of the
following abnormalities is most likely to be present in this patient?
Metastalic ovarian cancer
@Endometriosis
Acute pelvic inflammatory disease
Adenomyosis
A posteriorly located subserosal uterine leiomyoma
#
12
All the following endometrial changes are consistent with secretory endometrium EXCEPT
basal cytoplasmic vacuoles
secretions within glandular lumen
predecidual refction within stroma
neutrophil infiltrate
@plasma cell infiltrate
#
13
A 24-year-old woman presents with a 2-year history of infertility. An endometrial biopsy is obtained approximately 5 to 6
days after the predicted time ofovulation. This biopsy specimen reveals secretory endometrium, but there is a significant
difference (asynchrony) between the estimated chronologic menstrual date and the estimated histologic menstrual datNo
prolif-erative endometrium is seen. Which of the following is the most likely diagnosis?
Anovulatory cycle (no corpus luteum formed)
@Inadequate luteal phase (decreased functioning of the corpus luteum)
Irregular shedding (prolonged functioning of the corpus luteum)
Normal endometrium during the follicular phase of the cycle (no corpus luteum formed)
Normal endometrium during the luteal phase of the cycle (normal corpus luteum)
#
14
A 23-year-old woman presents with urinary frequency and abnormal uterine bleeding. A careful medical history finds that
her abnormal menstrual bleeding is characterized by excessive bleeding at irregular intervals. A pelvic examination finds
a single mass in the anterior wall of the uterus, this being confirmed by ultrasonography. Which one of the following clin -
ical terms best describes the abnormal uterine bleeding in this woman?
Amenorrhea
Dysmenorrhea
@Menometrorrhagia
Oligomenorrhea
Polymenorrhea
#
15
A 50-year-old woman presents with fatigue, insomnia, hot flashes, night sweats, and absence of menses for the last 5
months (secondary amenorrhea). Her urine hCG test is negativLaboratory tests reveal decreased serum estrogen and
increased serum FSH and LH levels. Which of the following is the most likely cause of this individual's clinical signs and
symptoms?
17-hydroxylase deficiency of the adrenal cortex
Prolaccin-secreting tumor of the anterior pituitary
Gonadotropin-releasing hormone-secreting tumor of the hypothalamus
@Menopause
Menarche
#
16
Histological exam of the uterine mucosa reveals anfractuous glands (sew- and bottlescrew-like), elongated overgrowths of
stroma with proliferation of its cells. What is your diagnosis?
Placental polyp
Acute endometritis
Leiomyoma
Molar pregnancy
@Glandular hyperplasia of endometrium
#
17
A microscopic investigation of an endometrium revealed coiled extended glands with 'saw' and a 'spin-like' pattern. A
stromal proliferation with hyperplasia of its cells was also determined. What is the most likely diagnosis?
@Glandular hyperplasia of endometrium
Acute endometritis
Leiomyoma
Hydatidiform mole
Placental polyp
#
18
A 45-year-old woman with a medical history of a menstrual cycle disturbance had curettage of the uterine cavity to obtain
endometrial tissue for histopathologic analysis. Microscopic investigation revealed the increased quantity of the
endometrial glands, their form irregularity. Some of glands were dilated or had a saw-like pattern. What is the most likely
diagnosis?
@Cystic glandular hyperplasia of endometrium
Placental polyp
Atypical hyperplasia of endometrium
Glandular polyp of endometrium
Endometrial adenocarcinoma
#
19
A 42-year-old woman presented with dysfunctional uterine bleeding. A histological investigation of a curettage material
revealed increased quantity of coiled glands and their cystic dilation. What is the most likely diagnosis?
@Cystic glandular hyperplasia
Atrophy
Metaplasia
Displasia
Hypertrophic vegetations
#
20
Cystic hyperplasia of the endometrium is associated with all the following EXCEPT
occurence at or just before menopause
increased estrogen administration or production
excessive uterine bleeding
@secretory cells lining the cystically bilated glands
functioning granulosa-theca cell tumors
#
21
A 60-year-old postmenopausal woman presents with the new onset of uterine bleeding. An endometrial biopsy is
diagnosed as atypical hyper-plasi. Which of the following histologic changes is most characteristic of this abnormality?
@Crowding of endometrial glands with budding and epithelial atypia
Lymphatic invasion by interlacing bundles of atypical spindle-shaped cells
Menstrual-type endometrial glands with focal atypical cystic dilatation
Secretory-type endometrial glands with hyperplasia of atypical polygonal cells clear cytoplasm
Stromal invasion by malignant glands with focal areas of atypical squamous differentiation
#
22
A 52-year-old woman, with a medical history of the menstrual cycle disturbances and hemorrhages, had curettage of the
uterine cavity. Histological investigation revealed the increased quantity of the endometrial glands. Many glands had a
morphological pattern of a proliferation phase, clumped together and had intraglandular papillary protrusions. Some
glandular cells exhibited the signs of nuclear and cytoplasm polymorphism. What is the most likely diagnosis?
@Atypical hyperplasia of endometrium
Cystic glandular hyperplasia of endometrium
Glandular hyperplasia of endometrium
Glandular polyp of endometrium
Adenocarcinoma of endometrium
#
23
A 46-year-old woman undergoes an abdominal hysterectomy for a "fibroid" uterus. The surgeon requests a frozen section
on the tumor, which is deferred because of the lesion's degree of cellularity. Which of the fol lowing criteria will be used
by the pathologist in determining behignancy versus malignancy in permanent sections?
@Milotic rate
Cell pleomorphism
Cell necrosis
Nucleus-io-cytoplasm ratio
Tumor size
#
24
Prolonged unopposed estrogen stimulation in an adult woman increases the risk of development of endometrial
hyperplasia and subsequent carcinomWhich of the following is the most common histologic appearance for this type of
cancer?
@Adenocarcinoma
Clear cell carcinoma
Small-cell carcinoma
Squamous cell carcinoma
Transitional cell carcinoma
#
25
A 23-year-old woman presents to her gynecologist for a routine physical examination that includes a Pap smear. Her
sexual history includes many sexual partners beginning at an early age, but she has never been pregnant. Physical
examination is unremarkabl. The Pap smear returns as abnormal with the presence of atypical squamous epithelial cells of
undetermined significance (ASCUS). She returns for a 6-month follow-up and a repeal pelvic exam is performe. Her
cervix is painted with iodine and an area near the cervical os is present that does not stain with iodine. This area is flat and
not papillary. Several biopsies are obtained from this pale area, and a representative histologic section shows poilocytosis,
which is most characteristic of infection with which one of the following organisms?
Cytomegalovirus
Epstein-Barr virus
Herpes simplex virus
@Human papillomavirus
ParvovirusBlO
#
26
An 18-year-old woman presents with amenorrhea and is found to, have normal secondary sex characteristics and normal-
appearing external genirali. Her first menstrual period was at age 13, and her cycle has been unremarkable until now. She
states that her last menstrual period was 8 weeks prior to this visit. A urine test for hCG is positive. Which of the
following is the most likely diagnosis?
Ectopic pregnancy
@Intrauterine pregnancy
Stein-Leventhal syndrome
Turner's syndrome
Weight loss syndrome
#
27
A 24-year-old woman delivers a normal 8-lb baby boy at 40 weeks of gestation. She has no history of drug abuse, and her
pregnancy was unremarkablExamination had revealed the placenta to be located normally, but following delivery the
woman fails to deliver the placenta and subsequently develops massive postpartum hemorrhage and shock. Emergency
surgery is performed to stop the bleeding. Which of the following is the most likely cause of her postpartum bleeding?
An abruptio placenta
A placenta previa
@A placenta accreta
A hydatidiform mole
A chorioncarcinoma
#
28
A 29-year-old woman presented to maternity department with termination of pregnancy at 20 weeks of gestation. The
fetus and fetal membranes were delivered with some blood clots. Histological investigation revealed fetal membranes,
chorion villi and decidual tissue. Name -the described pathology of pregnancy.
@Spontaneous complete abortion
Premature birth
Hydatidiform mole
Invasive hydatidiform mole
Artificial abortion
#
29
A 26-year-old woman acutely develops lower abdominal pain arid vaginal bleeding. While in the bathroom she passes a
cast of tissue composed of clot material and then collapses. She is brought to the hospital, where a physical examination
reveals a soft, tender mass in right adnexa and pouch of Douglas. Histologic examination of the tissue passed in the
bathroom reveals tilood clots and decidualized tissuNo chorionic villi or trophoblastic tissues are present. Which of the
following conditions is most likely present in this individual?
Aborted intrauterine pregnancy
Complete hydatidifbrm mole
@Ectopic pregnancy
Endometrial hyperplasia
Partial hydatidiform mole
#
30
A 23-year-old woman presents with her menstrual cycle disorder went trough a transvaginal puncture of Douglas’ cul-de-
sac resulted in aspiration of blood and some other material. A microscopic investigation of the puncture fluid revealed
blood, some decidual cells and chorion villi. What is the most likely diagnosis?
@Ectopic tubal pregnancy
Salpingitis
Rupture of Fallopian tube
Hydatidiform mole
Carcinoma uteri
#
31
A 35-year-old woman had an operation of uterine tube removal at gynecology department. A histological research of a
tube's wall reveled chorion villi and decidual cells. What is the most likely diagnosis?
@Tubal pregnancy
Placental polyp
Choriocarcinoma
Paper-doll fetus
Lithopedion
#
32
A young woman presented to the hospital with an acute pain. Her uterine tube was removed at gynecology department.
Macroscopic investigation of a tube revealed its local dilation at the middle third and blood clots accumulations within the
lumen. A histological research showed chorion villi, surrounded by areas of erythrocytes with some leucocytes. What is
the most likely diagnosis?
@Tubal pregnancy
Acute purulent salpingitis
Hemorrhage in tube of the uteri
Hemorrhagic salpingitis
Tuberculous salpingitis
#
33
A 39 y.o. woman went through an operation in course of which surgeons removed her uterine tube that was enlarged and
a part of an ovary with a big cyst. Histological examination of a tube wall revealed decidual cells, chorion villi. What was
the most probable diagnosis made after examination of the uterine tube?
Lithopedion
Choriocarcinoma
Papyraceous fetus
@Tubal pregnancy
Placental polyp
#
34
At the operation of a 39-year-old woman were removed an enlarged in size uterine tube and a part of ovary with a big
cyst. Histological exam of the mucous membrane of the tube wall revealed decidual cells and villi of chorion. What is the
most possible diagnosis?
Choriocarcinoma
Placental polyp
@Tubal pregnancy
Papyraceous fetus
Lithopedion
#
35
A 25-year-old woman with a medical history of recent childbirth presented to gynecology department with metrorrhagia.
The histological investigation of a curettage material revealed a pathologic growth, consisted of fibrin clots, chorion villi
and decidual tissue, undergoing organization. What is the most likely diagnosis?
@Placental polyp
Glandular hyperplasia of the endometrium
Hydatidiform mole
Choriocarcinoma
Endometritis
#
36
A woman harboring endometrial adenocarcinoma nearly always has antecedent
obesity
diabetes millitus
@endometrial polyps
endometrial hyperplasia
systemic hypertention
#
37
A 25-year-old woma in her 15th week of pregnancy presented with uterine bleeding and passage of a small amount of
watery fluid and tissue. She is found to have a uterus that is mach larger than estimated by her gestetional dates. Her
uterus is found to be filled with cystic, avascular, grapelike structures that do not penetrate the uterine wall. No fetal parts
are found. The most likely diagnosis for this adnormality is
partial hydatidiform mole
@complete hydatidiform mole
invasive mole
placental site trophoblastic tumor
chorioncarcinoma
#
38
A patient had an operation relatively uterine tumour. Macroscopically: a spongy mottled nodule in the myometrium.
Histological exam reveals giant bright epithelial cells, among which many dark polymorphous ones are noted. Stroma is
absent, vessels look like lumens covered with tumour cells. Multiple haemorrhages are detected, too. How can we term
the determined tumour?
@Chorionepithelioma
Destructive (malignant) molar pregnancy
Adenocarcinoma
Cavernous hemangioma
Medullary cancer
#
39
6 months after labour a woman had uterine hemorrhag E. Gynaecological examination of uterine cavity revealed a dark-
red tissue with multiple cavities resembling of a "sponge". Microscopic examination of a tumour revealed in blood
lacunas atypic light epithelial Langhans cells and giant cells of syncytiotrophoblast. What tumour is it?
Fibromyoma
Squamous cell nonkeratinous carcinoma
@Chorioepithelioma
Adenocarcinoma
Cystic mole
#
40
A 26-year-old woman in the third trimester of her first pregnancy develops persistent headaches and swelling of her legs
and facEarly during her pregnancy a physical examination was unremarkable; however, now her blood pressure is
170/105 mmHg and urinalysis reveals slight proteinuriWhich of the following is the most likely diagnosis?
Eclampsia
Gestational trophoblastic disease
Nephritic syndrome
Nephrotic syndrome
@Preeclampsia
#
41
A- post-mortem of a pregnant woman revealed a brain edema, hemorrhagic pneumonia, motley liver with a hemorrhage.
Microscopic study showed disseminated blood clotting of vessels, plural fine necroses and hemorrhages in internal
organs. In addition, in kidneys there were found cortical necroses and a fibrinoid necrosis of a tubules epithelium within a
nephron.
What is the most likely diagnosis?
@Eclampsia
Pre-eclampsia
Septicopyemia
Septicemia
Acute renal insufficiency
#
42
A 29-year-old woman presents with severe pain during menstruation (dysmenorrhea). During workup, an endometrial
biopsy is obtaineThe pathology report from this specimen makes the diagnosis of chronic endometritis. Based on this
pathology report, which of the following was present in the biopsy sample of the endometrium?
Neuirophils
Lymphocytes
Lymphoid follicles
@Plasma cells
Decidualized stromal cells
#
43
A 48-year-old woman with a grease bloody flux from her genital tracts had a diagnostic curettage of her uterus cavity.
Microscopical research of the uterus curettage material revealed a thinning of a mucosal layer, reduction of the
endometrial glands number, a fibrosis of a stroma and some lymphoid cells infiltration. What is the most likely diagnosis?
@Chronic atrophic endometritis
Acute purulent endometritis
Chronic cystic endometritis
Chronic hypertrophic endometritis
-
#
44
A 32-year-old woman died in post delivery period. At the autopsy were revealed the signs of suppurative endometritis,
purulent thrombophlebitis of venae uterinae, numerous abscesses of lungs, abscesses of kidneys and spleen, abscessing
myocarditis and purulent meningitis. Diagnose the type of sepsis.
@Septicopyemia
Septicemia
Chroniosepsis
Prolonged septic endocarditis
Tuberculous sepsis
#
45
A 25-year-old woman presents with lower abdominal pain, fever, and a vaginal discharge. Pelvic examination reveals
bilateral adnexal (ovarian) tenderness and pain when the cervix is manipulated. Cultures taken from the vaginal discharge
grow Neisseria gonorrhoea. Which of the following is the most likely cause of this patient's adnexal pain?
Adenomatoid tumor
Ectopic pregnancy
Endometriosis
Luteoma of pregnancy
@Pelvic inflammatory disease
#
46
A 33-year-old woman gave birth to a dead fetus with gestational age more than 43 weeks. The fetal skin was dry and
chap. An autopsy revealed a general hypotrophy and nuclei of ossification (Beclard's nuclei) in a proximal epiphysis of
tibial and humeral bones. Amniotic fluid, an umbilical cord and placental membranes were dyed by meconium. Name the
period of the described perinatal pathology?
@The antenatal period
The prenatal period
The intranatal period
The postnatal period
The progenesis period
#
47
A female newborn is being worked up clinically for several congenital abnormalities. During this workup, it is discovered
that normal development of the vagina and uterus in this female infant has not occurreFailure of the uterus to develop
(agenesis) is directly related to the failure of which one of the following embryonic structures to develop?
Urogenital ridge
Mesonephric duct
@Paramesonephric duct
Metanephric duct
Epoophoron
#
48
Microscopical exam of vena umbilicalis of a newborn, who died of intoxication, reveals diffuse inflammatory infiltration
of the wall; its lumen is obturated by thrombus with numerous neutrophilic leukocytes, facts of caryorrhexis and with
bacterial colonies. What is the most possible after-effect of the thrombus?
Transformation into thromboembolus
Aseptic autolysis
Organization and canalization of thrombus
@Septic autolysis
Petrifaction of thrombus
#
49
A 24-year-old woman, on a 3rd day after childbirth, presented with a diffusive edema and painful palpation of the right
breast. Physical examination revealed hyperemia of the breast and a fervescence. Histological research of a gland's tissue
showed diffusive leucocytes infiltration of stroma, an interstitial edema and hyperemia of vessels. What is the most likely
diagnose?
@Acute phlegmonous mastitis
Acute apostematous mastitis
Acute serous mastitis
Chronic purulent mastitis
-
#
50
A 23-year-old woman presents with a rubbery, freely movable 2-cm mass in the upper outer quadrant of the left brest.
Which of the following histologic features is most likely to be seen when examining a biopsy specimen from this mass?
Large numbers of neutrophils
Large numbers of plasma cells
Dust estasia with inspissation of breast secretion
Necrotic fat surrounded by lipid-laden macrophages
@A mixture of fibrous tissue and ducts
#
51
A 39-year-old woman present with the new onset of a bloody discharge from her right nipple. Phsicl exaination reveals a
1-cm freely mavable mass that is located directly beneath the nipple. Sections from this mass reveal multiple
fibrovascular cores lined by several layers of epithelial cells. Atypia is minimal. The lesion is completely contained
within the duct and no invasion into underlying tissue is seen. Which of the following is the most likely diagnosis?
Bening phyllodes tumor
@Ductal papilloma
Intraductal carcinoma
Paget;s disease
Papillary carcinoma
#
52
A 35-year-old woman present with 2.2-cm mass in her left breast. The mass is excised and histologic section reveal a
tumor composed of a mixture of ducts and cells, as seen in the photomicrograph below. The epithelial cells within the
duct not atypical in appearance. There is a marked increase in the stromal cellulary, but the stromal ceels are not atypical
in appearance and mitoses are not found. Which of the following is the most likely diagnosis?
Atypical epithelial hyperplasia
@Bening phyllodes tumor
Fibroadenoma
Malignant phyllodes tumor
Medullary carcinoma
#
53
A 48-year-old woman presents with a 1.5-cm firm mass in the upper outer quadrant of her left breast. A biopsy from this
mass reveals many of the ducts to be filled with atypical cells. In the.center of these ducts there is extensive necrosis. No
invasion into the surrounding fibrous tissue is seen. Which of the following is the most likely diagnosis?
Colloid carcinoma
@Comedocarcinoma
Infiltrating ductal carcinoma
Infiltrating lobular carcinoma
Lobular carcinoma in situ
#
54
A 46-year-old woman presents with a 4-month history of a discharge from the nipple. An excisional biopsy of the nipple
area reveals infiltration of the nipple by large cells with clear cytoplasm. These cells are found both singly and in small
clusters in the epidermis and are PAS-positive and diastase-resistant. Which of the following is the most likely diagnosis?
Ductal papilloma
Eczematous inflammation
Mammary duct ectasia
@Paget's disease
Phyllodes tumor, malignant
#
55
A 65-year-old woman presents with a pruritic red, crusted, sharply demarcated map-like lesion involving a large portion
of her labia major. Histologic sections from this lesion reveal individual anaplastic tumor cells infiltrating the epidermis.
Distinctive clear spaces are noted between these anaplastic cells and the surrounding normal epithelial cells. These malig-
nant cells stain positively for mucin and negatively with S100. Which of the following is the most likely diagnosis?
Clear cell adenocarcinoma
Malignant melanoma
@Extramammary Paget disease
Sarcoma botryoides
Squamous cell carcinoma
#
56
A 60-year-old woman presents with a slowly enlarging 2.5-cm firm, irregular mass in the upper outer quadrant of her left
breast. A biopsy from this mass is interpreted by the pathologist as being an infiltrating lobular carcinoma of the breast.
Which of the following histologic features is most characteristic of this tumor?
Expansion of lobules by monotonous proliferation of epithelial cells
Large cells with clear cytoplasm within the epidermis
Large syncytium-like sheets of pleomorphic cells surrounded by aggregates of lymphocytes
Small individual malignant cells dispersed within extracellular pools of mucin
@Small tumor cells with little cytoplasm infiltrating in a single-file pattern
#
57
A 35-year-old woman who underwent a modified radical mastectomy of her right breast for infiltrating ductal carcinoma
2 years ago presents with enlargement of her right breast. The breast has a swollen, red-discolored appearancIt is diffusely
indurated and tender on palpation. Multiple axillary lymph nodes are palpable in the lower axille. The working clinical
diagnosis is inflammatory carcinoma. Which of the following histologic features is most characteristic of this clinical
diagnosis?
Duct ectasia with numerous plasma cells
@Extensive invasion of dermal lymphatics
Infiltrating malignant ducts surrounded by numerous neutrophils
Malignant vascular tumor forming slitlike spaces
Marked dermal desmoplasia
#
58
A 48-year-old woman presents with a painless mass located in her left breast. Physical examination finds a firm,
nontender, 3-cm mass in the upper outer quadrant of her left breast. There was retraction of the skin overlying this mass,
and several enlarged lymph nodes were found in her left axill. The mass was resected and histologic sections revealed an
invasive ducial carcinoma, grade 2 with approximately 50% of the tumor being composed of tubular structures. Biopsies
from her axillary lymph nodes revealed the presence of metastatic disease to 4 of 18 examined axillary lymph nodes.
Further studies revealed that the tumor cells were positive for estrogen receptors and had high levels of urokinase
plasminogen acti-vator. Numerous new blood vessels were proliferating at the edge of the tumor, and the majority of the
tumor cells were diploid with low S-phase fraction. Which of the following clinical findings is associated with the worse
prognosis for this individual?
High levels of urokinase plasminogen activator being present in the tumor cells
Histologic grade 2 with approximately 50% of the tumor being composed of tubular structures
@Metastatic disease being present in four axillary lymph nodes
The majority of the tumor cells being diploid with low S-phase fraction
The tumor cells being positive for estrogen receptors
#
59
An ovary specimen stained by hematoxylin-eosin presents a follicle, where cells of follicular epithelium are placed in 1-2
layers and have cubic form, there is a bright-red membrane around the ovocyte. What follicle is it?
Mature
Secondary
Primordial
Atretic
@Primary
#
60
Ovarian tumour was diagnozed in a woman. Surgery should be perfomed. What ligament should be extracted by the
surgeon to disconnect the ovary and the uterus?
Broad ligament of uterus
Suspensory ligament of ovary
Lateral umbilical ligament
Round ligament of uterus
@The ovarial ligament
#
61
Ovarial cystadenomas or cystadenocarcinoma (serous or mucinous)
always produce androgen
seldom are bilateral
@can be papillary
usually occur during pregnancy
are extremely rare
#
62
The ovarian tumor with the highest degree of bilateral involvement is
endometrioid carcinoma
serous cystadenoma
mucinous cystadenoma
mucinous cystadenocarcinoma
@serous cystadenocarcinoma
#
63
A 23-year-old woman presents with pelvic pain and is found to have an ovarian mass of the left ovary that measures 3 cm
in diameter. Grossly, the mass consists of multiple cystic spaces. Histologically, these cysts are lined by tall columnar
epithelium, with some of the cells being ciliateWhich of the following is the correct diagnosis for this ovarian tumor,
which histologically recapitulates the histology of the fallopian tubes?
@Serous tumor
Mucinous tumor
Endometrioid tumor
Clear cell tumor
Brenner tumor
#
64
The majority of malignant tumors of the ovary take their origin from
@surface epithelium
urogenital stem cells
ovarial germ cells
stromal cells
hilar cells
#
65
A 32-year-old woman presents with the recent onset of oligomenorrhea followed by amenorrhea and then the loss of
female secondary characteristics. She has also developed acne, deepening of her voice, and temporal balding. Which of
the following types of ovarian tumor is most likely to produce these symptoms?
Epithelial tumor
@Stromal tumor
Germ cell tumor
Surface tumor
Metastatic tumor
#
66
A 51-year-old woman presents with abdominal distention. Surgical exploration finds multiple irregular masses within her
abdominal cavity. A biopsy of one of these masses is examined by a pathologist during a frozen section. His diagnosis is
"metastatic mucin-secreting neoplasm" and he advises the surgeon to examine the appendix. The surgeon finds the appen -
dix to be grossly normal in appearancIt is resected and examined histo-: logically but no malignancy is founThe next
step in searching for this primary tumor should be to look for a tumor in which one of the following sites?
Adrenal gland for a conical carcinoma
Gallbladder for a papillary cystic tumor
Kidney for a clear cell carcinoma
@Ovary for a non-germ cell tumor
Uterus for a smooth-muscle tumor
#
67
An uncircumcised 49-year-old man present with the sudden onset of severe pain in the distal portion of his penis. The
emergency room physical examines the patient and finds that the foreskin is retracted but cannot be rolled back over the
gland penis. The ER physical calls the urologist, who performs an emergency resection of this patient's foreskin. Which
of the following is the most likely diagnosis?
Balanoposthitis
Epispadias
Omphalocele
@Paraphimosis
Phimosis
#
68
Histologic examination of an excision specimen from a lesion on the dorsal surface of the penis reveals a papillary lesion
with clear vacuoliza-lion of epithelial cells on the surface and extension of the hyperplastic epithelium into the underlying
tissue along a broad front. Which of the following is the most likely diagnosis?
Condyloma acuminatum
Bowen's disease
Erythroplasia of Queyrat
@Verrucous carcinoma
Squamous cell carcinoma
#
69
A patient complains of frequent and difficult urination. Imperfection of what formation can cause it?
Bulb-uretic glands
@Prostate
Testicles
Testicle adnexa
Sperm bubbles
#
70
A 67-year-old man is found on rectal examination to have a single, hard, irregular nodule within his prostate. A biopsy of
this lesion reveals the presence of small glands lined by a single layer of cells with enlarged, prominent nucleoli. From
what portion of the prostate did this lesion most likely originate?
Anterior zone
Central zone
@Peripheral zone
Periurethral glands
Transition zone
#
71
A 47-year-old man presents with the sudden onset of fever, chills, and dysuriDuring the review of symptoms you discover
that he has no history of recurrent urinary tract infections. Rectal examination finds that the prostate gland is very
sensitive and examination is painful. .Which of the following is the most likely diagnosis?
@Acute prostatitis
Chronic bacterial prostatitis
Chronic abacterial prostatitis
Granulomatous prostatitis
Benign prostatic hyperplasia
#
72
A 69-year-old man presents with urinary frequency, nocturia, dribbling, and difficulty in starting and stopping urination.
Rectal examination reveals the prostate to be enlarged, firm, and rubbery. A needle biopsy reveals increased numbers of
glandular elements and stromal tissuThe glands are found to have a double layer of epithelial cells. Prominent nuclei or
back-lo-back glands are not seen. Which of the following is the most likely diagnosis?
Atrophic prostatitis
Atypical small acinar proliferation
High-grade prostatic intraepithelial neoplasia
@Benign prostatic hyperplasia
Prostatic adenocarcinoma
#
73
A physical examination of a 68-year-old man, with complains to an acute ischuria (retention or suppression of the urine),
revealed the nodular enlargement of a prostate. A microscopical investigation showed that the majority of knots were
constructed of polymorphic glandular structures, with some retention cysts. The latter ones had thick secreta. What is the
most likely diagnose?
@Glandular hyperplasia of prostate
Fibromuscular hyperplasia of prostate
Hyperplasia of prostate mixed form
Adenoma of prostate
Carcinoma of prostate
#
74
An autopsy of an elderly man revealed enlarged, soft, elastic, slightly nodular prostate. A gross investigation of a cut
section showed separate nodes parted by layers of a connective tissue. The compressed surrounding prostatic tissue
created a plane of cleavage about them. A microscopic study found the increase of glands quantity. There were variable
number of prostatic nodes and glands in them.
@Glandular nodular hyperplasia
Fibromuscular (stromal) hyperplasia
Mixed nodular hyperplasia
Adenocarcinoma
Undifferentiated carcinoma
#
75
At the section of a 73-year-old man was revealed an enlarged, soft, elastic, tuberous prostate. On the incision it consisted
of separated by strata of connective tissue nodules. Microscopical exam revealed the increment of glandular elements.
The size of the lobules and the amount of glandular elements were different. What process is the most possible?
@Glandular nodular hyperplasia
Muscular-fibrous nodular hyperplasia
Mixed nodular hyperplasia
Adenocarcinoma
Undifferentiated cancer
#
76
A 68-year-old man presented with difficulty of a miction. The prostate was operatively removed. A microscopical
investigation revealed an increase of glandular and muscular elements numbers. A lobular structure of prostate was also
altered. What process in a prostate is the most probable?
@Mixed nodular hyperplasia
Glandular hyperplasia
Fibromuscular hyperplasia
Prostatitis
Adenocarcinoma
#
77
Within prostatic glands, feature consistent with prostatic intraepithelial neoplasma (PIN) incude all the following
EXCEPT
cellular crowding
@absence of a basal cell layer
variation in nuclear size
nucleoli
hyperchromatism
#
78
Carcinoma of the prostate tends to do all the following EXCEPT
be adenocarcinoma
arise in the posterion lobe
cause elevation of serum acid phosphatase
@be estrogen-depended
form osteoblastic metastases
#
79
In histological specimen there is an organ, which is covered by tunica vaginalis and tunica albuginea. Stroma of the organ
consists of connective tissue with the Leydig’s cells. Parenchyma contains tubules, which are lined by the
spermatogeneous epithelium. What organ is there?
@Testis
Prostate gland
Epididymis
Mammary gland
Ovary
#
80
Primary germ cell tumors of the testise occur predominantly in the younger male with the exception of
embryonal carcinoma
@spermatocytic seminoma
polyembryoma
choriocarcinoma
teratocarcinoma
#
81
A 27-year-old man presents with a testicular mass, which is resected and diagnosed as being a yolk sac tumor. Which of
the following substances is most likely to be increased in this patient's serum as a result of being secreted from the cells of
this tumor?
Acid phosphatase
@a-fetoprotein (AFP)
Alkaline phosphatase
p-human chorionic gonadotropin (p-hCG)
Prostate-specific antigen (PSA)
#
82
A 44-year-old man presents with painless enlargement of one testiclPhysical examination reveals a single testicular mass
that does not transilluminatThe mass is resected, examined histologically, and radiation therapy is subsequently given
based on the pathologist's diagnosis. Which of the following best describes the expected microscopic appearance of this
tumor?
A mixture of malignant cytotrophoblasts and syncytiotrophoblasts
Abnormal tissue derived from all three germ levels with scattered immature neural elements
Large tumor cells with abundant eosinophilic, granular cytoplasm, and rare intracytoplasmic rhomboid crystals
@Numerous lymphocytes in the fibrous stroma between groups of tumor cells having distinct cell membranes and clear
cytoplasm
Sheets of undifferentiated tumor cells having focal glandular differentiation
#
Endocryne system
#
1
An endocrinal gland with parenchyma consisting of epithelium and neural tissue is under morphological examination.
Epithelial trabecules have two types of cells: chromophilic and chromophobic. Identify this organ:
@Hypophysis
Parathyroid gland
Hypothalamus
Thyroid gland
Adrenal glands
#
2
A patient has a decreased vasopressin synthesis that causes polyuria and as a result of it evident organism dehydratation.
What is the mechanism of polyuria development?
Acceleration of glomerular filtration
Reduced tubular reabsorption of protein
Reduced tubular reabsorption of Na ions
Reduced glucose reabsorption
@Reduced tubular reabsorption of water
#
3
A 23-years-old male presented to his physician with excess of the somatotropic hormone, the enlarged nose, lips, ears, a
mandible, hands and feet. What is the most likely diagnosis?
@Acromegaly
Pituitary nanism
Itsenko-Cushing disease
Addison's disease
Adiposogenital dystrophy
#
4
A 45-years-old male presented to the hospital with enlarged nose, ears, a mandible and feet. What is the most likely
diagnosis?
@Acromegaly
Nanism
Cerebro-hypophyseal cachexia
Adiposogenital dystrophy
-
#
5
A 42-year-old man presents with increasing fatigue and occasional headaches. He states that recently he has had to
change his shoe size from 9 to 10, and he also thinks that his hands and jaw are now slightly larger. Physical examination
reveals a prominent forehead and lower jaw, enlarged tongue, and large hands and feet. Initial laboratory examination
reveals increased serum glucosWhich of the following is the most likely explanation for this constellation of clinical
findings?
@Acromegaly
Apoplexy
Cretinism
Diabetes
Giganrism
#
6
A 45-years-old male presented to the hospital with gradual enlargement of feet, right hand, nose and lips. The adenoma of
a pituitary body was diagnosed. What is the most likely disease?
@Acromegaly
Nanism
Basedow's disease.
Addison's disease
Diabetes mellitus
#
7
A 46 year-old patient has complained of headache, fatigue, thirst, pains in the spine and joints for the last 2 years.
Clinically observed disproportional enlargement of hands, feet, nose, superciliary arches. He notes that he needed to buy
bigger shoes three times. What is the main reason of such disproportional enlargement of different parts of the body?
Increased sensitivity of the tissues to growth hormone
Joints dystrophy development
Joints chronic inflammation development
Increased sensitivity of the tissues to insulin
@Cartilaginous tissue proliferation under growth hormone influence
#
8
Gradually, patient’s feet, right hand, a nose and lips began to increase in size rateably. The hypophysial adenoma was
revealed. Diagnose the disease.
Addison’s disease
Nanism
Diffuse toxic goiter
@Acromegaly
Diabetes mellitus
#
9
Arterial hypertension, hyperglycemia, glucosuria were observed clinically for a long time in the patient with upper type of
obesity. Death was due to the cerebral haemorrhage. Basophilic hypophysis adenoma, hyperplasia of adrenal gland cortex
were revealed on pathomorphological examination. What is the likely diagnosis?
Adiposogenitalis dystrophy
Acromegaly
Diabetes mellitus
Hypophysis nanism
@Cushing disease
#
10
A 42-year-old woman presented with the obesity, chiefly of the trunk and face, steroid diabetes, an arterial hypertension
and secondary dysfunction of ovaries. An autopsy revealed a hypertrichosis, a hirsutism, strias on a skin of hips and a
abdomen. In the anterior lobe of the pituitary gland a tumour was found. Histological investigation showed the basophilic
adenoma of the pituitary and hyperplasia of a fascicular layer in adrenals. What of the listed diagnoses is the most likely?
@Itsenko-Cushing disease
Itsenko-Cushing syndrome
Simmonds' disease
Adiposogenital dystrophy
Pituitary nanism
#
11
A male patient presented with obesity of the trunk and face. He also had a long medical history of an arterial
hypertension. The death resulted from a hemorrhage in a brain. A morphological investigation revealed a basophilic
adenoma of a pituitary body and hyperplasia of adrenals cortex. What is the most likely preliminary diagnosis?
@Itsenko-Cushing disease
Diabetes mellitus
Acromegaly
Pituitary nanism
Adiposogenital dystrophy
#
12
A 44 year old woman complains of general weakness, heart pain, significant increase of body weight. Objectively: moon
face, hirsutism, AP is 165/100 mm Hg, height - 164 cm, weight - 103 kg; the fat is mostly accumulated on her neck,
thoracic girdle, belly. What is the main pathogenetic mechanism of obesity?
Reduced production of thyroid hormones
Increased insulin production
Increased mineralocorticoid production
Reduced glucagon production
@Increased production of glucocorticoids
#
13
An autopsy of a 40-year-old woman revealed the obesity, chiefly of the trunk and face ("moonface" and "buffalo hump"),
strias on a skin of hips and abdomen. She had in her medical records an osteoporosis with spontaneous fractures of bones,
an amenorrhea, an arterial hypertension, a hyperglycemia. What of the listed diagnoses is the most likely?
@Itsenko-Cushing syndrome
Addison's disease
Friderichsen-Waterhouse syndrome
Conn's syndrome
Sipple's syndrome
#
14
A patient complaining of weight loss (10 kg during 2 months), pallpitation and exophthalmos came to the
endocrinologist. For the hyperfunction of what endocrine gland (glands) are these complaints the most typical?
Pancreas
Ovaria
Parathyroid glands
Adrenal glands
@Thyroid
#
15
A 19-year-old female suffers from tachycardia in rest condition, weight loss, excessive sweating, exophtalmos and
irritability. What hormone would you expect to find elevated in her serum?
ACTH
Mineralocorticoids
Cortisol
Insulin
@Thyroxine
#
16
A 48-years-old woman presented with insomnia (inability to get enough sleep, a bad dream), a fatigue, irritability,
exophthalmos (protuberance of eyes) and tachycardia. The thyroid gland was enlarged. She died soon due to
cardiovascular insufficiency. Histological investigation of the thyroid tissue revealed a proliferation of an epithelium with
formation of papillas, a colloid colliquation, lymphocyte and plasmocyte infiltration and formation of lymphatic follicles
with the germinal centres. What is the most likely diagnosis?
@Diffuse toxic goiter. (Basedow's disease)
Endemic goiter
Sporadic goiter
Hashimoto's thyreoiditis
Benign tumor of thyroid gland
#
17
A thyroid gland of a 48-year-old male was removed at surgery department. Histological investigation revealed various
size follicles which were lined by tall columnar epithelium. The latter one was pilled up at places forming papillary
infoldings. Colloid appeared lightly staining, watery and finely vacuolated. The stroma showed increased accumulation of
lymphoid cells.
@Basedow's goiter.
B Hashimoto's thyreoiditis
Riedel's thyreoiditis
De Quervain's thyroiditis
Nodular goiter
#
18
A 40-year-old woman was made a thyroidectomy. Histological exam of the gland reveals big follicles of different size
with a foamy colloid. The follicular epithelium is high and forms papillae in some places. The stroma has a focal
lymphocytic infiltration. Diagnose the disease of the thyroid gland.
@Diffuse toxic goiter
Hashimoto’s thyroiditis
Riedel’s thyroiditis
de Quervain’s thyroiditis
Nodular goiter
#
19
A patient suffering from thyrotoxicosis symptoms of vegetoasthenic syndrome was revealed. What of the following
would show the histological appearance of a thyroid gland being stimulated by thyroid-stimulating hormone (TSH)?
Increased numbers of parafollicular cells
Decreased numbers of follicular cells
Decreased numbers of parafollicular capillaries
An abundance of colloid in the lumen of the follicle
@Columnar-shaped follicular cells
#
20
A patient is followed up in an endocrinological dispensary on account of hyperthyreosis. Weight loss, tachycardia, finger
tremor are accompanied by hypoxia symptoms - headache, fatigue, eye flicker. What mechanism of thyroid hormones
action underlies the development of hypoxia?
Competitive inhibition of respiratory ferments
Specific binding of active centres of respiratory ferments
Intensification of respiratory ferment synthesis
Inhibition of respiratory ferment synthesis
@Disjunction, oxydation and phosphorilation
#
21
Graves’ disease is associated with all the following EXCEPT
Tachycardia
Anti-TSH receptor antibodies
Localized myxedema
@Toxic nodular goiter
Conversion hysteria
#
22
A 48-year-old male, with a long history of living in the mountain district of Central Asia, presented with a thyroid gland
enlargement, resulted in complicated swallowing. A physical investigation revealed an increasing of a body mass,
slowness, drowsiness, and puffy face. A microscopic study of a thyroid gland showed irregular follicles with a
hypochromic colloid. What of diagnoses is the most probable?
@Endemic goiter.
Basedow's goiter.
Hashimoto's thyreoiditis.
Sporadic goiter.
Riedel's thyreoiditis.
#
23
The exam of the patient, who lives in the mountain region of Middle Asia, reveals the enlargement of the thyroid gland,
that makes it difficult to swallow, increased weight, retarded thinking, drowsiness and puffy face. Microscopic exam of
the gland reveals follicles of different size with hypochromic colloid. What is the most possible diagnosis?
@Endemic goiter
Diffuse toxic goiter
Hashimoto’s thyroiditis
Sporadic goiter
Fibrous goiter
#
24
A histological investigation of the removed enlarged thyroid gland revealed irregular form and sizes follicles. The
follicular epithelium proliferated both into follicles lumen and outside. Layers of the connective tissue separated various
sites of the thyroid. What is the most likely diagnosis?
@Macro-microfollicular goiter
Diffuse goiter
Autoimmune thyroiditis
Follicular carcinoma
Riedel's thyreoiditis
#
25
A 58-year-old woman presents with increased "fullness" in her neck. Physical examination finds nontender diffuse
enlargement of her thyroid glanClinically she is found to be euthyroid and her serum TSH level is within normal limits.
Sections from her enlarged thyroid gland reveal numerous, mainly enlarged follicles, most of which are filled with abun -
dant colloid material. There are areas of fibrosis, hemorrhage, and cystic degeneration. No papillary structures are
identified and neither colloid scalloping nor Hurthle cells are present. Which of the following is the most likely
diagnosis?
Colloid carcinoma
Diffuse toxic goiter
Graves' disease
Hashimoto's thyroiditis
@Multinodular goiter
#
26
At operation of excision of a diffusive thyroid gland struma a patient died from acute adrenals insufficiency. An autopsy,
besides the atrophy of adrenals, revealed a left ventricle hypertrophy in a heart, a hypertrophy and an intracellular edema
of cardiomyocytes, a serous edema and lymphoid infiltration of a myocardial stroma. What is the figurative name of a
heart pathology presented at post-mortem?
@Thyrotoxyc heart
Tabby cat heart
Cor bovinum
Cor pulmonary
Brown atrophy of heart
#
27
A patient presented to surgery with symptoms of hypothyrosis. Macroscopic investigation of the removed thyroid gland
revealed its considerable enlargement, dense-elastic texture and lobulated surface. Histological investigation showed the
diffuse infiltration of lymphocytes and plasmocytes of gland's parenchyma with formation of lymphoid follicles. The
latter ones had hyperplastic germinal centers with atypia and a metaplasia of a follicular epithelium. In addition, the
sclerotic alterations of a parenchyma were also determined. What is the most likely diagnose?
@Autoimmune thyroiditis
Diffuse toxic goiter
Colloid goiter
Nodular goiter
De Quervain's thyroiditis
#
28
A 44-year-old woman presents with anorexia and weight loss. Physical examination reveals a slightly decreased blood
pressure along with increased skin pigmentation. Laboratory examination reveals a low cortisol with increased ACTH.
After further work-up the diagnosisof adrenal cortical failure is made. One year later this woman is found to have
hyperglycemia and after appopriate work-up the diagnosis of type 1 diabetes mellitus is made. Which of the following
adnormalities is this individual most likely to develop?
@Autoimmune destruction of the thyroid gland
Bacterial infection of the antrum of the stomach
Fungal infection of the oral cavity
Metabolic hyperfunction of the parathyroid glands
Neoplastic development in the anterior pituitary
#
29
A perimenopausal woman complains of slight swallowing difficulty, fatigue, and a change in bowel habits. The
photomicrograph below is of her thyroid gland. This disorder is
Subacute thyroiditis
Thyrotoxicosis
@Autoimmune thyroiditis
Riedel’s thyroiditis
Conversion hysteria
#
30
Histological exam of a thyroid gland reveals moderate atrophy of the parenchyma, sclerosis and diffuse infiltration of the
stroma by lymphocytes and plasmatic cells with the formation of lymphoid follicles. What is your diagnosis?
@Autoimmune thyroiditis
Parenchymatous goiter
Thyrotoxicosis
Thyroiditis
Fibrous goiter
#
31
A histological investigation of a thyroid gland, presented to pathology, revealed a mild atrophy of a parenchyma,
sclerosis, diffusive infiltration of a stroma by lymphocytes and plasmocytes with formation of lymphoid follicles. What is
the most likely diagnose?
@Autoimmune thyroiditis
Parenchymatous goiter
Toxic goiter
Thyroiditis
Riedel's thyreoiditis
#
32
An 8-month-old infant is being evaluated for growth and mental retardation. Physical examination reveals a small infant
with dry, rough skin; a protuberant abdomen; periorbital edema; a flattened, broad nose; and a large, protuberant
tonguWhich of the following disorders is the most likely cause of this infant's signs and symptoms?
Graves' disease
@Cretinism
Toxic mulunodular goiter
Toxic adenoma
Struma ovarii
#
33
A morphological investigation of a thyroid removed due to thyrotoxicosis struma revealed a diffuse infiltration of a gland
by lymphocytes with a destruction of parenchyma and an area of connective tissue. What is the most likely diagnose?
@Hashimoto's thyreoiditis
Anaphylactic reaction
Thyrotoxicosis
Cancer of thyroid
Adenoma
#
34
At the operative biopsy of a thyroid gland, histologically, among the follicles filled with colloid, were revealed lymphoid
structures with the growth centers. Define the disease.
Diffuse toxic goiter
Endemic goiter
Sporadic goiter
@Hashimoto’s goiter
Fibrous goiter
#
35
During the intraoperative biopsy of thyroid gland the histological examination revealed lymphoid structures with growth
centers, which were among the folliculi full of colloid. What disease was it?
Endemic goiter
@Hashimoto's thyroiditis (lymphadenoid goiter)
Thyrotoxicosis (Basedow's goiter)
Ridel's goiter
Sporadic goiter
#
36
A histological investigation of a thyroid gland biopsy, presented to pathology, revealed lymphoid structures with the
germinal (growth) centers among the follicles filled with a colloid. What is the most likely disease presented in that case?
@Hashimoto's goiter.
Endemic goiter
Sporadic goiter
Basedow's goiter
Riedel's thyreoiditis
#
37
A histological investigation of a thyroid revealed a considerable infiltration of a gland by lymphocytes with formation of
lymphoid follicles, a destruction of parenchyma, growth of connective tissue fibers. What is the most likely disease
presented in that case?
@Hashimoto's goiter
Colloid goiter
Endemic goiter
Diffuse toxic goiter
Parenchymatous goiter
#
38
The thyroid gland of the patient is twice enlarged. The palpation shows: the gland is solid; its surface is unevenly
tuberous. Histological exam reveals diffuse infiltration of the tissue by lymphocytes, plasmatic cells with the formation of
follicles and intensive overgrowth of the connective tissue. What disease do these facts indicate?
@Hashimoto’s goiter
Endemic goiter
Sporadic goiter
Diffuse toxic goiter
Fibrous goiter
#
39
A 42-year-old man presented with a thyroid gland enlargement in two times. A physical investigation revealed firm,
irregular lobulated gland. Histological investigation showed a diffusive infiltration of thyroid gland by lymphocytes,
plasmocytes with follicles formation and the enhanced growth of a connective tissue. What is the most likely diagnosis?
@Hashimoto's goiter.
Endemic goiter.
Sporadic goiter.
Diffuse toxic goiter.
Riedel's thyreoiditis
#
40
Follicular carcinoma of the thyroid may show all the following features EXCEPT
Vascular invasion and hematogenous metastasis
@Multiple foci within the gland
A clear cell variant that resembles renal carcinoma
An insular type that is an aggressive form
Absence of ground-glass nuclei
#
41
A 37-year-old man presents with a single, firm mass within the thyroid glanThis patient's father developed a tumor of the
thyroid gland when he was 32 years of agHistologic examination of the mass in this 37-year-old man reveals organoid
nests of tumor cells separated'by broad bands of stroma, as seen in the photomicrograph below. The stroma stains •
positively with Congo red stain and demonstrates yellow-green birefringencWhich of the following is the most likely
diagnosis?
Follicular carcinoma
Papillary carcinoma
Squamous cell carcinoma
@Medullary carcinoma
Anaplasric carcinoma
#
42
A 2-year-old child experienced convulsions because of lowering calcium ions concentration in the blood plasma.
Function of what structure is decreased?
Thymus
Pineal gland
@Parathyroid glands
Adrenal cortex
Hypophysis
#
43
Kidneys of a man under examination show increased resorbtion of calcium ions and decreased resorbtion of phosphate
ions. What hormone causes this phenomenon?
Thyrocalcitonin
Vasopressin
Hormonal form D3
Aldosterone
@Parathormone
#
44
A 52-year-old woman presents with nausea, fatigue, muscle weakness, and intermittent pain in her left flank. Laboratory
examination reveals increased serum calcium and decreased serum phosphorus. The patient's plasma parathyroid hormone
levels are increased, but parathyroid hormone-related peptide levels are within normal limits. Urinary calcium is
increased, and microhematuria is present. Which of the following is the most likely cause of this patient's signs and
symptoms?
@Primary hyperparathyroidism
Primary hypoparathyroidism
Pseudohypoparathyroidism
Secondary hyperparathyroidism
Secondary hypoparathyroidism
#
45
Periodic renal colics attackes are observed in the woman with primery hyperparathyroidizm. Ultrasonic examination
revealed small stones in the kidneys. What is the cause of the formation of the stones?
Hypercholesterinemia
Hyperkalemia
@Hypercalcemia
Hyperphosphatemia
Hyperuricemia
#
46
Osteomalacia may best be characterized as
Failure of bone remodeling
@Failure of bone mineralization
Failure of osteoid formation
Reactive bone formation
Reduction in amount of normally mineralized bone
#
47
A 56-year-old woman with a long history of parathyroid glands disease, died from increased renal insufficiency. A post-
mortem revealed a bones deformation of extremities, a vertebral column, and ribs. Bones were fragile, soft and easily
deformed or cat. They had some motley tumorous formations. Kidneys were contracted. Histological investigation
revealed lacunar bone resorption. In addition, a microscopic study demonstrated giant cell granulomas within the centres
of tumorous formations, accumulations of erythrocytes and a hemosiderin. Name the described disease.
@Parathyroid osteodystrophy.
Metastases of cancer in a bone
Chronic renal insufficiency
Paget's disease
Myeloma
#
48
A 64-year-old patient died of a cachexia. A post-mortem revealed an adenoma of parathyroid glands, bones deformation
of extremities, a vertebral column, and ribs. Bones were soft, with slight porousness and easily deformed or cat. What of
the listed diagnoses is the most likely?
@Parathyroid osteodystrophy.
Osteopetrosis
Chondrodysplasia
Osteomyelitis
Fibrous dysplasia
#
49
At the section of a man, who died of cachexy, were revealed: adenoma of the parathyroid glands, deformity of bones,
especially of extremities, vertebra and ribs. The bones are soft, porous and can be easily cut. What is the most possible
diagnosis?
@Parathyroid osteodystrophy
Osteopetrosis
Chondrodysplasia
Osteomyelitis
Fibrous dysplasia
#
50
A 65-year-old man presents with bone pain and is found to have hypocalcemia and increased parathyroid hormone.
Surgical exploration of his neck finds all four of his parathyroid glands to be enlarge. Which of the following disorders is
the most likely cause of this patient's enlarged parathyroid glands?
Primary hyperplasia
Parathyroid adenoma
@Chronic renal failure
Parathyroid carcinoma
Lung carcinoma
#
51
The section of a 50-year-old woman, who died of uremia, revealed adenoma of parathyroid glands, deformation of the
limbs, vertebra and ribs. The bones are soft; on incision – the increased amount of pores, tumour-like nodules that have
mottled sight and contain cysts. At the microscopical exam a noticeable changeover of bone structures, focuses of lacunar
resolution of osteal and fibrous tissue are determined. What is the most possible disease?
@Recklinghausen’s disease
Chronic osteomyelitis
Osteoporosis
Fibrous dysplasia
Paget’s disease
#
52
A 55-year-old woman died from uraemia. A post-mortem revealed a parathyroid gland adenoma, bones deformation of
extremities, a vertebral column, and ribs. Bones were soft, with slight porousness. On a cut they had a motley pattern with
multiple cysts. Histological investigation revealed marked bone remodelling and lacunar resorption of osteoid and fibrous
tissues. What is the most likely diagnosis?
@Parathyroid osteodystrophy.
Chronic osteomyelitis
Osteoporosis
Fibrous dysplasia
Paget's disease
#
53
A 60-year-old woman has noted a dark red-black appearance to her great toe and second and third toes of her left foot for
the past month. On physical examination, the toes are cold to touch and have no sensation. The dorsalis pedis and
posterior tibial pulses are not palpable on the left. A transmetatarsal amputation is performed. These findings are most
typical for a patient with which of the following conditions?
@Diabetes mellitus
Monckeberg's arteriosclerosis
Blunt force trauma
AIDS
Type III hypersensitivity reaction
#
54
Components of diabetic glomerulopathy include all the following EXCEPT
Diffuse glomerulosclerosis
Nodular glomerulosclerosis
Thickening of capillary basement membranes
Mesangial proliferation
@Fibrin thrombi
#
55
Diseases of the urinary tract occurring with increased frequency in patients with diabetes mellitus include all the
following EXCEPT
Nodular or intercapillary glomerulosclerosis
Nephritic syndrome
Aterosclerosis of the renal artery
Renal pappilary necrosis
@Uric acid stones
#
56
One of the principal types of diabetes (type 1) is an autoimmune disease in which antibodies against endocrine cells of
pancreas (islets of Langhergans) and depresses activity of cells (production of insulin). Which cells activity is depressed?
@B-cells
A-cells
D-cells
PP-cells
D1-cells
#
57
A 35-year-old obese woman of normal height is found to have hyperglycemia that lasts for several hours following a
meal. Further work-up reveals normal fasting serum glucose levels. Physical examination is otherwise unremarkable.
Which of the following is the most likely cause of this patient's postprandial hyperglycemia?
Antibodies to insulin
Decreased functioning of hepatocyte nuclear factor
Decreased production of glucagon
Excess production of cortisol
@Impaired release of insulin
#
58
A 20-year-old woman is found to have elevated blood glucose levels on several occasions. She is otherwise asymptomatic
and is of normal height and normal weight. Laboratory evaluation does not detect the pres ence of islet cell autoantibodies
in this young woman. Several members in successive generations other family, however, have been diagnosed as having
diabetes mellitus. Further tests find that her mother also has mildly elevated blood glucose levels but is not obese and is
otherwise asymptomatic. Which of the following is the most likely diagnosis?
Insulin-dependent diabetes mellitus
@Mature-onset diabetes of the young
Non-insulin-dependent diabetes mellitus
Type 1 diabetes mellitus
Type 2 diabetes mellitus
#
59
A 45-year-old obese woman presents with increasing fatigue, malaise, and fullness in the right upper quadrant of her
abdomen. Pertinent clinical history includes type II diabetes mellitus and hyperlipidemia. Laboratory test finds elevated
liver enzymes along with increased serum cholesterol. Which one of the following clinical procedures or tests should be
used to confirm a diagnosis of nonalcoholic steatohepatitis?
Abdominal magnetic resonance imaging
@Liver biopsy
Liver ultrasonography
Oral cholecystogram
Quantitative serum ferritin
#
60
A renal puncture biopsy was taken from a patient with elevated blood sugar levels and high amounts of glucose in his
urine. Histological investigation revealed extension of mesangium with focal accumulation of membranous substances
and periglomerular sclerosis of some glomerules. In addition, there were found a hyalinosis and plasmatic infiltration of
arterioles; lymphocytes, histiocytes and some leucocytes infiltration of a stroma; glycogen deposits within nephrocytes of
narrow segment. What is the most likely diagnosis?
@Diabetic glomerulonephritis
Arteriolosclerotic nephrocirrosis
Pyelonephritis
Acute glomerulonephritis
Subacute glomerulonephritis
#
61
A 64-year-old man, with a medical history of a diabetes mellitus, presented augmented symptoms of renal failure. What
disease caused such changes in a pancreas?
@Glomerulosclerosis.
Necrosis of renal tubular epithelium.
Amyloidosis.
Proliferative glomerulonephritis
Membranous nephropathy.
#
62
A 64-year-old man died with symptoms of a hypoglycemic coma. A post-mortem revealed reduced, dense pancreas.
Histological investigation of pancreas showed the growth of a connecting tissue, an atrophy of Langerhans islets. What
disease caused such changes in a pancreas?
@Diabetes mellitus.
Mucoviscidosis.
Acute pancreatitis.
Cancer of pancreas.
Hypoplasia of pancreas.
#
63
A 62-year-old man with a 14 years medical history of a diabetes mellitus presented with significantly worsened eye vision
for the last two years. What characteristic processes resulted in worsening of the eye vision?
@Microangiopathy
Macroangiopathy
Calcinosis of hyaloid body
Dimness of cornea
-
#
64
Index of pH of the blood changed and became 7,3 in the patient with diabetus mellitus. Detecting of the components of
what buffer system is used while diagnosing disorder of the acid-base equilibrium?
Protein
Oxyhemoglobin
Phosphate
Hemoglobin
@Bicarbonate
#
65
A patient with a diabetes mellitus died of a chronic renal insufficiency with a development of uremia. At the section were
revealed all manifestations of uremia with the changes in kidneys. What microscopical changes do the kidneys have?
Necrosis of the nephral tubules’ epithelium
Spasm of the afferent arterioles
Hyaline cylinders in the lumen of the nephral tubules
@Hyalinosis and sclerosis of capillaries of glomeruli
Focal necrosis of the renal cortex
#
66
At the section of a 67-year-old man, who died of hypoglycemic coma, the areas of overgrowth of connective tissue,
necrotic focuses and the atrophy of pancreatic islets (islets of Langerhans) were revealed. What disease led to such
changes in pancreas?
@Diabetes mellitus
Mucoviscidosis
Acute pancreatitis
The cancer of the head of the pancreas
Hypoplasia of the pancreas
#
67
A 58-year-old man has a clinical picture of acute pancreatitis. The increase of what substance in the urine from
belowmentioned will confirm the diagnosis?
Albumin
Nonprotein (rest) nitrogen
Urea
Uric acid
@Amylase
#
68
A pancreatic islet cell tumor of the gastrin-secreting G cells will most likely produce
hypoglycemia
@severe peptic ulcreation of the duodenum
mild diabetes
steatorrhea
profuse watery diarrhea
#
69
A 30-year-old woman was diagnosed with insufficiency of exocrinous function of pancreas. Hydrolisis of what nutrients
will be disturbed?
Fats, carbohydrates
Proteins, carbohydrates
@Proteins, fats, carbohydrates
Proteins, fats
Proteins
#
70
A histological spacemen presents parenchymal organ, which has cortex and medulla. Cortex consists of epitheliocytes
bars with blood capillaries between them; the bars form three zones. Medulla consists of chromaffinocytes and venous
sinusoids. Which organ has these morphological features?
Lymph node
Kidney
@Adrenal gland
Thymus
Thyroid
#
71
Some diseases reveal symptoms of aldosteronism with hypertension and edema due to sodium retention in the organism.
What organ of the internal secretion is affected on aldosteronism?
Hypophysis
Testicle
Ovaries
Pancreas
@Adrenal glands
#
72
Examination of a patient revealed hyperkaliemia and hyponatremia. Low secretion of which hormone may cause such
changes?
@Aldosteron
Cortisol
Natriuretic
Vasopressin
Parathormone
#
73
A concentrated solution of sodium chloride was intravenously injected to an animal. This caused decreased reabsorption
of sodium ions in the renal tubules. It is the result of the following changes of hormonal secretion:
Aldosterone increase
Vasopressin increase
@Aldosterone reduction
Vasopressin reduction
Reduction of atrial natriuretic factor
#
74
A patient had been taking glucocorticoids for a long time. When the preparation was withdrawn he developed the
symptoms of disease aggravation, decreased blood pressure and weakness. What is the reason of this condition?
Hyperproduction of ACTH
@Appearance of adrenal insufficiency
Sensibilization
Habituation
Cumulation
#
75
Patient was on glucocorticoids for a long time, discontinuation of usage caused exacerbation of the illness, decreased BP,
weakness. How can you explain it?
Adaptation to the medicine
Cumulation
Hyperproduction of ACTH
Sensitization
@Insufficiency of adrenal glands
#
76
A 42-year-old man presents with weakness and dizziness associated with stress. Physical examination reveals a slightly
decreased blood pressure along with a diffuse increase in skin pigmentation. Laboratory examination reveals
hyponatremia and hyperkalemic acidosis with decreased aldosterone, decreased cortisol, decreased glucose, increased
ACTH, decreased sex steroids, and increased LH and FSH. Thyroid function tests are found to be within normal limits.
Which of the following is the most likely cause of this patient's signs and symptoms?
A benign adenoma of the adrenal cortex
A malignant tumor of the adrenal medulla
@Autoimmune destruction of the adrenal cortex
Bilateral hyperplasia of the adrenal cortex
Tuberculosis of the adrenal medulla
#
77
A 52-year-old man with a history of hematogenic tuberculosis presented to the hospital with hyperpigmentation skins
(melanosis) and mucosas, a cachexia, cardiovascular insufficiency. What disease caused such changes?
@Addison's disease
Pheochromocytoma
Simmonds' disease
Itsenko-Cushing disease
Graves' disease
#
78
At the autopsy of a dead man a bronzed color of the skin and mucous membrane of the oral cavity were noted. There were
also caseous masses in the adrenal glands. What disease did the patient have?
@Addison’s disease
Itzenko-Cushing’s syndrome
Diffuse toxic goiter
Acromegaly
Pheochromocytoma
#
79
A patient suffers from arterial hypertension strokes, accompanied by tachycardia, excessive sweat, sharp pain in the
epigastric region. Which of the mentioned tumorous diseases are these symptoms the most typical for?
@Pheochromocytoma
Glomerular adrenal adenoma
Pituitary basophil adenoma
Ovarian tumor
Thyroid adenoma
#
80
A 40-year-old patient complains of intensive heartbeats, sweating, nausea, visual impairment, arm tremor, hypertension.
From his anamnesis: 2 years ago he was diagnosed with pheochromocytoma. Hyperproduction of what hormones causes
the given pathology?
Aldosterone
@Catecholamines
Glucocorticoids
Thyroidal hormones
ACTH
#
81
A girl is diagnosed with adrenogenital syndrome (pseudohermaphroditism). This pathology was caused by hypersecretion
of the following adrenal hormone:
@Androgen
Estrogen
Aldosterone
Adrenalin
Cortisol
#
82
An 18-year-old man with asthenic body constitution: tall, narrow shoulders, broad pelvis and with poor hair on his face
came to the geneticist. There was marked mental retardatoin. The preliminary diagnosis was Klinefelter's syndrome. What
method of medical genetics can confirm the diagnosis?
Population-statistic
Dermatoglyphics
@Cytogenic
Gemellary
Genealogic
#
83
Parents of a 10 year old boy consulted a doctor about extension of hair-covering, growth of beard and moustache, low
voic E. Intensified secretion of which hormone must be assumed?
Of progesterone
Of oestrogen
Of cortisol
Of somatotropin
@Of testosterone
#
84
Testosterone and it's analogs increase the mass of skeletal muscles that allows to use them for treatment of dystrophy.
Due to interaction of the hormon with what cell substance is this action caused?
Ribosomes
@Nuclear receptors
Proteins-activators of transcription
Chromatin
Membrane receptors
#
85
During puberty man's sexual glands produce in a blood man's sexual hormone testosterone which is responsible for
development of the secondary sexual features. What cells of spermaries synthesize this hormone?
@Leydig cells
Sertoli cells
Spermatozoons
Fibroblasts
Spermatids
86
During pubescence the cells of male sexual glands begin to produce male sex hormon testosterone that calls forth
secondary sexual characters. What cells of male sexual glands produce this hormone?
@Leidig cells
Spermatozoa
Sertoli's cells
Sustentocytes
Supporting cells
#
87
Intake of oral contraceptives containing sex hormones inhibits secretion of the hypophysiae hormones. Secretion of which
of the indicated hormones is inhibited while taking oral contraceptives with sex hormones?
Thyrotropic
@Follicle-stimulating
Somatotropic
Oxytocin
Vasopressin
#
Disease of the nervous system
(костно-мышечный аппарат)
#
1
An elderly man presented to the hospital with progressive dementia (aphrenia), with marked emotional lability; thus the
focal neurologic symptoms were absent. After his death, a post-mortem revealed the cortex thinning in frontal, occipital
and temporal lobes of the brain. A histological study of a brain's tissue showed the considerable quantity of senile plaques
(especially in a cortex), reduction of neurones' sizes with cytoplasm vacuolation. In addition, the little Hirano bodies were
determined in dendrites. What is the most likely pathology in the central nervous system?
@Alzheimer's disease
Lateral amyotrophic sclerosis
Disseminated sclerosis
Tick-borne encephalitis
Viral meningoencephalitis
#
2
An autopsy of the 68-year-old man revealed the atrophy of brain's tissue. The cerebral atrophy mainly involved the
frontal, temporal, and occipital regions. A hydrocephaly was also found. A microscopical study showed atrophic zones in
the cerebral cortex, the senile (neuritic) plaques in hippocampus, neurofibrillary "tangles", damage of neurons and Hirano
bodies. What is the most likely main disease?
@Alzheimer's disease
Lateral amyotrophic sclerosis
Disseminated sclerosis
Encephalitis
Ganglioneuroma
#
3
An autopsy of the elderly woman revealed a massive purulent bronchopneumonia. A brain was slightly atrophic, gyri
were thinned and sulci were deep. A microscopical study of a brain showed perivascular deposits of amyloid, Hirano
bodies and neurofibrillary "tangles". What is the most likely main disease?
@Alzheimer's disease
Pick's disease
Parkinson's disease
Disseminated sclerosis
Lateral amyotrophic sclerosis
#
4
A 75-year-old man, with a history of Alzheimer's disease, died of a bronchopneumonia. A microscopical study of the
cerebral cortex revealed the senile (neuritic) plaques, neurofibrillary "tangles" and damaged neurons. What formations
characterize cytoskeleton pathology of proximal dendrites at the Alzheimer's disease?
@Hirano bodies
Councilman's bodies
Negri bodies
Mallory's bodies
Russell bodies
#
5
An autopsy of the 78-year-old woman revealed a massive purulent bronchopneumonia. There were numerous grey color
plaques, up to 15 mm in diameter, found in the brain and spinal cord. A microscopical study of brain and spinal cord
tissue showed plural perivenular sites of a demyelization with limphoplazmocytes infiltration and the plural foci of
gliosis. Diagnose a basic disease.
@Disseminated sclerosis
Pick's disease
Parkinson's disease
Alzheimer's disease
Lateral amyotrophic sclerosis
#
6
A 42- year-old forester, a week later after long stay in spring wood, presented to the hospital with a fever, a headache,
consciousness disturbance and epileptoid attacks. He died in 4 days. An autopsy revealed a brain edema, plural petechial
hemorrhages. A microscopical study showed perivascular and pericellular edema, plural perivascular, mainly
lymphocytic, infiltrates. Diagnose the basic disease.
@Tick-borne encephalitis
Meningococcal infection
Poliomyelitis
Purulent encephalitis
Cerebro-vascula disease
#
7
An autopsy of a patient, with a lifetime history of numerous pathological fractures, revealed changes of his long tubular
bones. The femurs and tibia bones were curved or spiral with uneven surface and the obliterated bone-marrow channels
on a cut. There was also a remodeling of a compact cortical layer into the spongy type. Microscopical investigation
showed mosaicity of osteal structures. There were numerous cavities of the resorptions combined with osteoblastic lying
down of new bone. These were accompanied with chaotic fme-fibered or lamellar structures of osteal fragments. The
arteries were twisted and dilated. What is the most likely diagnosis?
@Deforming osteodystrophy
Osteopetrosis
Parathyroid osteodystrophy
Fibrous dysplasia
Chronic osteomyelitis
#
8
A 20-year-old woman with a history of pathological cross- striated muscular weakness and ptosis died of asphyxia. An
autopsy revealed the follicular hyperplasia of a thymus, atrophy and a necrosis of skeletal muscles with focal lymphocytic
infiltrates among the cells. Specify the form of a muscles lesion.
@Myasthenia
Atrophy myositis
Progressive myopathy
Mussels Leyden's dystrophy
Mussels Erb's dystrophy
#
9
A 20-year-old woman presented with a several years history of muscular weakness, resulted in the motion, talking,
chewing and swallowing problems. A physical examination revealed the ptosis of both eyelids. The thorax
roentgenography found a tumorous conglomerate in anterior mediastinum. Histological investigation of the tumor biopsy
determined a thymoma. There were also some dystrophic alterations in skeletal muscles. Diagnose the disease.
@Myasthenia
Progressive mussels dystrophy
Mussels Erb's dystrophy
Dermatomyositis
Poliomyelitis
#
10
A 20-year-old woman presented with weakness of the eye, chewing, speech and swallowing groups of muscles. Thus, the
normal muscles contraction were completely ceased after the vigorous activity and restored again after the rest. Later, the
muscles of extremities and intercostals were also involved in pathological process. Inadequate ventilation of the lungs
resulted in secondary focal pneumonia, followed by patient's death. An autopsy showed the atrophy of cross- striated
muscles, their dystrophy with focal aggregates of lymphocytes in interstitium. In the enlarged thymic gland the follicular
hyperplasia was found out. Specify the most probable diagnosis:
@Myasthenia
Verdniga-Goffman's spinal amyotrophy
Dyushen's mussels dystrophy
Lateral amyotrophic sclerosis.
Erb's mussels dystrophy
#
Tooth
#
1
Painfulness of tooth and edema of lower part of face at the side of ill tooth are present in a patient with acute pulpitis.
What mechanism of edema development is leading one in this case?
@Disorders of microcirculation at the focus of injury
Increase in production of aldosterone
Disturbances of trophic function of nervous system
Disorder of neural regulation of water-salt metabolism
Hypoproteinemia
# INFECTIOUS DISEASES

Sepsis. Bacterial infections diseases.


#
1
Acute infective endocarditis differs from subacute endocarditis in all the following respects EXCEPT
The time required for the lesion to develop
The nature of the preponderant organism
Embolization and dissemination
@The nature of valvular vegetations
The causative organism
#
2
At the section of a young man, who died being in comatose condition, were revealed: extensive thromboembolic
infarction of the left hemisphere, large septic spleen, immune complex glomerulonephritis, and ulcers in the aortic valve
hidden by polyp-like thrombi with staphylococcal colonies. What disease led to the cerebral thromboembolism?
@Septic bacterial endocarditis
Septicemia
Acute rheumatic valvulitis
Septicopyemia
Rheumatic thromboendocarditis
#
3
An autopsy of a 48-year-old man revealed the infarct of the left hemisphere of a brain. A macroscopic study also showed
a big septic spleen, immunocomplex glomerulonephritis, ulcers in aortal valve leaflets, covered with polypous thrombi.
The latter ones had colonies of staphilococcuses, determined by microscopic study. What disease resulted in cerebral
thromboembolism?
@ Septic bacterial endocarditis
Septicemia
Acute rheumatic valvulitis
Septicopyemia
Rheumatic thromboendocarditis
#
4
Extensive thromboembolic infarction of the left cerebral hemispheres, large septic spleen, immunocomplex
glomerulonephritis, ulcers on the edges of the aortic valves, covered with polypous thrombus with colonies of
staphylococcus were revealed on autopsy of the young man who died in coma. What disease caused cerebral
thromboemboly?
Septicemia
Rheumatic thromboendocarditis
Acute rheumatic valvulitis
Septicopyemia
@Septic bacterial endocarditis
#
5
A 32-year-old man with a long history of a drug abuse, presented with a fever, the intoxication and a pyuria. An
echocardioscopy revealed massive superimpositions (vegetations) on the heart valves. Microbiological blood test showed
a presence of a staphylococcus. The patient died of a thromboembolism of a pulmonary artery. Specify the changes of
heart, revealed at post-mortem?
@Polypous-ulcerative endocarditis
Libman-Sacks endocarditis
Diffuse valvulitis
Acute warty endocarditis
Recurrent warty endocarditis
#
6
A post-mortem revealed on the external surface of the aortal valve the large, 1-2 cm, brownish-red, easily crumbling
superimpositions (vegetations). They covered ulcerative defects in the valves. What is the most likely diagnosis?
@Polypous-ulcerative endocarditis
Recurrent warty endocarditis
Acute warty endocarditis
Fibroplastic endocarditis
Diffuse endocarditis
#
7
On autopsy it was revealed: large (1-2 cm) brownish-red, easy crumbling formations covering ulcerative defects on the
external surface of the aortic valve. What is the most likely diagnosis?
Fibroplastic endocarditis
Recurrent warty endocarditis
Diffusive endocarditis
@Polypus-ulcerative endocarditis
Acute warty endocarditis
#
8
A 46-year-old man died of an odontogenic sepsis. An autopsy revealed the marked thickening of semilunar aortic valves.
They were whitish, opaque and stiff, with thrombotic masses, 1x1.5 cm in the size on the external surface. What is the
most likely form of an endocarditis?
@Polypous-ulcerative endocarditis
Diffuse endocarditis
Acute warty endocarditis
Fibroplastic endocarditis
Recurrent warty endocarditis
#
9
A 48-year-old patient with a history of croupous pneumonia died of the pulmonary and heart failure. A post-mortem
revealed a hyperplasia of a spleen, a thickening and defects of the aortal valve leaflet with large, up to 2 cm in diameter,
thrombuses attached to tissues underneath. What is the most likely diagnosis?
@Polypous-ulcerative endocarditis
Acute warty endocarditis
Recurrent warty endocarditis
Diffuse endocarditis
Fibroplastic endocarditis
#
10
A patient that had used drugs for a long time had fever, the facts of intoxication, pyuria. Echocardioscopy revealed
massive stratification on the cardiac valves. Staphylococcus was cultured in the blood. A patient died of pulmonary artery
thromboembolism. Indicate on the changes of the heart that were revealed during the section.
@Polypous-ulcerative endocarditis
Libman-Sacks endocarditis
Diffuse valvulitis
Acute verrucous endocarditis
Recurrent verrucous endocarditis
#
11
A patient fell ill with diphtheria. On the 2 nd week of the disease acute heart failure developed that caused the death. At the
section: the ventricles of the heart are dilated, myocardium is of soft consistence, mottled. Histologically were revealed
numerous small focuses of myolysis with small perifocal lymphoid infiltration. What caused acute heart failure?
Metabolic necroses of myocardium
Bacterial myocarditis
Septic myocarditis
Myocardial infarction
@Bacterial exotoxic myocarditis
#
12
A 28-year-old woman died in the postnatal period. A post-mortem revealed a purulent endometritis and thrombophlebitis
of a uterus veins, plural abscesses of lungs, kidneys and a spleen. In addition, there were an apostematous myocarditis and
purulent meningitis. What is the most likely kind of a sepsis?
@Septicopyemia
Septicemia
Chroniosepsis
The prolonged septic endocarditis
-
#
13
A 20 year old patient died from intoxication 8 days after artificial illegal abortion performed in her 14-15th week of
pregnancy. Autopsy of the corpse revealed yellowish colour of eye sclera and of skin, necrotic suppurative endometritis,
multiple pulmonary abscesses, spleen hyperplasia with a big number of neutrophils in its sinuses. What complication after
abortion was developed?
Septicemia
Chroniosepsis
Hemorrhagic shock
@Septicopyemia
Viral hepatitis type A
#
14
A 30-year-old woman with a history of abortion presented with purulent endometritis resulted in a death from sepsis. A
post-mortem revealed a purulent lymphadenitis, abscesses in lungs, kidneys, a myocardium, a spleen hyperplasia, and
dystrophy of parenchymatous organs. Define the clinico-anatomic form of a sepsis.
@Septicopyemia
Septicemia
Septic endocarditis
Granulomatous sepsis
Chroniosepsis
#
15
Purulent endometritis with fatal outcome was progressing in the woman after abortion performed not at the hospital. On
autopsy multiple lung abscesses, subcapsule ulcers in the kidneys, spleen hyperplasia were revealed. What form of sepsis
developed in the patient?
Chroniosepsis
@Septicopyemia
Lung sepsis
Septicemia
Urosepsis
#
16
A man with a history of a lower extremity wound, presented with persistent pyesis of a lesion, resulted in death of
intoxication. An autopsy revealed a cachexia, dehydration; a brown atrophy of a liver, a myocardium, a spleen and cross-
striated muscles. In addition, there was amyloidosis of kidneys. What of the listed diagnoses is most probable?
@ Chroniosepsis
Septicopyemia
Septicemia.
Chemogubov's disease.
Brucellosis
#
17
At the section of a 18-year-old man the spleen is 580g, of dark-red color with big smear of pulp. Histologically were
diagnosed a significant proliferation of reticular cells, presence of a big amount of mature neutrophiles in sinusoid
capillaries. How can we term such a spleen?
Sago
@Septic
Porphyry
Cyanotic
Leukosal
#
18
All the following have been commonly associated with pyogenic brain abscesses EXCEPT
Congenital heart disease
Sinusitis
Lung abscess
@Liver abscess
mastoiditis
#
19
Autopsy of a man who died from the sepsis in his femoral bone revealed phlegmonous inflammation that affected the
marrow, haversian canals and periosteum. Under the periosteum there are multiple abscesses, adjoining soft tissues of
thigh also have signs of phlegmonous inflammation. What pathological process was described?
Osteopetrosis
Chronic hematogenous osteomielitis
Osteoporosis
@Acute hematogenous osteomyelitis
-
#
20
A morphological research of a femur reveled the chronic purulent inflammation of compact substance and a bone marrow
with sequesters formation. What is the most likely disease?
@Osteomyelitis
Retikulosarcoma
Multiple (plasma cell) myeloma (Kahler's disease)
Osteoblastoclastoma
Periostitis
#
21
A 48- year-old man presented with advanced staphylococcal purulent periodontitis, resulted in a purulent inflammation of
alveolar process' bone marrow and then a mandible's body. Microscopical investigation showed thinned osteal beams, the
necrosis centers and the osteal sequesters, surrounded with a connective tissue's capsule. What is the most likely disease?
@Chronic osteomyelitis
Acute osteomyelitis
Parodontome
Chronic fibrous periostitis
Purulent periostitis
#
22
A 53- year-old man died of intoxication. An autopsy revealed the enlargement of his hip with some fistulas on a skin.
There was yellow - green thick liquid discharge from the fistulas. The gross investigation showed the thickened femur
with sequesters formation. A microscopical study determined sequestral cavities, surrounded by granulation and
connective tissue with neutrophil infiltration. The bone-marrow channels were obliterated; the compact layer was
thickened. What is the most likely diagnosis?
@Chronic purulent osteomyelitis
Acute gematogenic osteomyelitis
Tuberculous osteomyelitis
Parathyroid osteodystrophy
Osteopetrosis
#
23
A 52-year-old man with a 10 years history of chronic osteomyelitis and 3 years a nephritic syndrome, died of uremia. An
autopsy revealed dense, white kidneys, with some cicatrices in their cortical layer. They had sebaceous glitter on a cut
surface. Specify a pathology of kidneys which has developed.
@Secondary amyloidosis
Primary amyloidosis
Idiopathic amyloidosis
Chronic glomerulonephritis
Chronic pyelonephritis
#
24
A young man died of intoxication on 4th day after eating the crude eggs. An autopsy revealed the stomach and small
bowel mucosa inflamed, covered by a mucous exudate. In addition, abscesses were found in lungs, brain and liver. What
diagnosis is the most likely?
@Salmonellosis (septic form)
Dysentery
Salmonellosis (typhoid form)
Salmonellosis (intestinal form)
Typhoid
#
25
In the autopsy of a 56 year-old man the pathologist has found out in terminal fragment of a small bowel some ulcers about
1 cm in areas of lymphoid follicles, coated with greyish necrotic masses. Vidal reaction was positive. Diagnose this
disease:
@Typhoid fever.
Paratyphoid
Reccurent fever
Dysentery
Crohn’ disease
#
26
Multiple oval ulcers along the intestine were revealed on autopsy of the person, who died from diffuse of peritonitis in the
distant part of the small intestine. Bottom parts of the ulcers are clear, smooth, formed with muscular or serous covering,
edges of ulcers are flat, rounded. There are perforations up to 0,5 cm in diameter in two ulcers. What diseases’ can be
diagnosed?
Tuberculosis
Typhus
@Typhoid fever
Dysentery
Cholera
#
27
Few ulcers from 4 to 5cm in size were revealed in the terminal part of small intestine at the section of a 56-year-old man.
The edges of ulcers elevate above the surface of mucous membrane. The walls of ulcers are covered with friable greyish-
yellowish masses. The Widal’s reaction is positive. Make a diagnosis.
@Abdominal typhoid
Paratyphoid
Recrudescent typhus
Shigellosis
Crohn’s disease
#
28
Numerous ulcers of an oval form, located along the distal part of small intestine, were revealed at the section of a man,
who died of diffuse peritonitis. Bottoms of ulcers were clean, plain, formed of muscular or serous membrane. The edges
of ulcers were even and rotund. Two ulcers had perforative holes with the diameter to 0,5cm. What disease can we
suspect?
@Typhoid (abdominal typhoid)
Shigellosis
Cholera
Tuberculosis
Spotted fever
#
29
The death of a 16-year-old patient was caused by diffuse fibrinous-purulent peritonitis. At the autopsy an ulcer was
revealed in the lower part of small intestine, which repeated the form of Peyer’s plaque and perforated the wall of the
intestine. Microscopically: the picture of lymphoid tissue was erased and superseded by proliferative monocytes that form
granulomas. The complication of what disease is meant?
@Abdominal typhoid
Shigellosis
Cholera
Brucellosis
Non-specific ulcerative colitis
#
30
A 16-year-old patient died because of diffuse fibrinopurulent peritonitis. The autopsy in the inferior region of small
intestine revealed an ulcer with a form of Peyer's plaque, with the perforation of intestine wall. The microscopic
examination revealed lymph tissue pattern vagueness, the displacement of this tissue with proliferating monocytes, which
were forming granulomata. Complication of what disease caused the death?
Brucellosis
Nonspecific ulcerative colitis
Cholera
@Typhoid
Dysentry
#
31
A patient died of a diffuse peritonitis. An autopsy revealed in the distal part of a small bowel multiple oval ulcers with
rounded edges, which were parallel to the longitudinal axis of the intestine. A bottom of ulcers was clean, smooth and
glitter. Base of the ulcers presented either by muscular layer of ileum or its serosa. Two ulcers had perforation apertures
0,5cm in diameter each. What disease resulted on a death of a patient?
@Typhoid
Dysentery
Cholera
Tuberculosis
Epidemic typhus
#
32
An autopsy at the Forensic Pathology Department of an unknown man, with a history of alcohol intoxication, revealed in
his small bowel the enlarged Peyer's patches, which protruded above the mucosal surface. They were soft, with irregular
surface, which reminded brain's gyri and sulci. The cut surface had a gray-red coloring. For what disease the described
changes are characteristic?
@Typhoid
Dysentery
Lambliasis
Amebiasis
Enteritis due to Campylobacter
#
33
A patient presented with a sustained fever, enlarged and painful spleen. On the 10 th day of a disease a rash of flat, rose-
colored spots appeared. On the 21 st hospital day he died after developing peritonitis. An autopsy revealed in the ileum
deep ulcers within the area of necrotized group of lymphoid follicles (Peyer's patches). One of ulcers was perforated;
there was a fibrinopurulent diffuse peritonitis. What is the most likely disease?
@Typhoid
Dysentery
Intestine amebiasis
Cholera
Salmonellosis
#
34
A 38-year-old man died of intoxication. A post-mortem revealed in a small bowel an edema of lymphoid follicles groups.
The follicles protruded above the mucosal surface like a soft plaques with irregular surface pattern, which reminded
brain's gyri and sulci. What diagnosis is most probable?
@Typhoid
Dysentery
Salmonellosis
Acute enteritis
Cholera
#
35
A 38-year-old man died of diffuse fibrinopurulent peritonitis. A post-mortem revealed an ulcer in a small bowel, which
replicated a form of a Peyer's patch and perforated the intestine's wall. Microscopical investigation showed an alteration
of lymphoid tissue pattern with replacement of lymphocytes by proliferating monocytes, followed by granulomas
formation. The complication of what disease resulted in patient's death?
@Typhoid.
Dysenteries.
Cholera.
Brucellosis
Nonspecific ulcerative colitis.
#
36
A 44-year-old man presented to a surgery department with diffuse fibrinopurulent peritonitis. An operation revealed an
ulcer of ileum with a perforation. An ulcer closure, followed by abdomen cavity drainage was performed. Four days later
patient died. At post-mortem iliac ulcers of Peyer's patches, parallel to the longitudinal axis of the intestine, were found.
A base of the ulcers was presented by serosa. For what disease such changes are characteristic?
@Typhoid
Intestine tuberculosis
Dysentery
Intestinal tumours
An intestine amebiasis
#
37
An autopsy of a 45-year-old man revealed in his small bowel the enlarged, hyperemic groups of lymphoid follicles. The
follicles protruded above the mucosal surface; their surface reminded child's brain gyri and sulci. Microscopical
investigation showed monocytes, histiocytes and reticular cells proliferation. Macrophages formed aggregates (typhoid
granulomas), which replaced lymphocytes. For what disease the described changes are characteristic?
@Typhoid
Cholera
Dysentery
Salmonellosis
Amebiasis
#
38
A 52-year-old woman died in surgery department of a diffuse purulent peritonitis. A post-mortem revealed in a distal part
of a small intestine enlarged Peyer's patches, which projected into the intestine's lumen. There surface was covered by
brownish-greenish scab. In the centre of some patches were found deep defects, extended to a serous layer. For what
disease described intestinal changes are characteristic?
@Typhoid
Dysentery
Intestine tuberculosis
Crohn's disease
Staphylococcal enteritis
#
39
An autopsy of a 53-year-old man revealed few 4-5cm ulcers. The margins were elevated above the mucosa. The walls of
ulcers were covered by the yellowish-grayish crumble masses. The Widal test was positive. Diagnose a disease?
@Typhoid
Paratyphoid
Recurrent typhus
Dysentery
Crohn's disease
#
40
At the section of a man, who was ill with abdominal typhoid, the following changes in small intestine were traced:
enlarged aggregated lymphoid follicles, elevated above the surface of the mucous membrane, of gray-red color, succulent;
their surface had the outlook of convolutions and grooves. The microscopical exam revealed the formation of typhoid
granulomas. What stage of abdominal typhoid is this picture typical for?
Formation of ulcers
Necrosis
@Brain-like swelling
Clean ulcers
Healing
#
41
An autopsy revealed groups of enlarged, hyperemic lymphoid follicles in small bowel. The follicles protruded above the
mucosal surface; their surface reminded brain's gyri and sulci. The cut surface of follicles was succulent; with gray-red
coloring. Microscopical investigation showed monocytes and histiocytes proliferation. Macrophages formed aggregates
(typhoid granulomas). Name a morphological stage of typhoid.
@Medullar swelling
Necrotic
Formation of ulcers
Pure ulcers
Healing
#
42
An autopsy of a 45-year-old man, who died on the 5 th day of typhoid fever, revealed groups of enlarged, hyperemic
lymphoid follicles in small bowel. The follicles protruded above the mucosal surface; their surface reminded brain's gyri
and sulci. Microscopical investigation showed hyperemia, edema and granulomas formation. Specify, what period of
local changes of typhoid is the most likely?
@Stage of medullar swelling
Necrotic stage
Stage of healing
Stage of pure ulcers
Stage of ulcers' formation
#
43
An autopsy of a 48-year-old man, with a history of typhoid fever, revealed groups of enlarged lymphoid follicles in small
bowel. The follicles protruded above the mucosal surface; they had a grey- red coloring, juicy appearance and a surface,
which reminded gyri and sulci. Microscopical investigation showed granulomas formation. Specify, what of the listed
stages of typhoid is the most likely?
@Medullar swelling.
Necrotic.
Formation of ulcers.
Pure ulcers.
Healing.
#
44
A 42-year-old patient, with a history of dysentery, presented with symptoms of paraproctitis. What is the most probable
stage of local changes presented in that case?
@Formation of ulcers
Fibrinous colitis
Follicular colitis
Catarrhal colitis
Stage of healing
#
45
A post-mortem of a patient, who died of diffuse fibropurulent peritonitis, revealed in the wall of a small bowel multiple
oval ulcers with rounded edges, which were parallel to the longitudinal axis of the intestine. A base of the ulcers
presented either by muscular layer of ileum or its serosa. Two ulcers had perforation apertures 0,3cm in diameter each.
Define a typhoid stage.
@Stage of pure ulcers
Stage of "dirty ulcers" formation
Necrotic stage
Stage of a medullar swelling
Stage of healing
#
46
A man, with 4 weeks history of the typhoid, presented with symptoms of 'acute abdomen'. He died soon of intoxication.
At post-mortem a pathologist found a perforation of iliac wall and diffuse purulent peritonitis. For what stage of disease
the described complication is characteristic?
@Stage of pure ulcers
Stage of a medullar swelling
Necrotic stage
Stage of healing of ulcers
Bacteriemia
#
47
A post-mortem of a 57-year-old male, with a history of typhoid, revealed a dense, fragile whitish-yellowish color of the
rectus abdominis and hip muscles. They reminded a stearin candle. The described changes are the result of:
@Zenker's necrosis
Fibrinoid necrosis
Caseous necrosis
Colliqative necrosis
Apoptosis
#
48
A 48-year-old patient with severe typhoid developed an acute renal failure, resulted in a lethal outcome of a disease. An
autopsy revealed enlarged and swollen kidneys. On a cut section, the cortex was pale grey, while pyramids were dark red.
A histological investigation showed that in the majority of tubules the lumen was narrowed, epithelial cells were enlarged
and lost their nuclei; glomeruli were collapsed. A renal stroma was edematous with a small leukocytic infiltration and fine
hemorrhage. What pathology of kidneys presented in that case?
@Necronephrosis
Acute pyelonephritis
Acute glomerulonephritis
Pyonephrosis
Hydronephrosis
#
49
A patient with suspicion on epidemic typhus was admitted to the hospital. Some arachnids and insects have been found in
his flat. Which of them may be a carrier of the pathogen of epidemic typhus?
@Lice
Bed-bugs
Cockroaches
Spiders
Houseflies
#
50
A sick man with high temperature and a lot of tiny wounds on the body has been admitted to the hospital. Lice have been
found in the folds of his clothing. What disease can be suspected?
Malaria
@Epidemic typhus
Tularemia
Plague
Scabies
#
51
A young man presented with rash in the form of roseolas and small hemorrhages (petechias) on a skin of a stomach,
thorax and petechial rash on eyes conjunctiva. Lesion of his brain soon resulted in his death. A microscopical
investigation revealed in destructive-proliferative endo-trombovasculitis in medullar oblongata and pons of a brain, also
in a skin, kidneys and myocardium. What is the most likely disease?
@Epidemic typhus
Sepsis
Periarteritis nodosa
Systemic Lupus Erythematosus
Brucellosis
#
52
A patient died as a result of a heart failure. Macroscopically traces of a pre-existed rash in the form of maculae and spots
were recognized. A gross investigation revealed decubituses at the areas of breeches and spinous processes of vertebra.
Microscopical study showed a destructive-proliferative endo-trombovasculitis and Popov's granulomas in the central
nervous system, in a skin, and adrenals. An interstitial myocarditis was diagnosed in his heart. What is the most likely
disease?
@Epidemic typhus
Q fever
Typhoid fever
Periarteritis nodosa
HIV-infection
#
53
A 48-year-old male presented with a sustained high fever, severe headache, a dyspnea, palpitation. A physical
investigation revealed a pediculosis, a rash in the form of roseolas and small hemorrhages (petechias) on his thorax skin.
In addition, decubituses of legs and foot gangrene were also determined. He died of a heart failure. A histological study
showed in the medullar oblongata a hyperemia, stasis, perivascular plasmocytes infiltrates and a proliferation of a
microglia (Popov's granuloma). What is the most likely diagnosis?
@Epidemic typhus
Antrax meningocephalitis
Meningococcal meningitis
Typhoid fever
Rubella
#
54
A patient complained of fever, severe headache, dyspnoae, and heartbeating. Physical exam revealed pediculosis, roseola
and petechia on the skin of the thorax, bedsores of shanks, and a gangrene of a foot. The patient died with the signs of
heart failure. Histologically in the medulla tissue hyperemia, stases, perivascular ferrules of plasmatic cells, and focuses
of microglial proliferation (Popov’s granulomas) were detected. What is your diagnosis?
Siberian (anthrax) meningoencephalitis
@Epidemic typhus
Meningococcal meningitis
Abdominal typhoid
German measles
#
55
Skin of a man who died from cardiac insufficiency has an eruption in form of spots and specks. There are also bedsores in
the area of sacrum and spinous vertebral processes. Microscopical examination of CNS, skin, adrenal glands revealed in
the vessels of microcirculatory bed and in small arteries destructive-proliferative endothrombovasculitis with Popov's
granulomas; interstitial myocarditis. What diagnosis corresponds with the described picture?
@Spotted fever
HIV
Enteric fever
Q fever
Nodular periarteritis
#
56
A patient died 3 days after the operation because of perforated colon with the manifestations of diffuse purulent
peritonitis. The autopsy demonstrated: colon mucos membrane was thickened and covered with a fibrin film, isolated
ulcers penetrated into different depth. Results of histology: mucous membrane necrosis, leukocytes infiltration with
hemorrhages focuses. The complication of what disease caused the patient's death?
Nonspecific ulcerative colitis
Typhoid
Crohn's disease
Amebiasis
@Dysentery
#
57
Histological investigation of a rectum revealed large areas of necrosis in mucosa. Necrotic masses were impregnated by
fibrin, resulting in membrane formation. Mucous and submucous on periphery of necrotic zones were hyperemic,
edematous, with hemorrhages and leukocytic infiltration. What is the most likely diagnosis?
@Dysentery
Cholera
Amebiasis
Typhoid
Salmonellosis
#
58
A male patient, with 5 days history of diarrhea, had a colonoscopy. The investigation revealed a colon mucosa
inflammation with grey-green membranes, intimately connected with tissue underneath. What is the most likely
diagnosis?
@Dysentery
Typhoid
Nonspecific ulcerative colitis
Salmonellosis
Crohn's disease
#
59
An autopsy of a child, with a history of diarrhea, revealed an exicosis and a widespread fibrinous colitis. A mucosal
smear imprint study showed gram-negative rod bacteria. What is your diagnosis?
@Dysentery
Cholera
Staphylococcal intestinal infection
Typhoid
Salmonellosis
#
60
A 7-year-old child, on the 3 rd week of disease, presented with hectic fever, skin and sclera icterus. He died soon of the
hepatorenal failure. An autopsy reveled in a rectum many irregular form defects with uneven edges. These defects were
covered by grey-white membranes, intimately soldered to a tissue underneath. In addition, there was a purulent
thrombophlebitis of the intestinal vessels. Plural pylephlebitic abscesses and fatty dystrophy of hepatocytes were
diagnosed in a liver. An acute necrotic nephrosis was found in kidneys. What disease caused a lethal outcome?
@Dysentery
Salmonellosis
Cholera
Typhoid
Amebiasis
#
61
An autopsy of an 8-year-old child revealed in his colon multiple irregular form and various depth defects with uneven
edges. In addition, there were grey-white membranes intimately connected with tissues underneath. What is the most
likely diagnosis?
@Dysentery
Salmonellosis
Cholera
Typhoid
Amebiasis
#
62
A 30-year-old patient with bacteriologically proved dysentery developed the signs of paraproctitis. What is the stage of
local changes in this patient?
Follicular colitis
Fibrinous colitis
Catarrhal colitis
Healing of the ulcers stage
@Ulceration stage
#
63
A 39-year-old patient died on the 4 th day after operation concerning perforation of a colon wall, resulted in diffuse
purulent peritonitis. An autopsy revealed the mucosa of a colon wall thickened, covered by fibrinous membrane. Some
single ulcers got on different depth. Histological study of a colon wall showed a necrosis of mucous, a fibrinous
inflammation, infiltration by leucocytes with focuses of hemorrhages. What complication of disease became a cause of
patient's death?
@Dysentery.
Typhoid.
Nonspecific ulcerative colitis.
Crohn's disease
Amebiasis.
#
64
Autopsy of a 46-year-old man revealed multiple brown-and-green layers and hemmorhages on the mucous membrane of
rectum and sigmoid colon; slime and some blood in colon lumen; histologically - fibrinous colitis. In course of
bacteriological analysis of colon contents S.Sonne were found. What is the most probable diagnosis?
Crohn's disease
@Dysentery
Salmonellosis
Yersiniosis
Cholera
#
65
An autopsy of a 52- year-old man revealed in sigmoid and rectum plural brown-green membranes, hemorrhages, some
blood in the lumen of intestine. Histological study showed a fibrinous colitis. Bacteriological investigation found S.
Sonne. What is the most likely diagnosis?
@Dysentery
Cholera
Salmonellosis
Yersiniosis
Crohn's disease
#
66
An autopsy of a 54-year-old man revealed a fibrinous inflammation at colon. What is your diagnosis?
@Dysentery
Amebiasis
Typhoid fever
Cholera
Balantidiasis
#
67
A 65 -year-old patient addressed to a doctor his complains to frequent urge and painful passage of stool, with mucous and
some blood in feces. Proctosigmoidoscopy revealed marked narrowing of a colon's lumen, especially in its distal part. A
mucosa was edematous, hyperemic, with areas of necrosis and hemorrhages. A bacteriological study of feces determined
the Shigella flexneri. Define the disease?
@Dysentery
Typhoid
Anthrax
Crohn's disease
Paratyphoid

68
A patient who had come backfrom Asia,died from many abscesses of the liver. On the autopsy of the ascending colon
there were ulcers of iirregular forms with overhanging brims the mucous membrane above the gastric fundus collapses
when pushing by probe. What is the most possible disease?
@ Bacterial disentery
Amebiasis
Cholera
Abdominal typhoid
Salmonollosis
#
69
Patients with similar complaints applied to the doctor: weakness, pain in the intestines, disorder of GIT. Examination of
the faeces revealed that one patient with four nucleus cysts should be hospitalized immidiately. For what protozoa are
such cysts typical?
Balantidium
Intestinal amoeba
@Dysenteric amoeba
Trichomonas
Lamblia
#
70
An autopsy of 47- year-old man revealed in sigmoid and rectum plural red ulcers with irregularform. A mucous between
ulcers was covered by dirty grey membrane. What is the most likely etiology of the disease?
@Shigella
Ameba
Mycobacterium tuberculosis
Salmonella
Staphilococcus
#
71
Multiple red ulcers of irregular form were revealed in sigmoid colon and rectum at the autopsy. The mucous membrane
between them was covered with dirty-gray membrane. Name the etiology of the disease.
Mycobacterium tuberculosis
Amoeba
@Shigella
Salmonella
Staphylococcus
#
72
A patient died on the third day after the operation relatively a perforation of the wall of large intestine with the facts of
diffuse purulent peritonitis. At the section: the mucous membrane of large intestine is thickened is covered with fibrinous
membrane. Single ulcers penetrate on different depth. Histological exam reveals necrosis of mucosa, fibrin, and
leukocytic infiltration with focuses of haemorrhages. The complication of what disease caused the death of the patient?
Non-specific ulcerative colitis
Abdominal typhoid
@Shigellosis
Crohn’s disease
Amebiasis
#
73
Colonoscopy of a patient ill with dysentery revealed that mucous membrane of his large intestine is hyperemic, edematic,
its surface was covered with grey-and-green coats. Name the morphological form of dysenteric collitis:
Purulent
Catarrhal
Necrotic
@Fibrinous
Ulcerous
#
74
A 50-year-old man, who acutely fell ill, was diagnosed shigellosis. He died on the seventh day of the disease. The section
revealed thickened wall of sigmoid colon and of initial parts of rectum. There was also fibrous membrane on the surface
of mucosa. Histologically was detected a deep necrosis of the mucous membrane with fibrin infiltration of necrotic
masses. What kind of colitis is meant?
@Diphtheritic
Catarrhal
Ulcerative
Gangrenous
Follicular
#
75
On the autopsy of the patient, who lived in the seaside city in southern parts of the country, with obvious diarrheas and
vomiting, dead from dehydration, was foud the picture of acute gastroenteritis with serous-desquamative
inflammation in small intestine. Chosoe the most correct diagnosis.
@Cholera
Bacterial dysentery
Abdominal typhus
Amebiasis
Salmonellosis
#
76
A man, with a history of frequent drinking non- boiled water from the river, has developed vomiting and explosive 'rice-
water' diarrhea. The abdominal pains were absent; the body temperature did not rise. The intestine biopsy revealed a
plethora, a marked swallowing, an edema of enterocytes, and some infiltration of villi by lymphocytes, plasmocytes, and
few leucocytes. What is the most likely disease?
@Cholera
Salmonellosis
Typhoid fever
Dysentery
Lambliasis
#
77
A patients, the inhabitant of a seaside city in the south of the country, presented to the hospital with severe diarrheas and
vomiting, resulted in his death from dehydration. An autopsy revealed an acute gastroenteritis with serous- desquamative
inflammation in a small bowel. Choose the most likely diagnosis.
@Cholera
Bacterial dysentery
Typhoid
Amebiasis
Salmonellosis
#
78
A 45-year-old man presented in the infectious diseases hospital with a profuse diarrhea, exicosis, acute decline of his
body temperature. He died soon of an uremia. An autopsy revealed in his small bowel lumen a colorless liquid in the form
of rice broth and edematous mucosa. Microscopical investigation of a small bowel showed a hyperemia of vessels, a focal
hemorrhage, a desquamation of enterocytes, a hypersecretion of a beaker (goblet) cells and lympho- leucocytes
infiltration of a mucosal stroma. What is the most likely diagnosis?
@Cholera
Salmonellosis
Dysentery
Typhoid
Crohn's disease
#
79
An autopsy 54-year-old man revealed markedly dilated lumen of a small bowel filled with a liquid, which reminded "rice
broth". The intestine wall was edematous with multiple petechial hemorrhages. For what infectious disease the described
enteritis is characteristic?
@Cholera
Dysentery
Salmonellosis
Amebiasis
Typhoid
#
80
An autopsy of a 65-year-old man, who died in week from the beginning of a profuse diarrhea, revealed severe exicosis,
dry all tissues and thick, concentrated blood. A bacteriological study of contents of a small bowel, which reminded the
rice broth, determined vibrioes. What disease resulted in the patient's death?
@Cholera
Dysentery
Typhoid
Salmonellosis
Alimentary toxic infection
#
Tuberculosis. Syphilis.
#
1
A child died of a diffuse peritonitis. An autopsy revealed the primary intestinal tubercular complex with primary affect, a
lymphangitis and regional caseous lymphadenitis. In addition, an ulcer of an empty intestine with perforation was found
within the primary affect focus. What is the most likely way of tuberculosis infection in presented case?
@Alimentary (nutritional)
Transplacental
Aerogenic
Mixed
Contact
#
2
At the section of a died child was revealed primary intestinal tuberculous complex: primary affect-ulcer of jejunum,
lymphangeitis and regional caseous lymphadenitis. The death was of the ulcer perforation and development of diffuse
peritonitis. Name the way of contamination of the child with tuberculosis.
Transplacental
@Alimentary
Aerogenous
Mixed
Contagious
#
3
An autopsy of a 7-year-old child revealed in the apex of the right lung, under the pleura, a focus of caseous necrosis, 15
mm in diameter. There were also enlarged bifurcational lymph nodes with small necrotic foci. A microscopical
investigation of the pulmonal focus and the lymph node's lesion showed necrotic masses surrounded by epithelioid cells,
lymphocytes and solitary giant multinuclear Pirogov-Langhans' cells. What is the most likely disease?
@Primary tuberculosis
Gematogenic tuberculosis with the lesions of lungs
Gematogenic generalized tuberculosis
Secondary inflammatory tuberculosis
Secondary fibro-inflammatory tuberculosis
#
4
The girl within last 3 months ate the crude cow milk. She arrived in the hospital with the "acute abdomen" symptoms. An
operation revealed in a caecum a circular ulcer with perforation. A histological study of the ulcer edges determined
necrotic masses, lymphocytes, epithelioid cells, and few multinucleated giant cells
What is your diagnosis?
@Primary intestinal affect at tuberculosis
Nonspecific ulcerous colitis
Amebiasis
Carcinoma of blind gut (cecum)
Dysentery
#
5
A histological investigation of the enlarged supraclavicular lymph nodes, removed from a 40-year- old woman, revealed
an alteration of their structure with aggregates of slightly extended cells with a light nucleus. The multinuclear, large,
oval-round form cells were also determined. These cells were rich with a cytoplasm and their nuclei were located on
periphery in the form of a paling. The lymphoid cells created the next layer. The unstructured pinkish masses were seen
within the centers of lesions. What the described changes are characteristic for?
@Tuberculous lymphadenitis
Metastasis of tumor
Lymphogranulomatosis (Hodgkin's disease)
Hyperplasic lymphadenitis
Sarcoidosis (Boeck's disease)
#
6
After introduction of BCG vaccine to babies’ immunity against tuberculosis lasts until in an organism will be alive
bacteria of vaccinal strain. How to name such kind of immunity in the most correct way?
@Nonsterile
Humoral
Typospecific
Congenital
Cross-immunity
#
7
At carrying out of medical inspection of pupils Mantoux test was done. Presence of which from the listed below specific
factors determines the positive reaction?
@Т-lymphocytes
B-lymphocytes
Antibodies
Erythrocytes
Leukocytes
#
8
At 10 years old child Mantoux test (with tuberculin) was done. In 48 hours in the place of tuberculin injection papule 8
mm in diameter has appeared. Which type of hypersensitivity has developed after tuberculin injection?
@IV type of hypersensitivity
Phenomenon of Arthus
Atopic reaction
Serum sickness
II type of hypersensitivity
#
9
Test Mantoux that appeared negative was done in child at his registration to school for the decision of necessity of
revaccination. This result of test testifies about?
@About absence of cellular immunity to a tuberculosis
About presence of cellular immunity to a tuberculosis
About absence of antibodies to tubercular bacteria
About absence of antitoxic immunity to a tuberculosis
About presence of antibodies to tubercular bacteria
#
10
A lung lobe was removed at Surgery Department due to preliminary diagnosed purulent inflammation of a cyst. A gross
investigation of the operative material revealed within the 8 segment a dense focus with irregular roundish form, 1,8 cm
in diameter. On a cut, it had white-yellow coloring with a dim surface and fine sites of osteal density. The described
macroscopical picture characteristic for:
@Tuberculous affect with healing
Peripheral carcinoma of lung
Chondromas
Carnification
Chronic pneumonia
#
11
A post-mortem of a 43-year-old man revealed in the 3 segment of the right lung under the pleura an area of consolidation,
1,5 cm in diameter, with the accurate borders, surrounded with a whitish fibrous tissue. A gross investigation of a cut
section showed the white-yellow crumbling foci. Presence of the described focus is characteristic for:
@Encapsulation of primary affect
Peripheral carcinoma
Chondroma
Fibroma
Infarct of lung with organization
#
12
At the section of a 17-year-old girl, who died of lung insufficiency, were revealed confluent areas of caseous necrosis, the
facts of caseous necrosis in bronchopulmonary, bronchial and bifurcation lymphatic nodules. What is the most possible
diagnosis?
Tuberculoma
Haematogenous form of progressing of primary tuberculosis
Haematogenous tuberculosis with preferable affection of lungs
@The growth of the primary affect at primary tuberculosis
Caseous pneumonia at secondary tuberculosis
#
13
A post-mortem of the 17-year-old girl, who died of a pulmonary insufficiency, revealed confluent zones of a caseous
necrosis in the lower lobe of the right lung. The caseous necrosis was also determined in broncho- pulmonal, bronchial
and bifurcational lymph nodes. What of the listed diagnoses the most likely?
@ Growth primary affect at primary tuberculosis
Gematogenic form of progress of primary tuberculosis
Gematogenic tuberculosis with lesion of lungs
Tuberculoma
Caseous pneumonia at the secondary tuberculosis
#
14
An 8-year-old child, with no disease in his medical history, presented to the hospital with a dyspnea. An x-ray inspection
showed the air and a liquid in a pleural cavity. During operation in the right lung the subpleural cavity, 2,4 cm in diameter
was found. It had an irregular form with the residual caseous masses inside. The lesion was connected with the pleural
cavity. The similar caseous masses were found at a cut of the enlarged radical lymph nodes. A histological research of a
cavity's wall determined lymphocytes, epithelioid cells and multinuclear giant cells. How correctly to name a cavity?
@Primary pulmonary cavity (cavern) at tuberculosis
Abscess of lungs
Empyema of pleura
Cavity (cavern) at acute cavernous tuberculosis
Cavity at fibrocavernous tuberculosis.
#
15
An autopsy of a 44-year-old man revealed tuberculosis. A macroscopical investigation showed in the apex of the right
lung the foci of caseous pneumonia. There were caseous lymphadenitis of enlarged lymph nodes of mediastinum and
plural milium nodules in many organs. Name the described form of tuberculosis?
@Primary with mixed form of progressing
Primary with gematogenic spreading
Primary with lymphogenic spreading
Primary with growth of a primary affect
Gematogenic
#
16
A medical record of a dead child reveled meningeal symptoms in his medical history. His X-ray showed a dark patch in
the III segment of the right lung and enlarged radical lymph nodes. At autopsy, in cerebral arachnoids, the millet grains-
like nodules were found. Microscopical study of a nodule determined a focus of a caseous necrosis with wall of
epithelioid, lymphoid cells with giant cells between them. These giant cells had big nuclei, situated on periphery in the
form of a half moon. What is the most likely kind of meningitis?
@Tuberculous
Syphilitic
Brucellous
Influenzal
Meningococcal
#
17
A 26-year-old patient arrived in a hospital with complaints to a fever and weakness. One of the enlarged cervical lymph
nodes was taken for histological study. Microscopical investigation showed in lymphoid tissue the centers of necrosis,
surrounded with epithelioid cells, giant multinuclear Pirogov-Langhans cells and lymphocytes. What is the most likely
disease?
@Tuberculosis
Lymphogranulomatosis (Hodgkin's disease)
Lymphatic leukemia
Sarcoidosis (Boeck's disease)
Syphilis
#
18
A histological investigation of the cervical lymph node revealed the aggregation of epithelioid cells, lymphocytes and
giant Pirogov - Langhans cells. In the centre of a lesion a caseous necrosis was present. Specify the most likely pathology.
@Tuberculosis
Rhinoscleroma.
Sarcoidosis (Boeck's disease)
Glanders
Syphilis
#
19
A histological study of the enlarged cervical lymph nodes revealed nodules with giant, spherical-oval form cells. These
cells had a light pink cytoplasm and nuclei located on a periphery. The found out changes are characteristic for:
@Tuberculosis
Lymphogranulomatosis (Hodgkin's disease)
Sarcoidosis (Boeck's disease)
Brucellosis.
Nonspecific hyperplastic lymphadenitis
#
20
A 24-year-old man presented with enlarged cervical lymphatic node. A histological investigation revealed the growth of
epithelioid, lymphoid cells and macrophages with nuclei in the form of a horseshoe. In some centers the aggregates of
unstructured light pink color masses with nuclei fragments were determined. For what disease the described changes are
characteristic?
@Tuberculosis
Lymphogranulomatosis (Hodgkin's disease)
Actinomycosis
Metastasis of tumor
Syphilis
#
21
A histological investigation of the enlarged cervical lymph nodes revealed the nodules consisted mainly of flat, slightly
extended cells with pale nucleus. There were also giant cells with spherical-oval form, light pink cytoplasm and with
nuclei located on a cell periphery. The centre of some nodules was presented with the unstructured, light pink colored
masses. The described changes are characteristic for:
@Tuberculosis
Lymphogranulomatosis (Hodgkin's disease)
Sarcoidosis (Boeck's disease)
Brucellosis
Nonspecific hyperplasic lymphadenitis
#
22
A 7-year-old boy with a history of tonsillitis presented with markedly enlarged paratracheal, bifurcate and cervical lymph
nodes. A microscopical investigation of the removed cervical lymph node revealed the centers of a necrosis surrounded
with lymphocytes, epithelioid cells and Pirogov-Langhans cells. Specify the most likely pathology.
@Tuberculosis
Sarcoidosis (Boeck's disease)
Rhinoscleroma.
Glanders
Syphilis
#
23
A lungs roentgenogram investigation revealed a dark patch. During a diagnostic express biopsy of a lymph node of a
bronchus was removed. A histological study showed a caseous tissue necrosis, round which lied epithelioid and lymphoid
layers with few multinuclear large cells (Langhans' cells). Specify the cause of the lymphadenitis.
@Tuberculosis
Pneumonia
Syphilis
Metastases of cancer
Adenoviral infection
#
24
Microscopical exam of the patient’s lungs (52 years) reveals focuses of necrosis that is surrounded by a roller of
epithelioid cells and lymphocytes. Giant cells of round shape and with great amount of nuclei on the periphery are located
between epithelioid cells and lymphocytes. How can we term such histological changes?
Syphilitic granuloma
Sarcoid granuloma
@Tuberculous granuloma
Leprous granuloma
Scleromous granuloma
#
25
Granulemes were detected during the appendix’s biopsy, which consist of the epitheliod, giant Pirogov-Langxance cells
and with necrosis in granulem’s center. These changes give the reason to make a diagnosis:
@Tuberculosis
Superficial appendicitis
Syphilis
AIDS
Actinomycosis
#
26
42-year-old patient complained of a voice timbre change. The larynx biopsy was taken. A histological investigation
revealed the aggregates of large, oblong form cells with light nuclei, rich with a cytoplasm. The nuclei were located on
periphery in the form of a horseshoe. The described histological changes are characteristic for:
@Tuberculosis
Leprosy
Syphilis
Rhinoscleroma
Lymphogranulomatosis (Hodgkin's disease)
#
27
An autopsy of a child, who died of meningocephalitis revealed under pleura and in pulmonal tissue plural, punctual,
yellow-white lesions which reminded millet grains. Histological investigation showed granulomas with a necrosis zone in
the centre and a wall of epithelioid cells, lymphocytes, macrophages and some giant Pirogov -Langhans cells. What is the
most likely disease?
@Primary milliary tuberculosis
Secondary tuberculosis
Focal pneumonia
Croupous pneumonia
#
28
At the section of a 44-year-old man, who died of pulmonary-heart insufficiency, were diagnosed pneumosclerosis,
emphysema of lungs, and hypertrophy of the right ventricle of the heart. Multiple focuses with the diameter of 1cm were
localized in both lungs, mostly subpleural. Histologically: in the center of the focus – the area of necrosis, on the
periphery – a roller of epithelioid cells, lymphocytes, few macrophages and plasmatic cells. Giant Pirogov-Langhans cells
were determined, too. There was also small amount of blood vessels on the periphery of the focus. What disease did the
dead man have?
@Haematogenous tuberculosis
Actinomycosis of lungs
Sarcoidosis
Syphilis
Silicosis
#
29
A 52-year-old man with a long history of a tubercular prostatitis died of a meningocephalitis. An autopsy revealed in
arachnoids of the basis and lateral surfaces of a brain, in a spleen, kidneys and liver a considerable quantity of dense grey
color nodules 0, 5-1mm in diameter. Histological investigation of nodules showed epithelioid, lymphoid and some giant
cells with nuclei located on a cell periphery in the form of a horseshoe. What is the most likely diagnosis?
@Acute milliary tuberculosis
Acute tuberculous sepsis
Chronic milliary tuberculosis
Disseminated tuberculosis
Septicopyemia
#
30
At the section of a 48-year-old man, who died of pulmonary tuberculosis, were revealed: partly devastated symmetric
tuberculous cavernae in corticopleural areas with the domination of productive tissue reaction, delicate reticulated
pneumosclerosis and emphysema in both lungs, pulmonary heart, tuberculous caseous lymphadenitis in bifurcation
nodules. Chon focus was located in the III segment of the right lung. What is the most possible diagnosis?
Haematogenous generalization of primary tuberculosis
@Haematogenous disseminated tuberculosis
Chronic primary tuberculosis
Chronic miliary tuberculosis
Cirrhotic tuberculosis
#
31
An autopsy of a 74- year-old man revealed the enlargement and deformation of the right knee joint. A histological
investigation determined the massive foci of a caseous necrosis, surrounded by peripheral mantle of epithelioid, lymphoid
cells and some multinucleated giant cells.
What is the most likely disease?
@Tuberculous arthritis
Syphilitic arthritis
Rheumatic arthritis
Gonorrheal arthritis
Deforming osteoarthrosis
#
32
Tuberculous spondylitis (Pott’s disease) is characterized by all the following EXCEPT
Involvement of thoracic and lumbar vertebrae
Hematogenous spread
@Proliferative synovitis with pannus
Destruction of intervertebral disks
Formation of psoas abscass
#
33
A 10-year-old girl arrived in traumatological unit with symptoms of pathological fracture of the right femur. A
histological investigation of an operational material revealed in a bone marrow of a femur fragment, a plural foci of a
caseous necrosis with peripheral mantle of epithelioid cells, lymphocytes and solitary Pirogov-Langhans' giant cells.
Zones of a caseous necrosis extended on adjoining sites of a bone tissue, resulting in osseous destruction. What is the
most likely disease?
@Tuberculous osteomyelitis
Tuberculous spondylitis
Tuberculous koxitis
General(common) macrofocal tuberculosis
Secondary acute focal tuberculosis
#
34
A post-mortem of a 72-year-old vagabond, with marked kypho-scoliosis, revealed the partially destructed thoracal
vertebral bodies due to formation of cavities (sequesters), filled with white-yellow brittle masses. A histological
investigation determined within altered vertebras big foci of a caseous necrosis, surrounded with solitary giant
multinucleated Pirogov-Langhans' cells, epithelioid cells and lymphocytes. Similar changes were diagnosed at
mediastinum lymph nodes and disseminated in lungs and kidneys. What is the most likely diagnosis?
@Tuberculous osteomyelitis
Purulent osteomyelitis
Syphilitic osteomyelitis
Osteoporosis
Osteoarthrosis
#
35
An ultrasound investigation of the young 17-year-old man's right kidney revealed pyeloectasis, accompanied with
disturbance of outflow of urine by urethra. Microscopical investigation of the renal biopsy showed a diffusive infiltration
of the interstitium by lymphocytes and hystiocytes. In the cortical and medullar zones there were the foci of a destruction
with formation of the cavities, surrounded by peripheral mantle of epithelioid , lymphoid cells and some multinucleated
Pirogov-Langhans' giant cells. What disease is the most probable?
@Tuberculosis of kidney
Suppurative (apostematous) nephritis
Nonspecific tubular-interstitial nephritis
Polycystic kidney
Chronic pyelonephritis
#
36
A man died of liver cirrhosis. An autopsy revealed in the 1-2 segment of the right lung three grey- yellow, dense foci, 1-
1,5cm in diameter with a dim surface. A histological research showed the tubercular endobronchitis, foci of a caseous
bronchopneumonia, surrounded with a wall of epithelioid, lymphoid and giant Pirogov-Langhans cells. For what form of
a pulmonary tuberculosis the described changes are characteristic?
@Acute focal tuberculosis
Infiltrative tuberculosis
Fibrous-focal tuberculosis
Caseous pneumonia
Primary pulmonary tuberculous complex
#
37
An autopsy of a 63-year-old man revealed in the second segment of the right lung a tubercular panbronchitis, with the
focus of a caseous bronchopneumonia. The focus was surrounded by a wall of epithelioid cells with some lymphocytes
and few giant Pirogov -Langhans cells. What form of a pulmonary tuberculosis presented in this case?
@Acute focal pulmonary tuberculosis
Primary tuberculous complex
Growth of primary affect
Acute miliary tuberculosis
Infiltration tuberculosis
#
38
At histologic examination of tuberculoma resected from right lung a necrosis in center is found out. Call a kind of the
necrosis.
@Caseous
Fatty
Liquefactive
Coagulative
Gangrene
#
39
An autopsy of a 48-year-old man revealed in the first segment of the right lung the round formation, 5 cm in diameter,
surrounded with a thin connective tissue layer. The centre was presented by write, brittle masses. Diagnose the form of
secondary tuberculosis.
@Tuberculoma.
Caseous pneumonia
Acute cavernous tuberculosis
Acute inflammatory tuberculosis
Fibrocavernous tuberculosis
#
40
A patient, who died of pulmonary tuberculosis, has a white-gray focus that is surrounded by a capsule, diameter 3cm in I
segment of the right lung. Microscopical exam reveals a focus of necrosis with a capsule and the absence of perifocal
inflammation. Name the form of tuberculosis.
Caseous pneumonia
Acute cavernous tuberculosis
Fibrous-cavernous tuberculosis
Cirrhotic tuberculosis
@Tuberculoma
#
41
At the section of a 48-year-old man, in the area of I segment of the right lung was revealed a round formation (diameter
near 5cm) with well-defined contour that was surrounded by thin stratum of connective tissue and filled with friable
masses. Diagnose the secondary form of tuberculosis.
@Tuberculoma
Caseous pneumonia
Acute cavernous tuberculosis
Acute focal tuberculosis
Fibrous-cavernous tuberculosis
#
42
An autopsy of a man with a history of tuberculosis revealed in the second segment of the right lung the white-grey focus,
3 cm in diameter, surrounded with a capsule. Microscopic study showed a focus of necrosis with a capsule and lack of a
perifocal inflammation. Name the tuberculosis form.
@Tuberculoma
Acute cavernous tuberculosis
Fibrocavernous tuberculosis.
Cirrhotic tuberculosis
Caseous pneumonia
#
43
An autopsy of a 42-year-old man revealed in the second segment of the right lung the focus of consolidation, 5 cm in
diameter, surrounded with a thin capsule. The centre was presented by a dense dry crumbling tissue with a dim surface.
Morphological changes in a lung are characteristic for:
@Tuberculoma
Carcinoma of lung
Chondromas
The tumorous formof a silicosis
Postinflammation pneumosclerosis
#
44
An autopsy of a 40-year-old man with a history of AIDS revealed both lung lobes enlarged, dense, air free, with fibrinous
membranes on pleura. A gross investigation of a cut surface showed the yellow dim pulmonal tissue. A histological
investigation determined in alveoli the serofibrinous and fibrinous exudate with large foci of a pulmonal tissue necrosis.
For what disease the described changes in lungs are characteristic?
@Caseous pneumonia
Infiltrative tuberculosis
Croupous pneumonia
Lymphogranulomatosis of lungs (Hodgkin's disease)
Carcinoma of lung
#
45
A 50-year-old man with a history of tuberculosis died of a chronic pulmonary and heart failure. A post-mortem revealed a
lobar lesion of the right lung. The upper lobe of the right lung was enlarged, dense, with fibrinous membranes on pleura.
A gross investigation of a cut surface showed the yellow brittle tissue. What is the most likely form of secondary
tuberculosis described in that case?
@Caseous pneumonia
Fibrous-focal tuberculosis
Infiltrative tuberculosis.
Tuberculoma
Acute focal tuberculosis
#
46
A 50-year-old man was ill with tuberculosis and died at the background of the facts of pulmonary-heart insufficiency. The
section detects the lobar character of pulmonary affection: the upper part of the right lung is enlarged, solid, yellow on the
incision, friable with fibrinous strata on pleura. What form of secondary tuberculosis does this pathology belong to?
Tuberculoma
Fibrous-focal tuberculosis
Infiltrative tuberculosis
@Caseous pneumonia
Acute focal tuberculosis
#
47
A 52- year-old man with a long history of tuberculosis died of a bleeding from lungs. An autopsy revealed in lungs some
oval and round cavities. The walls of cavities were presented with necrotic masses and a pulmonal tissue. What is the
most likely form of tuberculosis presented in that case?
@Acute cavernous
Fibrocavernous
Tuberculoma
Caseous pneumonia
Acute inflammatory
#
48
A 40- year-old man died of a pulmonary bleeding. An autopsy revealed in the upper lobe of the right lung a cavity, 4cm in
diameter, with dense walls and a rough internal surface, filled with blood clots. A microscopical study of the wall showed
the inner layer, presented with fell to pieces leucocytes; the middle layer formed by epithelioid, lymphoid and giant
multinuclear cells with nuclei located like a horseshoe. The changes found out in a lung are characteristic for:
@Tuberculous cavern (cavity)
Bronchiectatic cavern (cavity)
Abscess of lungs
Congenital cyst
Infarct of lung with septic disintegration
#
49
A 47- year-old man died of an acute anemia, resulted from the pulmonary bleeding. An autopsy revealed in the 2 segment
of the right lung the cavity, 5-6 cm in the size, which had connection with the bronchus lumen. The cavity had irregular
roundish form and filled with blood. The internal surface of a cavity was rough and covered by a flabby unstructured
yellowish tissue. A wall was thin, presented by the condensed, inflammatorily changed pulmonary tissue A histological
investigation showed that the cavity inside layer consisted of the fused caseous masses with a considerable quantity of
segmented leucocytes. What is the most likely pathology presented in that case?
@Acute cavernous tuberculosis
Abscess of lung
Disintegrating carcinoma of lung
Infarction of lung with septic disintegration
Bronchiectatic cavity (cavern)
#
50
A died woman had a chronic pulmonal abscess in her medical records. An autopsy revealed in 2 nd segment of the right
lung a roundish cavity, 5cm in the size. The internal surface of a cavity was created by caseous masses; the external one -
by a dense pulmonal tissue. A pyogenic membrane was absent. What is the most likely pathology?
@Acute cavernous tuberculosis
Fibrocavernous tuberculosis.
Acute abscess.
Chronic abscess.
Primary pulmonary cavity (cavern).
#
51
At the section of a woman, who died with a clinical diagnosis chronic pulmonary abscess, was detected a cavity of a
round form and 5cm in size in II segment of a right lung. The internal surface of the cavity was formed of caseous
masses, the external one – of solid pulmonary tissue. Pyogenic membrane was absent. What process is meant?
Acute abscess
Fibrous-cavernous tuberculosis
@Acute cavernous tuberculosis
Chronic abscess
Primary pulmonary cavern
#
52
A man died from the progressive pulmo-cardiac insufficiency at the age of 48. On the autopsy pulmonary emphysema,
net-like pneumosclerosis, nodules of white-gray colour, in the medium lung lobes symmetrically located cavities 3,5 cm
in diameter and hypertrophy of the right ventricle were detected. For which pulmonary
@Fibrous-cavernous tuberculosis
Hematogenically disseminated pulmonary tuberculosis
Chronic abscess of the lungs
Bronchiectatic disease
Acute pulmonary tuberculosis
#
53
A post-mortem of a 40-year-old patient, with a 10 years history of tuberculosis, revealed a cavity in the 1 and II segments
of his right lung. It had dense walls. The inner layer was rough, with beams formed by the obliterated bronchi and the
thrombosed vessels. The middle and lower lobes were firm, with the yellow foci on a cut surface. What form of
tuberculosis is the most likely in that case?
@Fibrocavernous
Acute cavernous
Fibrous inflammatory
Tuberculoma
Cirrhotic
#
54
A post-mortem of a 58-year-old patient revealed plural pathological cavities in both lungs. Microscopical study showed
the inner layer of one cavity presented with necrotic masses and diffuse neutrophil infiltration; the middle one contained
an infiltrate of epithelioid cells, lymphocytes and multinuclear giant cells. The external layer consisted of a mature
connective tissue. Diagnose the form of secondary tuberculosis.
@Fibrocavernous tuberculosis
Acute inflammatory
Fibro-inflammatory
Acute cavernous
Cirrhotic tuberculosis
#
55
At the autopsy of patient, which suffered by tuberculosis for a long time, the cavity by the sizes 3х2 cm connected with a
bronchus was found out in the upper lobe of right lung. The wall of the cavity dense has three layers: internal - pyogenic,
middle - layer of tubercular granulation tissue, external – layer of connective tissue. What the most probable diagnosis
which this formation relative to?
@Fibrous-cavernous tuberculosis
Fibrous-focal tuberculosis
Tuberculoma
Acute focal tuberculosis
Acute cavernous tuberculosis
#
56
A post-mortem of a 58-year-old patient, with a history of tuberculosis, revealed in the upper lobe of the right lung a
cavity, 3x2cm in sizes, which connected with a bronchus. A cavity had dense walls and three layers. The internal layer
was pyogenic; the middle one was presented with tubercular granulation tissue and the external coating of connective
tissue. What of the listed diagnoses the most likely?
@Fibrocavernous tuberculosis.
Fibrous focal tuberculosis
Tuberculoma.
Acute focal tuberculosis.
Acute cavernous tuberculosis
#
57
A 40-year-old prisoner died in the house of correction of tuberculosis. The autopsy exam revealed deformation and
reduction in size of the both pulmonary apices, numerous cavities with solid walls 2-3mm thick in the upper lobes of both
lungs, disseminated focuses of caseous necrosis with the diameter 5mm-2cm in the lower lobes. Diagnose the form of
tuberculosis.
Secondary cirrhotic
Secondary fibrous-focal
Haematogenous focal with the affection of the lungs
Primary, primary affect growth
@Secondary fibrous-cavernous
#
58
A 40-year-old prisoner died in penitentiary of tuberculosis. An autopsy revealed a deformation and reduction of the both
lungs apexes, plural cavities with dense, 2-3 mm thick walls in the upper lobes of both lungs. In addition, disseminated
foci of a caseous necrosis, 5mm-2 cm in diameter, were found in the lower lobes of lungs. Diagnose the tuberculosis
form.
@Secondary fibrocavernous
Secondary fibro-inflammatory
Gematogenic inflammatory with lesion of lungs
Primary, growth of primary affect
Secondary cirrhotic
#
59
A 56-year-old man died of a progressing heart failure. A post-mortem revealed small, dense lungs; the upper lobes were
deformed, penetrated by a cicatrical tissue, with the encapsulated foci of a caseous necrosis, 0, 2-0,5cm in diameter. In the
upper and middle lobes there were cavities, 3-4 and 4- 5cm in the size, with dense walls and fiber internal surface, filled
with the opaque yellow-green liquid. The surrounding pulmonary tissue was sclerotic. The heart was enlarged at the
expense of the hypertrophied right ventricle. Define the form of secondary pulmonary tuberculosis
@Cirrhotic tuberculosis
Fibrous-focal tuberculosis.
Infiltrative tuberculosis
Acute cavernous tuberculosis
Fibrocavernous tuberculosis
#
60
A 68-year-old woman, with a 20 years history of fibrous - cavernous pulmonary tuberculosis, presented to the
Department of Nephrology with symptoms of uremia. Intravital test on amyloid in kidneys appeared positive. What is the
most likely form of an amyloidosis presented in this case?
@Secondary systemic
Primary systemic
Limited (local)
Familial congenital
Senile (gerontal)
#
61
A patient with a history of fibrous - cavernous tuberculosis died of a renal failure. A post-mortem revealed a urine smell,
left ventricle hypertrophy, fibrinous pericarditis, and fibrinous – hemorrhagic enterocolitis. Kidneys were small, reduced
in sizes, dense, with plural foci of shrinkage. Histological investigation of the slides with Congo-Red staining showed
pink masses within glomuleruli and vessels walls. In addition, destruction and an atrophy of the majority of nephrons,
accompanied with a nephrosclerosis were determined. What is the most likely pathology of kidneys?
@Amyloid- contracted kidneys
Contracted kidney
Arteriolosclerotic kidney
Atherosclerotic contracted kidneys
Pielonephritic contracted kidneys
#
62
56-year-old man with a long history of fibrous - cavernous pulmonary tuberculosis died of a chronic pulmonary and heart
failure. Within last months the expressed proteinuria was observed. A post-mortem revealed enlarged, firm kidneys, with
a waxy surface. What changes developed in kidneys at this form of tuberculosis?
@Amyloidosis of kidney
Tuberculosis of kidney
Glomerulonephritis
Nephrolithiasis
Necrotic nephrosis
#
63
A 10-year-old boy presented with Hutchinson's triad: parenchymatous keratitis, sensorineural deafness and deformed
Hutchinson's teeth (the barrel form), along with saddle-nose deformity. For what illness the presented changes are
characteristic?
@Syphilis.
Toxoplasmosis
Leprosy.
Tuberculosis.
Opisthorchiasis.
#
64
A 14-year old patient was diagnosed a Hutchinson’s triad: screwdriver teeth, parenchymatous keratitis, and deafness.
What disease do detected changes point on?
@Syphilis
Toxoplasmosis
Lepra
Tuberculosis
Opisthorchosis
#
65
A histological study of a biopsy from an oral cavity ulcer revealed the foci of caseous necrosis surrounded with
plasmocytes, epithelioid , lymphoid cells and solitary giant multinucleated Pirogov-Langhans' cells. There were also signs
of endo - and a perivasculitis in fine vessels. Specify the disease.
@Syphilis
Tuberculosis
Leprosy
Rhinoscleroma
Glanders

#
66
A man's oral cavity investigation revealed an oval, cartilaginous density, slightly elevated ulcer at his buccal mucosa. A
meaty, reddened floor was covered with grayish sebaceous deposits. A microscopical investigation showed lymphocyte
infiltration, situated mainly round fine vessels. There was also an endothelium proliferation within the vessel's walls.
What disease has developed in that case?
@Syphilis
Traumatic ulcer
Erosive-ulcerated leukoplakia
Ulcer-Carcinoma
Ulcero-necrotic Vensan's stomatitis
#
67
An ulcer of an oval form with little raised edges and cartilaginous infiltrate is presented on the mucous membrane of the
patient’s cheek. The bottom of the ulcer is of meaty-red color, covered with grayish greasy scurf. Microscopic exam
reveals lymphoplasmocytic infiltrate, mostly around small vessels with the proliferation of endothelium. What disease do
these signs indicate on?
Ulcerative-necrotic Vincent’s stomatitis
Traumatic ulcer
Erosive-ulcerative leukoplakia
Ulcer-cancer
@Syphilis
#
68
A painless ulcer with smooth varnished bottom and margins of cartilaginoid consistence is observed on the mucous
membrane of the right palatine tonsil. Microscopically an inflammatory infiltrate, which consists of lymphocytes,
plasmocytes, small amount of neutrophiles and epithelioid cells, and the presence of endo- and perivasculitis are revealed.
What disease is meant?
Faucial diphtheria
Actinomycosis
Tuberculosis
@Syphilis
Ulcerative-necrotic Vensan’s quinsy
#
69
The prepuce was removed from a young man. A microscopical investigation revealed the polymorphic infiltrate
consisting from randomly located plasmatic, lymphoid and epithelioid cells. There was marked vessels' vasculitis. For
what disease the described changes are characteristic?
@Syphilis
Tuberculosis
Actinomycosis
Periarteritis nodosa
Leprosy
#
70
An 18-year-old patient has enlarged inguinal lymph nodes, painless, solid when palpated. A small ulcer with solid
margins and varnished bottom of grayish color exists in the area of genital mucosa. Make a diagnosis.
Lepra
Tuberculosis
@Syphilis
Trophic ulcer
Gonorrhea
#
71
A 20-year old woman presented with enlarged, not painful, condensed inguinal lymph nodes. In her genital mucosa there
was a small ulcer with firm edges and the "varnished" grayish color floor. What is the most likely diagnosis?
@Syphilis
Tuberculosis.
Leprosy.
Trophic ulcer.
Gonorrhea.
#
72
A biopsy of cervix uteri was taken from the 34-year-old women. Histological investigation revealed the inflammatory
infiltration with involving of fine arteries and veins walls. The infiltrate composed of plasmocytes, lymphocytes, and
epithelioid cells. There were also zones of sclerosis and a tissue hyalinosis. What is the most likely disease?
@Syphilis
Tuberculosis
Leukoplakia
Cervical erosion
Condyloma
#
73
An autopsy of a 36-year-old man revealed in his liver s roundish formation, 0,5 cm in diameter. Microscopical
investigation showed in the centre of lesion necrotic masses, surrounded by the granulation tissue with some plasmatic
and lymphoid cells. There were also signs of the vasculitis in the blood vessels. What is the most likely diagnosis fn that
case?
@Solitary gumma of liver
Solitary adenoma of liver.
Solitary leproma of liver.
Chronic abscess of liver.
Carcinoma of liver.
#
74
At the microscopic examination of a biopsy material, taken from the liver, granulomas were revealed. They consisted of
plasmatic, lymphoid, giant multinuclear Pirogov-Langhans’ cells, small vessels with the sings of endo- and perivasculitis.
Focuses of caseous necrosis were also detected. What disease are these granulomas typical for?
@Syphilis
Tuberculosis
Lepra
Rhinoscleroma
Glanders (malleus)
#
75
At the microscopic exam of the biopsy material, taken from the oral cavity, were detected focuses of caseous necrosis,
surrounded by plasmocytes, epithelioid and lymphoid cells, and occasionally by giant multinuclear Pirogov-Langhans’
cells. Small vessels with the signs of endo- and perivasculitis could be also seen. Term the disease.
Glanders (malleus)
Tuberculosis
Lepra
Rhinoscleroma
@Syphilis
#
76
An autopsy of a 54-year-old man reveled above valvular rupture of an aorta with resulted in a cardiac tamponade.
Histological investigation of ascending part of an aorta showed in its external and middle layers inflammatory infiltrates.
They consisted of lymphoid, plasmatic and epithelioid cells. There were also recognized the necrosis centers and
proliferation of a vessels' wall. Changes in an aorta are characteristic for:
@Syphilitic aortitis
Septic aortitis
Rheumatic aortitis
Atherosclerosis
Hypertension diseases
#
77
A 38-year-old man died trying to elevate the weight. Collapse had developed. The autopsy revealed the rupture of the
extensive thoracic aorta aneurysm. During his life the patient had visceral syphilis. What pathological process caused the
atrophy of the aortic wall, its dilation and rupture?
Atrophy of the muscular stratum
Dissolution of collagen fibres
@Dissolution of elastic fibres
Changes in intima after the type of shagreen skin
Neoplasm of vessels
#
78
Tabes dorsalis is characterized by
Hydrophobia
Increased neutrophils in the cerebrospinal fluid (CSF)
Involvement of the motor neurons of the spinal cord
@Degeneration of posterior columns of the spinal cord
Infection of oligodendrocytes
#
79
A 69-year-old man in an underdeveloped country develops changes in his mental status along with ataxia, deformed
knees and ankles, and an abnormal gait during which he slaps his feet as he walks. Physical examination reveals
decreased vibration and proprioception in lower extremities along with absent pupillary light reflexes with normal
accommodation. Which of the following is the most likely cause of these signs?
Cysticercosis
@Neurosyphilis
Poliomyelitis
Rabies
Progresive multifocal leukoencephalopathy
#
Viral infection disease
#
1
A post-mortem investigation of a 42-year-old man revealed serous-hemorrhagic inflammation in a larynx and trachea
mucosa, a focal pneumonia, areas of atelectases and acute lungs emphysema. Histological research showed a vacuolar
dystrophy and loss of cilia within the epithelium layer, an exfoliating of cells with oxyphile inclusions. Such changes in
lungs are caused:
@Influenza viruses
Parainfluenza viruses
Measles virus
Adenoviruses
RS-viruses
#
2
A 30-year-old man was ill with acute respiratory disease and died at the background of the facts of acute pulmonary-heart
insufficiency. At the section were revealed fibrinous-haemorrhagic inflammation on the mucous membrane of larynx and
trachea, destructive panbronchitis. Lungs were enlarged in size and mottled because of abscesses, haemorrhages and
necrosis. What is the most possible diagnosis?
Measles
Paragrippe
Respiratory-sinticial infection
@Grippe
Adenoviral infection
#
3
A 38-year-old man presented with an acute fever, elevated temperature to 40°C, a headache, a cough and dyspnea. He
died on the 5th day of a disease. An autopsy revealed enlarged lungs with a motley pattern of a pulmonary tissue - "the big
motley lung". For what disease the described signs are characteristic?
@ Influenza
Adenoviral infection
Croupous pneumonia
Respiratory syncytial infection
Bronchiectasis
#
4
A 42-year-old man, with a history of an acute respiratory disease, died of pulmonary insufficiency. A post-mortem
investigation revealed fibrinous-hemorrhagic inflammation in a larynx and trachea mucosa, destructive panbronchitis and
enlarged lungs. Multiple abscesses, hemorrhages and necroses created a motley appearance of a pulmonal tissue. What of
the listed diagnoses the most likely?
@Influenza
Parainfluenza
Respiratory syncytial infection
Measles
Adenoviral infection
#
5
A 42-year-old man died at the background of the facts of severe intoxication and pulmonary insufficiency.
Macroscopically on the incision pulmonary tissue of all lobes is mottled with multiple small-focal haemorrhages and with
focuses of emphysema. Histological exam reveals haemorrhagic bronchopneumonia with abscess-formation, eosinophilic
and basophilic inclusions in the cytoplasm of the bronchial epithelium. Diagnose the disease that was revealed at the
section.
Adenoviral infection
Paragrippe
@Grippe
Respiratory-synticial infection
Staphylococcal bronchopneumonia
#
6
A 44-year-old man died of pulmonary insufficiency and severe intoxication. A post-mortem investigation of a lungs cut
surface revealed a motley pattern with plural focal hemorrhages and the zones of emphysema. Histological study showed
a hemorrhagic bronchopneumonia with abscesses formation. A cytoplasm of bronchial epithelium cells had eosinophilic
and basophilic inclusions. What is the most likely diagnosis?
@Influenza
Parainfluenza
Adenoviral infection
Respiratory syncytial infection
Staphylococcal bronchopneumonia
#
7
A 63 y.o. man fell ill with acute tracheitis and bronchitis accompanied by bronchial pneumoni a. On the 10th day the
patient died from cardiopulmonary insufficiency. Autopsy revealed fibrinous hemorrhagic laryngotracheobronchitis;
lungs were enlarged, their incision revealed the "coal-miner's" effect caused by interlacing of sections of bronchial
pneumonia, hemorrhages into the pulmonary parenchyma, acute abscesses and atelectases. Internal organs have
discirculatory and dystrophic changes. What is the most probable diagnosis?
Respiratory syncytial infection
Parainfluenza
Moderately severe influenza
Adenoviral infection
@Influenza, severe form
#
8
A 66-year-old man presented with acute tracheitis and bronchitis, soon resulted in bronchopneumonia. He died on the 12 th
day at the hospital from the pulmonal-heart failure. An autopsy revealed fibrinous -hemorrhagic inflammation in a larynx
and trachea mucosa, enlarged lungs. On a cut section lungs had a motley pattern due to areas of pneumonia, hemorrhages
in lung parenchyma, acute abscesses and atelectases. In addition, circulative disturbances and dystrophies were
determined in internal organs. What is the most likely diagnosis?
@Influenza, severe form
Influenza, moderately severe form
Parainfluenza
Respiratory syncytial infection
Adenoviral infection
#
9
A 48-year-old woman died of pulmonary insufficiency. A post-mortem revealed the enlarged lung with the dark red and
pinkish-yellow colour areas ("the big motley lung") and necrotic tracheobronchitis. What is the preliminary diagnosis in
that case?
@Influenzal pneumonia
Croupous pneumonia
Caseous pneumonia
Fibrosing alveolitis
Measles virus pneumonia
#
10
A 67-year-old man died of pulmonary insufficiency. A post-mortem investigation of a lungs cut surface revealed
multiple, bright, red, deaerated 2,5 cm foci with irregular form and inaccurate borders. A microscopic study showed
erythrocytes and some lymphocytes within alveoli. The described changes are characteristic for:
@Influenzal pneumonia
Croupous pneumonia
Hemorrhagic infarction of the lung
Measles virus pneumonia
Aspiration pneumonia
#
11
A 67-year-old patient had a severe form of grippe and died of it. At the section the lungs were big and diverse.
Microscopical exam revealed acute vessel plethora, haemorrhages, edema of pulmonary tissue, and exudate in the lumen
of bronchi and alveoli that contained mostly erythrocytes. What type of inflammation do these morphological changes
indicate on?
@Hemorrhagic bronchopneumonia
Catarrhal bronchopneumonia
Suppurative bronchopneumonia
Desquamative bronchopneumonia
Fibrinous pneumonia
#
12
In course of severe respiratory viral infection there appeared clinical signs of progressing cardiac insufficiency that
caused death of a patient in the 2nd week of disease. Autopsy revealed that the heart was sluggish, with significant cavity
dilatation. Histological examination of myocardium revealed plethora of microvessels and diffuse infiltration of stroma
by lymphocytes and histiocytes. What disease corresponds with the described picture?
@Myocarditis
Acute coronary insufficiency
Myocardium infarction
Stenocardia
Cardiomyopathy
#
13
An infection in 7-year old child started with mild cold-like symptoms, “seal bark” cough and then it results in subglottal
swelling. Which of the following organisms is most likely can cause laryngotracheobronchitis (croup)?
@Parainfluenza virus type 1-3
Mumps
Measles
Respiratory syncytial virus
14
RNA which is contained in AID virus, penetrated into the middle of leukocytes and, made the cell synthetic viral DNA
with the help of the enzyme revertase. This process is based on…
@Reverse transcription
Convariant replication
Reverse translation
Depression of operone
Repression of operone
#
15
It is known that the human immunodeficiency virus belongs to the family of retroviruses. Specify the basic attribute,
which characterizes this family.
@The presence of the enzyme - reverse transcriptase
Contain RNA
Simple viruses, which affect only men
Nucleic acid doesn.t integrate into the genome of the
Enzyme linked immunosorbent assay is used for the revealing of antigens
#
16
It was revealed that T-lymphocytes were affected by HIV. Virus enzyme - reverse transcriptase (RNA-dependent DNA
polymerase) - catalyzes the synthesis of:
Virus informational RNA on the matrix of DNA
DNA on virus ribosomal RNA
Viral DNA on DNA matrix
@DNA on the matrix of virus mRNA
mRNA on the matrix of virus protein
#
17
Examination of a young man in the AIDS centre produced a positive result of immune-enzyme assay with HIV antigens.
Patient's complaints about state of his health were absent. What can the positive result of immune-enzyme assay be
evidence of?
HBV persistence
Being infected with HBV
Being ill with AIDS
@HIV infection
Having had AIDS recently
#
18
The diagnosis of AIDS was put to the 20 years old patient. What populations of cells are most sensitive to a human
immunodeficiency virus?
@T-helpers
Hepatocytes
Endotheliocytes
Epithelial cells
B-lymphocytes
#
19
Blood analysis of a patient showed signs of HIV infection (human immunodeficiency virus). Which cells does HIV-virus
primarily?
Specialized nervous cells (neurons)
Mast cells
Cells that contain receptor IgM (B-lymphocytes)
Proliferating cells (stem hematoplastic cells)
@Cells that contain receptor T4 (T-helpers)
#
20
The human immunodeficiency virus has the follow antigens - gp 41 and gp 120 on the surface. This virus interacts with
the cells-target of the organism. Choose among the listed antigens of the human leukocyte with which gp 120 of the virus
complementary connected.
@CD 4
CD 3
CD 8
CD 19
CD 28
#
21
A post-mortem of a woman revealed a septicopyemia with metastatic abscesses of lungs and kidneys; pyosalpinx, a
purulent peritonitis, a cachexia and lymphadenopathy. Medical records indicated HIV- positive blood test results within
last 5 years. What clinical period of AIDS corresponds with the given pathomorphologic signs?
@Acquired immunodeficiency syndrome (AIDS)
Incubation period
Persisten generalised lymphadenopathy
Pre-AIDS (AIDS -related complex)
Reactive lymphadenopathy
#
22
The patient is 25 years old. From the numerous cutaneous pustules St.aureus in the association with St.epidermidis were
isolated. Pneumocystes carinii was detected in the sputum, Cryptosporidia, Proteus vulgaris and Candidas - in the feces.
In which disease you can see so plural infection by the opportunistic
@AIDS
Diabetes
Sepsis
Dysbacteriosis
Medicinal agranulocytosis
#
23
A 60-yer-old mail patient received several blood transfusions during the surgical operation. He did well for 2 weeks, but
then developed fever, vomiting, diarrhea. Liver biopsy revealed intranuclear inclusion bodies. Which organism is the
cause of the disease?
@Cytomegalovirus
Adenovirus
Hepatitis A virus
Influenza virus
Parainfluenza virus
#
24
At the pathomorphologic exam of a 30-year-old drug-user with HIV infection was traced that both lungs were
consolidated, of dark vinous-gray color, low-aerial; interalveolar septa were densely infiltrated with leukocytes; part of
alveolocytes were transformed into big cells. The nucleus of such cell is located in the centre and is round with light ring
that looks like an owl’s eye. What opportunistic infection caused pneumonia?
Atypical mycobacterium
Pneumocystis carinii
@Cytomegalovirus (CMV)
Herpes virus
Toxoplasma
#
25
A 35-year-old narcomaniac (drug abuser) with a history of HIV-infection died at the hospital. A post-mortem revealed
that both lungs were dense, dark-red-grey color, with little air. Histological study showed that interalveolar septs were
infiltrated by lymphocytes. Some alveolocytes were transformed to the big cells, with a centrally located round nucleus
with a light rim of cytoplasm ("an owl's eye"). What opportunistic infection caused pneumonia in that case?
@Cytomegalovirus
Pneumocystis carinii
Atypical mycobacterium
Herpes-virus
Toxoplasma
#
26
A post-mortem of a 40-year-old patient revealed a pneumocystosis carinii pneumonia, Kaposi's sarcoma and B-cell
lymphoma. Medical records suggested that he actively practiced unsafe sexual intercourse with multiple partners at his
life. What is the most likely diagnosis?
@A human immunodeficiency virus infection, AIDS stage
A human immunodeficiency virus infection, pre- AIDS stage
A secondary immunodeficiency as a result of primary B-cell lymphoma
A secondary immunodeficiency as a result of Kaposi's sarcoma
-
#
27
On the autopsy of the dead some crimson-red strains, plaques and nodules (sarcoma Kaposhi) were found on the skin of
the distal parts of the lower extremities. Also acute pneumonia was discovered. What is the disease?
@AIDS
Influenza
Chicken pox
Diphtheria
Anthrax
#
28
HIV-infected patient is examined periodically for the purpose of the exposure of the signs of process activation. Name the
most essential sign, which specifies the transformation of HIV-infection to the AIDS.
@Kaposi’s sarcoma. The amount of the helper-inducer T cells is less then 200 kl/mkl
Decreasing of amount of neutrophiles
Decreasing of amount of helper-inducer T cells
The amount of the helper-inducer T cells is less then critical level
The revealing of the gp 41
#
29
A patient presented to his physician with expressed immunodeficiency, lymphopenia with change of parity T-helpers to
T- suppressors, the lesion of a skin of the lower extremities. The cutaneous pathology was characterized by plural
tumorous nodules, dome-shaped purple plaques, involved in skin breakdown with resulting fungating lesions. A
histological investigation of the skin's biopsy showed the neoplasm of blood vessels, a dilatation of capillaries, slit-like
blood-filled vascular spaces. What is the most likely diagnosis?
@Kaposi's sarcoma
Basalioma
Dermatomycosis
Phoma of skin
Inflammatory dermatitis
#
30
A post-mortem of a patient, with a history of a drug abuse, revealed red-purple papular nodules and blotches on the skin
of his lower extremities (Kaposi's sarcoma). In addition, acute pneumocystosis carinii pneumonia was also determined.
For what disease the given symptoms are characteristic?
@AIDS
Influenza
Measles
Diphtheria
Anthrax
#
31
Due to the results of the section of a 44-year-old man, who was a citizen of Ukraine, was traced the combination of
pneumocystis pneumonia, Kaposi’s sarcoma and B-cell lymphoma. In the anamnesis is irregular sexual activity. What is
the most possible diagnosis?
HIV infection, pre-AIDS stage
@HIV infection, AIDS stage
Secondary immunodeficiency caused by primary B-cell lymphoma
Secondary immunodeficiency caused by Kaposi’s sarcoma
Superinfection at the background of immunodeficiency
#
32
A 23-year-old HIV-positive man presents with a cough and increased shortness of breath. A histologic section from a
transbronchial biopsy staied with Gomori's methenamine-silver stain is shown in the photomicrograph below. Which of
the following is the most likely diagnosis?
Pseudomonas pneumonia
Aspergillus pneumonia
@Pneumocystis carinii pneumonia
Cytomegalovirus pneumonia
Influenza pneumonia
#
33
A 25-year-old woman presents with fever, malaise, headaches, and muscle pain (myalgia). A chest x-ray reveals bilateral
infiltrates. You draw a tude of blood from the patient (the tude contains anticoagulant) and place the tude in a cup of ice.
After the blood has cooled, you notice that the red cells have agglutinated (not clotted). This agglutination goes away after
you wrm up the tude of blood. This patient's illness is most likely due to infection with which one of the following
organism?
Influenza A virus
@Mycoplasma pneumoniae
Streptococcus pneumoniae
Pneumocystis pneumoniae
Mycobacterium tuberculosis
#
34
A post-mortem of a patient, who died of a HIV- infection, revealed alterations of his brain. A histological investigation
determined in subcortical white matter, midbrain and a brain stem fine, perivascular necroses, microglial nodules with
multinuclear giant cells, focal gliosis and fibrosis. Name the most likely lesion of the central nervous system?
@Subacute meningocephalitis
Metabolic encephalopathy
Cytomegaloviral encephalitis
Primary lymphoma of CNS
Vacuolar myelipathy
#
35
Characteristics of AIDS-related neurologic abnormalities include all the following EXCEPT
Areas of demyelination
@Preferential infection of cortical neurons
Multinucleated giant cells
Vacuolar myelopathy
Increased incidence of progressive multifocal leukoencephalopathy (PML)
#
36
Disease that are classified as slow viral infections or unconventional agent (spongiform) encephalopathies include all the
following EXCEPT
@Reye’s syndrome
Subacute sclerosing panencephalitis
Creutzfeldt-Jakob disease
Progressive multifocal leukoencephalopathy
Kuru
#
37
A 3-year-old child has a temperature of 38,30C; discrete vesiculoulcerative lesions (Koplik’s sports) can be seen on the
mucous membrane of the mouth. The most probable diagnosis is:
@Measles
Rubella
Herpangina
Mumps
Scarlet fever
#
38
A child presented with catarrhal conjunctivitis, pharyngitis, a laryngotracheobronchitis. A gross investigation of the oral
cavity revealed white spot on the buccal mucosa opposite the first molar. In addition, a blotchy and papular rash appeared
on the patient's face, trunk and extremities later. What is the most likely diagnosis?
@Measles
Scarlet fever
Meningococcal infection
Epidemic typhus
Influenza
#
39
A child with a history of an acute catarrh of the upper respiratory tract, conjunctivas and blotchy, papular skin rash, died
of pulmonary insufficiency. A histological investigation revealed an interstitial pneumonia with focal serous
macrophageal inflammation and set of giant cells within alveoli walls. What is the most likely diagnosis?
@Measles
Influenza
Parainfluenza
Adenoviral infection
Cytomegaly
#
40
An 8-year-old child presented with raised body temperature up to 39° C, rhinitis, conjunctivitis, and cough. A physical
investigation revealed blotchy skin rash and whitish rashes on a buccal mucosa. Suddenly appeared pant resulted in
asphyxia, followed by patient death. What is the most likely diagnosis?
@Measles
Scarlet fever
Diphtheria
Meningococcal nasopharyngitis
Influenza
#
41
A 5-year-old child presented with raised body temperature, punctulated rash, and conjunctivitis. He died of the
superimposed pneumonia. A histological research of lungs revealed endo - mezo - panbronchitis and giant-cell
pneumonia. For what disease the presented changes are characteristic?
@Measles
Scarlet fever
Croupous pneumonia
Diphtheria
Smallpox
#
42
A 7-year-old child presented with raised body temperature up to 38,5°, rhinitis, conjunctivitis, and cough. A physical
investigation revealed blotchy and papular skin rash and whitish bran-like rashes in a buccal mucosa. On the fourth day
appeared a dyspnea and wet pulmonary rhonchuses. Soon, difficulty of breath resulted in patient death from asphyxia.
What is the most likely diagnosis?
@Measles
Meningococcemia
Influenza
The pulmonary form of a mucoviscidosis
Pneumonia
#
43
A child died of a spontaneous pheumothorax. A post-mortem revealed a collapse and interstitial emphysema of the right
lung, focal atelectases of lungs, a punctual hemorrhage in a brain and a conjunctiva of eyes. What is the most likely
diagnosis?
@ Whooping cough
Bronchopneumonia
Measles
Diphtheria of respiratory tracts
Scarlet fever
#
44
A 4-year-old child died of respiratory insufficiency as a result of a respiratory musculation paralysis. Histological
investigation revealed a hyperemia of the thoracal part of a spinal cord, the unclear pattern of the grey substance, punctual
hemorrhages and fine sinking down sites of an encephalomalacia. In addition, there was an inflammation with a neuroglia
proliferation around and by the necrotic neurons. What is the most likely diagnosis?
@Poliomyelitis
Meningococcal infection
Cytomegaly
Toxoplasmosis
Adenoviral infection
#
45
A 6-year-old girl presented to the hospital with suddenly raised temperature and catarrh of respiratory tracts. For 5th day
of illness there was a paralysis of her lower extremities muscles, accompanied with respiratory disorders. In the anterior
horns of a spinal cord there was a proliferation of a glia round necrotic neurons. What is the most likely diagnosis?
@Poliomyelitis
Measles
Diphtheria
Meningococcemia
Scarlet fever
#
46
A 6-year-old boy developed pain at swallowing and raised body temperature. On a second day of the disease, a physical
investigation revealed the appearance of the fine, densely set, widespread skin rash with a small unaffected circumoral
area. An oral cavity examination showed the bright reddening of a pharynx, enlarged tonsils and raspberry-red tongue.
What is the most likely diagnose?
@Scarlet fever
Adenoviral infection
Fauces diphtheria
Measles
Streptococcal angina
#
47
A sore throat and temperature elevation manifested in a 7-year-old child. On the second day of the disease red eruption,
like small densely located stains of poppy seed size, was noticed. They covered the whole body except nasolabial
trigonum. The exam of the oral cavity revealed flaming hyperemia of the fauces, enlarged tonsils, and crimson-red
tongue. What is your diagnosis?
@Scarlet fever
Adenoviral infection
Diphtheria of the fauces
Measles
Streptococcal angina
#
48
A boy is 7 y.o. Objectively: against the background of hyperemic skin there is knobby bright-pink rash on his forehead,
neck, at the bottom of abdomen, in the popliteal spaces; nasolabial triangle is pale. Examination of oropharyngeal surface
revealed localized bright-red hyperemia; tonsils are swollen, soft, lacunas contain pus, tongue is crimson. Cervical lymph
nodes are enlarged, dense and painful. What is the most probable diagnosis?
@Scarlet fever
Infectious mononucleosis
Rubella
Diphtheria
Whooping cough
#
49
A 7-year-old boy presented to the hospital with an acute catarrh in pharynx and tonsils which had extended on a mouth
mucosa, tongue and fauces. A tongue was fiery red (crimson red) with prominent papillae ('raspberry' or 'strawberry'
tongue). A gross investigation of tonsils showed superficial necroses. A casting-off necrotic tissue resulted in ulcers
formation. Cervical lymphatic nodules were enlarged. A punctuate erythematous rash was determined over the trunk,
arms and legs. Face was also involved but a small area about the mouth remained unaffected to produce circumoral pallor.
What is the most likely diagnosis?
@Scarlet fever
Diphtheria
Angina
Measles
Meningococcal nasopharyngitis
#
50
A 7-year-old child presented to the hospital with a punctuated bright-pink rash on hyperemic skin of a forehead, a neck, in
the bottom of abdomen and popliteal fossa. A nasolabial triangle remained pale. Physical investigation of a stomato-
pharynx revealed circumscribed bright red hyperemia of mucosa, 'raspberry' tongue. His tonsils were friable and
edematous; lacunas filled with pus. Lymph nodes of a neck were enlarged, dense and painful. What is the most likely
diagnosis?
@Scarlet fever
Rubella
Whooping cough
Diphtheria
Infectious mononucleosis
#
51
A 7-year-old girl developed an acute disease. She presented to the hospital with a sore throat, and a fever, accompanied
with a widespread rash. A physical examination revealed severe hyperemia of a pharynx, a 'raspberry' tongue, and
enlarged bright red tonsils with some grayish and yellowish dim areas. These zones also extended on peritonsillar tissues.
Submandibular lymph nodes were enlarged. For what disease the given changes are characteristic?
@Scarlet fever
Measles
Fauces diphtheria
Larynx diphtheria
Meningococcal nasopharyngitis
#
52
A 9-year-old child developed a sharp pain at swallowing and a neck edema. His body temperature had risen to 39°C. A
motley-red, punctulated , widespread rash appeared. A physical examination revealed severe hyperemia of a pharynx
('fiery pharynx') and a 'raspberry' tongue. The enlarged bright red tonsils presented with some grayish necrotic zones.
What is the most likely diagnosis?
@Scarlet fever
Meningococcal nasopharyngitis
Diphtheria
Influenza
Measles
#
53
A child complains of the general weakness, a pharyngalgia at swallowing and nausea. Objectively, a bright reddening
revealed in fauces; tonsils were enlarged. Later, the fine, densely located maculae, size about papaverous grain, appeared
on a neck, in the top of a back and a breast. Microscopical investigation showed in a mucosa and tonsils tissue an acute
plethora and grayish, dim areas of necroses. On the periphery of the necrotic zones, within the edema and fibrinous
inflammation areas, the chains of streptococcuses were defined. What is the most likely diagnosis?
@Scarlet fever
Diphtheria
Measles
Influenza
Meningococcal nasopharyngitis
#
54
In the autopsy of a 6-year-old child the pathologist has found out: the necrotic- purulent angina, phlegmon of a neck,
purulent otitis, purulent meningitis. These changes are most typical for:
@Septic scarlatina
Meningococcal infection
Toxic scarlatina
Diphtheria
Otogenic sepsis
#
55
An autopsy of a child revealed a necrotic tonsillitis, neck phlegmon, purulent otitis and purulent meningitis. These
alterations are the most typical for:
@Septic scarlet fever
Meningococcal infection
Toxic scarlet fever
Fauces diphtherias
Otogenic sepsis
#
56
The 6-years-old boy has a moderate increase of the temperature, his parotid glands are enlarged. The virus has been
allocated from the patient.s saliva. This virus is reproduced at the chicken embryos and tissue.s cultures, it has
hemagglutination properties and causes the formation of the symplast in the cells culture. Which organs may be affecting
by the disease caused by this virus most probably?
@Genital glands
Liver
Lungs
Throat tonsils
Brain
#
57
The laboratory of the diagnostic center obtained the modern test-system for the revealing of the TORCH-infection, which
gives the possibility to diagnose: 1) toxoplasmosis, 2) rubella infection, 3) cytomegalovirus infection, 4) herpes-infection.
Which from the named diseases do viruses cause?
@Rubella, cytomegalovirus infection, herpes-infection
Rubella, herpes-infection
Cytomegalovirus infection, herpes-infection
Rubella
All named
#
58
A 23-year-old woman presents to her gynecologist for a routine physical examination that includes a Pap smear. Her
sexual history includes many sexual partners beginning at an early age, but she has never been pregnant. Physical
examination is unremarkable. The Pap smear returns as abnormal with the presence of atypical squamous epithelial cells
of undetermined significance (ASCUS). She returns for a 6-month follow-up and a repeal pelvic exam is performe. Her
cervix is painted with iodine and an area near the cervical os is present that does not stain with iodine. This area is flat and
not papillary. Several biopsies are obtained from this pale area, and a representative histologic section is seen in. the
picture below. This histologic section shows koilocytosis, which is most characteristic of infection with which one of the
following organisms?
Cytomegalovirus
Epstein-Barr virus
Herpes simplex virus
@Human papillomavirus
Parvovirus BlO
#
59
A pregnant woman was registered in an antenatal clinic and underwent complex examination for a number of infections.
Blood serum contained IgM to the rubella virus. What is this result indicative of?
Of a chronic process
Of recurring infection with rubella virus
@Of primary infection
Of exacerbation of a chronic disease
The woman is healthy
#
60
A gross investigation of a brain at a post-mortem revealed the edema, a plethora, and fine hemorrhage in medulla
oblongata. A microscopical investigation showed a chromatolysis, hydropy and a necrosis of nervous cells. In addition,
the eosinophilic intracytoplasmic formations (little bodies the Babesh-Negri bodies) were defined in pyramidal neurons of
the hippocampus. What diagnosis answers the described morphological implications?
@Rabies
Meningococcal meningitis
Encephalitis
Encephalomyelitis
Brucellosis
#
61
A man who was bitten by the unknown dog applied to the surgeon. Wide ragged woundes were localized on the face.
What curative-prophylactic aid should be given to prevent rabies?
@Start immunisation with rabies vaccine
Immediately inject normal gamma globulin
Prescribe combined antibiotic therapy
Immediate injection of DPT(Diphtheria, Pertusis, Tetanus) vaccine
Hospitalize the patient and keep under the doctor's supervision
#
Quarantine and other infection
#
1
A histological investigation of a skin biopsy showed serous-hemorrhagic inflammation and a focus of necrosis. His
medical records revealed the beginning of the disease from a small red maculae formation with the bubble in the centre,
filled by serous-hemorrhagic liquid. Subsequently the central part became black. What disease is the most probable?
@Carbuncle at a malignant anthrax
Actinomycosis of skin
Allergic dermatitis
Streptococcal carbuncle
Chemical dermatitis
#
2
The worker of a stockbreeding farm presented to the hospital with acute disease and died soon of intoxication. An autopsy
revealed the enlarged, flaccid spleen, which had a dark-cherry colouring of its cut surface. The scrape of the spleen's pulp
was plentiful. Arachnoids of the brain's fornix and base were edematous, impregnated by blood, had a dark red colouring
("cardinal's hat"). Microscopical investigation found the serous- hemorrhagic inflammation of brain's membranes and
tissues with destruction of fine vessels walls. Diagnose a disease.
@Anthrax
Tularemia
Brucellosis
Plague
Cholera
#
3
A worker of the agricultural enterprise presented with an acute disease and died soon of intoxication. An autopsy revealed
the enlarged and flaccid spleen with a dark-cherry coloring of a cut surface. There were a superfluous pulp's scrape from
the lien cut surface. Arachnoids of the brain's fornix and base were edematous, impregnated by blood ('red cap' or
'cardinal's hat'). Microscopical investigation found the serous- hemorrhagic inflammation of brain's membranes and
cerebral tissues. What is the most likely diagnosis?
@Anthrax
Tularemia
Plague
Cholera
Brucellosis
#
4
The patient of 34 years has addressed to a doctor concerning a carbuncle on the face. During the review: leaky edema of a
hypodermic fat without a pain, in the center of the carbuncle is a black crust, on periphery of the carbuncle - vesicular
rashes. Microbiological research has found out presence of non-motile streptobacilli that are capable to form capsules.
Which microorganisms are pathogens of the given illness?
@Bacillus anthracis
Staptylococcus aureus
Bacillus anthracoides
Bacillus subtilis
Bacillus megaterium
#
5
During external survey the doctor has found out cone-like infiltrate of bright red color with sharply expressed edema of
tissues in 42 years old man, working as butcher. In the center of an infiltrate is black eschar. About which disease can you
think?
@Anthrax
Abscess
Furunculosis
Plague
Phlegmon of a brush
#
6
A 42-year-old woman, who worked at a fur atelier, suddenly died. An autopsy revealed dark red, impregnated by blood
arachnoids of the brain's fornix and base, which reminded a red cap. Microscopical investigation found the serous-
hemorrhagic inflammation of brain's membranes and tissues with necrosis of fine vessels walls and numerous
hemorrhages. What is the most likely diagnosis?
@Anthrax
Cerebral hemorrhage in case of hypertension disease
Cerebral hemorrhage due to trauma
Meningococcal infection
Tubercular leptomeningitis
#
7
A 48-year-old butcher died of a sepsis. A gross investigation of his right cheek revealed a dense, dark red, 6 cm in
diameter, corn-shape infiltrate with a black crust in its centre. The right half of his face and neck were edematous and
solid. A microscopical investigation of a lesion showed the peracute serous- hemorrhagic inflammation with epidermal
and adjacent tissue necrosis within the central zone of an infiltrate. What is the most likely diagnosis?
@Anthrax
Plague
Tularemia
Phlegmon of a neck
Furuncle
#
8
During the examination of a patient, who had been to the mountain pasture and had been hospitalized in a bad condition
with fever, the doctor found out the enlargement of inguinal lymph nodes to 8 cm, which were attached to the surrounding
tissues, immovable, the skin above them was red and tender. The microscopic examination of the node revealed acute
serohemorrhagic inflammation. What disease is it typical for?
Brucellosis
Syphilis
Anthrax
Tularemia
@Plague
#
9
A 38-year-old patient was on mountain pastures and then presented to the hospital in severe condition, with high body
temperature. A physical investigation revealed the lymph nodes were markedly enlarged, soldered to surrounding tissue,
motionless, a skin over them was red and very painful. A microscopical study of a lymph node showed the peracute
serous-hemorrhagic inflammation. For what disease it is characteristic?
@Plague
Tularemia
Syphilis
Brucellosis
Anthrax
#
10
An autopsy of the patient who died of a plague revealed a hemorrhagic syndrome, accompanied with the hemorrhagic
necrosis of a hip skin, a lymphangitis and an inguinal hemorrhagic lymphadenitis. Name the plague form.
@Dermo-bubonic
Bubonic
Primary septic
Primary pulmonic
Hemorrhagic
#
11
Eruptions on a skin, which were accompanied by a strong itch, increased body temperature up to 38°C, pain in joints has
appeared in a child sick with diphtheria in 10 days after introduction of antitoxic antidiphtheritic serum. What is the
reason of these phenomena?
@Serum illness
Anaphylactic reaction
Atopy
Hypersensitivity of delayed type
Contact allergy
#
12
A child with diphtheria 10 days after injection of antitoxic antidiphtherial serum has developed skin rash, accompanied by
severe itch, rising temperature up to 380C and joints pain. What is the cause of these symptoms?
Delayed type of hypersensitivity
Anaphylactic reaction
Contact allergy
Atopia
@Serum sickness
#
13
A child presented with diphtheria and died on the 2nd week of the disease from an acute heart failure. A post- mortem
revealed numerous fine centres of a myolysis, accompanied with some perifocal lymphoid infiltration. What became a
cause of death?
@Myocarditis, caused by bacterial ectotoxin
Bacterial myocarditis
Septic myocarditis
Myocardial infarction
Metabolic myocardial necrosis
#
14
A histological investigation of a myocardium revealed fatty dystrophy and plural necroses of cardiomyocytes, few focal
cellular interstitial infiltrates. A medical record showed that a child had diphtheria, resulted in the heart failure.
What kind of myocarditis is the most likely?
@Alterative myocarditis
Diffusive exudative myocarditis
Focal exudative myocarditis
Intersticial myocarditis
Granulomatous myocarditis
#
15
A child presented to a hospital with an acute fever, sore throat and swelling of submaxillary (submandibular) lymph
nodes. A gross investigation of fauces revealed the edematous and slightly hyperemic mucous membrane, enlarged
tonsils, covered by the grayish-whitish membranes. They left a raw surface, when forcibly removed. What is the most
likely diagnosis?
@Diphtheria
Catarrhal angina
Scarlet fever
Meningococcal infection
Measles
#
16
A child presented to the clinic with paralysis of a phrenic nerve and dysphagia, resulted in his death soon. An autopsy
revealed ulcers of tonsils with a cuticularization, a hemorrhage in adrenals, and flaccidity of a myocardium. Specify
disease for which the described changes and complications are characteristic.
@ Diphtheria
Scarlet fever
Agranulocytosis
Measles
Poliomyelitis
#
17
A 3-year-old girl presented with a sharp pain at the swallowing, a marked neck edema, and the body temperature raised to
40° C. A gross investigation of tonsils revealed the grayish-yellowish membranes, which left a raw surface with bleeding
ulceration, when forcibly removed. What is the most likely diagnosis?
@Diphtheria
Scarlet fever
Measles
Meningococcal nasopharyngitis
Influenza
#
18
An oral cavity examination of a 4-year-old child, revealed a hyperemia of the fauces and tonsils mucus membranes. Gross
investigation showed enlarged tonsils, covered by dense, whitish-yellowish membranes. They left a raw surface, with
deep defect, when forcibly removed. Soft tissues of a neck were edematous, regional lymph nodes were enlarged and
painful. What the most likely diagnosis is among listed below?
@Diphtheria
Scarlet fever
Measles
Parotitis
Adenoviral infection
#
19
A 4-year-old child presented to the hospital with a fever and a sore throat. An oral cavity examination revealed an edema
of a soft palate and grey membranes on tonsils. These membranes left a raw hemorrhagic surface when forcibly removed.
What the most likely diagnosis is among listed below?
@Diphtheria fauces
Simonovsky-Vensans' angina
Lacunar angina
Infectious mononucleosis
Necrotic angina
#
20
A 5-year-old girl has high temperature and pain in her throat. At the examination of this patient the following signs were
revealed: edema of soft palate, grey spots on tonsils, which were removed with difficulty and left after them deep
hemorrhagic defects of tissue. Which disease of below enumerated is the most possible one?
@Diphtheria of pharynx
Lacunar tonsillitis
Simanovsky-Vensan’s tonsillitis
Necrotic tonsillitis
Infectious mononucleosis
#
21
A 5-year-old girl has a high temperature and a pain in a throat. Objectively: a hypostasis of the soft palate, grey patches
on the tonsils, which are separated hardly and left deep bleeding defects of a tissue. Which from the listed below diseases
is the most probable?
@Diphtheria of a pharynx
Necrotic tonsillitis
Infectious mononucleosis
Lacunar tonsillitis
Simonovsky-Vensan’s angina
#
22
A 16-year-old girl presents with the symptoms of sharp pain during swallowing, lymph node enlargement of the neck, and
the body temperature of 38°C. The mucous membrane of the tonsils revealed grayish membranes with yellow tapes with
were not easily separated from the defect. The patient's state progressively worsened which death occurring on the 8th
day of the disease due to cardiac insufficiency. Which of following histological changes in the myocardial cells will be
the most likely finding?
@Fatty dystrophy
Hydropic dystrophy
Hyaline dystrophy
Ballooning dystrophy
Mucous dystrophy
#
23
An 18-year-old teenager acutely fell ill and died of infectious-toxic shock. The autopsy revealed the enlargement of
tonsils, covered with gray-white membranae that spread on the palatine arches, the edema of soft tissues of the neck. At
the histological exam were detected: the necrosis of tonsilar and arch’s epithelium; underlied tissues were sodden with
fibrous exudate that forms massive strata on its surface. Diagnose the disease.
Staphylococcal infection
Scarlel fever
Adenoviral infection
Infection mononucleosis
@Diphtheria
#
24
A young 19-year-old man, with an acute onset of disease, died of an infectious-toxic shock. An autopsy revealed the
enlarged tonsils, covered with grey-white membranes, which extended on the palatine arch. In addition, an edema of the
neck soft tissues determined. Histological investigation showed the necrosis of an epithelium of tonsilsand the palatine
arch. Tissues underneath were infiltrated by a fibrinous exudate, which formed massive stratifications on a surface. What
is the most likely diagnosis?
@Diphtheria
Scarlet fever
Adenoviral infection
Infectious mononucleosis
Staphylococcal infection
#
25
An autopsy of 2-year-old child revealed hemorrhagic enanthesis (skin rash), mild hyperemia and edema of nasopharynx
mucous, fine hemorrhages in mucosas and internal organs, severe dystrophy in a liver and myocardium, an acute necrotic
nephrosis, massive hemorrhages in adrenals. What disease is the most likely to present such symptoms?
@"Meningococcal infection
Scarlet fever
Diphtheria
Measles
Epidemic typhus
#
26
An autopsy of 3-year-old child revealed hyperemic, turbid arachnoids, which had a form of yellowish-greenish 'cap'. At
microscopic investigation a characteristic findings of arachnoids thickening, hyperemia, purulent with fibrin exudate
infiltration was present. What is the most likely diagnosis?
@Meningococcal infection
Tuberculosis
Anthrax
Ifluenza
Measles
#
27
A child presented with acute a catarrhal nasopharyngitis. In 2 days he died. An autopsy revealed arachnoids were
markedly hyperemic, edematous, infiltrated with turbid, thick, yellowish-greenish liquid. A brain was swelled, cerebellum
tonsils were enlarged in volume, and sulcus of the cerebrum impaction was well defined. The described changes are the
most typical for:
@Meningococcal infection
Influenza
Whooping cough
Diphtheria
Measles
#
28
At the section was revealed: pia mater of upper parts of the cerebral hemispheres was sanguineous, of yellow-green color,
sodden with purulent and fibrous exudate that looked like a cap. What disease is this picture typical for?
@Meningococcal meningitis
Tuberculous meningitis
Influenzae meningitis
Anthrax meningitis
Typhus (spotted fever) meningitis
#
29
On autopsy it was revealed: pia mater of the upper parts of cerebral hemisphere is plethoric, of yellowish-green color,
soaked with purulent and fibrose exudate, looks like a cap. What disease is it typical for?
Tuberculous meningitis
@Meningococcal meningitis
Meningitis connected with anthrax
Grippal meningitis
Meningitis connected with typhus
#
30
An autopsy revealed: soft arachnoid membrane of the upper parts of cerebral hemisphere is plethoric, of yellowish-green
color, soaked with purulent and fibrose exudate, it lookes like a cap. What disease is characterised by these symtoms?
Meningitis at anthrax
Meningitis at typhus
Tuberculous meningitis
@Meningococcal meningitis
Influenza meningitis
#
31
A 3-year-old child died being in severe cerebral comma. At the section were revealed purulent strata on the surface of pia
mater in frontal parts of cerebrum, and cerebral edema. Microscopically, plethora and neutrophilic infiltration of pia mater
were detected. Term the possible disease.
@Meningococcal meningitis
Anthrax meningitis
Influenzae meningitis
Tuberculous meningitis
Measles meningitis
#
32
An autopsy of 4-year-old child revealed hyperemic, yellowish-greenish arachnoids, infiltrated by purulent and fibrinous
exudate resembled a 'cap'. What is the most likely diagnosis?
@Meningococcal meningitis
Tubercular meningitis
Influenzal meningitis
Anthrax meningitis
Epidemic typhus meningitis
#
33
A 6 year-old child, was ill acutely with signs of intoxication. In 2 day the patient has died. In autopsy the pathologist has
found out: meninges of brain with edema, hyperemia, yellow-grey exudate. Tissue of brain was edematous. Microscopic
investigation: there were neutrophils, hyperemia, hemorrhages and edema in meninges. Described changes are most
typical for:
@Meningococcal meningitis
Flu
Pertussis
Diphtheria
Measles
#
34
11-year-old child presented with a body temperature up to 40°C, an acute headache, vomiting, anxiety and a fever. In four
days appeared a hemorrhagic skin rash, an oliguria and adrenal insufficiency, resulted in patient's death. A bacteriological
study of smear preparation from a pharynx revealed a meningococcus. What form of disease is found out?
@Meningococcemia
Meningococcal meningitis
Meningocephalitis
Meningococcal nasopharyngitis
-
#
35
An autopsy of 11-year-old child revealed plural hemorrhages, mainly in skin of breeches and lower extremities, serous
and mucosas, and also in a brain. Gross investigation determined a focal necrosis and massive hemorrhages .in adrenals, a
necrotic nephrosis in kidneys, a purulent arthritis, an iridocyclitis and vasculitis. Choose the correct diagnosis.
@Meningococcemia
Epidemic typhus
Periarteritis nodosa
Systemic lupus erythematosus
Radiation syndrome
#
36
Autopsy of a 12-year-old girl revealed: multiple cutaneous hemmorhages (mostly into the skin of buttocks, lower
extremities), serous and mucous memrane hemmorhages, cerebral hemmorhages. Adrenal glands show focal necrosis and
massive hemmorhages; kidneys show necrotic nephrosis, suppurative arthritis, iridocyclitis, vasculitis. What is the most
probable diagnosis?
Epidemic typhus
Radiation sickness
@Meningococcemia
Systemic lupus erythematosus
Periarteritis nodosa
#
37
A 38-year-old man died of cerebral coma. It was known, that after foreign business trip in one of the African countries,
the icteruses periodically developed. An autopsy revealed the enlarged, dense spleen, with a black pulp. A liver was also
enlarged, hyperemic, and grey-black. A gross investigation of the brain determined brown-grey coloring of a grey matter
and plural fine hemorrhages in white matter. What infectious disease should be suspected?
@Malaria
Meningococcemia
Prion infection
Generalized herpes infection
Generalized cryptococcosis
#
38
An autopsy of 48-year-old woman, resident of Middle Asia, with long history of exhausting fever, revealed enlarged liver
and a spleen. They were grey-flaky color. The same color had hyperplastic bone marrow. A cerebral cortex also had grey-
smoky coloring. What is the most likely diagnosis?
@Malaria
AIDS
Epidemic typhus
Sepsis
Hepatitis
#
39
A 48-year-old patient presented to the hospital with deep cerebral coma. It was known, that during lifetime he
experienced periodic attacks of a fever. An autopsy revealed grey-flaky colour of a bone marrow, lymph nodes; enlarged
a liver and a spleen. Histological investigation showed the haemomelanosis and a hemosiderosis in mentioned organs.
What of diagnoses is most probable?
@Malaria
Hemolytic anemia
Addison disease
Septicemia
Black pox
#
40
According to the data of WHO, for about 250 mln of Earth population fall ill with malari a. This disease is mostly spread
in tropical and subtropical regions. Range of its spread falls into the areal of the following mosquitoes:
Mansonia
@Anopheles
Culiseta
Aedes
Culex
#
41
A journalist’s body temperature has sharply increased in the morning three weeks after his mission in India, it was
accompanied with shivering and bad headache. A few hours later the temperature decreased. The attacks began to repeat
in a day. He was diagnosed with tropical malaria. What stage of development of Plasmodium is infective for anopheles-
female?
Sporozoites
Shizontes
@Gametocytes
Merozoites
Microgamete
#
42
2 weeks since the blood transfusion a recepient has developed fever. What protozoal disease can it be?
Trypanosomiasis
@Malaria
Amebiasis
Toxoplasmosis
Leishmaniasis
#
43
A doctor made the diagnosis of gonorrhoea. It was known from the anamnesis that a patient had had gonorrhoea before
and he had been treated completely. What type of infection can this new disease be attributed to?
Superinfection
@Reinfection
Secondary infection
Relapse
Autoinfection
#
44
During the bacteriological tests of the purulent secreta from urethra there were found bacteria, which according to Gramm
were negatively staining, looked like coffee beans. These bacteria were splitting glucose and maltose to acid, they were
located inside the leucocytes. The aetiological agent of what disease are these microorganisms?
Syphilis
Venereal lymphogranulomatosis
@Gonorrhoea
Chancroid
Melioidosis
#
45
A duodenal content smear of a patient with indigestion contains protosoa 10-18 mcm large. They have piriform bodies, 4
pairs of filaments, two symmetrically located nuclei in the broadened part of body. What kind of the lowest organisms is
it?
Balantidium
Intestinal ameba
Dysentery ameba
Trichomonas
@Lamblia
#
46
Parents with an ill child consulted an infectionist. They had been working in one of Asian countries for a long time. The
child has sallow skin, loss of appetite, laxity, enlarged liver, spleen, peripheral lymph nodes. What protozoal illness can
be suspected?
Amebiasis
@Visceral leishmaniasis
Balantidiasis
Lambliasis
Toxoplasmosis
#
47
A man was admitted to the hospital on the 5th day of disease that manifested itself by jaundice, muscle aching, chill, and
nose bleedings. In course of laboratory diagnostics a bacteriologist performed dark-field microscopy of the patient's blood
drop. Name a causative agent of this disease:
Rickettsia mooseri
@Leptospira interrogans
Bartonella bacilloformis
Borrelia dutlonii
Calymmatobacterium granulomatis
#
48
A histological research of an autopsy material revealed in a brain tissue a wide zone of proliferating microglia cells round
small vessels. In addition, the marked proliferation of an endothelium, an adventitia and pericytes were determined in the
skin vessels. Perivascular moderate lymphocytes infiltration with single neutrophiles added was also found. The vessels
walls underwent various signs of destruction. Name the causative organism of described infection disease?
@Rickettsia
Spirochete
Meningococcus
Streptococcus
Human immunodeficiency virus
#
49
A 39-year-old milkmaid died of cardiovascular insufficiency. A post-mortem reveled in the aortal valve a polypous -
ulcerative endocarditis. A microscopical study of the valve showed the polymorph cellular inflammatory infiltration,
zones of destruction and thrombuses with the organization. In addition, a sclerosis with granulomas, consisted of
randomly located epithelioid, giant, plasmatic, eosinophyle cells were diagnosed within a stroma of a myocardium. A
vasculitis was also determined in the heart vessels. For what disease the described changes in heart are characteristic?
@Brucellosis
Chronic sepsis
Rheumatic disease
Systemic lupus erythematosus
Systemic scleroderma
#
50
The worker of a cattle-breeding farm died in 4, 5 months of the disease beginning. A post-mortem revealed in his internal
organs the granulomas consisted of epithelioid, giant Pirogov- Langhans cells, plasmocytes and eosinophyles. There were
many vessels within a granuloma. A histological investigation showed a productive-destructive vasculitis in the vessels of
internal organs. What is the most likely disease?
@Brucellosis
Anthrax
Tuberculosis
Epidemic typhus
Lepra
#
51
A woman who was infected with toxoplasmosis during the pregnancy has a child with multiple congenital defects.This is
a result of:
@Teratogenesis
Chemical mutogenesis
Biological mutogenesis
Recombination
Cancerogenesis
#
52
A child complains of general weakness, loss of appetite, a troubled sleep, itching in the perianal are a. The provisional
diagnosis is enterobiasis. In order to specify this diagnosis it is necessary to perform:
Immune diagnostics
Duodenal contents analysis
@Scraping from perianal folds
Biopsy of muscle tissue
Roentgenoscopy
#
53
Larvae were detected occasionally on the microscopic examination of the sputum of the patient with pneumonia.
Eosinophiles were detected on the blood examination. What helminthiasis can be diagnosed?
@Ascariasis
Enterobiosis
Opistorchis
Trichocephaliasis
Paragonimiasis
#
54
Slime, blood and protozoa 30-200 microns of length have been revealed in a man's feces. The body is covered with cilias
and has correct oval form with a little bit narrowed forward and wide round shaped back end. On the forward end a mouth
is visible. In cytoplasm there are two nucleuses and two short vacuoles. For whom are the described attributes typical?
@Balantidium
Lamblia
Intestinal amoeba
Trichomonas
Dysenteric amoeba
#
55
During the section in the liver was detected a bladder-like formation with plain surface, diameter 5cm. A great amount
small vesicles with bright colorless contents is located in its cavity. The liver tissue around the bladder is sclerosed. What
is the most possible diagnosis?
Opisthorchiasis
Alveococcosis
Cysticercosis
@Hydatid Echinococcosis (hydatid disease)
Schistosomiasis
#
56
The patient with pneumonia was treated with antibiotics for a long period. After treatment patient complains of frequent
and watery stool, abdomenal pain. What is the reason of intestine function disorder?
Hereditary enzyme defect
Antibiotics toxic influence on the GIT
@Intestinal disbacteriosis development
Autoimmune reaction development
Bacteria toxins influence
#
57
A 54-year-old man presented to the hospital with reddening and edema of the neck's skin, and then resulted in a small
abscess formation. On a cut, a tissue was dense with yellow-green coloring. In purulent masses there were visible white
grains. Histological study showed fungus' druses, plasmatic, xanthome cells, and macrophages. Specify the most probable
kind of a mycosis.
@Actinomycosis
Aspergillosis
Candidosis
Sporotrichosis
Coccidioidomycosis
#
58
A 69-year-old man presented to the hospital with edema and a painfulness of a skin and soft tissues of his neck. The
phlegmon of a neck and a mediastinitis were diagnosed. The patient died. An autopsy revealed enlarged, dense left tonsil.
On a cut, the tonsil was yellow-green, with set of fine cavities similar to porous structures. Microscopical investigation
determined fine abscesses with basophilicformations in their centers. These formations consisted of short rhabdoid
elements. What is the mostlikely diagnosis?
@Actinomycosis
Brucellosis
Amebiasis
Lambliasis
Leishmaniasis
#
59
Firm phlegmon-like infiltrate has appeared in cervical - gnathic area in the man, inhabitant of countryside. A skin around
infiltrate is of blue-crimson color, in the center – necrotic infiltrate, pus with an unpleasant smell is allocated from an
ulcer. For confirmation of the diagnosis “Actynomycose of cervical – gnathic area" the microscopicl research of pus is
carried out. What should find out a bacteriologist for confirmation of such?
@The presence of grains in pus (druses)
Gram-positive streptococci
Gram-negative diplobacteria
Acid-fast rods
Gram-negative diplococci
#
60
During survey of trimensual child doctor - pediatrician has noted, that the mucosa of an oral cavity, and also tongue are
covered by a dense white deposit. In a material taken from a place of defeat, the bacteriologist has found out presence of
fungi of the yeast-like form, in this connection have suspected a mycosis:
@A candidosis
Favus
Epidermophytia
Actinomycosis
Trychophytia
#
61
The doctor has found out spherical and ellipse like formed cells, posed by bunch, 3-6 microns in size during microscopy
of a micro preparation from vaginal discharge of person with chronic colpovaginitis. About the pathogen of what fungoid
disease there may be a speech in this case?
@Candidosis
Coccidiosis
Epidermophytia
Microsporia
Cryptococcosis
#

You might also like